Sie sind auf Seite 1von 554

1- Question 1 of 5

A 36 yo lady presents to the ED with a short history of breathlessness a rash & feeling wheezy & SOB that came on
whilst out at a restaurant with friends. She is a known asthmatic & also has eczema

(a) Name 4 common causes of anaphylaxis:


Answer (Latex - Food (nuts, shellfish, wheat, strawberries) - Hymenoptera (bee stings) - Drugs & vaccines (many
including: NSAIDS, sux, Abx, Asprin, IV contrast etc)
(b) What is your initial management (assume airway compromise & a severe reaction)?
Answer (Assess ABC & treat accordingly -100% O2- IM adrenaline if needed (0.5mg 1:1000 IM) Salbutmaol nebs -
10mg IV chlorphemeramine- Fluid if not responding to adrenaline - Oral or IV steroid depending on if Pt can swallow
(c) After assessing ABC the Pt is stable. How long would you want to observe her for & why?
Answer (At least 4 hrs looking for delayed hypersensitivity, also Pt is asthmatic & they often have more severe react.
(d) Which medication is pertinent in the history of any Pt presenting with angio-oedema?
Answer (ACE inhibitor i.e. Ramipril)
(e) How do you treat Hereditary angio-oedema?
Answer (C1 esterase inhibitor)
‫ــــــــــــــــــــــــــــــــــــــــــــــــــــــــــــــــــــــــــــــــــــــــــــــــــــــــــــــــــــــــــــــــــــــــــــــــــــــــــــــــــــــــــــــــــــــــــــــــــــــــــــــــــــــــــــ‬
2- Question 2 of 5
A 65 yo gentleman presents to the department with a classic sounding history for cardiac chest pain which came on
suddenly across his chest 32 hrs ago.

(a) Describe the 2 important changes shown by the ECG (fig 1) & hence locate anatomically the pathology
Answer (1- ST elevation in II,III & aVF 2- Reciprocal changes in leads V1 & V2 3- This is therefore an acute inferior MI)
(b) What is it important to be vigilant for when a Pt presents with this type of picture (specific to the above
ECG changes) & what additional test could you do to access for this?
Answer (Rt ventricular infarct Do Rt ventricular leads (V4R))
(c) The Pt responds very quickly to GTN spray & the chest pain settles, also you notice that the repeat ECG
taken 5 minutes after the GTN now looks normal. What is a possible explanation for such quick resolution?
Answer (ECG changes resolving so quickly are more in keeping with Prinzmetal angina or variant angina)
(d) List 4 of the most important contraindications to thrombolysis
Answer (1- Severe HTN systolic >200 mmHg, diastolic >120mmHg
2- Head injury, CVA or recent TIA
3- Previous neurosurgery or cerebral tumour
4- Recent GI or GU bleed
5- Warfarin
6- Pregnancy 7- Recent major surgery
8- Puncture of non-compressible vessel )
(e) List 4 things that can cause a rise in Troponin levels
Answer (a. Acute myocardial infarction b. Sepsis c. Acute renal failure d. Pulmonary embolus)
‫ــــــــــــــــــــــــــــــــــــــــــــــــــــــــــــــــــــــــــــــــــــــــــــــــــــــــــــــــــــــــــــــــــــــــــــــــــــــــــــــــــــــــــــــــــــــــــــــــــــــــــــــــــــــــــــ‬
3- Question 3 of 5

A 39 yo man presents to the department after a fall from a 6 metre ladder onto his Rt side

1
(a) Describe the most important things that this CT chest shows (fig 1):
Answer (1- Pulm. contusion 2- Fractured ribs Also 3- Hameothorax 4- Pneumothorax (apical) 5- (Surgical emphysema )
(b) What's the most important initial management step? Where should this Pt be sent? What's your choice of
analgesia?
Answer (ABCD then as part of that ruling out a tension pneumothorax & insert. of a thoracostomy tube, Likely going to need
HDU/ITU care, Thoracic epidural but mark for mentioning that an intercostal nerve block may be a good intermediate choice)
(c) The Pt deteriorates in front of you with sats of 85% on 80% FiO2 you decide that he requires intubation &
IPPV. Describe the steps required for an RSI (including any drug doses)
Answer (Preoxygenate 3 mins if possible or ventilate with 100% O2, Ensure adequate monitoring ECG, Sats, BP &
secure IV access, Cricoid Pr., ensure not released until ET secured, Induction agent (Thiopentone 3-5mg/kg,
Etomidate 0.3mg/kg, Ketmaine 0.5-2mg/kg), Suxamethonium 1-1.5mg/kg, Confirm placement of tube (best by direct visualisat.
of tube passing through the cords), end tidal CO2. Listen to the chest both sides, inflate cuff & secure ET tube.)
(d) As part of the 1ry survey you notice that the Pt has a tender abd.. What do you do next?
Answer (FAST scan/DPL to rule out ? liver injury If stable CT Abd., if not laparotomy.)
(e) The CT is reported & the injuries are worse than you first thought, amongst other things there is a fracture
of the first rib. What is it essential to rule out & how will you do this?
Answer (Need to rule out an aortic injury, needs arch aortogram)
‫ــــــــــــــــــــــــــــــــــــــــــــــــــــــــــــــــــــــــــــــــــــــــــــــــــــــــــــــــــــــــــــــــــــــــــــــــــــــــــــــــــــــــــــــــــــــــــــــــــــــــــــــــــــــــــــــــــــــــــــــــــــــــــــــــــــــــــــــــ‬
4- Question id: 2003
A 26 yo male presents after having been hit around the head with a metal bar. His GCS on admission is 14 but after an
hr it has fallen to 13. His CT is shown (fig 1):

Fig 1
(a) Describe what it shows:
Answer (1- Rt frontal acute extradural haematoma 2- Skull fracture 3- Air bubble in the haematoma 4- Midline shift)
(b) Describe your initial management priorities, including the definitive management for this Pt
Answer (1- ABCDE 2- Intubation & controlled ventilation 3- Try to decrease ICP +/- mannitol 4- Arrange
appropriate neurosurgical referral/transfer 5- Definitive = Burr hole & evacuation of the haematoma.)
(c) List all indications for an immediate CT scan of the head following head trauma according to the NICE
guidelines.
Answer (1- GCS<13 when 1st assessed 2- GCS<15 when assessed 2 hrs after the injury 3- Suspected open or
depressed skull fracture 4- Sign of fracture at skull base (panda eyes, haemotypanum, Battle's sign, CSF leak from
nose or ears. 5- Post traumatic seizure 6- Focal neurological deficit 7- >1 episode of vomiting 8- Amnesia of events >
30 mins before impact)
(d) For a child under 16 how many episodes of vomiting after a head injury is acceptable before imaging is
required according to NICE?
Answer (3 or more = CT head)
(e) What is the role of hypothermia therapy after traumatic brain injury in children?
Answer (It does not improve outcome & may increase mortality)
‫ــــــــــــــــــــــــــــــــــــــــــــــــــــــــــــــــــــــــــــــــــــــــــــــــــــــــــــــــــــــــــــــــــــــــــــــــــــــــــــــــــــــــــــــــــــــــــــــــــــــــــــــــــــــــــــــــــــــــــــــــــــــــــــــــــــــــــــــــــــــــــــــ‬
5- Question id: 2004
A 68 yo lady is brought in to ED resus at 06:00 am. The crew say that she had a short history of waking up & being
unable to get her breath. O/E she has a RR of 36, sats of 89% on a re-breathe mask

2
(a) The ED in which you work has the resources to measure BNP or N-terminal- ProBNP. Is there any
diagnostic value in measuring this?
Answer (Yes, BNP> 500 pg/dL or NT- proBNP> 1000 pg/dL makes acute heart failure syndrome likely (approx +ve
likelihood ratio [LR+} = 6)
(b) The Pt deteriorates & you start her on CPAP 5 mmHg. Please explain briefly how CPAP works
Answer (1. CPAP splints the alveoli open, thereby preventing alveolar collapse & allowing unimpeded alveolar
ventilation (Recruits alveoli). 2. CPAP also ↓ preload & afterload, improves lung compliance,  FRC & ↓ work of
breathing. 3. NIV reduces trans-diaphragmatic Pr., Pr. time index of respiratory muscles & diaphragmatic EMG
activity. This leads to an  in tidal volume, ↓ in RR &  in minute ventilation. Also overcomes the effect of intrinsic
PEEP. CPAP reduces left ventricular transmural Pr. & therefore  CO. Hence it is a very effective for TTT of
pulmonary oedema. Causes increase in intrathoracic Pr. therefore improving cardiac output.)

(c) Her Bl. Pr. is 113/56 & decides to treat her with a nitrate infusion. Is there any evidence for or against
giving diuretics please discuss:
Answer: Yes but always in combination with nitrates. There is also some evidence to move away from diuretic
montherapy as it is unlikely to prevent the need for tracheal intubation & can worsen renal function which has been
shown to increase mortality.1 The advice is to use it in combination & to use them judiciously
(d) Explain the mechanism behind how diuretics work in the acute management of heart failure & how this fits
in with the pathogenesis of acute pulmonary oedema.
Answer: They work via venodilatat. The other acute HF syndromes (pulm. edema, HTN crisis & exacerbated HF) are
caused by a combinat. of progressive excessive vasoconstrict. superimposed on ↓ lt vent. functional reserve. The
impaired cardiac power & extreme vasoconstrict. induce a vicious cycle of afterload mismatch resulting in a dramatic
↓ of CO &  lt vent. end diastolic Pr., which is transferred backwards to the pulm. capillaries yielding pulm. oedema.
Therefore, the immediate TTT of these acute HF syndromes should be based on the administrat. of strong, fast-acting
IV vasodilators such as nitrates or nitroprusside. After initial stabilizat., therapy should be directed at ↓ recurrent episodes
of acute HF, by prevention of repeated episodes of excessive vasoconstrict. along with efforts to optimize cardiac funct.
(e) You decide to insert a central line as the Pt has very poor peripheral access. You insert a left sided internal
jugular line as there was some local scar tissue on the Rt. When you get the chest x-ray to confirm position you
see the film shown (see fig 1). Can the line be used? What would you want to do prior to using it?
Answer (It is actually venous, it's in a low brachiocephalic trunk but it doesn't look like it. You would want to check
the length of line inserted should be at least 14 cm. Aspirate all lumens & run it through a Bl. gas analyser to confirm
that it is venous Bl.., Attach it to a CVP monitor & transducer the line to look for a venous waveform. If further doubt
perform a venogram rarely the anatomy is unusual like in this case)
‫ـــــــــــــــــــــــــــــــــــــــــــــــــــــــــــــــــــــــــــــــــــــــــــــــــــــــــــــــــــــــــــــــــــــــــــــــــــــــــــــــــــــــــــــــــــــــــــــــــــــــــــــــــــــــــــــــــــــــــــــــــــــــــــــــــــــــــــــ‬
6- Question 1 of 50
A 50 yo man presented with an extremely painful lt ankle. There was no history of trauma & the pain had a gradual
onset over the previous 24 hrs. O/E there was minor redness & swelling.

(a) Besides acute gouty arthritis name 2 other possible diagnoses?


Answer (Pseudogout & septic arthritis.)
(b) Name three predisposing factors to acute gouty arthritis?
3
Answer (Trauma, surgery, starvation, dietary overindulgence, & ingestion of drugs affecting serum urate
concentrations (eg, allopurinol & uricosuric agents) may all promote gouty attacks. In men, increasing alcohol
consumption (beer & spirits, but not wine) is associated with proportionately a greater risk of developing gout.)
(c) What are the 2 most appropriate investigations to confirm acute gouty arthritis in this Pt?
Answer:  serum urate(however an appreciable No. of cases occur in Pts with normal or even ↓ serum urate
concentrate.) & aspirat. of synovial fluid from the affected joint & analysis of the fluid by Gram stain, culture &
polarized light microscopy.
(d) Name one feature on a plain radiograph suggestive of gouty arthritis?
Answer: Subcortical bone cysts. In chronic tophaceous gout bone erosions due to tophi may have delicate
"overhanging" edges & punctate to diffuse soft tissue calcification.
(e) Name 2 factors that predispose to chronic tophaceous gout?
Answer: Pts treated with cyclosporine (& often diuretics as well) are at  risk for the accelerated development of
chronic tophaceous gout. Other Pts at  risk for chronic tophaceous gout are those who have chronic kidney disease
that precludes full dose antihyperuricemic drug therapy & those who are allergic to or otherwise intolerant of
uricosuric agents & allopurinol.
‫ـــــــــــــــــــــــــــــــــــــــــــــــــــــــــــــــــــــــــــــــــــــــــــــــــــــــــــــــــــــــــــــــــــــــــــــــــــــــــــــــــــــــــــــــــــــــــــــــــــــــــــــــــــــــــــــــــــــــــــــــــــــــــــــــــــــــــــــ‬
7- Question 2 of 50
A 46 yo business man develops diarrhoea 3 days into his trip to SE Asia. He has 3 very important meetings over the
next three days & comes into the ED.

(a) What advice would you give him?


Answer (General advice re traveller s diarrhoea including strict hand hygiene not eating washed salad not drinking
the water. Could prescribe antibiotics, good evidence for early TTT. In SE Asia consider Campylobactor therefore
azithromycin rather than ciprofloxacin.1)
(b) What would you tell him about chemoprophylaxis for any further business trips?
Answer (Expert opinion supports the use of prophylactic antibiotics when the trip is vitally important or the
consequences of diarrhoea would be difficult to manage.)
(c) Why is chemoprophylaxis currently not recommended for most traveller s?
Answer (Potential complications i.e. predisposition to other conditions e.g. vaginal candidiasis, or C. difficle
diarrhoea, development of bacterial resistance, cost, lack of data & efficacy of antibiotics given for 2 or 3 weeks.)
(d) Are probiotics any good at preventing traveller s diarrhoea?
Answer (Yes as much as 15% risk reduction)
(e) When he returns he finds that the diarrhoea has persisted for 2 weeks he comes to the ED out of hrs, as he
couldn't get a GP appointment. What do you suggest?
Answer (Stool sample should be sent for giardia, cryptosporidium & other parasities & GP should follow up.)
‫ـــــــــــــــــــــــــــــــــــــــــــــــــــــــــــــــــــــــــــــــــــــــــــــــــــــــــــــــــــــــــــــــــــــــــــــــــــــــــــــــــــــــــــــــــــــــــــــــــــــــــــــــــــــــــــــــــــــــــــــــــــــــــــــــــــــــــــــ‬
8- Question 3 of 50
A 24 yo woman presented with left flank pain which had come on gradually & difficulty passing urine. O/E her temp.
was 38.4 degrees. Urinalysis revealed nitrites, leukocytes & Bl. in her urine.

(a) What is the most likely diagnosis?


Answer (Pyelonephritis.)
(b) How would you further investigate this Pt?(Four steps)
Answer (MSU to laboratory for microscopy, culture & sensitivity. FBC, UE, Bl. cultures, renal ultrasound.)
(c) How would you manage this Pt?(Three steps)
Answer (Antibiotics, IV fluids, analgesia, organise admission or follow up)
(d) What is the most common pathogen to cause this condition?Name another 2 pathogens to cause this
condition?
Answer (Most common- E.Coli. Others include klebsiella, enterococcus, & staph saprophyticus.)
(e) Name three risk factors for this condition?
Answer (Frequency (≥3 times per week) of sexual intercourse in the previous 30 days, UTI within the previous 12
months (OR 4.4), diabetes (OR 4.1), stress incontinence within the previous 30 days (OR 3.9), a new sex partner in
the previous yr (OR 2.2), recent spermicide use (OR 1.7), UTI history in the participant's mother (OR 1.6).)
‫ـــــــــــــــــــــــــــــــــــــــــــــــــــــــــــــــــــــــــــــــــــــــــــــــــــــــــــــــــــــــــــــــــــــــــــــــــــــــــــــــــــــــــــــــــــــــــــــــــــــــــــــــــــــــــــــــــــــــــــــــــــــــــــــــــــــــــــــ‬
9- Question 4 of 50
A 60 yo woman presented to the ED complaining of lethargy & weakness. Her husband reported that she seemed
confused at times over the previous day. She self reported that she had a history of addisons disease. Her medical
records were unavailable as she lived overseas & was visiting her daughter in the area. O/E she had a postural drop in
her Bl. Pr. & her HR was 110bpm.
4
(a) What is the most likely diagnosis?
Answer (Addisonian crises.)
(b) What are the common causes of this condition?(Name 2)
Answer (Withdrawal of longterm steroid therapy, intercurrent injury, infection or stress.)
(c) How would you investigate this Pt?
Answer (FBC, UE, Bl. glucose, Ca, Bl. cultures, urine cultures, sputum culture, CXR.)
(d) What findings would you expect from the electrolyte profile?(Name 2)
Answer (Hyponatremia, hyperkalaemia, uraemia)
(e) How would you manage this Pt?
Answer (Hydrocortisone 100mg IV stat, IV fluids, treat hypoglycaemia if present, treat with broad spectrum
antibiotics if infection is believed to be the precipitant, specialist consultation.)
‫ـــــــــــــــــــــــــــــــــــــــــــــــــــــــــــــــــــــــــــــــــــــــــــــــــــــــــــــــــــــــــــــــــــــــــــــــــــــــــــــــــــــــــــــــــــــــــــــــــــــــــــــــــــــــــــــــــــــــــــــــــــــــــــــــــــــــــــــ‬
10- Question 5 of 50
A 75 yo man was found collapsed at home by his son. There was no available past medical history.He was living
independently & had last been seen 2 days previously by his son. O/E his GCS was 7/15. There was no nuchal
rigidity, pupil reflexes were sluggish but fundi exam. was noraml. Tone was slightly  in all 4 limbs. The peripheral
reflexes were present & plantars were downgoing. The HR was 39 bpm & the Bl. Pr. was 76/42 mmHg. Heart sounds
were normal & the chest was clear. Hypothermia was suspected.

(a) How is hypothermia defined?


Answer (A fall in core temp. below 35 degrees)
(b) How should the temp. be measured?
Answer (Rectal (core) temp. with a low reading thermometer.)
(c) List some clinical features of this condition?(four)
Answer (Impaired consciousness, cardiac embarrassment, bradycardia, hypotension, hypopnoea, sluggish pupillary &
peripheral reflexes, muscle rigidity, coma, ventricular fibrillation, & asystole)
(d) List some ECG features of this condition?(list four)
Answer (Bradycardia, tremor artefact, J-waves, prolonged QT interval, prolonged PR.)
(e) How would you treat this man if the diagnosis is confirmed?
Answer (Gradual rewarming using space blankets. If the Pt is unconsciousness or the temp. is less than 32 warm IV
fluids may be given. The aim is to increase the temp. by one degree/hr.)
‫ـــــــــــــــــــــــــــــــــــــــــــــــــــــــــــــــــــــــــــــــــــــــــــــــــــــــــــــــــــــــــــــــــــــــــــــــــــــــــــــــــــــــــــــــــــــــــــــــــــــــــــــــــــــــــــــــــــــــــــــــــــــــــــــــــــــــــــــ‬
11- Question 2151
An 80 yo man who has CKD 4 is sitting in the waiting area waiting to get a pre-tibial laceration dressed when he
develops palpitations & feels unwell. A nurse kindly puts him in a bay & records an ECG (fig 1)

(a) What do the rhythm strips in figure 1 show?


Answer Torsades de pointes
(b) Name 2 causes for this rhythm
Answer Hypomagnesaemia, hypokalaemia, prolonged QT interval (congenital or drug related)
(c) Name to drugs that could have caused this rhythm?
Answer Sotalol, antipsychotics, antihistamines, antidepressants,
(d) Why is it a concerning pattern?
Answer It may degenerate into VF
(e) How is it treated?
Answer IV magnesium 2g IV over 10 mins
f) It appears to be refractory what do you do?
5
Answer: call cardiologist may require over-drive pacing, can consider an isoprenaline infusion whilst awaiting pacing
‫ـــــــــــــــــــــــــــــــــــــــــــــــــــــــــــــــــــــــــــــــــــــــــــــــــــــــــــــــــــــــــــــــــــــــــــــــــــــــــــــــــــــــــــــــــــــــــــــــــــــــــــــــــــــــــــــــــــــــــــــــــــــــــــــــــــــــــــــ‬
12- Question 18 of 50
A 55 yo man is brought to the ED with haematemesis. His medical history is of alcohol abuse.

(a) Give a differential diagnosis?(give five)


Answer: Peptic ulceration, mucosal inflammation (oesophagitis, gastritis or duodenitis), oesophageal varices,
mallory-weiss tear, gastric carcinoma, coagulation disorders, tumors.
(b) What is the most common cause?
Answer: PUD.
(c) How would you investigate this Pt?(Give five)
Answer: FBC, Coag screen, UE, Bl. glucose, group & cross-match, LFTs.
(d) How would you manage this Pt?
Answer: ABCs, 2 large IV cannulae, IV fluids, consider PPI, keep fasted, gastroenterology or surgical consultation.
(e) If the cause is thought to be oesophageal varices another medication which may be indicated?
Answer: Vasopressin/terlipressin & octreotide/somatostatin
‫ـــــــــــــــــــــــــــــــــــــــــــــــــــــــــــــــــــــــــــــــــــــــــــــــــــــــــــــــــــــــــــــــــــــــــــــــــــــــــــــــــــــــــــــــــــــــــــــــــــــــــــــــــــــــــــــــــــــــــــــــــــــــــــــــــــــــــــــ‬
13- Question 19 of 50
A 26 yo samoan painter & decorator was admitted with acute colicky central abd. pain associated with vomiting. The
only past medical history was of a viral illness associated with a rash 2 wks previously. O/E he was pale, his HR was
120 bpm & his Bl. Pr. was 140/80 mmHg. The abd. was generally tender but there was no guarding & bowel sounds
were present. Exam. of the CNS revealed reduced power & tone in the lower limbs & absent ankle & knee reflexes.
Investigat. were unremarkable apart from a microcytic anaemia. A diagnosis of lead poisoning is being considered.

(a) What investigation would confirm the diagnosis?


Answer: Serum lead concentration
(b) What is the TTT?
Answer: D-penicillamine
(c) Who is at risk for this condition?(Name four)
Answer: Scrap-metal workers, plumbers, individuals ingesting water from lead pipes, children ingesting old lead
based paint in the house, painters & decorators.
(d) What is the differential diagnosis for the above Pt?(Give 2)
Answer: AIP, arsenic poisoning, guillain-barre syndrome, PAN, sarcoidosis, alcohol abuse.
(e) A lead concentration above what value is considered toxic?
Answer: 4mmol/l
‫ـــــــــــــــــــــــــــــــــــــــــــــــــــــــــــــــــــــــــــــــــــــــــــــــــــــــــــــــــــــــــــــــــــــــــــــــــــــــــــــــــــــــــــــــــــــــــــــــــــــــــــــــــــــــــــــــــــــــــــــــــــــــــــــــــــــــــــــ‬
14- Question 21 of 50
A 30 yo woman was brought to the ED after an overdose of amitriptyline.

(a) Name 8 possible clinical features?


Answer: Tachycardia, dry skin, dry mouth, dilated pupils, urinary retention, ataxia, drowsiness, delerium,
hallucinations, dysarthria & jerky limb movements. If the Pt is unconscious they may have increased muscle tone,
increased reflexes, myoclonus, an extensor plantar response, convulsions & a divergent squint.
(b) What Bl. investigations should also be considered? (Give 2)
Answer: Fingerstick glucose, to rule out hypoglycemia as the cause of any alteration in mental status, paracetamol &
salicylate levels to rule out these common co-ingestions, & serum beta-HCG.
(c) What ECG changes might be expected in this condition?
Answer: Sinus tachycardia, increased PR interval, increased QRS complex, ventricular arrhythmias.
(d) How would you manage this Pt?
Answer: ABCs, monitoring, activated charcoal if within one hr, IV lorazepam for seizures, consider treating acidosis
with 8.4% Na bicarbonate, IV fluids & poisons information consultation.
(e) What are the possible complications of Na bicarbonate infusion?(Give 2)
Answer: Anaphylaxis, volume overload, hypernatremia, & metabolic alkalosis may result from prolonged
bicarbonate infusion. Other listed adverse effects include cerebral hemorrhage, CHF (aggravated), oedema, tetany,
gastric distension, hypernatremia, hyperosmolality, hypocalcemia, hypokalemia, intracranial acidosis & pulm. edema.
‫ـــــــــــــــــــــــــــــــــــــــــــــــــــــــــــــــــــــــــــــــــــــــــــــــــــــــــــــــــــــــــــــــــــــــــــــــــــــــــــــــــــــــــــــــــــــــــــــــــــــــــــــــــــــــــــــــــــــــــــــــــــــــــــــــــــــــــــــ‬
15- Question 22 of 50
A 28 yo gentleman from Poland attends after falling over in the garden & cutting his finger on a piece or metal that
was supporting a plant in his vegetable patch. The wound appears to be trivial.

6
(a) What important questions do you need to illicit in the history? (2 marks)
Answer: Essentially is it a tetanus prone wound? How old is the wound, was there any manure in the ground? Has he
had immunisations against tetanus?
(b) What are the signs of tetanus infection & at what stage after sustaining a wound do they present? (2 marks)
Answer: Presentat.: is 4-21 days (average 10) after infect., with agonising contract. superimposed on muscular rigidity
(c) What features constitute a tetanus prone wound? List 5 things.
Answer: a significant degree of devitalised tissue, puncture type wound, contact with soil or manure, clinical
evidence of sepsis occurring more than 6 hrs before presentation.
‫ـــــــــــــــــــــــــــــــــــــــــــــــــــــــــــــــــــــــــــــــــــــــــــــــــــــــــــــــــــــــــــــــــــــــــــــــــــــــــــــــــــــــــــــــــــــــــــــــــــــــــــــــــــــــــــــــــــــــــــــــــــــــــــــــــــــــــــــ‬
16- Question 23 of 50
A 69 yo smoker who lives alone is brought in acutely dyspnoeic by the crew. Initial observations show the she is
drowsy GCS 13. RR 33, HR 146, BP 78/47. They let you know that according to her next door neighbour she only
came home from hospital last week & hasn't left the house since. She has O2 at home & is on lots of medication.
Apparently she is awaiting placement in a nursing home as she can no longer manage with 3 calls a day. You
diagnose a severe exacerbation of COPD & are not concerned about sepsis.

(a) Your initial impression is that the Pt is peri-arrest. List 3 important things that you need to try to do in the
next 5 minutes in order of priority.
Answer: 1. Treat what you can treat i.e. ABCDE assessment gain IV access, take Bl. off etc. Try to ascertain a
diagnosis an ABG will be very helpful as will a CXR (may be too unstable?) 2. Get the arrest trolley out & organise
your team. 3. Try to speak with any family or the GP if possible, track down hospital notes; is there a plan in place for
this Pt if she should become very unwell?
(b) You notice that the O2 is flowing at 15 litres from the wall supply via a non-rebreathe mask. Is this of
relevance to the Pt's condition?
Answer: It might be, You need to access if the Pt is adequately O2ating & ventilating. The sats probe will help with
the former but not the later. An ABG will guide you. It is possible that she has been over oxygenated on route to
hospital & that the CO2 is raised causing the lowered GCS. Turning down the O2 may improve the Pt's condition.
(c) What amount of O2 should you give this type of Pt prior to obtaining an ABG?
Answer: This is clearly not an exact science but it is much better to start low & titrate up when the history points
towards COPD. If the Pt is known to have had hypercapnic respiratory failure in the past then give an FiO2 of 24%
via a venture mask. For all other Pts & when the diagnosis is unclear give 40% FiO2 until an ABG has been obtained.
(d) List 3 therapies that you gave the Pt on admission.
Answer: Salbutmaol nebs 2.5mg or 5 mg, Ipratropium nebs 500mcg & steroids, prednisolone 30mg if could swallow
(unlikely) therefore 200mg of IV hydrocortisone. Stat dose of doxycycline also given 200mg.
(e) The medical registrar demands that this Pt is put on NIV Rt now & sent straight to medical HDU. The Pt s
observations have now worsened. What is your response?
Answer: No. The Pt is clearly unstable is peri-arrest & would not tolerate NIV at present. Moving the Pt would be
catatrophic. ITU need to be involved with this Pt. If a decision is made that invasive ventilation is not appropriate then
a trial of NIV is an option although it may not be successful.
‫ـــــــــــــــــــــــــــــــــــــــــــــــــــــــــــــــــــــــــــــــــــــــــــــــــــــــــــــــــــــــــــــــــــــــــــــــــــــــــــــــــــــــــــــــــــــــــــــــــــــــــــــــــــــــــــــــــــــــــــــــــــــــــــــــــــــــــــــ‬
17- Question 24 of 50

Fig 1 Fig 2:
(a) Describe how the QT interval is measured
Answer: From the start of the Q wave to the end of the T wave. See fig 1
(b) What dangerous arrhythmias can be precipitated from long QT intervals?
7
Answer: Torsades de Pointes ,VF & hence sudden cardiac death
(c) The QT interval gets shorter as the HR speeds up & longer as it slows down. What is the QTc & why's it important?
Answer: It is the corrected QT interval i.e. it takes the rate out of the equation. Normal range is <440 ms. See fig 2
(d) There are 2 types of LQTS congenital & acquired. Name 2 causes of acquired LQTS
Answer: 1- Antiarrhythmics: Quninidine, procainamide, disopyramidine, flecanide, propafenone, sotalol, ibutilide,
dofetilide, amaiodarone (rare) 2- Antimicrobials: Erythromycin, clarithromycin, trimethoprim, ketoconazole,
itraconazole, choloroquine. 3- Antihistamines: terfenadine Electrolyte imbalances Severe bradycardia
(e) Name another cause of sudden cardiac death (SCD)?
Answer: 1- Hyperthrophic cardiomyopathy (HCM), risk of SCD is  with early age of diagnosis, family hx of SCD,
Non-sustained VT on 24hy tape, Abnormal BP in response to exercise, certain genetic mutations. 2- Arrhythmogenic
Rt Ventricular Cardiomyopathy (ARVC) is probably the 2nd most common cause of unexpected sudden death in the
young. 3- DCM 4- Restrictive Cardiomyopathy is the rarest of the cardiomyopathies. 5- Myocarditis 6- Brugada
Syndrome 7- Progressive Cardiac Conduction Defect (Lev-Lenegre's Syndrome) 8- Idiopathic VF (without Brugada
ECG changes) Catecholaminergic Polymorphic VT
‫ـــــــــــــــــــــــــــــــــــــــــــــــــــــــــــــــــــــــــــــــــــــــــــــــــــــــــــــــــــــــــــــــــــــــــــــــــــــــــــــــــــــــــــــــــــــــــــــــــــــــــــــــــــــــــــــــــــــــــــــــــــــــــــــــــــــــــــــ‬
18- Question 25 of 50
A 39 yo woman is brought into the department acutely SOB c/o of chest pain (worse on inspiration) She has a RR of
45 & her sats are 89% in air. She was completely well 30 minutes ago but collapsed suddenly at work

(a) Give 3 differential diagnoses?


Answer: 1. Massive PE 2. Pneumonia 3. Cardiac collapse, M.I.; now with arrhythmia? 4. Sepsis
(b) Name 3 crucial tests that need to be done.
Answer: 1. ABG 2. CXR 3. ECG 4. Bedside ECHO
(c) Are d-dimers useful (comment on specificity & sensitivity?
Answer: It depends; they need to be interpreted within the realms of clinical probability. Current BTS guidelines are
as follows: If Pt has features consistent with PE namely a): Raised RR +/- haemoptysis +/- pleuritic chest pain Plus 2
other factors: 1. Absence of another reasonable clinical explanation. 2. Presence of a major risk factor. If Pt has: a)
plus 1&2: HIGH pre-test probability b) plus 1 or 2: INTERMEDIATE pre-test probability c) alone: LOW pre-test clinical
probability There are 2 assays simpliRED & VIDAS. They vary in their sensitivity simpliRED 99% VIDAS 87%. Specificity is
however poor 60-70% so both = high false +ve rates. Basically a -ve d-dimer in Pts who were low (simpliRED) or
intermediate (VIDAS) don't need further imaging. A -ve test isn't useful where the pre-test probability is high (so shouldn't be
done)These pts will need imaging anyway! D-dimer becomes less reliable the longer a pt has been in hospital.
(d) The Pt arrests in front of you. Rhythm is PEA. You start the resuscitate according to current ALS guidelines.What will
you do?
Answer: Need to consider thrombolysis in this young Pt you has likely had a massive P.E. BTS guidelines would
support thrombolysis where clinical probability suggests massive P.E. causing cardiovascular collapse.
(e) List the 4 H's of cardiac arrest
Answer: Hypoxia, Hypovolaemia, Hypothermia, Hyperkalaemia/Hypokalaemia, Hypomagnesaemia (metabolic etc)
‫ـــــــــــــــــــــــــــــــــــــــــــــــــــــــــــــــــــــــــــــــــــــــــــــــــــــــــــــــــــــــــــــــــــــــــــــــــــــــــــــــــــــــــــــــــــــــــــــــــــــــــــــــــــــــــــــــــــــــــــــــــــــــــــــــــــــــــــــ‬
19- Question 26 of 50
A 36 yo preg. lady presents suicidal stating that she took an overdose 5 hrs ago. She wants to kill herself & her unborn child

(a) What important things do you need to establish from the Pt?
Answer: The amount taken Is she high risk for toxicity i.e. on any liver enzyme inducing drugs Is she anorexic,
alcoholic, HIV +ve, malnourished (as all of these things lead to a decreased glutathione store).
(b) How will the fact that she is pregnant affect your choice of antidotes?
Answer: It won t; still treat the same. Parvolex & Methionine have no harmful effects to the fetus.
(c) How do you approach the Pt that has taken a staggered overdose of paracetamol?
Answer: You need to base it on the time since the 1st OD, its easy if the Pt has taken >150mg/kg they need to be
treated. If not then you need to take Bl. for paracetamol levels & check U&E, LFTs, clotting, & paracetamol levels. This proves if
any paracetamol has actually been taken, if it has then you need to treat. If in any doubt start TTT & obtain expert advice
(d) List the symptoms of late overdose
Answer: Late presenters are more likely to have taken larger significant overdoses, they may have severe abd. pain &
vomiting which are symptoms of acute liver failure. Take caution when using the normogram as there is insufficient
data on Pts who present >15 hrs after ingestion
(e) Using the modified sad persons scale. what score would mandate a psychiatric opinion?
Answer: 6 - 8
‫ـــــــــــــــــــــــــــــــــــــــــــــــــــــــــــــــــــــــــــــــــــــــــــــــــــــــــــــــــــــــــــــــــــــــــــــــــــــــــــــــــــــــــــــــــــــــــــــــــــــــــــــــــــــــــــــــــــــــــــــــــــــــــــــــــــــــــــــ‬
20- Question 27 of 50
You perform a CXR on a 59 yo man you presents with SOB
8
(a) Describe what it shows: (fig 1)
Answer: Large Rt sided pleural effusion
(b) He doesn't have heart failure, you decide to do a pleural tap for diagnostic purposes, what do you need to
send samples for?
Answer: Check pH (can use Bl. gas syringe), Send to micro in culture bottles & plain tube for gram stain MC+S also
for AFB cultures. Send plain sample pot to biochemistry for LDH & protein. Send as much as you can in a sterile pot for cytology
(c) The protein is 29g/L, how do you work out if it is a transudate or an exudate?
Answer: Using Light's criteria; effusion is an exudates if it meets 1 of the following criteria: If the fluid protein/serum
protein ratio >0.5 Pleural fluid LDH/serum LDH ratio >0.6 Pleural fluid LDH> 2/3 the upper limit of normal serum LDH
(d) List the 4 most common causes for a pleural effusion in the Uk
Answer: Cardiac failure, pneumonia, malignancy, pulmonary embolus
(e) The pH comes back at 7.16 what will you do?
Answer: Needs an intercostal drain, empyema unlikely to resolve without drainage
‫ـــــــــــــــــــــــــــــــــــــــــــــــــــــــــــــــــــــــــــــــــــــــــــــــــــــــــــــــــــــــــــــــــــــــــــــــــــــــــــــــــــــــــــــــــــــــــــــــــــــــــــــــــــــــــــــــــــــــــــــــــــــــــــــــــــــــــــــ‬
21- Question 28 of 50
A man is brought in who has been involved in a , which makes mattresses. He looks drowsy but is rousable he is
complaining of a headache & feeling dizzy.

(a) You do a Bl. gas, which reveals a CO level of 17%. You’re concerned about CO poisoning. What do you do?
Answer: This represents possible severe poisoning Give high flow O2 Do an ECG check for arrhythmias & or MI If
becomes unconscious the consider IPPV Consider hybebaric O2 if there is a centre nearby
(b) The ABG showed a profound met. acidosis a nurse thinks that is breath smells sweet what do you consider?
Answer: Although the detection of almond smelling breath is not reliable 50% of people cannot smell it this may
represent cyanide poisoning
(c) What antidotes could you use if your suspicions above are correct?
Answer: Dicolbat edetate or Na thiosulphate
(d) What level of CO can smokers have in normality?
Answer: Around 8% would be a normal level
(e) What must you be careful of when treating this Pt?
Answer: Avoid getting contaminated yourself. Ensure that Protective clothing is worn!
‫ـــــــــــــــــــــــــــــــــــــــــــــــــــــــــــــــــــــــــــــــــــــــــــــــــــــــــــــــــــــــــــــــــــــــــــــــــــــــــــــــــــــــــــــــــــــــــــــــــــــــــــــــــــــــــــــــــــــــــــــــــــــــــــــــــــــــــــــ‬
22- Question 29 of 50
You are asked to see a 44 yo immediately who has a GCS of 7/15.
(a) You clear ABC & move to assessing D. His pupils are equal, normal sized & reactive. What do you do? You
have no history, he was found like this
Answer: Need to establish why GCS is 7, need to assess the need for airway protection Check BMG Look for
evidence of opiate use Look for medi alert bracelet Look for signs of head injury Look for any focal neurological
signs suggestive of CVA or SAH Evidence of ETOH? Evidence of any other overdose? Insulin?
(b) You decide to do a Bl. gas. It is normal apart form the glucose reads 1mmol/Litre. What is your
management?
Answer: Due to low GCS likely will not be able to give oral glucose therefore needs IV glucose, current
recommendations are 50mls of 10% glucose (previously 50mls of 50%) Different in different hospitals, author
advocates using 20% glucose. Glucagon 1mg IM/IV or SC Reassess BM after 5 minutes constantly reassess GCS
(c) What risk factors are there giving IV glucose & how can they be minimised?
Answer: Risk of thrombophlebitis, extravasation can cause severe tissue necrosis, can result in loss of limb in
extreme cases. Reduce the risk by using lower concentration of IV glucose.
(d) You find out from Pt’s wife that she thinks that he deliberately took an insulin overdose. What will you do?
Answer: Needs to be managed on ITU/HDU May need to be on a sliding scale for 24 hrs. Hypokalaemia can be
problematic Block excision of the injection site has been used as successful TTT for insulin OD but there is no clear
cut evidence that it works
(e) You reassess but after 15 minutes the GCS is only 8/15. What do you need to consider now?
Answer: Could there be another cause? CVA etc Or might represent development of cerebral oedema due to
hypoglycaemia, which has a high mortality. Will need urgent imaging of the brain
‫ـــــــــــــــــــــــــــــــــــــــــــــــــــــــــــــــــــــــــــــــــــــــــــــــــــــــــــــــــــــــــــــــــــــــــــــــــــــــــــــــــــــــــــــــــــــــــــــــــــــــــــــــــــــــــــــــــــــــــــــــــــــــــــــــــــــــــــــ‬
23- Question 30 of 50
A 72 yo man is phoned through presenting with chest pain, the crew have thrombolysed him as he had ST elevation

(a) Name some different thrombolytics

9
Answer: Streptokinase, alteplase (rtPA), retaplase (modified rtPA), tenecteplase (mosified rtPA).
(b) Name a some side effects of the first thrombolytic agent
Answer: Allergenic reaction to streptokinase, Causes hypotension. Also can’t be used again as antibodies are
produced against it
(c) What are the requirements for thrombolysis?
Answer: >1 mm ST elevation in the limb leads or >2mm in 2 contiguous chest leads or LBBB (with typical M.I.
history NB DOES NOT HAVE TO BE NEW!)
(d) What are the anterior leads?
Answer: V1-V3 = anteroseptal, V2-V4 = anterior, V5-V6 = anterolateral
(e) What is a Rt ventricular infarct? What type of M.I is it likely to occur with? 6.
Answer: When the Rt ventricle is taken out by an inferior M.I. ST elevation in V1 with inferior M.I. suggest it,
especially if it is greater then in V2 & V3.
f) How do you diagnose it & what is it important to treat it with?
Answer: Perform ECG with V4R. Ensure that IV fluid is given to maintain adequate filling Pr. in Rt ventricular
failure. 40% of Pts with inferior wall infarctions have Rt ventricular &/or posterior wall involvement, which
predisposes them to more complications & increased mortality
‫ـــــــــــــــــــــــــــــــــــــــــــــــــــــــــــــــــــــــــــــــــــــــــــــــــــــــــــــــــــــــــــــــــــــــــــــــــــــــــــــــــــــــــــــــــــــــــــــــــــــــــــــــــــــــــــــــــــــــــــــــــــــــــــــــــــــــــــــ‬
24- Question 2130
35 yo presents with tingling of the hands & feet, she has been feeling very weak over the last 2/7. Knee reflexes are
absent. There is blurring of the vision & diplopia on lateral gaze. She admits to a recent URTI

(a) Give the nerve roots for the reflexes:


Answer: Jaw: trigeminal V Biceps: The biceps & brachioradialis reflexes are mediated by the C5 & C6 nerve roots
Triceps The triceps reflex is mediated by the C6 & C7 nerve roots, predominantly by C7. Supinator C5-6 Knee: The knee
jerk reflex is mediated by the L3 & L4 nerve roots, mainly L4. Ankle: The ankle jerk reflex is mediated by the S1 nerve root.
(b) What is the most likely diagnosis
Answer: Guillain Barre Syndrome Antecedent illness Up to 2/3 of Pts with GBS report an antecedent illness or event
1-3 wks prior to the onset of weakness. Upper resp. & GIT illnesses are the most commonly reported condit.
Symptoms generally have resolved by the time of medical presentat. for the neurologic condit. Ophthalmoparesis may
be observed in up to 25% of Pts with GBS. The most common limitation of eye movement is from a symmetric palsy
associated with cranial nerve VI Lower extremity weakness usually begins 1st & ascends symmetrically &
progressively over the 1st several days. Upper extremity, trunk, facial & oropharyngeal weakness is observed to a
variable extent. Marked asymmetric weakness calls the diagnosis of GBS into question. Despite frequent complaints
of paresthesias, objective sensory changes are minimal. Reflexes are absent or hyporeflexic early in the disease course
& represent a major clinical finding O/E of the Pt with GBS
(c) Give 2 differential diagnoses
Answer: Poliomyelitis, Nutritional neuropathies, Toxic neuropathies (eg, arsenic, thallium, organophosphates, lead),
Multifocal motor neuropathy, Mononeuritis multiplex, Critical illness polyneuropathy, Botulism, Vasculitic
neuropathies, Diphtheritic polyneuritis, Acute myasthenia gravis.
(d) Name three things to do in the ED to get the diagnosis
Answer: Speak to neurologist Spirometry Frequent evaluations of these parameters should be performed at bedside to
monitor respiratory status & the need for ventilatory assistance LP The  in CSF protein is thought to reflect the
widespread inflammatory disease of the nerve roots MRI brain Imaging studies such as MRI or CT scan of the spine
may be more helpful in excluding other diagnoses, such as mechanical causes of myelopathy, than in assisting in the
diagnosis of GBS Nerve conduction studies: EMG studies can be very helpful in the diagnostic workup of Pts with
suspected GBS. Abnormalities in the NCS consistent with demyelination are sensitive & represent specific findings
for classic GBS Basic laboratory studies, such as complete Bl. counts & metabolic panels, are of limited value in the
diagnosis of GBS. They often are ordered, although, to exclude other infectious or metabolic causes of the weakness
(e) What needs to be monitored whet the Pt is admitted?
Answer: FVC to see any deterioration in respiratory function
‫ـــــــــــــــــــــــــــــــــــــــــــــــــــــــــــــــــــــــــــــــــــــــــــــــــــــــــــــــــــــــــــــــــــــــــــــــــــــــــــــــــــــــــــــــــــــــــــــــــــــــــــــــــــــــــــــــــــــــــــــــــــــــــــــــــــــــــــــ‬
25- Question 32 of 50
A 29 yo Nepalese man presented with haemoptysis. He had moved to the UK 2 months previously to train in hotel
management.There was no history of trauma

(a) How could you confirm with that the Bl. is from the lungs & not the stomach?(Give 2 methods)
Answer: Alkaline pH, foaminess, or the presence of pus may sometimes suggest the lungs as the 1ry source of
bleeding rather than the stomach
(b) What is the differential diagnosis?(Give eight)
10
Answer: Infection(URTI,pneumonia,TB,lung abscess), carcinoma, bronchiectasis, pulmonary oedema, PE, inherited
or acquired coagulation disorder, wegener's granulomatosis, goodpastures syndrome.
(c) How would you investigate this Pt?(Give eight)
Answer: FBC, Coag screen, UE, LFTs, Bl. group & crossmatch, ABG, SpO2, CXR, ECG, Sputum M/C/S & dipstick urine
(d) How would you manage this Pt?(Give four)
Answer: ABCs, O2, suction, face mask, 2 large bore IV cannulae, IV fluids, Bl. transfusion if indicated,correct
coagulopathy, respiratory consultation
(e) How would you define massive haemoptysis & what is its significance?
Answer: Massive hemoptysis is variably defined as expectoration of Bl. exceeding 100 to 600 mL over a 24-hr
period. Although only 5% of haemoptysis is massive some studies report a mortality rate of up to 80% in this group.
Cahill, BC, Ingbar, DH. Massive hemoptysis. Assessment & management. Clin Chest Med 1994; 15:147
‫ـــــــــــــــــــــــــــــــــــــــــــــــــــــــــــــــــــــــــــــــــــــــــــــــــــــــــــــــــــــــــــــــــــــــــــــــــــــــــــــــــــــــــــــــــــــــــــــــــــــــــــــــــــــــــــــــــــــــــــــــــــــــــــــــــــــــــــــ‬
26- Question 33 of 50
A 70 yo man was brought in by his family as he was feeling generally unwell. Bl. investigations revealed a potassium
level of 7.1 mmol/litre.His only medical history was of HTN for which he had recently been started on a medication.
He had no known history of hyperkalaemia

(a) Give a differential diagnosis for hyperkalaemia(Give four)?


Answer: ARF, CRF, potassium sparing diuretics, crush injury, burns, tumor cell necrosis, acidosis from any
cause(potassium cellular shift), drugs such as suxamethonium & beta-blockers, addisons disease, haemolysis of the
sample or a sample taken from a limb with a potassium infusion
(b) What medication for his HTN might this man have been started on which may exacerbate his
hyperkalaemia
Answer: Amiloride or spironolactone
(c) What are the clinical features of this condition?(Name three)?
Answer: Muscle weakness, muscle cramps, paraesthesiae, hypotonia & may cause focal neurological deficits
(d) What ECG features may be present?(Give four)
Answer: Peaked T waves,small broad or absent P waves, widening QRS complex, sine wave pattern QRST,
ventricular tachycardia/ventricular fibrillation.
(e) How would you manage this Pt?
Answer: Confirm result, monitored ECG, 10ml 10% Ca chloride, 10units Actrapid with 50ml of 50% dextrose, 5mg
nebulised salbutamol, IV fluids, treat underlying cause, specialist consultation
‫ـــــــــــــــــــــــــــــــــــــــــــــــــــــــــــــــــــــــــــــــــــــــــــــــــــــــــــــــــــــــــــــــــــــــــــــــــــــــــــــــــــــــــــــــــــــــــــــــــــــــــــــــــــــــــــــــــــــــــــــــــــــــــــــــــــــــــــــ‬
27- Question 34 of 50
A 21 yo girl is brought in to the department looking desperately unwell, she has been at an all night rave & has taken
7 ecstasy tablets she is pale & hot but is lucid

(a) She then becomes unresponsive, you fluid resuscitate her & check her BM which is normal. You send off a
full set of investigations, which show the following: INR 3.4, WBC 18.3, Ur 14 Cr 312, temp 38.2, urine dip +ve
for Bl., CK 1203. pH 6.31 What is going on?
Answer: Rhabdodyloysis from raving all night & ecstasy. She has also developed DIC. Acute renal failure
(b) Where should this Pt go & what should be done?
Answer: ITU, Prompt correction of fluid deficits & acidosis are crucial. Will likely need renal support
(c) What electrolytes can easily become deranged & need to be corrected in this condition?
Answer: K+ & Ca2+
(d) Give 3 other causes of the conditon:
Answer: compartment syndrome direct injuries & severe burns exertional: raving, fitting, metabolic disorders:
myxodema, neuroleptic malignant syndrome, myositis due to infection
‫ـــــــــــــــــــــــــــــــــــــــــــــــــــــــــــــــــــــــــــــــــــــــــــــــــــــــــــــــــــــــــــــــــــــــــــــــــــــــــــــــــــــــــــــــــــــــــــــــــــــــــــــــــــــــــــــــــــــــــــــــــــــــــــــــــــــــــــــ‬
28- Question 35 of 50
A 15 yo male presented to the ED with sudden onset of rapid palpitations which were not associated with chest pain
or dizziness.O/E he was well perfused & his Bl. Pr. was 120/80 mmHg. His ECG revealed an SVT.

(a) List 2 forms of TTT which may be attempted?


Answer: Vagotonic manoeuvres, IV adenosine
(b) After TTT the Pt's rest ECG revealed WPW. What are the ECG features of this condition?
Answer: Short PR interval, a wide QRS complex with a slurred upstroke(delta wave).
(c) What is the underlying pathology of this condition?
Answer: An accessort conduction pathway between th atria & ventricles.

11
(d) What is the definitive TTT in a young Pt with this disorder?
Answer: Radiofrequency ablation of the accessory pathway.
(e) What medication should be avoided in WPW?
Answer: Both digoxin & verapamil are contraindicated in WPW as they  the risk of malignant ventricular agents
‫ـــــــــــــــــــــــــــــــــــــــــــــــــــــــــــــــــــــــــــــــــــــــــــــــــــــــــــــــــــــــــــــــــــــــــــــــــــــــــــــــــــــــــــــــــــــــــــــــــــــــــــــــــــــــــــــــــــــــــــــــــــــــــــــــــــــــــــــ‬
29- Question 36 of 50
You see an 84 yo lady who lives alone at home who is acutely SOB. You suspect that she's in failure The paramedics
have surprisingly good notes on this lady as she was only discharged 2 days ago from a health care for the elderly
ward following a UTI. She was found to be in AF & had an echo, which showed normal lt vent. Funct. with a good EF

(a) On clinical exam. you here basal crepitations, she has a RR of 38 & sats of 92%. The chest x-ray has
widespread air space shadowing with upper lobe diversion. What is going on?
Answer: 2 main options: 1 either has non-cardiogenic pulmonary oedema or 2 has diastolic heart failure
(b) Name 4 causes of acute pulmonary oedema other than heart failure
Answer:  pulm. capillary Pr. (hydrostatic):  lt atrial Pr.: Mitral valve disease, atrial myxoma, arrhythmias.  lt vent.
end diastolic Pr.: Ischaemia, aortic valve disease, cardiomyopathy, uncontrolled HTN, fluid overload, high output
states Neurogenic: IC hge, cerebral oedema, post-ictal HAPE (rare obviously unless been up Everest recently)
Increased pulmonary capillary permeability ARDS Hypoalbuminaemia
(c) Explain the pathophysiology of diastolic heart failure
Answer: Essentially it occurs in the elderly who are hypertensive with LV hypertrophy, the ventricle has impaired
relaxation in diastole this leads to pulmonary oedema. With tachycardia diastolic filling time shortens & as the
ventricle is stiff in diastole left atrial Pr. is increased & pulmonary oedema occurs
(d) How would you manage a Pt in pulm. oedema who you new had a prosthetic mitral valve if they didn't
respond to initial therapy?
Answer: Need to involve cardiologist & cardiothoracic surgeon. Emergency thransthroacic or TOE to confirm
diagnosis of presumed prosthetic valve failure
‫ـــــــــــــــــــــــــــــــــــــــــــــــــــــــــــــــــــــــــــــــــــــــــــــــــــــــــــــــــــــــــــــــــــــــــــــــــــــــــــــــــــــــــــــــــــــــــــــــــــــــــــــــــــــــــــــــــــــــــــــــــــــــــــــــــــــــــــــ‬
30- Question id: 2050
A 66 yo woman presented to the ED with a 3 week history of progressive SOB & purulent cough. She had
tuberculosis treated in East Timor 24 yrs previously. O/E her O2 saturation was 92% on room air.

(a) Describe the main finding in the the CXR shown?Name 2 other findings which are seen in CXR of Pt with TB?
Answer: There is a Rt upper zone opacity with cavitation. Other findings include hilar adenopathy, sometimes associated with
Rt middle lobe collapse, infiltrates or cavities in the middle or lower lung zones, pleural effusions, solitary nodules.
(b) Besides tuberculosis give a differential diagnosis of 2 other conditions?
Answer: Neoplasm, pneumonia.
(c) What further investigations should be carried out in the ED?(Name three)
Answer: Full Bl. count, urea & electrolytes, random sample sputum staining for acid-fast bacilli & Bl. cultures
(d) How should this lady be managed in the ED?(three steps)
Answer: Isolation with barrier nursing, -ve Pr. room if available, supplemental O2 therapy & respiratory consultation

(e) Name 2 complications of this condition?


Answer: Haemoptysis, pneumothorax, bronchiectasis & extensive pulmonary destruction.
‫ـــــــــــــــــــــــــــــــــــــــــــــــــــــــــــــــــــــــــــــــــــــــــــــــــــــــــــــــــــــــــــــــــــــــــــــــــــــــــــــــــــــــــــــــــــــــــــــــــــــــــــــــــــــــــــــــــــــــــــــــــــــــــــــــــــــــــــــ‬
12
31- Question 38 of 50
A 74 yo man deliberately ingested 20 sulphonylurea tablets

(a) What are the clinical features of sulphonylurea overdose?(Name four)


Answer: Hypoglycaemia(confusion, difficulty speaking, dizziness, hemiparesis, seizures or coma, anxiety, nausea,
sweating & palpitations) hypokalaemia
(b) How would you investigate this Pt?(Give four)
Answer: UE, Bl. glucose, check for paracetamol & salicylate level, ECG
(c) How would you manage this Pt?(Give four)
Answer: Observe for at least 24 hrs, oral or IV glucose as needed, correct hypokalaemia, consider octreotide, expert
advice in severe poisoning, & mental health consultation.
(d) How would the management of this Pt be different if he had renal failure?
Answer: Renal failure results in impaired drug clearance & this increases the risk of hypoglycaemia
(e) Why may octreotide be indicated?
Answer: Octreotide blocks pancreatic insulin release.
‫ـــــــــــــــــــــــــــــــــــــــــــــــــــــــــــــــــــــــــــــــــــــــــــــــــــــــــــــــــــــــــــــــــــــــــــــــــــــــــــــــــــــــــــــــــــــــــــــــــــــــــــــــــــــــــــــــــــــــــــــــــــــــــــــــــــــــــــــ‬
32- Question 39 of 50
A 45 yo woman presented with pleuritic chest pain.

(a) What is the differential diagnosis of pleuritic chest pain?(Give four)


Answer: PE, pericarditis, viral pleurisy, Pneumonia, Pneumothorax, Collagen vascular diseases, including systemic
lupus erythematosus, mixed connective tissue disease & rheumatoid arthritis, drug-induced lupus, inflammatory
bowel disease, familial Mediterranean fever & radiation pneumonitis
(b) How would you investigate this Pt?(Give four)
Answer: ECG, D-Dimer, CXR, FBC, ABG, BNP, troponin, CTPA, V/Q Scan
(c) What %age of Pts with a PE have abnormal D-Dimer value?(not including sub-segmental PE)
Answer: D-dimer levels are abnormal in 95% of Pts with PE.They are abnormal in only 50% of Pts with subsegmental PE.
(d) If PE is confirmed how would you manage this Pt?(three steps)
Answer: O2, hemodynamic support, analgesia, & anticoagulation
(e) What is the prognosis of untreated PE?
Answer: PE is associated with a mortality rate of approximately 30% without TTT, primarily the result of recurrent embolism
‫ـــــــــــــــــــــــــــــــــــــــــــــــــــــــــــــــــــــــــــــــــــــــــــــــــــــــــــــــــــــــــــــــــــــــــــــــــــــــــــــــــــــــــــــــــــــــــــــــــــــــــــــــــــــــــــــــــــــــــــــــــــــــــــــــــــــــــــــ‬
33- Question 40 of 50
A 34 yo gentleman attends with sudden onset of left sided chest pain worse on inspiration. His sats are 94% in room
air. You consider the diagnosis of a spontaneous pneumothorax

(a) What element of the social history that you don't have would support the likelihood of a spontaneous pneumothorax?
Answer: Smoking the lifetime risk of developing a pneumothorax in healthy smoking men may be as much as 12%
compared with 0.1% in non-smoking men.1
(b) What is a 1ry pneumothrax compared to a 2ry one?
Answer: 1ry pneumothoraces arise in otherwise healthy people without any lung disease. 2ry pneumothoraces arise in
subjects with underlying lung disease.
(c) You perform a CXR which confirms your diagnosis, which 2 features would lead you to considering
aspiration according to current BTS guidelines?
Answer: If the rim was >2cm from the chest wall or if the Pt was breathless as a result of it.
(d) You attempt aspiration but it is unsuccessful, what would you do next?
Answer: Could consider repeat aspiration or if that fails again insert an intercostal drain
(e) Explain why it is crucial to obtain an erect chest x-ray in Pts with a suspected pneumothorax
Answer: On supine CXR lung markings will extend to the chest wall as air in the pleural cavity moves anteriorly. A lateral
or lateral decubitus CXR should be performed if the clinical suspicion of pneumothorax is high, but a PA CXR is normal
(f) What type of x-ray may be of benefit in these cases?
Answer: lateral decubitus film.
(g) In an older Pt with underlying COPD who develops a spontaneous pneumothorax that is 4 cm in size what
condition must you be wary of post aspiration?
Answer: re-explansion pulmonary oedema, this is especially important if the Pt has waited a few days before
seekingmedical attention as the incidence is higher the longer the lung has been collapsed.
‫ـــــــــــــــــــــــــــــــــــــــــــــــــــــــــــــــــــــــــــــــــــــــــــــــــــــــــــــــــــــــــــــــــــــــــــــــــــــــــــــــــــــــــــــــــــــــــــــــــــــــــــــــــــــــــــــــــــــــــــــــــــــــــــــــــــــــــــــ‬
34- Question 41 of 50
A 61 yo lady is sitting in the cubicle area on a trolley, you go to see her & think that she looks unwell, she is sweaty
13
clammy & tachycardic. You re-check her observations: she has a pulse of 105 sats of 96% in air & BP 0f 145/70. She
is with her partner who says that she fitted earlier & that's why they have come in. You find out that she is an
alcoholic & hasn't drunk for 2 days now

(a) What is going on?


Answer: She appears to be withdrawing from alcohol. Although the differential diagnosis is wide & one needs to
constantly assess & monitor this Pt
(b) How would you treat this situation?
Answer: benzodiazepines normally chlordiazepoxide 20mg but higher doses may be required to control symptoms.
Maximum does of 200mg in 24 hrs. Also IV vitamin complexes (Pabrinex) will need vitamin B co-strong & thiamine.
(c) What is delirium tremens?
Answer: It is a medical emergency occurring in alcohol withdrawal. The Pt may have all the signs of withdrawal but
in addition have hallucinations, sinister delusions, confusion & disorientation. Deaths occur from arrhythmias (2ry to
acidosis, electrolyte disturbances or alcohol related cardiomyopathy).
(d) Explain the type of picture you might see on an ABG of a Pt that you suspect has alcoholic ketoacidosis
Answer: It develops from an alcoholic withdrawing, not eating & vomiting repeatedly. A metabolic acidosis is
normally seen with a high anion gap but the pH is actually variable as vomiting could lead to an alkalosis & they
could have an element of respiratory alkalosis
(e) The magnesium you sent earlier comes back at 0.017 what will you do?
Answer: IV replacement but not too fast (likely chronically low) cannot do too much harm by giving high doses.
good place to start is to give 8mmol of Mg sulphate over 20 min then to give another 8 mmols over 4-6 hrs then re-
checking the level. Ensure that all other electrolytes are checked carefully this Pt should be managed on HDU
‫ـــــــــــــــــــــــــــــــــــــــــــــــــــــــــــــــــــــــــــــــــــــــــــــــــــــــــــــــــــــــــــــــــــــــــــــــــــــــــــــــــــــــــــــــــــــــــــــــــــــــــــــــــــــــــــــــــــــــــــــــــــــــــــــــــــــــــــــ‬
35- Question 42 of 50
A 56 yo lady has taken 56 of her amitriptyline tablets

(a) List features that would commonly be apparent in a Pt who had done this?
Answer: Dry mouth Tachycardia Dry skin Dilated pupils Ataxia Urinary retention Jerky limb movements Coma
(b) What dose is toxic?
Answer: When >10mg/kg is taken
(c) What ECG changes can be seen?
Answer: Sinus tachycardia is common, with severe poisoning PR & ORS duration increase. The rhythm can look like
VT as the P waves are superimposed on the preceding T wave & the QRS duration is prolonged. Any arrhythmia can
occur & bradycardia normally indicates a per-arrest scenario
(d) She becomes unconscious & requires mechanical ventilation, whilst being ventilated develops a bizarre
tachyarrhythmia, what do you do?
Answer: Don't treat with antiarrhythmics, instead treat the acidosis & correct hypoxia, use NaHCo3 8.4% (adult 50-100mL IV)
(e) Is there a role for activated charcoal in an amitriptyline overdose?
Answer: Yes it binds it but must ensure that the Pt can protect their own airway & that it's given within an hr or so of presentat.
‫ـــــــــــــــــــــــــــــــــــــــــــــــــــــــــــــــــــــــــــــــــــــــــــــــــــــــــــــــــــــــــــــــــــــــــــــــــــــــــــــــــــــــــــــــــــــــــــــــــــــــــــــــــــــــــــــــــــــــــــــــــــــــــــــــــــــــــــــ‬
36- Question 43 of 50
A 60 yo man presented with SOB. He complained that he was waking at night with SOB & couldn't lie flat.His
previous history was of MI.O/E there was pulmonary rales & mild lower extremity edema

(a) What is the diagnosis?


Answer: Symptomatic left ventricular systolic dysfunction
(b) Name 2 other possible findings O/E in this condition?
Answer: S3 gallop, a decrease in tissue perfusion,pulsus alternans & elevated jugular venous Pr
(c) How would you investigate this Pt?(Name six)
Answer: SpO2, FBC, UE, LFTs, Bl. glucose, thyroid function tests, iron studies , plasma BNP , CXR, ECG
(d) How would you manage this Pt in the ED?(Name four)
Answer: O2 +/- assisted ventilat., IV loop diuretic , vasodilator therapy(NTG), morphine sulfate, +ve inotropic agents
(e) What are the four most common cause of this condition?
Answer: The most common causes of systolic dysfunction are IHD, idiopathic DCM, HTN & valvular disease
‫ـــــــــــــــــــــــــــــــــــــــــــــــــــــــــــــــــــــــــــــــــــــــــــــــــــــــــــــــــــــــــــــــــــــــــــــــــــــــــــــــــــــــــــــــــــــــــــــــــــــــــــــــــــــــــــــــــــــــــــــــــــــــــــــــــــــــــــــ‬
37- Question 44 of 50
A 23 yo man presented with a 1 day history of a painful itchy Rt eye. O/E the eye was red & there was a small amount
of white discharge

14
(a) What pathogens commonly cause bacterial conjunctivitis?
Answer: Staphylococcus aureus, Streptococcus pneumoniae, Haemophilus influenzae, & Moraxella catarrhalis
(b) What virus typically causes viral conjunctivitis?
Answer: Viral conjunctivitis is typically caused by adenovirus
(c) What is the cardinal symptom distinguishing allergic conjunctivitis from a viral aetiology?
Answer: Itching is the cardinal symptom of allergy, distinguishing it from a viral etiology, which is more typically
described as grittiness, burning, or irritation. Eye rubbing can worsen symptoms. Pts with allergic conjunctivitis often
have a history of atopy, seasonal allergy, or specific allergy (eg, to cats).
(d) How is the vision affected in conjunctivitis?
Answer: The diagnosis can be made in a Pt with a red eye & discharge only if the vision is normal & there is no
evidence of keratitis, iritis, or angle closure glaucoma.
(e) Name 4 'red flags' which should alert the clinician that there may be a more serious underlying condition
than simple conjunctivitis?
Answer: Reduction of visual acuity, ciliary flush: A pattern of injection in which the redness is most pronounced in a
ring at the limbus (the limbus is the transition zone between the cornea & the sclera), photophobia, severe FB sensate.
that prevents the Pt from keeping the eye open, corneal opacity, a fixed pupil & severe headache with nausea.
‫ـــــــــــــــــــــــــــــــــــــــــــــــــــــــــــــــــــــــــــــــــــــــــــــــــــــــــــــــــــــــــــــــــــــــــــــــــــــــــــــــــــــــــــــــــــــــــــــــــــــــــــــــــــــــــــــــــــــــــــــــــــــــــــــــــــــــــــــ‬
38- Question id: 2128
A 55 yo man is on the observation ward/ CDU after a minor head injury. You are urgently called to see him as the
nurses feel he is acutely withdrawing from alcohol

(a) List 6 features that would support this diagnosis


Answer: Tremor, anxiety, hallucinations, sweaty, agitation, confusion
(b) What are the risk factors for severe withdrawal to occur?
Answer: Fitting, electrolyte disturbances, arrythmias, infection, CVS collapse, hpoglycaemia, alcoholic ketoacisosis
(c) List the medical management of severe alcohol withdrawal
Answer: See figure 1 next page
(d) What are the diagnostic features of Wernicke-Korsakoff syndrome & how might it be precipitated?
Answer: Ophthalmoplegia, Ataxia, nystagmus, Mental status changes, Apathy, indifference, paucity of speech
Hallucination, agitation Confabulation: Pt fills in gaps of memory with data that can be recalled at that moment.
Debate remains as to whether this action represents a deliberate attempt by the Pt to hide his memory deficits or if it is
an unconscious mechanism.
‫ـــــــــــــــــــــــــــــــــــــــــــــــــــــــــــــــــــــــــــــــــــــــــــــــــــــــــــــــــــــــــــــــــــــــــــــــــــــــــــــــــــــــــــــــــــــــــــــــــــــــــــــــــــــــــــــــــــــــــــــــــــــــــــــــــــــــــــــ‬
39- Question 46 of 50
A 36 yo man was brought to the ED because his mother had found him that morning in his bedroom confused &
drowsy. She had heard him vomiting during the night.She had found an empty packet of anti-histamines by his
bedside. He had a medical history of alcoholism & IV drug abuse. O/E he was febrile at 38 degrees , his HR was 112,
his SpO2 was 99% on RA & his BP was 140/90. His pupils were dilated & his skin was hot to touch. He was
disorientated & answering questions inappropriately with incoherent speech

(a) What is the most likely diagnosis? What other important diagnosis should be considered?
Answer: Most likely diagnosis-Anti-cholinergic toxicity. Other important diagnosis to consider is sepsis or meningitis
given fever,tachycardia & disorientation
(b) What other points may be found O/E of the Pt?(Three points)
Answer: Decreased or absent bowel sounds, "Red as a beet" (cutaneous vasodilation), "Dry as a bone" (anhidrosis),
"Hot as a hare" (anhydrotic hyperthermia), "Blind as a bat" (nonreactive mydriasis), "Mad as a hatter" (delirium;
hallucinations),& "Full as a flask" (urinary retention)
(c) How would you investigate this Pt?(Give four points)
Answer: Bl. glucose, FBC, UE, ECG, Paracetamol level
(d) How would you manage this Pt?(Give five points)
Answer: Stabilization of the ABC. Pts should have IV access, O2, cardiac monitoring, & continuous pulse oximetry.
Consultat. with a medical toxicologist or regional poison center.Agitation & seizures may be treated with benzodiazepines.
Hyperthermia should be treated in typical fashion. Charcoal should be withheld in Pts who are sedated
(e) What is the anidote for this condition?
Answer: Physostigmine.
‫ـــــــــــــــــــــــــــــــــــــــــــــــــــــــــــــــــــــــــــــــــــــــــــــــــــــــــــــــــــــــــــــــــــــــــــــــــــــــــــــــــــــــــــــــــــــــــــــــــــــــــــــــــــــــــــــــــــــــــــــــــــــــــــــــــــــــــــــ‬
40- Question 47 of 50
A-35 yo man who has recently come to the UK from Liberia (West Africa) Presents with a 2 day history of worsening
symptoms of joint pains, fever, chills, rigors & waking up drenched in sweat. O/E he is febrile at 40 C & he is
15
complaining of retrosternal pain

(a) What initial investigations should you carry out?


Answer: Bl. cultures (all febrile Pts)FBC,U&E Bl. films, malaria antigen dip stick testing LFTs Clotting Urinalysis CXR
(b) The malaria screen is -ve what do you do?
Answer: Contact an expert centre in infections/tropical diseases & arrange transfer. Consider the possibility of viral
haemorrhagic fevers i.e. Lassa fever which is endemic to Liberia. 1 malaria screen that is -ve doesn’t necessarily mean
that malaria is not present
(c) Name at least 2 subtypes of malaria & describe which one is potentially fatal?
Answer: P. Falciparum (malignant) potentially fatal P. Ovale P. Vivax P. malariae
(d) Describe features that would make this a severe case of falciparum malaria?
Answer: Renal failure, Acidosis, Coagulopathy, Hypoglycaemia, Coma, WBC >12, Hb <7, Pulmonary oedema,
Retinal haemorrhages, > than 2% Schizonts on Bl. Film
(e) What is the first line TTT of falciparum malaria?
Answer: Parenteral. If the Pt is seriously ill or unable to take tablets, quinine should be given by IVI. The adult
dosage regimen for quinine by infusion is:loading dose(2) of 20 mg/kg(3) (up to maximum 1.4 g) of quinine salt(1)
infused over 4 hrs then 8 hrs after the start of the loading dose, maintenance dose of 10 mg/kg(4) (up to maximum 700
mg) of quinine salt(1) infused over 4 hrs every 8 hrs (until Pt can swallow tablets to complete the 7-day course
together with or followed by either doxycycline or clindamycin as above). Specialist advice should be sought in
difficult cases (e.g. very high parasite count, deterioration on optimal doses of quinine, infection acquired in quinine-
resistant areas of south east Asia) because IV artesunate may be available for named-Pt use

Fig. 1 of Q 38
‫ـــــــــــــــــــــــــــــــــــــــــــــــــــــــــــــــــــــــــــــــــــــــــــــــــــــــــــــــــــــــــــــــــــــــــــــــــــــــــــــــــــــــــــــــــــــــــــــــــــــــــــــــــــــــــــــــــــــــــــــــــــــــــــــــــــــــــــــ‬
41- Question 48 of 50
A 39 yo woman presented with a generalised macular erythematous rash. She had been feeling unwell for the previous 2 days but
16
the rash had relatively quickly on the day of presentat.The wk before presentat. she had been started on a new medicat. for a
chronic medical condit. She also complained of difficulty eating 2ry to oral pain. O/E she was febrile & had a generalised
erythematous macular rash. She had multiple oral ulcerated areas. A diagnosis of stevens-johnsons syndrome was made

(a) What is the differential diagnosis of this condition?(Name three)


Answer: Erythematous drug eruptions, pustular drug eruptions, phototoxic eruptions, staphylococcal scalded skin
syndrome (SSSS) & toxic shock syndrome (TSS)
(b) Name 2 medications which commonly cause this condition
Answer: Anti-gout agents (especially allopurinol), antibiotics (sulfonamides > penicillins > cephalosporins), antipsychotics
& anti-epileptics (including carbamazepine, dilantin, lamotrigine & phenobarbital) & analgesics & NSAID
(c) How would you investigate this Pt?(Name three appropriate investigations)
Answer: FBC with differential cell count, LFTs (Mild elevations in serum aminotransferase levels (2 to 3 times
normal) are present in about 1/2 of Pts), skin biopsy,CXR (condition may be caused by infect. as well as medication).
(d) How would you manage this Pt? (Five management steps)
Answer: Prompt removal of offending agent. Supportive care includes wound care, fluid & electrolyte management,
nutritional support, ocular care,oral care, temp. management & monitoring for & TTT of superinfections. Consider
transfer to a burn unit as massive loss of the epidermis may occur
(e) What is the prognosis of this condition?
Answer: Mortality of 1 to 3%.
‫ـــــــــــــــــــــــــــــــــــــــــــــــــــــــــــــــــــــــــــــــــــــــــــــــــــــــــــــــــــــــــــــــــــــــــــــــــــــــــــــــــــــــــــــــــــــــــــــــــــــــــــــــــــــــــــــــــــــــــــــــــــــــــــــــــــــــــــــ‬
42- Question 49 of 50
A 18 yo girl was brought to the ED after overdosing on her mothers iron tablets

(a) What are the clinical features of iron toxicity?


Answer: Nausea, vomiting, diarrhoea, abd. pain, black stools, hyperglycaemia, shock, hypoglycaemia, jaundice,
metabolic acidosis, hepatic encephalopathy & renal failure & coma.
(b) How would you investigate this Pt?(Give four)
Answer: Check serum iron, FBC, UE, glucose, ABG
(c) How would you manage this Pt?
Answer: Charcoal doesn't absorb iron, gastric lavage if within 1 hr, expert advice, supportive measures, desferrioxamine
(d) What are the complications of desferrioxamine?(Give four)
Answer: Iron-desferrioxamine complex makes the urine orange or red, desferrioxamine causes hypotension if infused
too rapidly, rashes, anaphylaxis, pulmonary oedema, ARDS
(e) What gastrointestinal sequelae may survivors exhibit? (Give one)
Answer: Gastric strictures & pyloric obstruction
‫ـــــــــــــــــــــــــــــــــــــــــــــــــــــــــــــــــــــــــــــــــــــــــــــــــــــــــــــــــــــــــــــــــــــــــــــــــــــــــــــــــــــــــــــــــــــــــــــــــــــــــــــــــــــــــــــــــــــــــــــــــــــــــــــــــــــــــــــ‬
43- Question 50 of 50
A 46 yo man was came into his house after being in the garden playing with his dog. Over the course of an hr he
developed facial swelling & SOB resulting in his wife calling the ambulance

(a) Name three common causes of anaphylaxis?


Answer: Drugs, vaccines, bee/wasp stings, nuts, shellfish, strawberries, wheat, latex
(b) Pts taking what medication may have particularly severe symptoms?
Answer: B-Blockers.
(c) What are the basic initial management steps with this man?
Answer: Cardiac monitoring, SpO2 monitoring, IV access, supplemental O2, monitor BP.
(d) What dose of adrenaline should be given if there is shock, airway swelling or respiratory difficulty?
Answer: 0.5mg(0.5ml of 1:1000) IM. Repeat in five minutes if there is no improvement
(e) Caution should be observed in administering adrenaline if the Pt is taking what medications?(Name 2)
Answer: TCAs, MAOIs, B-Blockers.
‫ـــــــــــــــــــــــــــــــــــــــــــــــــــــــــــــــــــــــــــــــــــــــــــــــــــــــــــــــــــــــــــــــــــــــــــــــــــــــــــــــــــــــــــــــــــــــــــــــــــــــــــــــــــــــــــــــــــــــــــــــــــــــــــــــــــــــــــــ‬
44- Question 3 of 20
A 73 yo man presents to the ED acutely SOB. He is found to be in fast AF with a PR of 153. He suffered 3 M.I.s last yr & has a
10 yr history of HTN. His BP was 100/53 & auscultat. of his chest revealed insp. crackles & exp. wheeze with a  JVP

(a) What should happen to this Pt according to resus council guidelines?


Answer: Attempt synchronised shocks up to 3 attempts. According to tachyarrhythmia guidelines the Pt is unstable &
requires electrical cardioversion
(b) Which of the following drugs would be the best for restoring sinus rhythm? 1. IV digoxin 2. IV amiodarone
3. IV flecanide 4. IVesmolol 5. IVdofetolide
17
Answer: IV amiodarone. - digoxin & esmolol wouldn’t restore sinus ryhtm, flecanide is contraindicated in heart
failure & amiodarone is the least -vely ionotropic Dofetolide = good option but amiodarone = better
(c) What type of Pt should one avoid digoxin in?
Answer: Renal Pts/ Pts with known renal failure.
(d) List the investigations that you would want to help you work out why the Pt was in fast AF.
Answer: CXR, MSU, Bl. cultures, ECHO,
‫ـــــــــــــــــــــــــــــــــــــــــــــــــــــــــــــــــــــــــــــــــــــــــــــــــــــــــــــــــــــــــــــــــــــــــــــــــــــــــــــــــــــــــــــــــــــــــــــــــــــــــــــــــــــــــــــــــــــــــــــــــــــــــــــــــــــــــــــ‬
45- Question id: 2110
The following ECG was recorded on a 67 yo male in the ED. He was sweaty & clammy & felt SOB but said that he
had no chest pain although he described discomfort in his mouth & neck

(a) What changes are shown in yellow & blue, what is the diagnosis?
Answer: Anterior M.I. yellow = ST segment elevation most pronounced in the anterior leads V1-V4. Blue =
reciprocal changes in the inferior leads.
(b) Which coronary vessel is likely to have been occluded?
Answer: Likely left LAD to be exact.
(c) Name 3 conditions that could mimic the picture shown above
Answer: Pericarditis, trauma to the myocardium, WPW, hyperkalaemia, pneumothorax, cardiac amyloid/sarcoid,
cardiac tumours, cardiomyopathy, LBBB, LVH or RVH, pancreatitis
(d) When do troponin levels rise post M.I.? How long do they remain elevated for?
Answer: They start to rise 3 hrs post M.I. but peak at 24-48 hrs they can remain elevated for 7-14 days
(e) Give 3 contraindications to thrombolysis, (appreciating that most are relative, choose ones where you would
be very hesitant to administer thrombolytic agents)
Answer: Arterial or major surgery within 4 wks Previous hgic stroke Prolonged CPR Pregn. Possible aortic dissect. Severe HTN

‫ـــــــــــــــــــــــــــــــــــــــــــــــــــــــــــــــــــــــــــــــــــــــــــــــــــــــــــــــــــــــــــــــــــــــــــــــــــــــــــــــــــــــــــــــــــــــــــــــــــــــــــــــــــــــــــــــــــــــــــــــــــــــــــــــــــــــــــــ‬
46- Question 7 of 20
A 69 yo man presented with a painful swollen Rt knee which had come on insidiously over the course of the previous
48 hrs. He had no history of joint disease. There was no history of trauma. His backgound was of type II DM & was
on warfarin for a prosthetic heart valve.O/E there was an effusion with restricted range of movement

(a) Give a differential diagnosis of four conditions?


Answer: Infective arthritis, hemorrhagic effusions, noninflammatory effusions (osteoarthritis) & inflammatory
effusions (RA, gout, pseudogout)
(b) Name three characteristics of normal synovial fluid?
Answer: Highly viscous, clear, essentially acellular, protein concentration approximately one-third that of plasma &
glucose concentration similar to that in plasma
(c) Approximately what is the cell count in bacterial joint infections?
Answer: Bacterial joint infections typically are purulent with leukocyte counts (most of which are neutrophils) of
50,000 to 150,000 cells/mm3
(d) What is seen in the synovial fluid in acute crystal-induced synovitis?
Answer: MonoNa urate (MSU) crystals(Gout) & Ca pyrophosphate dihydrate (CPPD) crystals(pseudogout)
(e) Name 2 conditions in which an eosinophilia may be prominent in the synovial fluid?
Answer: Eosinophilia in the synovial fluid suggests parasitic infection, allergy, neoplasm, or Lyme disease.
‫ـــــــــــــــــــــــــــــــــــــــــــــــــــــــــــــــــــــــــــــــــــــــــــــــــــــــــــــــــــــــــــــــــــــــــــــــــــــــــــــــــــــــــــــــــــــــــــــــــــــــــــــــــــــــــــــــــــــــــــــــــــــــــــــــــــــــــــــ‬
47- Question 10 of 20
A 21 yo university student comes in having ingested is Aunties complete supply of digoxin tablets. He is fully
conscious but has a rate of 38 b.p.m
18
(a) What is your initial management?
Answer: ABCDE, ECG- IV access & atropine- in increments start with 0.5mg
(b) Is digoxin +vely or -vely ionotropic?
Answer: +vely
(c) Is there a role for charcoal in this Pt?
Answer: Yes- give it down an NG tube
(d) Assuming your initial TTT works where should he be admitted?
Answer: CCU
(e) After 20 minutes he starts to feel dizzy & his HR drops to 36 b.p.m what is the next step in management?
Answer: Digoxin specific fab fragment (Digibind)
(f) The Pt gets worse & the monitor shows VT, he has a pulse but his BP is 85/49. What will you do now?
Answer: He is haemodynamically unstable & has a VT due to digoxon toxicity. DC cardioversion is relatively
contraindicated here unless all other measures have been exhausted. The most useful drugs in this setting are
lignociane & phenytoin. Amiodarone would increase digoxin levels & is contraindicated
‫ـــــــــــــــــــــــــــــــــــــــــــــــــــــــــــــــــــــــــــــــــــــــــــــــــــــــــــــــــــــــــــــــــــــــــــــــــــــــــــــــــــــــــــــــــــــــــــــــــــــــــــــــــــــــــــــــــــــــــــــــــــــــــــــــــــــــــــــ‬
48- Question id: 2092
A 76 yo gentleman with prostate cancer comes in with severe back & lt leg pain. He also cannot pass urine. & is off
legs & confused

(a) What systems must you examine carefully?


Answer: He needs everything examining but careful attention should be paid to lower limb neurological exam..
(cauda equina) PR is absolutely crucial (cadua equina & is the rectum full of hard stool?) Mini mental state exam.
(level of confusion & cause?)

(b) What 2 investigations are critical?


Answer: Serum Ca, Radiographs of the left leg, Bladder scan
(c) How do you manage his pain? At home he takes MST 60mg BD & oramorph 10mg PRN?
Answer: In the acute setting IV morphine but consider the dose carefully, will need to be titrated. He may well be
fairly opiate tolerant.
(d) Some lab results become available Hb- 9.5 WBC 14.8 Plt 93 Na 146 K+ 3.6 Ur 12 Cr 345 Ca2+ 3.9 What
concerns you & how would you manage these results?
Answer: WBC increasing Ca2 increasing & ARF! All concerning. Initial therapy is going to be around rehydration &
careful monitoring of ARF. Best managed in medical HDU. Decisions about filtration will need to be had.
(e) The radiograph of the left femur is shown (fig 1). Describe what it shows
Answer: A pathological fracture, mid-shaft with angulation & minimal displacement.
‫ـــــــــــــــــــــــــــــــــــــــــــــــــــــــــــــــــــــــــــــــــــــــــــــــــــــــــــــــــــــــــــــــــــــــــــــــــــــــــــــــــــــــــــــــــــــــــــــــــــــــــــــــــــــــــــــــــــــــــــــــــــــــــــــــــــــــــــــ‬
49- Question 12 of 20
A 58 yo man presents to the ED with retrosternal chest pain for the last 2 hrs. He appears unwell & is sweaty. He has
smoked for the last 20 yrs but has no significant medical history of note. HR is 110 & Bp is 82/45. His JVP is raised & he
has an audible S3, he also has creps to the midzones of the lungs. ECG reveals St elevat. in V1-V4 & depression in II,III & aVF

(a) What 2 oral medications should he be given immediately?


Answer: Asprin & Clopidogrel 300mg each.
(b) What has occurred?
Answer: Large anterior M.I. complicated by cardiogenic shock
(c) What investigations would be very helpful in this case?
Answer: CXR- To look for widening of the mediastinum to help rule out dissection (although NB that it doesn’t rule
it out) Also bedside ECHO to look for pericardial tamponade.
19
(d) How should this Pt be managed?
Answer: IV vasopressors to treat the shock & intra-aortic balloon pump also reperfusion of the coronary arteries via
PCI. The SHOCK trial showed that emergently revascularisat. was better than thrombolysis with regard to improving mortality
‫ـــــــــــــــــــــــــــــــــــــــــــــــــــــــــــــــــــــــــــــــــــــــــــــــــــــــــــــــــــــــــــــــــــــــــــــــــــــــــــــــــــــــــــــــــــــــــــــــــــــــــــــــــــــــــــــــــــــــــــــــــــــــــــــــــــــــــــــ‬
50- Question id: 4519
A 68 yo man presents to the department with a feeling of being generally unwell & weak. His wife tells you that he
collapsed earlier this morning but he is denying any such thing. He has lost 6kg in weight over the last 3 months. Abd.
exam. reveals some left sided loin tenderness. Urinalysis reveals Bl. 4+, protein 2+.

(a) You do a chest x-ray as part of your collapse? Cause work up & find the following findings (see figure 1).
What does the radiograph show? List a few differential diagnoses for this picture.
Answer: Cannonball metastasis- could be from renal, testicular, colon, osteosarcoma.
(b) The Pt's haemoglobin is 19g/dL what could be the cause of this?
Answer: With a diagnosis of renal cell carcinoma from the history given the relative polycythemia could be due to an 
amount of circulating rennin. Renal tumours often secrete rennin & ertythropoetin along with other peptide hormones
(c) What lab tests are especially important in this case?
Answer: Ca level- could be raised & need treating. U&E- again could show decreased renal function. LFTs; any
evidence of liver involvement
(d) Given the likely diagnosis what is the management & prognosis for this Pt?
Answer: Very poor. If it has spread metastatically to other organs, the 5-yr survival rate is<5 %. Management would still be
surgical to remove the tumour from the lt kidney if the Pt was fit enough for surgery as removal of the 1ry tumour has been
shown to improve survival & cause regression of the metastasis. Also likely to go on to have palliative chemo & radiotherapy

51- Question 14 of 20
A 46 yo factory worker comes in with chest pain that started yesterday after some heavy lifting. His ECG shows T
wave inversion in the lateral leads & his 12 hr troponin came back at 0.08. He is pain free when you see him

(a) What does this represent?


Answer: troponin <0.1 is not an NSTEMI but > 0.03 may well represent unstable angina
(b) What does this Pt need?
Answer: Needs to be admitted. He needs an aniogram. In Pt exercise stress testing should be avoided in view of the
abnormal troponin
(c) List 3 causes of a raised troponin other than myocardial ischaemia?
Answer: Renal failure, infection- sepsis, pulmonary embolism, myocarditis/Pericarditis, prolonged tachycardia.
(d) The Pt was sent home on asprin, clopidogrel & sivastatin. What other medicat. should the Pt receive to improve his
prognosis?
Answer: ACEi/angiotensin receptor blocker. The HOPE study looked at the role of ACEi in high risk populations
without any evidence of left ventricular dysfunction
(e) How long does he effect of clopidogrel last for after it is discontinued?
Answer: The life of the platelet as it irreversibly changes the platelets ability to aggregate- 10-14 days. It stops ADP
from binding to its receptor on the platelet surface
‫ـــــــــــــــــــــــــــــــــــــــــــــــــــــــــــــــــــــــــــــــــــــــــــــــــــــــــــــــــــــــــــــــــــــــــــــــــــــــــــــــــــــــــــــــــــــــــــــــــــــــــــــــــــــــــــــــــــــــــــــــــــــــــــــــــــــــــــــ‬

20
52- Question 15 of 20
A 26 yo Pt was admitted via ED with acute asthma for the 5th time in the last 2 yrs. The Pt had recently had
aminophyline added to her inhaled therapy, which consisted of seretide 500 & salbutamol. On admission she was
unwell with sats of 88% in air & a RR of 44. CXR revealed consolidate. at the Rt base. She was started on erythromycin as
she had a penicillin allergy. She improved but 2 days later suffered 3 grand mal seizures & needed to be ventilated on ITU

(a) What step is this Pt on with regard to her asthma management according to BTS guidelines?
Answer: 4
(b) What is Seretide a combination of?
Answer: Salmeterol (LABA) & Fluticasone (steroid)
(c) What do you think might be the major problem with this Pt's asthma?
Answer: Poor compliance
(d) The Pt had no previous history of fitting from the following options which do you think was the cause of the
seizures & why? 1. Hypoxia 2. Meningitis 3. Benign IC HTN 4. React. to erythromycin 5. Theophylline toxicity 6.
Herpes encephalitis
Answer: Theophylline toxicity: the erythromycin inhibits the metabolism of theophylline therefore potentiating its effects
(e) What is the cross over for penicillin allergic Pts when considering giving cephalosporins?
Answer: Quoted as 10%
‫ـــــــــــــــــــــــــــــــــــــــــــــــــــــــــــــــــــــــــــــــــــــــــــــــــــــــــــــــــــــــــــــــــــــــــــــــــــــــــــــــــــــــــــــــــــــــــــــــــــــــــــــــــــــــــــــــــــــــــــــــــــــــــــــــــــــــــــــ‬
53- Question 16 of 20
A 38 yo samoan lady presented to the ED with an ulcer on the lateral aspect of her Rt small toe & a surrounding
cellulitis. She had a one yr history of NIDDM & was prescribed metformin but had not been taking this medication.

(a) How would you investigate this Pt?(Four points)


Answer: FBC, UE, Bl. cultures, swab from lesion, X Ray foot
(b) How would you manage this Pt?(Four points)
Answer: IV antibiotics, analgesia, subcutaneous insulin sliding scale, IV fluids, endocrinology consultation
(c) Name three categories of bacteria which may be causing infection in this Pt?
Answer: Aerobic gram +ve organisms , gram -ve organisms & anaerobic organisms.
(d) Name four factors which make Pts with diabetes at high risk for foot infections?
Answer: Sensory neuropathy(which causes a decreased appreciation of temp. & pain) motor neuropathy(which can
cause foot deformities), autonomic neuropathy(which can cause decreased sweat & sebaceous gland secretion
resulting in dry, cracked skin), peripheral arterial disease(which can reduce the Bl. supply needed for healing of ulcers
& infections) & hyperglycemia(which impairs neutrophil function & defects in host defenses)
(e) Name a complication of this condition?
Answer: Osteomyelitis & systemic infection
‫ـــــــــــــــــــــــــــــــــــــــــــــــــــــــــــــــــــــــــــــــــــــــــــــــــــــــــــــــــــــــــــــــــــــــــــــــــــــــــــــــــــــــــــــــــــــــــــــــــــــــــــــــــــــــــــــــــــــــــــــــــــــــــــــــــــــــــــــ‬
54- Question 17 of 20
A 45 yo man has now been in the ED for 2 hrs he is profoundly unwell. He presented in what appeared to be septic
shock & has been treated as such. He was febrile 39.2, tachycardic with a BP of 67/40. There is no history available
about what happened

(a) As yet no source for sepsis has been found. You do a tox screen which is also -ve. List the DD apart from sepsis
Answer: MDMA(ecstasy), Thyrotoxic storm, Malignant hyperthermia, Heat stroke (malignant hyperpyrexia), EBV,
Serotonin syndrome.
(b) TSH is 8 & T4 is 20. What will you do now?
Answer: Treat as a thyrotoxic crisis
(c) List 4 precipitants of a crisis such as this?
Answer: Thyroid surgery, Withdrawal of antithyroid drugs, Iodinated contrast dyes, Thyroid palpation, Sepsis, P.E.,
DKA, Trauma or emotional stress.
(d) What would giving salicylates do?
Answer: Make it worse by displacing the T4 from thyroid binding globulin (TBG)
(e) management steps?
Answer: CVP & accurate fluid resuscitation B-blockers if no contraindicate. Active cooling techniques Treat any
infect. High dose antithyroid drugs Propylthiouricil is better than carbimazole Hydrocortisone inhibits the conversion
of T4-T3 Monior glucose levels
‫ـــــــــــــــــــــــــــــــــــــــــــــــــــــــــــــــــــــــــــــــــــــــــــــــــــــــــــــــــــــــــــــــــــــــــــــــــــــــــــــــــــــــــــــــــــــــــــــــــــــــــــــــــــــــــــــــــــــــــــــــــــــــــــــــــــــــــــــ‬
55- Question 18 of 20
The nurse in charge takes a phone call from the ambulance staff who are en route to the ED with a 60 yo woman who
has arrested. She had called the ambulance as she had chest pain but had arrested soon after the ambulance staff got to
her home.They arrive in the ED. The Pt had received CPR & has IV access but hasn't received any medicat. thus far.
21
(a) What is the first step in the management if this Pt is in VF?
Answer: Give one shock. Resume CPR immediately after the shock.
(b) After this step how many cycles of CPR should be given before the next rhythm check?
Answer: Give five cycles of CPR & then check rhythm.
(c) After the cycles of CPR there is another rhythm check & the Pt is still in VF. What are the 3 next
management steps?
Answer: Give 1 shock. Resume CPR. Give adrenaline 1mg IV/IO,repeat every 3-5 minutes.
(d) What anti-arrhythmic medication should be given during CPR?
Answer: Amiodarone 300mg IV/IO once, then consider additional 150mg IV/IO.
(e) If the Pt is in torsades de pointes what medication should be given & at what dose?
Answer: Magnesium, loading dose 1 to 2g IV/IO.
‫ـــــــــــــــــــــــــــــــــــــــــــــــــــــــــــــــــــــــــــــــــــــــــــــــــــــــــــــــــــــــــــــــــــــــــــــــــــــــــــــــــــــــــــــــــــــــــــــــــــــــــــــــــــــــــــــــــــــــــــــــــــــــــــــــــــــــــــــ‬
56- Question 2080
A 35 yo man who has a known personality disorder says that he has taken 45 300mg asprin tablets. He is sweating
profusely & is agitated, he has been vomiting & says that his ears will not stop ringing

(a) Explain the results of the ABG: pH 7.36 paO2 12.3 paCO2 2.8 BE -16 HCO3- 17
Answer: Shows a mixed picture, shows a mixed metabolic acidosis with respiratory alkalosis which is typical in
salicylate poisoning, the danger is that the acidosis will worsen

(b) What does could be fatal?


Answer: >500mg/kg could cause fatal poisoning.
(c) Is there a role for activated charcoal with asprin od?
Answer: Yes, if 2 levels are taken & the 2nd 1 ↑ you could consider giving another dose of 50g of activated charcoal.
(d) A CXR is taken (fig 1): What is shown & how would you manage this?
Answer: ARDS needs ITU for ventilation & supportive care, likely to need renal support.
(e) What is the definitive management in severe cases?
Answer: ITU for haemodialysis, Paralysis & ventilation often helpful IV glucose as brain levels can get very low
‫ـــــــــــــــــــــــــــــــــــــــــــــــــــــــــــــــــــــــــــــــــــــــــــــــــــــــــــــــــــــــــــــــــــــــــــــــــــــــــــــــــــــــــــــــــــــــــــــــــــــــــــــــــــــــــــــــــــــــــــــــــــــــــــــــــــــــــــــ‬
57- Question 3 of 50
A 44 yo male presents with retrosternal chest pain & a mild fever. He says that the pain is sharp & worse on
inspiration. It gets better when he leans forward. His ECG is shown in fig 1

(a) What is the diagnosis?


Answer: Pericarditis
(b) Can you name 4 causes?
Answer: Viral (coxsackie B, HIV), Post MI (Dressler’s syndrome). Bacterial pneumonia or septicaemia, TB,
Cancer, Rheumatic fever, Uraemia, Collagen vascular disease ( SLE, PAN, Rheumatoid arthritis), After cardiac
surgery, Drugs ( hydralazine, procainamide, methyldopa, minoxidil)
(c) He subsequently tells you that he has had unprotected homosexual intercourse & is concerned about HIV.
What other important test does this Pt need?
Answer: TB, as this is a cause of pericarditis & he may well have TB due to being HIV +ve.
22
(d) Should this Pt have troponins measured?
Answer: There is no evidence that it guides management or provides prognostic information.
(e) The Pt responds well to simple TTT & you decide that he can be discharged however he returns to see his
GP a week later with symptoms of tiredness & further chest pain. The GP runs some routine tests including an
ESR which is found to be elevated. What further tests would you like to do & what diagnosis needs to be ruled out?
Answer: diagnosis to rule out is myocarditis, test: ESR is ↑ in 60% of cases, will need cardiology work up & possibly
an endomyocardial biopsy which continues to be of use in diagnosing myocarditis. Also an echo will be useful.
‫ـــــــــــــــــــــــــــــــــــــــــــــــــــــــــــــــــــــــــــــــــــــــــــــــــــــــــــــــــــــــــــــــــــــــــــــــــــــــــــــــــــــــــــــــــــــــــــــــــــــــــــــــــــــــــــــــــــــــــــــــــــــــــــــــــــــــــــــ‬
58- Question 5 of 50
A 74 yo gentleman is called through to the department by the ambulance crew. He was found collapsed at home &
was unresponsive when they arrived. He had an aneurysm repair 2 yrs ago.

(a) In the 1ry survey you establish that he is shocked & has a GCS of 14 as he is confused but he is breathing
spontaneously & is maintaining his own airway. You can see a laparotomy scar. You gain IV access & attach
fluids aiming to maintain a BP of around�.. what?
Answer: The Pt is shocked, you have heard that he had an aneurysm repair 2 yrs ago. You want to aim for a MAP of
around 70 or a systolic of around 90 or less.
(b) How do you calculate MAP?
Answer: Diastolic Pr. + 1/3 of pulse Pr. (systolic-diastolic)= MAP approx.
(c) In what other circumstances should Pts be managed in this way & what’s the underlying principle known as?
Answer: Permissive hypotension; trauma is the other situation. If someone is shocked in trauma the principle should
be to maintain a similar MAP whilst aiming to prevent the dilution of clotting factors.
(d) What fairly new agents are you aware of that can help to stem bleeding in the shocked trauma Pt?
Answer: Activated factor VIIa
(e) He then proceeds to have a large PR bleed; the Bl. appears to be fresh. Name 2 DD for what is happening?
Answer: 1 large fresh rectal/lower GI bleed could be from numerous causes including diverticular disease,
angiodysplasia etc 2 aortoenteric fistula, rare but fits with the history.
(f) You alert the surgeons & arrange imaging. You manage to stabilise the Pt with resuscitation. What is the
best way to ensure that this Pt is adequately monitored at this time?
Answer: an arterial line will be very helpful to detect the beat to beat variation in Bl. Pr., easy & quick to insert.
(g) What is octaplas & in what situations is it used?
Answer: It is Fresh Frozen Plasma used to reverse the effects of warfarin very quickly
‫ـــــــــــــــــــــــــــــــــــــــــــــــــــــــــــــــــــــــــــــــــــــــــــــــــــــــــــــــــــــــــــــــــــــــــــــــــــــــــــــــــــــــــــــــــــــــــــــــــــــــــــــــــــــــــــــــــــــــــــــــــــــــــــــــــــــــــــــ‬
59- Question 2165
An 80 yo man presented feeling generally unwell. O/E his HR was 30 bpm

(a) Name three basic initial steps in the management of this man?
Answer: O2, ABCs, monitor ECG, monitor BP, SpO2, establish IV access.
(b) What are the signs & symptoms suggesting poor perfusion caused by bradcardia?(Name three)
Answer: Acute altered mental status, ongoing chest pain, hypotension or other signs of shock.
(c) If the Pt has poor perfusion what medication should be considered?What is the dose?
Answer: Atropine 0.5mg IV. May repeat to a total of 3mg.
(d) If this is ineffective & no specialist consultation is available what is the next step?
Answer: Transcutaneous pacing.
(e) If the therapeutic modality of step 4 is ineffective what medicat. may be added to try & ↑ its effectiveness?
Answer: Adrenaline(2-10ug/min) or dopamine(2-10 ug/kg per minute) infusion.
‫ـــــــــــــــــــــــــــــــــــــــــــــــــــــــــــــــــــــــــــــــــــــــــــــــــــــــــــــــــــــــــــــــــــــــــــــــــــــــــــــــــــــــــــــــــــــــــــــــــــــــــــــــــــــــــــــــــــــــــــــــــــــــــــــــــــــــــــــ‬

Fig 1
23
60- Question 20 of 50
A 73 yo woman presented to the ED complaining of severe pain. Her vital signs were within the normal range. Exam.
revealed the rash shown in the picture.

(a) What is the causative organism?


Answer: Herpes zoster(shingles) results from reactivate. of endogenous latent VZV infect. within the sensory ganglia.
(b) How does the rash begin?
Answer: The rash of herpes zoster starts as erythematous papules, which quickly evolve into grouped vesicles or bullae
(c) How long is the Pt infective?
Answer: In immunocompetent hosts, the lesions crust by 7 to 10 days & are no longer infectious
(d) Which 2 dermatomal regions are most commonly involved?
Answer: The thoracic & lumbar dermatomes are the most commonly involved sites of herpes zoster. Zoster is
generally limited to one dermatome in previously healthy hosts.
(e) What percentage of Pts develop systemic symptoms & name three systemic symptoms?
Answer: Fewer than 20% of Pts have significant systemic symptoms, such as headache, fever, malaise, or fatigue.
‫ـــــــــــــــــــــــــــــــــــــــــــــــــــــــــــــــــــــــــــــــــــــــــــــــــــــــــــــــــــــــــــــــــــــــــــــــــــــــــــــــــــــــــــــــــــــــــــــــــــــــــــــــــــــــــــــــــــــــــــــــــــــــــــــــــــــــــــــ‬
61- Question 25 of 50
Eligibility criteria for the TTT of acute ischemic stroke with recombinant tissue plasminogen activator (rt-PA) include:

(a) Within how long a period of time can TTT be given after a clearly defined symptom onset?
Answer: Thrombolytic TTT should be initiated within 3 hrs of a clearly defined symptom onset.
(b) Name three features of the history which may exclude the use of thrombolysis
Answer: Features of the history which exclude the use of thrombolysis include stroke or head trauma within the prior
3 months, any prior history of intracranial hemorrhage, major surgery within 14 days, gastrointestinal or
gentitourinary bleeding within the previous 21 days, myocardial infarction in the prior 3 months, arterial puncture at a
noncompressible site within 7 days & lumbar puncture within 7 days.
(c) Name three clinical features which exclude the use of thrombolysis.
Answer: Clinical features which may exclude the use of thrombolysis include rapidly improving stroke symptoms,
only minor & isolated neurologic signs, seizure at the onset of stroke is an exclusion if the residual impairments are
due to postictal phenomenon (Seizure isn't an exclusion if the clinician is convinced that residual impairments are due
to stroke & not to postictal phenomenon), symptoms suggestive of subarachnoid hemorrhage, even if the CT is
normal, clinical presentat. consistent with acute MI or post-MI pericarditis, persistent systolic BP>185, diastolic BP>110
mmHg or requiring aggressive therapy to control BP, preg. or lactation, active bleeding or acute trauma (fracture).
(d) Name three laboratory features which exclude the use of thrombolysis.
Answer: Lab. features which exclude the use of thrombolysis include platelets <100,000/mm3, serum glucose <50 mg/dL
(2.8 mmol/L) or >400 mg/dL (22.2 mmol/L), INR >1.7 if on warfarin & an elevated PTT if on heparin
(e) Name 2 head CT scan features which exclude the use of thrombolysis.
Answer: Head CT scan which exclude the use of thrombolysis include evidence of hemorrhage & evidence major
early infarct signs, such as diffuse swelling of the affected hemisphere, parenchymal hypodensity, &/or effacement of
>33 % of the middle cerebral artery territory
‫ـــــــــــــــــــــــــــــــــــــــــــــــــــــــــــــــــــــــــــــــــــــــــــــــــــــــــــــــــــــــــــــــــــــــــــــــــــــــــــــــــــــــــــــــــــــــــــــــــــــــــــــــــــــــــــــــــــــــــــــــــــــــــــــــــــــــــــــ‬
62- Question id: 2146
A 65 yo woman presents with chest pain. Her ECG is shown in figure 1.

Fig1
(a) What are the 4 criteria according to the UK Resuscitat. Council 2005 guidelines that constitute an unstable
tachyarrhythmia?
Answer: 1. Presence of chest pain 2. Systolic BP <90 3. Evidence of heart failure 4. Decrease in conscious level
24
(b) What is the TTT of choice? What is it crucial to appreciate from an anaesthetic viewpoint?
Answer: DC cardioversion is the TTT of choice. Must be done in synchronised mode so that the shock is delivered on
the R wave to avoid precipitating VF. Need to appreciate that the circulation time & cardiac output are obviously
markedly reduced therefore a gentle anaesthetic is required, also high risk of aspiration as not starved
(c) How many joules would you select for the above rhythm?
Answer: 200 monophasic, 120-150 biphasic for starters
(d) How many shocks would you deliver if your first were not successful?
Answer: 3
(e) What would you do after the No. of shocks you stated in part d?
Answer: Give 300mg of amiodarone over 10-20 mins & rpt the shock.
‫ـــــــــــــــــــــــــــــــــــــــــــــــــــــــــــــــــــــــــــــــــــــــــــــــــــــــــــــــــــــــــــــــــــــــــــــــــــــــــــــــــــــــــــــــــــــــــــــــــــــــــــــــــــــــــــــــــــــــــــــــــــــــــــــــــــــــــــــ‬
63- Question 27 of 50
A 29 yo woman who was 38 weeks pregnant called an ambulance because she felt palpitations. The ambulance staff
called in that the Pt had a narrow complex tachycardia.

(a) What are the symptoms & signs that suggest that this Pt may be unstable?(Give three)
Answer: Altered mental status, ongoing chest pain,& hypotension
(b) If it is decided that the Pt is stable give four basic steps prior to TTT?
Answer: O2, monitor, IV access, 12 lead ECG.
(c) If the rhythm is regular&QRS complex is narrow how would you procede prior to administering any medication?
Answer: Vagal maneuvers.
(d) If this fails, with what medication would you treat the Pt?What is the dose of the medication?
Answer: Adenosine 6mg IV push. If no conversion give 12mg rapid IV push;may repeat 12mg dose once.
(e) If the rhythm fails to convert after this medication what other diagnoses should be considered?(Give 2)
Answer: Atrial flutter, ectopic atrial tachycardia or junctional tachycardia. The rate should be controlled with a Ca
channel blocker or a beta blocker, treat the underlying cause & consider expert consultation
‫ـــــــــــــــــــــــــــــــــــــــــــــــــــــــــــــــــــــــــــــــــــــــــــــــــــــــــــــــــــــــــــــــــــــــــــــــــــــــــــــــــــــــــــــــــــــــــــــــــــــــــــــــــــــــــــــــــــــــــــــــــــــــــــــــــــــــــــــ‬
64- Question 28 of 50
A 20 yo male presented to the ED with intermittent headaches & malaise since a head injury at work the previous
week. The head injury had caused a LOC & he had been brought to another hospital immediately afterwards & a CT
brain scan had not revealed any intracranial pathology. Three days later he had re-attended the same hospital as he
still had headaches & malaise. He had undergone a 2nd CT brain scan, which again was unremarkable

(a) What are the common post concussion symptoms? (Give four)
Answer: Headache, lethargy, low mood, poor concentrating ability, dizziness
(b) What are the characteristics of post concussion headaches? (Give 2)
Answer: May last for several months, intermittent, become worse during the day, become worse on exercise
(c) What factors may contribute to dizziness caused after a concussion? (Give one)
Answer: Codeine based analgesia, Pts are more sensitive to the effects of alcohol
(d) Name 2 categories of Pts who are prone to developing a chronic subdural haematoma?
Answer: Elderly, Pts with bleeding disorders, alcoholics.
(e) How would you manage this Pt? (Give four)
Answer: History should cover symptoms of other types of headache e.g. photophobia, meningismus, full neurological
exam, investigations to rule out other causes of headache if appropriate, check the reports of the CT Brain radiologist
report from the initial hospital, explanation of symptoms to Pt, arrange follow up with GP.
‫ـــــــــــــــــــــــــــــــــــــــــــــــــــــــــــــــــــــــــــــــــــــــــــــــــــــــــــــــــــــــــــــــــــــــــــــــــــــــــــــــــــــــــــــــــــــــــــــــــــــــــــــــــــــــــــــــــــــــــــــــــــــــــــــــــــــــــــــ‬
65- Question 29 of 50
A 50 yo truck driver presented with dysuria & painful wrists, shoulders, knees & ankles.He also complained of purulent eye
discharge. O/E he was febrile (38.5)&had a small joint effusion in his Rt knee. His dipstick urine revealed nitrites,leukocytes&Bl.

(a) What is the diagnosis?


Answer: Reiters syndrome.
(b) Name five investigations which should be carried out?
Answer: FBC, UE, MSU, Bl. cultures, knee synovial fluid aspiration, stool culture (as enteric infections can cause a
reactive arthritis), CRP, ESR & plain radiographs to exclude other diagnoses.
(c) Name three pathogens which can cause a reactive arthritis?
Answer: The classical pathogens for reactive arthritis are: Chlamydia trachomatis, Yersinia, Salmonella, Shigella &
Campylobacter & perhaps Clostridium difficile & Chlamydia pneumoniae.
(d) Name three management steps in the ED?

25
Answer: NSAIDS, Rheumatology consultation, Infectious diseases consultation to discuss appropriate additional tests
& medications for symptomatic relief or microbiologic cure & to ensure follow-up TTT
(e) What is the prognosis of this condition?
Answer: Most Pts remit completely or have little active disease six months after presentation. Chronic persistent
arthritis, lasting more than six months, occurs in only a small proportion of Pts.
‫ـــــــــــــــــــــــــــــــــــــــــــــــــــــــــــــــــــــــــــــــــــــــــــــــــــــــــــــــــــــــــــــــــــــــــــــــــــــــــــــــــــــــــــــــــــــــــــــــــــــــــــــــــــــــــــــــــــــــــــــــــــــــــــــــــــــــــــــ‬
66- Question 30 of 50
A 30 yo women presented with redness & pain around her Rt eye.

(a) What is the most likely diagnosis?What is the most serious differential diagnosis?
Answer: Preseptal(periorbital) cellulitis & orbital cellulitis.(Preseptal cellulitis is much more common than orbital cellulitis)
(b) What are the most common pathogens to cause this condition?(Name 2)
Answer: The most common inciting organisms of preseptal cellulitis include St. pneumoniae, Staph.aureus, other St.
species & anaerobes.
(c) Name 2 indications for CT scanning?
Answer: Inability to accurately assess vision, gross proptosis, ophthalmoplegia, bilateral edema or deteriorating
visual acuity & signs or symptoms CNS involvement.
(d) How would you manage this Pt?(2 points)
Answer: Broad-spectrum oral antibiotics, consider anaerobic cover, opthalmology consultation, close observation.
(e) Name some complications of this condition?(Name three)
Answer: Recurrent preseptal cellulitis, orbital cellulitis, vision loss, death
‫ـــــــــــــــــــــــــــــــــــــــــــــــــــــــــــــــــــــــــــــــــــــــــــــــــــــــــــــــــــــــــــــــــــــــــــــــــــــــــــــــــــــــــــــــــــــــــــــــــــــــــــــــــــــــــــــــــــــــــــــــــــــــــــــــــــــــــــــ‬
67- Question 31 of 50
A 69 yo man developed a sudden onset painful left upper limb while at rest. He also complained of paraesthesia. O/E
he was in severe distress & his left upper limb was pulseless distal to the brachial pulse & extremely pale.

(a) What is the most likely diagnosis?


Answer: Ischaemic limb
(b) Name three other medical conditions that frequently co-exist with this condition?
Answer: Other maladies that often coexist with PVD are coronary artery disease, myocardial infarction (MI), atrial
fibrillation, transient ischemic attack, stroke & renal disease
(c) Name 2 other points that should be noted in the exam. of the Pt?
Answer: Presence or absence of heart murmurs. Investigate all peripheral vessels, including carotid, abd. & femoral,
for pulse quality & bruit. Note that the dorsalis pedis artery is absent in 5-8% of normal subjects, but the posterior
tibial artery usually is present. Both pulses are absent in only about 0.5% of Pts.
(d) What are the mangement steps in the emergency deparment?
Answer: Analgesia.Urgent vascular surgery consultation. +/- heparin infusion.
(e) Name three risk factors for this condition?
Answer: Smoking, hyperlipidemia, DM & hyperviscosity.
‫ـــــــــــــــــــــــــــــــــــــــــــــــــــــــــــــــــــــــــــــــــــــــــــــــــــــــــــــــــــــــــــــــــــــــــــــــــــــــــــــــــــــــــــــــــــــــــــــــــــــــــــــــــــــــــــــــــــــــــــــــــــــــــــــــــــــــــــــ‬
68- Question 32 of 50
A 65 yo man with lung cancer comes in as he has noticed a swelling to his Rt arm & has had worsening SOB over the
course of the day. He looks stressed & a bit red in the face. His arm is indeed swollen & you can see his ext. JV easily

(a) What do you suspect might be causing his symptoms?


Answer: Need to think about SVC obstruction.
(b) How would you investigate him?
Answer: Urgent admission & CT chest to see what is causing the obstruction
(c) Is there any particular therapy that could be started prior to any investigations?
Answer: Dexamethosone 8mg BD (high dose)
(d) What is the definitive TTT?
Answer: Radiotherapy (within 24hrs) +/- stenting.
(e) What is tumour lysis syndrome & when does it occur?
Answer: Occurs after starting chemotherapy, more common bulky chemosensative disease (lymphoma, germ-cell
tumours, high blast-count leukaemias) can occur within hrs or days. It is caused by rapid lysis of tumour cells by the
chemotherapeutic agents. The cell death leads to leak of metabolites & can lead to ARF.
‫ـــــــــــــــــــــــــــــــــــــــــــــــــــــــــــــــــــــــــــــــــــــــــــــــــــــــــــــــــــــــــــــــــــــــــــــــــــــــــــــــــــــــــــــــــــــــــــــــــــــــــــــــــــــــــــــــــــــــــــــــــــــــــــــــــــــــــــــ‬
69- Question 35 of 50
A 23 yo lady is wheeled into the resus department complaining of chest pain & dizziness. She also says that she can’t
feel her fingers. The crew report that she has no medical history, she’s a non-smoker & is normally very fit & well.
26
Her observat. in the ambulance show a RR of 45 but are otherwise unremarkable apart from her seeming very anxious

(a) What is the most likely diagnosis from the history given above?
Answer: 1ry hyperventilation, pyschongenic (panic attack)
(b) What tests must you do to confirm your initial thoughts?
Answer: Need to rule out 2ry causes for hyperventilation i.e. DKA Kussmal's breathing therefore to a BM,
Saturations: pneumothroax/PE ECG: cardiac cause
(c) What will you do with this Pt?
Answer: Reassure her that there is nothing serious going on & encourage her to take control of her respirations
perhaps counting breathe in through the nose, count for 6, breathe out through the mouth count for 6, hold for 3 etc.
(d) The RR doesn’t come down & despite your efforts the Pt isn’t changing or improving. What tests would you
do now?
Answer: ABG, CXR, U&E, Bl. glucose consider tox screen
(e) Name a group of presentations common to the ED which could present in this way.
Answer: Overdose of: Aspirin/CO/ Methanol/ cyanide/ ethylene glycol
‫ـــــــــــــــــــــــــــــــــــــــــــــــــــــــــــــــــــــــــــــــــــــــــــــــــــــــــــــــــــــــــــــــــــــــــــــــــــــــــــــــــــــــــــــــــــــــــــــــــــــــــــــــــــــــــــــــــــــــــــــــــــــــــــــــــــــــــــــ‬
70- Question 36 of 50
The same 46-yo septic Pt that you met in a previous question is now on the ITU. He has sepsis from pneumonia.

(a) He has a Hb of 8 g/dL & you consider giving a Bl. transfusion. What is the current best evidence around this?
Answer: RBC transfusions for adults should occur only when Hb is < 7.0 g/dL to a target Hb between 7 & 9 g/dL
(b) His platelets have been falling & are currently 20,000mm/3 Should you give a platelet transfusion?
Answer: No unless there's a very high bleeding risk. When drops below 5000/mm3 then they should be given regardless
(c) What platelet level is normally considered minimum when considering surgery or other invasive procedures?
Answer: 50,000mm/3
(d) When considering how a ventilator should be set with this Pt what are the important things to consider to
reduce the chances of ALI/ARDS?
Answer: Lower tidal volume mechanical ventilation (6 mL/kg based on ideal body weight) can reduce mortality rates
to 22.1% from 39.8% compared with conventional methods (12 mL/kg based on ideal body weight) Tidal volumes
should be ↓ over 1 to 2 hrs to a low TV (6mL/kg predicted body weight) as a goal (grade 1B recommendation) in
conjunction with the goal of maintaining peak airway Pr.s below 30 cm H2O (grade 1C recommendation).
(e) What other therapies need to be considered in this Pt?
Answer: Stress ulcer prophylaxis, DVT prophylaxis
‫ـــــــــــــــــــــــــــــــــــــــــــــــــــــــــــــــــــــــــــــــــــــــــــــــــــــــــــــــــــــــــــــــــــــــــــــــــــــــــــــــــــــــــــــــــــــــــــــــــــــــــــــــــــــــــــــــــــــــــــــــــــــــــــــــــــــــــــــ‬
71- Question 7 of 10
A 22 yo gentleman presents to the ED at 03:00 am, he has been out at a party. He is accompanied by a friend who
tells you that he was previously completely well & has no medical history. When you examine him he is only
responding to pain & has a GCS of 10 (E2, V3, M5). He has a temp. of 38.8 �C, pulse of 120 bpm & Systolic BP of
85. You make a diagnosis of septic shock

(a) What are the 4 most important initial management interventions?


Answer: O2 & airway protection (+/- intubation & IPPV) Obtaining IV access, taking Bl. cultures & starting broad
spectrum antibiotics early, Aggressive fluid therapy, Early involvement of ITU
(b) Define SIRS
Answer: Body temp. of >38 C or <36 C HR >90 bpm RR> 20 or PaCO2 <4.3 KPa WCC >12x 109/litre or <4 x
109/litre or > 10% immature band forms
(c) What investigations would you like to carry out?
Answer: ABG, CXR, MSU, Bl. Cultures, Bl. Glucose, CT head
(d) Assuming a working diagnosis of septic shock what parameters would be sensible TTT targets in this
previously fit young man? Where should this Pt be managed?
Answer: MAP >65 mmHg, CVP 8-12 mmHG (12-15mmHg if ventilated), ITU with arterial & central venous lines
(e) Describe the technique of inserting an internal jugular central line using ultrasound guidance:
Answer: Full aseptic technique, Describe landmarks & how to use probe to identify the vein, Description of Sell-
dinger technique, Ensure all ports aspirate freely, 4 point fixation with skin sutures, Ensure CXR is ordered prior to use
‫ـــــــــــــــــــــــــــــــــــــــــــــــــــــــــــــــــــــــــــــــــــــــــــــــــــــــــــــــــــــــــــــــــــــــــــــــــــــــــــــــــــــــــــــــــــــــــــــــــــــــــــــــــــــــــــــــــــــــــــــــــــــــــــــــــــــــــــــ‬
72- Question 1 of 5
A 53 yo man presents with a swollen hot lt knee. He has also noticed that his calf muscle is hurting & there is
erythema over the calf. He has a background of gout, angina & HTN, he is allergic to penicillin. He is sure that there
has been no trauma to the knee. His temp. is 38.2

(a) Give three differential diagnoses in this scenario


27
Answer: 1: Septic arthritis 2: Gout/CPPD with local concurrent cellulitis of the leg 3: Reactive arthritis/Rieter's
syndrome 4: Gonococcal arthritis
(b) What are the 2 key investigations that you must perform in the ED?
Answer: 1: Bl. cultures� 2: Joint aspiration, gram stain, microscopy & polarized light microscopy.
(c) What test is used to determine the presences of crystal disease within a joint?
Answer: The use of polarized light microscopy. Gout -vely birefringent crystals that are needle shaped.
‫ـــــــــــــــــــــــــــــــــــــــــــــــــــــــــــــــــــــــــــــــــــــــــــــــــــــــــــــــــــــــــــــــــــــــــــــــــــــــــــــــــــــــــــــــــــــــــــــــــــــــــــــــــــــــــــــــــــــــــــــــــــــــــــــــــــــــــــــ‬
73- Question 4 of 5
A 46 yo gentleman is brought in by the ambulance crew. He has been unwell at home for the last 3 days with a cough.
His wife found him drowsy & unresponsive when she arrived home from work & dialled 999. His initial observations
are as follows, pulse 125 regular, Bp un-recordable, temp 38.7, RR28, GCS 13

(a) You presume a working diagnosis of sepsis. What are the 4 parameters that need to be aggressively
achieved within the golden hr- first 6 hrs of TTT?
Answer: Maintain strict parameters of normal values of CVP between 8 & 12 mm Hg (12 to15 mm Hg in the
mechanically vented Pt) with crystalloid or colloid infusions, MAP≥ 65, ScvO2≥ 70% or SvO2≥ 65%, UO≥ 0.5 mL/kg/hr
(b) You instigate aggressive fluid resuscitation with Hartman's & place a central line. The Pt initially responds
& now has a MAP of 50. You have infused 4 litres but the MAP remains 50 & his GCS is 14, UO is <0.5ml/hr,
SvcO2 is 60%. What are the next crucial steps for this Pt?
Answer: Needs urgent Bl. cultures take 20mls from 2 sites. Needs early antibiotic therapy. As he is not meeting the
targets despite fluid resuscitation you need to instigate the following. As the SvcO2 has not improved need to consider
the following, Additional fluid, Transfusion of red Bl. cells (RBCs) as needed to hematocrit ≥ 30%, Inotropic agents
(dobutamine 2.5 to 20 micrograms (mcg)/kg/min)
(c) Considering all comers what is the most likely cause of sepsis?
Answer: Lung: 35%; Abd.: 21%; Urinary tract: 13%; Skin & soft tissue: 7%; Other site: 8%; & Unknown: 16%..
(d) What are the recommended 1st line vasopressors?
Answer: Dopamine & norepinephrine
(e) What is the current recommendation for the use of steroids in septic shock?
Answer: Steroids. IV steroids (hydrocortisone 200 to 300 mg/day) for 7 days or 4 divided doses or by continuous
infusion is suggested only for Pts who, despite adequate fluid replacement, require vasopressor therapy to maintain
Bl. Pr. (grade 2C recommendation). This approach has only demonstrated ↓ mortality in those with relative adrenal
insufficiency (defined as postadrenocorticotropic hormone [ACTH] cortisol ≤9 mcg/dL).[10,11,35] Despite the long
-standing recommendat. to limit use of steroids to Pts with sepsis who (a) remain hypotensive despite adequate fluid replacement
& vasopressor therapy & (b) have insufficient rise in cortisol level from corticotropin challenge, steroids continue to
be widely used for those with septic shock.[36] For this reason, Sprung & colleagues[36] of the Corticotherapy for
Septic Shock (CORTICUS) study put this question to the test: Does the use of steroids for septic shock improve
mortality in a broader range of Pts with septic shock? Results from this landmark trial showed that hydrocortisone did
not reduce mortality Pts with sepsis at large & did  the risk for superinfection. Coupling this data (available but not
yet published at the time of the phase 2 SCC clinical guideline update) with the results of the study by Annane &
associates[35] published in 2002, the phase 2 SCC guidelines reiterated the restricted use of steroids to the population
described, & the strength of the rating was downgraded from the original guidelines published in 2004. The experts
who participated in SCC phase 2 debated about how best to communicate this recommendat. to clinicians, put
different wording options to a vote & the result was the following statement: "We suggest that IV hydrocortisone be
given only to adult septic shock Pts with Bl. Pr. poorly responsive to fluid resuscitat. & vasopressor therapy"(grade 2C)
(f) Who should receive Recombinant human activated protein C (rhAPC)?
Answer: An APACHE II score of 25 or greater; Sepsis-induced multiple organ failure; & No absolute
contraindications, related to bleeding risks. No mortality benefit in Pts with single-organ dysfunct. or APACHE ll < 25
‫ـــــــــــــــــــــــــــــــــــــــــــــــــــــــــــــــــــــــــــــــــــــــــــــــــــــــــــــــــــــــــــــــــــــــــــــــــــــــــــــــــــــــــــــــــــــــــــــــــــــــــــــــــــــــــــــــــــــــــــــــــــــــــــــــــــــــــــــ‬
74- Question 5 of 5
A 66 yo man presents with sudden severe ripping chest pain radiating to his back. He has a history of HTN. O/E there
is a diastolic murmur. You suspect an aortic dissection.

(a) What are the risk factors for this condition?(Name four)
Answer: The most important predisposing factor for acute aortic dissection is systemic HTN. Inflammatory diseases
that cause a vasculitis (giant cell arteritis, takayasu arteritis, rheumatoid arthritis, syphilitic aortitis) disorders of
collagen (eg, marfan syndrome, ehlers-danlos syndrome, annuloaortic ectasia) a bicuspid aortic valve, aortic
coarctation, turner syndrome, crack cocaine, previous aortic valve replacement, cardiac catheterization, trauma, high-
intensity weight lifting or other strenuous resistance & a history of CABG surgery are other associations.
(b) What other features (besides a diastolic murmur) in the exam. of this Pt may indicate an aortic
dissection?(Name 2)
28
Answer: Assymetry or absence of peripheral pulses or a pulse deficit, hypotension with features of tamponade, HTN,
neurological signs 2ry to carotid or spinal artery involvement.
(c) How would you investigate this Pt? (Name four)
Answer: ECG, CXR, FBC, UE, Glucose, Coag, Bl. group & crossmatch, TOE , CT Angiography.
(d) What features on the CXR give additional evidence to the suspected diagnosis?(Give three)
Answer: A widened mediastinum, a left sided pleural effusion, deviation of the trachea or NG tube to the Rt,
separation of 2 parts of the wall of a calcified aorta by >5mm (the Ca sign) & a double knuckle aorta.
(e) How would you manage this Pt if you suspected an aortic dissection?(Give four)
Answer: O2, 2 large bore IV cannulae, cross match Bl., IV opioid, specialist consultation, arterial line & BP control.
‫ـــــــــــــــــــــــــــــــــــــــــــــــــــــــــــــــــــــــــــــــــــــــــــــــــــــــــــــــــــــــــــــــــــــــــــــــــــــــــــــــــــــــــــــــــــــــــــــــــــــــــــــــــــــــــــــــــــــــــــــــــــــــــــــــــــــــــــــ‬
75- Question 2 of 20
A 24 yo woman has taken 37 paracetamol tablets & downed them with half a bottle of white rum. She said that she
wanted to end it all� She presents 3 hrs after taking the tablets. She weighs 55kg.

(a) What do you do?


Answer: Treat initially as she has taken> 150mg/kg. This is generally the threshold that will receive N-acetylcysteine.
(b) Give some risk factors that would make you want to treat Pts you had taken small overdoses
Answer: Taking enzyme inducing drugs such as Rifampicin, St John�s Wort, Phenytoin, Carbamazepine,
Barbiturates, Sulphonylureas, Alcohol (chronic excess) these increase the rate of paracetamol breakdown & hence
increase the amount of NAPQI which is toxic!
(c) What does the toxic metabolite of paracetamol bind to in the liver?
Answer: Glutathione
(d) Which test is the first to change if there has been liver damage?
Answer: Coagulopathies therefore the INR is the first to change & is sensitive for acute liver damage.
(e) Name an alternative to Parvolex�?
Answer: Methionine
‫ـــــــــــــــــــــــــــــــــــــــــــــــــــــــــــــــــــــــــــــــــــــــــــــــــــــــــــــــــــــــــــــــــــــــــــــــــــــــــــــــــــــــــــــــــــــــــــــــــــــــــــــــــــــــــــــــــــــــــــــــــــــــــــــــــــــــــــــ‬
76- Question 3 of 20
A 26 yo male presented to the ED with a stiff jaw & being unable to open his mouth. 3 days previously he was
immunized with tetanus toxin after lacerating his finger. O/E he had evidence of 'lock jaw'. His injured finger was
swollen , painful & exuding pus.

(a) What is the most likely diagnosis?


Answer: Tetanus.
(b) What is the organism?
Answer: Clostridium tetani.
(c) Where is the organism commonly found?
Answer: Soil , & faeces of domestic animals.
(d) What are the features of this condition?(Name four)
Answer: Rigidity, stiffness, reflex spasms,tachycardia, Bl. Pr. instability dysphagia,laryngeal spasm & opisthotonus.
(e) What is the immediate management?
Answer: Intramuscular human tetanus immunoglobulin, cleaning & if necessary surgical debridement of the wound
‫ـــــــــــــــــــــــــــــــــــــــــــــــــــــــــــــــــــــــــــــــــــــــــــــــــــــــــــــــــــــــــــــــــــــــــــــــــــــــــــــــــــــــــــــــــــــــــــــــــــــــــــــــــــــــــــــــــــــــــــــــــــــــــــــــــــــــــــــ‬
77- Question 4542
A 62 yo man presents acutely SOB. His vital signs are as follows: BP 78/49 pulse 110, he has a raised JVP, heart
sounds are muffled & inspiratory crackles are heard throughout the chest. He has a metal mitral valve & he admits to
having forgotten to take is warfarin the last week as he was on holiday in Spain.

(a) You perform a quick transthoracic ECHO which shows a mitral valve thrombus, what will you do?
Answer: Thrombolysis. He has an acute valve thrombosis resulting in cardiogenic shock. If he was stable then
surgery would be a better option.
(b) How do you measure the effect of your TTT?
Answer: Serial ECHO
(c) What is the best way to investigate/image a pt like this?
Answer: TOE gives much better views.
(d) List the differential diagnosis for the above Pt if you didn�t have access to ECHO
Answer: CCF, Cardiac tamponade, Tension pneumothorax? Cardiogenic shock post M.I.
‫ــــــــــــــــــــــــــــــــــــــــــــــــــــــــــــــــــــــــــــــــــــــــــــــــــــــــــــــــــــــــــــــــــــــــــــــــــــــــــــــــــــــــــــــــــــــــــــــــــــــــــــــــــــــــــــــــــــــــــــــــــــــــــــــــــــــــــــ‬
78- Question id: 2109

29
A 55 yo man presents with a 6 hr history of palpitations that woke him at 05:00am. His BMI is 29 but he is otherwise
well & takes no medication.

Fig1
(a) An ECG (fig 1) reveals the following rhythm: what is it?
Answer: Atrial fibrillation with rapid ventricular response
(b) What do you need to establish quickly?
Answer: Whether this rhythm is compromising the Pt or not i.e. are they stable? Reduced conscious level Systolic BP
<90 Chest pain Signs of heart failure
(c) What questions need to be asked in the history to try to establish a cause?
Answer: Any history of IHD or family Hx of structural (HOCM) or coronary disease, HTN, alcohol binge, caffeine
intake, hyperthyroidism, recent PE, acute pericarditis, acute pulmonary disease etc.
(d) You consider this Pt to be stable & he seems otherwise well. Would he be a candidate for pharmacologic
cardioversion? What would contraindicate this?
Answer: Probably yes, if there is any suspicion of cardiac failure LVF then it is contraindicated. Many drugs that
could be used including sotalol, flecanide, quinidine, propafenone, disopyramide.
(e) Later on that day another Pt comes in who is in what seems to be the same rhythm shown in the ECG in
part a, she is 78 & has a history of palpitations on & off over the yrs. She takes digoxin & aspirin. She is
haemodynamically stable. Where does your management focus lie?
Answer: The cornerstones of AF management are controlling Pts symptoms & preventing thromboembolic
complications, not restoration of sinus rhythm. 1st line TTT would be beta blockers or dihydrpyridine Ca channel
blockers (verapamil or diltiazem) which are effective during exercise & at rest, digoxin is only effective at rest &
should be considered a second line agent.
‫ـــــــــــــــــــــــــــــــــــــــــــــــــــــــــــــــــــــــــــــــــــــــــــــــــــــــــــــــــــــــــــــــــــــــــــــــــــــــــــــــــــــــــــــــــــــــــــــــــــــــــــــــــــــــــــــــــــــــــــــــــــــــــــــــــــــــــــــ‬
79- Question 10 of 20
You are alerted that a man (looks about 50) has collapsed outside the department after leaving the hospital from a
renal out Pt appointment. You rush outside with a portable defibrillator & some equipment. When you arrive at the
scene there is a crowd & the Pt appears to have arrested. You decide that it is too far to try to move him to the ED &
you don't have a trolley so you shout for help & start the resuscitation

(a) You have a good team & you quickly intubate the Pt. The rhythm is VF & you deliver a shock. What size
ET tube did you use? What is the ratio of ventilations to compressions now?
Answer: 8 or 9 normally for an adult male. 7 or 8 for an adult female. When intubated the compressions are
continuous as are the ventilations.
(b) You get a pulse back after the third shock with one dose of adrenaline given. You quickly transport the Pt
to the resus room. What do you do now?
Answer: The Pt is intubated so you need to assess for signs of life & check if he is making any respiratory effort it is
likely that you will need to continue ventilating him. Check an ABG & send off Bl.s, get an ECG
(c) The potassium is 7.2mmol/L. What do you do?
Answer: Consider that this has caused the VF arrest, needs to be treated. Give 10mls of 10% Ca gluconate. Consider
Na bicarbonate particularly if there is severe acidosis/renal failure, which there clearly will be in this case. Give
insulin & glucose Consider haemodyalysis on ITU
(d) What ECG changes are seen in hypokalaemia?
Answer: Prominent U waves & flattened T waves.
30
(e) What is the recommended maximum infusion rate for potassium? What is essential for giving IV K+?
Answer: 20mmols/hr is the recommended maximum infusion rate but sometimes i.e. peri-arrest arrhythmias/cardiac
arrest due to hypokalaemia can be given faster but ideally this should be through a central line. Must have cardiac
monitoring to give IV replacement especially at the rates described.
‫ـــــــــــــــــــــــــــــــــــــــــــــــــــــــــــــــــــــــــــــــــــــــــــــــــــــــــــــــــــــــــــــــــــــــــــــــــــــــــــــــــــــــــــــــــــــــــــــــــــــــــــــــــــــــــــــــــــــــــــــــــــــــــــــــــــــــــــــ‬
80- Question 11 of 20
A 27 yo female presents with palpitations of sudden onset which she has had before.

(a) What does the ECG show (fig 1)?


Answer: Supraventricular tachycardia (SVT)
(b) In a young healthy individual with a normal resting ECG what is this type of rhythm likely due to?
Answer: It is likely to be an atrioventricular nodal re-entrant tachycardia (AVNRT) i.e. the most common cause of
narrow complex tachycardia in Pts with normal hearts.
(c) How do you treat it in the ED providing that she is stable? (Include exact drug doses & sequences)
Answer: Attempt vagal manoeuvres, valsalva with 50 ml syringe, carotid sinus massage etc. If fails try adenosine
6mg, 12mg, 12mg
(d) Describe what you might need to explain to a Pt prior to the interventions you gave in part c.
Answer: Warn of the side effects of adenosine feeling of sudden chest discomfort & flushing Pts say that it feels terrible!
(e) What will the definitive management options?
Answer: Radio frequency ablation is the 1st line TTT for recurrent symptomatic episodes, which is curative. Or AV
blocking drugs B-blockers, diltiazem or verapamil can be used as a pill in the pocket, type approach to terminate an event.
‫ـــــــــــــــــــــــــــــــــــــــــــــــــــــــــــــــــــــــــــــــــــــــــــــــــــــــــــــــــــــــــــــــــــــــــــــــــــــــــــــــــــــــــــــــــــــــــــــــــــــــــــــــــــــــــــــــــــــــــــــــــــــــــــــــــــــــــــــ‬
81- Question 12 of 20
A 55 yo lady is brought in complaining of severe abd. pain. She is an epileptic & takes Carbamazepine for this; she's
normally fit & well her only other medicines are the OCP. You examine her abd. which is soft with no signs of
peritonism. She is tachycardic but hypertensive. She also complains of loss of sensation in her lower limbs. She is also
agitated. Her Bl. reveal a Na of 125mmmols/L. Her urine sample that was taken 30 minutes ago looks brown/red.

(a) What could be going on here? What would you do to confirm your suspicions?
Answer: Although the differential is wide the history is suggestive of acute porphyria Other differentials include
acute abdo pain (any cause of) Guillain-Barr syndrome, Systemic lupus erythematosus Test the urine for
porphobilinogen (PBG) (send a urine sample that is protected from light)
(b) Urinary porphobilinogen is markedly  which confirms your suspicions about what is going on. What will you do?
Answer: Manage pain! Normally requires opiod analgesia In severe attacks, a glucose 10% infusion is commenced,
which may aid in recovery. Supportive TTT ensure that high carbohydrate feed is given. Haem arginate are the drugs
of choice in acute porphyria Consider propanolol to treat HTN
(c) What are the causes of the condition described?
Answer: Abnormalities of haem-biosynthesis, They are broadly classified as hepatic porphyrias or erythropoietic
porphyrias, based on the site of the overproduction & mainly accumulation of the porphyrins (or their chemical
precursors). They manifest with either skin problems or with neurological complications (or occasionally both).
(d) List things that can precipitate an attack of the condition described?
Answer: ETOH, lead poisoning, iron deficiency, drugs(carbamazipine, OCP, sulphonamides, methyldopa,
barbiturates, danazol, chloramphenicol, tetracyclines, some antihistamines,) smoking, sudden dieting, emotional &
physical stress, pregnancy etc .
‫ـــــــــــــــــــــــــــــــــــــــــــــــــــــــــــــــــــــــــــــــــــــــــــــــــــــــــــــــــــــــــــــــــــــــــــــــــــــــــــــــــــــــــــــــــــــــــــــــــــــــــــــــــــــــــــــــــــــــــــــــــــــــــــــــــــــــــــــ‬
82- Question 14 of 20
1 of your staff nurses (aged 28) asks your advice because she has had loose bowel motions for 2 wks since returning
from India. She is worried she may have dysentry. She has 8 loose stools per day with abd. cramps & for 3 days has
noticed some Bl. in the stool. She is previously healthy.

(a) Give 3 possible differential diagnosis.


Answer: Bacterial GE, IBD, Viral GE, Parasitic disease, Coeliac disease, Tropical sprue, Amoebic dysentry
(b) Give one indication for antibiotics in a Pt who presents with diarrhoea.
Answer: Severe invasive disease Bl. / refractory, prolonged diarrhoea.
(c) Other than ABs, what 2 other medicat. may you consider & give the rationale for their use in Pts with diarrhoea
Answer: Simple analgesia e.g paracetamol for cramping pain Consider immodium to ↓ motility Fluid & electrolyte
replacement e.g diaralyte
(d) After discussion with microbiology, you decide to prescribe a course of antibiotics for her. What other 4
pieces of advice would you give her?
31
Answer: Hydration, Hand washing/hygiene, Occupational health clearance prior to return to work, Caution with local
contacts (family/friends/food preparat. etc), Follow up stool culture, Avoid lactose containing foods until diarrhoea stops
‫ـــــــــــــــــــــــــــــــــــــــــــــــــــــــــــــــــــــــــــــــــــــــــــــــــــــــــــــــــــــــــــــــــــــــــــــــــــــــــــــــــــــــــــــــــــــــــــــــــــــــــــــــــــــــــــــــــــــــــــــــــــــــــــــــــــــــــــــ‬
83- Question id: 4536
A 44 yo woman comes in to the ED c/o headache & visual disturbance. She has essential HTN. Her BP is 235/119.

Fig 1
(a) What is occurring?
Answer: You don't know yet until full exam is performed. This is HTN urgency which differs from a HTN crisis/
malignant HTN. A HTN emergency is a condition in which  Bl. Pr. results in target organ damage. HTN urgency
must be distinguished from emerg. Urgency is defined as severely  Bl. Pr. (ie, systolic >220 mm Hg or diastolic>120
mm Hg) with no evidence of target organ damage. For malignant HTN to be diagnosed papiloedmea must be present.
(b) What exam. is critical here?
Answer: Need to look at the fundi for papiloedema or other changes associated with vascular damage such as flame-
shaped haemorrhages or soft exudates, but without papilloedema.
(C) How would you treat her?
Answer: Depends if this turns out to be a HTN emergency or not, if not then aim to reduce the BP slowly if no
contraindications for a B-blocker then this is a good option i.e. Atenolol 25mg. HTN emergencies require immediate
therapy to ↓ Bl. Pr. within minutes to hrs. In contrast, no evidence suggests a benefit from rapidly ↓ Bl. Pr. in Pts with
HTN urgency. In fact, such aggressive therapy may harm the Pt, resulting in cardiac, renal, or cerebral hypoperfusion.
(d) The funoscopic picture reveals the following (see figure 1). What do you do?
Answer: Once the diagnosis of HTN emergency is made, the most commonly used IV drug is nitroprusside. An
alternative for Pts with renal insufficiency is IV fenoldopam. Labetalol is another common alternative, providing easy
transition from IV to oral dosing. B-blockade can be accomplished IV with esmolol or metoprolol. Also available
parenterally are diltiazem, verapamil & enalapril. Hydralazine is reserved for use in pregnant Pts, while phentolamine
is the drug of choice for a pheochromocytoma crisis. Pt should be managed with an arterial line on ITU.
‫ـــــــــــــــــــــــــــــــــــــــــــــــــــــــــــــــــــــــــــــــــــــــــــــــــــــــــــــــــــــــــــــــــــــــــــــــــــــــــــــــــــــــــــــــــــــــــــــــــــــــــــــــــــــــــــــــــــــــــــــــــــــــــــــــــــــــــــــ‬
84- Question 20 of 20
Its 03:00 am & you are asked to see a 24 yo girl with chest pain who has been out clubbing. A friend accompanies her &
they are both very talkative although the girl does indeed appear to be in severe pain. They admit to occasional substance misuse.

(a) The ECG seems to show widespread changes that look like an ST elevation M.I. What do you need to do?
Answer: Need to get a through history very quickly. Ask about risk factors for coronary disease. Need to establish if
she has taken illicit substances particularly cocaine, how much & when exactly she took it.
(b) What important & potentially life threatening condition should be ruled out in this Pt at this stage?
Answer: Could the pain be related to aortic dissection? Unlikely given the nature of pain described but increased risk
with cocaine use. Would be pertinent to at least do CXR before treating anything else.
(c) Yours suspicions were correct; she is having a myocardial infarction. How do you manage this Pt?
Answer: General measures are the same as anyone presenting with acute M.I.: MONA. In addition IV GTN to be
given at higher doses titrate but aim for high dose > 10mg/hr final level. Benzodiazepines to reduce anxiety
(d) You instigate initial measures as described above, what second line pharmacological agents could you use?
Answer: Verapamil: in high doses reduces cardiac work load & hence restores O2 supply & demand as well as
reversing coronary vasoconstriction. Phentolamine: α-adrenergic antagonist & reverses vasoconstriction. Labetalol:
both α & β adrenergic effects it can be used after verapamil & phentolamine if Pt remains hypertensive.
(e) The Pt fails to improve what should happen next?
Answer: PCI. Evidence for thrombolysis is weak & generally associated with poor outcome 2ry to HTN induced
haemorrhagic complicat.
‫ـــــــــــــــــــــــــــــــــــــــــــــــــــــــــــــــــــــــــــــــــــــــــــــــــــــــــــــــــــــــــــــــــــــــــــــــــــــــــــــــــــــــــــــــــــــــــــــــــــــــــــــــــــــــــــــــــــــــــــــــــــــــــــــــــــــــــــــ‬
85- Question id: 2029
A 45 yo man presented to the ED with a 6 hr history of progressive neck swelling & fever.
32
(a) What is the most common cause of the condition in the picture?
Answer: The most common cause of Ludwig's angina is dental infection especially of the second & third lower
molars5. Predisposing factors include dental carries, recent dental TTT, systemic illnesses such as DM, malnutrition,
alcoholism & immunosuppression & immunocompromise.

(b) What are the common causative organisms?


Answer: The most common causative organisms result from polymicrobial odontogenic infections extending into the
deep facial spaces. F. nucleatum, Bacteroides & Prevotella spp, Peptostreptococcus spp., Actinomyces spp.,
Streptococcus spp & Staphylococci are the most common isolates
(c) What is appropriate antibiotic cover?
Answer: Appropriate antibiotics include high dose penicillin(+/- metronidazole), & clindamycin or more broad
spectrum antibiotics such as ticarcillin-clavulanate, piperacillin-tazobactam or ampicillin-sulbactam.
(d) Discuss some non-antibiotic TTTs for this condition.
Answer: Dexamethasone reduces oedema & cellulitis & provides an initial chemical decompression protecting the
airway as well as allowing improved antibiotic penetration into the area. Nebulised adrenaline (1ml of 1:1000 diluted
to 5ml of 0.9% normal saline) is also safe & effective in reducing upper airway obstruction.
(e) What is the mortality for this condition?
Answer: It had a 50% mortality before the common use of antibiotics due to upper airway oedema but therapeutic
advances including antibiotics & supportive therapy have reduced the mortality to 8%.
‫ـــــــــــــــــــــــــــــــــــــــــــــــــــــــــــــــــــــــــــــــــــــــــــــــــــــــــــــــــــــــــــــــــــــــــــــــــــــــــــــــــــــــــــــــــــــــــــــــــــــــــــــــــــــــــــــــــــــــــــــــــــــــــــــــــــــــــــــ‬
86- Question 7 of 20
A 74 yo lady presents with severe left sided loin pain that radiates to the left side of the abd.. She seems distressed &
cannot keep still on the bed. You consider a diagnosis of renal colic.

(a) What are the initial priorities?


Answer: Clearly ABCD as ever- analgesia is key! Also hydrat. as Pts are often dehydrated from prolonged vomiting.
(b) What is the investigation of choice?
Answer: CT of the abd. 95% specific & 95% sensitive- also has the added advantage of picking up other things i.e. AAA
(c) What %age of Pts do not have haematuria?
Answer: Around 15% (e Medicine)
(d) What is the sex ratio for renal calculi?
Answer: The overall lifetime rate of kidney stones in the general population is approximately 12% for men & 4% for
women. (Emedicine)
(e) When can a Pt be discharged with renal colic?
Answer: If the pain has settled & the imaging has shown no signs of obstruction (NB that sometimes the pain going
can represent obstruction) As long as the Pt is otherwise well.
‫ـــــــــــــــــــــــــــــــــــــــــــــــــــــــــــــــــــــــــــــــــــــــــــــــــــــــــــــــــــــــــــــــــــــــــــــــــــــــــــــــــــــــــــــــــــــــــــــــــــــــــــــــــــــــــــــــــــــــــــــــــــــــــــــــــــــــــــــ‬
87- Question id: 4518
An 88 yo man is brought in from a nursing home with a GCS of 12. He had been a little sleepy over the last few days.
You find out that he was found slumped in a chair with a left sided facial droop.
(a) What are your first priorities?
Answer: You need a full history of events from the home. Full ABC assessment, rule out the simple correctable
causes for a lowered GCS for example Bl. glucose, any evidence of poisoning etc, Full neurological assessment.
33
(b) You organise a CT head, which is shown in figure 1. Describe what it shows
Answer: Large Rt sided subdural haematoma with evidence of an acute on chronic bleed. There is also global
cerebral atrophy & midline shift.

Fig 1
(c) What must have happened to this gentleman?
Answer: Must have had head trauma at some stage.
(d) What is the TTT?
Answer: Consideration of burr hole evacuation to improve symptoms. Generally, because the lesion represents
clotted Bl., the burr hole is not curative, & emergent craniotomy is necessary.
(e) Why are alcoholics especially susceptible to the above problem?
Answer: Often they have coagulopathies, which puts them at high risk, also prone to falling over when intoxicated.
‫ـــــــــــــــــــــــــــــــــــــــــــــــــــــــــــــــــــــــــــــــــــــــــــــــــــــــــــــــــــــــــــــــــــــــــــــــــــــــــــــــــــــــــــــــــــــــــــــــــــــــــــــــــــــــــــــــــــــــــــــــــــــــــــــــــــــــــــــ‬
88- Question id: 2056
A 76 yo man presented with a sudden onset tearing chest pain radiating to his back. His CXR is shown.

Fig 1
(a) What is the differential diagnosis?(Name five)
Answer: Myocardial ischemia due to an ACS with or without ST segment elevat., pericarditis, PE, aortic regurgitat.
without dissection, aortic aneurysm without dissection, musculoskeletal pain, mediastinal tumors, pleuritis,
cholecystitis, atherosclerotic or cholesterol embolism, PUD or perforating ulcer, acute pancreatitis.
(b) Name four findings on a CXR which are consistent with aortic dissection?
Answer: Widening of the aorta, pleural effusion, widening of the aortic contour, displaced calcification, aortic
kinking, a pleural cap & opacification of the aorticopulmonary window.
(c) What are the risk factors for aortic dissection(Name five)?
Answer: HTN, preexisting aortic aneurysm, inflammatory diseases that cause a vasculitis , disorders of collagen , a
+ve family history, bicuspid aortic valve, aortic coarctation, turner syndrome, coronary artery bypass graft surgery
(CABG), previous aortic valve replacement, & crack cocaine.
(d) What are the potential complications of an ascending aortic aneurysm?(Name five)
Answer: Acute aortic insufficiency, acute myocardial ischemia or MI, cardiac tamponade & sudden death ,
hemothorax & exsanguination , neurologic deficits, horner syndrome & vocal cord paralysis.
(e) How are aortic dissections classified?
Answer: The Daily system classifies dissections that involve the ascending aorta as type A, regardless of the site of
the 1ry intimal tear & all other dissections as type B. In comparison, the DeBakey system is based upon the site of
origin with type 1 originating in the ascending aorta & propagating to at least the aortic arch, type 2 originating in &
confined to the ascending aorta & type 3 originating in the descending aorta & extending distally or proximally.
‫ـــــــــــــــــــــــــــــــــــــــــــــــــــــــــــــــــــــــــــــــــــــــــــــــــــــــــــــــــــــــــــــــــــــــــــــــــــــــــــــــــــــــــــــــــــــــــــــــــــــــــــــــــــــــــــــــــــــــــــــــــــــــــــــــــــــــــــــ‬

34
89- Question 4513
An 18 yo male presented to the ED following a collapse at a local night club. O/E he was drowsy. His temp. was 40
degrees & he was sweating profusely. His HR was 120 bpm & regular. His Bl. Pr. was 170/100 mmHg. His pupils
were dilated & reacted poorly to light. His Bl. investigations revealed a Na of 124 mmol/l.

(a) What is the most likely cause of his presentation?


Answer: Ecstasy (MDMA) abuse.
(b) What are the possible complications of this presentation?(list four)
Answer: Rhabdomyolysis, ARF, DIC, acute hepatitis, MI & CVA.
(c) Give 2 therapeutic steps in the management of his temp..
Answer: Cooling/tepid sponging, paracetamol & IV dantrolene.
(d) Give 2 explanations for the hyponatremia?
Answer: SIADH & excessive Na loss from skin during profuse perspiration.
(e) Name some other causes of hyperpyrexia? (Name four)
Answer: Septicaemia, malaria, viral infections, neuroleptic malignant syndrome, malignant hyperpyrexia, cocaine
abuse, malignancy & aspirin toxicity.
‫ـــــــــــــــــــــــــــــــــــــــــــــــــــــــــــــــــــــــــــــــــــــــــــــــــــــــــــــــــــــــــــــــــــــــــــــــــــــــــــــــــــــــــــــــــــــــــــــــــــــــــــــــــــــــــــــــــــــــــــــــــــــــــــــــــــــــــــــ‬
90- Question id: 2116
A 59 yr old lady with a history of IHD presents with tiredness & SOB. Her initial observations show a HR of 59
b.p.m. Bp of 140/84 & sats of 100%.

Fig 1
(a) What part of the history is key to making any diagnosis here?
Answer: Is she on beta-blockers? If yes then these may be normal observations, also if she was an especially fit 59 yo
it is possible that this represents a normal HR.
(b) What is first degree AV block?
Answer: Prolonged PR interval i.e. > than 0.2 seconds (5 small squares on standard ECG) in itself it is benign but it
may represent IHD, digoxin toxicity, electrolyte disturbances, acute rheumatic carditis
(c) Mobitz type 2 & mobitz type1 (Wenkebach type) are both types of 2ry degree heart block. Which one is
benign & which can lead to complete heart block?
Answer: Wenchebach is normally benign. Mobitz 2 & 2:1 block can lead to third degree �complete heart block.
(d) What does this ECG show (fig 1)?
Answer: Complete heart block (CHB)/third degree block
(e) How would you treat it in the ED if the Pt were unstable?
Answer: Atropine, adrenaline then transcutaneous pacing. Temporary measures before transvenous pacing can be arranged.
‫ـــــــــــــــــــــــــــــــــــــــــــــــــــــــــــــــــــــــــــــــــــــــــــــــــــــــــــــــــــــــــــــــــــــــــــــــــــــــــــــــــــــــــــــــــــــــــــــــــــــــــــــــــــــــــــــــــــــــــــــــــــــــــــــــــــــــــــــ‬
91- Question 3 of 10
A 60 yo man with diet controlled type II DM & HTN was found collapsed at the bottom of the stairs in his home by
his son. He was on a thiazide diuretic. On exam he was drowsy, his HR was 40 bpm, his BP was 150/95mmHg. His
temp. was 36.4 degrees & his JVP was not raised. The heart sounds were normal & his chest was clear.His Rt lower
limb was externally rotated & painful to move, there was extensive bruising on his Rt buttock & thigh.

(a) How would you investigate this Pt?(List 6)


Answer: FBC, UE, LFTs, Glucose, CK, urinalysis, Hip X Ray, ECG,CXR & CT Brain.
(b) His renal profile revealed that his urea was 15 mmol/l & his creatinine was 700 ummol/l.His CK was also
grossly elevated. What is the diagnosis?
Answer: Rhabdomyolysis
(c) What causes the renal failure in this condition?
Answer: Skeletal muscle trauma, inflammat. or infarct. causes  myoglobin levels in the Bl. which is toxic to the renal tubules.
(d) Name four other causes of this condition?
Answer: Electrocution, hypothermia, status epilepticus, ecstasy/amphetamine abuse, burns, septicaemia, statins,
strenuous exercise, neuroleptic malignant syndrome.
(e) How should this Pt be treated?
Answer: Hydration with alkalinization of the urine.
35
92- Question 4 of 10
A 26 yo male presents to the ED with abd. pain & diarrhoea. He tells you that he hasn't felt well for about 6 months but in
the last few wks he has noticed that he has been losing weight & opening is bowels up to 8 times a day, since yesterday
he has had worse abd. pain. O/E he was pale & slim, is abd. was soft & mildly tender throughout with no guarding.

(a) What are the likely differentials in this case?


Answer: 1. IBD- UC/Crohn’s 2. Coeliac disease 3. Any other cause of abd. pain is of course is a possibility here i.e.
appendicitis (diarrhoea can be associated!) 4. Gastroenteritis (unlikely due to chronic Sx but need to ask about foreign
travel for things like giardia & tapeworms etc) 5. Intestinal lynphangiectasia 6. Hypogammaglobulinaemia 7. Small
bowel lymphoma 8. C.diff?? ask re Abx (despite age being young; it is known!)
(b) What is the current laboratory test for celiac disease?
Answer: IgA anti-endomyosial antibodies.
(c) You take some Bl.s what other important investigation needs to be done in the ED?
Answer: Plain abd. film (1 of the few reasons that 1 still needs to do a plain abd. film) to rule out a toxic megacolon.
(d) How do you treat confirmed case of c.difficle?
Answer: Oral metronidazole in the first instance- vancomycin orally if not working.
(e) What group of Pts get caecal volvulus?
Answer: 20-40 presenting with an acute onset of small bowel obstruction.
‫ـــــــــــــــــــــــــــــــــــــــــــــــــــــــــــــــــــــــــــــــــــــــــــــــــــــــــــــــــــــــــــــــــــــــــــــــــــــــــــــــــــــــــــــــــــــــــــــــــــــــــــــــــــــــــــــــــــــــــــــــــــــــــــــــــــــــــــــ‬
93- Question 5 of 10
A 75 yo lady presents with acute pulm. oedema. Your system tells you that she has presented 7 times in the last 5
months with pulm. oedema. An ECHO last time showed a near normal LV function with a gradient across the aortic
valve of 69mmHg. O/E she is SOB has an ESM & a slow rising pulse. She is normally SOB on minimal exertion.

(a) What is the diagnosis here?


Answer: Severe aortic stenosis
(b) What does this Pt need to definitively treat her condition?
Answer: An aortic valve replacement, she is symptomatic with severe AS.
(c) Is this Pt more or less likely to develop Torsades de pointes than a gentleman with a similar presentation?
Answer: More likely, this sex difference is unexplained.
(d) Can Pts with a near normal LV function be classed as having heart failure?
Answer: Yes population based studies report that around a third of Pts with CCF have normal or near normal EF. The
concept of diastolic heart failure is perhaps under recognised in clinical practise. It can only be separated from systolic
heart failure by Doppler echocardiography.
(e) Which of the following medications have been shown to improve prognosis post M.I.? Nicorandil, flecanide,
nitrates, clopidogrel, Ca antagonists
Answer: Clopidogrel
‫ـــــــــــــــــــــــــــــــــــــــــــــــــــــــــــــــــــــــــــــــــــــــــــــــــــــــــــــــــــــــــــــــــــــــــــــــــــــــــــــــــــــــــــــــــــــــــــــــــــــــــــــــــــــــــــــــــــــــــــــــــــــــــــــــــــــــــــــ‬
94- Question id: 4538
A 56 yo male attends the ED with an episode of syncope. It occurred whilst he was out walking his dogs. He smokes
15 a day & has done for yrs; he has controlled HTN & is on a statin. O/E he has an ESM & a soft second heart sound.

Fig 1
(a) He feels completely fine now & wants to go home, what do you need to do in the ED?
Answer: The Pt likely has cardiac syncope related to severe AS. He needs to be admitted & needs a fairly urgent
echocardiogram to assess the aortic valve. He will likely need to have it replaced & will therefore also need
angiography prior to this to guide the cardiac surgeons
36
(b) You perform an ECG (figure 1). Describe what it shows:
Answer: ECG showing gross left ventricular hypertrophy (LVH) with strain in case with severe aortic stenosis. The R
waves in V5 & V6 are so tall that they are overlapping with the tracing in the channel above. ST segment depression
& T wave inversion are seen in inferior & lateral leads. This is a Pr. overload pattern which can be seen also in severe
systemic HTN & hypertrophic obstructive cardiomyopathy.
(c) What is the next step in this Pts' management? 1. Start an ACEi? 2. Tredmill test 3. Percutaneous aortic
balloon valvulotomy 4. Give flecanide 5. Amiodarone 300mg IV over 30 min. 6. All of the above 7. None of the above
Answer: Answer= 7. This Pt if shown to have what you believe clinically to be severe aortic stenosis will need his
valve replacing, valvulotomy is only really used as a bridge to surgery in unstable Pts.
(d) What is the current guidance regarding AB prophylaxis for Pt undergoing dental procedures who have valvular HD?
Answer: That it is NOT REQUIRED. New guidance in 2006 from British Society for Antimicrobial Chemotherapy
(BSAC) states that it is no longer required as there is no evidence that it leads to BE. HOWEVER THIS IS
CONTROVERSIAL & a lot of cardiologists do not agree- we await NICE guidance on this.
‫ـــــــــــــــــــــــــــــــــــــــــــــــــــــــــــــــــــــــــــــــــــــــــــــــــــــــــــــــــــــــــــــــــــــــــــــــــــــــــــــــــــــــــــــــــــــــــــــــــــــــــــــــــــــــــــــــــــــــــــــــــــــــــــــــــــــــــــــ‬
95- Question id: 4527
An 88-yo lady is brought in by ambulance. They were on route to the medical admissions unit but felt that the Pt was
too unwell. She is from a nursing home & has long standing dementia she was sent in by the GP due to a general deteriorate.
& possible dehydrate. The reason the crew became concerned was due to brief periods of unresponsiveness that
seemed to be occurring quite frequently. She has a past history of CVA, IHD, HTN, NIDDM & dementia. She is on asprin,
clopidogrel, simvastatin, ramipril, digoxin, bisoprolol & metformin. She is normally bed bound & fully dependant for all ADLs.

(a) List 4 investigations that are important in the initial care of this Pt.
Answer: BP, pulse, sats, ECG, CXR, Bl. gases (for electrolytes & Bl. sugar) & to see if acidotic. ECG is crucial.
(b) What does the ECG in figure 1 show?
Answer: Complete heart block. The ventricular pacing rate has taken over as there is complete dissociation between
the atria & the ventricles.
(c) What is occurring & how would you manage this Pt initially?
Answer: She is having syncopal episodes related to runs of asystole. She needs to be fully monitored using a
defibrillator. Try atropine in 500mcg increments to a max of 3mg.
(d) What are the adverse signs according to the resus council UK that you need to treat when considering
bradyarrhythmias?
Answer: Systolic <90, HR <40, Ventricular arrhythmias compromising BP, Heart failure
(e) Thinking of possible causes of the picture described in this Pt what potential reversible causes can be
identified from the history given?
Answer: Drugs! Digi-toxic or B-blocker overdose? Consider addressing these 2 issues need to check to digoxin level
also consider glucagon for reversing B-blocker effect. Electrolyte abnormalities are also potential reversible causes to
be considered ion complete heart block.
(f) The Pt's HR appears to drop to around 20 b.p.m & she continues to have runs of asystole associated with no
output. What will you do?
Answer: On the one hand the Pt needs to be paced urgently- this could be done by transcutaneous pacing until trans-
venous pacing can be established. However in the above Pt the entire picture needs to be considered. She is very
unlikely to do well in this scenario & there are significant risks involved with placing a trans-venous pacing wire. It
may be better to simply monitor the Pt & aim to keep her comfortable.
‫ـــــــــــــــــــــــــــــــــــــــــــــــــــــــــــــــــــــــــــــــــــــــــــــــــــــــــــــــــــــــــــــــــــــــــــــــــــــــــــــــــــــــــــــــــــــــــــــــــــــــــــــــــــــــــــــــــــــــــــــــــــــــــــــــــــــــــــــ‬
96- Question 8 of 10
A 19 yo student presented to the ED with a headache. He lived with 2 other students who found him after he failed to
answer a wake up call. O/E he was flushed & drowsy.There was a cherry red discoloration to his lips. He was afebrile,
he didn't have a skin rash. His HR was 95 b/m & his Bl. Pr. was 130/90 mmHg. His GCS was 11/15. There was no
nuchal rigidity. The CNS & PNS exam. were normal. His investigations revealed a normal CBC, renal profile & electrolyte
profile. His ABG  a pH in the normal range, a ↓ PaO2(7.8 kPa) & a ↓ PaCO2 (3.6 kPa). His SpO2 was 98% on room air
(a) What is the most likely diagnosis?
Answer: Carbon monoxide poisoning.
(b) Explain the arterial Bl. gas results?
Answer: Carbon monoxide displaces O2 from Hb.
(c) Explain why the pulse oximeter reading is normal?
37
Answer: Pulse oximeter analysers cannot differentiate between oxyHb & carboxyHb. PaO2 is low when there is
significant carbon monoxide poisoning.
(d) Name some common sources of this condition?(Name 2)
Answer: Combustion engines, faulty stoves, paraffin heaters with poor ventilation facilities.
(e) What is the TTT for this condition?
Answer: Administration of 100% O2. Pts with neurological signs & symptoms, ECG abnormalities, myocardial
ischaemia, pulmonary oedema & shock require hyperbaric O2 at a specialised centre.
‫ـــــــــــــــــــــــــــــــــــــــــــــــــــــــــــــــــــــــــــــــــــــــــــــــــــــــــــــــــــــــــــــــــــــــــــــــــــــــــــــــــــــــــــــــــــــــــــــــــــــــــــــــــــــــــــــــــــــــــــــــــــــــــــــــــــــــــــــ‬
97- Question id: 2084
This ECG is from a 76 yo man who presented with central chest pain & nausea.

Fig 1

Fig 2
(a) What does the ECG in figure 1 show?
Answer: Infero-posterior M.I., would accept inferior M.I., with lateral reciprocal changes.
(b) The Pt seems to deteriorate & a repeat ECG (fig 2) shows the following: Explain why this has occurred
referring to the anatomy of the coronary arteries.
Answer: When The Pt has suffered an occlusion of the Rt coronary artery (RCA) the infero-posterior ischaemic
changes in the first ECH demonstrate this.The RCA supplies the SA node, the AV node & the entire posterior surface
of the heart. They can therefore lead to dangerous arrhythmias.
(c) When faced with the ECG in (figure 1) what additional investigations would you like to perform?
Answer: Posterior leads. To do true posterior leads, here�s what you do: take all the chest lead wires off. Now stick
on three more chest electrodes along the same line of V5 & V6, along the fifth intercostal space, using the same
spacing that you used for the chest leads, ending up under the scapula: V7, V8, & V9. Now start reattaching the wires:
put the V1 lead wire on the V4 electrode. See? The V2 lead goes on the V5 electrode. & so on around the chest. Now
when you do your 12-lead, you�ll get a clear picture of what the entire RV is doing: inferiorly & posteriorly.
(d) Name 3 acute complications of STEMI
Answer: Continuing chest pain, fever, new systolic murmur (VSD, MR or Pericarditis), dysrrhythmia (VT, AV block
ectopics & bradycardia), cardiogenic shock.
‫ـــــــــــــــــــــــــــــــــــــــــــــــــــــــــــــــــــــــــــــــــــــــــــــــــــــــــــــــــــــــــــــــــــــــــــــــــــــــــــــــــــــــــــــــــــــــــــــــــــــــــــــــــــــــــــــــــــــــــــــــــــــــــــــــــــــــــــــ‬
98- Question id: 2115
A 76 yo gentleman presents with the ECG down, his HR is as shown & he is symptomatic.

(a) What is the rate?


Answer: Accept 35-39 b.p.m. 300/8 = 37.5, 8-8.5 large squares.
(b) What is the rhythm called?
Answer: Its 2ry degree heart block, mobitz type 2.
(c) What is your initial pharmacological management including dose?
38
Answer: Atropine 500 mcg or glycopyrolate 200-600 mcg.

(d) After the drug you gave there was no response. What do you do next?
Answer: Repeat atropine up to 3mg, rpt glycopyrolate as necessary.
(e) You have an external pacing device available, explain exactly what you would do & how it works to set it up
including anything you would do to the Pt.
Answer: Consider sedative & analgesia as can be uncomfortable, if clinical state will allow then give morphine &
midazolam (cautious in elderly) Explain to the Pt that will feel uncomfortable Apply sticky pads to the chest & to the
back (AP paddles) Select external demand pacing mode on the defibrillator & set the rate to 70 b.p.m Then start to
dial up the pacing current from zero until you see that a beat had been captured on the monitor. Clinically a capture
beat results in a peripheral pulse & an improvement in the Pts condition. Ensure that this occurs despite the monitor
showing a captured beat.
(f) When do ventricular pauses become concerning?
Answer: always of concern but generally if pauses are lasting > 3 seconds then something needs to be done sooner
rather than later.
‫ـــــــــــــــــــــــــــــــــــــــــــــــــــــــــــــــــــــــــــــــــــــــــــــــــــــــــــــــــــــــــــــــــــــــــــــــــــــــــــــــــــــــــــــــــــــــــــــــــــــــــــــــــــــــــــــــــــــــــــــــــــــــــــــــــــــــــــــ‬
99- Question id: 2111
A known alcoholic lady downs an entire bottle of antifreeze she wrote a suicide note & was found
unconscious by her neighbour
(a) Explain why it is toxic, what does it contain?
Answer: The substance is ethylene-glycol, it is toxic due to its metabolites.
(b) Give 2 antidotes & explain how they work?
Answer: Alcohol & fomepizole. They work by inhibiting the metabolism of ethylene glycol & hence preventing the
formation of metabolites.
(c) What are the presenting features assuming that you have no history of antifreeze ingestion?
Answer: Pt appears drunk (but doesn't smell of ETOH) Ataxia Dysarthria Nausea/vomiting Haematemesis
(d) In severe poisoning like the lady described above what is going to be the likely course of management &
where will she be managed?
Answer: On ITU Haemodyalysis Correction of acidosis
(e) What metabolic disturbance must one be especially vigilant for? & how is it treated?
Answer: Hypocalcaemia, which can be severe, treated with Ca gluconate.
‫ـــــــــــــــــــــــــــــــــــــــــــــــــــــــــــــــــــــــــــــــــــــــــــــــــــــــــــــــــــــــــــــــــــــــــــــــــــــــــــــــــــــــــــــــــــــــــــــــــــــــــــــــــــــــــــــــــــــــــــــــــــــــــــــــــــــــــــــ‬
100- Question id: 4549
A 68 yo gentleman is brought in by ambulance after being found collapsed at his home. He is covered in faeces &
appears very thin & unkempt. He has had some malaena. The paramedics do not know what happened today & had to
break into his house. The Pt's daughter called them, as she hadn't heard from him in 2 days. He is an alcoholic who
drinks a litre of vodka a day.

(a) O/E he had following observat.: A- own B- Clear sats 98% in air C- PR 120 BP 81/40 D- GCS 13 E- abd. soft -
maleana present on the sheets. You discover from the notes that he has had 2 previous GI bl. & on the last time he
refused an OGD & was treated as a presumed bleed. You instigate initial resuscitat.- list 8 things that you do?
Answer: 1- IV fluids 2- IV pabrinex 3- Check Bl. glucose 4- ECG 5- CXR 6- Bl. test including cultures 7- Bl. gas 8-
Speak to any family, try to gain as much pre morbid functional status as possible
(b) He stabilises a little & his Bl. Pr. improves. His GCS remains 13. Pending Bl. test results you speak to the
on-call endoscopist. What is the next most important investigation/intervention? Bl. glucose is 6.1
Answer: Need to work out why GCS is 13. Look for signs of trauma will likely need a CT head if hasn't improved
after initial resuscitation in the ED. ? has had a sub-dural etc
(c) Bl. tests come back as follows ALT- 112 GGT- 980 Bili- 73 ALKP- 442 Alb- 38 Lactate- 10.3 Na+ 149 K+ 4.8
Ur 3.2 Cr 172 Hb 12.6 Plts 263 WBC 12.3 Clotting normal Lipase- 5479 Amylase 332 How does this affect the
DD? What could be going on? Which tests results are most concerning?
Answer: Pt is clearly unwell with a lactate of 10.3, liver function is grossly deranged but renal function is not too far
abnormal- the urea is normal & the haemoglobin is also normal meaning that any GI Bl. loss is likely not to be the
39
most significant thing occurring here. The lactate is the most concerning Bl. test. Differential- diagnosis: pancreatitis?
Ischaemic gut? Alcoholic ketoacidosis? GI bleed with perforation? ?
(d) His CXR is normal as is the ECG. There is no sign of ascites & he is not septic, abdominal exam. is
unremarkable. Urinalysis reveals 4+ ketones. What is the diagnosis?
Answer: This case is alcoholic ketoacidosis
‫ـــــــــــــــــــــــــــــــــــــــــــــــــــــــــــــــــــــــــــــــــــــــــــــــــــــــــــــــــــــــــــــــــــــــــــــــــــــــــــــــــــــــــــــــــــــــــــــــــــــــــــــــــــــــــــــــــــــــ‬
101- Question id: 2097
A 69 yo male attends with a history of general malaise over the past 3 weeks. On further questioning he has had bony
pains in his back & in his ribs for several wks that he attributed to ?old age?. A CXR is normal. His vision has been a
bit ?blurry? over the last few days. Bl.s show: Hb 9.0, MCV 83 fL, MCH 29pg, MCHC 34g/dl WCC 8.4, Plts 334
Urea 35.6; Creat 587; Na 138; K 7.9 Ca 3.05; Alk P 220u/L

(a) Give two possible diagnoses


Answer: Multiple myeloma: Hyperparathyroidism (& renal failure) Vit D excess (sarcoidosis & thyrotoxicosis)
Hypercalcaemia of malignancy (expect elevated Alk P)
(b) Give 5 TTT options available (multiple TTTs for the same abnormality not accepted)
Answer: Re-hydraton is fairly crucial, Rx hyperkalaemia: nebs, Insulin/Glc. Not Ca (?) ARF: Dialysis,
haemofiltration? Hypercalcaemia: Fluids & steroids- hydrocortisone Pamidronate (malignancy) if the hypercalcaemia
persists: For myeloma also consider prednislone 30-60mg Calcitonin
(c) How could you confirm the diagnosis?
Answer: BJ protein, serum/urine electrophoresis, bone marrow,
‫ـــــــــــــــــــــــــــــــــــــــــــــــــــــــــــــــــــــــــــــــــــــــــــــــــــــــــــــــــــــــــــــــــــــــــــــــــــــــــــــــــــــــــــــــــــــــــــــــــــــــــــــــــــــــــــــــــــــــ‬
102- Question id: 4510
A 65 yo male presented to the ED with a 3 hr history of chest pain, palpitations & breathlessness. His P/H was of
HTN & a MI His regular medicat. include aspirin 75mg , frusemide 40mg & atorvastatin 20mg.

(a) What is shown on the ECG?


Answer: Ventricular Tachycardia
(b) List the ECG features of this condition(List four)
Answer: Broad complex QRS, Extreme axis deviation, positive or negative concordance in the precordial leads, RSr
pattern in V1, Deep S-wave in V6, Fusion & Capture beats, Dissociated p-waves
(c) What drug may be used to treat this condition?
Answer: Amiodarone
(d) Shortly after administering the drug the Pt becomes clammy & cyanosed. His conscious level deteriorates &
his Bl. Pr. is low. What is the next step in management?
Answer: DC Cardioversion
(e) From which ventricle does the above rhythm usually arise?
Answer: Left ventricle.
‫ـــــــــــــــــــــــــــــــــــــــــــــــــــــــــــــــــــــــــــــــــــــــــــــــــــــــــــــــــــــــــــــــــــــــــــــــــــــــــــــــــــــــــــــــــــــــــــــــــــــــــــــــــــــــــــــــــــــــ‬
103- Question id: 2019
A 64 yo lady is brought in vomiting she has had haematemesis for the last 2 hrs & has just passed a large volume of PR Bl.
(a) What anatomical point differentiates an upper from a lower gastronintestinal bleed?
Answer: Ligament of Treitz it inserts as nonstriated muscle commonly into the third & fourth portions of the
duodenum & frequently into the duodenojejunal flexure as well.
40
(b) Is it safe to assume that the above Pt is having an upper or a lower gastrointestinal bleed (LGIB)?
Answer: It is unclear. You need to know if the Bl. is classical malaena or fresh Bl. but be warned LGIB can present
with fresh PR Bl. loss. 15% of LGIB present as UGIB.
(c) List 5 potential causes of an upper GI bleed.
Answer: ? Ulceration stomach or duodenum ? Inflammation: oesophagitis/gastritis/duodenitis ? Mallory Weiss tear ?
Warfarin or clotting disorders ? Gastric or oesophageal malignancy ? Oesophageal varices
(d) Name a scoring system for risk stratifying upper GI bleeds & list 4 criteria that it focuses on.
Answer: The Rockall score 1

(e) What is the commonest type of LGIB?


Answer: UGIB followed by diverticular bleeds.
(f) The Pts Hb comes back at 4.4 how many units of Bl.s will you give?
Answer: The aim of transfusion should be to minimize risk whilst improving the clinical situation. Therefore the
smallest volume of Bl. should be given aiming for a Hb> 7. 3-4 units initially but if ongoing bleeding then more Bl.
will be needed as will clotting factors & potentially platelets.
(g) What methods can be used to control a variceal UGIB prior to endoscopy?
Answer: Telipressin 2mg IV 4-6 hrly & possible insertion of a Sengstaken/Minnesota tube. 1.
‫ـــــــــــــــــــــــــــــــــــــــــــــــــــــــــــــــــــــــــــــــــــــــــــــــــــــــــــــــــــــــــــــــــــــــــــــــــــــــــــــــــــــــــــــــــــــــــــــــــــــــــــــــــــــــــــــــــــــــ‬
104- Question id: 2153
You see a 94 yo nursing home resident who has been brought in as she is confused. The nurse with her states that she
is not normally confused.
(a) List 3 simple ED tests that are crucial in this case.
Answer: Urinalysis Baseline observations Temperature BM AMMT Breath alcohol
(b) You take a very detailed history & perform a through exam.. All base line observations are normal apart
from a temp of 38.3?C. The nurse tells you that she had been cold this morning & they had out on the gas fire
in her room for the first time this yr. Would you do any further tests in light of the above information?
Answer: Need to perform an ABG for CO. Possible that she could have carbon monoxide poisoning.
(c) What other investigations would you like?
Answer: CXR, ECG, FBC, U&E Bl. glucose.
(d) The urinalysis is grossly positive & on repeat questioning the nurse had noticed that her urine had been
rather pungent over the last 48 hrs. What will you do now?
Answer: Pt needs admission Bl.s cultures Send MSU to the lab Empirically treat likely with oral antibiotics initially if
Pt can swallow them. Trimethoprim 200mg/BD
(e) Name 2 pathogens that commonly cause UTIs
Answer: E coli spec Enterococcus faecalis Klebsiella pneumoniae Proteus mirabilis Bacteriodes Pseudomonas aeruginosa
‫ـــــــــــــــــــــــــــــــــــــــــــــــــــــــــــــــــــــــــــــــــــــــــــــــــــــــــــــــــــــــــــــــــــــــــــــــــــــــــــــــــــــــــــــــــــــــــــــــــــــــــــــــــــــــــــــــــــــــ‬
105- Question id: 2120
A 71 yo lady presents "off legs" she has been in a nursing home for the last few months as she could no longer cope at
home due to her metastatic breast malignancy. She appears dehydrated & a little confused. The nursing home staff
state that she has mobile yesterday, they also tell you that she was doubly incontinent today which is unusual for her.

(a) If you could only perform 2 aspects of clinical exam. in this case to ascertain the main problem which 2
would you chose? (e.g. cardiovascular exam & exam. of the fundi)
Answer: A PR (to check for anal tone & sensation) A complete lower limb neurological exam.. Looking for evidence
of spinal chord compression.
41
(b) What investigation do you try to organise?
Answer: MRI to image the spinal chord
(c) Which Bl. tests are you especially interested in?
Answer: U&E & Ca are of particular interest, hypercalcaemia is a very common cause of confusion in these Pts.
(d) How will the primary problem described in a) be managed?
Answer: Normally radiotherapy but sometimes it may be appropriate for no TTT to occur & analgesia might be the
mainstay of TTT.
‫ـــــــــــــــــــــــــــــــــــــــــــــــــــــــــــــــــــــــــــــــــــــــــــــــــــــــــــــــــــــــــــــــــــــــــــــــــــــــــــــــــــــــــــــــــــــــــــــــــــــــــــــــــــــــــــــــــــــــ‬
106- Question id: 4525
A 57 Yo with known COPD comes in- she appears to be having an exacerbat, her initial observat. are as follows- a-
talking in broken sentences but drowsy, B- sats 83% widespread wheeze poor AE, RR 43, c- PR 115, d GCS 14 (drowsy)

(a) In general who should receive O2?


Answer: Only hypoxaemic pts should get O2 therapy- there is no evidence at all that O2 benefits pts who are
dyspnoeic but not hypoxic. There is increasing evidence that too much O2 may actually do harm.
(b) What are the first & second things that you should do?
Answer: Firstly: ensure that the pt is on O2- BTS guidelines (see reference) would advice checking for an O2-
warning card (i.e. to find out if known to retain & are they on home O2 etc, in a perfect world they would come in
with their own venturi!), This pt would be treated in accordance with table 1 in the guideline as she is crtically ill so
would be put on a non-rebreathe mask pending gases then maintained at 94-98% sats if normal CO2; if CO2  then
aim for 88-92%. Then on this basis go for either 28% via venturi or 24% via venturi. Of course if this doesn't work
(aiming for sats 88-92%) then put on higher flow O2/consider NIV etc. Also note that the guidelines suggest  the
flow to 50% above the minimum flow rate suggested on the venturi if the pts R is >30, The 2nd step needs to be an ABG.
(c) What is now encouraged for the crews to be doing pre-hospital?
Answer: ? Answer to start these pts on 28% O2 via venturi!?
(d) What should happen if sats go above 92%?
Answer: O2 concentration should be turned down?
(e) What are the 2 sats ranges to aim for hypoxic Pts suggested in the BTS guideline on emergency O2 use?
Answer: 88-92% for known COPD, or others at risk of hypercapnic respiratory failure (pending ABG result) Adjust
to the higher range if the pCO2 is normal & no history of hypercapnic respiratory failure. 94-98% for anyone else. ?
(f) How should nebulised therapy be given for the above Pt?
Answer: Should be driven in air, any O2 required should be given via nasal cannulae. If this is not available then
drive by O2 but limit this to 6 minutes.
‫ــــــــــــــــــــــــــــــــــــــــــــــــــــــــــــــــــــــــــــــــــــــــــــــــــــــــــــــــــــــــــــــــــــــــــــــــــــــــــــــــــــــــــــــــــــــــــــــــــــــــــــــــــــــــــــ‬
107- Question id: 2177
A 46 yo woman with bipolar disorder presented as she had mistakenly taken too many of her lithium tablets. Her
previous medicat. had been discontinued & she had been started on lithium the previous wk. She was taking the lithium
tablets according to her previous medication?s dosing regime. This had resulted in her taking excess lithium tablets
over the course of the week , a fact which she had only discovered on the day of presentation.

(a) What are the clinical features of lithium poisoning?


Answer: Nausea, vomiting, diarrhoea, tremor, ataxia, confusion,  in muscle tone, clonus, convulsions, coma & renal failure.
(b) What are the two most important Bl. investiagtions?
Answer: UE & lithium level.
(c) What is the normal range for therapeutic lithium levels?
Answer: <1.2mmol/L
(d) What is the role of activated charcoal in this Pt's TTT?
Answer: Activated charcoal does not absorb lithium.
(e) How would you manage this Pt?(Give four)
Answer: Gastric lavage is indicated if within 1 hr of a single large overdose,consider poisons consultation, control
convulsions with benzodiazepines, haemodialysis for severe poisoning.
‫ــــــــــــــــــــــــــــــــــــــــــــــــــــــــــــــــــــــــــــــــــــــــــــــــــــــــــــــــــــــــــــــــــــــــــــــــــــــــــــــــــــــــــــــــــــــــــــــــــــــــــــــــــــــــــــ‬
108- Question id: 4553
A 27 yo Turkish man presents to the ED at the weekend sent in from the walk in centre, he has painful legs & has
noticed a rash. He lives in a poor area of the inner city.
(a) What is the rash shown in figure 1?
Answer: Erythema nodusom?
(b) List 5 causes of this rash?
Answer: 1. Crohn’s/colitis 2. TB 3. HIV 4. Drug induced 5. St. infect. (beta haemolytic) 6. Sarcoidosis 7. Leprosy &
other infections( Yersina, toxoplasmosis, histomplasmosis, Chlamidya) 8. SLE 9. Behcet's disease ?
42
(c) What important questions do you ask in the history to try to help you with the diagnosis? Limit the answer
to the 4 most important questions. (Remembering that common things are common)
Answer: ? Ask about bowel habit/abdo pain & rectal bleeding/ features of IBD ? Ask about recent travel history &
possible TB contact ? Take a complete drug history ? Ask about recent infection (anything to suggest strep sore throat
etc) This probably covers the most common causes of erythema nodusum ?

Fig 1
(d) What base line investigations would be useful & why?
Answer: CXR: look for evidence of TB & sarcoid, FBC- looking for anaemia (IBD), ESR & CRP looking for
inflammation (vadculitis/IBD), ASO titer, Urinalysis, Throat culture, Intradermal tuberculin test
(e) He is s smoker & tells you that he has a cough from time to time. From your screening questions that you
chose above you decide that he doesn't seem to have any of the risk factors for common causes of this type of
rash. You decide to investigate further. You find out that he has had some urthethritis but denies sexual
intercourse in the last 6 months he has also noticed that he has intermittently painful joints. You notice some
mouth ulcers O/E. What could the diagnosis be?
Answer: Behcet's disease?
‫ــــــــــــــــــــــــــــــــــــــــــــــــــــــــــــــــــــــــــــــــــــــــــــــــــــــــــــــــــــــــــــــــــــــــــــــــــــــــــــــــــــــــــــــــــــــــــــــــــــــــــــــــــــــــــــ‬
109- Question id: 4562
A 56 yo manager comes in after experiencing some palpitations. He tells you that he has experienced palpitations off
& on for a No. of yrs but has never worried about them. Today he felt as if they lasted longer than previous episodes.
He is found to be in atrial fibrillation with a rate of 76 b.p.m

(a) What information is useful to know about this gentleman?


Answer: Does he have any structural HD (ideally has he had an ECHO), Is he on any medicat. that could precipitate AF, Does he
have a possible driving factor for AF i.e. infect./ETOH/thyroid disease etc, Are there any old ECG tracings showing NSR?
(b) According to NICE guidance what will determine if the Pt needs to be anti-coag. prior to attempting rhythm control?
Answer: The duration of the AF. If it is less than 48 hrs then no need for anticoagulation. Most parties would give
LMWH to Pts who they thought might be cardioverted acutely.
(c) What drug therapies are available in the above scenario?
Answer: Flecanide would be a good option provided that he doesn't have any structural heart disease. If he does then
amiodarone would be the preferred agent.
(d) Before you decide what you are going to do the Pt becomes unresponsive & drops his BP to 76/43. The
anaesthetist is unavailable for the next 30 minutes as there is a trauma in the next bay what do you do?
Answer: Options here are variable of course- could give amiodarone IV to attempt to revert the arrhythmia. Ideally
needs electrical cardioversion.
(e) List 4 scenarios where rhythm control is felt to be superior to rate control when faced with a Pt in AF.
Answer: Age, < 65 aim for rhythm control, When the Pt is very symptomatic, 1st presentation of lone AF, When the
AF is 2ry to a treated or corrected precipitant, With congestive cardiac failure.
‫ــــــــــــــــــــــــــــــــــــــــــــــــــــــــــــــــــــــــــــــــــــــــــــــــــــــــــــــــــــــــــــــــــــــــــــــــــــــــــــــــــــــــــــــــــــــــــــــــــــــــــــــــــــــــــــ‬
110- Question id: 4560
An 87 yo lady is brought in from a nursing home after having had a few episodes of collapse over the last wk. She has
clearly had a fall as she has a bruised face. She seems to be well on initial assessment. She is on a plethora of medicat.

(a) List the 4 most important bits of history you want from this lady
Answer: 1. Medicat.- particularly warfarin/anticoagulants & any medications that cause bradycardia/hypotension etc
2. Normal functional status i.e. is she independent etc 3. What is her mental status today & what is normal for her. 4.
Any pre-syncopal features, i.e. is she aware that she is going to collapse
43
(b) What investigations do you want immediately in the ED?
Answer: 1. ECG 2. Bl. glucose 3. Postural BP recordings 4. routine Bl. tests, FBC,UE, Ca.
(c) ECG revealed: see figure 1: What does it show?
Answer: Mobitz type 2- this is mobitz 2 with 3:1 block. Mobitz Type 2 2nd degree Heart Block is considered an
important warning signal of the potential progression to 3rd degree Heart Block, which requires prompt attention.
(d) What will you do about it?
Answer: Depends if the pt is stable or unstable- If stable then can prepare for a pacemaker at the next available
opportunity If unstable then requires a temporary pacing wire to be inserted.
(e) Her heart rate drops to 38 b.p.m, what measures do you take?
Answer: Measure the BP & re-assess the Pt, if unstable then may need to instigate immediate pacing- could use
transcutaneous pacing If BP is relatively maintained could consider giving atropine (best titrated in this scenario)
‫ــــــــــــــــــــــــــــــــــــــــــــــــــــــــــــــــــــــــــــــــــــــــــــــــــــــــــــــــــــــــــــــــــــــــــــــــــــــــــــــــــــــــــــــــــــــــــــــــــــــــــــــــــــــــــــ‬
111- Question id: 4546
A 26 yo man attends the ED with haemoptysis & SOB- he appears very unwell & has had large amounts of haemoptysis for
the last hr. He says that he has felt sick & has had aching joints for the last 2 days he also mentions that he hasn’t been to
the toilet today. You do an ABG & order a CXR

Fig 1
(a) Gas on 15 litres of O2 pH 7.29 pO2 9.7 pCO2 4.3 HCO3 16 BE -5.3 Describe the Bl. gas picture shown
Describe the CXR (figure 1) & state 4 potential causes of the appearances.
Answer: The x-ray shows bilateral airspace shadowing which in this case is Bl. from pulmonary haemorrhage- but it
could be fluid (pulmonary oedema), lymph or consolidation from infection. The gas shows a metabolic acidosis.
(b) What important investigations do you want to do immediately?
Answer: Clotting, renal function, complete biochemical profile, CBC, vasculitic screen- i.e. ANA, ANCA etc
(c) Some of his initial results come back- Hb 7.3 Plt 98 WBC 10.9 Na 134 K+ 6.1 Ur 25.6 Cr 435 What is the
most likely cause of the Bl. results above?
Answer: Most likely renal failure due to ATN as part of the vasculitic illness that is underlying this presentation.�
(d) What important step needs to be taken in light of the Bl. results?
Answer: ECG- look for signs of potassium toxicity i.e. tented T waves, widening of the QRS complex, slurred ST
segments, arrythmias, can lead to VF. Then if present treat with Ca gluconate to protect the myocardium.
(e) His initial observations are as follows: Pulse 100, BP 120/67, sats 94% in O2, RR 26, temp 37.4, GCS 15
Considering the Bl. gas & x-ray findings what is the DD?
Answer: Vasculitis- Goodpasture's, PAN, Wegnener's, microscopic polyangititis, Churg-Struass.
(f) The Pt is ANCA negative what is the most likely diagnosis?
Answer: Churg-Strauss
‫ــــــــــــــــــــــــــــــــــــــــــــــــــــــــــــــــــــــــــــــــــــــــــــــــــــــــــــــــــــــــــــــــــــــــــــــــــــــــــــــــــــــــــــــــــــــــــــــــــــــــــــــــــــــــــــ‬
112- Question id: 2101
A 54 yo lady with bipolar affective disorder tells you that she took a months worth of her lithium tablets you have the
boxes that she has brought in (they are slow release tablets)

(a) Would you consider using activated charc. for this lady as she has presented within an hr of having taken the tablets?
Answer: No as it doesn't absorb lithium
(b) Could you perform gastric lavage?
Answer: No as the slow release tablets are too large to pass up the nasogastric tube.
(c) What do you do?
44
Answer: In contact with a poison's specialist could consider whole bowel irrigation when slow release tablets have been taken.
(d) What are the symptoms of lithium overdose?
Answer: Nausea, vomiting, diarrhoea, are followed by tremor, ataxia & confusion. In severe cases there may be renal
failure, convulsions & coma.
(e) How would you control seizures if they occurred?
Answer: benzodiazepines: lorazepam, diazepam
‫ــــــــــــــــــــــــــــــــــــــــــــــــــــــــــــــــــــــــــــــــــــــــــــــــــــــــــــــــــــــــــــــــــــــــــــــــــــــــــــــــــــــــــــــــــــــــــــــــــــــــــــــــــــــــــــ‬
113- Question id: 4533
A 31 yo business man developed a sudden onset of sore throat, fever, diarrhoea & lethargy. He developed a rash over the
next few days affecting the face/trunk/palms & soles. He had been in Singapore 2 months previously. O/E he had cervical
lymphadenopahy a widespread rash, temp 38.4 & an erythematous pharynx. He was also c/o a non-productive cough.

(a) List some DD:


Answer: HIV seroconversion, EBV, MV infection, Acute hepatitis, TB.
(b) If he had glandular fever what would the most likely cause of the rash be?
Answer: Amoxicillin administration.
(c) What is common cause of diarrhoea in a Pt infected with HIV?
Answer: Cryptosporidium (supportive TTT)
(d) A CXR taken in the above Pt showed bilateral diffuse interstitial shadowing. What is the likely diagnosis?
Answer: HIV/AIDS- pneumocystis jiroveci pneumonia (formerly PCP)
(e) What is the TTT?
Answer: IV co-trimoxazole
‫ــــــــــــــــــــــــــــــــــــــــــــــــــــــــــــــــــــــــــــــــــــــــــــــــــــــــــــــــــــــــــــــــــــــــــــــــــــــــــــــــــــــــــــــــــــــــــــــــــــــــــــــــــــــــــــ‬
114- Question id: 2052
A 68 yo man presented with severe Rt sided eye pain which had come on over the course of two hrs. He had no
previous history of eye disease.A diagnosis of acute angle closure glaucoma has made.

(a) What are the risk factors for this condition?(Name four)
Answer: Family history of angle closure, age older than 40 to 50 yrs, female, history of symptoms suggesting angle-
closure, hyperopia (farsightedness), pseudoexfoliation (a condition in which abnormal flaky deposits on eye surfaces
can weaken the zonules that support the lens & cause it to shift forward) & race(the highest rates of angle closure are
reported in Asian populations).
(b) Besides pain what are the other symptoms a Pt may complain of?(Name four)
Answer: Decreased vision, halos around lights, headache, severe eye pain,nausea & vomiting.
(c) What signs may be found on exam?(Name four)
Answer: Conjunctival redness, corneal edema or cloudiness,a shallow anterior chamber & a mid-dilated pupil (4 to 6
mm) that reacts poorly to light.
(d) What time of the day is this condition most likely to occur?
Answer: Signs & symptoms of acute glaucoma often occur in the evening, when lower light levels cause mydriasis,
& folds of the peripheral iris block the narrow angle.
(e) What are the management steps in the ED?
Answer: Name two eye drops which may be of benefit?
‫ــــــــــــــــــــــــــــــــــــــــــــــــــــــــــــــــــــــــــــــــــــــــــــــــــــــــــــــــــــــــــــــــــــــــــــــــــــــــــــــــــــــــــــــــــــــــــــــــــــــــــــــــــــــــــــ‬
115- Question id: 2103

Fig 1
(a) What type of M.I. is shown in the ECG (fig 1), describe what is shown.
45
Answer: Lateral M.I. ST elevation in leads 1 & aVL can't see elevation in 2 & v6 but you don't always get a complete
set. Also note the inferior reciprocal changes 2,3 & aVF.
(b) Which coronary vessel is likely to be occluded?
Answer: Left circumflex. When the picture shows antero-lateral changes i.e. ST elevation in all the precordial &
lateral leads the occlusion is higher up in the left coronary artery before it splits into the LAD & LCx.
(c) You consider thrombolysis for this Pt, what 5 medications have you already given?
Answer: O2, morphine, aspirin, clopidogrel, LMWH.
(d) Name 2 agents that you could use for thrombolysis & describe how they are given.
Answer: Streptokinase give 1.5 mega units in a continuous infusion over 1 hr. Alteplase: 15mg bolus followed by
0.75mg/kg (max 50mg) IVI for 30 mins, then 0.5mg/kg (max 35mg) over 60 mins. Give heparin or s/c LMWH.
Reteplase: 2 IV boluses of 10 units each 30 mins apart (give heparin as above). Tenecteplase: single IV bolus over 10
seconds, Dose according to weight (also give heparin as above)
(e) After you give thrombolysis the Pt seems to still be in pain & after 30 minutes there is no resolution of the
ST segments. What will you do?
Answer: Will need to transfer to an interventional centre for rescue PCI
‫ــــــــــــــــــــــــــــــــــــــــــــــــــــــــــــــــــــــــــــــــــــــــــــــــــــــــــــــــــــــــــــــــــــــــــــــــــــــــــــــــــــــــــــــــــــــــــــــــــــــــــــــــــــــــــــ‬
116- Question id: 4537
A 71 yo man presents with central crushing chest pain. An ECG shows ST elevation in leads V1-V4. He receives
thrombolysis. 3 hrs later his ECG shows (see fig 1).

Fig 1
(a) What does the ECG show?
Answer: Second degree heart block- Mobitz type II.
(b) What size does the ST elevation need to be in the chest leads for thrombolysis?
Answer: Answer: >2 mm. In 2 anatomically contiguous leads (>1mm in limb leads, >2mm in V leads)�
(c) What has occurred with the above Pt?
Answer: Pt has had an anterior MI which has led to Mobitz type II which could lead to complete heart block.�
(d) With regard to the changes seen in figure 1 what does this Pt need, please chose the best option? 1.
temporary venous pacing wire 2. Atropine 3. Angiography 4. No TTT 5. temporary transcutaneous pacing
Answer: Answer: 1tempory venous pacing wire - Mobitz type 2 in this setting is very dangerous; the rhythm could
quickly turn into complete heart block.
(e) List the reasons that one would need to instigate urgent pacing after an M.I.
Answer: Complete HB, Asystole, Symptomatic bradycardia or Mobitz type 1 that isn't responding to atropine, New
BBB with 1st degree heart block, Old RBBB with 1st degree AV block & a new fasicular block.
‫ــــــــــــــــــــــــــــــــــــــــــــــــــــــــــــــــــــــــــــــــــــــــــــــــــــــــــــــــــــــــــــــــــــــــــــــــــــــــــــــــــــــــــــــــــــــــــــــــــــــــــــــــــــــــــــ‬
117- Question id: 4541
A 68 yrs old man presents with central chest pain. ECG shows an anterior STEMI.

(a) What is the best TTT?


Answer: Primary coronary angioplasty, lower mortality & lower re-infarction rates.
(b) What is the advantage of tenectaplase & reteplase as thrombolytic agents?
Answer: ease of administration as they are given as single bolus doses.
(c) What is the risk of major bleeding with thrombolysis?
Answer: 2-3%
(d) List 4 absolute contraindications to thrombolysis:
Answer: Active internal bleeding, suspected aortic dissection, recent head trauma/intracranial tumour, haemorrhagic
stroke at any time, ischaemic stroke within the last yr, previous allergic reaction to fibrinolytic agent, trauma or
surgery within the last 2 weeks at risk of bleeding.
(e) What drug is give along side thrombolytic agents?
Answer: IV heparin
‫ــــــــــــــــــــــــــــــــــــــــــــــــــــــــــــــــــــــــــــــــــــــــــــــــــــــــــــــــــــــــــــــــــــــــــــــــــــــــــــــــــــــــــــــــــــــــــــــــــــــــــــــــــــــــــــ‬
118- Question id: 4547
A 62 yo obese lady attends complaining of difficulty in getting her breath- she has insp. stridor & a grossly swollen tongue

(a) She is on some medication for HTN, angina & asthma. Which of the following medications most likely
caused this presentation?: Nicorandil, asprin, ramipril, simvastatin, monteleukast
46
Answer: Ramirpil- well described angioedematous reaction can occur yrs after stating an ACEi. Also can occur with
angiotensin 2 receptor blockers.
(b) How would you treat the Pt?
Answer: Ensure that the airway is protected- consider ENT & anaesthetic input if required- nurse in an area where pt
is monitored closely, be alert to any changes in pt condition. O2, consider IM adrenaline, chlorpenamine,
hydrocortisone. (treat as for anaphylaxis) These Pts need 24 hrs in hospital as there have been reported cases of
airway obstruction after early discharge.
(c) The Pt doesn't appear to improve & also complains of some abdominal pain. O/E you notice that she has
prominent cervical lyphadenopathy. What could explain her lack of improvement?
Answer: This could be acquired C1 esterase deficiency- seen in lymphoma. C1 estersae deficiency can be congenital
or acquired It can be treated with synthetic preparations of C1 esterase.
(d) When you look through her Bl. tests you note that the GP did a recent fasting glucose which came back as
8.4. What does this mean & which of the following should be instigated?: Rosiglitazone, metformin, insulin,
diet modification, gliclazide, pioglitazone.
Answer: She is diabetic & obese- likely to need drug therapy but start with diet modification & then first option
would be metformin as obese. Can't have glitazones as has IHD.
‫ــــــــــــــــــــــــــــــــــــــــــــــــــــــــــــــــــــــــــــــــــــــــــــــــــــــــــــــــــــــــــــــــــــــــــــــــــــــــــــــــــــــــــــــــــــــــــــــــــــــــــــــــــــــــــــ‬
119- Question id: 2131
A 49 yo man presents to the ED with an acutely painful left knee. The knee is swollen & painful. He felt fine 3 hrs
ago. His only medical history is of mild HTN for which he takes bendroflumethiazide. There is no history of trauma

(a) Give your top 3 DD:


Answer: You need to consider the DD for an acute monoarthritis, you must consider a septic joint until proven
otherwise. (must include 1&2 to score marks) 1. Septic arthritis 2. Acute gout/other crystal arthropathy 3. Ist
presentation of RA, SLE, seroneagtive arthritis (psoriatic/Rieter's) 4. Tramuatic causes: traumatic synovitis,
haemarhtrosis, fracture, ruptured ACL.
(b) Give 4 things can precipitate acute gout:
Answer: Diet, ETOH, diuretics, renal failure (ask about renal stones), trauma, cytotoxic drugs, myeloproliferative
disease, high cell turn over states i.e. lymphoma, psoriasis, Polycythaemia.
(c) What do you do?
Answer: The joint needs to be aspirated & the fluid sent for gram stain & culture but also for polarized light
microscopy to look for crystals. Also need to do Bl. cultures & send samples for WBC/CRP etc.
(d) List 2 contraindications to your suggestion in c)
Answer: Presumably answered aspirate the joint in c) 2 contraindications would be overlying sepsis i.e.
cellulitis/abscess, & bleeding diathesis.
(e) How would you treat septic arthritis?
Answer: Admit to orthopaedics Cultures as above Aspirate the joint to dryness check that there are not also crystals
as can be present in septic arthritis IV antibiotics flucloxacillin & benzypenicilin, in children use cefotaxime to cover
H. influenzae Strict bed rest Analgesics NSAIDS
‫ــــــــــــــــــــــــــــــــــــــــــــــــــــــــــــــــــــــــــــــــــــــــــــــــــــــــــــــــــــــــــــــــــــــــــــــــــــــــــــــــــــــــــــــــــــــــــــــــــــــــــــــــــــــــــــ‬
120- Question id: 2036
A 45 yo man presented with cough & SOB. O/E he was febrile & had bronchial breathing in the Rt lower zone.

Fig 1
(a) What is the condition shown in the radiograph?
Answer: Pneumonia.
47
(b) Name 3 common microbes which cause this condition?
Answer: St. pneumoniae, resp. viruses, mycoplasma pneumoniae, chlamydia pneumoniae&haemophilus influenzae.
(c) Name four risk factors?
Answer: Alcoholism, COPD, smoking, structural lung disease, aspiration, lung abscess, HIV infection, age &
exposure to birds droppings.
(d) What is an initial appropriate anti-biotic regime for a Pt not admitted to ICU?
Answer: Combination therapy with ceftriaxone (1 to 2 g IV daily) or cefotaxime (1 to 2 g IV every 8 hrs) plus
azithromycin (500 mg IV or orally daily). Alternatively monotherapy with a respiratory fluoroquinolone given either
IV or orally except as noted (levofloxacin 750 mg daily or moxifloxacin 400 mg daily or gemifloxacin 320 mg daily
[only available in oral formulation]).
(e) How long after discharge should the radiographic abnormalities be resolved?
Answer: CXR at 7 to 12 weeks after TTT is recommended for selected Pts who are over age 40 yrs or are smokers to
document resolution of the pneumonia & exclude underlying diseases, such as malignancy.
‫ــــــــــــــــــــــــــــــــــــــــــــــــــــــــــــــــــــــــــــــــــــــــــــــــــــــــــــــــــــــــــــــــــــــــــــــــــــــــــــــــــــــــــــــــــــــــــــــــــــــــــــــــــــــــــــ‬
121- Question 1 of 20
A 10 month old boy was sent to the ED by his GP with a 1st episode of wheezing. A diagnosis of bronchiolitis was made

(a) Give a differential diagnosis of 4 conditions?


Answer: Viral-triggered asthma or wheezing, pneumonia, chronic lung disease, foreign body aspiration, GERD &/or
dysphagia leading to aspiration, congenital heart disease, heart failure, & vascular rings.
(b) What is the most common etiological agent to cause this condition?
Answer: Respiratory syncytial virus (RSV) is the most common cause.
(c) Name 4 risk factors for this condition?
Answer: Prematurity (gestat. age <37 wks), low birth weight, age<6 to 12 wks, chronic pulm. disease (bronchopulmonary
dysplasia, cystic fibrosis, congenital anomaly), hemodynamically significant congenital HD (eg, moderate to severe
pulm. HTN, cyanotic HD or congenital HD that requires medicat. to control heart failure), immunodeficiency,
neurologic disease & congenital or anatomical defects of the airways. Having older siblings, concurrent birth siblings,
passive smoke, household crowding, child care attendance & high altitude.
d)Name 3 factors associated with increase illness severity?
Answer: Toxic or ill appearance, O2 saturation <95% by pulse oximetry while breathing room air, age younger than 3
months, RR ≥70 breaths per minute & atelectasis on chest radiograph.
(e) How would you manage this child?(Name 3)
Answer: Supportive measures to ensure that the child is clinically stable, well hydrated & well oxygenated. Consider
a trial of inhaled bronchodilators.
‫ــــــــــــــــــــــــــــــــــــــــــــــــــــــــــــــــــــــــــــــــــــــــــــــــــــــــــــــــــــــــــــــــــــــــــــــــــــــــــــــــــــــــــــــــــــــــــــــــــــــــــــــــــــــــــــ‬
122- Question 2 of 20
A 6-yr-old boy brought in acutely SOB. The ambulance crew let you know that he is known to have asthma & that he
has been very wheezy over the course of the last 6 hrs. They have given him 1 dose of nebulised salbutamol.

(a) What important features do you want to elicit to differentiate moderate severe asthma from life-threatening
asthma? Please state 3 things that would guide your decision. (3 marks)
Answer: Any 3: Cyanosis, PEFR<33%, Silent chest, Agitation or reduced GCS, Exhaustion
(b) Why is heart rate sometimes not a useful guide of severity?
Answer: Tachycardia produced by salbutmaol
(c) The initial observations show that he has moderate-severe asthma. The sats are 90% in air what is your
initial management please include drug doses.
Answer: Give high flow O2, Nebulised salbutamol 5mg (driven by O2)/as sats<92% if were above could give inhaled
salbutamol via a spacer. Prednisolone 40mg (as over 5 yrs)
(d) The child deteriorates & his sats drop to 86% his RR is now 65 & he appears to be tiring. What do you do know?
Answer: (5 marks drop marks for incorrect dosing), Continue with back to back neds driven by high flow O2, Give
nebulised ipratropium 500 mcg via neb, Give IV salbutamol loading dose of 15mcg/kg, IV aminophylline 5mg/kg
over 20 mins then loading dose then maintenance of 500mcg/kg/hr (if already on theophylline omit loading dose),
Importantly call anaesthetist & set up kit for tracheal intubation, alert PICU
(e) Discuss the role of Mg sulphate in the management of life threatening asthma in children.
Answer: No marks for mentioning that used in adults.(2 marks 1 for stating that it might be of benefit another for
stating that it is still undergoing trials) Answer: IV Mg does work & there is good evidence for it. Nebulised Mg may
work but there are a No. of ongoing clinical trials. So you can give it but ongoing research is needed for nebulised
route. Despite a suggestion of benefit in the sub-group of Pts with acute severe asthma this TTT isn't advocated at this
time by the current BTS/SIGN national asthma guidelines (2004). It is mentioned in the most recent edition of the
BNF as an unlicensed indication for Pts with acute severe asthma.
48
123- Question 3 of 20
A 6 yo boy is brought to the ED by his parents as he was drowsy & poorly communicative. O/E his heart rate was 40
& his extremities appeared poorly perfused.

(a) Name 2 basic initial management steps with this child?


Answer: O2, Attach monitor/defibrillator, support ABC's.
(b) If despite the above steps the child is still bradycardic with poor perfusion what is the next manag. step?
Answer: Perform CPR if depite oxygenation & ventilation HR <60/min with poor perfusion.
(c) If the bradycardia is persistent & symptomatic what medication is indicated? (Assume the child does not
have increased vagal tone or 1ry AV block)
Answer: Adrenaline(IV/IO) 0.01mg/kg(1:10000; 0.1mL/kg) or 0.1mg/kg (1:1000: 0.1mL/kg) via ETT. This can be
repeated every 3-5 minutes.
(d) If the bradycardia is persistent & symptomatic & the child has  vagal tone or 1ry AV block what medicat. is
indicated?
Answer: Atropine 0.02mg/kg & may be repeated.
(e) What other TTT modality should be considered?
Answer: Cardiac Pacing.
‫ــــــــــــــــــــــــــــــــــــــــــــــــــــــــــــــــــــــــــــــــــــــــــــــــــــــــــــــــــــــــــــــــــــــــــــــــــــــــــــــــــــــــــــــــــــــــــــــــــــــــــــــــــــــــــــ‬
124- Question 4 of 20
A 2 yo boy presents with a 3 day history of intermittent fever & tummy ache. No diarrhoea. He vomited once
yesterday. O/E his temp. is 37.5C & exam. of his ear, nose throat & chest are normal. His abd. is soft & non-tender.

(a) You suspect a UTI. Give 3 other possible diagnosis that are important to rule out in a boy of this age (3)
Answer: Appendicits, Mesenteric adenitis, Orchitis, Intussusception.
(b) The urine dipstick is positve for nitirites & leucocytes. Name 3 of the most likely organisms. (3)
Answer: Escheria Coli, Strep B, Klebseiella, Proteus, Enterobacter, Staph.
(c) Give 4 indications for admission in a child with UTI? (4)
Answer: Dehydrat./inability to tolerate oral fluids/repeated vomiting, Toxic child requiring IV antibiotics, Co-morbidities,
Parental concerns / inability to cope, Age<3/12 (some guidelines <6/12), Pyelonephritis / renal angle tenderness clinically
(d) How should you obtain a urine sample?
Answer: Not with a bag, should be clean catch MSU
(e) What would you treat this child with?
Answer: More than 3 months of age with signs of pyelonephritis Treat with oral antibiotics for 10 days if sufficiently
well5 <1 yo, Cephradine or Co-amoxiclav >1 yo, Cephradine or Trimethoprim If IV antibiotics required Cefuroxime
is the drug of choice. IV antibiotics should be continued until the pyrexia has settled & culture is available from which
an appropriate oral antibiotic can be given (total duration of TTT 10 days) More than 3 months of age with signs of
cystitis Treat with oral antibiotics for 3 days if sufficiently well but review if no improvement after 24-48 hrs <1 yo,
Cephradine or Co-amoxiclav (Augmentin) >1 yo, Cephradine or Trimethoprim
‫ــــــــــــــــــــــــــــــــــــــــــــــــــــــــــــــــــــــــــــــــــــــــــــــــــــــــــــــــــــــــــــــــــــــــــــــــــــــــــــــــــــــــــــــــــــــــــــــــــــــــــــــــــــــــــــ‬
125- Question 5 of 20
A family present to the ED with 4 children who have all developed a widespread rash as seen below. They have high
temp.s & have generalised coryzal symptoms including sore throat, conjunctivitis. They tell you that none of the
children have had their immunisations as they don't agree with it.
(a) What is the diagnosis? (1 mark)
Answer: Measles
(b) What are the 2 life threatening complications that you need to be vigilant for? (2 marks)
Answer: Pneumonia & encephalitis
(c) What actions do you take? (3 marks)
Answer: Advice family of the condit., Inform the HPA as measles is a notifiable illness, Look for 2ry bacterial infect.
(d) The mother is very concerned about her youngest child aged 4 yo & demands that she is admitted to
hospital. You think that she is relatively well with normal vital signs. What do you tell her? (1 mark)
Answer: Explain that it is self limiting disease & that if things were not improving in 3 days then she needs to seek
medical attention. Or if the child becomes more unwell i.e. any features of pneumonia or encephalitis.
(e) How long will the children be infective for? (1 mark)
Answer: From onset of symptoms until 5 days after the rash appears.
‫ــــــــــــــــــــــــــــــــــــــــــــــــــــــــــــــــــــــــــــــــــــــــــــــــــــــــــــــــــــــــــــــــــــــــــــــــــــــــــــــــــــــــــــــــــــــــــــــــــــــــــــــــــــــــــــ‬
126- Question 6 of 20
A 4 yo boy was brought to the ED by a parent with a painful ear.
(a) What is the diagnosis?
49
Answer: Otitis media.
(b) What are the risk factors for this condition?(Name 4)
Answer: The peak age-specific attack rate occurs between 6 & 18 months of age, the spread of bacterial & viral
pathogens is common in daycare centers, non-breast fed babies, Exposure to tobacco smoke & ambient air pollution 
the risk of OM, children who use a pacifier, children in developing areas, family history, social & economic condit.,
sleep position, season ( incidence during the fall & winter months), altered host defenses & underlying disease (eg,
cLt palate, Down syndrome, allergic rhinitis).
(c) What are the common species of bacteria accounting for most of the bacterial isolates from middle ear
fluid?(Name 2)
Answer: St. pneumoniae, Haemophilus influenzae, & Moraxella catarrhalis.
(d) How would you manage this Pt?(2 points)
Answer: Analgesia(paracetamol or ibuprofen), antibiotics(amoxicillin) & organise follow up to ensure resolution.
(e) What are the complications of this condition?(Name 4)
Answer: Mild conductive hearing loss, vestibular, balance & motor dysfunct., tympanic membrane perforat., inflammat.
of the mastoid &/or mastoiditis, petrositis & labyrinthitis. IC complications are rare in developed countries; they
include meningitis, epidural abscess, brain abscess, lateral sinus thrombosis, cavernous sinus thrombosis, subdural
empyema & carotid artery thrombosis.
‫ــــــــــــــــــــــــــــــــــــــــــــــــــــــــــــــــــــــــــــــــــــــــــــــــــــــــــــــــــــــــــــــــــــــــــــــــــــــــــــــــــــــــــــــــــــــــــــــــــــــــــــــــــــــــــــ‬
127- Question 7 of 20
A 2 yo boy presented with inspiratory stridor & a barking cough. O/E he was febrile & mildly tachycardic.

(a) What is the most likely diagnosis?


Answer: Croup.
(b) Give a differential diagnosis?(3 conditions)
Answer: Acute epiglottitis, peritonsillar & retropharyngeal abscesses, foreign body aspirat. or ingest., allergic react.,
acute angioneurotic edema, upper airway injury, congenital anomalies of the upper airway & laryngeal diphtheria
(c) What is the most common etiological agent?
Answer: Parainfluenza virus type 1 is the most common cause of acute laryngotracheitis, especially the fall & winter epidemics.
(d) Name 4 aspects of the exam. which are helpful in assessing the degree of up. airway obstruct. & severity of illness?
Answer: Overall appearance, quality of the voice, degree of resp. distress, TV, lung exam. & assessment of hydration status
(e) How would you manage this child?(Give five steps)
Answer: Administration of humidified air or humidified O2, antipyretics, encouragement of fluid intake, a single
dose of oral dexamethasone (0.6 mg/kg) (if fit for discharge), nebulized epinephrine, pulse oximetry, observation.
‫ــــــــــــــــــــــــــــــــــــــــــــــــــــــــــــــــــــــــــــــــــــــــــــــــــــــــــــــــــــــــــــــــــــــــــــــــــــــــــــــــــــــــــــــــــــــــــــــــــــــــــــــــــــــــــــ‬
128- Question 8 of 20
A 3 month old baby was brought to the ED as his mother had come upon the child in his cot apnoeic & off colour.
This was thought to be an apparent life threatening event (ALTE).

(a) How would you manage this Pt?


Answer: Take Bl. (FBC, UE, glucose, calcium, Mg, phosphate), admit for monitoring.
(b) What are the risk factors for SIDS(Name 4)?
Answer: Passive smoking, males, winter months, sleeping prone, premature babies, twins, apnoeic spells in first week
of life, lower socioeconomic groups, sibling with SIDS, maternal illicit drug abuse.
(c) What advice can you give the above Pt to prevent SIDS?(Give 4)
Answer: Avoid overheating, avoid duvets & excess bedding, sleep supine, consider apnoea alarm, avoid infant
sharing bed with parent.
(d) What is the definition of SIDS?
Answer: Sudden infant death in infancy with no cause identified after autopsy.
(e) What is the aetiology of SIDS?
Answer: Aetiology is unknown.
‫ــــــــــــــــــــــــــــــــــــــــــــــــــــــــــــــــــــــــــــــــــــــــــــــــــــــــــــــــــــــــــــــــــــــــــــــــــــــــــــــــــــــــــــــــــــــــــــــــــــــــــــــــــــــــــــ‬
129- Question 2005
A 6 yr old child is brought in fitting; he has been fitting for 5 min., is attached to monitors & is receiving O2. He has
been unwell for the last 3 days with a runny nose. He hasn't had his immunisat. His temp. is 39.6C & he weighs 20kg.
(a) Fill in the names, doses & timings of the drugs in the algorithm (see fig 1 for answers) for treating the fitting
child: There should be 5 boxes in the algorithm that you draw.
Answer: See fig 1
(b) Some lesions were noted in the child’s mouth (fig 2): What are the lesions called & what is the diagnosis?
Answer: Kopliks spots & Measles
(c) Name 8 other notifiable diseases:
50
Answer: Any 8 from Acute encephalitis, Anthrax, Botulism, Bruscellosis, Cholera, Diphtheria, Dysentery, Food
poisoning, HIV/AIDS, Legionella, Leptospirosis, Leprosy, Malaria, Measles, Meningitis, Meningococcal
Septicaemia, Mumps, Opthalima neonatorum, Paratyphoid, Plague, Polio, Rabies, Relapsing fever, Rubella, SARS,
Scarlet fever, Small pox, Syphilis, TB, Tetanus, Typhoid fever, Typhus, Viral haemorrhagic fever, Viral hepatitis,
Whooping cough, Yellow fever.

Fig 2
‫ــــــــــــــــــــــــــــــــــــــــــــــــــــــــــــــــــــــــــــــــــــــــــــــــــــــــــــــــــــــــــــــــــــــــــــــــــــــــــــــــــــــــــــــــــــــــــــــــــــــــــــــــــــــــــــ‬
130- Question 10 of 20
A 4 yo girl with leukaemia is brought in by her mother, they are on holiday in the area & normally would have gone
straight into their local chemotherapy suite as she has not been well & has a temp.

(a) What defines febrile neutropaneia?


Answer: Temp>38.5C on 1 occasion Temp>38 on 2 or more occasions recorded >1 hr apart Neutropaenia: Absolute
neutrophil count < 500/ml (< 0.5 x 109) ANC <1.0 & rapidly falling count after chemotherapy
(b) You assess the child, her observations are as follows: Pulse 160, cap refill 4secs, RR 52, sats 99% in air,
what do you do (include any calculations)?
Answer: Pt is shocked, needs urgent IV fluid bolus. Ages 4 yrs so (4+4) x2= 16kg therefore 20mls pre kg = 320mls of crystalloid.
(c) Which investigations do you send?
Answer: Relevant immediate investigations: Bl. cultures, systemic & waste Urine dipstix & culture Throat swab,
bacterial & viral Biochemistry, UE, LFT, CRP Haematology, DIC screen if septic
(d) Which areas should you pay particular attention to O/E?
Answer: 1. Mouth, teeth, gums, pharynx. 2. ENT, especially examining for tenderness over the sinuses & mastoid
sites. Consider NPA for Pts with coryzal symptoms 3. Respiratory, RR & O2 saturations & requirements must be
recorded & documented. Hypoxaemia & normal auscultation may be associated with Pneumocystis pneumonia
(PCP). 4. CVS, Bl. Pr. must be documented. 5. Upper GIT painful swallowing may be suggestive of herpetic or
candidal oesophagitis. 6. Abd. tenderness Rt lower quadrant pain may suggest typhilitis (neutropaenic caecal
inflammat.), as well as appendicitis discuss with senior member of staff. 7.Perineum symptoms of perianal discomfort
or pain should always be asked about. If there are symptoms, the perineum should be inspected. 8. Skin lesions look
for petechiae & purpura (evidence of thrombocytopaenia or DIC), consider Pseudomonas, herpetic, fungal aetiology
9. Central venous line sites erythema, swelling, tenderness are suggestive of infect. tracking along the line 10.
Procedure sites e.g. Gastrostomy sites, lumbar puncture, posterior superior iliac crests
(e) What do you need to do prior to completing a full history & exam.?
Answer: Treat the shock aggressively, need to cover with powerful antibiotics Discuss with senior paeds oncologist if
available But don't delay giving antibiotics: 1st line antimicrobials: 1. IV Ceftazidime 50mg/kg every 8 hrs (max 2 gm
tds) 2. IV Gentamicin* (see below for exceptions) a. <12 yrs 2.5mg/kg every 8 hrs b. >12 yrs 1.5mg/kg every 8 hrs
(max 120 mg tds) c. Levels must be taken after the 3rd gentamicin dose (pre + post) d. Adjust initial dose if Pt requires
gentamicin but has renal impairment & take earlier levels usually just give normal dose & take levels with 2nd dose
(must be taken). Wait for level before giving dose. 3. Oral fluconazole prophylaxis 3mg/kg once daily continue while
neutropaenic IV Flucloxacillin should be used instead of gentamicin in the following cases: Renal impairment discuss
with senior staff member Renally toxic chemotherapy protocols, i.e. those containing cisplatin or ifosfamide Dose of
flucloxacillin 25mg/kg every 6 hrs (max 1 gram qds) If Pt shows signs of septic shock contact a Senior member of
staff as they may require gentamicin irrespective of renal impairment or TTT with renally toxic chemotherapy
Additional antibiotics 1. Consider adding glycopeptide as 1st line agent if: CVL related infect. suspected Severe
mucositis Previous MRSA isolate IV Teicoplanin 10mg/kg (max 400mg) every 12 hrs for 3 doses, then once daily 2.
If significant perianal inflammation or possible typhilitis (Neutropaenic colitis) add: IV Metronidazole 7.5mg/kg (max
500mg) every 8 hrs Febrile at 48 hrs Discuss possible 2nd line antibiotics with Consultant: If Pt is unwell add: IV
Teicoplanin 10mg/kg (max 400mg) every 12 hrs for 3 doses, then once daily Febrile at 96 hrs Discuss possible 3rd line
antibiotics with Consultant: Consider empirical TTT for possible fungal infection (Consultant decision only): IV
Liposomal amphotericin (Ambisome) Dose 3mg/kg od (remember to prescribe test dose as per cBNF) Discuss change
of antibiotic with Consultant on call: IV Imipenem* a. <12 yrs 15mg/kg qds (max 500mg qds) b. >12 yrs 12.5mg/kg
qds (max1g qds) *Use Meropenem if evidence of renal impairment or history of CNS disorders
‫ــــــــــــــــــــــــــــــــــــــــــــــــــــــــــــــــــــــــــــــــــــــــــــــــــــــــــــــــــــــــــــــــــــــــــــــــــــــــــــــــــــــــــــــــــــــــــــــــــــــــــــــــــــــــــــ‬

51
131- Question 11 of 20
A 6 month old boy is brought in by his Mum, he was fine yesterday but this morning she noticed that he was crying >
normal & that he wasn’t moving his Lt leg as normal. There was no story of trauma. His x-ray is shown below:

(a) Describe what is shown in the x-ray


Answer: Transverse femur fracture of the diaphysis that is angulated but not shortened.
(b) What will you do?
Answer: Need to complete a full history & exam.. Then need to contact senior paediatrician. Social services will need
to be contacted & the child will need to be admitted not only for TTT of the fracture but for full NAI investigation.
(c) What is Munchausen by proxy (factitious ilness?)
Answer: Severe form of child abuse where adult (usually mother) fakes illness in their child, often they will induce
physical symptoms by giving medications i.e. headaches (GTN) & they will demand investigations.
(d) What else could have caused the appearances seen above?
Answer: Osteogenesis imperfecta,
‫ــــــــــــــــــــــــــــــــــــــــــــــــــــــــــــــــــــــــــــــــــــــــــــــــــــــــــــــــــــــــــــــــــــــــــــــــــــــــــــــــــــــــــــــــــــــــــــــــــــــــــــــــــــــــــــ‬
132- Question 13 of 20
A 9 yo boy was brought to the ED because after a prodrome of fever, malaise & pharyngitis he had developed a
pruritic rash shown in the picture.
(a) What is the diagnosis?
Answer: Chicken pox.
(b) What is the incubation period & how is this disease transmitted?
Answer: The average incubation period for varicella infection is 14 to 16 days although this interval can range from
10 to 21 days. Transmission occurs in susceptible hosts via contact with aerosolized droplets from nasopharyngeal
secretions of an infected individual or by direct cutaneous contact with vesicle fluid from skin lesions.
(c) What areas do the lesions cover?
Answer: The Pt with varicella typically has lesions in different stages of development on the face, trunk & extremities.
(d) Name 3 complications of this disorder?
Answer: The most frequent complication among healthy children is bacterial skin superinfection. Others include
encephalitis, reye syndrome, hepatitis & pneumonia.
(e) Name 3 general measures to treat this condition?
Answer: Antihistamines, fingernails should be closely cropped & paracetamol. Acyclovir may be used in selected cases.
‫ــــــــــــــــــــــــــــــــــــــــــــــــــــــــــــــــــــــــــــــــــــــــــــــــــــــــــــــــــــــــــــــــــــــــــــــــــــــــــــــــــــــــــــــــــــــــــــــــــــــــــــــــــــــــــــ‬
133- Question 14 of 20
A 13 yo national standard ice skater is sent in by her podiatrist who has recently made her some new inserts for her
skates. She is very distressed as she has an important competition in 2 wks time. She is complaining of pain in the Rt forefoot &
says that its gets progressively worse when she walks & has been unable to skate for the last 2 days. There is no history of trauma

(a) What could be going on & what do you do to investigate?


Answer: Frieberg’s disease. Foot x-ray to start. Might show flattening, widening or fragmentation of the metatarsal
head, or narrowing of the MTPJ.
(b) What would you advise?
Answer: NSAIDS & rest in the first instance. For this girl I would arrange orthopaedic referral. Persistent cases can
be treated with excision of the MT head or osteotomy.
(c) What is Osgood-Schlatter’s disease?
Answer: It is another osteochondritis affecting the tibial tuberosity. It is a traction apophysitis of the tibial attachment
of the patellar tendon normally seen in teenagers boys >girls. The tuberosity is prominent & tender.
(d) How should you treat plantar fasciitis?
Answer: NSAIDS & rest, elevate the foot. Heel pads can help. Severe persistent cases can be treated with local
steroid injections & sometimes surgical division of the plantar fascia.
‫ــــــــــــــــــــــــــــــــــــــــــــــــــــــــــــــــــــــــــــــــــــــــــــــــــــــــــــــــــــــــــــــــــــــــــــــــــــــــــــــــــــــــــــــــــــــــــــــــــــــــــــــــــــــــــــ‬
134- Question 15 of 20
A 10 yo boy was in a cubicle in the ED waiting to be seen & suddenly his mom ran out of the cubicle shouting for
help as her son had become unresponsive. The nursing staff confirm that the boy has no pulse & begin CPR. Other
staff members arrive & attach a monitor which shows asystole.

(a) After resuming CPR what medication should be given?


Answer: Adrenaline IV/IO: 0.01mg/kg(1:10000: 0.1mL/kg). This may be repeated every 3-5 minutes.
(b) After how many cycles of CPR should the rhythm be checked again?
Answer: 5 cycles.
(c) What is the compression to breath ratio for one cycle of CPR?
52
Answer: 15:2
(d) Name eight possible contributing causes to asystole?
Answer: Hypovolaemia, hypoxia, hydrogen ion(acidosis), hypokalaemia/hyperkalaemia, hypoglycaemia, hypothermia,
toxins, tamponade(cardiac), tension pneumothorax, thrombosis(coronary or pulmonary)& trauma.
(e) If after the 1st rhythm check the monitor shows VF what is the energy level the child will be shocked at initially?
Answer: 2J/kg.
‫ــــــــــــــــــــــــــــــــــــــــــــــــــــــــــــــــــــــــــــــــــــــــــــــــــــــــــــــــــــــــــــــــــــــــــــــــــــــــــــــــــــــــــــــــــــــــــــــــــــــــــــــــــــــــــــ‬
135- Question 16 of 20
A 1 yo boy is brought in by his parents after hitting his face on a wooden bar in the park, his mouth bled profusely
after the event & he appears to have lost his front tooth. By the time you see him the bleeding has settled

(a) What piece of information is crucial in this case?


Answer: Did the parents see the tooth; do they have it with them? If they didn’t there is a chance that it could have
been aspirated & go on to cause a lung abscess.
(b) What would you do?
Answer: Chest x-ray including lateral to look for foreign body.
(c) Another child presents with a lacerat. to the lip. What important features must you look for during the exam?
Answer: Must ensure that the laceration doesn’t cross the vermilion border. If it does it will need suturing as even a
1mm discrepancy will leave a scar.
(d) Unlike the cosmetically important facial lacerations that are almost always closed primarily, certain small
intraoral lacerations may be Lt open without repair. What are the indications for closure?
Answer: Indications for intraoral closure are (Mucosal laceration that creates a flap that interferes with chewing,
Mucosal laceration that is large enough to trap food particles, Wounds longer than 2 cm).
‫ــــــــــــــــــــــــــــــــــــــــــــــــــــــــــــــــــــــــــــــــــــــــــــــــــــــــــــــــــــــــــــــــــــــــــــــــــــــــــــــــــــــــــــــــــــــــــــــــــــــــــــــــــــــــــــ‬
136- Question 17 of 20
A 10 yo boy was in a cubicle in the ED waiting to be seen & suddenly his mom ran out of the cubicle shouting for
help as her son had become unresponsive. The nursing staff confirms that the boy has no pulse & begins CPR. Other
staff members arrive & attach a monitor which shows VF.

(a) At what energy level(J/kg) should the child be shocked initially?


Answer: 2J/kg.
(b) After the shock is delivered what is the next step?
Answer: Give 5 cycles of CPR & recheck rhythm after 5 cycles.
(c) At what energy level should the second shock be delivered at?
Answer: 4J/kg.
(d) What medication should be given at this stage & at what dose?
Answer: Adrenaline IV/IO: 0.01mg/kg(1:10000: 0.1mL/kg). This may be repeated every 3-5 minutes.
(e) If CPR is ongoing what medication should be considered & at what dose?
Answer: Amiodarone 5mg/kg IV/IO (or lidocaine 1mg/kg IV/IO).
‫ــــــــــــــــــــــــــــــــــــــــــــــــــــــــــــــــــــــــــــــــــــــــــــــــــــــــــــــــــــــــــــــــــــــــــــــــــــــــــــــــــــــــــــــــــــــــــــــــــــــــــــــــــــــــــــ‬
137- Question 18 of 20
The above film is of a 4 yo girl who presents 5 days after a fall onto her Lt arm. Mum is concerned as she has noticed
a swelling over the outer aspect of the arm.

(a) What is the abnormality on the film? How would you describe it?
Answer: Mid-shaft fracture of the ulna, this is a plastic deformity with clear bowing of the ulna.
(b) 1 of your consultants happens to be a round & casually lets you know that the films are inadequate. What
do they mean & what do you need to do now? What abnormality do you not want to miss here?
Answer: You need a true lateral at the elbow joint as you don’t have one. You don’t want to miss a dislocation of the
radial head & hence a Monteggia fracture dislocation. This is a common pitfall if you don’t request the correct films.
(c) What is a greenstick fracture?
Answer: Almost exclusively occurs during infancy & childhood. The bending of a bone with incomplete fracture
which involving the convex side only. Green stick fractures are characterized by a break in the bone which partially
extends across & then along the length of the bone forming the characteristic fracture pattern for which it is named.
‫ــــــــــــــــــــــــــــــــــــــــــــــــــــــــــــــــــــــــــــــــــــــــــــــــــــــــــــــــــــــــــــــــــــــــــــــــــــــــــــــــــــــــــــــــــــــــــــــــــــــــــــــــــــــــــــ‬
138- Question 19 of 20
A neonate is brought to the ED by his parents as he had a fever, cough & wasn’t feeding well. The treating emergency
clinician suspected pneumonia.
(a) What are the common pathogens involved in neonatal pneumonia?(Name 2)
Answer: E.Coli, beta-haemolytic strep, chlamydia trachomatis, listeria monocytogenes, CMV.
(b) How would you investigate this Pt? (Give 4)
53
Answer: Throat swabs, FBC, cultures, viral titres, mycoplasma antibodies, SpO2, urine cultre, CXR.
(c) How would you treat this Pt?
Answer: O2, IV fluids, specialist referral, benzylpenicillin & gentamicin alternatively cefuroxime or co-amoxyclav.
(d) What are the risk factors for neonatal pneumonia? (Give 4)
Answer: Prolonged rupture of the fetal membranes (>18 hrs), maternal amnionitis, premature delivery, fetal
tachycardia, maternal intrapartum fever, anomalies of the airway (eg, choanal atresia, tracheoesophageal fistula &
cystic adenomatoid malformations), severe underlying disease, prolonged hospitalization, neurologic impairment
resulting in aspiration of gastrointestinal contents.
(e) What are the factors which determine outcome? (Give 4)
Answer: Increased mortality is associated with preterm birth, pre-existing chronic lung disease, or immune
deficiencies. Severity of the disease, the gestational age of the Pt, underlying medical conditions & the infecting
organism affect the prognosis of the disease.
‫ــــــــــــــــــــــــــــــــــــــــــــــــــــــــــــــــــــــــــــــــــــــــــــــــــــــــــــــــــــــــــــــــــــــــــــــــــــــــــــــــــــــــــــــــــــــــــــــــــــــــــــــــــــــــــــ‬
139- Question 20 of 20
A 10 yo girl presents with an earring embedded in the earlobe with an associated local infect. You decide to do a nerve block.

(a) draw a diagram to indicate the site of injection & the nerve involved
Answer: Great auricular nerve block Subcutaneous injection infiltrate 1cm below the ear lobe from the posterior
border of the SCM to the angle of the mandible.
(b) Calculate the dose of Lidocaine 1% for this girl, show calculation
Answer: 10 yrs = 28kg (age+4) x2 Max dose = 3mg/kg, ie 84 mg Max dose of 1% lidocaine is 8.4mls
(c) What systems & symptoms does LA affect in overdose
Answer: Perioral & lingular paraesthesia & numbness CNS: Lightheaded, dizzy, LOC, Seizure CVS: Arrhythmia, Cardiac arrest
‫ــــــــــــــــــــــــــــــــــــــــــــــــــــــــــــــــــــــــــــــــــــــــــــــــــــــــــــــــــــــــــــــــــــــــــــــــــــــــــــــــــــــــــــــــــــــــــــــــــــــــــــــــــــــــــــ‬
140- Question 1 of 30
A 3 yo child is sent in by their GP as having a non-blanching rash & the GP wonders if it might be meningococcal
disease. He gives the child IM penicillin & sends them straight in to see you.

(a) Please give 4 differential diagnoses for a true non-blanching rash. Not including ITPP, HUS, HSP or acute
leukaemias which are all distinct & usually not difficult to diagnose.
Answer: Meningococcal disease (MCD), Sepsis with other bacteria, Viral illness, Trauma/NAI
(b) Describe how a child with ITP normally presents.
Answer: Usually well children with multiple bruises & petechiae noticed over severall days. Often seen after a viral
illness. And can get conjunctival haemorrhage, nose bleeds & bleeding gums.
(c) Does the fact that the child has been treated with penicillin affect the management principles that you will follow?
Answer: No, you would treat as you would another child but these children do require a senior paed. review prior to D/C.
(d) Define a purpuric rash.
Answer: Lesions >2mm in diameter that are non-blanching. Spontaneous bleeding into the skin usually appears as a
rash known as purpura
(e) If the lesions were purpuric & the child had a mild temp. what would be your initial management?
Answer: To give IV broad spectrum antibiotics; a third generation cephalosporin. Ceftriaxone 80mg/kg (od) or
cefotaxime 50mg/kg (tds)
(f) The lesions are confined to the area above the nipple line & you think that the child is otherwise quite well.
Explain the thoutht process that you will use to decide whether or not to admit him to hospital.
Answer: If the lesions are not purpuric i.e. they are less than 2mm & the child is well, i.e. not irritable, lethargic &
haemodynamically stable then you can look for the distribution of the rash if it is confined to the SVC distribution
then the child can be discharged as long as there is a focus of infection & there are no concerns over NAI.
‫ــــــــــــــــــــــــــــــــــــــــــــــــــــــــــــــــــــــــــــــــــــــــــــــــــــــــــــــــــــــــــــــــــــــــــــــــــــــــــــــــــــــــــــــــــــــــــــــــــــــــــــــــــــــــــــ‬
141- Question 3 of 30
A red phone call tells you that a 6 yo girl is on the way who is shocked. She is a type 1 DM & has been well over the
last few days; today she had some vomiting & Abd. pain. You assess her & begin to treat her gaining IV access &
instigating a fluid bolus. Her BM is 1.4. Mother tells you that she has been getting recurrent low BM readings over the
last few weeks that they haven't been able to explain.

(a) What will you do?


Answer: Give 5ml/kg 10% dextrose bolus followed by maintenance fluids. If unable to gain IV access & not drowsy
or unresponsive give sugar orally (eg.100ml coke, lemonade, orange juice, 2-3 dextrose tablets, milk feed, Glucogel)
If drowsy or unresponsive give IM Glucagon 0.5 mg < 25 kg, 1 mg > 25 kg
(b) You sent off some routine Bl.s initially & they come back, WBC normal, K+ 6.1, Na+ 128. You are
concerned as the child has not responded to your initial fluid bolus; you give another bolus & seek advice from
the consultant paediatrician. What is the possible diagnosis?
54
Answer: Undiagnosed 1ry adrenal insufficiency with acute adrenal crisis. Other autoimmune diseases may be a clue
to the presence of Addison’s disease. e.g. recurrent hypoglycaemia in a child with type 1 diabetes mellitus
(c) What is the management? (include any drug doses)
Answer: IV hydrocortisone 25mg (<10 kg), 50 mg (10-25 kg), 100mg (> 25kg) & continue 6 hrly until well with no
diarrhoea/vomiting & stable Bl. sugar & electrolytes. If unable to gain IV access give IM hydrocortisone
(d) Explain the pathophysiology of diabetes insipidus.
Answer: Diabetes insipidus is a condition characterized by excretion of large amounts of severely diluted urine,
which can't be ↓ when fluid intake is ↓. It denotes inability of the kidney to concentrate urine. DI is caused by a
deficiency of ADH, also known as vasopressin, due to the destruction of the back or "posterior" part of the pituitary
gland where vasopressin is normally released from, or by an insensitivity of the kidneys to that hormone. It can also
be induced iatrogenically by various drugs.
‫ــــــــــــــــــــــــــــــــــــــــــــــــــــــــــــــــــــــــــــــــــــــــــــــــــــــــــــــــــــــــــــــــــــــــــــــــــــــــــــــــــــــــــــــــــــــــــــــــــــــــــــــــــــــــــــ‬
142- Question 5 of 30
A 4 yo boy was playing with his brother magnetic set. He swallowed 4 magnetic balls yesterday but Mum has only
just found out. He is completely well in himself but she wanted to get him checked over.

(a) What do you tell Mother?


Answer: That you need to confirm that they are in his gut by taking an Abd. x-ray.
(b) What signs do you ask Mum to look out for?
Answer: Look out for any Abd. pain or vomiting, ensure that he is having normal bowel movements. If he appears at
all unwell then needs to be seen immediately.
(c) What do you do with this child? Do you admit them or send them home?
Answer: Needs surgical referral, may not need to come in but some authors propose that if multiple magnets are
found on imaging then they should be removed. Probably should come in for close observation.
(d) What complications could potentially ensue?
Answer: There are case reports of volvulus, perforation & obstruction when 1 or more magnets has been ingested. Or
when 1 magnet & a further metallic object has been ingested.
‫ــــــــــــــــــــــــــــــــــــــــــــــــــــــــــــــــــــــــــــــــــــــــــــــــــــــــــــــــــــــــــــــــــــــــــــــــــــــــــــــــــــــــــــــــــــــــــــــــــــــــــــــــــــــــــــ‬
143- Question 8 of 30
A 9 month baby girl is brought in with difficulty breathing & short history of being generally unwell & poor feeding.
She has a temp. of 38.2◦C & has saturations of 93% in room air.
(a) What is the normal RR in a child of 9 months? (1 mark)
Answer: Answer: 30-40 breaths a minute.
(b) When is the bronchiolitis season? (1 mark)
Answer: Answer: Late autumn to early spring
(c) Name 3 viruses that cause bronchiolitis stating which one is most common. (3 marks)
Answer: Respiratory syncitial virus (RSV) is the most common (70-80%). Others include, adenovirus, influenza,
parainfluenza, metapneumo-virus.
(d) Some children are at increased risk of severe illness & would almost always be admitted. List 3
circumstances where this would be the case. (3 marks)
Answer: Any of the following: Infants <6 wks, Preterm Infants, CLD, Congenital HD, Immunodeficiency
(e) On the history & information above what would you do with this child, list 3 criteria on which you would
base your decision please include some specific objective measurements? (3 marks)
Answer: It depends but answer should include a senior paeds review. Any one of these features may be sufficient to
prompt admission & not all are required. Marked recession/resp. distress or grunting resp. O2 saturations <92% * RR
>70/m Taking <50% usual feeds or concerns regarding hydration status History of apnoea Appears unwell or lethargic
(f) Describe the role of drug TTTs in bronchiolitis (2 marks)
Answer: Nebulised Ribavirin: minimal evidence shows it to be effective in reducing length of hospital stay &
ventilatory support in severely affected Pts. Consultant approval required before commencing ribavarin therapy. 1
mark awarded for stating that the mainstay of TTT is supportive rather than medication based.
(g) What is the most important thing that needs to be considered when admitting & nursing children with bronchiolitis?
Answer: Limiting cross infection by any sensible means described in the answer.
‫ــــــــــــــــــــــــــــــــــــــــــــــــــــــــــــــــــــــــــــــــــــــــــــــــــــــــــــــــــــــــــــــــــــــــــــــــــــــــــــــــــــــــــــــــــــــــــــــــــــــــــــــــــــــــــــ‬
144- Question 9 of 30
A 5 yo boy has eaten some mushrooms & is brought in vomiting he is haemodynamically stable & only ate them 30
minutes ago he found them in his 17 yo brother's room. Parents are concerned that they may be magic mushrooms
(a) What is the dose of activated charcoal in children?
Answer: 1g/kg
(b) What 2 conditions are essential to be met prior to considering its administration?
55
Answer: That it is within one hr of ingestion (unless slow release preparations are ingested could consider giving
later) That there is adequate airway protection either via an ET tube or a fully conscious Pt. (Aspiration can be fatal)
(c) Is there any place for giving it in this scenario?
Answer: Firstly the child is vomiting so is unlikely to be able to take the charcoal.Activated charcoal can be used if
death cap mushroom (Amanita phalloides) are ingested but there is no role for it in this scenario.
(d) Name to TTTs for ingestion of antifreeze.
Answer: Ethanol or alcohol dehydrogenase inhibitor (fomepizole)
(e) What is the dose of naloxone in children?
Answer: Neonate 5-10 micrograms/kg, repeated every 2-3 minutes if required Child 1 month-12 yrs 5-10
micrograms/kg; if response inadequate, give a subsequent dose of 100 micrograms/kg (max. 2 mg) Child 12-18 yrs
1.5-3 micrograms/kg; if response inadequate, give subsequent doses of 100 micrograms every 2 minutes
‫ــــــــــــــــــــــــــــــــــــــــــــــــــــــــــــــــــــــــــــــــــــــــــــــــــــــــــــــــــــــــــــــــــــــــــــــــــــــــــــــــــــــــــــــــــــــــــــــــــــــــــــــــــــــــــــ‬
145- Question 16 of 30
A 4 wk old baby boy is brought in by his parents who say that he has been vomiting after every feed they say that the
vomiting is projectile in nature. You wonder about pyloric stenosis. You establish venous access & give a fluid bolus of 10ml/kg

(a) What age group are affected by pyloric stenosis & what exactly is it?
Answer: Pyloric stenosis is hypertrophy of the muscles surrounding the pylorus of the stomach. It is uncertain
whether there is a real congenital narrowing or whether there is a functional hypertrophy of the muscle that develops
in the 1st few wks of life. Age affected: Usually presents between 3 & 6 wks of age Late presentat. up to 6 months can occur1
(b) What is helpful when making the diagnosis?
Answer: Palpable 'tumour' in Rt upper quadrant best felt from Lt during test feed Visible peristalsis often seen
Diagnosis can be confirmed by Abd. ultrasound Needs assessment of length, diameter & thickness of the pylorus A
wall thickness of great than 3mm supports the diagnosis Biochemically a hypochloraemic alkalosis exists
(c) How is it treated?
Answer: Correct dehydrate. over a 24 - 72 hr period, NGT is often required Ramstedt's pyloromyotomy 1st described
in 1911 Transverse Rt upper quadrant or circumumbilical incision Longitudinal incision in pylorus down to mucosa
Incision extend from duodenum onto the gastric antrum Need to try & avoid mucosal perforation pyloromyotomy
(d) Another child comes in with similar symptoms but doesn’t appear too dehydrated & the vomiting isn't
really projectile. What do you need to do to try to establish the diagnosis?
Answer: Do a test feed to assess the nature of the vomiting Also establish the total amount that they are feeding,
should be about 150mls per kg if they are massively overfeeding then this may represent the main problem.
‫ــــــــــــــــــــــــــــــــــــــــــــــــــــــــــــــــــــــــــــــــــــــــــــــــــــــــــــــــــــــــــــــــــــــــــــــــــــــــــــــــــــــــــــــــــــــــــــــــــــــــــــــــــــــــــــ‬
146- Question 17 of 30
A 6 yo boy is brought to the ED after a fall on his Rt wrist. X Ray reveals a colles fracture. The decision is made to
manipulate the boy's wrist using ketamine for procedural sedation

(a) What are the advantages of using ketamine for procedural sedation? (Give 2)
Answer: Ketamine provides sedation, analgesia, amnesia, & immobilization, while usually preserving upper airway
muscle tone, airway protective reflexes, & spontaneous breathing.
(b) What is the dose range when using ketamine for procedural sedation intravenously?
Answer: 0.5mg to 2 mg/kg.
(c) What is the duration of action of ketamine?
Answer: 10 to 20 minutes. (though typical duration of effective dissociation is 5-10 min)
(d) Name 3 side effects?
Answer: Side effects of ketamine include  salivat., vomiting, unpleasant hallucinations, laryngospasm rarely occurs.
(e) What are the disadvantages of giving ketamine via the IM route?(Give 2)
Answer: Longer recovery times & more vomiting.
‫ــــــــــــــــــــــــــــــــــــــــــــــــــــــــــــــــــــــــــــــــــــــــــــــــــــــــــــــــــــــــــــــــــــــــــــــــــــــــــــــــــــــــــــــــــــــــــــــــــــــــــــــــــــــــــــ‬
147- Question 2 of 20
A 3 yo child attends the department with worried parents who tell you that she refuses to use her Lt arm. There is no
history of trauma. O/E the arm appears to move normally with out discomfort but the child cries when you palpate the
arm. There is no swelling or deformity.
(a) What is the next appropriate course of action?
Answer: With no accurate history a fracture or other soft tissue injury can't be confidently ruled out. Therefore an x-
ray is the next step. If reasonable doubt surrounds the diagnosis, performing radiography of the extremity before
attempting reduct. is prudent to avoid manipulat. of an extremity with an elbow fracture this is a medico-legal pitfall.
(b) Explain how you would manipulate a pulled elbow
Answer: TTT consists of manipulating the child's arm so that the annular ligament & radial head return to their
normal anatomic positions. a. This is accomplished by immobilizing the elbow & palpating the region of the radial
56
head with 1 hand. b. The other hand applies axial compression at the wrist while supinating the forearm & flexing the
elbow. c. As the arm is manipulated, a click or snap can be felt at the radial head. A click noted by the examiner has a
+ve predictive value of more than 90% in 2 published case series & a -ve predictive value of 76% in one case series.
Some authors believe the likelihood of successful reduction is  if pr. is applied over the radial head. Nursemaid's
elbow can be reduced by extension of the forearm instead of flexion; however, extension was less effective in
achieving reduction in 1 case series. A recent abstract reports that pronation may be more effective than supination.1
(c) What is the age range for a pulled elbow, which arm is more commonly affected & is there any sex
preponderance?
Answer: Normally 1-4 yrs but 4 months to 15yrs have been reported, Lt arm more common as more care givers are
Rt handed. Girls more common than boys.
(d) How would you manage a failed attempt at manipulation of a pulled elbow?
Answer: Attempt again up to 3 times but must x-ray if still unsuccessful (if not already x-rayed) If radiographic
findings demonstrate no fracture, repeat attempts at reduction are unsuccessful, & the child doesn't regain normal
function after 30-40 min., the safest management is to support the arm in a sling (or splint & sling) & have the child
re-evaluated by a physician (usually a 1ry care physician, not an orthopedist) in 1-2 days. 1 case series reported 7 Pts
meeting these criteria had either spontaneous return of funct. or successful reduct. at follow-up evaluat. by day 4.1
‫ــــــــــــــــــــــــــــــــــــــــــــــــــــــــــــــــــــــــــــــــــــــــــــــــــــــــــــــــــــــــــــــــــــــــــــــــــــــــــــــــــــــــــــــــــــــــــــــــــــــــــــــــــــــــــــ‬
148- Question 3 of 20
A 4 yo boy was brought to the ED by his mother as he had a 2 week history of cough.

(a) What are the clinical features of whooping cough?(Give 4)


Answer: Cough may persist for several weeks, an inspiratory noise (whoop) after a bout of coughing, coughing may
culminate in vomiting, cough typically worse a night, conjunctival haemorrhage 2ry to severe coughing.
(b) What is the pathogen involved?
Answer: Bordetella pertusis.
(c) How would you investigate this Pt?(Give 4)
Answer: Bl. for viral titres, mycoplasma antibodies, FBC, CXR.
(d) How would you treat this Pt?(Name 3)
Answer: Erythromycin, avoidance of other children, arrange GP follow up, prophylaxis for unimmunised infant
siblings, notifiable disease.
(e) What are the complications of this condition? (Name 2)
Answer: Prolonged illness, neurological damage, bronchiectasis.
‫ــــــــــــــــــــــــــــــــــــــــــــــــــــــــــــــــــــــــــــــــــــــــــــــــــــــــــــــــــــــــــــــــــــــــــــــــــــــــــــــــــــــــــــــــــــــــــــــــــــــــــــــــــــــــــــ‬
149- Question 10 of 20
A 4 yo boy is brought to the ED by his parents. They complained that he had been distressed that morning but could
not explain why. He had no history of trauma or fever but had become abruptly distressed. O/E his heart rate was
200/min. There was also evidence of poor perfusion.

(a) Before ordering an ECG give 3 basic management steps?


Answer: ABC's, O2, attach monitor/defibrillator.
(b) His ECG revealed narrow QRS complexs & was thought to be a probable SVT. What is the next step?
Answer: Vagal maneuvers.
(c) If this fails & but IV access is readily obtained what medication should be given & at what dose?
Answer: Adenosine. 0.1mg/kg by rapid bolus.
(d) What other TTT modality should be considered?
Answer: Synchronised cardioversion. 0.5 to 1J/kg. If this isn't effective increase to 2 J/kg.
(e) Name eight possible contributing causes to paediatric tachycardias with poor perfusion(5H's & 5T's)?
Answer: Hypovolaemia, hypoxia, hydrogen ion(acidosis), hypokalaemia/hyperkalaemia, hypoglycaemia,
hypothermia, toxins, tamponade(cardiac), tension pneumothorax, thrombosis(coronary or pulmonary) & trauma.
‫ــــــــــــــــــــــــــــــــــــــــــــــــــــــــــــــــــــــــــــــــــــــــــــــــــــــــــــــــــــــــــــــــــــــــــــــــــــــــــــــــــــــــــــــــــــــــــــــــــــــــــــــــــــــــــــ‬
150- Question id: 2093
A 13 day baby boy is brought in, he Lt the neonatal unit 3 days ago. Mum says that he has a bowel disorder that needs
an operation, they are currently doing washouts of his rectum as he cannot pass stool himself. Mum is a little
concerned as he has vomited his feeds this afternoon & wonders if his abd. is a little distended.
(a) You ask if the baby had a rectal biopsy whilst on the NICU, they says yes & you also find out that he is
otherwise completely healthy & was not ventilated whilst on the NICU. What is the diagnosis?
Answer: Hirschprung's disease
(b) You examine the child carefully. What important signs are you looking for? Do you order any tests?
Answer: General ABCDE approach need to assess if the baby is septic, need to think about intestinal obstruction.
Looking for distended bowel loops. Check BM, temp. full set of observations cap refill etc. Order a plain Abd. Film
57
looking for obstruction & signs of necrotising enterocolitis NEC. Full set of Bl.s including cultures/CRP/WBC
(c) The AXR (fig 1)was taken in the ED what does it show & what will you do?
Answer: It shows Pneumatosis intestinalis, which is pathognomonic for NEC. An urgent surgical consultation is
needed & further imaging ultrasound if a skilled provider is available or Lt lateral decubitus imaging to rule out a
pneumoperitoneum. If this is the case then surgery will be indicated. Baby needs to be NBM, have an NGT inserted &
have IV antibiotic started. Needs to be managed on PICU. If free gas was shown then needs an urgent laparotomy.

Fig 1
(d) What is the approximate mortality of this condition?
Answer: 50% mortality but higher in severe NEC
‫ــــــــــــــــــــــــــــــــــــــــــــــــــــــــــــــــــــــــــــــــــــــــــــــــــــــــــــــــــــــــــــــــــــــــــــــــــــــــــــــــــــــــــــــــــــــــــــــــــــــــــــــــــــــــــــ‬
151- Question 8 of 20
8 yr old presents lethargic & dehydrated. Weighs 22 kg. Looks unwell. RR 40, Sats 98% on O2. Started on re-hydrat. fluids

(a) You decide to give a bolus what would you give?


Answer: 20mls/kg or 10mls/kg = appropriate so either 440mls or 220mls depending on the level of dehydration they
thought was appropriate.
(b) Give 4 features of 5% dehydration
Answer: Clinically useful signs for detecting 5% dehydration were capillary refill time, abnormal skin turgor &
abnormal resp. pattern (table). Dry MM, sunken eyes & poor overall appearance are moderately useful
(c) Calculate the maintenance fluid required in the first 8 hrs
Answer: first 10 kg = 100ml/kg/24 hrs second 10 kg = 50ml/kg/24 hrs subsequent kgs = 20ml/kg/24 hrs i.e for 22kg
child = 1540 mls/24 hrs = 513 mls / 8 hrs
(d) Give 3 investigations to establish the underlying diagnosis in the ED with the diagnosis considered for each test
Answer: Stool spec for GE., BM for new presentation of DM, Urinalysis for Urinary sepsis, CXR for pneumonia.
‫ــــــــــــــــــــــــــــــــــــــــــــــــــــــــــــــــــــــــــــــــــــــــــــــــــــــــــــــــــــــــــــــــــــــــــــــــــــــــــــــــــــــــــــــــــــــــــــــــــــــــــــــــــــــــــــ‬
152- Question id: 2025
A 5 yo child presents with a rash & some Abd. pain. He also has pains in his knees & ankles. Mother tells you that he
had a cold 2 weeks ago & hasn't been Rt since the rash appeared within the last 24 hrs. The rash looks similar to the
one seen in this photo (see fig 1): It is present on the extensor surface of the lower limbs as well as the buttocks.

Fig 1
(a) What is the diagnosis? (1 mark)
Answer: Henoch-Schonlein purpura
(b) Which are the 3 main areas that are affected by this condition? (3 marks)
Answer: GI, renal & skin
58
(c) What investigations must be performed? (3 marks)
Answer: Urinalysis, BP & FBC/UE
(d) The child is admitted under the paediatricians but later in the day develops some bloody diarrhoea. What
investigation will you do & what condition is important to rule out? (2 marks)
Answer: Intussussception, Abd. ultrasound scan
(e) What informations will you give the parents with regard to prognosis of the condition?
Answer: HSP is an acute self-limited illness & usually resolves without TTT, but may rarely lead to complications.
Initial attacks of Henoch-Schonlein purpura can last several months. One third of Pts have one or more recurrences.
Children younger than 3 yrs have a shorter, milder course & fewer recurrences. The long-term prognosis of Henoch-
Schonlein purpura is directly dependent on the severity of renal involvement.1
‫ــــــــــــــــــــــــــــــــــــــــــــــــــــــــــــــــــــــــــــــــــــــــــــــــــــــــــــــــــــــــــــــــــــــــــــــــــــــــــــــــــــــــــــــــــــــــــــــــــــــــــــــــــــــــــــ‬
153- Question id: 2016
A 5 yo girl is brought in by her parents who say that she isn't Rt but they cannot identify exactly why. When you ask
the girl if she is ok she says that she feels funny

(a) Her initial observation show that she is tachycardic at 260 b.p.m, what will you do? (2 marks)
Answer: ABCD, apply O2 Attach 3 lead monitoring & get a 12 lead ECG Measure BP & cap refill time essentially
assess if hameodynamically stable Obtain IV access in a large proximal vein.
(b) Her ECG is shown (see fig 1): What does it show (1 mark)
Answer: Narrow complex tachycardia. (SVT)
(c) Her BP is 90/50, but her heart rate is still 260. What will you do? (2 marks)
Answer: As is haemodynamically stable can try Vagal techniques: Try valsalva but in 5 yr old better to elicit diving
reflex, Facial cooling with ice for 15 seconds Immersion wrap the child in a towel & immerse the whole head in a
bucket of ice water for 5 seconds (no need to obstruct mouth or nose).
(d) Name a drug that could be used for this child & give the correct dose based on her age. (2 marks)
Answer: Adenosine dose (5+4=9) x2 = 18kg (estimated weight) therefore giving 0.05mg/kg= 0.9mg or 900mcg.
(e) Are there any drug interactions that you need to know about with your chosen drug? (1 mark)
Answer: Yes; adenosine's action is prolonged by a factor of 4 by dipyridamole!
(f) The drug that you chose failed to work what will you do next?
Answer: reassess check that still haemodynamically stable then give further adenosine at doses of 0.1mg/kg then 0.2mg/kg g)
(g) The child fails to respond & seems to be drowsy now you repeat the BP which is now not reading what will
you do? (3 marks) Must have dose for 1st shock to gain any marks
Answer: Get someone to urgently call the paeds on call anaesthetist. Draw up some drugs that they may need. Get the
defibrillator attached in sync mode & dial up 0.5joules/kg in this case 10 joules. Give synchronised DC shock.
‫ــــــــــــــــــــــــــــــــــــــــــــــــــــــــــــــــــــــــــــــــــــــــــــــــــــــــــــــــــــــــــــــــــــــــــــــــــــــــــــــــــــــــــــــــــــــــــــــــــــــــــــــــــــــــــــ‬
154- Question 4 of 50
A 34 yo man is involved in motor cross accident- he was partially impaled on a wooden stake at the side of the course
which penetrated the Rt side of his back. He is flown in. When he arrives his observations are as follows. GCS 14, RR
35 sats 88%, BP 145/70 pulse 110. He is screaming in pain intermittently. You can see a large open wound with a
wooden stake sticking through it on the Rt side of the back between T5-L3.

(a) What are you most concerned about? & what would you do about it?
Answer: A pneumothorax! The low sats & high RR rate with tachycardia along with the site of the injury must raise
the possibility of a haemopneumothorax. Examine the chest- if not tensioning get a CXR. Insert a chest drain.
(b) With fluid resuscitation & good analgesia the BP remains 145/68 & the tachycardia comes down to 85.
What analgesia is best in this situation? please give doses.
Answer: Why not use a fast acting opioid like fentanyl- 50-100 mcg in increments- start with 50 mcg then titrate the
rest Morphine takes too long to work in this situation.
(c) What would you do now?
Answer: FAST scan for free fluid would be good idea: if present then should go for a laparotomy but as is
haemodynamically stable CT chest/abd./pelvis would be the investigation of choice. May need MRI of the spine later
but that can wait.
(d) You perform a quick secondary survey: on neurological assessment you discover that the Rt leg is
hyperreflexic & has decreased power but the Lt leg seems normal. Light touch is normal both sides but the Lt
leg there is no sensation to painful stimulus. What could explain these findings?
Answer: Brown- Sequard syndrome
‫ــــــــــــــــــــــــــــــــــــــــــــــــــــــــــــــــــــــــــــــــــــــــــــــــــــــــــــــــــــــــــــــــــــــــــــــــــــــــــــــــــــــــــــــــــــــــــــــــــــــــــــــــــــــــــــ‬
155- Question 7 of 50
A 30 yo man presented to the ED with a lacerat. to his Rt middle finger over the middle phalanx. There were no associated
sensory symptoms or tendon damage. The treating clinician decided to repair the lacerat. using a digital nerve block.
59
(a) What anaesthetic agents are suitable for this procedure? What additional agent should be avoided?
Answer: Lignocaine, bupivacaine. Avoid adrenaline.
(b) What volume of fluid should be used on each side of the finger?
Answer: 1-2ml on each side of the finger.
(c) What alterat. should be made to the procedure if the lacerat. was over the proximal portion of the middle phalanx?
Answer: An additional injection of LA should be given across the dorsum of the base of the proximal phalanx.
(d) How long does it take anaesthesia to develop?
Answer: About 5 minutes.
(e) How does the skin feel if the block is working?
Answer: Warm & dry as the autonomic nerves are blocked also.
‫ــــــــــــــــــــــــــــــــــــــــــــــــــــــــــــــــــــــــــــــــــــــــــــــــــــــــــــــــــــــــــــــــــــــــــــــــــــــــــــــــــــــــــــــــــــــــــــــــــــــــــــــــــــــــــــ‬
156- Question 15 of 50
A 42 yo man presented with sudden onset pain in his Lt ankle during a game of squash. The Pt had heard a snap &
reported that it had felt like a baseball bat had hit the back of his ankle. The treating clinician thought that an achilles
tendon rupture was likely
(a) Name 3 risk factors for this condition?
Answer: This typically occurs in men over the age of 30 who sporadically engage in sports & do not do a regular leg
conditioning program, fluoroquinolone antibiotic use, corticosteroid use & genetic predisposition.
(b) What features may be present O/E if the Pt has an achilles tendon rupture?(Name 3)
Answer: The Pt may be unable to stand up on the toes, thompson's test may be positive & there may be a palpable
gap in the area of the achilles tendon.
(c) How would you investigate this Pt?(Give one investigation)
Answer: Ultrasound.
(d) How would you manage this Pt?
Answer: Orthopedic consultation for immobilization or repair is necessary for Pts with tendon rupture.
(e) What is the risk of non-operative TTT?(Give one)
Answer: Nonoperative TTT appears to be associated with a higher risk of rerupture.
‫ــــــــــــــــــــــــــــــــــــــــــــــــــــــــــــــــــــــــــــــــــــــــــــــــــــــــــــــــــــــــــــــــــــــــــــــــــــــــــــــــــــــــــــــــــــــــــــــــــــــــــــــــــــــــــــ‬
157- Question 24 of 50
A 15 yo boy comes in to see you with a swollen Rt knee. He was playing football yesterday. He went in for a tackle,
his studs were planted in the ground & he inwardly rotated on the knee joint causing severe pain. He heard a popping
sound as it happened
(a) He has a large swelling & can’t flex the knee >10 degrees, he doesn’t respond to pain killers. What do you need to do?
Answer: Need to examine the knee fully. Rule out a bony injury. Need to x-ray the joint to look for haemarthrosis or
lipohaemarthrosis. If the effusion is so large that it is causing severe pain that is unresponsive to analgesics then 1
could consider aspirat.
(b) What are the indications for aspiration of a knee after trauma?
Answer: As above; if the pain cannot be relieved then can aspirate a large effusion. If you are concerned about
compartment syndrome.
(c) What would differentiate an effusion from a haemarthrosis?
Answer: Predominantly the history, if it occurs soon after injury more likely to be haemarthrosis if later more likely
to be an effusion.
(d) How would you manage this Pt?
Answer: As above consider aspiration for pain relief. Need to put in a Richard’s splint & give crutches for comfort.
RICE. Fracture clinic follow up.
(e) If you clinically suspected a fracture of the patella but the x-rays appeared normal what could you do?
Answer: Obtain skyline or oblique views.
(f) How do you treat infrapatellar bursitis?
Answer: avoid the causative activity, NSAIDS & rest. Persistent symptoms can lead to elective excision of the bursa,
if any systemic symptoms then watch for infective bursitis
‫ــــــــــــــــــــــــــــــــــــــــــــــــــــــــــــــــــــــــــــــــــــــــــــــــــــــــــــــــــــــــــــــــــــــــــــــــــــــــــــــــــــــــــــــــــــــــــــــــــــــــــــــــــــــــــــ‬
158- Question id: 2041
A 15 yo girl slipped rushing for a bus & fell on her outstretched hand. She complained of a painful wrist.
(a) What is name of the fracture shown?
Answer: Colles' fracture. Colles' fractures involve dorsal displacement of the distal radius fragment.
(b) What is the name of the characteristic deformity associated with this fracture?
Answer: Dinner fork deformity.
(c) What nerve can be compressed by severely displaced fractures of this kind? Where should sensate. be tested?
60
Answer: Median nerve. Sensation should be tested over the thumb & index fingers.

Fig 1
(d) If the distal radius fragment was displaced towards the palmer aspect what is the fracture called?
Answer: Smith's fractures involve palmar displacement of the distal radius fragment.
(e) What is the name of the classification system used for distal radius fractures?
Answer: The Frykman classification system.
‫ــــــــــــــــــــــــــــــــــــــــــــــــــــــــــــــــــــــــــــــــــــــــــــــــــــــــــــــــــــــــــــــــــــــــــــــــــــــــــــــــــــــــــــــــــــــــــــــــــــــــــــــــــــــــــــ‬
159- Question 41 of 50
An 18 yo woman presented with severe Rt iliac fossa pain. O/E she had a low grade fever.

(a) List 3 possible causes?


Answer: Appendicitis, cecal diverticulitis, meckel's diverticulitis ,acute ileitis,UTI & pelvic inflammatory disease.
(b) Name 3 non-imaging investigations which may be helpful?
Answer: A urinalysis is frequently obtained to rule out a UTI. The inflamed appendix is often in close proximity to
the bladder & ureter & as a result, microscopic hematuria & pyuria are found in up to 1/3 of Pts with acute
appendicitis. Pelvic cultures may be useful in sexually active, menstruating women. A beta-HCG is mandatory to rule
out an ectopic preg. Although leukocytosis is common 30% of Pts with acute appendicitis have a normal WBC count.
(c) Name 3 possible findings on plain radiograph in acute appendicitis?
Answer: Radiographic findings associated with acute appendicitis include, Rt lower quadrant appendicolith, localized
Rt lower quadrant ileus, loss of the psoas shadow, free air (occasionally), deformity of cecal outline & Rt lower
quadrant soft tissue density.
(d) Name 3 possible findings on standard CT scan in acute appendicitis?
Answer: Findings on standard CT scan which are consistent with a diagnosis of acute appendicitis include a thick
wall (>2 mm),appendicolith(seen in approximately 25% of Pts),target structure (concentric thickening of the inflamed
appendiceal wall),phlegmon,abscess,free fluid & fat stranding/Rt lower quadrant inflammation.
(e) Name two bacterial groups which should be covered if a perforated appendix is suspected?
Answer: In Pts with perforated appendicitis, the antibiotic regimen should cover enteric gram -ve rods & anaerobes.
‫ــــــــــــــــــــــــــــــــــــــــــــــــــــــــــــــــــــــــــــــــــــــــــــــــــــــــــــــــــــــــــــــــــــــــــــــــــــــــــــــــــــــــــــــــــــــــــــــــــــــــــــــــــــــــــــ‬
160- Question id: 2042

Fig 1
A lactating 38 yo woman (G1 P1) presented with a painful red area on her Rt. breast. On her initial presentation there
was no fluctuant mass palpable.
(a) What is the diagnosis? What is the most common differential diagnosis?
Answer: Mastitis. The most common differential diagnosis is plugged ducts. Plugged ducts usually present as
palpable lumps with tenderness without associated shooting pains or fever.
(b) Name two common aetiological agents?
Answer: Staphylococcus aureus, streptococcus, & Escherichia coli.
(c) Name 3 supportive measures used in the TTT of this condition?
Answer: Supportive measures include continued nursing, bed rest, NSAID such as ibuprofen for pain control.
(d) What is the initial antibiotic of choice for this condition? How long should antibiotic therapy continue for?
Answer: Antibiotic TTT should be started with flucloxacillin for 10 to 14 days.
(e) Despite antibiotic therapy the above lady represented 4 days later with a breast abscess. Name two risk
factors this lady has for the development of a breast abscess?
61
Answer: Risk factors for breast abscess formation include maternal age over 30 yrs of age, primiparity, gestational
age ≥ 41 weeks gestation, & mastitis.
‫ــــــــــــــــــــــــــــــــــــــــــــــــــــــــــــــــــــــــــــــــــــــــــــــــــــــــــــــــــــــــــــــــــــــــــــــــــــــــــــــــــــــــــــــــــــــــــــــــــــــــــــــــــــــــــــ‬
161- Question 47 of 50
A 46 yo man injured his Rt knee while pivoting during a Tae-kwon-do kick. There was a valgus, abducting stress on the knee. He
was tender over the medial aspect of the knee. There was pain with opening of the joint on valgus stress test of the knee.

(a) What is the diagnosis?


Answer: Medial collateral ligament injury.
(b) Name two other ligaments which provide support with valgus stress?
Answer: The ant. & post. cruciate ligaments also provide support with valgus stress & injuries to these ligaments may
accompany a MCL injury.
(c) How are these injuries classified?
Answer: First degree, second degree & third degree separations.
(d) Name four management steps?
Answer: Ice, elevation, crutches & activity limitation are advised during the first 7 to 14 days of therapy.
(e) What is the role of surgery in this condition?
Answer: In contrast to tears of the ant. or post. cruciate ligaments, surgery is rarely necessary for MCL repair, even in
Pts with third degree tears.
‫ــــــــــــــــــــــــــــــــــــــــــــــــــــــــــــــــــــــــــــــــــــــــــــــــــــــــــــــــــــــــــــــــــــــــــــــــــــــــــــــــــــــــــــــــــــــــــــــــــــــــــــــــــــــــــــ‬
162- Question 48 of 50
A 29 yo man presented with severe Rt sided flank pain which had occurred suddenly while driving to work on the
morning of presentation. He had no urinary symptoms & his only medical history was of depression. O/E he was
afebrile & his abd. was soft & non tender. Dipstick urine test revealed microcopic Bl. & a trace of protein.

(a) What is the most likely diagnosis?


Answer: Nephrolithiasis.
(b) What is the differential diagnosis for this condition? (Name four)
Answer: Renal cell carcinoma, aortic aneurysm , acute intestinal obstruct. or appendicitis & drug seeking behaviour.
(c) How would you investigate this Pt? (Name 3)
Answer: CT KUB(if unavailable IVP or plain film of the abd.)U/S if it's necessary to avoid radiat., UE, FBC & MSU.
(d) How would you manage this Pt?(Name four)
Answer: NSAIDs & opioids, consider calcium channel blockers or alpha blockers, treat infection if present & urology
consultation for pain that is not controllable in the ED or large stone size.
(e) What is the major determinant as to if this man will need a further procedure or can be managed
conservatively?
Answer: Stone size is the major determinant of the likelihood of spontaneous stone passage, although stone location
is also important. Most stones ≤4 mm in diameter pass spontaneously. For stones > 4 mm in diameter, there is a
progressive decrease in the spontaneous passage rate, which is unlikely with stones ≥10 mm in diameter.
‫ــــــــــــــــــــــــــــــــــــــــــــــــــــــــــــــــــــــــــــــــــــــــــــــــــــــــــــــــــــــــــــــــــــــــــــــــــــــــــــــــــــــــــــــــــــــــــــــــــــــــــــــــــــــــــــ‬
163- Question 49 of 50
A 14 yo girl has been assaulted by another girl in the park she comes in with epistaxis & a swollen nose. You suspect
a nasal fracture.

(a) What do you need to look for & document O/E?


Answer: Clinical diagnosis, no need to x-ray Ensure that can breathe through both nostrils Look for septal
haematoma as will need I&D (especially in children) Assess deviation (ask Pt to look in the mirror) NB is a head
injury, ensure that you check carefully for other facial/head injuries i.e. zygoma fracture.
(b) The nose is deviated, what do you do?
Answer: Give them an ENT follow up appointment for 5-7 days so that an MUA can be done within 10 days.
(c) She also complains of diplopia & on close inspect. she can’t look up properly on the Lt side, what is your
next course of action?
Answer: Arrange facial x-rays, need to consider orbital floor fracture
(d) Explain how you would manage prolonged epistaxis in an adult Pt?
Answer: Initial measures include ABC, Aggressive TTT of hypovolaemia, Check anticoagulant status if unsure or known to
be on anticoagulants then send Bl. for clotting, Pr. on the fleshy part of the nose for 10 min, Could then move on to cautery
with silver nitrate &/or put a cotton wall soaked in lidocaine with adrenaline in the nostril, never cauterize both sides
of the nasal septum, If continues insert nasal pack. Post. bleeding can respond to tamponade with a folley catheter.
‫ــــــــــــــــــــــــــــــــــــــــــــــــــــــــــــــــــــــــــــــــــــــــــــــــــــــــــــــــــــــــــــــــــــــــــــــــــــــــــــــــــــــــــــــــــــــــــــــــــــــــــــــــــــــــــــ‬
164- Question 4 of 10
An 18 yo man is brought to the ED after a hit & run. His GCS is 6/15 & the decision is made to intubate.
62
(a) What type of muscle relaxant is suxamethonium?
Answer: Depolarising muscle relaxant.
(b) What is the dose range for suxamethonium in adults?
Answer: 600micrograms-1mg/kg.
(c) In what circumstances is suxamethonium contraindicated? (Give two)
Answer: Hyperkalaemia, burns, paraplegia, crush injuries.
(d) What is the result of the administration of suxamethonium administration on intracranial pressure(ICP)?.
Answer: Suxamethonium causes a rise in ICP.
(e) What is the normal duration of action of suxamethonium?
Answer: About 5 minutes but longer in Pts with abnormal pseudo-cholinesterase enzymes.
‫ــــــــــــــــــــــــــــــــــــــــــــــــــــــــــــــــــــــــــــــــــــــــــــــــــــــــــــــــــــــــــــــــــــــــــــــــــــــــــــــــــــــــــــــــــــــــــــــــــــــــــــــــــــــــــــ‬
165- Question 8 of 10
A 70 yo lady fell on her outstretched Lt. wrist. X Ray revealed a colles fracture & it was decided to manipulate the
fracture using a biers block technique.

(a) What should be recorded before the pocedure? (Name two)


Answer: Pt consent, BP, how long the Pt is fasted, pre-op assessment, ECG, SpO2.
(b) What are the contraindications to biers block?(Name four)
Answer: Severe HTN, obesity, peripheral vascular disease, raynauds syndrome, methaemoglobinaemia, children < 7
yrs, sickle cell disease or trait, uncooperative or confused Pt. & procedures needed in both arms.
(c) What is the local anaesthetic of choice?
Answer: Prilocaine 0.5%
(d) How much above the systolic BP should the pressure cuff be inflated to?
Answer: The tourniquet should be inflated to at least 100mmHg above the systolic BP.
(e) At least how long must the tourniquet be inflated for?
Answer: The tourniquet must be inflated for at least 20 minutes.
‫ــــــــــــــــــــــــــــــــــــــــــــــــــــــــــــــــــــــــــــــــــــــــــــــــــــــــــــــــــــــــــــــــــــــــــــــــــــــــــــــــــــــــــــــــــــــــــــــــــــــــــــــــــــــــــــ‬
166- Question 10 of 10
A 26 yo man is brought to the ED by ambulance after a MVA. He has a low GCS & is intubated on arrival. The
decision is made to insert a CV line. During the attempted insertion the Pts O2 saturation begins to drop.

(a) What are the clinical signs to suggest a tension pneumothorax?(Give four)
Answer: Absent breath sounds on the affected side, hyper-resonance over the affected lung, distended neck veins,
tachycardia, hypotension & tracheal deviation.
(b) What would you do if you suspected a tension pneumothorax on the side of the central venous line?
Answer: Immediate decompression by inserting an IV cannula into the second intercostal space in the mid-clavicular
line just above the third rib.
(c) What is the next step if this initial TTT is successful?
Answer: Insertion of an axillary chest drain. Obtain a CXR.
(d) Why does tension pneumothorax cause cardiac arrest?
Answer: Movement of the mediastinum causes kinking of the great vessels & a decrease in venous return.
(e) If a Pt is receiving intermittent positive pressure ventilation (IPPV) what feature may cause the treating
clinician to suspect a tension pneumothorax?
Answer: A sudden increase in airway pressure.
‫ــــــــــــــــــــــــــــــــــــــــــــــــــــــــــــــــــــــــــــــــــــــــــــــــــــــــــــــــــــــــــــــــــــــــــــــــــــــــــــــــــــــــــــــــــــــــــــــــــــــــــــــــــــــــــــ‬
167- Question 5 of 10
An 18 yo man is brought to the ED after a hit & run. His GCS is 6/15 & the decision is made to intubate.

(a) What class of medication is thiopentone?


Answer: A barbituate.
(b) What are the effects of overdosage?
Answer: Hypotension, respiratory depression.
(c) What is the duration of action?
Answer: Duration of action of 5 to 10 minutes.
(d) What is the induction dose in an adult?
Answer: 4mg/kg
(e) What is the induction dose range in a child?
Answer: 2-7mg/kg.
‫ــــــــــــــــــــــــــــــــــــــــــــــــــــــــــــــــــــــــــــــــــــــــــــــــــــــــــــــــــــــــــــــــــــــــــــــــــــــــــــــــــــــــــــــــــــــــــــــــــــــــــــــــــــــــــــ‬

63
168- Question 2 of 5
A 35 yo man suffered a burns injury at work.

(a) Name four common signs of significant smoke inhalation injury?


Answer: Persistent cough, stridor or wheezing, hoarseness, deep facial or circumferential neck burns, nares with
inflammat. or singed hair, carbonaceous sputum or burnt matter in the mouth or nose, blistering or edema of the oropharynx,
depressed mental status, including evidence of drug or alcohol use, resp. distress & hypoxia or hypercapnia.
(b) Describe immediate burn care & cooling?(3 steps)
Answer: Any hot or burned clothing, any jewelry & any obvious debris should immediately be removed to prevent
further injury & to enable accurate assessment of the extent of injury, cool water or saline soaked gauze should be
applied. Ice & freezing should be avoided to prevent frostbite & systemic hypothermia.
(c) What points are important to attain in the history?(Four points)
Answer: What burned (eg, chemicals, textiles). The locat. of the fire (eg, enclosed or open space).Whether an explosion
occurred. Whether the Pt used alcohol or drugs. Whether there was associated trauma (eg, from falling debris)
(d) Describe burns in terms of superficial, superficial partial thickness, deep partial thickness & full thickness?
Answer: Superficial burns involve only the epidermal layer of skin. They are painful, dry, red & blanch with
pressure. Superficial partial-thickness burns involve the epidermis & superficial portions of the dermis. They are
painful, red & weeping, usually form blisters & blanch with pressure. Deep partial-thickness burns extend into the
deeper dermis, damaging hair follicles & glandular tissue. They are painful to pressure only. They almost always
blister (easily unroofed), are wet or waxy dry & have variable color from patchy cheesy white to red. Full-thickness
burns extend through & destroy the dermis. They are usually painless. Skin appearance can vary from waxy white to
leathery gray to charred & black. The skin is dry & inelastic & does not blanch with pressure.
(e) In an adult what percentage of total body surface area does each leg, arm, ant. & post. trunk & head
represent?
Answer: Each leg represents 18% TBSA, each arm represents 9% TBSA, the ant. & post. trunk each represent 18%
TBSA, & the head represents 9% TBSA.
‫ــــــــــــــــــــــــــــــــــــــــــــــــــــــــــــــــــــــــــــــــــــــــــــــــــــــــــــــــــــــــــــــــــــــــــــــــــــــــــــــــــــــــــــــــــــــــــــــــــــــــــــــــــــــــــــ‬
169- Question 1 of 20
A 69 yrs old lady is brought into the resuscitat. room. She is too unwell to give a history but her daughter tells you she
has complained abd. pain for the last 3 days. Over the last 24 hrs she has become increasingly unwell. Clinical exam.
reveals a tender abd. in the epigastrium & Rt upper quadrant. She is clearly icteric. PR 135, BP 88/45, temp 39.5, RR
35/min, Sats 98% (on high flow O2) Hb 12.2, WCC 21.9, Plt 290 Na 137, K 4.1, Cl 105, HCO3 12, Urea 10.0,
Creatinine 125 AST 56, Gamma GT 37, Alk Phosphate 742, total Billi 65, T. protein 65, Albumin 35
(a) What is the most likely diagnosis?
Answer: Biliary sepsis: Ascending cholangitis
(b) What secondary condition has arisen as a result?
Answer: Septic shock
(c) What 3 physical findings are classically described as Charcot’s triad?
Answer: Jaundice, Pain (R upper quadrant) & Fever
(d) Give 2 commonly encountered bacterial pathogens that you might expect to isolate in this case?
Answer: E. Coli Klebseilla Enterococcus Bacteroids sp.
(e) The Pt is being resuscitated with fluids & inotropes. IV antibiotics have been commenced. What imaging
modality would be most appropriate in this unstable Pt at this time, to confirm the clinical diagnosis?
Answer: Abd. USS
‫ــــــــــــــــــــــــــــــــــــــــــــــــــــــــــــــــــــــــــــــــــــــــــــــــــــــــــــــــــــــــــــــــــــــــــــــــــــــــــــــــــــــــــــــــــــــــــــــــــــــــــــــــــــــــــــ‬
170- Question id: 2102

Fig 1
A 23 yo male from Poland presents to the department in the early hrs. He has been badly beaten with a wooden club
to the face. He is stable but you suspect facial fractures.
64
(a) What are the important things to test for & document?
Answer: Sensation of the face (Anaesthesia over the region supplied by the infraorbital nerve (lower lid, cheek, side
of nose, upper lip, upper teeth & gums). Check for tenderness over the zygomatic arches, maxilla, mandible & TMJ
Assess mouth opening Look for bruising oedema, subcutaneous emphysema Nasal deviation Visual acuity Eye
movements any diplopia Uneven pupilary levels due to orbital floor damage) CSF rhinorrhoea Sunconjunctival
haemorrhage without a post. border (suggests an orbital wall or ant. cranial fossa fracture)
(b) What name is given to the fracture type shown in the figure 1?
Answer: Le Fort II
(c) The Pt has the fracture shown above, you notice that he has Bl. & clear fluid coming from his nostrils
known as the tramline effect. What do you need to do
Answer: Contact neurosurgeons immediately Ensure Pt doesn’t blow nose Give antibiotics Ensure TT prophylaxis
(d) He is unsure of his tetanus status what will you do?
Answer: If unsure then give a dose of combined DTP (ensure GP follow up) then make an assessment of if it is a
tetanus prone wound.
(e) What constitutes a tetanus prone wound?
Answer: Heavy contamination with soil or faeces, Devitalised tissue Infection, wounds >6hrs old, Puncture wounds
& animal bites
‫ــــــــــــــــــــــــــــــــــــــــــــــــــــــــــــــــــــــــــــــــــــــــــــــــــــــــــــــــــــــــــــــــــــــــــــــــــــــــــــــــــــــــــــــــــــــــــــــــــــــــــــــــــــــــــــ‬
171- Question id: 4544
A 21 yo female presents to the ED as she was sent by NHS direct due to tingling in her Rt arm, it seems to worsen
when she lifts her arm upwards.

(a) What does the CXR in figure 1 reveal?


Answer: Rt cervical rib
(b) How does this explain her symptoms?
Answer: Sometimes a cervical rib can press on the subclavian artery & brachial plexus causing transient vascular
insufficiency or paraesthesiae. This is known as thoracic outlet syndrome.
(c) What is the investigation of choice upon discovering this finding?
Answer: CT angiography with the arms down & the arms raised will show if there is any vascular compromise.
(d) In the ED what things do you need to arrange?
Answer: Although that vast majority of TOS is non-urgent a vascular & in this care arterial cause is more urgent as
the possibility of throwing off an embolus are increased. A vascular surgery consultation is needed. They may want
the Pt to be started on heparin & admitted?
(e) If this was to be addressed operatively what risks would you need to tell the Pt about?
Answer: Lymphocele & phrenic nerve damage are the main 2 significant ones.
‫ــــــــــــــــــــــــــــــــــــــــــــــــــــــــــــــــــــــــــــــــــــــــــــــــــــــــــــــــــــــــــــــــــــــــــــــــــــــــــــــــــــــــــــــــــــــــــــــــــــــــــــــــــــــــــــ‬
172- Question 5 of 20
The following CT scan (fig 1) was performed in the ED on a 75 yo man who had fallen earlier in the day. His GCS
was 13/15 due to him being confused & having his eyes shut.
(a) Describe the main findings (fig 1)
Answer: Large acute subdural haematoma on the Rt extending from the frontal lobe to the occipital lobe. Midline
shift & surrounding oedema.
(b) What is the most common intracranial injury post traumatic head injury?
Answer: Subdural haematoma
(c) What is the pathophysiology of the findings seen on the CT scan? (simple description required)
Answer: Essentially, injury to the bridging veins between the brain surface & the dura matter caused by shearing
forces that tear the veins.
(d) What is the approximately mortality for the condition seen on the CT scan?
Answer: Answer: 30-90% around 60% is typical1
(e) What is the definitive management?
Answer: Surgery for emergent decompression has been advocated if the acute subdural hematoma is associated with
a midline shift greater than or equal to 5 mm. Surgery also has been recommended for acute subdural hematomas
65
exceeding 1 cm in thickness. These indications have been incorporated into the Guidelines for the Surgical
Management of Acute Subdural Hematomas proposed by a joint venture between the Brain Trauma Foundation & the
Congress of Neurological Surgeons released in 2006.
‫ــــــــــــــــــــــــــــــــــــــــــــــــــــــــــــــــــــــــــــــــــــــــــــــــــــــــــــــــــــــــــــــــــــــــــــــــــــــــــــــــــــــــــــــــــــــــــــــــــــــــــــــــــــــــــــ‬
173- Question 6 of 20
A fit & well 68 yo lady is brought in to the ED early in the morning. She slipped whilst getting out of the shower &
now has a very painful Lt hip. She thinks that she has dislocated her prosthesis again. X-rays confirm that the
prosthesis has dislocated posteriorly.
(a) What is the best method to reduce the hip?
Answer: There are no RCTs to address this quest. but recent Best Bets would suggest that sedat. with propofol in the
ED in the Rt hands has got a very good success rate. This is highly dependant on the doctor having the necessary
anaesthetic experience to manage any complicat. of propofol sedat. The alternative is reduct. under GA.
(b) What important questions do you need to ask in the history before considering the above?
Answer: This question is trying to test the candidate's ability to think ahead. It is important to ensure that the Pt is
starved if you are going to use propofol sedation.
(c) What is it paramount to test prior to considering reduction?
Answer: The function of the sciatic nerve. The finding of sciatic nerve dysfunction mandates surgical exploration to
release or repair the nerve.
(d) What method would you use to reduce the hip?
Answer: Allis method The Pt should be supine & under procedural sedat. The combined weight of the Pt & physician
may exceed the weight limit of the stretcher. It is generally unsafe for the physician to be standing on a stretcher. For
these reasons, placing the Pt on the floor rather than on the stretcher is often useful. An assistant should stabilize the
pelvis. The physician should initially be toward the Pt's feet, providing in-line tract. The physician should then gently
flex the hip 60-90o while maintaining in-line tract. At this point, the physician is standing directly above the Pt's hip,
providing traction in-line with the deformity. Gently adducting the hip can force the head of the femur laterally & help
it clear the acetabular rim. Alternately, gentle lateral traction can be applied to the proximal femur. Reduction can be
confirmed by a click that is felt & may be heard as well. The Pt should assume normal anatomical position.
Stimson method: This method is mechanically the same as the Allis method, but the positioning is opposite.
Although some physicians prefer this method because of its technical ease & high success rate, this method has some
important disadvantages. It requires the Pt to be in a prone posit., which may not be possible for the Pt with multiple
trauma. Monitoring the Pt during procedural sedat. is also difficult. The prone Pt is placed so the pelvis on the
affected side hangs either over the end or over the side of the stretcher. The hip & knee are flexed to 90o. Downward
pressure is applied to the popliteal fossa, providing traction in-line with the deformity. An assistant stabilizes the
pelvis & trunk preventing the Pt from being pulled off the stretcher.
Whistler technique: The Pt is placed supine with ipsilateral knee flexed to 120o. The physician stands on the
affected side & places an arm under the ipsilateral knee with his or her hand resting on the contralateral knee. The
pelvis & ankle are stabilized by an assistant or the physician's free hand. The physician raises his or her arm, which
applies an ant. force to the knee & subsequently to the affected hip. Ant. dislocations A modified Allis technique may
be used. The Pt is placed supine. The physician stands at the foot of the stretcher. Traction is applied to a neutral hip
while an assistant stabilizes the pelvis. Gentle lateral traction applied to the proximal femur facilitates the femoral
head clearing the acetabular rim.
(e) What should the further care post reduction include?
Answer: After reduct. of the hip dislocate., obtain repeat AP & lateral radiographs of the hip, as well as repeat CT scans
or MRIs of the hip to verify proper reduct. After either open or closed reduct. of a hip dislocat., the Pt is instructed to
remain on bed rest with his or her legs abducted & with skeletal traction designed to keep the hip from displacing
post.ly. The durat. of tract. is approximately 2 wks, but the recommended period with no weight bearing is
controversial & varies from 9 days to 3 months.
‫ــــــــــــــــــــــــــــــــــــــــــــــــــــــــــــــــــــــــــــــــــــــــــــــــــــــــــــــــــــــــــــــــــــــــــــــــــــــــــــــــــــــــــــــــــــــــــــــــــــــــــــــــــــــــــــ‬
174- Question 7 of 20
A 52 yo man presented with ant. shoulder pain which had come on over the previous 24 hrs since a trip to the
gymnasium in which he had been using various exercise machines. The pain was agravated by lifting & overhead
reaching. He was tender in the bicipital groove.
(a) What is the most likely diagnosis?
Answer: Bicipital tendonitis.
(b) What clinical test may be used to aid in the diagnosis?
Answer: Yergason's test This test of supination against resistance is positive if it elicits pain in the bicipital groove.
Also the pain of bicipital tendonitis is frequently aggravated by the painful arc maneuver.
(c) What is the role of plain X-Rays in this condition?
66
Answer: Plain x-rays of the shoulder (including PA, external rotation, Y-outlet & axillary views) are not necessary in
most Pts with bicipital tendonitis.
(d) How would you manage this Pt?
Answer: Ice , advise Pts to eliminate lifting & restrict over-the-shoulder positions & reaching, weighted pendulum
stretching exercise for 5 to 10 min. a day acutely, & then 3 times/wk as symptoms improve to reduce the chance of
recurrent tendonitis, isometric toning exercises of elbow flexion should begin 3 to 4 wks after the acute pain has
resolved. If symptoms persist then a corticosteroid injection or orthopaedic referral should be considered.
(e) The Pt went on to develop a lump just proximal to the antecubital fossa. What are the risk factors for this
complication? (Name 3)
Answer: Risk factors for rupture include recurrent tendonitis, previous rotator cuff or contralateral biceps tendon
rupture, age greater than 50, poor general shoulder conditioning & rheumatoid arthritis.
‫ــــــــــــــــــــــــــــــــــــــــــــــــــــــــــــــــــــــــــــــــــــــــــــــــــــــــــــــــــــــــــــــــــــــــــــــــــــــــــــــــــــــــــــــــــــــــــــــــــــــــــــــــــــــــــــ‬
175- Question id: 2038
A 19 yo man had fallen on his Rt shoulder while playing soccer.His X Ray is shown.

Fig 1
(a) What is the diagnosis?
Answer: Ant. shoulder dislocation.
(b) Name 3 findings O/E?
Answer: An anteriorly dislocated shoulder causes the arm to be slightly abducted & externally rotated. The Pt resists
all movement. The acromion appears prominent in thin individuals & there is loss of the normal rounded appearance
of the shoulder. Axillary nerve dysfunction manifests as loss of sensation in a "shoulder badge" distribution, although
this finding is not reliably present.
(c) Name two factors associated with fracture?
Answer: Factors associated with fracture include age over 40, 1st time dislocate. & traumatic mechanism (eg, fight or fall)
(d) Describe what finding would be expected on the Y radiographic view?
Answer: When an ant. dislocation is present, the humeral head appears medial to the "Y".
(e) Name two associations of post. shoulder dislocations?
Answer: Violent muscle contract. following a seizure or electrocution represent common causes of post. shoulder dislocat.
‫ــــــــــــــــــــــــــــــــــــــــــــــــــــــــــــــــــــــــــــــــــــــــــــــــــــــــــــــــــــــــــــــــــــــــــــــــــــــــــــــــــــــــــــــــــــــــــــــــــــــــــــــــــــــــــــ‬
176- Question 9 of 20
A 38 yo man was punched in the face last night. He has come in today as he cannot see properly in his Lt eye it keeps
going blurry & he is seeing double.
(a) Explain how you would test visual acuity, write down how you would record it?
Answer: Use a Snellen chart 6 metres away from the Pt get them to read off the chart covering one eye at a time,
instruct them to go down the chart until they cannot read the letters any more. The line they reach will determine their
acuity i.e. 6/12 or 6/5 (best it could be) If Pts read additional letters form the line below record it as such: 6/12 +2.
(b) His Rt eye appears to be slightly sunken & he had a subconjunctival haemorrhage. What do you need to
establish with regard to the subconjunctival haemorrhage?
Answer: Can you see the back of it? If not then it could represent an orbital wall fracture or an ant. cranial fossa fracture.
(c) When you assess eye movements what are you looking out for?
Answer: Restriction of upward gaze due to the inferior rectus muscle being trapped in the broken orbital floor.
(d) What is the tear drop sign seen on facial x-rays?
Answer: It represents soft tissue mass in the top of the maxilla. sinus from muscle that has slipped down through the orbital floor
‫ــــــــــــــــــــــــــــــــــــــــــــــــــــــــــــــــــــــــــــــــــــــــــــــــــــــــــــــــــــــــــــــــــــــــــــــــــــــــــــــــــــــــــــــــــــــــــــــــــــــــــــــــــــــــــــ‬
177- Question 14 of 20
A 25 yo woman presented to the ED after coming home from a sking holiday. She had fallen on her last day & had
persistent pain in her Rt hand at the base of her thumb. There was point tenderness over the ulnar side of the
67
metacarpophalangeal joint of the thumb.
(a) What is the diagnosis?
Answer: Ulnar collateral ligament injury (gamekeeper's thumb or skier's thumb).
(b) What is the mechanism of injury?
Answer: Forced radial deviation of the thumb.
(c) How is this condition tested for clinically?
Answer: Valgus stress testing determines the irritat. & integrity of the ulnar collateral ligament (stress is applied across
the MP joint to the collateral ligament located on the ulnar side of the thumb [ie the thumb is pushed away from the palm]).
(d) Why is this condition significant?
Answer: The strength or holding power of the thumb & first finger may be compromised.
(e) How should this Pt be treated if X Ray is unremarkable but there is clinical evidence of a severe injury?
Answer: Referral to an orthopedist or hand surgeon is indicated for Pts with evidence of complete ligament tear. For
lesser injuries a thumb spica splint or a dorsal hood splint for approximately 3 weeks is appropriate.
‫ــــــــــــــــــــــــــــــــــــــــــــــــــــــــــــــــــــــــــــــــــــــــــــــــــــــــــــــــــــــــــــــــــــــــــــــــــــــــــــــــــــــــــــــــــــــــــــــــــــــــــــــــــــــــــــ‬
178- Question id: 2106
A 23 yo university rugby player was tackled whilst leaping to touch down a try he was airlifted by 2 players & landed
with some force on his Lt shoulder.

Fig 1 Fig 2Fig 3


(a) What is shown in the radiograph (fig 1)?
Answer: AC joint dislocation, grade III. The AC ligament is ruptured along with the conoid & trapezoid ligaments.
(b) How would you manage this?
Answer: Good analgesia broad arm sling & orthopaedic follow up, may require internal fixation. Type IV-VI:
Account for more than 10-15% of total acromioclavicular dislocat. & should be managed surgically. Failure to reduce
& fix these will lead to chronic pain & dysfunct. Type III the acromioclavicular joint capsule & coracoclavicular
ligaments are completely disrupted. The coracoclavicular interspace is 25-100%>the normal shoulder. Type IV This is
a type III injury with avulsion of the coracoclavicular ligament from the clavicle, with the distal clavicle displaced
posteriorly into or through the trapezius. Type V This is type III but with exaggerate. of the vertical displacement of
the clavicle from the scapula-coracoclavicular interspace 100-300%>the normal side, with the clavicle in a SC
position. Type VI This is a rare injury. This is type III with inf. Dislocat. of the lateral end of the clavicle below the coracoid.
(c) If a Pt sustains a fracture of the scapular what should you do?
Answer: Check for other injuries, it takes considerable force to fracture the scapula so look for rib fractures &
particularly any evidence of a lung contusion.
(d) What does the radiograph in figure 2 show?
Answer: Fracture of the humeral head, 3 part fracture
(e) What does the radiograph in figure 3 show? How would you manage this in the ED?
Answer: First do no harm! Do not pull this! You will make it worse, will need ORIF. Give good analgesia & support
in sling Answer: Salter Harris 4 fracture of the distal tibia, medial malleolus.
‫ــــــــــــــــــــــــــــــــــــــــــــــــــــــــــــــــــــــــــــــــــــــــــــــــــــــــــــــــــــــــــــــــــــــــــــــــــــــــــــــــــــــــــــــــــــــــــــــــــــــــــــــــــــــــــــ‬
179- Question 16 of 20
A 40 yo woman presented with severe abd. pain. This had come on gradually over the course of the previous 24 hrs.
O/E her vital signs were within the normal range & her abd. was soft. Bl. investigations revealed a serum amylase
level which was five times the upper limit of normal. A diagnosis of pancreatitis was made.
(a) What is the most common cause of this condition in women?
Answer: Women-Gallstone pancreatitis.Men-Alcohol.
(b) Name five findings which may be present O/E in this condition?
Answer: Tachycardia, fever, shock, abd. tenderness, abd. distention, guarding, ecchymotic discoloration in the flank
(Grey-Turner's sign) or the periumbilical region (Cullen's sign), an epigastric mass due to pseudocyst formation may
become palpable in the course of the disease.
68
(c) How quickly does serum amylase rise in acute pancreatitis?
Answer: Serum amylase  within 6-12 hrs of onset. In uncomplicated attacks, serum amylase is usually  for 3 to 5 days
(d) What other conditions may cause a rise in serum amylase?(Give four conditions)
Answer: Post-ERCP, acute cholecystitis, parotitis, intestinal trauma, intestinal surgery, intestinal obstruction,
intestinal infarction, alcoholism, anorexia, cirrhosis, ruptured ectopic pregnancy, salpingitis, renal failure, acidosis &
malignancy with ectopic amylase production.
(e) What is the role of CXR is this Pt?
Answer: Approximately 1/3 of Pts with acute pancreatitis have abnormalities visible on the chest X Ray such as  of
a hemidiaphragm, pleural effusions, basal atelectasis, pulmonary infiltrates, or acute respiratory distress syndrome.
‫ــــــــــــــــــــــــــــــــــــــــــــــــــــــــــــــــــــــــــــــــــــــــــــــــــــــــــــــــــــــــــــــــــــــــــــــــــــــــــــــــــــــــــــــــــــــــــــــــــــــــــــــــــــــــــــ‬
180- Question 17 of 20
A 39 yo man comes into the department in triple immobilisation with a hard collar, blocks & tape. He was the driver
in a rear end shunt a few hrs earlier. He walked from the vehicle, which was hit at around 30 m.p.h.
(a) According to the Canadian c-spine rules what are the 3 high risk factors that mandate radiography?
Answer: 1. Any dangerous mechanism of injury 2. Any paraesthesia in the extremities 3. Age >65
(b) Over what GCS range is it acceptable to apply the Canadian c-spine rules?
Answer: Only validated when GCS is 15/15
(c) He doesn't fit any of the high risk factors. Name as many of the low risk factors, which if present would
mean you could assess the range of movement in the neck. (3 marks if all 6, 2 marks 4-5, 1 mark 2-3)
Answer: Sitting in the ED -Simple rear end shunt* (excludes roll-over, hot by large truck, high speed crash, pushed into
traffic)-Ambulatory at any time-Delayed onset of neck pain (not immediate)-Absence of midline c-spine tenderness
(d) You decide that it is safe to ask him to rotate his neck. However he can't move his neck to 45 degrees. What
do you do now?
Answer: Needs x-ray
(e) Do you know of any other validated rules for assessing the need for radiography with regard to neck
injuries in the alert & stable trauma Pt?
Answer: The NEXUS rules.
‫ــــــــــــــــــــــــــــــــــــــــــــــــــــــــــــــــــــــــــــــــــــــــــــــــــــــــــــــــــــــــــــــــــــــــــــــــــــــــــــــــــــــــــــــــــــــــــــــــــــــــــــــــــــــــــــ‬
181- Question id: 2014

A 55 yrs old lady presents with a 2 day history of abd. pain. Clinically she is mildly tachypnoeic & has a sinus
tachycardia. Abd. exam. confirms generalised tenderness.
(a) What is the radiological diagnosis (see fig 1)?
Answer: Small bowel obstruction
(b) Name 3 symptoms associated with this x-rays appearance.
Answer: Persistant vomiting / bilious vomiting, Colicky abd. Pain, Abd. Distension & Absolute constipation
(c) Name 3 potential causes of this x-rays appearance.
Answer: Adhesions, Hernia, Intraluminal obstruction (e.g. gallstones, or food bolus), Stricture & Neoplasm
(d) Give 3 steps in the early management of this Pt. Post immediate ABCD assessment, assume O2 has been
started
Answer: Nil by mouth / NGT, IV fluid resuscitation, Analgesia, Urinary catheter, General surgical referral & Bl. tests
‫ــــــــــــــــــــــــــــــــــــــــــــــــــــــــــــــــــــــــــــــــــــــــــــــــــــــــــــــــــــــــــــــــــــــــــــــــــــــــــــــــــــــــــــــــــــــــــــــــــــــــــــــــــــــــــــ‬
182- Question 20 of 20
A 26 yo presents to the ED after being hit in the face by a baseball bat. The treating clinician suspects a facial bone fracture

(a) What are the imporant points on inspection of the facial bones?(Give four)
69
Answer: Asymmetry, flattening of the cheek suggests a depressed zygomatic fracture, a flattened & elongated face
may be due to post. & downward displacement of the maxilla (the so called dish face deformity), nasal deviation,
saddle deformity, an orbital floor fracture may cause uneven pupil levels, CSF rhinorrhoea, subconjunctival
haemorrhage without a post. border.
(b) If there is hypo/anaesthesia of the cheek , side of the nose & upper lip which nerve may be affected?
Answer: Infraorbital nerve.
(c) What is the significance of subcutaneous emphysema in this Pt?
Answer: Subcutaneous emphysema suggests a compound fracture often of the maxillary sinus.
(d) If a mandibular fracture is suspected what X Ray should be requested?
Answer: Orthopantomogram.
(e) If there is no evidence of facial fracture on X Ray but there is a strong clinical suspicion of facial fracture
how would you proceed?
Answer: Expert consultation or follow up.
‫ــــــــــــــــــــــــــــــــــــــــــــــــــــــــــــــــــــــــــــــــــــــــــــــــــــــــــــــــــــــــــــــــــــــــــــــــــــــــــــــــــــــــــــــــــــــــــــــــــــــــــــــــــــــــــــ‬
183- Question 5 of 20
A 36 yo man was hit by a car. He complained of a painful Rt knee. X Ray revealed a tibial plateau fracture.
(a) Which side of the plateau is usually involved?
Answer: Tibial plateau fractures most commonly involve the lateral plateau after a direct blow that produces a strong
force to the lateral knee.
(b) Besides an AP film which other films should be requested?
Answer: Lateral & oblique.
(c) What is the typical finding on radiography?
Answer: Radiographs typically reveal a depression of the lateral tibial plateau in moderate to severe fractures.
(d) When clinical suspicion is high but radiographs are equivocal how should the clinician proceed?
Answer: Further imaging should be with CT or MRI.
(e) Name four management steps?
Answer: Compression, icing, knee splinting in full extension, elevation,orthopaedic referral & strict non-weight
bearing are the initial phase of TTT of a tibial plateau fracture.
‫ــــــــــــــــــــــــــــــــــــــــــــــــــــــــــــــــــــــــــــــــــــــــــــــــــــــــــــــــــــــــــــــــــــــــــــــــــــــــــــــــــــــــــــــــــــــــــــــــــــــــــــــــــــــــــــ‬
184- Question id: 2071
A 60 yo woman presented with a 2 day history of abd. pain & bloating. She had not passed a bowel motion for 24 hrs
& felt nauseated. Her past medical history was signoficant for an appendicetomy & a cholecystectomy. O/E she was
febrile at 38degrees & her abd. was bloated & diffusely tender.

Fig 1
(a) What is the diagnosis?
Answer: Small bowel obstruction.
(b) What is the differential diagnosis?(Give two)
Answer: Intestinal pseudo-obstruction & paralytic ileus.
(c) What are the causes of this condition?(Give five causes)
Answer: Adhesions, hernia, volvulus, congenital malformat. Duplicate., atresia, stenosis, neoplasm, inflammatory
stricture, radiat. enteritis, intussusception, gallstones, feces or meconium, bezoar, & traumatic intramural hematoma.
(d) How would you further investigate this Pt?(Name four)
Answer: Urea & creatinine & the hematocrit can be used to gauge the degree of dehydrate. Leukocytosis with Lt
ward shift may be present. Metabolic alkalosis can be seen in Pts who have frequent emesis. Metabolic (lactic)
acidosis can result if the bowel becomes ischemic or if dehydrate. is severe enough to cause hypoperfusion of the gut
70
& other tissues. Serum lactate is found to be elevated in Pts with mesenteric ischemia & is a sensitive. CT abd. may
give more information on the level & cause of the obstruction
(e) How would you manage this Pt?(Name 3 steps)
Answer: Nasogastric tube, IV fluids, surgical consultation
‫ــــــــــــــــــــــــــــــــــــــــــــــــــــــــــــــــــــــــــــــــــــــــــــــــــــــــــــــــــــــــــــــــــــــــــــــــــــــــــــــــــــــــــــــــــــــــــــــــــــــــــــــــــــــــــــ‬
185- Question 13 of 20
A 44 yo presents with a short history of a Rt sided neck swelling that seems to be going up & down. It is quite tender
& it seems to be most prominent when he eats. O/E he appears well, he is afebrile & there are no skin changes. The
mass appears to be just below the angle of the Rt mandible & is approximately 3cm by 2cm.

(a) What further clinical exam. would you perform in the ED?
Answer: Full exam. of the oral cavity & neck. Need to look inside the mouth the check for any lesions/salivary
calculi/ signs of tooth decay/infection/. Also full systemic exam. to ensure no signs of sepsis.
(b) What investigations should you perform?
Answer: If you think that it is infected then it might be worth doing some baseline inflammatory markers & of course
Bl. cultures. Organise an OP silaogram through the maxillofacial team.
(c) What is the most likely diagnosis given the above history?
Answer: submandibular calculus.
(d) What are the potential TTTs?
Answer: Gentle probing into the duct from inside the mouth with a thin blunt instrument can sometimes free a stone
which then falls into the mouth. This is done by a doctor. Therapeutic sialendoscopy. This is a similar procedure to
that described above. It also uses a very thin endoscope (tube) with a camera & light at the tip. The tube is pushed into
the duct. If a stone is seen, then a tiny 'basket' or pair of 'grabbers' that are attached to the tube is used to grab the stone
& pull it out. This technique can successfully remove about 17 in 20 stones. Local anesthetic is usually injected into
the duct first to make this procedure painless. In some cases, where the stone is rather large, the stone is broken up
first & then the fragments are pulled out. A small operation to cut out the stone is the traditional TTT, but is done less
& less as therapeutic sialendoscopy has become available. It may still be needed if therapeutic sialendoscopy is not an
available option, or if it fails. 'Shock wave' TTT (lithotripsy) may be an option. This uses ultrasound waves to break
up stones. The broken fragments then pass out along the duct. This is a relatively new TTT for salivary stones
(although it has been used for some yrs to treat kidney stones). However, it is not done commonly. Sometimes shock
waves are used to break up a large stone when therapeutic sialendoscopy is done to make smaller fragments which
can be more easily removed.
‫ــــــــــــــــــــــــــــــــــــــــــــــــــــــــــــــــــــــــــــــــــــــــــــــــــــــــــــــــــــــــــــــــــــــــــــــــــــــــــــــــــــــــــــــــــــــــــــــــــــــــــــــــــــــــــــ‬
186- Question id: 4532
A 24 yo drug dealer has been shot in the abd., he comes in the resus room & is haemodynamically unstable. You have
been pre-alerted.

(a) As part of your primary assessment B seems normal clinically to you, although you have not yet seen the
back. In E what do you need to look for?
Answer: This question is stressing the importance of fully looking around the body including the back/loin/groin &
sides for entry & exit points of the bullets. An abd. emergency could fast become a thoraco-abd. emergency if what
first seems like a isolated abd. gun shot wound turns out to have an exit point high in the back of the chest.
(b) Where is this Pt going & what measures need to occur prior to that?
Answer: Needs urgent laparotomy, Needs good IV access first ideally central lines & arterial lines.
(c) The Pt is bleeding from the wound site- the Hb on the initial gas is 6.9. What will you do?
Answer: The Pt will need transfusion of whole Bl., FFP, cryoprecipitate & consideration should be given to rV111a,
evidence is emerging stating that giving it early although not yet proven to decrease mortality has been shown to
reduce ICU days, reduce the amount of Bl. required etc.
(d) What is it important to remember to give in all cases?
Answer: Tetanus prophylaxis & anaerobic antibiotic cover.
‫ــــــــــــــــــــــــــــــــــــــــــــــــــــــــــــــــــــــــــــــــــــــــــــــــــــــــــــــــــــــــــــــــــــــــــــــــــــــــــــــــــــــــــــــــــــــــــــــــــــــــــــــــــــــــــــ‬
187- Question id: 2039
A 28 yo man fell while ice skating with his son. His X Ray is shown below.
(a) What is the diagnosis?
Answer: Scaphoid Fracture.
(b) Name two mechanisms of injury?
Answer: Fractures of the scaphoid can occur either with direct axial compression or with hyperextension of the wrist
such as a fall on the outstretched hand
(c) Name two findings O/E?
71
Answer: Grip strength is typically reduced. On palpation pain is typically in 1 of 3 places: The volar prominence at
the distal wrist crease for distal pole fractures; in the anatomical snuff box for waist fractures & just distal to Lister's
Tubercle (a longitudinal bony prominence on of the distal radius just to the ulnar side of the extensor carpi radialis
tendon) for proximal pole fractures.
(d) Name 3 radiographs which should be requested if this injury is suspected?
Answer: For suspected scaphoid fractures, standard radiographs include a PA, true lateral & a scaphoid view. The
scaphoid view is made in PA format with the wrist in full pronation & ulnar deviation.
(e) What findings on a lateral radiograph are suggestive of this condition?
Answer: The scapholunate angle is formed by a line bisecting the scaphoid in its longitudinal axis & a line bisecting
the lunate. This angle should be between 40 & 60o. More or lesser degrees indicate ligamentous instability &/or fracture.
‫ــــــــــــــــــــــــــــــــــــــــــــــــــــــــــــــــــــــــــــــــــــــــــــــــــــــــــــــــــــــــــــــــــــــــــــــــــــــــــــــــــــــــــــــــــــــــــــــــــــــــــــــــــــــــــــ‬
188- Question 19 of 20
A 32 yo woman was bitten by a dog 2 days prior to presentation. She had attended her primary care physician who
had treated her with anti-tetanus prophylaxis but had not prescribed any anti-biotics.O/E there were bite marks on
either side of the thenar eminence of the Rt hand. The area was red, swollen & tender. She was afebrile.
(a) Name two bacterial classes that may have caused this wound infection?
Answer: The predominant organisms in animal bite wounds are the oral flora of the biting animal (notable pathogens
include Pasteurella, Capnocytophaga & anaerobes) as well as human skin flora (such as staph. & streptococci).
(b) Name 3 possible complications of this wound infection?
Answer: Subcutaneous abscesses, associated crush injury, osteomyelitis, tenosynovitis & septic arthritis.
(c) Name 3 management steps?
Answer: TTT of animal bites includes wound care, antibiotic therapy, vaccinat., radiographic imaging & surgical evaluat.
(d) What antibiotics should be started in this lady?
Answer: Options for empiric gram--ve & anaerobic coverage include 1.Monotherapy with a beta-lactam/beta-
lactamase inhibitor, such as one of the following: Ampicillin-sulbactam (3 g every six hrs) , Piperacillin/tazobactam
(4.5 g every eight hrs) , Ticarcillin-clavulanate (3.1 g every four hrs) , or 2. A third generation cephalosporin such as
ceftriaxone (1 g IV every 24 hrs) PLUS metronidazole (500 mg IV every eight hrs).
(e) Name two circumstances where rabies vaccination should be considered?
Answer: Rabies prophylaxis should be considered in the setting of bites from unvaccinated pets, wild animals & in
geographic areas where the prevalence of rabies is high.
‫ــــــــــــــــــــــــــــــــــــــــــــــــــــــــــــــــــــــــــــــــــــــــــــــــــــــــــــــــــــــــــــــــــــــــــــــــــــــــــــــــــــــــــــــــــــــــــــــــــــــــــــــــــــــــــــ‬
189- Question id: 2066
A 56 yo man presented with a severe headache which had a sudden onset. His CT Brain scan is shown.

(a) What is the diagnosis?


Answer: SAH
(b) Name four risk factors for this condition?
Answer: Smoking, HTN, alcohol, family history, phenylpropanolamine in appetite suppressants & oestrogen deficiency
(c) How would you manage this Pt?
Answer: Neurosurgical consultation, intensive care setting for constant hemodynamic & cardiac monitoring, stool
softeners, bedrest, analgesia to diminish hemodynamic fluctuations & lower the risk of rebleeding & pneumatic
compression stockings to limit risk of deep vein thrombosis should be utilized while Pts are immobile. Antithrombotic
discontinuation, ICP monitoring & nimodipine therapy.

Fig 1
(d) What are the complications of this condition?(Name four)
72
Answer: Rebleeding, vasospasm & delayed cerebral ischemia, hydrocephalus, increased intracranial pressure,
seizures, hyponatremia, cardiac abnormalities, hypothalamic dysfunction & pituitary insufficiency.
(e) Name two prognostic factors?
Answer: 1.Level of conscious. & neurologic grade on admission, 2.Pt age, 3.Amount of Bl. on initial head CT scan.
‫ــــــــــــــــــــــــــــــــــــــــــــــــــــــــــــــــــــــــــــــــــــــــــــــــــــــــــــــــــــــــــــــــــــــــــــــــــــــــــــــــــــــــــــــــــــــــــــــــــــــــــــــــــــــــــــ‬
190- Question 15 of 20
A 24 yo man was brought to the ED after being mugged. He was stabbed in the abd.. His Bl. Pr. was 140/80 mmHg &
his heart rate was 82bpm. Exam. revealed a stab wound superior to his umbilicus.
(a) In relation to the stabbing instrument what points are important in the history? (Four points)
Answer: What instrument was used, how long it was, how wide it was, how he was positioned during the stabbing &
what path the implement traveled.
(b) How would you investigate this Pt?
Answer: Local wound exploration, FBC, UE, Bl. group & hold, CT scan.
(c) How would you manage this Pt? (Four points)
Answer: Provide initial resuscitation based upon protocols from Advanced Trauma Life Support, Monitored bed, two
wide bore IV lines, IV fluids, surgical consultation.
(d) Name two points in the exam. that if present are strong indicators for urgent laporotomy?
Answer: Immediate laparotomy was traditionally indicated in the presence of hemodynamic instability, evisceration,
or unequivocal peritoneal signs on physical exam. Others are signs of GIT hemorrhage & an implement in situ.
(e) What is the role of plain radiographs in this Pt?
Answer: Plain film radiographs add little to the management of stab wounds.
‫ــــــــــــــــــــــــــــــــــــــــــــــــــــــــــــــــــــــــــــــــــــــــــــــــــــــــــــــــــــــــــــــــــــــــــــــــــــــــــــــــــــــــــــــــــــــــــــــــــــــــــــــــــــــــــــ‬
191- Question id: 2037
A 30 yo man presented with severe Lt flank pain,nausea, & difficulty urinating.He had microscopic Bl. in his urine.
(a) List 2 possible diagnosis?
Answer: Renal colic, pyelonephritis & renal cell carcinoma.
(b) Name 3 risk factors for nephrolithiasis?
Answer: For calcium stones, urinary risk factors include hypercalciuria, hyperoxaluria, hyperuricosuria,
hypocitraturia, & dietary risk factors such as a low calcium intake, high oxalate intake, high animal protein intake,
high sodium intake, or low fluid intake. A history of prior nephrolithiasis, Pts with a family history of stones have an
increased risk of nephrolithiasis, frequent upper UTIs , & HTN.
(c) Name two complications of nephrolithiasis?
Answer: Nephrolithiasis may lead to persistent renal obstruction, staghorn calculi, & infection.
(d) Name one type of stone which is radiolucent on abd. X Ray?
Answer: Uric acid stone.
(e) What investigation should be used in pregnant Pts?
Answer: Ultrasound is the initial diagnostic test in pregnant women or in Pts in whom cholecystitis or a gynecologic
process is a prominent consideration.
‫ــــــــــــــــــــــــــــــــــــــــــــــــــــــــــــــــــــــــــــــــــــــــــــــــــــــــــــــــــــــــــــــــــــــــــــــــــــــــــــــــــــــــــــــــــــــــــــــــــــــــــــــــــــــــــــ‬
192- Question 12 of 50
A 29 yo who has recently had a baby & is breast-feeding comes to see you as she has developed diarrhoea & is concerned

(a) She takes oral aminophyline for asthma. Which antibiotic should you avoid in this Pt?
Answer: Ciprofloxacin&erythromycin are both liver enzyme inhibitors&can therefore ↑plasma concentrate. of theophyline
(b) She came off lithium prior to conception but is worried about her depression what do you advice?
Answer: Cannot go on it whilst still breast feeding due to risks to baby of involuntary movements.
(c) What drug can cause cleft lip & palate if taken during pregnancy? Which drug should be used instead?
Answer: Phenytoin. Monotherapy with carbamazepine is probably the safest. Risks of major congenital
malformations related to specific anti-epileptic drugs Carbamazepine taken as a single drug TTT (known as
monotherapy) carries the lowest risk, with 2.2% Taking Na valproate as monotherapy at a daily dosage under
1000mg, carries a risk of 5.1% Taking Na valproate as monotherapy at daily doses over 1000mg carries a risk of 9.1%
Drug combinat. that include Na valproate have a significantly higher risk of MCMs than combinations that don't
include this drug. Taking lamotrigine as monotherapy at daily dosages of 200 mg or less carries a risk of 3.2% Taking
lamotrigine as monotherapy at a daily dosage above 200 mg carries a risk of 5.4% Taking carbamazepine & Na
valproate together carries a risk of 8.8% Taking Na valproate & lamotrigine together carries a risk of 9.6% The
information from the study didn't include any specific data on vigabatrin, gabapentin, topiramate, tiagabine,
oxcarbazepine, levetiracetam & pregabalin.1
(d) She is sexually active again & doesn t want to conceive what advice do you give regarding contraception?
Answer: Can't go on the OCP due to risks associated with breast feeding. The progesterone only pill/condoms/cap etc
73
are other options.
‫ــــــــــــــــــــــــــــــــــــــــــــــــــــــــــــــــــــــــــــــــــــــــــــــــــــــــــــــــــــــــــــــــــــــــــــــــــــــــــــــــــــــــــــــــــــــــــــــــــــــــــــــــــــــــــــ‬
193- Question 18 of 50
A 29 yo lady with hyperemesis grvidarum attends the ED as she is on holiday in the local area visiting her mother.
(a) Considering hyperemesis gravidarum, when is it most common & when would you expect it to have resolved by?
Answer: 8-12 weeks, settled by 20 weeks, more likely to get it if you are younger than 30.
(b) Is a family history relevant?
Answer: Yes there's a genetic component. It is more likely in sisters & daughters of women who have suffered with it
(c) How is it treated?
Answer: NB that it is a diagnosis of exclusion; need to rule out other things by investigat. Can use lots of antiemetics-
In the Uk normally start with antihistamine then proclorperazine or metocloparimde then ondansetron. Anti-emetic
medicat. appears to ↓ the frequency of nausea in early preg. There is some evidence of adverse effects. Of newer TTT,
pyridoxine (vit. B6) appears to be more effective in ↓ the severity of nausea. The results from trials of P6 acupressure
are equivocal. Evidence from observational studies suggests no evidence of teratogenicity from any of these TTT.
‫ــــــــــــــــــــــــــــــــــــــــــــــــــــــــــــــــــــــــــــــــــــــــــــــــــــــــــــــــــــــــــــــــــــــــــــــــــــــــــــــــــــــــــــــــــــــــــــــــــــــــــــــــــــــــــــ‬
194- Question 36 of 50
A 29 yo woman presented with severe Rt lower quadrant pain which had begun during exercise. She had no history of
PV bleeding & wasn't sexually active. She was at the mid-point of her menstrual cycle. O/E she had moderate
tenderness in her Rt lower quadrant but had no guarding. She was afebrile & haemodynamically stable

(a) What is the differential diagnosis? (Give five)


Answer: Ruptured ovarian cyst, ectopic pregnancy, endometriosis, PID, appendicitis, renal calculi, mittelschmerz
pain, ovarian torsion & pain from a leiomyoma.
(b) How would you investigate this Pt in the ED? (Give five investigations)
Answer: Urinalysis, urine or serum HCG, FBC, UE, pelvic ultrasound & a vaginal swab.
(c) How would you manage this Pt in the ED if she has an uncomplicated ovarian cyst rupture? (Give two points)
Answer: Analgesia, IV fluids & organise follow up.
(d) What is the indication for surgery if this Pt has a complicated ovarian cyst rupture?
Answer: Emergency surgery is performed to control ongoing significant hemorrhage.
(e) How would the presentation of a dermoid cyst rupture vary from that of a simple cyst rupture?
Answer: Shock & hemorrhage are the immediate sequelae of rupture of a dermoid cyst due to spillage of sebaceous
material into the abd. cavity.
‫ــــــــــــــــــــــــــــــــــــــــــــــــــــــــــــــــــــــــــــــــــــــــــــــــــــــــــــــــــــــــــــــــــــــــــــــــــــــــــــــــــــــــــــــــــــــــــــــــــــــــــــــــــــــــــــ‬
195- Question 2 of 10
A 25 yo woman presented with persistent vomiting at 9 wks gestation. U/S confirmed a viable intrauterine pregnancy.
(a) How would you investigate this Pt? (give three investigations)
Answer: Measurement of weight, orthostatic Bl. Pr, serum free T4 concentrat., serum electrolytes & urine ketones.
(b) How would you manage this Pt?(Name three points)
Answer: Gut rest, IV rehydration, avoidance of precipitants, anti-emetic medication.
(c) How is the diagnosis of hyperemesis gravidarum made?
Answer: The diagnosis of hyperemesis gravidarum is made clinically in a woman with onset of persistent vomiting accompanied
by weight loss exceeding 5% of prepreg. body weight & ketonuria in the 1st trimester, unrelated to other causes.
(d) List three possible maternal complications of hyperemesis gravidarum?
Answer: Micronutrient deficiency, wernicke encephalopathy (from deficiency of vit. B1) & sequelae of malnutrition
(immunosuppression, poor wound healing) have been reported. Oesophageal tears & rupture are other possible complicat.
(e) What birth defects are associated with hyperemesis gravidarum?
Answer: There's no clear  in the risk of birth defects among offspring of gravida with hyperemesis gravidarum.
‫ــــــــــــــــــــــــــــــــــــــــــــــــــــــــــــــــــــــــــــــــــــــــــــــــــــــــــــــــــــــــــــــــــــــــــــــــــــــــــــــــــــــــــــــــــــــــــــــــــــــــــــــــــــــــــــ‬
196- Question 3 of 10
A 19 yo woman attends with a history of lower abd. pain.

(a) What are the 3 common features of pelvic inflammatory disease?


Answer: Institute empiric TTT of PID when a Pt has all of the following minimal clinical criteria in the absence of an
established cause other than PID: Lower abd. tenderness on palpationm, Adnexal tendernessm, CMT, Additional
criteria, , Oral temp. >38.3C -Abnormal cervical or vaginal discharge - ESR - CRP –Lab. Documentat. of cervical
infect. with N gonorrhoeae or C trachomatis.
(b) What combination of drugs would you use to treat this condition?
Answer: 1st line antibiotics for pelvic inflammatory disease: Ceftriaxone 250mg IM stat (for gonococcal cover) plus
Doxycycline 100mg bd for 14 days & Metronidazole 400 mg bd PO for 5 days
74
(c) In what circumstances would you give IV antibiotics?
Answer: Severe disease is indicated if : A surgical emergency can't be excluded, Lack of response to oral therapy,
Clinically severe disease (temp >38oC, signs of pelvic peritonitis, signs of a tubo-ovarian abscess), Intolerance to oral
therapy, Disseminated gonococcal infection.
(d) What do you want to rule out in this Pt?
Answer: Ectopic pregnancy. Or other serious cause of abdo pain
(e) What investigations would you do?
Answer: FBC, U&E, CRP, LFTs, CXR, ECG, beta HCG, amylase/lipase, clotting screen, +/- cross match.
‫ــــــــــــــــــــــــــــــــــــــــــــــــــــــــــــــــــــــــــــــــــــــــــــــــــــــــــــــــــــــــــــــــــــــــــــــــــــــــــــــــــــــــــــــــــــــــــــــــــــــــــــــــــــــــــــ‬
197- Question 5 of 10
A 26 yo woman presented with right iliac fossa pain & PV bleeding. She was afebrile. Urine HCG was positive.

(a) What is the diagnosis? What is the differential diagnosis?


Answer: Ectopic pregnancy. Normal pregnancy with threatened/missed or incomplete abortion, ruptured or torsed
corpus luteum cyst & degenerating uterine leiomyoma.
(b) Name two investigations which should be performed?
Answer: Transvaginal +/- transabd. ultrasound & serum quantitative HCG.
(c) Name three management steps?
Answer: Analgesia, Bl. Group, Anti-D if Pt is rhesus negative, Obstetrical consultation.
(d) Name five risk factors for this condition?
Answer: Previous ectopic, previous tubal surgery, tubal ligation, intrauterine DES exposure, current IUD use, tubal
pathology, infertility, previous cervicitis (gonorrhea, chlamydia), history of pelvic inflammatory disease, multiple
sexual partners, smoking, previous pelvic/abd. surgery, vaginal douching & early age of intercourse (<18 yrs).
(e) The above lady was haemodynamically stable & there was no fetal cardiac activity. It was decided to treat
this lady with methotrexate. Below what serum hCG is methotrexate suitable?
Answer: hCG <5000 mIU/mL
‫ــــــــــــــــــــــــــــــــــــــــــــــــــــــــــــــــــــــــــــــــــــــــــــــــــــــــــــــــــــــــــــــــــــــــــــــــــــــــــــــــــــــــــــــــــــــــــــــــــــــــــــــــــــــــــــ‬
198- Question 6 of 10
A 23 yo who is gravida 2 para 1 attends with PV bleeding in the 8th week of her current pregnancy. She has also had
some mild lower back & abd. pain.
(a) What is a threatened miscarriage?
Answer: Vaginal bleeding through a closed cervical os. 50% will go on to miscarry.
(b) What is cervical shock?
Answer: Severe pain & bleeding accompanied by hypotension & bradycardia might be due to a vagal response
caused by the presence of retained products stuck in the cervical os.
(c) How do you treat it?
Answer: Remove any products of conception from the cervical os using sponge forceps.
(d) What is it important to do in a woman who has had an inevitable abortion?
Answer: Need to be referred to gynaecologists for in Pt care- may well go on to require a D/C. Also need to consider
Rheus status- if mother is -ve & non-immune then will need anti-D.
(e) List 4 risk factors for ectopic pregnancy
Answer: 1-Previous pelvic surgery 2-Previous ectopic 3-PID 4-Assisssted fertilisat. 5-Endometriosis 6-IUCD 7-POP.
‫ــــــــــــــــــــــــــــــــــــــــــــــــــــــــــــــــــــــــــــــــــــــــــــــــــــــــــــــــــــــــــــــــــــــــــــــــــــــــــــــــــــــــــــــــــــــــــــــــــــــــــــــــــــــــــــ‬
199- Question 1 of 10
A 26 yo Lady presents to the ED with vomiting. She is primi., 13 wks. She tells you that she can't hold anything down
& that it has been getting worse over the last few days. The 12 wks scan showed a healthy intrauterine foetus. O/E she
appears dehydrated & is tachycardic.
(a) What are the possible diagnoses?
Answer: Any cause of vomiting- normal part of preg., GE, etc.. hyperemesis gravidarum Appendicitis, Acute Ovarian
Torsion Cholecystitis & Biliary Colic, Pancreatitis, DKA, Pregnancy, Preeclampsia Gastritis & PUD UTI, Female
Gastroenteritis Hepatitis, Pyelonephritis, Molar pregnancy, Pseudotumor cerebri Acute fatty liver of pregnancy
(b) What investigations should be performed?
Answer: Lab. Studies * Obtain electrolyte levels. * Measure urine gravity & ketones. * Perform LFT if hepatitis is a
concern. Of note, LFTs can be slightly  with hyperemesis gravidarum. * Perform a CBC & urinalysis to rule out
other causes, with particular concern for pyelonephritis. * Hyperthyroidism causing nausea & vomiting is rare, a T3 &
T4 level should be drawn if this is a concern. (TSH can be suppressed in hyperemesis gravidarum.) * Obtain serum
amylase-to-creatinine ratio if pancreatitis is a concern. * Serum hCG levels are not clinically useful in a Pt with a
known intrauterine pregnancy & hyperemesis. Imaging Studies * The Pt should have an ultrasonographic evaluation
of her pregnancy to look for molar pregnancy or multiple gestations.
75
(c) What would prompt you to admit the Pt?
Answer: Presence of ketones is an important marker clearly if the Bl. tests revealed significant renal failure or other
concerning features then the Pt would need to be admitted
(d) What analgesics are good in the first trimester?
Answer: Paracetamol or opiod based, avoid NSAIDS
‫ــــــــــــــــــــــــــــــــــــــــــــــــــــــــــــــــــــــــــــــــــــــــــــــــــــــــــــــــــــــــــــــــــــــــــــــــــــــــــــــــــــــــــــــــــــــــــــــــــــــــــــــــــــــــــــ‬
200- Question 3 of 5
A 24 yo woman presented with a 2 week history of lower abd. pain & pustular vaginal discharge.

(a) What is the differential diagnosis?(Name three)


Answer: PID, intrauterine pregnancy complication, obstetrical malignancy, cystitis, & urethritis.
(b) How would you investigate this Pt?(Name five)
Answer: Preg. test to rule out ectopic preg. & complications of an intrauterine pregnancy, urinalysis, vaginal swab,
gram stain & microscopic exam. of vaginal discharge, FBC, ESR, CRP, transabd. +/- transvaginal U/S, CT pelvis.
(c) What are the risk factors for pelvic inflammatory disease?(Name three)
Answer: Multiple partners, age (PID occurs in highest frequency among those 15 to 25 yrs of age) previous PID, IUD
insert. (the risk of PID is primarily limited to the 1st 3 wks after IUD insert.) while barrier contracept. protects against PID.
(d) Name the two most common pathogens which cause PID?
Answer: Neisseria gonorrhoeae & Chlamydia trachomatis.
(e) Name three complications of PID?
Answer: Infertility, ectopic pregnancy & chronic pelvic pain.
‫ــــــــــــــــــــــــــــــــــــــــــــــــــــــــــــــــــــــــــــــــــــــــــــــــــــــــــــــــــــــــــــــــــــــــــــــــــــــــــــــــــــــــــــــــــــــــــــــــــــــــــــــــــــــــــــ‬
201- Question id: 2073
A 33 yo man presented with pleuritic chest pain which was relieved by sitting forward. O/E there was an audible frict. rub.

(a) What is the diagnosis?


Answer: Pericarditis.
(b) What is the aetiology of this condition?(Name three)
Answer: Viral (adenovirus, enteroviruses, CMV, influenza virus, HBV & HSV), TB, other bacterial, "autoreactive"
(immune-mediated), uremia, neoplastic, idiopathic.
(c) Name four laboratory findings which support the diagnosis?
Answer: CK-MB, troponin, CRP, elevated WBC count, elevated ESR.
(d) Name three features of the ECG which help to distinguish this condition from an acute MI.
Answer: The ST segment elevation in acute pericarditis begins at the J point, rarely exceeds 5 mm & usually retains
its normal concavity. The distribution of ST elevation is different. Acute STEMI is often associated with reciprocal
ST segment changes, which are not seen with pericarditis except in aVR & V1.
(e) How should this Pt be managed?
Answer: TTT of the underlying condition or if viral or idiopathic, NSAIDs.
‫ــــــــــــــــــــــــــــــــــــــــــــــــــــــــــــــــــــــــــــــــــــــــــــــــــــــــــــــــــــــــــــــــــــــــــــــــــــــــــــــــــــــــــــــــــــــــــــــــــــــــــــــــــــــــــــ‬
202- Question 14 of 50
You see a 74 yo gentleman with known hypertension, he is c/o feeling sick & has a headache, he also appears to be
confused. His BP is 220/130.
(a) Is this mild, moderate or severe hypertension?
Answer: Severe: >125 diastolic = severe
(b) What is the diagnosis?
Answer: Hypertensive encephalophay.
(c) What must your physical examination look for? (What conditions need ruling out?)
76
Answer: Need to examine the retinal for changes (exudates, haemorrhages, papilodema), Need to examine for any
focal neurology, accurately record GCS & mental status. Need to rule out a stroke or subarachnoid haemorrhage. If
suspected needs urgent CT head.
(d) When & who should start therapy? How should it be commenced?
Answer: Not in the ED, medical team to start TTT may require ICU/HDU care. Aim is to reduce the BP slowly.
‫ــــــــــــــــــــــــــــــــــــــــــــــــــــــــــــــــــــــــــــــــــــــــــــــــــــــــــــــــــــــــــــــــــــــــــــــــــــــــــــــــــــــــــــــــــــــــــــــــــــــــــــــــــــــــــــ‬
203- Question 16 of 50
A 72 yo man presented with a low grade fever, weight loss, & fatigue. He complained of a severe new unilateral
temporal headache & jaw claudication.

(a) Name three other symptoms associated with this condition?


Answer: Temporal arteritis may cause visual complaints such as amurosis fugax, polymyalgia rheumatica, upper
respiratory tract symptoms, arm claudication, symptoms 2ry to aortic aneurysms & aortic dissection may occur.
(b) Name three possible findings on physical exam?
Answer: Pulses may be diminished in the setting of large vessel disease.Tender or thickened temporal or other cranial
arteries can occur.Some Pts have cotton wool spots in the retina, depending on the site of critical vascular lesions.
Fundoscopic examination shows changes of ischemic optic neuropathy with a swollen pale disc & blurred margins. In
Pts with PMR, active range of motion of the shoulders, neck, & hips is limited due to pain. Bruits may be heard on
auscultation of the carotid or supraclavicular areas.
(c) Name three likely laboratory findings?
Answer: A characteristic lab. abnormality seen in most Pts with GCA is a  ESR, which often reaches 100 mm/h or
more. Serum CRP levels in GCA tend to parallel those of the ESR while the WBCs is usually normal, even in the
setting of widespread systemic inflammat.  serum concentrate. of hepatic enzymes, such as aspartate
aminotransferase & alkaline phosphatase, occur in 25 to 35% of Pts. The serum albumin level is often moderately ↓ at
diagnosis & a normochromic anemia is generally present prior to therapy.
(d) What is the TTT for this condition?
Answer: If temporal arteritis is not complicated by symptoms or signs of ischemic organ damage (eg, visual loss) an
initial dose of glucocorticoid equivalent to 40 to 60 mg of prednisone in a single dose is appropriate. If potentially
reversible symptoms persist or worsen, the dose may increased until symptomatic control is achieved.
(e) What is the prognosis for this condition?
Answer: In most Pts temporal arteritis tends to run a self-limited course over several months to several yrs. The
glucocorticoid dose can eventually be reduced & discontinued in the majority of Pts.A sizable minority have more
chronic disease & require low doses of prednisone for a number of yrs to control symptoms.
‫ــــــــــــــــــــــــــــــــــــــــــــــــــــــــــــــــــــــــــــــــــــــــــــــــــــــــــــــــــــــــــــــــــــــــــــــــــــــــــــــــــــــــــــــــــــــــــــــــــــــــــــــــــــــــــــ‬
204- Question 17 of 50
A 25 yo lady is brought in by ambulance. She has taken 7 bottles of 650ml methadone & 2 bottles of vodka. The ED
doctor who laid eyes on her wrote looks dreadful in the notes. She has pinpoint pupils.
(a) What is your immediate management?
Answer: ABCD with focused history if available Main priority will usually be A in such Pts Ensure that airway is
maintained (may require adjuncts or actually ventilating them, always have bag valve mask on hand) Can give IM
naloxone if doesn't have IV access
(b) You cannot get IV access quickly in this lady what will you do?
Answer: Give IM naloxone likely better to start with a reasonable dose i.e. 600mcg. Can give 200mcg-2mg in
repeated doses up to 10mg.
(c) What is the dose of naloxone in children?
Answer: 10mcg/kg with subsequent doses 100mcg/kg
(d) You get a response to your TTT. However it appears transient what will you do? (Please describe exactly
showing the workings of any calculations)
Answer: (600/100 x 60) = 360mcg (This is the amount that you want to infuse over 1 hr. Make up a syringe
containing 4mg of naloxone in 20mls (therefore 200mcg per ml) Therefore 1.8ml per hr needs IVI of naloxone. BNF
says: By continuous IVI using an infusion pump, 4 mg diluted in 20 mL IV infusion solution [unlicensed concentrate.]
at a rate adjusted according to response (initial rate may be set at 60% of initial IV injection dose (see above) &
infused over 1 hr)
(e) You take an ABG & the results are shown below: pH 7.107 pCO2 8.93kpa pO2 36.05 HCO3 16.2 BE -10.6
HB 15.1 Glucose 13.2 Describe what it shows?
Answer: Metab. acidosis -ve BE & ↓ pH. Also comment on the glucose. Comment on the high CO2 Resp. depression
(f) What could explain these results?
Answer: Could be DKA, high CO2 due to respiratory depression. Effects of alcohol & methadone could explain these
results. Could be septic?
‫ــــــــــــــــــــــــــــــــــــــــــــــــــــــــــــــــــــــــــــــــــــــــــــــــــــــــــــــــــــــــــــــــــــــــــــــــــــــــــــــــــــــــــــــــــــــــــــــــــــــــــــــــــــــــــــ‬
77
A 55 year old man, smoker and H/O hypertension presents with a 2 hour history of
central chest pain and shortness of breath, sweaty.

List 8 points of the initial management. 4 points

O2 via face mask, cannula, aspirin, BP, pulse, ECG, pulse oximetry, GTN SL

List 2 history criteria for eligibility for thrombolysis 1 point

20 min continuous ch pain < 12h ago


Intermittent pain <24 hours, worst episode <12h

List 3 ECG criteria for eligibility for thrombolysis. 1.5 points

1 mm ST elev 2 limb leads


2mm ST in 2 chest
New onset LBBB

List 7 absolute contraindications for thrombolysis. 3.5 points

Haemorrhagic stroke
Ischaemic stroke last 6/12
CNS damage or neoplasms
Recent major trauma/surgery/HI
GI bleed last 6/12
Known bleeding disorder
Aortic dissection

1) Regarding anaesthesia in the A&E department.


Name 4 different methods of anaesthesia which can be used to reduce a Colle’s fracture.
2 points

GA, Bier’s block, haematoma block, axillary block, Nitrous Oxide, sedation

What is the maximum dose of lignocaine with and without adrenaline. 2 points

3-4.5 mg/kg and with adrenaline 7 mg/kg

Name 3 general anaesthetic drugs which can be used in RSI and their doses. 3 points

Ketamine- 1-2 mg/kg


Propofol- 2-3 mg/kg
Thiopental – 3-5mg/kg

PRAY FOR ME Dr. Ashraf Elshehry Page 1


Midazolam- 0.1-0.2 mg/kg
Fentanyl- 5-15 µg/kg
Etomidate 0.2-0.3 mg/kg

Until what age is straight blade laryngoscope preferable and why? 2 points

1-5 years old


Epiglottis is floppy and obstructs view.

Name 2 methods of ensuring that the femoral nerve block is being injected in the correct
area. 1 point

Nerve stimulator
USS

2)Regarding Bier Block.

Which local anaesthetic is contraindicated in Bier’s block ? 1point

Bupivicaine

To what pressure should the cuff be inflated in Bier’s block and how long can it stay
inflated? 2 points

100mmHg > systolic


20-90 min

Name 6 contraindications to Bier block. 6 points


Peripheral vascular disease
Raynaud´s syndrome
Sickle cell disease
Cardiac conduction abnormalities
Hypertension, BP > 200 mmHg systolic
Cellulitis/infected extremity
Uncooperative patients
Young children (<5 years)
Local anesthetic allergy
What is the most common side effect of the block and how can it be minimised. 1 point

Cuff pain- inflate the proximal cuff after the LA injection and deflate the proximal cuff.

PRAY FOR ME Dr. Ashraf Elshehry Page 2


23 year old woman presents to A&E with SOB, difficulty in finishing sentences. She has
a history of asthma and takes regular inhalers.

Describe your initial management of this patient 3 points

O2, nebulised salbutamol 5mg and ipratropium 0.5 mg, steroids- 40-50mg pred or 100-
200mg Hydrocort
Check PEFR, O2 sats

List 6 signs of life threatening asthma 3 points

Silent chest, pO2<8kPa or sats < 92%, normal pCO2, bradycardia, cyanosis,
hypotension, feeble resp effort, PEFR <33%,, confusion, coma

List 6 therapies except the initial management which can be used in severe or life
threatening asthma 3 points
Mg 2+, BiPAP, continuous nebulised Beta agonist, IV aminophylline after senior
consultation, salbutamol infusion, Heliox- although not currently recommended,
anaesthetic gases, ketamine, adrenaline

What is the best predictor of outcome in acute asthma presentation in ED 1 point

Response to treatment

Paracetamol is the most commonly used drug in suicide attempts.


What is the best antidote and after what dose and how soon after the ingestion of

PRAY FOR ME Dr. Ashraf Elshehry Page 3


paracetamol should it be started. 3 points

N- acetyl cysteine
150mg/kg, or 75mg/kg in high risk pt, after 8 hours or immediately if staggered.

What is the alternative antidote, when should it be given instead of the above and at
what time post ingestion. 2 points

Methionine if H/O true anaphylaxis or no IV access (although oral N-acetyl cysteine


is available in the US) in < 10h

Name 5 other common poisons with a specific drug or substance used in the treatment
of their overdose. 5 points

B blockers – glucagons
Digoxin- dig specific antibody
TCA- NaHCO3
FE- Desferroxamine
Methanol etc- ethanol
Opiates- Naloxone
Insulin- Glucose
Paracetamol- N-Acetylcysteine
Sulphonylureas- Octreotide
Cyanide- Thiosulphat/dicobalt ededate/ hydroxycobalamine
Oral anticoagulants- vit K
Heavy Metals- EDTA, DMSA, DMPS

Drug overdose most commonly associated with the admission to ICU is the tricyclic
antidepressant group of drugs.
Name 6 signs of TCA overdose which can be found on examination- 3 points

The anticholinergic properties of TCAs produce


Sinus tachycardia, warm dry skin, brisk reflexes, sedation, seizures,. agitation to
delirium, confusion, amnesia, hallucinations, slurred speech, ataxia, sedation, and
coma.
Peripheral antimuscarinic symptoms include poorly reactive pupils, blurred vision,
hyperthermia, hyper- or hypo- tension, decreased oral secretions, ileus, urinary
retention, increased muscle tone, and tremor

PRAY FOR ME Dr. Ashraf Elshehry Page 4


Name 4 ECG changes associated with TCA overdose- 2 points
QRS, QT, and PR prolongation, and right axis deviation, cardiac ectopy, tachy and
brady arrhythmias

What is the main drug used in the treatment of TCA overdose, when should it be used
and how would you monitor its effects and what is its main side effect 4 points
NaHCO3 1-2 meq/kg until serum pH is 7.5-7.55
if pt is acidotic, cardiac arrhythmias, QRS prolongation of greater than 100 ms, or
hypotension
Hypokalaemia

Name 2 other drugs which can be used in the A&E for the treatment of TCA overdose
1 point
hypertonic saline, benzodiazepines, activated charcoal, barbiturates, MgSO4

2 days ago 40 year old woman flew back to Liverpool from skiing in Breckenridge,
Colorado. She fractured her left tibia and fibula and has her leg in plaster. Today she
presents to A&E with a sudden onset sharp, pleuritic left sided chest pain and SOB.
Describe your initial management. 2 points
High flow O2, IVA, analgesia, heparin either LMW or unfractionated

She has a RR of 25, sats of 100%, BP 140/85, ECG shows a sinus tachy of 110.
According to BTS guidelines, what should be the next investigation? 1 point
CTPA

Name other investigations which can be used as alternatives. 2 point


Perfusion scan, pulmonary angiography, MRI angio, USS – thoracic, ECHO or leg

She is not improving and suddenly deteriorates.


What are the clinical signs of massive PE. 4 points
Hypotension/collapse and unexplained hypoxia and engorged neck veins and gallop
rhythm

PRAY FOR ME Dr. Ashraf Elshehry Page 5


What bedside investigation would help to confirm the diagnosis and what is the
treatment of choice. 1 point
ECHO- showing R V dilatation, 50 mg alteplase as a bolus

What is the diagnosis and what is the most common cause 1 point
What is the most common disease predisposing to this condition and give a
differential diagnosis of this X-ray. 1 point
Gas gangrene due to Clostridium perfringens infection,
DM, surgical emphysaema from pneumothorax on ventilator or laparoscopic VV
surgery,
Describe your immediate management of this patient. 3 points
O2, IV access, IV fluids – 0.9% saline large volume, IV antibiotics- benzyl
penicillin or cephalosporin plus gentamycin ,
analgesia, BM

PRAY FOR ME Dr. Ashraf Elshehry Page 6


This, normally healthy, 4 year old child presented with a 6 day H/O temperature up
to 40, also has red eyes. Has not been
eating well and is crying on passing water. What is the most likely diagnosis, what
treatment can be given in A&E,
and what investigation should be done within 24 hours? 3 points
Kawasaki Disease or Mucocutaneous Lymphnode syndrome, Aspirin 80-100
mg/kg, ECHO

What is the treatment of choice for this condition and what is the most serious long
term complication? 2 points
Immune Globulins
Coronary Artery aneurisms

List the Ottawa knee rules – 5 points


Age >55
Inability to FWB both immediately and in the ED (4 steps)
Inability to flex to 90%
Tenderness over the head of Fibula
Isolated tenderness over the patella

PRAY FOR ME Dr. Ashraf Elshehry Page 7


List the Ottawa ankle rules- 5 points
Tenderness posterior med mall
Tenderness over navicular
Tenderness over lat mall
Tenderness over head of 5th metat
Inability to wt bear immediately and in the ED
Higher index of suspicion in young, old and intoxicated (½ point if one of the
others missing)

Name 10 features associated, directly or indirectly, with liver disease which can be found in the hand. 10
Anaemia
Jaundice
Bruising
Palmar erythaema
Dupitrien’s contracture
Leuconychia
Tremor
Tattoos
Clubbing
Spider naevi

Describe this ECG- 2 points

Junctional Tachycardia ( ½ point for supraventricular), Rate of 115 ( 110-120), Left Axis
Deviation, retrograde p waves buried within QRS

According to the APLS SVT algorithm what is the first treatment in children with SVT? 1 point
Which 6 drugs can be used to treat SVT? 6 points

PRAY FOR ME Dr. Ashraf Elshehry Page 8


Vagal manoeuvre
Adenosine50/100/250 mcg/kg
Verapamil
Digoxin
Porpranolol 50mcg/kg
Flecanide 2mg/kg
Amiodarone 5mg/kg

EXTRA ½ POINTS FOR THE DOSES


Which of the above drugs should be avoided in children under 1?

Verapamil

A 3 year old boy present to your department because his parents noticed him limping over the
last 3 days. There is no history of trauma and he had been walking and running around
normally until now. Name 10 acquired, non traumatic, causes of a limp in a child which are
localised to the leg- 5 points
Cellulitis
Septic arthritis
Osteomyelitis
Perthe’s
Irritable hip
Juvenile RA
Neoplasms
Warts
Rickets
Scurvy
Polio

A 2 year old boy presents to A&E with 1 day H?O NWB on his L foot. There is a vague
history of possible fall yesterday at grandparents but he was walking well all day until
developing a slight limp in the evening. He has chicken pox all over his body, temperature of
38.4 oC, tachycardia of 140 regular. He is covered in ch. Pox rash and slightly flushed on his
face. He is refusing to put any wt on his L leg. On close examination he appears to be tender
just above his lateral malleolus with a slight swelling there. His hip, knee etc appear to be
fine.
List 3 investigations which are most likely to give you the diagnosis? 3 points
X-ray, USS, bone scan, ½ point for MRI or CT

Give 2 most likely diagnosis 2 points

PRAY FOR ME Dr. Ashraf Elshehry Page 9


Fracture distal fibula, osteomyelitis fibula- there is an association between ch pox and
osteomyelitis/septic arthritis

What does the picture show? 1point


Angiodaema or swelling of the lips

PRAY FOR ME Dr. Ashraf Elshehry Page 10


Fill in the blank boxes for the anaphylaxis pathway. 1 point each
Give 4 causes of angiodaema. 2 points
Hereditary, acquired, drug induced and allergic, also idiopathic for ½ point
What is the significance in the treatment between these classifications? – 2 points
HAO, AAO and ACE induced do not respond to antihist/adren/steroid Rx
Vapour heated C1-INH concentrate is used for the HAO & AAO alternatively FFP
(extra 2 points)

PRAY FOR ME Dr. Ashraf Elshehry Page 11


Name 5 Ps pathognomonic to compartment syndrome and what is the main problem
with applying them in clinical practice to diagnose this condition? 6 points
pain, pallor, paresthesia, paralysis, pulselessness
These are late signs, probably too late to save the compartment.

Give 6 causes of compartment syndrome- 3 points


Trauma, fractures, bleeding in enclosed space, external compression of the limb,
vigorous exercise, small thrombo- embolic events, and intramuscular injections.

What compartment pressures are considered abnormal? – 1 point


15mm Hg resting or 30 mm Hg exertional, pressures above 20mmHg at 5 min post
exercise are also abnormal (extra ½ point)

PRAY FOR ME Dr. Ashraf Elshehry Page 12


A 38 year old woman comes in feeling rather unwell, complaining of a headache and
nausea. She noticed that her ankles started swelling last couple of weeks and she is
concerned as she is 32 weeks pregnant.
Describe the initial investigations and management of this patient. 3 points
IV access, check BP and perform UA and BM, Lie down in a quiet room(1/2 extra
point)
What features would constitute a diagnosis of preeclamsia? 1 point
Pregnancy, Hypertension and proteinuria &/or oedma
As you are attending to her patient suddenly becomes vacant and unresponsive. She
then starts twitching on her mouth. What is the treatment at this point. 1 point
MgSO4 2-4 g
What are the main management points following this? 5 points
Place in L lateral position, give O2, control BP – diastolic 90-100 with labetolol or
hydralazine, refer to O&G for urgent delivery

Name 3 abnormalities on this ECG.


What is the likely diagnosis?
4 points
T wave inversion inferiorly, T wave flattening in chest leads, U wave in V2-V3.
Hypokalaemia
What is the normal NaHCO3 concentration? 1 point
23-26 mmol
How do you calculate anion gap and what is its normal value? 2 points
Na- (Cl+NaHCO3)= 3 to 11 (used to be 8-15)
Give 4 causes of clinically important increases anion gap acidosis, other than lactic
acidosis. – 4 points
Renal failure, ketoacidosis, alcohol poisoning, salicylate poisoning

PRAY FOR ME Dr. Ashraf Elshehry Page 13


A 7-year-old boy presents to his primary care physician with a swollen left eye. Three
days ago, what was thought to be a pimple developed below his left eye, adjacent to his
nose. Two days ago, the left eyelids appeared erythematous and swollen. Yesterday, the
swelling became worse, and the patient's mother noted discharge from his left eye. In
addition, he had a low-grade fever and complained of having a headache and body aches.
The patient's father is in the US Army Special Forces and is being deployed to Iraq next
week.What is the diagnosis? What is the likely pathogen and how would you treat it? 5
points

Periorbital cellulitis
Vaccinia of the eyelid caused by inadvertent inoculation
vaccinia immunoglobulin (VIG) together with systemic antibiotics to prevent and treat any
secondary bacterial infection

A 55-year-old man with a fever for 10 days developed a crusted plaqaue on his left flank
(picture on the R). There was a black eschar with surrounding erythema and small
vesicles. Two weeks earlier his grandson developed a similar lesion associated with high
fever, sepsis, seizures and death.
What is the diagnosis? 1 point
What investigations would you perform? 2 points
How would you treat it? 1 point

PRAY FOR ME Dr. Ashraf Elshehry Page 14


Anthrax,
BC, CXr, ELISA blood test
IV abio- penicillin, ciprofloxacin

A 23 year old man presents alone to A&E department shouting that we have to contact “the
famous people in London” because the Earth is in danger from an Alien invasion. If we don’t
contact the famous people than he will have to kill his next door neighbour as he is directing
the Alien fleet towards Earth. He is very difficult to control and says he has a gun at home
with which he can kill the neighbour.
He allows himself to be booked in but the contact numbers he supplies are unavailable or not
answering and he is not previously known to the department.
You decided that he needs to be kept in hospital under the mental health act. You try to
contact the psychiatrist on call but she cannot answer you as it turns out that she is
currently dealing with a violent patient on one of the wards in psychie home who is
holding a nurse a hostage. Patient is becoming increasingly anxious and agitated
threatening to run off any minute. Under which section of the act could you detain this
patient “off your own back” in the department and what do you need to do for the
patient to become “eligible”. How long can you keep the patient, who do you need to
inform that you have done so? 3 points
Section 5 (2) after you persuaded the patient to get admitted to CDU/SSW (has to be a
voluntary pt) for 72 hours, you need to inform the hospital manager – extra ½ point for
section 5 (4) and 6 hours
Or section 4 with the use of an approved social worker- pt would not get admitted then for 1
extra point
You manage to calm the patient down and he agrees to wait for a little longer while you try
get him some transport to London sorted. After telephone conversation with psychiatrist he
says he is coming in because he thinks they need to section him and asks if you can assemble
the relevant people. Who would you call and what section will he then be able to apply for? 2
points
Need an approved social worker or a close relative of the patient for section 4& 2 + ideally a
doctor who knows the patient for section 2
What are the 4 categories of mental disorder?- 4 points
Mental illness
Severe Mental Impairment
Mental Impairment
Psychopathic disorder
What is the legal definition of Mental Illness? 1 point
There isn’t one

PRAY FOR ME Dr. Ashraf Elshehry Page 15


this boy 5years had vesicles on the palms with erythema around the lesions. Lesions affect
predominantly the palms. He has dysphagia and 2 aphthous-like lesion on the tongue.
What is it?- 1 point
Hand Foot and Mouth disease
A 22 year old man presents to the A&E department complaining of left testicular pain. He is
mildly tachycardic and pyrexial at 38.5oC. Yesterday he was started by his GP on antibiotics
for a tooth abscess and was planning to see a dentist today but for the testicular pain and
swelling.
He denies any unprotected intercourse with strangers and is married, his wife is 20 and 2
months pregnant.
What is the most likely diagnosis? 2 points
How would you treat him?- 2 points
What other important advice would you give him and why?- 2 points
Mumps
Rx with NSAIDs, icepacks and scrotal support
Find out if wife has been immunised but either way isolate from her for at least 8 days-
mumps causes spontaneous abortion in 27% of cases in 1st trimester

This 22 year old primary school teacher developed a diffuse symmetric red papular eruption
all over her body.
What is it? 1 point
What else is this patient likely to complain of and how would you treat those complaints? 2
points
Erythema infectiosum- Fifth disese
Arthralgia- worsening throughout the day, Rx with NSAIDs

PRAY FOR ME Dr. Ashraf Elshehry Page 16


3 year old boy has fallen out of a second floor window and landed on his head on grass floor.
He does not have any obvious external injuries, is breathing spontaneously, his heart rate in
150 BPM and BP is 100/65. On further assessment his eyes are closed but he opens them
when his mum speaks, he is moving all his limbs from time to time.
GCS- list all the Best Verbal Response criteria- 5 points
Alert, Babbles, coos, words to usual ability
Less than usual words, spontaneous irritable cry
Cries to pain
Moans to pain
No response
As you are putting a cannula in his arm, he becomes less responsive, he tries to withdraw his
hand but not very sternly and he only moans when you put the needle in him. You quickly
check his vital signs, RR is 25, good air entry, sats 99%, pulse 140, BP 100/70.
According to APLS temporary manoeuvres could you institute in this situation (not
necessarily in this patient)? 4 points
Head up to 20o
ventilate to PCO2 of 3.5-4.0
IV mannitol 0.5-1.0 mg/kg
Colloid infusion if hypotensive
What size ETT would you have ready?- 1 point
4-4.5-5.0 (1/2 point for 4.5)
Or one that fits into a nostril +/- 1/2 size

A 40 year old man presents to the department with severe abdominal pain. It is central and
going through to his back. He has been vomiting profusely. He thinks he’d eaten something
yesterday at his best friends stag night. His RR is 25, pulse of 120, BP 110/55.
Describe the first 5 steps in the initial treatment of this patient. 21/2 points
O2, IVA, IV fluids and IV analgesia, NGT
What is the most likely diagnosis and what investigations could you do to diagnose it? 21/2
points
Pancreatitis
Amylase, lipase, ERCP, CT abdomen, USS abdomen on occasions
List the Ranson’s criteria which can be used in the A&E. If a patient scores all of them what
would be his/her predicted mortality? 6 points

Age>55
Glucose>11 mmol
LDH> 350IU/l
AAT> 250U/l
WCC>16 000/ml
1 point for each of the above
under 3 Predicted mortality about 1%
3-4 Predicted mortality of 15%
5-6 Predicted mortality of 40%
over 6 Predicted mortality of 100%

PRAY FOR ME Dr. Ashraf Elshehry Page 17


The others are within 48 h
Packed Cell Volume decrease
Urea increase >1.8 mmol/l
Calcium < 2 mmol/l
PO2< 9kPa
Base deficit>4 mmol/l
Fluid sequestration> 6l

A 35 year old man comes in to A&E c/o a swollen and painful L wrist. It turns out he had
fallen off a horse and landed on the palm of his hand as he tried to save himself from falling.
Name 6 possible bony injuries. 3 points
Colle’s #, Scaphoid #, Galleazzi type #, Lunate and Perilunate dislocations, Ulnar styloid #,
Ulnar dislocation
What are the next 4 steps in your management of this type injury? 2 points
Analgesia, check neuro-vascular integrity give some form of wrist support, remove rings
Because you are in a DGH and it is now midnight, you are unable to get a X-ray without a
warrant from the minister of Health. You therefore decide to put a backslab on the patient and
tell him to return in the morning for imaging. The patient turns up 4 hours later in agony, he
can’t move his fingers now and can’t sleep because of the pain. What would you do first? 1
point
Loosen the plaster
Describe your management of the patient afterwards. 4 points
IV access and analgesia, bloods for K+, CK, and pH, order urgent X-ray and call the
orthopaedic surgeons .
1 point for surgeons, 1/2 point for the others

According to the BTS guidelines.


What are the5 core criteria for the severity of community acquired pneumonia and
what constitutes “severe pneumonia”.- 6 points
New confusion
Urea > 7
RR > 30
BP < 90/60
Age >64
Any 3 of the above criteria
What antibiotic regime would you use for severe community acquired pneumonia
requiring hospitalisation- 3 points
Co-amoxiclav 1.2mg tds or cefuroxime 1.5 mg tds or cefotaxime 1mg tds or
ceftriaxone 2g od
+
Erythromycin 500 mg qds or clarithromycin 500 mg bd

Alternatively
Levofloxacin 500 mg bd (oral or IV) + Benpen 1.2 mg qds
Name 2 other criteria associated with bad outcome – 1 point
Multi segment involvement

PRAY FOR ME Dr. Ashraf Elshehry Page 18


Sats < 92% or pO2 <8
Apyrexial
High or low WCC >20 and < 4
CRP>50
Positive BC

A 15 year old boy presents to A&E with 3 hour history of sudden onset severe Left
lower abdominal pain, radiating to his groin and a tender L testicle. He feels sick but
has not vomited, on examination he has exquisitely tender L testicle which feel larger
than than the right. List 3 differential diagnoses and how would you treat this patient?
4 points
Torsion, appendiceal torsion, epididymoorchitis, orchitis, trauma, testicular tumour
IV analgesia and refer to urology
A 20 year old man presents with 2 week
history of gradual swelling to his L
testicle (see pic). He has a soft but firm
swelling to his L testicle which is non
tender.
What is the most likely diagnosis and
how would you confirm it at the bed
side?
2 points

Hydrocele, transiluminate or USS


A 14 year old boy comes in with his
father. His dad had noticed a strange
lesion on the patients penis the previos
day at a swimming pool (see picture)
The boy says that he’s had it for aweek or
so but he did not bother with it as it was
painless and he was hoping it would just
go away.
What is the most likely diagnosis?
How would you treat it?
What else do you need to ask him
regarding the history of this illness?
What other consideration do you need to
take into account whet discharging this
boy home?
4 points
Syphylis
Procaine benzylpenicillin 750mg OD IM for 14 days
Sexual history and partners
Under age sex

PRAY FOR ME Dr. Ashraf Elshehry Page 19


A 65 yr old patient presents to department with sudden onset pain in the left eye. He had been
reading a book in bed when it happened. It is a severe pain and he finds it difficult to see out
of that eye.
List 3 features on simple examination which would be suggestive of glaucoma? – 3 points
Red eye, reduced visual acuity, fixed pupil, hazy sclera
Which topical agents can be used in the initial management?- 3 points
Topical beta blockers- eg timolol,
Pilocarpine drops
Apraclonidine- alpha agonist
What features on the examination of the eye would make you suspect bacterial
conjunctivitis rather than glaucoma? 4 points
Frank puss
Conjunctival oedma
Reactive pupil
Sensation of foreign body or grit in the eye

What is the diagnosis?


How would you treat it?
2 points

HSV keratitis or dendritic ulcer


Rx with topical antiviral agent- eg Acyclovir
What is the diagnosis?
2 points

PRAY FOR ME Dr. Ashraf Elshehry Page 20


Acute angle glaucoma
This patient presented to the department
with altered vision in one eye. A few days
before he had noticed a sudden onset of
black specks in front of that eye that
seemed to settle but now he keeps seeing
cobwebs in front of his eye and they not
getting better by rubbing. What is the
diagnosis?
What are the worrying features in the
history?
3 points
Retinal detachment
Delayed presentation and black floaters (pathognomonic of vitreous haemorrhage)
What is this condition called?
1 point

Papiloedema

This 8 year old boy presented with a


weeks history of erythematous rash on his
flank( see pic). He has been treated by the
GP with an antifungal and steroid
ointment but it is not improving. The
child does not seem to be bothered by the
rash except when the cream is being put
on he seems to be in a lot of pain.
He is otherwise well and healthy,
immunisation UTD.
What is the diagnosis? 1 point
What is the key question in the past
medical history? 1 point
What is the most common complication
of this condition and which patient
subgroup is most likely to suffer from it?
2 points
Shingles(herpes zoster)
H/O chicken pox (extra point for very young age)
Post herpetic neuralgia (pain >120 days) elderly
Bell’s palsy is defined as an abrupt, isolated, unilateral, peripheral facial paralysis

PRAY FOR ME Dr. Ashraf Elshehry Page 21


without detectable causes. What is the treatment of choice currently? 1 point
Name 5 factors which would give a worse prognosis.- 5 points
Prednisolone and aciclovir/valaciclovir
Complete facial palsy
No recovery by three weeks
Age over 60 years
Severe pain
Ramsay Hunt syndrome (herpes zoster virus)
Associated conditions—hypertension, diabetes,
pregnancy
Severe degeneration of the facial nerve shown by
electrophysiological testing

A 30 year old woman comes in to A&E complaining of abdominal pain. She is 32 weeks
pregnant and has been feeling unwell for a couple of days with pain coming on more severely
today. She feels nauseous and her appetite is down but she has not vomited. What factors
make a diagnosis of abdominal pain more difficult in pregnancy? 4 points
Anatomical differences- certain organs get displaced,
Peritonism less likely- peritoneum gets pulled off its usual place
WCC is raised,
Presentation tends to be delayed
Imaging is more difficult
Presence of abnormal organs- e.g. placenta, enlarged uterus
Presence of foetus- additional source of potential pain
On examination the lady has diffuse abdominal tenderness maximal in the RUQ.
What are the 2 most likely diagnoses? 2 points
Cholecystitis
Appendicitis
What would be the investigation of choice to distinguish between the 2? 1 point
USS
List 3 factors which would make you think of an obstetric rather than surgical cause
of pain in a woman in 3rd trimester of pregnancy. 3 points
PV bleed
Foetal distress
Absence of foetal movements
Absence of foetal HR
H/O lower abdominal trauma
Waters breaking
Visible foetal parts PV

PRAY FOR ME Dr. Ashraf Elshehry Page 22


A 23 year old girl comes in to A&E C/O L breast pain. The pain started 3 days ago and it got
so bad yesterday that she was unable to feed her 5 week old son. On examination she has a
large area of erythaema over the medial aspect of the breast. The skin feels firm and
indurated, it is very tender.
What is the likely diagnosis? - 1 point
What advise would you give her? - 2 points
How would you treat it? - 2 points
Mastitis
Carry on feeding, warm compress, plenty of analgesia, return if symptoms worsen or
notices discharge from nipple other than milk.
Flucloxacillin 500 mg QDS for 7 days +/- phenoxymethyl penicillin- 250 mg qds
What features in history or examination would point towards an abscess? 3 points
Fluctuant mass
Discharge from nipple or the mass
Very localised
Nipple inversion
History of previous abscess
One point each
How could you treat a breast abscess in A&E? 1 points
What investigation could you use to help the management?- 1 point
Needle aspiration and antibiotics, 1/2 point for referring to surgeons
USS to guide the needle

40 year old man presents with 6h


history of sudden onset of severe
stabbing pain in his perianal region.
What is the diagnosis? 1point
How would you treat it? 1 point

Perianal haematoma
I&D or conservatively
This man was hit in the eye
yesterday. What is the diagnosis? 1
What grade is it? 1

Hyphaema
Grade 1 - Layered blood occupying less than one third of the anterior chamber
Grade 2 - Blood filling one third to one half of the anterior chamber
Grade 3 - Layered blood filling one half to less than total of the anterior chamber
Grade 4 - Total clotted blood, often referred to as blackball or 8-ball hyphema

PRAY FOR ME Dr. Ashraf Elshehry Page 23


This 18 year old young man was started
yesterday on amoxicillin for ? tonsillitis.
What is the true diagnosis? 1 point
What caused the rash? 1 point

Infectious mononucleosis
Amoxicillin reaction
This 4 day old bay was brought in by his
17 year old mum concerned about the
rash. He was a FTB, NVD, being breast
fed. Apyrexial, good appetite, sleeping
well etc.
What is the diagnosis? 1 point
What is the treatment? 1 point

Erythaema Toxicum Neonatorum


No Rx- observe

A 20 year old man comes to A&E


complaining of being generally unwell. 3
weeks ago he returned from Papua New
Guinea, where he’d been on Raleigh
International for 3 months building a
bridge to a village in the middle of a
forest.
He started feeling unwell last week and
developed a rash. His GP diagnosed a
viral illness but he is feeling worse and
worse.
On examination he is pyrexial 40, has
slightly distended abdomen and palpable
spleen. The rash is easily blanching.
His bloods are:
Hb 11, WCC 15, plat- 80
Na 130, K 3.3, Ur 8, Cr 140,
GGT 200, ALP 75, Bili 40
What is the likely diagnosis? 2 points
Typhoid fever
What is the investigation of choice to confirm the diagnosis? 2 points

PRAY FOR ME Dr. Ashraf Elshehry Page 24


Who would you manage this patient and what particular precautions do you need to
take? 5 points
Bone marrow aspirate
IV access, IV fluids, Blood cultures, start antibiotics- 3rd gn cephalosporin or
ciprofloxacin, refer to infectious diseases
Very thorough hand hygiene
What else do you need to do once the patient has left the department? 1 point
Notifiable disease

68 year old man is brought in by ambulance. He has been complaining of abdominal pain for
the last 4 days. Pain has gradually been getting worse and his GP prescribed him senna
yesterday. Today he started vomiting profusely and is unable to keep any food down. He has
not opened his bowels for 5 days.
PMH – hypertension, mild arthritis, he had a Laparotomy 30 years ago following a
motorcycle accident, which left him with a limp.
DH- atenolol, thiazide diuretics and enalapril
His HR is 80 with a BP of 100/75, he looks sweaty and pale. His abdomen is distended and
generally tender with no guarding with loud bowel sounds.
Describe how you would manage this patient initially. 3 points
Oxygen,
IV access & IV fluids – N Saline 1 litre stat,
analgesia- morphine IV –
1point for mentioning each of these
Name 6 investigations you ought to order which could alter your
immediate management. 3 points
Abdo X-ray,
Chest X-ray,
ECG,
Ca,
amylase,
U&Es,
blood gases
½ point each
Diagnosis of small bowel obstruction is confirmed. Describe your management from
then on. 4 points
NGT,
continue IVF- wait for K+ results to see whether you need to give any to pt, he might
be in renal failure
catheterise,
refer to surgeons for further treatment
analgesia PRN in mean time

ENT
Name 4 symptoms useful in the diagnosis of acute bacterial sinusitis.
4 points
Unilateral face pain
Purulent nasal discharge

PRAY FOR ME Dr. Ashraf Elshehry Page 25


Pain during mastication
Anosmia
Headache
Fever
Nasal congestion
Halitosis
Toothache
Metallic taste
Cough
What is the investigation of choice in the diagnosis of sinusitis- 1 point and when is it
indicated- 2 points?
CT
Chronic disease/treatment failure, exclusion of alternative diagnosis, evaluation of
the severity of complications e.g. orbital cellulitis 1 point each
Name 3 signs which would distinguish orbital from periorbital cellulites. – 3 points
Proptosis
Chemosis
Visual Disturbace
Eye lid paralysis
Pain on movement

Neurology. (NICE)
Write out the Adult Glasgow Coma Score for an intubated patient – 10 points for all
Eye opening
No eye opening -1
To pain- 2
To voice- 3
Spontaneous- 4

Motor
No movement- 1
Extensor to pain- 2
Flexor to pain- 3
Withdrawal from pain- 4
Localising pain- 5
Obeys commands- 6
½ point for above

Grimace
No grimace to pain- 1
Slight grimace to pain- 2
Vigorous grimace to pain- 3
Less than usual spontaneous or to touch only- 4
Normal spontaneous grimace/facial movement- 5
1 point for above

PRAY FOR ME Dr. Ashraf Elshehry Page 26


What does SAD PERSONS score stand for?- 10 points
Sex
Age
Depression- H/O
Previous attempt at suicide
Excessive alcohol/drugs
Rational thinking loss
Separated/divorced/widowed
Organised/serious attempt
No social support
Suicide intent for the future

A 5 month old boy is brought in by his parents because of intermittent screeming. They got a
bit concerned in the morning when he appeared to be less active than usual. Was not eating
and for the last 2 hours started screaming intermittently and kicking with his legs up in the air.
On the way to the hospital he vomited once.
On examination he is lethargic but calm initially. HR is 140, BP 80/65. Abdomen is firm with
a fullness felt in the RUQ. When you are examining him child starts to cry as above and
vomits some bile stained fluid. Describe your initial management.
5 points
IV access, IV fluids maintenance, morphine IV, NG tube, refer surgery and paeads
What is the likely diagnosis? 1 points
Intussusception
What bedside investigation could aid with the diagnosis? 1 point
FOB or USS
What is the investigation of choice in this patient? 1 points

Ba/double contrast enema

What is the treatment of choice in this patient?- 1 point


Name an absolute contraindication to this.- 1 point
Hydrostatic enema
Signs of perforation- peritonitis/ air on AXr

PRAY FOR ME Dr. Ashraf Elshehry Page 27


This 6 year old patient noticed a few
these spots on his forearm last week.
They are not itchy but are not going
away. Parents are concerned.
What is the diagnosis?
How serious is this disease? 2 points

Molluscum contagiosum
Self terminating with no morbidity
This 14 year old boy has started
complaining of painful L knee. The re is
no history of trauma.
What is the diagnosis and treatment?

Osgood Schlatter
Rest and NSAIDs for the pain
A 6 year old boy presents to A&E with
difficulty in walking. He has been unwell
for a few days with temperatures. GP
diagnosed a viral illness and advised
ibuprofen and paracetamol. Pt has not
been getting any better and has had
several episodes of shakes and sweats. He
has an unremarkable PMH except a heart
murmur which is thought to be benign
and has not been investigated. Yesterday
he c/o painfull foot but today he is having
problems walking on it.
O/E he has a temp of 38.7, is a bit
lethargic, tachycardic at 140, BP 75/60.
He has a systolic murmur best heard at
the apex. His foot examination reveals
tender nodules(see pic)
What does the picture show?- 1 point
What is the diagnosis? – 1 point
Describe your immediate management. 4
points
Osslers nodes
Endocarditis
Oxygen, IV access and culture from 3 sites, IV N sal bolus- 20ml/kg, start a-bio, refer
to paediatrics, baseline bloods, analgesia

PRAY FOR ME Dr. Ashraf Elshehry Page 28


Blood transfusion

A 60 year old man is brought in following a bout of fresh haematemesis. He is known to


suffer from duodenal ulcers. He looks pale but is conscious and his BP is 140/100 his pulse is
95, his sats are 98 % on RA and his RR is 18. Apart from his PUD he has no other medical
problems. Describe initial management and investigations. 3 points
2 large bore cannulae
Group and save serum
FBC, U&E
ECG

His Hb is 6 so you start blood transfusion and called for an urgent endoscopy. While
this is being sorted pt develops a temperature of 38.8 oC. What would you do next? 2
points
Slow down the transfusion
Recheck the blood bag
List 4 symptoms of acute haemolytic blood reaction, other than pyrexia. 2 points
Chills
Burning along the vein
Anxiety
Back pain
Chest tightness
Flushing
Nausea
Tachycardia
Hypotension
What is the definition of massive blood transfusion? 2 points
Replacement of ½ volume of blood immediately or whole volume of blood within 24
hours
Which blood groups could you transfuse to someone who is A Rh-ve? 1 point
A-ve ½ point
O-ve ½ point
A+ or O+ if this is their first transfusion for extra ½ mark each

PRAY FOR ME Dr. Ashraf Elshehry Page 29


Delirium Tremens

36 year old patient is brought in to the department by ambulance. He is agitated and keeps
trying to push thing off himself when there is nothing there. He is known to the department as
a regular who comes in intoxicated with alcohol. According to the ambulance he had not been
seen for 3 days and when his friend went to see him he was found in this condition.
List 6 signs or symptoms of alcohol withdrawal other than mentioned above. - 3 points
Seizures, sweat, tachycardia, mild temperature, hypertension, tremour, insomnia, alcohol
craving, tachypnea, nausea/vomit, incontinence
Before you get to see the patient he has a grand mal seizure. It lasts 2 minutes and self
terminates before you give him anything. His temp is now 38, HR 130, BP 140/65, BM is
1.5.GCS is 11. Assuming O2 is on and pt has IV access, describe your immediate
treatment.4 points
IV thiamine (pabrinex, glucose- 500 ml 10% (but -1 if thiamine not mentioned as it can
precipitate Wernicke’s), N Saline 1 l stat, benzodiazepines of your choice titrated to pt
response, e.g. diazepam 10mg IV- according to the literature that is the drug of choice
What is the Wernicke’s encephalopathy triad? 3 points
Ocular abnormalities
Global confusional state
Ataxia

Epistaxis

Give 3 signs which would suggest posterior bleed. 3 points


Inability to visualise anterior source
Bilateral bleed
Blood dripping down the nasopharynx
A 40 year old man presents to A&E with a 30 minute left sided epistaxis.
He is haemodynamically stable etc.. What would be your initial management. 2 poins
Firm pressure to both sides of the nose- so that pt is uncomfortable for 10 min +/- ice
Ask pt to lean forward
This is a healthy man with a PMH of hypertension only on ACE inhibitors, no
previous bleed. What blood tests would you perform. – 2 points
None
A 69 year old man is brought in by ambulance he has been bleeding for 4 hours,
rather profusely following an injury to his nose at home. His BP is 100/65, HR is 85
irregular. He takes B blockers for hypertension and is known to have AF and is on
warfarin, INR last week was 3.5. No other PMH of significance. GCS 15 His nose is
obviously deformed and swollen and it continues to bleed. Describe your immediate
management. 3 points for 6 steps
Pressure and ice on the nose as much as pt will tolerate
Oxygen if necessary

PRAY FOR ME Dr. Ashraf Elshehry Page 30


IVA
Analgesia
Bloods- FBC, U&E, INR, X-match 2-4 units
Give FFP a.s.a.p.
Monitor BP/sats
Contact ENT
Contact anaesthetist and ICU
If bleed continuing consider packing/baloons

F Fractured Femur

A 25 year old man is brought in following a fall off a scateboard in a park- he was playing on
a pipe. He is in a lot of pain from an isolated injury to his R thigh, which is obviously
deformed and swollen. His ABC is intact, he has a cannula in situ. List 3 methods of pain
relief you would use in this patient. 3 points
IV opiates
Femoral block
Nitrous Oxide gas
GA if indicated
IV NSAIDs
As you are dealing with his pain you notice that his heart rate begins to rise and he
looks rather sweaty. His BP remains reasonable at 120/90. What would be your
initial single form of management. 2 points
Apply traction splint- e.g. Thomas
What would be the single most important blood test at this stage and the single most
important part of the bedside examination? 2 points
X-match 4 units of blood
Check distal pulses
His blood pressure is now starting to drop and he is becoming less responsive.
Describe you management at this stage. 3 points
Give IV fluids- hypertonic saline, saline or colloid of your choice
Insert another cannula
Ask for O neg blood
Organise an urgent thigh X-ray
Call trauma team + vascular surgeon

Headache - Migraine

PRAY FOR ME Dr. Ashraf Elshehry Page 31


A 55 year old lady presents to A&E with a 2 day H/O gradual onset headache. It is unilateral,
tight feeling associated with nausea. She had a few problems sleeping last night and
paracetamol is not helping.
On examination there is no abnormality at all, she has no PMH of significance and she’d
never suffered from headaches like this before. What urgent investigations would you order. 4
points
FBC
U&E
ESR+/- CRP
CT head
What treatment would you give her initially? 2 points
NSAID + antiemetic e.g. ketorolac 30 mg + prochloperazine 12.5 mg IM
List 4 signs or symptoms which are likely to be present in a migraine type headache-
4 points
History: Moderate-to-severe headache with or without a prodrome
Aura (20%):
Visual aura (most common)
o Scotoma (blind spots)
o Fortification spectra
o Geometric visual patterns
o Hemianopia
o Hallucinations
· Headache
o Unilateral, also known as hemicrania (30-40% are bilateral)
o Throbbing or pulsatile
o Lasts between 4-72 hours
· Systemic manifestations
o Nausea (80-90%)
o Vomiting (40-60%)
o Photophobia (80%)
o Phonophobia (75-80%)
o Lightheadedness (70%)

DVT and Polycythaemia

A 48 year old man presents to A&E with a swollen R calf. I had been bothering him for a few
days and it got worse this morning. On examination his right calf is 5 cm thicker than the left
and his foot appears oedmatus. List 4 factors from the Well’s criteria which would increase
your suspicion of DVT. 4 points
H/O lower limb trauma/surgery or POP
Swelling >3cm at 10 cm below tibial tuberosity
Pitting oedma
H/O malignancy
Bed ridden for more than 3 days
Entire limb swollen
Tenderness along the popliteal or femoral veins AS WELL AS calf tenderness
Dilated collateral veins
Previous TED

PRAY FOR ME Dr. Ashraf Elshehry Page 32


The patient’s bloods are as follows
Hb 18.5 g/dl
PCV 0.57
MCV 89 fl
WCC 9 x 109/l
Neutrophils 92 %
Platelets 388
List 4 secondary causes of polycythaemia. – 4 points
PHYSIOLOGIC INCREASE IN ERYTHROPOIETIN
Chronic lung disease
Cyanotic heart disease
High altitude
Smoker
Familial

INAPROPRIATE INCREASE IN ERYTHROPOIETIN


Renal- carcinoma, polycystic kidney, transplant
Adrenal tumors
Hepatocellular carcinoma
Ovarian fibroma
Cerebellar haemangioblastoma

Erythropoietin abuse by athletes

What abnormalities in the above results would give you a likely diagnosis of primary
polycythaemia- 1 point
Raised WCC AND platelets
What other blood tests would you perform before sending this patient for a diagnostic
investigation? – 1 point
None

A 20 year old woman, from Armenia, presents to A&E department c/o abdo pains and
sore arms and legs. Her English is not very good but from what you gather she has
had these pains before. She is otherwise fit and healthy and normally takes colchicines
on regular basis but has not taken any last few months since she ran out.
She also tells you that she cannot have babies but is unable to tell you why.
On examination she is in a lot of pain, has a rigid abdomen with decreased bowel
sounds and you note an appendicectomy scar. She has severe pains on trying to move
her legs and arms.
What is the diagnosis? 1 point
Describe your initial management. 2 point
Familial Mediterranean Fever
IV fluids(N sal large volumes) and opiate analgesia, give colchicine
What investigations would you carry out? 5 points
FBC
U&E
Amylase/lipase

PRAY FOR ME Dr. Ashraf Elshehry Page 33


UA
Pregancy test
What is the most common complication of this condition and what bedside test should
you carry out to give you and indication? 2 points
Renal failure
UA for protein

A 20 year old man from Uganda presents to A&E dept. He came over a few days
earlier to visit his brother. Today he started feeling unwell with recurrent pains in his
jaw and neck which are gradually getting worse. He keeps getting painful spasms of
his shoulders and neck muscles and is finding it difficult to breathe.
On examination he has a relatively fresh wound on his shoulder which turns out to
have been inflicted a week prior to coming over by a spear during a tribal scuffle.
Describe your initial A&E treatement.- 3 points
What is the diagnosis?- 1 point
What bedside test would confirm the diagnosis? – 1 point
Oxygen, IV analgesia and benzodiazepines, Human Tetanus Immunoglobulin
Tetanus
+ve spatula test- pt bites on the spatula when attempting to elicit gag reflex, negative
is a normal gag reflex
A 35 year old woman presents to the department confused and agitated and extremely
uncooperative. On examination she is tachycardic, pyrexial and has uncoordinated
movements. You also notice that she is drooling at the mouth.
She returned from France 5 days ago where she’d been on holidays but started to feel
unwell soon after. She attended the department a couple of days earlier c/o sore throat,
headache and temperature. At the time she also mentioned that she’d been bitten by a
small bat while camping in France towards the end of her stay there and now the
small wound felt odd being both painful and numb. She was discharged home with
some NSAIDs and advised to see the GP if things get worse.
What is her diagnosis?- 1 point
What is her prognosis?- 1 point
What treatment should she have received at first presentation?- 1 points
What other issues should you consider? – 2 point
Rabies
Death (100 % mortality once symptomatic)
1st shot rabies vaccine and immunoglobuline
Any 2 of:
Isolation of patient
Rabies prophylaxis for any of the staff exposed
Rabies prophylaxis to any family members
Reportable disease
Legal issues for the doctor who saw her 1st time

PRAY FOR ME Dr. Ashraf Elshehry Page 34


28 year old man had returned from holidays
in Central America a month ago. He had
suffered multiple insect bites while out there
but did not take any notice. However one of
the bites is not healing up and, what initially
looked like a small abscess now has broken
down and ulcerated.
What was the bite caused by? 1 point
What is the likely diagnosis? 1 point
What would be your management? 1 point
Sand Flies
Leishmaniasis,
refer to infectious diseases

What is the classical description of this


picture? 1 point
What disease is it typically associated with?
1 point
What pathogen is it caused by? 1 point

Strawbery tongue
Scarlet Fever
Group A Strep
A 40 year old man presented with a 3 day
history of pain and itching of his forearm,
following an insect bite.
What is the likely diagnosis? 1 point
In up to 30% of these there is a silent,
potentially serious complication- what is it?
1 point
What investigation would you order to
exclude this? 1 point
What is the usual treatment for this? 1
point
Superficial Thrombophlebitis
Deep Thrombophlebitis
USS venous system
NSAIDs and anticoagulation

PRAY FOR ME Dr. Ashraf Elshehry Page 35


A 15 year old girl presents to your department with abdominal pain and confusion She
had been complaining of the pain for a couple of days, it was epigastric and colicky in
nature. She had put it down to the fact that she had not opened her bowel for a few
days. On examination she has tender, soft abdomen which is not distended, bowel
sounds are present, she is crying and uncooperative, she vomited twice prior to
arrival. You also notice that she is confused in time and space.
She has no past medical history of significance.
Describe your initial management. 3 points
IVA and fluids,
analgesia- morphine
Oxygen if patient tolerates it
While you are doing all this she suddenly starts screaming that she can’t move her
legs. On closer inspection she is moving them but appears to have significant
weakness in her thighs.
Apart from the usual U&Es and FBC, what investigations would you do? 4
What would be your initial treatment? 1 point
Amylase, LFTs, ABGs, urine porphyrins
10 % dextrose
While you are waiting for the bloods you notice that the weakness in her legs appears
to be worsening and she is less active.
What is the likely diagnosis? 1 point
What further treatment could you instigate? 1 point
Acute porphyria- any abdo pain with mental and/or neurological changes
Haem infusion ( Haem arginate)

A 68 year old farmer presents to A&E with a puncture wound to his foot. It turns out
he was cleaning the stables when he stepped on a rake.
He has a puncture wound to the sole of his foot.
He is otherwise well and healthy.
Describe your initial treatment. 2 points
Thorough irrigation and debridement of the wound if necessary.
Give Human Tetanus Immunogobuline
The patient is not certain when he last had his tetanus, nor how many he had. Under
what circumstances could tetanus vaccine not be given? 2 points
History of 5 or more vaccines gives life long immunity and no further boosters are
necessary even in high risk wounds.
source
Chief Medical Officer et al. Update on immunisation issues. [PL/CMO/2002/4,

PRAY FOR ME Dr. Ashraf Elshehry Page 36


PL/CNO/2002/4, PL/CPHO/2002/2]. Department of Health, August 2004.
How could you attempt to determine whether this pt needs a TD booster? 1 point
Call GP
According to the DoH guidelines, what are considered to be tetanus prone wounds?
5 points
Any wound sustained more than 6 hours prior to the wound or burn presenting to the
department
Puncture wounds
Wounds contaminated with soil/manure
Heavily devitalised tissue
Clinical signs of sepsis

In spinal cord injury.


What does power 0-5 mean. (5 points)
0- No movement
1- Minimal movement
2- Movement not against gravity
3- Movement against gravity
4- Movement against some resistance
5- Movement against full resistance
Below what level of spinal injury does neurogenic shock become less pronounced and
why? (2 points)
Below C6- sympathetic nerves leave the SC between C7 and L1
A 75 year old man presents to A&E c/o weakness in his hands. He fell over the
previous evening and hit his forehead on the table on the way down. Today he noticed
that he keeps dropping things and was unable to brush his teeth.
What is this syndrome called? 1 point
What is the most common condition predisposing to this injury? 1 point
Central cord syndrome
Cervical Spondylosis
In what age group is SCIWORA most likely? 1 point
The young – especially <9 yrs old

PRAY FOR ME Dr. Ashraf Elshehry Page 37


A 12 year old boy presents to your department with a 4 hour H/O headache, preceded by a
day H/O cold, runny nose and temperatures. Mum noticed a fine petechial rash on his belly.
Name 3 investigations which would could alter your management in A&E and why. (3 points)
LP- to check for the signs of meningitis
WCC- check for leukopenia,
Platelets-thrombocytopenia
Hb- anaemia
U&E- Hyponatraemia
BM/Glucose- hypoglycaemia
Name 3 contraindications to LP (3 points)
Coagulopathy
Raised intracranial pressure
Cellulitis/infection at the site of LP
Patient/carer refusal
Spine abnormalities

Name 4 abnormalities in LP results suggestive of bacterial meningitis. (4 points)


Pressure- raised >15-18cm
High WCC- typically thousands, except in pt with shunts who tend to have high WCC
Presence of microorganisms on gram stain
Glucose- lower than serum
Protein- high (>150 usually >1000)
Frank puss

Hypothermia

PRAY FOR ME Dr. Ashraf Elshehry Page 38


At what temperature would you consider pt to be hypothermic? 1 point
A CORE temperature of 35oC or less
List 4 symptoms of mild hypothermia. 4 points
Shivering, tachycardia, tachypnea, peripheral vasoconstriction, increase in BP,
ataxia, amnesia, dysarthria, ataxia, apathy

This ECG shows 2 signs typical of moderate hypothermia. What are they? 2 points
J waves (Osbourne waves), bradycardia
List 2 non environmental, chronic conditions which would predispose you to
hypothermia. 2 points
Hormonal
• Hypothyroidism
• Hypopituitarism
• Diabetes
• Hypoadrenalism
CNS tumours
What effect does hypothermia has on the urine output and why? 1 point
Polyuria- impairs renal concentrating ability leading to “cold diuresis”

A 5 year old boy comes with his mum to A&E, with difficulty in breathing. HE was well until
a couple of hours ago except for a bit of a cold and runny nose. He has no past medical
history but is looking very unwell now. His RR is 40 with an obvious stridor and significant
accessory muscle use, he is holding his head forward, unable to swallow saliva, his temp is
39.5 0C. Give 2 differential diagnoses. 2 points

PRAY FOR ME Dr. Ashraf Elshehry Page 39


Epiglottitis
Severe Croup
Tracheitis
What would be your first action in management of this patient? 1 point
Give O2
Despite your care the child is not improving, appears septic and anxious. Describe the
subsequent steps in your management. 3 points
Keep child calm
Call paediatric help
Call ICU/anaesthetist
Set up for urgent intubation/surgical airway
Point subtracted for IV access and antibiotics
The X-ray on the right was performed in
a 2 year old who presented in a similar
way.
Name 3 abnormalities on this X-ray.
3 points

hypopharyngeal dilatation,
the swollen epiglottis,
lack of definable aryepiglottic folds
What are the 2 most important steps in the management of this disease? 1 point
Secure airway
IV antibiotics

PRAY FOR ME Dr. Ashraf Elshehry Page 40


A 19 year old male was found on the beach at 0100. His friends brought him to A&E but
promptly disappeared. On arrival in the department he is very drowsy- GCS=
Eye- 1
Verbal- 4
Motor- 5
There are no obvious signs of external injuries but he is wet and covered in sand.
Describe 4 important aspects of your initial treatment. 2 points
O2, immobilise neck, IV Access, remove wet clothing and actively re-warm
His pulse is 120, BP 100/60. Apart from the usual FBC, U&Es what further
investigations would you carryout? 3 points
Sats, BM, ABG, C spine X-ray, CXr, core temperature ½ point each
His electrolytes are as follows:
Na- 150
K- 3.6
Cl- 101
Ur- 6
Cr- 115
HCO3- 20
Gluc- 3.5
What are the 2 abnormalities? 2points
Hypernatraemia and increased anion gap
What is the likely initial cause of the state the patient is in? 1point
Ethanol intoxication- 30-50% of near-drownings are associated with drinking.
He was found on a sea beach. What difference does that make to your initial
management? 1point
Which of the above tests suggest that he might have spent some time in the sea and
why? 1point
No difference to management
Na is high suggests swallowing sea water

A 70 year old man presents to A&E with severe L sided colicky abdominal and loin pain. It
was of sudden onset and started 2 hours ago. He has no significant PMH but has been
increasingly unwell over the last couple of months. He is currently on NSAIDs for neck pain
and iron for anaemia., both of which have been started in the last few weeks. Over the past
few days he noticed that he is having problems walking, upstairs in particular and his legs
gave way a few times causing him to fall. He has also been c/o tingling sensation and
numbness in his feet.
O/E he is slim, pale looking, you notice that he walks with a very careful gait even for
someone in severe pain. Abdomen is soft but his left side is very tender, as is his loin.
His UA shows blood +++, protein +++ no WCC no nitrites.
What is the most likely cause of his back pain, what should be your investigation of choice to
investigate that? 2 points
Renal colic
CT KUB
What 3 simple blood tests would you perform in A&E to further investigate his other
complaints and why? 6 points
FBC- looking for anaemia, thrombocytopaenia and leukopenia
U&E- looking for signs of renal failure
Ca- looking for hypercalcaemia

PRAY FOR ME Dr. Ashraf Elshehry Page 41


You performed a Cspine X-
ray. What is the most
significant abnormality? 1
point
What is the most likely
diagnosis in this patient,
linking all the above?
1 point

Lack of body of C3, Multiple Myeloma

Lumbar puncture
What are the 4 landmarks in performing the lumbar puncture? 2 points
2 pelvic bones and 2 spinous proceses- L4 and L5
What size needle would you use in an adult? 1 point
20 or 22
What are the normal values for SCF- pressure, glucose, protein, WCC 4 points
Pr- 80-200 mm
Gluc-
Prot- 15-40 mg/dl
WCC- 0-5
Name 3 contraindications to LP- 3 points
Coagulopathy
Raised intracranial pressure
Cellulitis/infection at the site of LP
Patient/carer refusal
Spine abnormalities

PRAY FOR ME Dr. Ashraf Elshehry Page 42


Memingitis
A 6 year old boy arrives in A&E with 4 hour H/O feeling unwell. He has a temperature,
complaining of headache and does not like the light being on. You suspect he might have
meningitis.
Name 2 signs which can be elicited on examination of pt which would suggest meningeal
irritation and describe them. 2 points
Kernig Sign- flex knee to 90o followed by flexion of hip to 90o pain in hamstrings and
resistance to knee extension is +ve
Babinski- flexion of neck leads to hip flexion
Nuchal rigidity- inability to passively flex the neck
In what percentage of patients with bacterial meningitis are these tests positive? 1
points
50%
Name 4 signs suggestive of CNS irritation- 2 points
Lethargy, seizure, focal neurological signs, raised fontanelle, irritability, confusion,
reduced GCS, papiloedema, positioning eg decorticate, decerebrate, raised BP with
bradycardia
What are the signs of early compensated shock? 3points
Tachycardia
Cool peripheries
Increased capillary refill > 3 or 4 sec
Tachypnea/ Sats <95%/hypoxia on ABGs
Base deficit > -5 mmol/l
Confusion/drowsiness/decrease conscious level
Poor urine output- <1ml/kg/h
Reduced pulse pressure
½ mark each
In a patient with no signs of raised intracranial pressure or shock what 2 drugs should
you use in A&E to treat suspected meningitis? 2 points
Cefuroxime 80mg/kg max 2g
Dexamethasone 0.4 mg/kg

PRAY FOR ME Dr. Ashraf Elshehry Page 43


Sickle Cell
A 24 year old man is brought in by ambulance c/o severe back pain and a painful erection. He
is of Afrocarribeal decent and is known to suffer from sickle cell disease.
On examination he is in considerable discomfort & SOB his abdomen is soft and he has
obvious priapism.
Describe 4 steps in your initial management. 2 points
Give O2, IV access, IV analgesia morphine 10 mg, IV fluids- 3-4 l NS
Name 3 investigations would you perform and why? 3 points
FBC- check for the severity of anaemia
U&Es- check for renal failure
ABGs- check pO2(lung crisis)
CXr- lung involvement/pneumonia
ECG- exclude possibility of an infarct
List 3 indications for exchange transfusion in sickle cell anaemia 3 points
Neurological- 1 TIA
2 CVA
3 Seizures
4 Cranial nerve palsies
5 Lung involvement – pO2 <9 on FiO2 of >60%
6 Priapism
7 Sequestration syndromes
List 4 causes of sickle cell crisis – 2 points
Cold, infection, hypoxia, dehydration, alcohol intoxication, emotional stress,
pregnancy, acidosis

PRAY FOR ME Dr. Ashraf Elshehry Page 44


Wrist injuries
Name this injury 1 point

Perilunate dislocation
Describe your management in A&E- 3 points
Analgesia, immobilisation- below elbow backslab, urgent referral ortho/hand surgeon
Name 6 carpal bones and ossification ages. 6 points
Capitate- 6/12
Hamate-6/12
Triquetrum- 2-4 years
Lunate- 3-5 years
Scaphoid- 4-6 years
Trapezium- 4-6 years
Trapezoid- 4-6 years
Pisiform- 12 years+

Regarding burns pt.


According to ATLS list 4 clinical indications of inhalation injury. 4 points
Singeing eyebrows,
Facial burns
Carbon deposits/acute inflammation of oropharynx
Carbonatious sputum
HbCO > 10%

PRAY FOR ME Dr. Ashraf Elshehry Page 45


H/O confinement in a burning environment
Explosion with burns to head/torso
25 year old man is brought in from his
house with burns to the R side of his
body. He fell asleep, drunk next to a
gas fire. He appears to be average (70
kg) size and has no significant PMH
Estimate the area of his burns and the
fluid requirement for the first 12
hours.3 points
What fluid would you use? 1 point

18%,
70*4*18= 5040 ml 2520 ml in 8h + 630 in next 4h= 1968.75ml in first 12h
Hartman’s is the best choice
How do you know whether the fluid management is adequate in this patient? 1 point
Aim for minimum of 50ml/h of u.o. or 1ml/kg
How would you differentiate between superficial and partial thickness burns in the
acute stages? 1point
Formation of blisters, more sensitive to stimulation, if deep tend to be much darker in
colour and blisters more likely to be burst

PRAY FOR ME Dr. Ashraf Elshehry Page 46


Paracetamol overdose
Name 3 conditions which would increase pt risk in paracetamol OD and describe the
mechanism responsible for that increase. 4 points

Malnourishment
Anorexia Nervosa
Bulaemia
Cystic Fibrosis
HIV
Act by depleating glutathione stores
List 3 drugs which would increase pt risk in paracetamol OD and describe the
mechanism responsible for that increase. 4 points
Phenytoin
Carbamezapine
Rifampicin
Phenobarbitone
isoniazid
? Alcohol
Act by stimulating the coenzyme p450 system
Name 2 other factors which would put the pt at an increased risk of liver damage from
paracetamol OD 2 points
Late presentation > 8hours
Staggered overdose

PRAY FOR ME Dr. Ashraf Elshehry Page 47


A 28 year old man is brought in from an RTA. His car rolled over an embankment and hit a
tree with the driver side door. No one else was involved. The accident happened 6 hours ago
in the middle of the night and the pt was unable to get out on his own. He has obvious
deformity to his mid thigh, ABCs are stable and does not appear to have any other injuries.
His Pulse is 130, BP 110/90, cap refill is up to 3 secs.

His CXr and C spine are clear, what does the hip X-ray show? 1 point
What 3 nerves are likely to be damaged with this injury and how could you test for it in
A&E? 3 points
Medial dislocation L hip or L acetabular fracture with hip dislocation
Sciatic nerve- ask pt to dorsiflex foot ,check sensation on the lateral aspect below the knee
Superior gluteal- hip abduction
Inferior gluteal- hip extension= check tensing of gluteus maximus
Name 5 other injuries and nerves which are associated these injuries. 5 points
Axillary Nerve- Shoulder dislocation
Radial nerve- distal 1/3 of humerus #
Median nerve- fracture dislocations of mid carpal/wrist
Ulnar nerve- median epicondyle fracture
Common perineal nerve – fracture Fibular neck

PRAY FOR ME Dr. Ashraf Elshehry Page 48


The pt is suddenly becoming more
SOB and confused. You notice a few
purple spots on his face and upper
trunk similar to the one on the lip
here

What is the likely cause for his sudden deterioration? 1 point


Fat embolus

Salicylate poisoning
At what dose/plasma concentration is salicylate poisoning thought to be mild? 1 point
List 2 symptoms of mild salicylate poisoning. 2 points
<150 mg/kg or 300-600 mg/l (250-400 in elderly or children- extra mark)
lethargy
tinnitus
nausea
vomiting
dizziness
At what dose/plasma concentration is salicylate poisoning severe? 1 point
List 3 signs/symptoms of severe salicylate poisoning -3 points
>500mg/kg or >800mg/l (>700 in elderly or children- extra mark)
Metabolic acidosis
Coma
Hypotension
Convulsions
Renal Failure
A 23 year old girl gets admitted after taking 50x 300mg aspirin tablets 45 min ago.
She weighs 50 kg.What would be the initial treatment? 2 points
Give activated charcoal 50g orally, consider multiple doses
Start bicarbonate infusion- 1l of 1.26% NaHCO3 over 3 hours

Her levels come back at 900mg/l, other biochem as follows:


Na 137
K 4.6
Cr 150
Ur 9.4
Venous pH 7.15
What treatment will she require? 1 point
Haemodialysis

PRAY FOR ME Dr. Ashraf Elshehry Page 49


Regarding Trauma
Which 3 parameters go into calculating the Revised Trauma Score and what is the normal
value and at what level should you take the pt to trauma centre and alert trauma team? 4
points
Systolic BP, GCS, Respiratory Rate
12 is normal, alert if <11
A 32 year old, 33 weeks pregnant woman is brought in from a simple RTC. She was
driving at 20 mph and hit a stationary car in front. She was wearing a seat belt and did
not suffer any direct injuries and was mobilising on scene. Now all she is complaining
of is some lower abdominal pain. What would be your main concern regarding this
pain and how would you investigate it? 2 points
Placental abruption, CTG for 4 hours at least
Only ½ point for USS- it only picks up up to 50%
Her vital signs are
BP- 130/85, pulse 110, RR 18, Sats 99% on air
While you are waiting you get her usual blood tests back.
Hb- 10.4, WCC- 17, plat- 350
What treatment is required so far and why. 2 points
analgesia- all parameters are within normal range for pt in 3rd trimester
The patient is concerned about the baby. How could you reassure her about its health.
2 points
Any 2 of the following
Ask about the foetal movements
Currently no signs of any significant problems and mechanism of injury not serious
Perform a dopler/USS of the foetal heart
Perform CTG

PRAY FOR ME Dr. Ashraf Elshehry Page 50


A 26 year old man, known H/O depression, is brought in drowsy into the department. His
girlfriend says she found him difficult to wake at home. He currently takes Amitryptyline for
his depression but she could no find the bottle. He has a GCS of 13, sats 98 RA, pulse 115,
BP 135/90. Describe your initial management and name 3 investigations you would perform
immediately which could alter your management. He has no other medical history. 4 points
A&B are intact, gain IV access, take paracetamol levels, do a blood gas analysis and
ECG
Name 3 signs of severe tricyclic overdose 3 points
Hypotension – systolic BP <90
Repeated seizures
Malignant tachyarrhythmias
Complete heart block
Blood pH<7.1
QRS duration > 160 ms
How would you treat this patient and what would be the aim of your treatment. 2
points
NaHCO3 boluses until blood pH is 7.5-7.55
If this patient shows no signs of tricyclic antidepressant overdose and any other
significant pathology has been excluded how long would you keep him in hospital? 1
point
6 hours post ingestion

A previously healthy 12 year old boy is brought in by his parents. He has been well until a
few hours ago. He started complaining of feeling dizzy and belly ache, later developed watery
diarrhoea. He is looking very unwell, his temperature is 39.8, pulse 140 and BP 110/85, cap
refill 3 sec. Describe your initial management and investigations. 6 points
O2 1point
Give anti-pyretics – parac or ibuprofen 1 point
IV access and cultures, FBC, U&Es, CRP (1/2 point each)
IVF- N Sal 20ml/kg bolus 1 point
Ceftriaxone IV 80mg/kg up to 2 g or equivalent 1point
As you are setting things up you notice that he now develops a few spots around the
creases of his arms and legs, he is becoming increasisngly unwell, temp is now 40.5,
BP 85/40, Pulse 160. What is the condition called and what is its mechanism? 2 points
Waterhouse–Friderichsen syndrome- spontaneous haemorrhage into the adrenal
glands from overwhelming bacterial sepsis
Whose help would you request at this point? 2 points
Paediatric and ICU specialists

PRAY FOR ME Dr. Ashraf Elshehry Page 51


An 85 year old woman is brought in by ambulance. She was found today at home
unresponsive in a chair. She now has a GCS of 10. Her HR is 56, BP 95/60. Her sats are 99%
on a non re-breather. There is no H/O trauma and the family are on their way. What 2
investigations would you carry out immediately in this pt and why? 2 point
BM- hypoglycaemia is a common reversible cause of altered mental state in the
elderly
Temp- bradycardia, reduced GCS again hypothermia is a likely common condition in
the elderly
On arrival of the family you get some Hx- pt has little in PMH, takes some pills for
high BP and a water tablet. Last few months has become less & less independent,
would not leave the house and tended to sleep a lot, she seemed to spend a lot of time
sitting in front of the fire and has put on a lot of weight, her GP started on an anti-
depressant a couple of weeks ago. Yesterday she seemed to catch a cold- with a sore
throat, and cough. Today, in the morning the family found her asleep in her chair in
the living room and could not wake her up.
What is the likely diagnosis and name 3 ECG changes associated with this condition.
4 points
Myxoedema coma
Bradycardia, low voltage QRS, 1st degree HB, RBBB, prolonged QT
When you take off her O2 sats drop to 85%. Name 2 abnormalities you could find on
the CXr and explain them. 2 points
Enlarged cardiac silhouette- cardiac effusion
Consolidation- pneumonia which caused the acute episode
What is the definitive treatment for this pt? 2 points
IV T3 25- 50 mcg or T4 400-500 mcg
Hydrocortisone 100 mg IV 6hly
Treat the underlying cause

PRAY FOR ME Dr. Ashraf Elshehry Page 52


A 22 year old man is BBA to your department. He fell off a ladder while cleaning windows
and landed on a fence, across his abdomen. The fall was about 3m. He is conscious and has
not sustained any other injuries but his abdomen is obviously tender and swollen. What does
AMPLE history stand for? 5 points
Allergies
Medications
Past MH
Last Meal
Events leading to presentation
He is well, has a cannula in situ. His pulse is 110, BP 120/85, O2 sats 100% with
mask on, and is talking freely. Abdomen is bruised and tender. What would be you
first action as far as treatment is concerned? 1 point
IV opiate analgesia- e.g. morphine 5-10 mg
Name 4 indications for immediate surgical intervention in abdominal trauma. 2 points
Gunshot wound,
Penetrating injury through the peritoneaum- both now questioned in the literature
Signs of shock without any obvious external cause- again a bit tricky in view of the
permissive hypotension theory
Evisceration
Diaphragmatic Rupture
Air under diaphragm on CXr
Frank peritonism

Name 1 absolute and 1 relative contraindication to DPL- 2 points


Absolute- Obvious need for Laparotomy
Relative- Morbid obesity, pregnancy and multiple previous abdominal ops

PRAY FOR ME Dr. Ashraf Elshehry Page 53


A 30 year old man comes in to A&E c/o pain in his R shoulder. It began a few hours ago and
is getting steadily worse. He is currently on holidays and has been doing some scubadiving-
today he did 2 dives to 20 meters in the North Sea with all the stops as per computer there
were no untoward events. He is normally fit and healthy and does not take any medications.
What would your initial 2 treatments be? 1 point
NSAIDs analgesia- eg Ibuprofen 800 mg
Oxygen non re-breather
What test could you do in A&E to exclude Cerebral DIS – 1 point and describe it 1
points
Modified Rhomberg’s
Pt stands tip to toe with arms across his chest, closed eyes once in balance, best out of
4 tries, needs to manage to stand still for at lest 1 min
Name 3 factors which would increase you risk of developing DIS even if your dive is
“By The Book”- 3 points
Cold water
Multiple dives
Multiple ascents during a dive
Physical exertion during or shortly after dive
Ascent to high altitude following a dive (including a plane flight)
Old age
Name 4 other injuries associated with diving 2 points
Drowning/near drowning
Barotrauma- pneumomediastinum, pneumothorax, perforated ear drum, GI
rupture,toothache, maxillary pain
Oxygen toxicity
Nitrogen Narcosis
Mask squeeze
The patient suddenly appears somewhat confused and agitated although remains
cooperative. What would be your management now?2 points
Lie pt flat in a quiet room, arrange urgent transfer to the nearest hyperbaric unit,
keep the O2 going, consider lignocaine infusion for an extra mark

A 2 year old is brought in by mum, he has been getting increasingly SOB over the last few
hours. She is getting rather worried about him.
How would you assess the child’s effort of breathing on examination. 3 points
Under what circumstances are these signs likely to be absent. 3 points
Respiratory Rate
Intercostal recessions
Flaring alae nasi
Accessory muscle use
Inspiratory or expiratory noises
Grunting
½ point for each of the above
Absent in- exhausted child
Child with neuromuscular disease

PRAY FOR ME Dr. Ashraf Elshehry Page 54


Cerebral depression- drugged, encephalopathic, raised ICP etc
On examination, what signs of respiratory inadequacy affecting other organs would
you look for ? 3 points
Skin colour- pale or cyanotic
Heart rate- initially raised, reduced in pre-terminal stages
Mental status- agitation or later drowsiness
What is the most useful bedside test which assesses the effectiveness of breathing? 1
point
Pulse Oximetry

A 35 year old man comes in to you’re A&E c/o SOB. He is a known asthmatic but does not
feel too bad, he had ran out of inhalers while on holidays yesterday.
He receives all the appropriate treatment but does not seem to be getting any better. List 4
signs/symptoms of moderate/severe asthma.- 2 points
PEFR 33-50% best or predicted
RR > 25
Pulse > 110
Difficulty in finishing sentences.
What are the indications for chest X-ray in acute asthma? 3 points
? pneumothorax
?pneumomediastinum
?consolidation
no improvement on treatment
life threatening asthma
requirement for intubation
½ point for each of the above
After a further nebuliser he does seem to improve and is virtually symptom free, his
PEFR is now 80% his best. Under what circumstances would you prefer to keep him
in hospital? 3 point
Lives alone/poor access to help
Continuing symptoms
Exacerbation despite steroid pre hospital
Psychological problems/difficulty in learning
Concerns regarding compliance
Pregnant
Previous near fatal/brittle asthma
Presentation at night
½ point for each of the above
Just as you are about to discharge the pt your SHO comes in with a set of ABG he had
done a few minutes ago before you came to review the pt.
pO2- 12
pCO2- 3.0
pH- 7.46
BE- 3

What would you say to the SHO? – 2 points


Why did he do the ABGs- they are not indicated in non life threatening asthma and it
has been shown that patients present late because they fear that someone will do the
ABGs on them.

PRAY FOR ME Dr. Ashraf Elshehry Page 55


A 72 year old lady comes in c/o dizzyness. She has been gradually getting more and more
tired recently and has had some abdominal pains last couple of days. Today she started to feel
sick and vomited once. Her main problem however is dizziness esp on standing up. AN ECG
has been done:

Describe 4 abnormalities on this ECG – 4 points


Supra Ventricular Bradycardia
AF
T wave inversion inferior
Downwards sloping ST segment V4-V6(reverse tick)
The pt tells you that she normally has an irregular heart beat and takes digoxin for it
and a tablet to thin her blood down, she also takes a water tablet for her BP. What
investigations would you do which would help with the treatment? 2 points
Digoxin levels, U&Es, Mg 2+
What is the most common cause of digoxin toxicity? 1 point
Renal failure/impairment
List 4 indications for Digibind. 2 points
K+ > 5
Haemodynamically unstable bradyarrhythmia
Ventricular tachycardia
Cardiac arrest
Digoxin levels > 10-15
Altered mental status attributable to dig toxicity
Ingestion of >10mg in an adult or >0.3 mg/kg in child
Pt suddenly develops VT of 150, and becomes sweaty and SOB, BP is 80/50- what
easily available drug could you use to treat this immediatelly? 1 point
Lidocaine 100 mg bolus

PRAY FOR ME Dr. Ashraf Elshehry Page 56


Your SHO comes over to ask you to assist him with chest drain insertion. He has a 25 year
old man with acute chest pain since yesterday, no history of trauma and a pneumothorax on
his X-ray.
List 6 indications for chest drain insertion? 3 points
Pneumothorax- Recurrent post repeated aspiration
Symptomatic-i.e. SOB
Large traumatic
Tension post needle decompression
Any ventilated pt
Large 2-ry in pt >50
Empyema
Traumatic haemopneumothorax
Malignant pleural effusion
Post op- e.g thoracotomy
What are the A@&E treatment options for such a patient? 3 points
If < 2cm air and no symptoms discharge and advise re X-ray in 2/52 or return if symptoms
worsen
Otherwise try aspiration, if unsuccessfull can be tried one more time or go on to drain
insertion.
This is the pt CXr. What
would you tell the SHO? 2
points

This is a small L sided pneumothorax if pt is not SOB then I does not need a drain.
What advice would you give to pt discharged after a diagnosis of a spontantaneous
pneumothorax? 2 points
Advise not to fly until resolved on a re X-ray, most air lines advise 6/52 later for extra ½
mark
Do not dive unless bilateral surgical pleurectomy.

PRAY FOR ME Dr. Ashraf Elshehry Page 57


A 25 year old man is brought in by his friend. He had come off his motorcycle the previous
night. He slid off at about 50-60mph and hit a kerb with his L shoulder. His shoulder has been
very painful since and today, when he sobered up, he noticed that he is having problems
holding anything with his hand which also feels numb.
What is the most likely injury he sustained? 1 point
List 2 signs of this injury which might be obvious on simple observation.2 points
Brachial plexus injury
Horner’s
Winged scapula
Klumke’s or
Erb’s palsies
He does not feel SOB and his pulse is 75 regular. He does not appear to have any
other injuries. What would be your initial management of this patient? 1point
Give analgesia, apply cervical collar, order C spine X-rays, shoulder X-ray
On examination you find that he has
reduced sensation over the area on right.
Which 3 dermatomes are likely to be
affected? 1 points
Which movements are likely to be
affected by this injury? 3 points

C5-C7- 1 point for all 3, ½ point for 2


C5-shoulder abduction, extension and external rotation, some elbow flexion
C6- elbow flexion, forearm pronation and supination, some wrist extension
C7- global weakness of the upper limb without paralysis of a specific group, lat dorsi

1 point for at least 2 movements mentioned in each group, ½ point for just one
List 2 other bony injuries likely in this patient? 2
Avulsion injury to spinous processes/C spine injury
Clavicular fracture
Acromio-clavicular injury
Shoulder dislocation
Fracture humeral head

PRAY FOR ME Dr. Ashraf Elshehry Page 58


Fill in the empty boxes in the fitting child algorhythm. 10 points for all (2 points for the 2nd
from the bottom)

PRAY FOR ME Dr. Ashraf Elshehry Page 59


At 0800 a 65 year old man is brought in by ambulance. He was found outside in a field and is
unconscious. There is no evidence of injury, his clothes are covered in mud and he smells of
alcohol. His pulse is 55, BP 90/60, RR 8, sats 95% with a poor trace, he has a cannula inserted
in his R cubital fossa. What further 2 tests would you carry out immediately? 2 points
Core temperature- rectal or NG (no points for just temp)
BM
Name 4 signs or symptoms of moderate hypothermia other than the ones mentioned
above. 2 points
Core temp- 29-32oC
Polyuria
Irregular heart rate
Sluggish/dilated pupils
Absence of shivering
Stupor
The pt temperature is 30oC. Except for the ABC, what would be you first action as far
as treatment is concerned and what must you always remember when doing it?
2 points
Remove wet clothes CAREFULLY- these pts are very unstable and prone to cardiac
arrest.
List 8 methods of re-warming available to you in A&E. 4 points
Non invasive- Warm water- e.g. bath
Warm objects- e.g hot water bottle or electric blanket
Radiant heat- esp in babies
Forced warm air
Invasive- Warm gastric lavage
Warm bladder irrigation
Warm inhaled gases
Warm IV fluids
Surgical- Warm peritoneal lavage
Warm mediastinal lavage
Warm pleural lavage

PRAY FOR ME Dr. Ashraf Elshehry Page 60


A baby boy is brought in to A&E. He has been coughing a little all day and is now finding it
difficult to breathe. On examination he has widespread wheeze and fine crackles, his sats are
94% on air and his RR is 60.
Your SHO thinks it’s probably Bronchiolitis. List 4 factors which would make you worry that
a child is at an increased risk of a severe attack. 2 points
Premature birth
Under 6 weeks old
Chronic lung disease
Congenital heart problem
Immunodeficiency
He is apyrexial and haemodnamically stable. What treatment would you instigate in
this boy? 1 point
High flow Oxygen therapy
He does not improve greately despite your management and suddenly starts fitting
and goes into a respiratory arrest. He is 2 years old. Calculate the following.
Current weight 1point
ETT sizes 1point
Emergency fluid requirements 1 point
Adrenaline dose 1 point
Monophasic Defibrilator charges as per 2005 guidelines 1point
According to the current guidelines how many breaths should you give at the
beginning of CPR 1 point
What is the current recommendation for the compressions:ventilation ratio in this age
group? 1 point
(Age+4)x2= 12kg
Age/4 + 4= size 4,4.5 and 5
Wt x 20= 240 ml N Saline bolus
0.01 mg/kg= 0.12 mg adrenaline – 1.2ml of 1:10000
4J/kg single shock= 48 J or the nearest available dose on the defib
5 breaths- effective
15:2 or 30:2 if single person and problems switching

PRAY FOR ME Dr. Ashraf Elshehry Page 61


A 13 year old boy fell off a tree. He has not been able to walk since. On arrival he tells you
that he fell 2 m and landed on his feet and his L knee is really sore. He has an obvious
deformity of his L thigh.
List 6 signs which would alert you to circulatory inadequacy in this child. 3 points
Tachycardia
Low BP
Reduced UO
Reduced GCS or mental state
Pale skin
Cold extremities
Increased capillary refill
Tachypnea
Reduced pulse pressure
Signs of heart failure are not relevant in this case
How could you control his pain? 4 points
Intravenous morphine- 0.1 mg/kg or intranasal diamorphine
Nitrous Oxide
Femoral block- 3mg/kg Lignocaine
Leg splint
When you examine his legs you notice that the L foot looks a bit pale. List 3 signs
which might alert you to a vascular injury. 3 points
Abnormal pulses
Impaired capillary return
Decreased Sensation
Expanding haematoma
Bruit

PRAY FOR ME Dr. Ashraf Elshehry Page 62


A 5 year old child is brought in by his uncle. Apparently he became rather unwell after eating
a chocolate. He is known to have an allergy to nuts. The child is looking pale and quiet with
somewhat swollen lips and is struggling for breath.
List 4 signs and 4 symptoms which would alert you that this was a moderate or severe
anaphylactic attack. 4 points
Symptoms- Signs-
Coughing/wheezing Bronchospasm
Loose bowel motions/uncontrolled Tachycardia
defeacation Pallor
Sweating Laryngeal oedma
Irritability Shock
Collapse Respiratory arrest
Vomiting Cardiac arrest
As you approach the child you not that he has a quiet stridor and his RR is 60. What
treatment would you instigate immediately? 2 points
IM adrenaline- 10 micrograms/kg IM
Nebulised adrenaline 5ml 1:1000 through O2
As the treatment is being instigated describe you further management. 4 points
IV access
Hydrocortisone IV 4mg/kg ½ mark for drug and ½ for dose
Chlorpheniramine IV 2.5-5 mg ½ mark for drug and ½ for dose
Call anaesthetist
Call paediatrician
Presumably Ranitidine etc can be given but APLS does not mention it

PRAY FOR ME Dr. Ashraf Elshehry Page 63


A 54 year old man comes in to
the department feeling unwell.
He has just come back from
“cowboy holidays” in
Oklahoma, where he spent 2
weeks cattle rustling and
sleeping in tents. He developed
flu-like symptoms on the last
day of the holidays but did not
think much of it. Now, 3 days
later, he is complaining ao
hedache and temperature but
his main concern was a strange
rash he noticed this morning
(right). What is the main
question you would ask him? 1
point
What is the diagnosis? 1 point
What is the causative
organism? 1 point
Had he been bitten by a tick?
Rocky Mountain spotted fever
Ricketsia ricketsii

On examination he has a BP of 95/60, pulse 110, temp of 38.5oC. When you get back
to him he appears lethargic and asks you to get his mum, it turns out she has been
dead for 5 years. Describe your treatment. 3 points
Give Oxygen,
IV access & Bolus of fluids- 1 l N Sal stat
IV antibiotics- Doxycyclin 200mg
What is the likely cause of his confusion? 1 point
List 3 investigations you would perform and how what clinically important
abnormalities are they likely to show? 3 points
FBC- anaemia and thrombocytopaenia
U&E- hyponatraemia & renal failure
CXr- may show pulmonary oedema
Blood gases- may show lactic acidosis
BM- exclude hypoglycaemia

LFTs are likely to be abnormal but do not alter Rx, BC is very insensitive and few
labs perform it due to bio-hazard. Serology gets done later.

PRAY FOR ME Dr. Ashraf Elshehry Page 64


Pt is complaining of a foreign
body sensation in his eye.
What is the most likely
diagnosis?
1 point

Subtarsal foreign body- multiple vertical scratches


A 24 year old woman comes in
to A&E c/o painful L eye. Pain
gets worse on movement and she
thinks she can’t see very well
out of the eye. Thre is no H/O
trauma and slit lamp
examination is unremarkable.
Fundoscopy shown on the R-
what does it show? 1 point
List 3 other tests which should
be performed in A&E to help
reach the diagnosis. 3 points
What is the diagnosis and what
is the treatment? 2 points

Optic disc oedema


Visual acuity- often reduced, afferent pupilary defect- often present and red
desaturation test- usually posistive
Optic neuritis- Rx either with IV steroids or conservative, oral steroids have a
worsening outcome (-1/2 for that)
A 60 year old man is brought in by his daughter. He had been unwell recently and and
seems to have lost some weight and has been feeling increasingly tired and his GP has
started him on immigrant a couple of weeks ago. Today the pt noticed that he could
not see out of his L eye. What is the diagnosis? 1 point
What 2 blood tests would you perform to confirm it? 1 point
How would you treat it? 1 point
Ischaemic optic neuropathy due to Giant Cell Arteritis
ESR and CRP
Steroids- IV hydrocort or methyl pred

PRAY FOR ME Dr. Ashraf Elshehry Page 65


Painful red eye,
pain worse on
light.
Name the
condition and the
treatment.
2 points

Iritis/uveitis, Rx with topical steroid drops

What does the


picture show and
what is the
condition
causing it? 1
points
How would you
dispose of the
pt? 1 point
Hypopyon, bacterial keratitis; refer for urgent ophthalmic opinion
Painful L eye.
Name the
condition and 4
drugs which
could be used to
treat this in
A&E.
3 points

Acute angle closure glaucoma- topical b-blockers, topical a-agonists, pilocarpine


drops and acetazolamide IV, analgesia, IV mannitol
Very itchy, watering eye for last
few hours no other symptoms.
Pt noticed this swelling and is
concerned. What is it?
Which part of PMH would be
important ?
How would you treat this?
3points
Conjunctival oedema/chemosis; ask for H/O allergies esp hey fever; antihistamine eye
drops
PRAY FOR ME Dr. Ashraf Elshehry Page 66
A young man is brought into your
department. He had been assaulted and
has sustained the following injury

What two important structures are likely


to be damaged in this picture? 2 points
How would you go about repairing this
wound? 1 point

Lacrimal duct and tarsal plate are likely to be damaged


Do not repair-wet dressing and refer plastics/ophthalmology
You suspect he might have sustained damage to his cornea and are concerned about a
penetrating injury. How would you investigate investigate that and what must you
never do while examining such an eye and why? 2 points
Methods- stain washout: fluorescein gets washed away on examination
Anterior chamber thickness- shine light from side if does not illuminate the
/ whole of iris it suggests a shallow chamber= penetration
You should never press on the eye- you might cause more fluid leakage
He keeps
complaining of pain
around the eye and
on examination does
not seem to be able to
look upwards.
You X-rayed the
man’s face. List 3
abnormalities on this
X-ray.
3 points

Fracture R medial orbital wall


Fracture R orbital floor
Fluid in the R maxillary sinus
? FB L orbit for extra mark
List 2 indications for orbital blow out repair. 2points
Persistent diplopia in a functional field of gaze
> 50% floor involved
enophthalmos > 2mm

PRAY FOR ME Dr. Ashraf Elshehry Page 67


A 25 year old man presented to the
department because of a strange
lesion in his groin (upper pic) and
sore eyes (bottom picture). He had
been unwell for a while with sore
joints and mouth and has lost some
weight. A couple of weeks ago he
had noticed that his shins
developed some strange bruises.
Describe the top lesion. 1 point
List 3 abnormalities in the bottom
picture and what is their cause? 3
points
What is the unifying diagnosis and
what blood tests would you do to
confirm it? 2 points

A single well defined ulcer Major features


Recurrent aphthous ulceration of the oral mucous membrane
in the left groin. Skin lesions - Erythema nodosum–like lesions, subcutaneous thrombophlebitis,
Injected cornea, uneven folliculitis (acnelike lesions), cutaneous hypersensitivity
Eye lesions - Iridocyclitis, chorioretinitis, retinouveitis, definite history of
pupil, hypophyon= uveitis chorioretinitis or retinouveitis
Behcet’s disease Genital ulcers
Minor features Arthritis without deformity and ankylosis
No bloods- there are no Gastrointestinal lesions characterized by ileocecal ulcers
diagnostic test it is a Epididymitis
Vascular lesions
clinical diagnosis: Central nervous system symptoms
Diagnosis
Complete - Four major features
Incomplete - (1) 3 major features, (2) 2 major and 2 minor features, or (3) typical
ocular symptom and 1 major or 2 minor features
Possible - (1) 2 major features or (2) 1 major and 2 minor features
List 8 other systemic, non traumatic, conditions which can present with a skin ulcer.
4 points
Arterial insufficiency
Diabetes Mellitus
Venus insufficiency
pyoderma gangrenosum
Syphilis
TB/Leprosy
Trigeminal trophoc syndrome
Leishmaniasis
Drug reactions e.g. methotraxate
Skin secondaries
Calciphylaxis
SLE

Methaemoglobinaemia

PRAY FOR ME Dr. Ashraf Elshehry Page 68


Name 2 congenital causes of methaemoglobinaemia. 1 point
Congenital NADH methaemoglobin reductase deficiency
Haemoglobinopathy – haemoglobin M
List 2 drugs commonly used in A&E department which can cause
methaemoglobinaemia.- 1 points
GTN- IV or patch
Local anaesthetics- Lidocaine, bupivicaine
Silver nitrate
What is the normal level of methaemoglobin and at what level does toxicity become
symptomatic? 1 point
< 1% is normal
>20% symptomatic
List 2 signs or symptoms of mild OD and 2 of severe OD- 2 points
Mild- nausea, vomit, tachycardia, anxiety, headache, lethargy, dyspnoea, weakness,
syncope, may get chocolate cyanosis
Severe- coma, seizures, cardiac arrhythmias, cardiac conduction defects, death,
“chocolate cyanosis”
A 14 year old youth is brought in unconscious to the department. He was found in the
school playground in the evening after his friends rang for ambulance. You suspect he
has methaemoglobinaemia, what would you ask the ambulance crew regarding the
scene where pt was found and why? 2 points
Were there any cans around and smell of petroleum products
butyl-, isobutyl-, amyl- nitrites and naphthalene used as recreational drugs cause
methaemoglobinaemia.
What is the treatment of methaemoglobinaemia? 1 point
List 4 indications for obtaining methaemoglobin levels. 2 points
Methylene blue- 1-2 mg/kg,- extra point for dose
(revision points- it is ineffective in pt with G6PD deficiency and should not be given
to pt that are already blue (in overdose it causes methaemoglobinaemia))

Cyanosis unresponsive to Oxygen


Chocolate cyanosis- dark brown colour of arterial blood on testing
High or normal pO2 with low sats
History of nitrate/ite abuse/exposure associated with symptoms/signs of
methaemoglobinaemia

PRAY FOR ME Dr. Ashraf Elshehry Page 69


Anticholinergics
List 5 peripheral and 5 central effects of anticholinergic effects of a drug overdose. 5
points.
CENTRAL PERIPHERAL
Agitation Warm, dry skin
Mydriasis and blurred vision Tachycardia
Confusion Ileus
Coma Low-grade temp
Aggression Dry mucous membranes
Hallucinations Urinary retension
Delirium
P.S. Feel free to add others if you know of
any
List 4 groups of drugs responsible for such a presentation. 2 points
Antihistamines- older H1
Tricyclic antidepressants
Antipsychotics- e.g. phenothiazines, clozapine, risperidone
Anticholinergic drugs- Atropine, benzhexal, benztropine, scopolamine
Amantadine,
Carbamezapine
½ point for either group or a single drug representing it.

A patient with the signs and symptoms of anticholinergic drug OD and has a broad
complex tachycardia on the ECG- what treatments could you offer? 2 points
Sodium Bicarbonate ½-1 mmol/kg bolus until blood pH 7.5-7.55
If arrhythmia persists ALS protocol for an extra ½ mark

What is the antidote that can be used in anticholinergic crisis and when should it be
avoided? 1point
Physostigmine or tacrine
Do not use if broad complex tachy or due to TCA OD- can cause asystole

PRAY FOR ME Dr. Ashraf Elshehry Page 70


General toxicology
List 3 effects causing tachycardia due to poisoning and a drug responsible. 3points
Sympathomimetic- Amphetamines, caffeine, cocaicne, theophylline and others
Anticholinergic- tricyclics, antihistmines, beztropine, phenothiazides and others
Reflex response to hypotension- alcohol, salicylates, metals and others
Reflex response to hypoxia- paraquat

List 3 drugs responsible for bradycardia- 3points


B-blockers, Ca channel blockers, Digoxin, clonidine

List 4 toxic causes of hyperthermia- 4points


Salicylates, amphetamines, cocaine, MAO inhibitors, Serotonin syndrome,
anticholinergics, presumably thyroxin?

PRAY FOR ME Dr. Ashraf Elshehry Page 71


A 32 year old lady comes in c/o PV discharge and lower abdominal pain. She is fit
and healthy normally and has been feeling unwell for 3 days or so.
What are the minimal criteria for the diagnosis of PID- 3 points
Adnexal tenderness
Lower abdominal pain/tenderness
Cervical excitation

What is the first test you should do in such a patient? 1 point


Pregnacy test

What is the most common method of acquiring the infection, what organism is
responsible and how would you treat it normally? 3 points
Sexually transmitted
N Gonorrhoeae or C trachomatis
Ofloxacin and metronidasole orally at home (400mg bd for both, orally for 14 days
for extra mark)

What blood tests are likely to help with the diagnosis- 1 point
None- WCC is rarely raised, ESR and CRP too non specific it is a clinical diagnosis

List 4 criteria for admission in patients with PID 2 points


Failure to respond to oral treatment in 24-48 h
Unable to exclude a surgical cause e.g appx
Tubo-ovarian abscess
Pt unable to comply with oral treatment
Pregnancy
Pt systemically unwell- signs of sepsis
Adolescent age
Other medical conditions e.g. Immunocompromise, diabetic

Radiology
These 2 radiographs show the same injury.
Name the injury 1 point
How do you tell- 1 point
What age group does it typically affect? - 1 point

PRAY FOR ME Dr. Ashraf Elshehry Page 72


Why is the injury on the left rarer than the right? 1 point
What is the treatment in A&E and definitive? 1point

Slipped upper femoral epiphysis


Line drawn along the superior margin of the femoral neck does not transect the head
Girls 10-13, boys 12-15
L is a girl- 8 times rarer
A&E= non weight bearing, definitive- internal fixation
What are the 2 most likely
places for the coin to be in.
1point
What makes you say that? 1
points

In the oesophagus or on the skin


It is face on- tracheal coins TEND to be side on since they have to go between vocal
cords, it is also below the bifurcation ½ point each

PRAY FOR ME Dr. Ashraf Elshehry Page 73


This patient came to A&E with this
rather odd rash on her trunk. You
noticed that her breathing is a bit
more laboured than you’d expect a 30
year old healthy female to be and she
has fine crackles all over her lungs.
Describe the X-ray- 1point
What is the disease? 1 point

Bilateral hilar lymphadenopathy with generalised infiltrates


Sarcoidosis- X-ray stage 2 for an extra point

PRAY FOR ME Dr. Ashraf Elshehry Page 74


A 35 year old man comes to A&E with sore throat, temperature and generalised aches
and pains. He is concerned as he just came back from France and is worried that he
might have bird flu. What 4 questions would you ask him from the WHO
recommendations? 2 points
Ask whether he has been in the region where birds have been infected?
Has he been in contact with dead birds while there?
Has he visited any poultry farms/markets while there?
Does he think he might have eaten a poorly cooked/raw bird or egg while on
holidays?
Name 4 EU countries, other than France, in which Bird flu has been documented. 4
points
Germany, Croatia, Austria, Slovenia, Greece, Hungary, Cyprus & Italy
What are the WHO recommendations regarding visits to infected regions, except the
questions in 1st part? 2 points
Wash your hands frequently with soap and water
Do not try to bring any live or dead birds to UK
What symptoms would you expect a patient with bird flu to have early on during the
disease? 2 points
Temp > 38oC
Sore throat
Sore eyes
Achy joints and muscles
Generally feeling unwell
Cough
SOB
SAME AS ANY FLU

PRAY FOR ME Dr. Ashraf Elshehry Page 75


A 68 year old man is brought in c/o problems with speech. He is slightly drooling and
R corner of his mouth is sagging. This all started 45 minutes ago.
List 4 independent risk factors which predict the probability of CVA pot TIA?
2 points
What are the chances of the pt having a stroke within 90 days if all 5 factors are
present? 1 point
AGE- > 60
Duration > 10 min
Signs of weakness
Speech difficulty
Diabetes
34%
List 4 symptoms compatible with a posterior circulation TIA/CVE. 2points
Virtigo
Diplopia
Bilateral simultaneous visual loss
Bilateral simultaneous weakness
Bilateral simultaneous sensory disturbance
Crossed sensory/motor loss

Can also be- Dysarthria, homonymous hemianopia, dysphagia.

What does “locked in” syndrome signify? 1point


Pt remains conciouss but cannot move- stroke affecting ventral pontine motor areas
(pt is capable of looking upwardsonly)

What are the indications for emergency thrombolysis in CVEs? 3 points


Onset< 3 hours
No improvement in symptoms
No haemorrhage on CT
Age > 18
No contraindications to thrombolysis

PRAY FOR ME Dr. Ashraf Elshehry Page 76


A 17 year old girl has come in with suspected appendicitis. Tour SHO goes to see her
and returns ½ hour later looking concerned. He tells you that he saw the patient who
was initially in pain and was having problems moving on bed, she had localised
peritonitis in RIF and he referred her to surgeons. When he went back to see her she
appeared to be in some distress. You go and see the patient who is sittnig in bed
looking worried and keeps looking all around her. She says she can’t stop herself.
What is the condition called? 1 point
Occulogyric crisis- part of the dystonic reactions

What is the likely cause in this pt?


List 2 other causes. 3points
Antiemetic drug
Antipsychotics, antidepressants

List 6 other symptoms the pt might develop. 3 points


Buccolingual crisis
Protrusion of tongue
Forced Jaw opening
Difficulty in speaking
Facial Grimacing
Torticoliis
Opisthotonic crisis (Any idea what this is?)
Lordosis or Scoliosis
Tortipelvic crisis

List 4 factors which would predispose a pt to this attack.2 points


Young age
Female gender
Viral infection
Cocaine abuse
Alcohol abuse
Family history of dystonia

What is the treatment? 1 point


Antimuscarinic drugs e.g. benzatropin 1-2 mg IV or procyclidine 5-10 mg IV extra
point for doses

What 2 pieces of advice would you give the SHO? 2 points


Antiemetics are relatively contraindicated in young especially females BNF advises to
avoid.
They have no effect on the nausea related to opioid use (2 randomised double blind
trials in A&E and an observational study), except 5HT3 antagonists.
Not a clinically serious event, just scary.

PRAY FOR ME Dr. Ashraf Elshehry Page 77


Name 8 complications of transfusion. 4 points
Immediate-
Acute haemolytic reaction due to ABO incompatibility
Febrile non haemolytic transfusion reaction
Allergic reaction
Hyperkalaemia
Delayed-
Infections
Hypocalcaemia
Acid base disturbances
Fluid overload
Hypothermia
Delayed haemolytic reaction
Graft-vs-host disease
Which group of patients are most likely to get the most severe type reaction? 1 point
Unconscious or intubated
List 4 indications for FFP transfusion 2 points
Pt on warfarin or other anticoagulants
Pt with liver disease or DIC
Pt with congenital factor deficiencies when the specific factors not available
Pt with thrombotic thrombocytopaenic purpura
Pt with massive transfusion and signs of coagulopathy and ongoing bleed
Pt with antithrombin 3 deficiency

Which products are used to treat haemophilia A and which haemophilia B? 2 points
A- factor VIII
B- factor IX
What drug can be used to reverse heparin and how quickly does it act? 1point
Protamine acts within 5 min

PRAY FOR ME Dr. Ashraf Elshehry Page 78


A 48 year old pt is brought into the department complaining of a headache and sore
neck. This was a sudden onset severe headache and he passed out for a minute or so.
He is conscious but keeps his eyes closed until you talk to him, he can give you full
history and there is no obvious neurological deficit. The most likely diagnosis is a
subarachnoid haemorrhage. List 4 factors in the past medical history would increase
the likelihood of that event. 4 points
Smoking
Family history
Hypertension
Connective tissue disease- particularly polycycstic kidney and neurofibromytosis
Sickle cell disease
Α-1 antitrypsin deficiency

What factors go into the WFNS grading system of SAH, what scale does it run and
what grade bleed is the pt? 3 points
GCS and presence of motor deficit The other grading system is Hunt and
1-5 Hess also 1-5 and takes into account
Grade 2 severity of headache and extent of
neurological deficit

List 6 methods of investigating acute intracranial bleed 3 points


CT
MRI
MRA
USS in neonates
Angiogram
Lumbar puncture

Spinal trauma
According to the ATLS manual. Describe the dermatomes and the myotomes corresponding
to the following levels of spinal injury:

C5- derm= area over deltoid, myo= shoulder abduction, (deltoid)


C7- derm= middle finger, myo= elbow extension- (triceps)
T8- derm= xiphisternum, myo= intercostals, (difficult to test)
T12- derm= pubis symphysis, myo= intercostals and rectus abdominis, (difficult to
test)
L4- derm= medial aspect of calf, myo= knee extension, quadriceps
L5- derm= 1st web space toes, myo= ankle and big toe dorsiflexion (tib ant and EHL)
partial knee flexion (hamstrings)
S1- derm= lateral aspect foot, myo= ankle plantar flexors (gastrocnemius, soleus)
S4- derm= perianal region, myo= levator ani, test on PR

8 points

Briefly describe the diagnostic features of the following injuries:


Anterior cord syndrome- Paraplegia with loss of pain and temperature, (vibration,
proprioception and deep sensation are intact)
Brown-Sequard’s cyndrome- hemisection of the cord= ipsilateral paralysis and loss of
proprioception, contralateral temperature and pain

PRAY FOR ME Dr. Ashraf Elshehry Page 79


beginning at the level 1-2 below the injury, variations
in extent are frequent

2 points

A 30 year old man presents to A&E c/o difficulty in breathing and generalised weakness. He
is known to suffer from myasthenia gravis. List 4 other signs or symptoms of MG. 2 points
Ptosis
Diplopia
Weakness on exercise, improving with rest
Absent gag reflex
Improvement of symptoms in cold
Inability to cough

What two life threatening conditions could this patient suffer from and how could you
distinguish between the two? 4 points
Myasthenia crisis
Cholinergic crisis
Give small dose of edrophonium- if pt improves= MG crisis, if worsens= cholinergic
If pt has ptosis- place ice pack over the eyes, improvement in MG crisis

You put the O2 on, IVA in situ, all the bloods and investigations are done. As you are
waiting for the appropriate drugs the patient suddenly deteriorates and stops
breathing. Anaesthetist is unavailable because they are in OT with a AAA. Briefly
describe your management from then on quoting the necessary drugs and doses if
possible. 3 points
Place pt in supine position and insert an oropharyngeal or nasopharyngeal airway.
Preoxygenate the patient for at least 1 min with BMV, maintain cricoid pressure at all
times (personnaly I think that is load of cods wallop but most people are still insisting
on it)
Procede to RSI-
Propofol 2-3mg/kg and atracurium 0.3-0.6 mg/kg or another non depolarising ( -1
point for sucs)
Intubate with size 7-8 depending on pt, inflate cuff
Attach capnograph

What is the treatment for acute myasthenic crisis? 1 point


Edrophonium in small boluses, max 10 mg- monitor pt at all times for arrythmias

PRAY FOR ME Dr. Ashraf Elshehry Page 80


Pt presented with a sore shoulder. What is
the diagnosis?- 1 point
What syndrome might you elicit on
examination and what is the syndrome? 2
points
What would be the initial treatment. 1
point

Clacific tendonitis
Painful arch syndrome- pain on shoulder abduction 60o-120o
Rest and NSAIDs eg ibuprofen 400-800 mg TDS

A 55 year old man comes in to the department c/o hand pain. It has been bothering
him for a while but seemed to get worse today. It mainly seemed to bother him at
night and has been affecting the palm of his hand and the middle 3 digits. Now it is
also hurting his forearm. What is the likely diagnosis? 1 point
Name a sign you could elicit in A&E to “help” with the diagnosis and describe it. 1
points
Carpal Tunnel syndrome
Tinnels sign- tap over median nerve causes paraesthesia in the hand
or
Phalen sign- paraesthesia on hyperflexion for 60 secs

What might you find on examination which would confirm your suspicion. 2 points
Thenar wasting- in advanced cases
Reduced pain sensation on the palm over the median nerve distribution
Weakness of resisted thumb abduction

1 point for each


How would you treat it initially? 2 points
Splint the wrist and NSAIDs- e.g. ibuprofen 400-800mg TDS

PRAY FOR ME Dr. Ashraf Elshehry Page 81


A 20 year old girl presents with shortness of breath and generally feeling unwell.
She has a long standing history of weight loss and polyarthropathy and was recently
diagnosed with syphilis and started on antibiotics.
On admission you not a rash
on R
She is acutely SOB and her
BP is 90/40, she remains
apyrexial. She has muffled
heart sounds and appears to
have dilated neck veins.
Describe your initial
management. 4 points

Give Oxygen
IV access
Bloods for FBC, U&Es, CRP, blood cultures and ABG
Order a CXr and an ECG
Minus a point for IV fluids
Her ECG lookes like this:
What is the likely cause for
her current state? 1 point
What urgent investigation
and treatment is required? 2
points
What is the likely diagnosis?
1 point

Pericardial effusion/tamponade- low voltage QRS, muffled heart sounds, distended


neck veins, hypotension
ECHO and pericardiocenthesis
SLE- typical rash [(less than 50% get the malar rash)= symmetric hyperpigmented
plaques with scale, central hypopigmentation/depigmentation, follicular plugging,
and atrophy], polyarthralgia, wt loss, +ve VDRL in a young woman
List 4 other complications of this condition 2 points
Pleural effusion
Mucocutaneous ulcers
Malar rash
Pyrexia of unknown origin
Pleuritis
Broncho pneumonia
Myocarditis, pericarditis, endocarditis, heart failure
Pretty much any sort of -itis by the look of things ☺

PRAY FOR ME Dr. Ashraf Elshehry Page 82


20 Sore joints

What are the 4 features of Reiter syndrome. 2 points


What is the original cause of it? 1 point
What is the treatment? 1 point
Urethritis, Arthritis, Conjunctivitis (also keratitis, iritis, occasionally optic neuritis),
Skin and mucosal lesions
Chlamydial GUD or salmonell, shigella, campylobacter or yertsinia GIT infections
NSAIDs and reassurance- lasts 6/52 to 6/12
A young boy presented with this
rash and generally feeling unwell.
He has a temperature, sore joints.
What is the rash? 1 point
What is the diagnosis? 1 point
List 3 features, other than
mentioned above, of the disease.
3 points
What is the treatment. 1 point

Erythaema marginatum
Rheumatic fever
Subcutaneous nodules, carditis, chorea, 1st degree HB, recent URTI/pharyngitis
(H/O sore throat is missing in 20-30% of cases)
Penicillin oral for 10 days or Penicillin G IM-2.4 milion IU once, also haloperidol for
chorea, digoxin for heart failure

This pt was brought


in from an RTC.
Name 5 abnormalities
on this X-ray, what is
the diagnosis?
3points

Mediastinal widening
Deviation of trachea to the R
Obliteration of aortic knuckle
L Haemothorax
Obliteration of the AP window (the space between L pulmonary artery and the Aorta)

PRAY FOR ME Dr. Ashraf Elshehry Page 83


Extra ½ point for inadequate X-ray- apices not visible
Traumatic aortic rupture
Mane 2 other abnormalities associated with this injury which can be visible on a chest
X-ray. 2 points
Depression of L main stem bronchus
Deviation of oesophagus (NGT)
Widened paratracheal stripe
Widened paraspinal interfaces
Presence of pleural/apical cap
Fractures 1st/2nd rib+/- scapula
This pt had a large amount of
blood noted in his trachea on
intubation. Apart from all the
iatrogenic bits, name 2
abnormalities on this X-ray.
2 points
Following chest drain
insertion there was a very
large air leak. What is the
likely diagnosis. 1 point
How clould you confirm that
and how would you treat the
hypoxia in this case? 2 points

Pneumomediastinum and surgical emphysaema


Tracheo-bronchial tree injury
Bronchoscopy and reposition ETT into the intact bronchus.

Addisons Question

A 67 year old smoker, with COPD, attends ED with an episode of collapse. She has recently
lost weight and complained of feeling weak to her husband. On arrival she has a resting BP of
100/60 which drops to 80/38 on standing.

Her initial lab tests are as follows: Meds: Ventolin accuhaler prn
Na 125 Seretide accuhaler bd
K 6.2 Prednisolone 5mg od
Urea 7.9 (recently reduced from 20mg)
Creat 98
Chloride 105
Bicarb 14

ABG shows a mild metabolic acidosis.


CXR shows a peripheral mass lesion on the right apex.

1) What is the likely diagnosis and give 4 possible causes. (3)

Adrenal Insufficiency (Addisons) due to metastatic Ca Lung(1)

PRAY FOR ME Dr. Ashraf Elshehry Page 84


Causes: Metastasis, TB, Sarcoid or Cessation of corticosteroids(1/2 each)
Primary: Autoimmune, TB, Thrombosis/haemorrhage, Infiltrative diseases e.g.sarcoid,
Metastases
Secondary: Corticosteroid use, Radiotherapy, Infiltrative e.g. sarcoid, tumours, Head trauma
Must have TB and Ca Lung plus 2 others for full 2 marks.

2) The ABG showed mild metabolic acidosis. Calculate the Anion Gap.(2)

Anion Gap = ( Na + K) – (Cl + HCO3) = 12.2mmol/l

3) Give 5 Causes of Metabolic Acidosis with an elevated anion gap. (5)

Methanol
Uraemia
DKA, alcohol or starvation
Paraldehyde
Isoniazid or Iron
Lactate
Ethylene Glycol
Salicylates

MA/CO SAQ

OSCE April 2003

Question 1

Given data for a feverish hypoxic asthmatic child.

4 immediate treatments

Protect airway and administer high concentration (85%) Oxygen

Nebulised salbutamol

Paracetamol 15mg/kg

Hydrocortisone 4mg kg intracvenously

PRAY FOR ME Dr. Ashraf Elshehry Page 85


3 more treatment options

Call Paediatric anaesthetist

Repeat salbutamol neb and add ipratropium

Intravenous Salbutamol/ Aminophylline/ Magnesium according to age/drug history and local


guidelines

4 signs of a severe asthma attack in a child ?

Age 2-5 or over 5 respectively

Pulse greater than 130 or 120

Respiratory rate greater than 50 or 30

Unable to talk

PEFR<50% best or predicted

Question 2

Shown picture of a vehicle involved in side impact RTA.

5 likely injuries from such a side impact

Pick any five from the right side of the body

PRAY FOR ME Dr. Ashraf Elshehry Page 86


4 treatments you can begin at the roadside

Protect airway and administer high concentration (85%) Oxygen

Intravenous fluids

Analgesia

Splint limbs

What investigations you would request once at the hospital ?

(Pregnancy Test)

Chest and pelvic radiographs

Spinal series

FBC/U&E/X-match 6 units/ COAG/ amylase

ECG

Blood glucose

Urinalysis

CT/US as indicated

Question 3

Elderly diabetic chap with chest pain. shown ECG, CXR, and ABG results.

Asked to describe them and give differentials?

LBBB

PRAY FOR ME Dr. Ashraf Elshehry Page 87


Pulmonary oedema and consolidation right lower lobe

Respiratory acidosis with hypoxia

MI, LVF, Pneumonia

Question 4

Shown CXR and Right Shoulder X-Ray of trauma victim (one passenger died)

Comment on x-ray findings and possible complications

# Right 1st and 2nd ribs

AC joint dislocation

Pneumothoraces

OA etc.

Question 5

Elderly chap with chest pain radiating to back.

3 possible causes

MI

Dissecting thoracic aneurysm

Acute pancreatitis

3 investigations

ECG

CXR

Spiral CT chest with intravenous contrast

then shown a thoracic CT final diagnosis? – Dissecting thoracic aortic aneurysm

Question 6

Shown x-ray of index finger of netball player.

PRAY FOR ME Dr. Ashraf Elshehry Page 88


Asked for diagnosis and complication ?(Volar plate injury, OA)

Question 7

Photo of atraumatic swollen right knee in a young Caucasian.

Differential diagnosis

Septic arthritis

Gout

CPPD

Seronegative arthritis (e.g. assoc with UC/Ank spond/Reiters syndrome)

Haemarthrosis

Specific tests from lab?

Gram stain, microscopy and culture on joint aspirate

Polarisation studies on the joint aspirate

FBC/CRP/PV

Clotting studies and factor 8 assays

Question 8

Young West-Indian man on way home from night-club. Develops pain in his legs and
abdomen.

Differential diagnosis

Rhabdomyolysis due to MDMA (or cocaine or PCP)

Sickle crisis (abdominal crisis)

PRAY FOR ME Dr. Ashraf Elshehry Page 89


Question 9

Given data for a pyrexial child with purpuric rash.

Management steps

Protect airway and administer high concentration (85%) Oxygen


Blood cultures/FBC/U&E/COAG/PCR
Blood glucose – correct with 5mls/kg 10% dextrose if less then 3mmols/l
Intravenous Cefotaxime 80mg/kg
Intravenous fluids if shocked (20mls/kg N.Saline or HAS)
Intravenous dexamethasone (0.15mg/kg)
Paracetamol 15mg/kg
Inform paediatrics +/- PICU
Urinalysis
Throat swab

Question 10

25 yr old man collapses

Hb 9

WBC 2

Plt 90

Give 3 possible causes of the collapse

Overwhelming sepsis /Intracerebral infection

Intracerebral bleed

Toxicological reasons eg MDMA

Investigation of choice?

Blood glucose

PRAY FOR ME Dr. Ashraf Elshehry Page 90


Initially CT head followed by Bone marrow biopsy under admitting team

Question 11

Shown spinal xrays of young man fallen from ladder.

Diagnoses

C1 Jefferson's #

C2 Hangman's #

Wedge # L3

3 management procedures?

ABC (Protect airway with cervical spine immobilization, Support breathing if compromised
with high concentration oxygen/ bag and mask ventilation, Circulation – intravenous fluids
initially for hypotension)

Analgesia

Involve spinal surgeons early to guide further investiogations and prepare for surgical
stabilization

Question 12
12 Another set of blood gases...............Can't remember

T OSCE April 2004


Question 1
X ray - Radiolucent lesion developing at distal radius 10 yr old
a. Describe x ray

b. Differential diagnosis

Simple Bone cyst Osteosarcoma


Osteomyelitis Ewings sarcoma
Aneuysmal bone cyst Giant cell tumour

PRAY FOR ME Dr. Ashraf Elshehry Page 91


Osteoid osteoma
Enchondroma
Fibrous Dysplasia
Sickle Infarction

c. Investigations, management
Bone Scan
MRI
Bone biochemistry (Ca/Phosphate/ALkP)
FBC/PV/ESR/CRP

Question 2
Picture of leg with escharotomy
What is it?
Escharotomy for burn, resulted from electrical injury

What else might be needed?


Fasciotomy

Tissues preferentially conducting electricity


Vessels, nerves, skin, muscle

Showed picture of catheter bag: what does this suggest?


Myoglobinuria secondary to rhabdomyolysis

Question 3
Picture and x ray of thumb
a. Describe x ray findings:
osteomyelitis
b. Management

PRAY FOR ME Dr. Ashraf Elshehry Page 92


Analgesia
Antibiotics (intravenous benzylpen. And fluclox.)
Blood/swab cultures
Refer to hand surgeons for curette
FBC/U&E/PV/ESR/CRP

Question 4
8 yr old with fracture tib and fib.
a. Describe x ray

b. What are compartment syndrome symptoms?


Disproportionate pain to injury
Pain on passive movement of tendons /m uscles through that compartment
Parasthaesia
Swelling
Pallor

c. Management
Be aware of associated injuries if mechanism suggests (ATLS management)
If isolated injury:-
Check not an open fracture and neuro vascular statuse
Give Analgesia (intravenous opiates)
Above knee backslab
Keep limb at level ofsupine patient
Measure blood pressure and compartment pressures if compartment syndrome a
possibility (Diastolic minus compartment pressure >30mmHg)
Refer to orthopaedic team

d. Other investigations

PRAY FOR ME Dr. Ashraf Elshehry Page 93


Knee and ankle radiographs
CXR/Pelvis/Spinal series as indicated
Compartment pressures if concerns

20 year old, distal tibia with radiolucent lesion


a. Describe x ray
b. Differential
c. Management
d. Investigations (not bloods)

Question 5
Child on return visit to A&E; Kawasaki now diagnosed– already has conjunctivitis,
pyrexia,?nodes
a. 2 additional signs to look for
Fissured lips/Strwaberry tongue
Desquamation of palms/soles

b. Investigations – not bloods


ECG (30% get coronary artery aneurysms)
ECHO
c. Management
High dose salicylates
Immunoglobulin
Oral Hygiene

PRAY FOR ME Dr. Ashraf Elshehry Page 94


Question 6
Told pt has suspected thyroid storm
a. 6 clinical features
Tachcardia/irregular pulse
Fever
Irritable/Confused/Coma
Tremor
Sweating
Warm peripheries
D&V

b. 3 drugs used to treat


Propranalol
Hydrocortisone
Carbimazole

c. Investigations
ECG
TFT’s
Glucose
U&E’s
FBC
Septic screen
COAG
Calcium

Question 7

PRAY FOR ME Dr. Ashraf Elshehry Page 95


Paracetamol overdose
a. Name class of drugs affecting treatment level used
P450 CYTOCHROME OXIDASE ENZYME INDUCERS
Phenytoin
Carbamazepine
Barbiturates
Rifampacin
Alcohol
St.John’s Wort/ Sulphonylureas

b. 3 other conditions affecting which treatment line to use


Malnutrition/ anorexia
Alcoholism
AIDS

c. Calculate dose Parvolex

150 mg/kg Over 15 mins 200mls 5% Dextrose


50 mg/kg Over 4hrs 500mls 5% Dextrose
100 mg/kg Over16hrs, 1000mls 5% Dextrose

Question 8
Given history; 60 yrs old, collapse, initially, paramedics had difficulty finding pulse
a. ECG: 5 abnormalities

PRAY FOR ME Dr. Ashraf Elshehry Page 96


b. Diagnosis
Tri-fascicular block (1st degree block , LAD, RBBB)

Pt has further ‘funny do’; new ECG


What does the ECG show?
CHB

Treatment options?
Protect airway and administer high concentration oxygen (85%)
Atropine 500 microg intravenously repeated up to 3 mg total
External pacing
Adrenaline 2-10microg/min intravenously
Call cardiology re: Pacing wire

What asystolic rhythm might he develop?


P wave asystole (ventricular standstill)

Question 9
20 yr old female; short hx illness, no hx alcohol, drugs etc. Jaundiced. Bloods:
Hb 5.1
Plt 91
WCC N
Bilirubin raised
Clotting normal

PRAY FOR ME Dr. Ashraf Elshehry Page 97


a. What else would you ask in the Hx?
D&V
Fever
Recent travel
Last menstrual period/ ?on OCP/? Pregnant

b. Further investigations
Pregnancy test
U&E
Blood film
Coombs test
Urinalysis

c. 2 differential diagnoses
TTP
HELLP

Question 10
Hx CCP ? Dissection
a. Chest x ray: Name 4 abnormal features
Double knuckle sign
Wide mediastinum
Calcuium sign (>5mm step in aortic walll calcification)
Loss of aortic-pulmonary window
Pleral cap
Left Plearal effusion

PRAY FOR ME Dr. Ashraf Elshehry Page 98


Tracheal deviation to right

b. Further investigations
Spiral CT with contrasts or TOE
Aortography if cardiothoracics request
ECG
c. Management
Oxygen
2 large iv access
Opiate analgesia
FBC/U&E/COAG
X-Match 6 units blood
Cardiac monitor
Labetalol to control hypertension
Refer to Cardiothoracic team and involve ITU as will need invasive monitoring to monitor
BP

Question 11
Hx abdo pain, polyuria; bloods raised urea
a. CXR – 4 abnormalities
Bilat consolidation ?Sarcoid
b. Diagnosis
Hypercalcaemia
?DKA
c. Investigations
d. Management

Question 12

PRAY FOR ME Dr. Ashraf Elshehry Page 99


19 yr old male, RTA, GCS 6-7
a. Give initial treatments
A
B
C
D
E

Primary survey shows isolated head injury, CT no definite injury


b. 2 reasons why
Diffuse Axonal Injury
Brain stem injury
Too soon for signs of contusion or infarct changes to appear on CT

he April 1999: OSCE


Question 1
50 year old female smoker X-ray of Lateral Thoracic Spine

a. Describe the abnormality(s)


Generalised loss of density in T12

b. What is your differential diagnosis?


Metastatic lung Carcinoma
Myeloma
Osteomyelitis (TB)

c. What else do you want to examine?


Chest (collapse/consolidation/tracheal deviation/pleural effusion)
TB scar
Clubbing/ hepatomegaly/ lymphadenopathy/ anaemia
Neurological signs (limbs/ sacral reflexes)

Question 2

PRAY FOR ME Dr. Ashraf Elshehry Page 100


X-ray Left Shoulder
a. Describe the abnormality(s)
# mid clavicle
# posterior 2nd rib
pneumothorax

b. How would you treat the shoulder injury?


Broad arm sling

Question 3
X-ray abdomen. 80 year old chronic schizophrenic from the local long stay mental
health ward.

a. Describe the abnormality(s)


Huge dilated loop of bowel originating in the left iliac fossa (inverted U)
(Bent inner tube/ coffee bean sign)
?prominent small bowel loops

b. What is the diagnosis?


Sigmoid volvulus

c. Name two predisposing factors.


Elderly
Chronic constipation (possibly from antipsychotic medication)

d. What is the treatment?


Compression by insertion of soft rectal tube (flatus tube)
May need laparotomy if this fails.

PRAY FOR ME Dr. Ashraf Elshehry Page 101


Question 4
Chest X-ray
a. Describe the abnormality(s)
Calcification at the left heart border
Borderline cardiac enlargement

b. What is the diagnosis?


Ventricular aneurysm

c. Why does the patient have haematuria?


Warfarin therapy

d. Why does the patient have a left hemiparesis?


Embolic CVA secondary to mural thrombus
Question 5
CT Head of 60 year old man fallen 2/52 ago and again 1/7 ago. Now presents with
depressed level of conciousness

a. Describe the abnormality(s)


(Acute SDH typically appears on a noncontrast head CT scan as a hyperdense (white)
crescentic mass along the inner table of the skull, most commonly over the cerebral convexity
in the parietal region)
Biconcave haematoma in the left parietal area
Blood in the ventricle
Blood in the falx
Mass effect with midline shift

b. What is the diagnosis?


Subdural haematoma (?acute on chronic)

Question 6
Man hit in the eye by the branch of a tree. Photo of eye.

a. Name three abnormalities on the picture


PRAY FOR ME Dr. Ashraf Elshehry Page 102
Hyphaema
Irregular pupil
Subconjuntival haemorrhage.

b. Name four possible complications


Scleral/corneal Perforation
Corneal blood staining (decreased visual acuity)
Endophthalmitis
Glaucoma

c. Give three causes of a fixed dilated pupil


Third nerve palsy (eg raised intracranial pressure)
Mydriatics (e.g. atropine)
Death or Glaucoma
Question 7
20 year old man, unwell 2/7 with cough and pyrexia. Presents with reduced level of
consciousness after an apparent fit. Photo of crusted lesions on the lips

a. What are the two most likely causes for this appearance?
Stevens-Johnson syndrome
Herpes simplex encephalitis

b. List four investigations


Blood glucose
Blood culture
CT scan
EEG
LP when more alert

c. What treatment would you initiate before diagnosis?


Airway, breathing, circulation with high concentration oxygen via facemask
iv Cefotaxime 80mg/kg
iv Acyclovir (check dose with BNF)

d. What are the likely causative agents?

PRAY FOR ME Dr. Ashraf Elshehry Page 103


Herpes simplex
Mycoplasma
Drugs (e.g. suphonamides, barbiturates)

Question 8
Photo of person using both hands to hold a sheet of paper between thumbs and fingers. The
left thumb is adducted onto the paper, the right thumb is flexed and opposed onto the paper.
What is the name of the test demonstrated? Froment’s sign
On which side is the abnormality? Right
What does this indicate? Inability to adduct thumb (1st dorsal interossei paralysed)= Ulnar
nerve palsy,
What mechanism is used on the abnormal side? Flexion (flexor pollicis longus) via
median nerve
Question 9
20/40 pregnant woman involved in RTA sustained blunt abdominal trauma. Initial
observations; pulse 110, BP 90/50, RR 32
Give four physiological changes associated with the pregnancy in

a. the cardiovascular system


Increased blood volume
Increased cardiac output
Increase in pulse rate
Decrease in BP

b. the respiratory system


Increased tidal volume
Increased respiratory rate
Decreased residual volume
Hypocapnoea

c. What should the height of fundus be?


The umbilicus

d. Give five signs of foetal compromise

PRAY FOR ME Dr. Ashraf Elshehry Page 104


Uterine tetany
Active labour
PV bleeding
Loss of foetal movement
Foetal bradycardia/absent heart sounds

Question 10
40 year old woman recently returned from a trip to Malawi. Flu like symptoms for one
week, then a generalised seizure. On arrival temp 40°C.

a. What is your working diagnosis?


Cerebral malaria, (falciparum malaria)

b. Give four methods of temperature control


Exposure (Remove clothing)
Evaporation (Tepid sponging/fan)
Conduction (ice packs in groin/axilla)
Pharmacological (Rectal paracetamol)

c. What test would help with your diagnosis?


Thin blood films (+Thick films if still used in your centre)

d. Name three other useful tests.


Blood glucose
CT scan
Blood cultures
Urinalysis
PRAY FOR ME Dr. Ashraf Elshehry Page 105
(Chest X-ray)

Question 11
A six week old baby presents with shortness of breath and wheeze after a brief coryzal
illness. On examination he has a respiratory rate of 80/min with recession. Auscultation
shows bilateral creps and wheeze.

a. What is the diagnosis?


Bronchiolitis

b. What is the most likely viral agent?


RSV

c. Name two other possible viral agents


Adenovirus
Parainfluenzae virus

d. Give three conditions that may increase the severity


Bronchopulmonary dysplasia
Cystic fibrosis
Congenital heart disease

Question 12
Scenario. Asked to give basic life support to a baby manikin, talking through it as you
performed.

Back ache

In a patient presenting to the department with back pain

What features in the history might raise suspicion of cancer? (2 marks)

Onset > 50 or <17 yrs

PRAY FOR ME Dr. Ashraf Elshehry Page 106


Night pain
No improvement after 4-6 weeks
Weight loss
History of cancer

What features might suggest Cauda equina syndrome? (2 marks)

Urinary incontinence or retention


Saddle anaesthesia
Decreased anal tone of faecal incontinence
Bilateral lower extremity weakness or numbness
Progressive neurological deficit

What features might suggest vertebral fracture? (2 marks)

Prolonged use of steroids


Mild trauma + age >50
Age > 70
History of osteoporosis
Recent significant trauma

If these three were excluded, give the differential diagnosis for a 67yr old man presenting to
the department with acute onset back pain (4 marks)

AAA
Mechanical back pain
Lumbar disc protrusion
Infection

AASAQ back pain man

A 76 year old man presents with 5 weeks of back pain and gradual loss of mobility. His
daughter says that whilst he used to be completely independent and walk several miles a day,
he now needs help around the house and his exercise tolerance is reduced to 50m before he
becomes short of breath. Over the past 24 hrs he has become drowsy

FBC
Hb 8.2
Plt 45
WCC 4.3

Na+ 145
K+ 5.5
U 18.7
Cr 324

PRAY FOR ME Dr. Ashraf Elshehry Page 107


Give two possible causes of his drowsiness 2marks

Renal failure
Hypercalcaemia

What is the cause of his decreased exercise tolerance? 1 mark

Anaemia

What is the unifying diagnosis? 2 marks

Multiple myeloma

Name one blood and one non-blood test you cold use to confirm the overall diagnosis, and the
expected results 4 marks

Urine Bence-Jones protein


Plasma elecrophoresis – monoclonal bands

Name one further Biochemical test you would like to perform urgently 1 mark

Calcium
--------------------------------------------------
Bad rash

A 4 yr old child presents to you’re A&E department with a rash like this

(rash is meningococcal, in case you can’t see)

What features would you assess in your initial examination of the child (6 marks)

Must include features of shock (4 marks ) and raised ICP (2 marks)

Tachy decreased/fluctuating GCS


Cool peripheries Hypertension +relative brady

PRAY FOR ME Dr. Ashraf Elshehry Page 108


Cap refill pupils unequal/sluggish/dilated
Inc RR/SaO2 <95% Focal neurology
Hypoxia on ABG Abnormal posture/seizures
BE worse than -5 Papilloedema (late)
Poor UOP
Hypotension (late)

What fluid bolus would you give if necessary (how much and of what)? (1 mark)

20 ml/Kg colloid (HAS 4.5%)

If no response after 3 boluses of fluid, what would be your next action (1 mark)

Arrange intubation

Give 2 drugs + doses that would be used, sequentially, to treat fitting in this child (2 marks)

Lorazepam or Midazolam 0.1 mg/kg


Phenytoin 18mg/kg

Meningitis research foundation


Early management of meningococcal disease in children 4th Edition
A man is brought into the ED having been on a live electric train line. He has 30% burns to
arms and trunk, which are circumferential, and burns to his neck and right side of the face and
mouth. He looks about 70kg. His Breathing is noisy and his resp rate is 40/min with poor air
entry. His pulse is 120/min and his BP is 90/50. His GCS is 10/15.

Outline 8 steps in his initial management (and investigations) (4)


Oxgen 15l/min
C spine immobilisation
Monitoring
Call anaesthetist(senior!!)
Intubatewith RSI. Care with suxamethonium (raised K+)
IV analgesia with morphine + antiemetic
Dressings
IV fluid resuscitiation ( see below)
FBC, U&E, CK, ABG,Xmatch/G and S
Urine for myoglobin/haemoglobin
ECG
CXR
Catheterise
Contact plastics/burns unit
Tetanus prophylaxis

Calculate his fluid requirements. How much do you give in 24 hours , over what time period
do you divide the fluid and what type of fluid do you give? (3)
30 x 70 x 4mls in 24 hours = 8400mls (can also have 2mls instead of 4 mls) = 4200 if 2mls
Give half over first 8 hours = 4200mls (= 2100 if 2mls)

PRAY FOR ME Dr. Ashraf Elshehry Page 109


525mls/hour of normal saline or hartmans for first 8 hours (262.5mls/hr if using 2 mls)

He is intubated and the anaesthetist says she is finding it increasingly difficult to bag the
patient.
What one thing can you do to improve his breathing? (1)
Escharotomy of chest

Give 4 complications of electrical injury?(2)


Arrhythmia e.g. AF and other ECG changes
Dislocation or fracture
Renal failure due to myoglobinuria
Compartment syndrome
Neurolgical effects e.g. coma, seizures, headaches, transient paralysis
Ophthalmic e.g. cataracts and glaucoma
Burns full thickness

Bells Palsy or CVA?

A 63 yr old woman presents with L sided facial weakness. The triage nurse has diagnosed
Bells Palsy. The patient thinks it’s a stroke.

How would you differentiate between Bells Palsy and CVA on examination of the face (2
marks)
CVA spares frontalis muscle, so there is normal brow furrowing.
In CVA there is normal eye closure and blinking. In Bell’s Palsy there isn’t

What other features would you look for if you had diagnosed Bell’s Palsy? (2 marks)
Rash behind ear (Ramsay Hunt Syndrome)
Rash elsewhere might signify Lyme disease or Sarcoid

PRAY FOR ME Dr. Ashraf Elshehry Page 110


What symptoms might the patient have experienced in Bells Palsy?(2 marks)
Pain behind ear
Numbness on affected side of face
Loss of taste on ipsilateral 2/3 of tongue

What two drugs, and within what time limits, can be used to treat Bells Palsy (3 marks)
Prednisolone 1 mg/kg for 1/52 then taper over 2 nd week. Effective in 1st 7 days
Acyclovir 800mg 5x/day. Effective in 1st 72 hrs

What percentage of patients recover spontaneously (1 mark)

2/3

10 minute consultation Bells Palsy BMJ 2005;330:1374

Bone xray

PRAY FOR ME Dr. Ashraf Elshehry Page 111


An 8 yr old child presents with a limp of 7 weeks duration, and mild swelling around the right
ankle. The child has been systemically unwell and febrile. An x-ray has been taken in triage
and marked thus.

Describe the Xray 2 marks


There is quite a well demarcated lucent area in the metaphysis of the humorous of the tibia.
This lucent area lies immediately underneath the epiphyseal plate. There is no evidence of
infiltrative type destruction of the bone adjacent to it and there is no periosteal reaction.

Name the abnormality and the causative process. (2 marks)


Brodie's abscess, a chronic abscess of bone surrounded by dense fibrous tissue and sclerotic
bone

What is the best radiological investigation for this disease process? 1 mark
MRI

If a 5 year old presented with a limp, give 5 possible causes (5 marks)


Irritable hip, septic arthritis, toddlers fracture, osteomyelitis, FB in foot, ill fitting shoes…

Bradycardia
A 76 yr old woman presents to your ED complaining of dizziness and shortness of breath.
Here is her ECG

Describe what you see on this ECG, concerning the rhythm (2 marks)
Increasing PR interval and then missed beat

What is this rhythm called (1 mark)


Mobitz type 1 (wenkebach)

What are the adverse signs in bradycardia? (2 marks)

PRAY FOR ME Dr. Ashraf Elshehry Page 112


SBP < 90 mmHg
HR< 40
Vent arrythmias requiring suppression
Heart Failure

Assuming one of these was present, What would be your action? (1 mark)
Atropine 500mcg iv

If there was satisfactory response to this, how would you asses risk of asystole? (2 M)
Recent asystole
Mobitz 2 AV block
Complete Heart block with broad QRS
Ventricular pause > 3s

If risk of asystole was there, what interim measures could be institued in the ED before
definitive management? (2M)
Atropine 500mcg iv up to 3 mg
Transcutaneous pacing
Or Epinephrine 2-10 mcg/min

Broad complex tachy

A 57 yr old woman presents to your department after sudden onset of dizziness about 20
minutes ago. She has had 3 similar episodes over the past 12 months. Initially her ECG is
normal, but her symptoms return and her ECG is as follows

What is the diagnosis for this ECG? (1marks)

Ventricular Tachycardia

PRAY FOR ME Dr. Ashraf Elshehry Page 113


What is an important differential diagnosis? (1 mark)

SVT+ aberrant conduction

Name the features marked P, C and F and describe them. (3 marks)

Capture Beat: Narrow QRS followed by an upright T wave


Fusion beat: Wider complexes followed by inverted T waves
AV dissociation: p waves seen throughout ECG, not related to QRS complexes

What adverse signs would cause you to consider electrical cardioversion? 2 marks
SBP < 90
Chest pain
Heart Failure
Rate >150/min

If adverse signs are not present, give 2 treatment options (3 marks)


Amiodarone 150 mg iv over 10 minutes
Or lidocaine 50 mg over 2 mins repeated every 5 mins to 200mg

Postgrad Med J 2000;76:802 ( December )

Broken femur baby

A 2 month old child is brought to your A&E department with a grossly swollen right upper
leg. It has been X-rayed from triage and this shows a fractured femur. Mum says she rolled
off the bed when she wasn’t looking.

Apart from analgesia (next question) what would be your approach to the child? (1 mark)

ABCD. Check in particular CRT and other C indices. Gain iv access. Oxygen

What analgesia could you provide for the child (3 marks)

Splinting
Iv opiates
Femoral nerve block/catheter

The nurse says this sounds like a suspicious injury. What features in the history would make
you suspect non-accidental injury? (2 marks)

Delay in presentation
History inconsistent with development
Injuries inconsistent with history
Vague history
Changing history
Abnormal parental attitude/affect/concern
Frequent attendances

PRAY FOR ME Dr. Ashraf Elshehry Page 114


What features in the examination would make you suspect NAI? (2 marks)

Bilateral bruising esp to the eye


Bruising to non-trauma areas eg medial arms and thigh
Imprints of belts, teeth, slap
Bruising in a pre-toddler
Facial petechiae

What age would you expect a child to do these things? (2 marks)


Roll over 2-4 months
Sit up 6 months
Walk alone 1 year
Have finger-thumb grasp 9 months

AASAQ calfpain

A 21 yr old woman who is 16/40 pregnant presents with a woollen, erythematous, tender right
calf. She has no medical problems, takes no medication and her pregnancy is thus far
uncomplicated.

1. What is the differential diagnosis? 1 mark

Deep vein thrombosis, cellulitis, muscle sprain. 1 mark for > 1 answer

2. What are your options to investigate her? 1 mark

1 mark for any of U/S, AV1000 etc. No marks if D-Dimer included

3. What are the factors in the history for risk stratification of DVT? 3 Marks

3 marks for full house, 2 marks for most, 1 mark for some of

Recent trauma, surgery, medical illness, or immobilisation from any other cause; especially
in the presence of known thrombophilia, previous venous thromboembolism, pregnancy,
puerperium, age over 40 years, obesity, varicose veins, oestrogen use, malignancy, heart
failure, inflammatory bowel disease, or nephrotic syndrome

4. The patients in whom VTE can be excluded varies from hospital to hospital. Explain this.
3 marks

1 mark hospitals use different test kits


1 mark “sensitivity/specificity”
1 mark “low/medium risk groups” OR “pre-test probability”

5. How would you manage this patient if a DVT was confirmed? Why? 1 mark

1 mark for Heparin. 1 mark for Warfarin is teratogenic, contraindicated in pregnancy.

PRAY FOR ME Dr. Ashraf Elshehry Page 115


SIGN Guidelines

SAQ Chest trauma

According to ATLS

a)List 6 immediately life threatening conditions in chest trauma (2 marks)

Airway blockage Massive Haemothorax


Tension Pneumothorax Flail Chest
Open Pneumothorax Cardiac Tamponade

b)What criteria define Massive Haemothorax (2 marks)

>1500 ml immediate blood loss


Ongoing blood loss requiring transfusion

c)Name 8 potentially life threatening chest injuries which might be found on the secondary
survey (3 marks)

Myocardial contusion Traumatic aortic rupture


Pulmonary contusion Oesophageal rupture
Tracheo-bronchial disruption Simple pneumothorax
Traumatic diaphragmatic injury Mediastinal traversing wounds

d)Describe how you would perform pericadiocentesis (2 marks)

cardiac monitoring advance, withdrawing syringe


2 cm inf to xiphisternum withdrew if ECG changes
Aim for L scapula

e)A 70 Kg patient has just been intubated. At what minute volume would you set the
ventilator? (1 mark)

5-8 L min -1 (100 ml/kg/min)

Chest X-ray

PRAY FOR ME Dr. Ashraf Elshehry Page 116


PART 2

A 26 yr old male attends A&E complaining of a cough for the past 3-4 weeks, and now
increasing shortness of breath. He says he has been told to attend by his wife because he is too
short of breath to work. He is an air steward. PMH and DH nil.

On examination

Hr 96 regular HS 1+2 + nil


RS AE al zones, nil added
RR 36
SaO2 89% on air

You note these lesions on his chest

What other elements of history do you specifically require? (1 mark)

Sexual history, iv drug abuse

PART1
On the above CXR, name the anatomical features numbered
(3 marks)

PRAY FOR ME Dr. Ashraf Elshehry Page 117


1 R Atrium
2 Superior vena cava
3 Left ventricle
4 left atrial appendage
5 pulmonary artery
6 aortic knuckle

His CXR looks like this

What is the skin diagnosis? 1 mark


Kaposis sarcoma

What is the CXR diagnosis? 1 mark


Pneumocystis carinii pneumonia

What is the overall diagnosis? (2 marks)


AIDS with CD4 count < 200

Name 2 other conditions with which this illness might present (2 marks)
Oral candidiasis, CMV retinitis, lymphoma, M avium cellulare, TB, + many more

Child primary assessment

A six month old has been brought in to your department. It has taken reduced feeds over the
past 2 days and today has been refusing all. It is now drowsy.

In the primary assessment of the child (APLS)

Give 3 modes of assessing airway patency (1 mark)


Look for chest/abdo movement
Listen for Breath sounds/stridor
Feel for expired air

PRAY FOR ME Dr. Ashraf Elshehry Page 118


Give 6 ways of assessing the effort of breathing (3 marks)
Resp rate
Recession
Wheeze
Accessory muscle use
Stridor
Grunting
Flaring alae nasi

Give 2 ways of assessing the efficacy of breathing (1 mark)


Chest expansion/abdominal excursion
Breath sounds
SaO2 in air

Give 2 effects of breathing on other physiology (1 mark)


Heart rate
Skin colour
Mental status

Give 3 ways of assessing the circulation in a child (2 marks)


Heart rate
Blood pressure
Cap refill time
Skin temp and colour

Give 4 elements of assessing Disability in an unwell child (2 marks)


Mental status (AVPU)
Pupils
Posture
Neck stiffness/ raised fontanelle

PRAY FOR ME Dr. Ashraf Elshehry Page 119


Intensely itchy. 3 children have it.

Childhood dermatology
For each picture name the condition, causative organism and treatment.
(2.5 marks each)

Staphylococcal scalded skin


Staph aureus
Flucloxacillin

PRAY FOR ME Dr. Ashraf Elshehry Page 120


Impetigo
Staph aureus OR Strep. Pyogenes
Flucloxacillin or Fucidin cream

Molluscum contagiosum
Pox virus
No Rx as resolves spontaneously

Scabies
(Sarcoptes scabiei)
Permethrin, Benzyl benzoate or Malathion

Collapse with long prodrome


This 65 yr old woman is brought to your
department. She has been found on the floor,
after only a few minutes. She lives with her
husband who says she has been increasingly
withdrawn and depressed, with poor memory
and concentration over the past few months.
Her weight has increased considerably.

She has recently been started on


antidepressants and a Atorvastatin by her GP .

On examination GCS is 11, HR 40, BP 90/65,


T = 32 degrees

ABG on 2L O2
pH 7.1
pO2 12

PRAY FOR ME Dr. Ashraf Elshehry Page 121


pCO2 7
BE 0.8
U+E on ABG
Na+ 121
K+ 3.5
Gluc 3.5

What is the underlying problem? How do you arrive at this conclusion? (5 marks)
Hypothyroid coma

Hyperlipidaemia
Altered mental state, Weight gain, Low temp, Hyponatraemia, Hypoventilation
Typical facies

What two therapies are required? (2 marks)

Thyroxine (t3 or t4)


Corticosteroids

What could you ask the husband to confirm your diagnosis? (2 marks)

Other features of hypothyroidism: large tongue, hair loss, yellow skin, cold intolerance,
hoarseness, myalgia, goitre

What is the mortality of this condition? (1 mark)


50%

According to the BTS guidelines.


What are the5 core criteria for the severity of community acquired pneumonia and
what constitutes “severe pneumonia”.- 6 points

What antibiotic regime would you use for severe community acquired pneumonia
requiring hospitalisation- 3 points

Name 2 other criteria associated with bad outcome – 1 point

PRAY FOR ME Dr. Ashraf Elshehry Page 122


What is the diagnosis?
How would you treat it?
2 points

What is the diagnosis?


2 points

This patient presented to the department


with altered vision in one eye. A few days
before he had noticed a sudden onset of
black specks in front of that eye that
seemed to settle but now he keeps seeing
cobwebs in front of his eye and they not
getting better by rubbing. What is the
diagnosis?
What are the worrying features in the
history?
3 points

What is this condition called?


1 point

FFAEM April 2005 – as best remembered.

PRAY FOR ME Dr. Ashraf Elshehry Page 123


Question 1

A 75-year-old woman presents to the department complaining of chest pain. This was severe
and central associated with sweating and a feeling of nausea but no vomiting. It had lasted
about 2 hours by presentation. She had a past medical history of diabetes, ischaemic heart
disease and a raised cholesterol level.

Her blood results and ECG are as follows:

Hb 11.4 WCC 9.2 Plt 300


Na 140 K 4.4 Urea 5.1 Creat 60
Gluc 10.4 Chol 8.1 TnT 0.9

a) What is the diagnosis? (2)


_______________________________________________________________________
_______________________________________________________________________

b) List the TIMI criteria (5)


_______________________________________________________________________
_______________________________________________________________________
_______________________________________________________________________
_______________________________________________________________________
_______________________________________________________________________

c) You treat her with oxygen, 300mg aspirin, opiate analgesia and a nitrate. What other
treatments would you give her? (3)

PRAY FOR ME Dr. Ashraf Elshehry Page 124


_______________________________________________________________________
_______________________________________________________________________
_______________________________________________________________________

Question 2

A 35-year-old male presents to the department with a five-day history of generally feeling
unwell. In the past couple of days he had developed visible jaundice and abdominal pain in
his right upper quadrant. He described his urine as being dark. Previously he was fit and well
with no significant past medical history. He admitted drinking 18 units of alcohol per week.

Hb16.5 WCC 11.0 Plt400


Na 149 K 5.0 Urea 7.5 Creat 120 Glu 6.2

PRAY FOR ME Dr. Ashraf Elshehry Page 125


Ca 2.4 Bil 100 ALT 100 AST 900 Alk P 261 GGT 50

a) What is the pathological process? (2)


___________________________________________________________________
___________________________________________________________________
________

b) List the possible causes. (4)


___________________________________________________________________
___________________________________________________________________
___________________________________________________________________
___________________________________________________________________
________________

c) What further investigations need arranging? (2)


___________________________________________________________________
___________________________________________________________________
________

Question 3

A 70-year-old male presents to the department and is clearly very ill. In the later stages
of his resuscitation you place urinary and central venous catheters successfully in order
to monitor his condition. His bloods results show the following:

Hb 9.0 WCC 4.1 Plt 100


Na 143 K 7.3 Urea 44 Creat 500

a) For each of this mans problems list a treatment you would give explaining the
rationale for its use. You cannot, for example, give two treatments for his
potassium etc. (5)
___________________________________________________________________
___________________________________________________________________
___________________________________________________________________
___________________________________________________________________
___________________________________________________________________
____________________

b) List two causes for this problem. (2)


___________________________________________________________________
___________________________________________________________________
________

Question 4

A standby call is received in the department warning of a 9-year-old child who is fitting.

a) Complete the algorithm below including drug doses and timings. (7)

Airway
High flow oxygen
Glucose?

PRAY FOR ME Dr. Ashraf Elshehry Page 126


Vascular access

Vascular access
After achieving control of the seizures you examine the child and note a temperature of
38.2 and these lesion in the mouth.

b) What are the oral lesions? (1)


______________________________________________________________________
___

c) What is the diagnosis? (1)


______________________________________________________________________
___

d) List 8 notifiable diseases. (2)


______________________________________________________________________
______________________________________________________________________
______________________________________________________________________
______________________________________________________________________
______________________________________________________________________
______________________________________________________________________
______________________________________________________________________
______________________________________________________________________
________________________

Question 5

A twenty six year old female tax inspector presents to the department with an
exacerbation of her pelvic inflammatory disease.

a) What are the three diagnostic criteria for this condition? (3)

PRAY FOR ME Dr. Ashraf Elshehry Page 127


___________________________________________________________________
___________________________________________________________________
___________________________________________________________________
____________

b) List one further criterion. (1)


___________________________________________________________________
____

c) What are the indications for admission in this condition? (3)


___________________________________________________________________
___________________________________________________________________
___________________________________________________________________
____________

d) List three serious complications of this condition. (3)


___________________________________________________________________
___________________________________________________________________
___________________________________________________________________
____________

Question 6

You receive into the resuscitation room a 32-year-old male who has fallen from a
significant height. His chest x-ray is below:

a) Draw a diagram of the primary survey in this man. (5)

PRAY FOR ME Dr. Ashraf Elshehry Page 128


b) List two diagnoses on the chest x-ray. (2)
___________________________________________________________________
___________________________________________________________________
________

c) List two management priorities complete with appropriate anatomical landmarks.


(3)
___________________________________________________________________
___________________________________________________________________
___________________________________________________________________
____________

Question 7

An elderly patient presents to the department having been unwell for a couple of
days. She has had a cough with associated shortness of breath. She is an diabetic
using insulin for control. On examination she is febrile and has a fast respiratory rate.
Arterial blood gases are taken on high flow oxygen and a chest x-ray performed.

PH 7.2 PCO2 7.5 PO2 10 BE –10 HCO3 16

PRAY FOR ME Dr. Ashraf Elshehry Page 129


a) List 3 abnormalities on the chest x-ray. (3)
___________________________________________________________________
___________________________________________________________________
___________________________________________________________________
____________

b) What is the exact metabolic derangement on the blood gas? (2)


___________________________________________________________________
___________________________________________________________________
________

c) What antibiotics would you prescribe and why? (2)


___________________________________________________________________
___________________________________________________________________
________

d) List 4 other treatments and explain why they are of use. (4)
___________________________________________________________________
___________________________________________________________________
___________________________________________________________________
___________________________________________________________________
________________

Question 8

A 70 year old attends the department at 11o’clock at night. They have allegedly been
unable to get an appointment with the GP for a rash they have had for two weeks.
There are no systemic features.

PRAY FOR ME Dr. Ashraf Elshehry Page 130


a) Describe the rash. (2)
___________________________________________________________________
___________________________________________________________________
________

b) List the differential diagnoses. (3)


___________________________________________________________________
___________________________________________________________________
___________________________________________________________________
____________

c) What investigations are necessary in the emergency department? (2)


___________________________________________________________________
___________________________________________________________________
________

d) What follow up should be arranged for the patient? (2)


___________________________________________________________________
___________________________________________________________________
________

Question 9

PRAY FOR ME Dr. Ashraf Elshehry Page 131


A patient attends the department having recently returned from West Africa. Four days
after return to the UK they had started with a fever that had lasted now for 10 days.

a) What is the commonest group of diseases in travellers? (2)


___________________________________________________________________
____

b) List the differential diagnoses. (3)


___________________________________________________________________
___________________________________________________________________
___________________________________________________________________
____________

c) List four signs that you would look for. (4)


___________________________________________________________________
___________________________________________________________________
___________________________________________________________________
___________________________________________________________________
________________

d) List one important investigation. (1)


___________________________________________________________________
____

Question 10

Look at the picture below.

a) Describe the abnormality in the picture. (2)


___________________________________________________________________
___________________________________________________________________
________

b) List two systemic illnesses that are associated with this condition. (2)

PRAY FOR ME Dr. Ashraf Elshehry Page 132


___________________________________________________________________
___________________________________________________________________
________

c) List six steps in examining the eye. (6)


___________________________________________________________________
___________________________________________________________________
___________________________________________________________________
___________________________________________________________________
___________________________________________________________________
___________________________________________________________________
________________________

d) List two causes of a misshapen pupil. (2)


___________________________________________________________________
___________________________________________________________________
________

Question 11

A young woman is brought into the department. She has been found by her partner who
reports that she has recently been involved with the police and is due in court soon. The
problems have led to her having a depressed mood for which she is under treatment
from her doctor. On examination she is breathing slowly but she has a fast pulse rate.
Her blood pressure is low and her GCS is E1 V4 M4. Just as you begin to read her ECG
she begins to have a generalised tonic-clonic seizure.

PRAY FOR ME Dr. Ashraf Elshehry Page 133


a) What does the ECG show? (2)
___________________________________________________________________
___________________________________________________________________
________

b) List three immediate management procedures before any specific therapy. (3)
___________________________________________________________________
___________________________________________________________________
___________________________________________________________________
____________

c) List the specific pharmacological treatment including the dose. (2)


___________________________________________________________________
___________________________________________________________________
________

d) How does it work and how is the effect measured? (2)


___________________________________________________________________
___________________________________________________________________
________

PRAY FOR ME Dr. Ashraf Elshehry Page 134


Question 12

The department is put on standby by the local ambulance service for a 2-year-old child
from a nursery. He had apparently been chewing through a live electrical cable before it
exploded and the child had been thrown across the room. A nearby curtain had then
caught fire and burnt the child around the mouth and neck. Initial assessment by the
paramedics estimated a 20% partial thickness burn.

a) List the equipment you would prepare including drugs with specific doses. (5)
___________________________________________________________________
___________________________________________________________________
___________________________________________________________________
___________________________________________________________________
___________________________________________________________________
____________________

b) Describe the picture below and indicate any significance to the injury. (2)

___________________________________________________________________
___________________________________________________________________
________

c) What are the fluid requirements for this child in the first 24 hours? (3)
___________________________________________________________________
___________________________________________________________________
___________________________________________________________________
____________

Question 13

A three-year-old child is brought to the ED by her parents. She has had a cough for 2
days but today developed noisy breathing. There is no significant past medical history
and the vaccination schedule to date is complete. On examination the child is reasonable
well but with inspiratory and expiratory stridor. The chest is otherwise clear.

a) List the differential diagnoses of this problem. (4)

PRAY FOR ME Dr. Ashraf Elshehry Page 135


___________________________________________________________________
___________________________________________________________________
___________________________________________________________________
___________________________________________________________________
________________

b) What is the most likely diagnosis? (1)


___________________________________________________________________
____

c) List two specific pharmacological treatments with doses. (2)


___________________________________________________________________
___________________________________________________________________
________

d) List the criteria of the croup score. (5)


___________________________________________________________________
___________________________________________________________________
___________________________________________________________________
___________________________________________________________________
___________________________________________________________________
____________________

Question 14

A self-employed businessman attends the department with a laceration to his knee. He


smells strongly of alcohol and his speech is a little slurred. You notice that the history
provided to the triage nurse is different to the one provided to you and you are concerned
about this.

a) What are the signs of Wenicke’s encephalopathy? (4)


___________________________________________________________________
___________________________________________________________________
___________________________________________________________________
___________________________________________________________________
________________

b) Assuming the ABCs are normal, list the steps in his management and explain
why they are necessary. (6)
___________________________________________________________________
___________________________________________________________________
___________________________________________________________________
___________________________________________________________________
___________________________________________________________________
___________________________________________________________________
________________________

c) List the signs of alcohol withdrawal between 24 and 48 hours. (4)


___________________________________________________________________
___________________________________________________________________
___________________________________________________________________
___________________________________________________________________
________________

PRAY FOR ME Dr. Ashraf Elshehry Page 136


Question 15

An elderly diabetic patient is brought into the department with a left hemi neglect,
weakness and dysphasia. You diagnose an acute CVA.

a) What is the likely arterial region involved? (2)


___________________________________________________________________
___________________________________________________________________
________

b) List four interventions that have been shown to improve outcomes for this group
of patients. (4)
___________________________________________________________________
___________________________________________________________________
___________________________________________________________________
___________________________________________________________________
________________

c) List three criteria for thrombolysis in this group of patients. (3)


___________________________________________________________________
___________________________________________________________________
___________________________________________________________________
____________

Dermatology

Question 1:

PRAY FOR ME Dr. Ashraf Elshehry Page 137


A 34 year old man returned 1 week ago from a weekend in the ‘countryside’ with his
family. He has presented to the ED with a 3 day history of this lesion on his left shoulder.

1. Describe the lesion. 2 marks


2. What is this skin rash called? 1 mark
3. What is the causative organism? 1 mark
4. Is the patient contagious? 1 mark
5. List 2 recognised complications. 2 marks
6. What is the treatment? 1 mark

1. Extending macular rash with a central scaling puncum


2. Erythema chronicum migrans
3. Borrelia burgdorferi
4. Potentially via body fluids
5. Carditis and heart block
Bell’s palsy
Mononeuritis multiplx
Late: arthritis

6. Doxycycline, amoxicillin or cephalosporin for 2-3 weeks


Question 2:

PRAY FOR ME Dr. Ashraf Elshehry Page 138


A 64-year-old man presented with a large, painful ulcer on the lateral aspect of his left
calf. The lesion had begun as a small pimple a year earlier and, after breakdown, had
rapidly and continuously expanded. It was surgically debrided a few months ago,
when about half its present diameter. The patient had long-standing polycythemia
rubra vera for which he had undergone frequent phlebotomies. His medications
included furosemide, allopurinol, and hydroxyurea.

In laboratory studies, the hemoglobin concentration was 14.6 gm/dL, the white blood
cell count was 28,000/mm3, and the platelet count was 191,000/mm3. Electrolyte and
creatinine levels were normal, as were results of liver function tests.

1. What are your next 2 A and E investigations? 2 marks


2. What are your differential causes for this lesion? 2 marks
3. What is your treatment of this gentleman? 3 marks

Answers

This is hard! The actual diagnosis is pyoderma gangrenosum so just think of it as a


differential when you have an ulcer.

1. Doppler – localization of pulses


BM/blood glucose
Ankle/brachial pressure index
Wound swab
Xray for osteomyelitis

2. Ischaemic ulcer

PRAY FOR ME Dr. Ashraf Elshehry Page 139


Venous ulcer
Malignancy
Pyodema gangrenosum

3. Analgesia IF painful
Wound swab
Clean with N Saline
Dress
Advise leg elevation
Consider antibiotics – flucloxacillin 500 mg qds
Refer to tissue viability service/
Plastics for further care ?SSG/
Dermatology for help with diagnosis
District nurses/GP for regular review and dressings

The treatment of pyoderma gangrenosum is steroids po

Describe this injury (1)


Posterior dislocation of the thumb at the metacarpophalangeal joint

PRAY FOR ME Dr. Ashraf Elshehry Page 140


Which nerves would need to be blocked to reduce this injury under local anaesthetic? (2)
Radial and median nerves

What does this picture show? (1)

Bone scan of wrist showing increased uptake in left scaphoid region

What are the complications of this injury (2)


Avascular necrosis of proximal pole of scaphoid
Non union and Sudeks Atrophy/ RSD/Chronic regional pain syndrome
Describe the x ray below. (2)

PRAY FOR ME Dr. Ashraf Elshehry Page 141


There is a fracture of the distal radius.(1)
There is shortening and palmar/anterior angulation (1)
Smiths Fracture (1)
-1 for colles!!!

How would you manage this patient? (1)


Analgesia
Backslab POP
Refer orthopaedics. Unstable injury
½ each

A 56 year old gentleman attends the Ed with vertigo. He feels as though the room is
spinning and he has intractable vomiting. When you enter the room he is lying still on the
bed, every time he moves his head he feels like he is going to fall and vomits.
He has no previous medical illnesses and until this morning he was well except for a
minor upper respiratory tract infection.

What is the likely diagnosis?(1)


Viral labyrithitis

Give 8 other causes of vertigo. (4)


Menieres disease (vertigo, tinnitus and deafness)
Benign positional vertigo (elderly, lasts approx 2 mins with positional change)
Otitis Media
Acoustic neuroma (giddiness more than vertigo V, VI and VIII nerves plus ipsilateral
cerebellar signs, loss of corneal reflex)
CVA
Cholesteatoma
Trauma
Wax or FB in the ear
MS
Alcohol intoxication
Ramsay Hunt

You examine the patient and indentify no evidence of a central cerebellar lesion.
What are 6 signs of cerebellar involvement?(3)

PRAY FOR ME Dr. Ashraf Elshehry Page 142


Dysarthria / dysdiadochokinesis
Ataxia, truncal
Nystagmus
Intention tremor / past pointing
Staccato speech
Hypertonia / hyperreflexia

Name 2 drugs, with doses which you could use for this patient (2)
Prochlorperazine 5mg PO, 3mg Buccal or 12.5mg IM
Betahistine (SERC) 8mg PO
Cyclizine 50mg PO/IM/IV

A 75 year old female is brought to the ED by paramedics following an episode of


collapse. She has a history of MI, 4 years before and suffers with hypertension.
On arrival she has a heart rate of 52/min and BP of 110/75. She takes atenolol 50mg od ,
aspirin, ramipril and a statin. She states that she feels fine now but has has a few of these
episodes in the last month.

Her ECG is show below


Comment on the ECG (3)

1st degree heart block (prolonged PR)


Sinus Brady rate approx 50/min
LAD (Left anterior hemiblock)
RBBB
PRAY FOR ME Dr. Ashraf Elshehry Page 143
What is this this “block” called and what dangerous condition does it predispose to? (2)
Trifascicular block
Leads to complete heart block and risk of asystole

Whilst in the ED she becomes unwell, pale and clammy with chest tightness. She feels
dizzy and faint. She looks unwell but has an output and is semiconscious. Her last BP
reading was 60/30.
Her monitored rhythm is shown below

What rhythm is this? (1)


Complete heart block

Give 4 possible interventions/treatments in her management, excluding oxygen, IV fluids


and monitoring. (4)
Atropine 500mcg boluses to a maximum of 3 mg
Transcutaneous Pacing. May require sedation/opiates to tolerate
IV adrenailine infusion 2-10mcg/min
Venous pacing wire
Drowsy male

A 64 yr old west Indian male is admitted to your ED after his daughter has noticed he has
become increasingly drowsy over the past 48 hrs. He has a 10 yr history of diabetes for
which he was taking chlorpropamide. Before the deterioration he had complained of
increasing urinary frequency and excessive thirst. He recently saw his GP and was noted
to have a BP of 170/100 for which he was prescribed bendrofluazide.

O/E drowsy, apyrexial, answers appropriately. Decreased skin turgor. HR110 regular, BP
115/60. HS normal, chest clear. Abdo and Neuro exam normal.

Hb 15.0
WCC 13.0
Plt 179

Na+ 151
K+ 5.0
U 17
Cr 140

Urine Gluc +++


Protein +

What is the diagnosis? (1 marks)

PRAY FOR ME Dr. Ashraf Elshehry Page 144


HONK (hyper Osmolar non Ketotic Diabetic coma

List six investigations for this patient (3 marks)

Plasma osmolality, Blood glucose, Blood cultures, Urine culture, Chest Xray, ECG

List 2 possible precicpitants

MI, infection, thiazide diuretics, consumption of sugary drinks

List 4 steps in your management (4 marks)

Rehydration
Insulin infusion 1-3 units/hr
Anticoagulation
Treat underlying cause

Drowsy woman

A 67 yr old woman presents to your department with increasing thirst and drowsiness.
Her Calcium is 3.45 , U 10.1, Cr73

What symptoms are associated with this level of Calcuim? (2 marks)

Nausea/ Vomiting,
Alterations of mental status
Abdominal or flank pain (The workup of patients with a new kidney stone occasionally
reveals an elevated calcium level.)
Constipation
Lethargy Depression
Weakness and vague muscle/joint aches
Polyuria Headache

Give 5 causes of this level of Calcium (5 marks)

Primary hyperparathyroidism
Malignancy
Granulomatous disease
Sarcoidosis
Tuberculosis
Drugs
Thiazide diuretics
Vitamin D toxicity
Lithium
Milk alkali syndrome

PRAY FOR ME Dr. Ashraf Elshehry Page 145


Familial hypercalciuric hypercalcaemia
Endocrine
Thyrotoxicosis
Adrenal crisis
Immobilisation
Renal Failure
Aluminum intoxication

Describe your initial therapy of this patient (1 marks)

Rehydration with N saline

IF this is not effective, what would be your next tow therapeutic choices (2 marks)

calcitonin
biphosphonate

ECG Axis and LVH

On this ECG

1.Comment on the ECG

Left axis deviation with voltage criteria for LVH

2. Give three causes of Left axis deviation

inferior myocardial infarction, ventricular pre-excitation, hyperkalaemia, tricuspid


atresia, ostium primum atrial septal defect, artificial cardiac pacing, hypertension, aortic
stenosis or regurgitation, subaortic stenosis, mitral regurgitation, and left ventricular
conduction defects

PRAY FOR ME Dr. Ashraf Elshehry Page 146


3. Give three causes of Right axis deviation

COPD, pulmonary emboli, valvular disease, septal defects, and pulmonary hypertension.

4. What are the ECG criteria for Left Ventricular Hypertrophy

S wave in V2 plus R wave in V5 > 35.

5. Give two causes of Left Ventricular Hypertrophy

Aortic stenosis, hypertension

A 57 yr old man with previous MI’s presents to the A&E department complaining of
dizziness and shortness of breath for 4 hours. Discussion ensues as to the nature of the
rhythm on his ECG.

Name 3 possible rhythms this might be 2 marks


Ventricular Tachycardia, SVT with aberrant conduction, AF with preexcitation

Describe the following (which would support a diagnosis of VT) 3 marks


Capture beats: Normal QRS complex occurring after a p wave
Fusion beats: Combined normal and abnormal complexes
Concordance: All QRS complexes of the same polarity

Name 4 other feature which support a diagnosis of VT 2 marks

PRAY FOR ME Dr. Ashraf Elshehry Page 147


Very broad QRS, AV dissociation, bifid upright QRS with taller 1st paek in V1, Deep s
wave in V6

What actually is the rhythm? 2 marks

AF with preexcitation

How would you manage this patient if he was pain free and cardiovascularly stable?
1 mark
Chemical cardioversion

This 39 year old man attends the ED after an prolonged episode of palpitations. He has
had frequent episodes in the past of palpitations in the past but has never sought medical
advice. On his arrival the palpitations have stopped and he feels fine again.

Comment on the ECG. (2)

Sinus rhythm rate 90/min


L axis deviation
Short PR interval
Delta waves
Dominant R wave in V1
½ mark each

What is the likely diagnosis? (1)


Wolfe-Parkinson-White syndrome

Give 4 causes of a dominant R wave in V1 (2)


Normal in children
RVH
RBBB
Posterior MI
Ventricular pre-excitation (WPW)
Duchene muscular dystrophy
½ mark each

Give 6 ECG changes associated with hyperkalaemia. (3)


Small or absent p waves

PRAY FOR ME Dr. Ashraf Elshehry Page 148


Long PR
AF
Broad QRS
Axis deviation
Tall tented T waves
Shortened or absent ST segment
VF
Decreased R wave size
½ mark each

Give 4 causes of a prominent U wave. (2)


Hypokalaemia
Hypocalcaemia
Digoxin
Thyrotoxicosis
Class 1A (procainamide) and class 3 antiarrhythmics (amiodarone, sotalol)
Intracranial haemorrhage
Exercise
Congenital long QT syndrome
½ mark each

ECG young man

A 25 yr old man presents to your department with a 3 day history of chest pain, which is
retro-sternal and worse on lying down. Here is his ECG

What is the diagnosis? 2 marks

PRAY FOR ME Dr. Ashraf Elshehry Page 149


Pericarditis

What features on the ECG are typical of this? 2 marks

Widespread mild ST elevation and PR depression

What is the treatment? 2 marks

Oxygen and cardiac monitoring


NSAIDS

Name 1 possible life-threatening complication 1 mark

Cardiac Tamponade

Give 3 possible causes 3 marks

Viral (PR depression in 80% of these)


SLE, RA
TB
Acute MI or Dresslers syndrome

http://www.emedicine.com/EMERG/topic412.htm
Asked Nov 2002

Elbow injury

PRAY FOR ME Dr. Ashraf Elshehry Page 150


An eight year old child attends A&E having fallen off a trampoline onto her left arm. The
elbow is swollen and she does not want to move it. The arm is neurovascularly intact.

Give two radiographic features that suggest a fracture on this X-ray, and explain them (4
marks)

Fat pad sign


Displaced anterior humeral line: should pass through middle third of capitellum

Name the ossification centres of the elbow in the order they appear, and at what age (6
marks)
Capitellum 1
Radial head 5
Internal (medial) epicondyle 7
Trochlear 10
Olecranon 10
Lateral epicondyle 11

http://uwmsk.org:8080/residentprojects/stories/storyReader$77

Elderly trauma

PRAY FOR ME Dr. Ashraf Elshehry Page 151


A 72 yr old man is brought into your ED having fallen off a roof, where he was trying to
mend an aerial for a friend. His GCS is 15, HR 97, BP 170/90, sSaO2 97% on
100% O2. According to the ATLS manual

Name two problems for each of A, B, C, D and E which are specific to trauma of the
elderly? 5 marks
A:Dentures, Nasopharyngeal mucosal fragility, macroglossia/microstomia, cervical
arthritis
B:Diminished resp reserve, Use of O2 mandatory, chest injuries poorly tolerated, COPD,
Hypoxic drive
C: dec CVS reserve, fixed HR, Hypertension, loss of renal function, anticoagulants,
pharmacology
D:Acute and chronic subdurals, altered sensorium (cerebral atrophy) spinal arthritis->
more frequent fractures
E:Inc risk of undetected hypothermia, fractures harder to spot, pre-existing deformities,
osteoporosis

Name 2 factors in the history which should be emphasised in trauma in the elderly (2
marks)

Medications
Pre-existing conditions
Easy marks!

Name 4 groups of medications which may affect your resuscitation efforts? (2 marks)

Beta blockers, NSAIDS, anticoagulants, hypoglycaemics, Ca channel blockers, steroids,


diuretics, psychotropics

How would fluid resuscitation be altered in trauma in the elderly? (1 mark)

More aggressive resuscitation and monitoring

Emergency delivery

A 22 year old woman presents to the A&E department with severe cramping abdominal
pain which she recognises as labour pains. This is her third child and her second child was
a rapid but uneventful birth.

What would be your initial actions? (2 marks)

Call anaesthetist, obstetrician


Check BP and HR, gently examine abdomen
Provide Entonox
Iv access

PRAY FOR ME Dr. Ashraf Elshehry Page 152


Before transfer to Labout ward can be arranged baby’s head starts to appar at the vagina.
Describe the process of Delivery ( 6 marks)

Put on sterile gloves. Stand on patient’s right.


When head crowns, encourage mother to stop bearing down. Encourage rapid shallow
breaths
Use left hand to control rate of emergence of head
With right hand press on either side of anus with thumb and fingers
Once head is delivered, allow it to extend
Feel for cord around neck. If possible slip it over, if not, divide and clamp
Allow anterior shoulder to deliver first
Give 5U oxytocin and 500 mcg syntometrine im
Deliver baby

If the baby’s head emerges but the shoulder does not, what action should you take? (2
marks)

Shoulder dystocia:
Lay mother flat and bend knees up onto chest
Apply gentle digital pressure to try to deliver anterior shoulder. Gently bend baby’s neck
towards mother’s anus

A 21 year old man attends the ED with an injury to his right eye. A squash ball hit him in
the eye 1 hour ago. He is complaining of pain around the eye with intermittent double
vision. He is usually fit and well. He has a deep laceration to the medial canthus by the
lower lid.

What features would you examine for in this injury? (3)


Enophthalmos
Surgical emphysema
Visual acuity
Slit lamp examination and funduscopy e.g. hyphaema, pupil response
Eye movements
Diplopia esp. on upward gaze
Infraorbital nerve paraesthesia
½ mark each

What important structure is the laceration likely to have involved? (1)


Lacrimal cannaliculi/duct

You arrange for facial Xrays.


Comment on the x ray. (2)

PRAY FOR ME Dr. Ashraf Elshehry Page 153


Air-fluid level in right maxillary antrum
Orbital floor fracture with “tear drop” sign

He is referred to the MFU outpatient clinic in 2 days


What advice would you need to give this person? (2)
No nose blowing-risk of surgical emphysema and increased orbital pressure
Return if vision deteriorates-probably due to raised orbital pressure.

Whilst in the ED he complains of worsening visual acuity and pain. There is increased
bruising around the eye and the eye now looks proptosed and injected. His eye
movements are grossly reduced and his VA is 6/60 in the right eye.

What important complication may have arisen and what surgical procedure can be
performed in the Ed to save his sight?
Retrobulbar haemorrhage
Lateral canthotomy

Falls at home

A 76 yr old female was admitted to hospital after being found on the floor by the home
help. She complained of frequent dizzy spells , particularly on standing. She had several
falls at home and 2 hospital admissions, and was increasingly dependent on social
services. She also complained of lethargy, hoarse voice , intermittent confusion and
weight gain of 7 Kg in the past seven months. PMH OA. DH Paracetamol.

PRAY FOR ME Dr. Ashraf Elshehry Page 154


O/E pale and frail. HR 60 /min. BP 200/60 lying, 90/50 standing. JVP not raised. Slightly
displaced apex. Soft 1st and 2nd HS, nil added. Chest and abdo normal. Neuro normal.

Hb 10
WCC 6
Plt 149
Na+ 118
K+ 5.3
U 3.0
Cr 69
LFT Normal
ECG Normal
CXR enlarged heart

What is the case of her falls? (2 marks)


Postural hypotension

What condition could be causing this? (2 marks)

Addisons disease

What test could be used to confirm this? (2 marks)

Short synacthen test

What other biochemical abnormality do the symptoms suggest? (2 marks)

hypothyroidism

Name two tests, one to confirm the dignosis, and one to investigate an important
biochemical side effect of this condition (2 marks)
TFTs, random lipid profile

A 15 year old farmers daughter presents with a 5 day history of RUQ pain. On
examination she is pyrexial at 38.40 C. She looks unwell and jaundiced. Her parents say
she has been unhappy at home for the past month, since they came back from holiday in
Egypt

Investigations are as follows

Hb 8g/dl
WCC 18.8
Plt 140

PRAY FOR ME Dr. Ashraf Elshehry Page 155


Retic 20%
No fragmented red cells seen
PT 36 (control 12)
APTT 59 (control 45)

Na+ 136
K+ 4.9
U 26
Cr 400
Bil 50
AST 900
ALP 300
Albumin 33

HepB sAg Absent


HCV Ab Absent
CMV serologyAbsent
Monospot Negative

1.Suggest two possible infective diagnoses. (4 marks)


Leptospirosis (Weil’s disease)
Hepatitis A

2. Suggest two possible non-infective diagnoses (4 marks)


Paracetamol overdose
Iron overdose

3.Give 2 possible causes for the anaemia (2 marks)


Blood loss 2ndry to poor coagulation
Haemolysis

4. If fragmented red cells had been seen, what would this suggest? (1 mark)
Haemolytic uraemic syndrome

Question 1

A 76 year old man presents to the hospital with sudden onset shortness of breath. He is
afebrile. You arrange a CXR.

PRAY FOR ME Dr. Ashraf Elshehry Page 156


1. Describe four features of the CXR (2 marks)

2. What is the diagnosis? (1 mark)

3. Describe or draw the Starling Curve, and describe how it is altered in this
condition. (2 marks).

4. Give three drug treatments (other than oxygen) which you might use in this
condition, with routes and doses where appropriate. (3 marks)

5. Describe the stimulus for release of B-type natriuretic peptide, and describe it’s
action. (2 marks).
Question 2.

PRAY FOR ME Dr. Ashraf Elshehry Page 157


1. Give three predisposing factors to this condition (3 marks)

2. Give four signs on ocular examination which would assist in distinguishing


between orbital and periorbital cellulitis. (4 marks)

3. Give three possible causative organisms (3 marks)

Question 3

PRAY FOR ME Dr. Ashraf Elshehry Page 158


A 36 year old woman is brought to the department by police. She has a five week old
baby. She is withdrawn and refuses to answer questions, and soon after presentation
becomes aggressive and starts shouting and kicking walls. She has a bag which contains
baby milk and nappies.

1. Outline the four principles of the treatment of violent patients in the ED according
to NICE guideline.

2. How would you manage her acutely?

3. Give four possible organic causes of her presentation.

Question 4

A 36 week old woman presents complaining of abdominal pain. Her urinalysis is


suggestive of a UTI.

1. Outine three clinical features which would make you suspect pyelonephritis (3
marks).

2. In an otherwise healthy woman, what are the two commonest causative


organisms? (2 marks)

3. Give three complications of pyelonephritis in pregnancy (3 marks).

PRAY FOR ME Dr. Ashraf Elshehry Page 159


Question 5
An eight year old boy presents with a history of painful joints, associated with a rash. He
had a sore throat two to three weeks ago. You suspect he has acute rheumatic fever.

1. List the features which make up the diagnosis of acute rheumatic fever (3 marks) –
Whose criteria.

2. What single laboratory test is required to confirm the diagnosis (1 mark)

3. Outline four principles of treatment of acute rheumatic fever (four marks).

Question 6

A 46 year old woman presents with a history of cramping lower abdominal pain, which
has been coming on for the last few months. It is associated with her period, which has
been getting heavier. She has been depressed, and has seen her GP who has commenced
her on medication for this.

PRAY FOR ME Dr. Ashraf Elshehry Page 160


1. Give four of the recommendations concerning intimate patient examinations given
in recent GMC guidance.

2. Give three possible diagnoses to explain her symptoms.

3. Give the dermatomal innervation for the following pelvic organs:


a. Ovaries
b. Uterine fundus
c. Labia and perineum

Question 7

A 16 year old girl is brought to the ED. She admits to taking some paracetamol tablets 30
minutes ago. Although she’s not exact about the number of tablets taken, three packets are
missing, with 16 tablets in each.

1. What historical features would you wish to illicit? (3 marks)

2. What would be you immediate interventions? (2 marks)

3. What factors would result in her being in a high risk category for paracetamol
poisoning? (2 marks)

PRAY FOR ME Dr. Ashraf Elshehry Page 161


4. What clinical and laboratory features would be consistent with a diagnosis of
severe paracetamol poisoning (give six) (3 marks).

Question 8

An elderly woman is brought into hospital from her nursing home, where she has been
having bandage treatment of ‘infected leg ulcers.’ She’s become confused today, and has
a GCS of 11/15 on arrival.
Her investigation results are shown below:

Na 156; K 4.1; Urea 18; Creatinine 130; Chloride 120.


pH 7.23; P02 8.6; PC02 3.2; HCO3 20; BE -8.0

1. What is the likely diagnosis? (1 mark)

2. Calculate the serum osmolality (show your working) (2 marks)

3. Outline the principles of fluid resuscitation in this case (2 marks)

4. Give two drug treatments you would use in this circumstances (2 marks)

PRAY FOR ME Dr. Ashraf Elshehry Page 162


Question 9

A fifty six year old man is brought to the ED after an RTA. No other cars were involved,
but he veered off the road, and hit a tree. There was fifty cm of intrusion of the car.
Although restrained, he seems to have collided with the steering wheel, which is
deformed. There was no air bag in the car.

1. What four additional features do you require in the history at this stage? (2 marks)

He begins to become more short of breath, and his respiratory rate increases to 32.

2. What is the most important diagnosis to exclude now? (1 mark)

3. Name six clinical signs of this diagnosis (3 marks)

4. He deteriorates further and a chest drain is inserted, which drains a haemothorax. What
are the indications for a thoracotomy following a traumatic haemothorax? (4 marks)

Question 10

PRAY FOR ME Dr. Ashraf Elshehry Page 163


A thirty seven year old woman presents with a short history of abdominal pain and
nausea. She has become vomiting before presenting to the ED. She doesn’t remember
when her bowels were last open, and she isn’t passing flatus.
She had an appendicectomy aged six, but is otherwise fit and well.

1. Give two radiological abnormalities (2 marks)

2. Give four common causes (2 marks)

Question 11

Give three diagnostic criteria for SIRS (3 marks)

Give the diagnostic criteria for severe sepsis(3 marks)

Question 12

Pyeloneprhitis in pregnancy

1. Give two physiological reasons that pregnant women are more at risk of UTI.
2. In an otherwise healthy woman, name the two most likely causative organisms.

PRAY FOR ME Dr. Ashraf Elshehry Page 164


3. Give three clinical findings which would increase your suspicion of
pyelonephritis.
4. Give three potential complications of pyelonephritis in pregnancy.

Question 13

Neonatal jaundiced baby from Africa.

1. Give four causes which should be considered in any neonate presenting with
jaundice.
2. Give six investigations which you would consider in the ED.
3. Give six supplements or replacements which you would give to a neonate with
jaundice in the emergency department.

Question 14

A sixty five year old man presents to the ED with a painful swollen 1st MTPJ. He’s had
gout before, and you suspect this is the diagosis now.

1. Give the biochemical abnormality in gout and pseudogout.

2. Give two classes of drug, and an example of each, which may precipitate an
acute attack of gout.

PRAY FOR ME Dr. Ashraf Elshehry Page 165


You decide to offer him some dietary advice to try and reduce further attacks.

3. Give three foods which you would advise him to avoid, and give the reason
behind this.

Food:

Food:

Food:

Reason:

Allopurinol and colchicine may both be used to prevent attacks of gout. By what
mechanism do they do this?

Colchicine:

Allopurinol:

Question 15

A seventy six year old woman comes into the department complaining of sudden onset
shortness of breath. She has had numerous previous admissions and is on inhalers for
COPD.
Her observations are: P – 110; BP – 140/85; Sp02 – 94% on 24% 02

1. Give two drug treatments which will be effective in the next five minutes (give
names, routes and doses where appropriate).

You do an ABG which shows: pH 7.25; p02 7.9; pC02 8.4; HC03 36. You decide to
commence NIV.

2. What type of NIV would you commence?

PRAY FOR ME Dr. Ashraf Elshehry Page 166


3. You have a medical student with you. Give four points which you would make in
describing the type of NIV which you have commenced:
i)
ii)
iii)
iv)

She suddenly deteriorates and you arrange a CXR.

4. Give four abnormalities on the CXR (can’t get a picture to match theirs)

5. Give the next two steps which you would carry out.

Question 16

A thirty six year old woman presents to the ED with cramping abdominal pain and
diarrhoea. Although she feels nauseated, she hasn’t vomited. The diarrhoea is not bloody.
She’s just returned from holiday in North Africa, and the night before leaving had a
Bedouin barbeque on the beach.

1. What’s the commonest causative organism in travellers diarrhoea.

2. Give two organisms which might cause bloody diarrhoea.

3. Give four worrying clinical features in a patient with a history of bloody


diarrhoea.
i)
ii)

PRAY FOR ME Dr. Ashraf Elshehry Page 167


iii)
iv)

4. She still has some crampy abdominal pain, but a non-tender abdomen, her
temperature is 37.3, and her pulse is 90. You decide to treat her. Give two
treatment options.

i)

ii)

Question 17

A seventy six year old man presents with difficulty passing urine. Clinically he has a
palpable bladder. He is on lisinopril, aspirin and ‘a diuretic.’
His urea and electrolytes are shown below:
Na 135; K 5.6; Urea 17; Creatinine 180.

1. Give four steps, in order of priority, which you would carry out after
catheterisation

i)
ii)
iii)
iv)

2. Give four causes for his renal impairment.


i)
ii)
iii)
iv)

PRAY FOR ME Dr. Ashraf Elshehry Page 168


Question 18

A fifty six year old woman presents to the ED complaining of extreme nausea, and an
abnormal gait. Every time she tries to sit up, her symptoms get worse.

1. Give three features on clinical examination which would help to distinguish a


central from a peripheral cause of her vertigo.

i)
ii)
iii)

2. You decide that her vertigo has a peripheral cause. Give two differential
diagnoses.

i)

ii)

3. Give the name of a diagnostic test to determine between the two of these, and
describe how to carry it out.

Question 19 (Part a)

You receive a call about a nineteen year old man who is being brought to the
department. His temperature is 26 degrees, but he still has a palpable carotid pulse.

1. Except for resuscitation equipment, name four items which you would ensure
were present in the department prior to his arrival.

i)
ii)
iii)
iv)

En route to the ED he loses his output.

2. Give four changes to the standard cardiac arrest protocol for patients who arrest
with a temperature below 30 degrees.

i)
ii)
iii)
PRAY FOR ME Dr. Ashraf Elshehry Page 169
iv)

Question 19 (part b)

A different patient is brought to the department, having spent three nights living rough in
extremely cold weather. This is his hand:

1. Give two interventions which are indicated, and two which are contraindicated.

Question 20

A nineteen year old gardener presents to the department, having cut his top lip on a
branch.
You decide to repair his lip under a nerve block.

1. Describe how you would carry out this block

He didn’t have an initial tetanus course as a baby, but thinks he had a pre-school booster
12 years ago.

2. Describe four actions which would reduce his risk of tetanus.

i)
ii)
iii)
iv)

Describe four features which make a wound ‘tetanus prone.’

i)
ii)
iii)

PRAY FOR ME Dr. Ashraf Elshehry Page 170


iv)

PID

A 27 year old woman presents to the A7E department with low abdominal pain and
vaginal discharge. She has a tender lower abdomen with no signs of peritonism.

Hb 15
WCC 12
CRP 20
MSU -ve

Give four differential diagnoses (4 marks)

PID
Ectopic pregnancy
UTI
Appendicitis

Give three diagnostic criteria(3 marks)

Low abdominal pain


Adnexal tenderness
Cervical excitation

What are the indications for admission? (3 marks)

Unable to take oral medications


Unable to control pain with oral analgesia
Sepsis
Adolescent with poor circumstances
------------------------------------------------re is now a second OSCE exam involving
more interactive skills. This could involve life able to defend them!

PRAY FOR ME Dr. Ashraf Elshehry Page 171


AASAQ Femsim

A 35 yr old female is brought to your emergency department by her family who say that
she has suddenly started behaving strangely. She has had low in mood for a few days but
today has become completely catatonic. Her eyes are open but she will not respond.

Her airway, breathing and circulation are normal.

1. What features would you look for in the history that would favour an organic from a
psychiatric cause?
Disorientation, poor concentration, fluctuating course, Anxious, irritable, depressed,
Muddled, ideas of reference, delusions, misinterpretations, illusions, visual
hallucinations. Impaired memory, drug history. No previous psych involvement.
Headaches.

2.What features would you look for in your examination of the patient?
Features of sepsis, neurological examination, neck stiffness, rash

3. No psychiatric history. Gravida 4, Para 2. Recent joint and muscle pains. Examination
is unremarkable. T= 38.7
Bloods:

Na+ 140
K+ 4.0
U 12.0
Cr 107
CRP 8
ESR 120

Hb 8.7
WCC 2.3
Plt 350

Urine; Protein +++, Blood –ve, Glucose –ve, Pregnancy negative.

4. Give a differential diagnosis


SLE, encephalitis

5. What would be your next two investigations?


CT, LP

6. If these are normal, what would be your next investigation?


Anti Nuclear antibody - SLE

A 60 year old man presents to your department having tripped over the cat. He has no
other apparent injuries except for the fracture of his right femur as shown

PRAY FOR ME Dr. Ashraf Elshehry Page 172


Describe the Xray (2 marks)

Transverse fracture of the R femur, completely off ended. Bone shows patchy osteolytic
and sclerotic areas

What is the Diagnosis? What alterations in Biochemistry would you expect? (3 marks)

Paget’s disease. Increased Alk Phos all others normal.

How would you assess the cognitive function of this gentleman?


(5 marks)

Age
Time
Register address
Month
Year
d.o.b.
Place
Start of WW2
Monarch
Count backwards from 20-1
Remember address

<8 implies cognitive impairment

FFAEM APRIL 2002

Critical appraisal paper: JAMA Feb 14, 2001-vol 285 No 6, 761-68.

PRAY FOR ME Dr. Ashraf Elshehry Page 173


Diagnostic accuracy of a bedside D-dimer assay and alveolar dead-space
measurement for rapid exclusion of pulmonary embolism.

OSCE:

1.ECG of a man with marked ST elevation in the antero-lateral leads.


How would you manage?
A chest x-ray of the man showing florid acute pulmonary oedema 24 hours after
admission-what has happened?

2. History of a man who collapsed in the garden and brought by ambulance.


Vital signs pulse 120, BP 105/60, ECG shows ventricular tachycardia.
What drug would you give?
The drug has no effect, what would you do now?
Nurse then tells you his pulse has disappeared. What would you do?
List 4 causes of this problem

3.Picture of a hand with a pointing index finger and wasting of the first dorsal interossei.
What is the likely cause?
What are the possible causes?
Picture of difficulty with thumb abduction-what does this demonstrate?

4. 2 pictures of a man who has fallen from a tree with marked bruising of his shoulder and
neck. He has developed paralysis of the upper limb on the same side.
What is the most likely diagnosis?
What other injuries could have occurred?
What parameters comprise the revised trauma score (RTS)
What is the maximum score?

5. Chest-x-ray of a 65 year old male with bilateral shadowing as well as right hilar
shadowing. He has become increasing breathless over the past month.
Give 5 pulmonary causes.

6. CT scan of the head of a 12 year old boy in a road traffic accident.


Normal vital signs and GCS 15/15.
Give 3 abnormalities (intracranial air, small subdural, ?petechial haemorrhages)
How would you prepare him for transfer
What advice would you give to him as regards transfer?

7. 2 year old found by mother collapsed at home.


She was complaining of abdominal pain. On examination she was pale, sweaty,
tachycardic, tachynpoiec and hypotensive.
How would you first treat this patient?
YOU order a variety of tests-FBC, U&E & LFT- what other blood test is needed?
Arterial Blood gases show a metabolic acidosis, give 2 differentials.

8. Picture of 3 different ears:


a. inflammation of the anti-helix-management.
PRAY FOR ME Dr. Ashraf Elshehry Page 174
b. ? bitten ear
c. ? infected earlobe from embedded ear-ring

9.Chest x-ray of a young man showing large right-sided pneumothorax?


Vital signs normal. He keeps shouting I don’t want the police here.
Give 2 methods of treatment.
He refuses treatment, what would you do?
While you getting things ready, he leaves the department, what would you do?

10. Cervical spine x-ray.


Give 3 abnormalities-
subluxation of C5 on C6.
Tear drop fracture of C5
# lamina C5
Give 3 cardiovascular complications of this injury.

11.Elderly lady with septic shock (from the vital signs)


What is your first line treatment?
Baseline blood tests done (FBC,U&E)- what other blood test do you need to do?-Glucose.
After 2 litres of fluid, no improvement, what would you do?
Give other tests that you would do?
Urinalysis shows blood, protein and nitrites, what further treatment would you consider.
What is the likely diagnosis?

12. Chest x-ray of a man that has been stabbed in the chest.
Widespread shadowing left hemithorax ; Radiopaque object visible in the subcutaneous
tissue on the same side.
What is the treatment?
He wants to leave your department, what would you do?

MANAGEMENT VIVA

1.IN TRAY
a. Letter of compliant about a 60 year old man with chest pain seen by your SHO who
discharged the patient on the basis of a normal clinical examination and ECG.
Patient then collapses and dies at home. Post-mortem reveals an inferior MI.
The consultant cardiologist in the hospital confirms that the ECG did infarct show an
MI.
b. Your specialist registrar doing nights writes you a letter to say that the nurses are so
short-staffed at night that the department is not safe.

c. PCT meeting regarding out of hours provision by GPs while you are on holiday.
d. Letter from the radiology department to say as a result of staff shortage they would
no longer be able to provide a service for GP requests.
e. Advert on CHI inspection course
PRAY FOR ME Dr. Ashraf Elshehry Page 175
f. Advert on mattress that would prevent bed ulcers
g. Advert on medico-legal course
h. Advert on a symposium

In depth discussion on items a, b, c .

MANAGEMENT SCENARIO FOR ALL:


You are 6 months into post as a new consultant when you receive a letter from the
compliants/litigation officer saying you are negligent. The letter is accompanied by a
copy of an independent medical report prepared by a senior consultant in the hospital
accusing you of been negligent as regards a case you saw 3 months earlier.
What would you do? What would be your response?

Other SCENARIOS
1. 12 year old boy involved in RTA arrives critically injured.
Parents are Jehaviour Witnesses and would not allow blood transfusion. Would you still
give blood transfusion against their wish? And if so, how?

2.The media want to interview you regarding an approved female boxing match that is
due to take place in your department’s catchment area. What you be your response?

FFAEM April 2005

Short Answer Questions

1) A diabetic female in her 40’s presents with cardiac sounding chest pain. You are given
a set of blood results that are normal except for a Trop of 2.2 and an ECG that is normal
except for biphasic T waves in V1-3.
i) What is the diagnosis? (2)
ii) List the 5 components of a TIMI score. (5)
iii) Give 3 medications you would give her for ongoing pain other than 02, Aspirin
and nitrates. (3)

2) A male in his 30’s has had right upper quadrant pain for 5 days. It is associated with
jaundice, dark urine and nausea. He drinks 10u alcohol per week. His blood results show
elevated AST, GGT and Bili but a normal Alk Phos.

i) What type of picture is this? (2)


ii) List 4 causes of it (4)
iii) Give two further investigations you would perform on this patient? (2)
iv) (One other question - can’t remember what)

3) A 70 year old male presents with a two week history of pruritis and a generalised

PRAY FOR ME Dr. Ashraf Elshehry Page 176


rash. He is otherwise well. A photo shows his axilla with multiple clear vesicles and
scratch marks.

i) Describe the rash (2)


ii) Give 3 differential diagnoses (3)
iii) Give 2 tests you would perform in A&E to confirm the diagnosis (2)
iv) Who would you refer the patient to and what investigations would you expect
them to perform? (3)

4) A 2 year old child has bitten through the cable of a DVD player. He was thrown
backwards and set on fire. He has 20% burns and is en-route to your A&E Department.

i) Give 5 things you would prepare before his arrival. Include doses and the
equipment you would need as applicable. (5)

Picture of the child’s mouth. Shows burns and soot on the tongue.

ii) Describe the abnormality and immediate management (2)


iii) Calculate the child’s 24 hour fluid requirement. (3)

5) A 70 year old man presents with a 9 month history of severe lower back pain. His
blood results are: Hb 9, WBC 4, Plt 65. Na 140, K 7.0, Ur 40, Creat 500, Ca 3.1, Phos
2.9. LFT’s normal. He has a CVP line, urinary catheter and has been cannulated.

i) Give 2 differential diagnoses


ii) List 5 different treatments and the reasons why they are necessary.

An ECG is enclosed that has features of Hyperkalaemia (I think)

iii) Describe 3 ECG abnormalities

6) A young female; with a history of depression, is due in court tomorrow and is found
collapsed at home. She is drowsy with a BP 120/70 and HR of 120.
Her ECG is pictured and shows a broad complex tachycardia.

i) Describe the ECG abnormalities (1)


ii) What is the diagnosis (2)

Whilst you are examining her she starts to fit and becomes hypotensive.

iii) List 3 immediate management steps (3)


iii) What drug would you use to terminate her fit, include the dose (2)
iv) Why would you use it and what ECG changes would you expect? (2)

7) A young man presents with a painful red eye.


PRAY FOR ME Dr. Ashraf Elshehry Page 177
A photograph is included that shows a red eye with a an irregular pupil

i) Describe two abnormalities (2)


ii) List six points you would include in the examination (3)
iii) What is the diagnosis (1)
iv) Give 2 related systemic illnesses (2)
v) Give 2 differential diagnoses (2)

8) A 40 year old man presents with a temp of 39.5oC. He returned from a 10 day holiday
in West Africa 4 days ago. His blood results show very low platelets, a WBC of 13 and
slightly low Na and K.

i) What is the most common cause of fever in travellers returning to the UK? (1)
ii) List 3 specific tropical diseases that would account for the patients findings (3)
iii) List 4 things you would look for in the examination. (4)
iv) Give one diagnostic test you would perform to confirm the diagnosis (2)

9) A 70 year old man with no known allergies presents with a 3 day history of a cough
and shortness of breath. He is hypotensive and tachycardic.
His ABG shows pH 7.2, PO2 10.2, PCO2 7.2, BXS –10, HCO3 16.
A CXR has a fluid level/effusion in the left base.

i) Give an exact description of his ABG(1)


ii) List 3 abnormalities on his CXR (3)
iii) Which antibiotics would you prescribe (2)
iv) List 5 treatments and why (5)

10) A 9 yr old boy with a temp of 39.5oC presents fitting.

i) Complete the flow chart below including doses and times

Blank flow chart from APLS manual for fitting child (7)

Photograph of inside of child mouth showing spots on mucosa. (Koplick)


ii) What is the diagnosis (1)
iii) List eight notifiable diseases

11) A 28 year male has fallen from a significant height. His ABC’s have been evaluated;
he has a pulse of 120, a Bp of 80/40 and a palpable central pulse only.

i) Draw a flow chart covering the steps of the primary survey (5)

A CXR is pictured. (Tension pneumothorax)


PRAY FOR ME Dr. Ashraf Elshehry Page 178
ii) Describe the CXR abnormalities (2)
iii) Describe two interventions you would perform including land marks (3)

12) Your SHO has examined a 28 year old female who she thinks has PID.

i) What are the 3 cardinal features of PID (3)


ii) List one other important feature (1)
iii) List 3 reasons for admission (3)
iv) List 3 serious complications (3)

13) A 3 year old child presents with recession, a RR 45, PR 160 and inspiratory and
expiratory stridor. His immunisations are up to date.

i) List 4 differential diagnoses (2)


ii) What is the most common cause in the UK (1)
iii) List 2 medications that are known to improve outcome (2)
iv) Which 5 components make up the croup severity score (5)

14) A 40 year old chronic alcoholic has sustained a knee laceration during a fall. His
story is inconsistent and keeps changing.

i) List the 4 diagnostic features of Wernickes encephalopathy


ii) List 5 immediate management steps
iii) List 4 features of acute withdrawal that are seen in the first 24-48 hours

15) A 70 year old male presents with left hemiplegia, left facial weakness, inattention,
dysarthria but no dysphasia.

i) Which vascular territory is the lesion (2)


ii) List 4 things known to improve outcome other than thrombolysis(4)
iii) List the 4 criteria that must be met before thrombolysis (4)

FFAEM NOVEMBER 2005 SAQ

Q1 Paediatric AXR picture Dilated small bowel loops


13/12 old Hx of vomiting, bloody diarrhoea wt 10.5kg
(intussception?)

PRAY FOR ME Dr. Ashraf Elshehry Page 179


Diagnosis
Causative factors
Fluid Requirements first 12hrs
2 treatment options

Q2 Picture of rash on palm


Hx of possible scabies

Describe picture
What causes symptoms of itching?
2 treatments you would prescribe
What will you tell the patient?

Q3 Retinal Photograph of ‘stormy sunset’


(aka Central Retinal Vein Occlusion)

2 Abnormalities
Diagnosis
Risk Factors/Causative factors

Q4 Child with Meningitis type Hx with low Sodium

Differential Diagnosis
What may cause low Na? (adrenal haemorrhage?)

5 A&E tests you would do to look for cause of low Na

Q5 Seizure in alcoholic patient

3 treatments
4 causes why alcoholics are prone to fitting

Q6 Young male with Hx of possible PE

What 3 risk factors may be present according to BTS guidelines?


What tests to do in A&E?
What 2 treatments given in A&E

Suddenly collapses in A&E with low BP


What 1 treatment would you use?

Q7 Young male brought in by sister. Suicidal intent but wants to leave

What elements in Hx may indicate serious harm?


PRAY FOR ME Dr. Ashraf Elshehry Page 180
Describe elements of Mental Health Act (which parts?)
3 Fold test for testing capacity/consent

Q8 Man attends poisoned on train by Organophosphates


Others affected
(aka Major Incident Management)

As well as calling A&E consultant, what else do you do?


Describe muscarinic effects of Organophosphate poisoning
3 drugs used in treatment

Q9 X ray picture of Supracondylar # Humerus in a child


Hx of fall; c spine x rays taken

Describe x ray
What 2 neurological complications may result?
Name 3 criteria you would use to ‘clear the neck’
Name another joint injury and describe its neurovascular complications

Q10 X ray picture of Facial bones with fracture orbital floor

Describe 3 abnormalities in x ray


Describe assessing parts of orbital cavity/contents
Describe management

Q11 X ray picture of C spine #dislocation


Hx of major trauma; CT Head – DAI; CT CAP NAD
BP 90/60 PR 60 after 2L fluid

Describe X ray
What accounts for Cardiovascular status?
Describe clinical features of spinal injury at this level (C5/6)

Q12 ECG of Broad Complex Tachycardia


Hx of Lawyer depressed recently; court case tomorrow; brought in drowsy, low
BP, fits.
(aka TCA overdose??)

Which 1 drug used in treatment


How does it work?

Further management

Q13 35/52 Pregnant confused fitting; high BP low Hb, Plts, Elevated LFTs, Renal
Failure & K=6.0
PRAY FOR ME Dr. Ashraf Elshehry Page 181
Comment on FBC & cause
What drug would you use to control BP?
Describe further management

Q14 3yr old boy; croup type Hx

5 features of Risk scoring (aka Westley)


3 drugs that may be used
What 2 criteria used for admission

Q15 ECG of bradycardia following Digoxin overdose; high K; pH 6.96; renal failure

Describe ECG
3 indications for Digibind
Further management

Q16 X ray picture of CXR RUZ shadowing


Hx 3/12 hx cough, fever; partner is HIV +ve with TB

Describe X ray
3 possible diagnoses
6 Ix you would do in A&E
3 chest pathogens in HIV

1.

35 yr old male awakens short of breath and sats 92%


a) 4 risk factors you would seek
b) BTS guidelines, Three other ED investigations to investigate other causes

Ddimer 300 (n <224)


Pt collapses hypotensive, GCS 12
c) What single management would you consider

2. Young child with stiff neck, drowsy, no rash, na 124

1) 2 neurological diagnosis
2) Explain the low sodium
3) 6 investigations for cause of low sodium

PRAY FOR ME Dr. Ashraf Elshehry Page 182


3. 35/40 pregnant lady BP 165, proteinuria +++++ Haematuria, plt 38, wcc14
bili 79 Hb 8

1) 2 features of FBC, and explain


2) what is the diagnosis (pre-eclampsia +- HELLP)
3) what is the management of the blood pressure (drug and route)
what 3 other things to do in ED

4.
Picture of rash (poor)
Your SHO thinks this is scabies
a) describe the rash (2points)
b) Diagnosis + differential diagnosis
c) Two treatment options
d) Two pieces of advice for patient

5.
68 male, Overdose of digoxin, level 16ng/ml (n<2)
bloods showed renal faliure, K+ 6

12 lead ECG (atrial flutter, variable block, reverse tick, ??prolonged QT)

a) 4 points on the ECG


b) 3 reasons for digibind
c) 4 other immediate management points

6.
60 male quiet withdrawn male

a) 6 features you would seek suggesting risk of suicide


b) 4 key features of MHA in your country of work
c) 3 key features of assessment of competance
7.
34 woman appearing in court today for fraud

unrousable from sleep


Pt drowsy. Pupils 4mm

ECG

1) describe the ECG, what is the cardiac diagnosis


2) what is the underlying diagnosis
3) what drug would you give for the cardiac abnormality (route & dose), how does it
work, how would you monitor it?
4) What 4 other management steps apart from O2

8. Man Assaulted. Facial XR, laceration under eye.


1) 3 features on XR (blood in sinus, crescent sign, teardrop sign)
2) 4 points on examination of orbit and its contents
PRAY FOR ME Dr. Ashraf Elshehry Page 183
3) 3 points in management

PRAY FOR ME Dr. Ashraf Elshehry Page 184


9. Fundoscopy picture
1) what 6 points would you explore in history
2) describe the picture (2 points), what is the diagnosis
3) what 6 associations with this condition

10. young child with stridor, with recession


1) 4 other diagnoses
2) croup scoring: 4 out of 5 features to assess severity
3) 3 drugs and routes
4) 2 reasons to admit

11. Man arrives with 3/12 history of SOB, cough


Male partner HIV +ve being treated for TB

1) describe XR (RUZ collapse)


2) 3 differentials from TB
3) 3 lung problems (?organisms) in HIV
4) 6 points for Ix in the ED

12. Man arrives in department saying exposed at railway, collapes


Other people affected
1) apart from calling ED consultant what 4 steps would you take
2) what 4 features of muscarinic poisoning woould you expect
3) what three treatments would you give (apart from O2)

13. Chronic alcoholic ongoing seizure BM 2.2


1) define Status Epilepticus
2) what 3 treatment options would you give this patient
3) still fitting 10 mins later, two treatment options
4) 4 things predisposing this man to seizures

14.
Intermittent abdo pain in child after viral infection (with rash)

a)what is the diagnosis


b)what 3 predisposing factors
c) what analgesia (dose and route)
d) what 2 options
d) 3 points for your SHO to manage fluid requirments

15.
Head injury kite surfing. GCS 8
CT loss of grey/white differentiation
Xray of C-spine + tomogram
2 litres fluid P65, bp 95/40

1)4 features of the xray and tomogram (c5 # ant vertebral, ?peg,?soft tissue,)
2)comment on cardiovascular status
3)given the cardiovascular status and xray what is the diagnosis
PRAY FOR ME Dr. Ashraf Elshehry Page 185
4) what 4 other features

16.
XR of supracondylar #

1) Describe the Xray


2) two neurological complications, and how would test this
3)
4) another joint + injury leading to neurovascular compromise

Fractured wrist

An 8 yr old boy attends your department having fallen out of a tree. His only injury is to
his left ankle, which is swollen and painful. The X-ray is thus:

Describe the X-ray appearances (2 marks)


Fracture through the epiphysis and epiphyseal plate of the tibia

What is the name of the classification of fractures of this type, and what type is this? (2
marks)

Salter Harris 3

Describe the other types of fracture in this classification (4 marks)

1 slip at growth plate


2 through metaphysic and growth plate
3 through epiphysis and growth plate
PRAY FOR ME Dr. Ashraf Elshehry Page 186
4 through both
5 crush at growth plate

What will be your disposal of this patient and why? 2 marks

Refer Ortho. SH 3 needs fixation as involves articular surface

1) A 24 year old women presents to the ED with high fever and vomiting and diarrhoea.
She has a widespread blanching macular erythematous rash and looks unwell. Her pulse is
120/min and her BP is 93/54. Her Temp is 39.4°C. She is currently on day 4 of her
menstrual period and has a tampon in situ. She is receiving 15l /min O2 and is receiving
intravenous fluid bolus when your SHO asks you to see the patient. She is attached to
continuous monitoring.

What is the most likely diagnosis? (1)


Toxic Shock Syndrome

What is a usual causative organism? (1)


Staph. aureus. (endotoxin)
Strep pyogenes (exooixin)

Other than oxygen and intravenous fluids, outline your initial management and
investigations. (3)
(half mark each)
Vaginal examination
Remove Tampon
Vaginal Swabs
U&E, FBC, LFT, ABG
Blood Cultures
ECG
CXR
IV Flucloxacillin +/- Benzylpenicillin or other anti staph cover
Contact ITU if refractory hypotension despite fluids

2) After dealing with this patient, another SHO asks you about a 32 year old patient with
panless vaginal bleeding who is 10 weeks pregnant. He asks you about indications for
anti-D.

Which patients need anti-D? (1)


Rhesus –ve mothers within 72 hours.

Which blood test can we do to check for fetomaternal haemorrhage?(1)


Kleihauer

Which conditions should you consider giving anti-D? (3)


delivery of Rh D-positive infant
threatened or spontaneous abortion
- any after 12 weeks
- any before 12 weeks that require instrumentation (e.g. dilatation and curettage)

PRAY FOR ME Dr. Ashraf Elshehry Page 187


- any before 12 weeks if the bleeding is heavy or associated with abdominal pain
antepartum haemorrhage (APH)
closed abdominal injury (e.g., in road traffic accident)
ectopic pregnancy
intrauterine death.

A 42 year old man with a history of many years of alcohol abuse presents to your
department reporting an episode of haematemesis. In the department he has a further,
large haematemesis. His BP is 90/45, HR 127, GCS 15

What would be your initial steps in the 1st 5-10 minutes, management and investigation( 3
marks)

Gain 2 x large bore iv access


x-match 6u blood
Check clotting screen, FBC, U&E, LFT

There is no endoscopy available for 6 hours. What pharmacological agents are available,
and what is their mechanism of action? (4 marks)

Vasopressin – reduces portal blood flow, portal systemic collateral blood flow and
variceal pressure

Somatostatin – causes splanchnic vasoconstriction, reducing portal pressure and blood


flow

These agents have no effect. What further two steps could you take to try to arrest the
bleeding. (2 marks)

Correct clotting
Sengstaken Blakemore tube

If these and gastroscopic banding are ineffective, what further intervention may be tried?
(1 mark)

TIPSS Transjugular intrahepatic portosystemic stent shunt

A 68 year old male is brought to the ED by ambulance. He has become confused and
unwell over the last 2/7. He is clearly confused. His temperature is 38° C. He has a pulse

PRAY FOR ME Dr. Ashraf Elshehry Page 188


rate of 110/min. His oxygen saturation on air is 91%. He has no other PMH of note other
than mild OA. He takes no regular medication and is a non smoker. He has no allergies.

Comment on his CXR (1)

Left lower lobe consolidation

How do we assess severity in community acquired pneumonia and which other


information do we require to complete the assessment of severity?(4)
CURB-65
Urea > 7
Respiratory rate >30/min
Bp (Systolic <90mmHg or diastolic <60mmHg)

Outline your management of this patient. (3)


Oxgen, 15l/min by rebreather mask. Sats monitoring
Iv access. Iv fluids 1l saline over 1hour
FBC, U&E, Blood Cultures, Sputum culture
ABG
IV antibiotics (hospital, severe so Augmentin or 3rd gen cephalosporin plus
Clarithromycin)
Refer HDU/ICU
½ mark each.

Give 2 pulmonary complications of community acquired pneumonia. (2)


Lung abscess andeEmpyema or parapneumonic pleural effusion

House fire
PRAY FOR ME Dr. Ashraf Elshehry Page 189
A 45 year old man is brought in having jumped form a first floor window in a house fire.
He is immobilised on a spinal board. HR 126, BP 100/40, SaO2 92% in air, GCS
E3V3M5

What factors would you consider in deciding the management of this patient’s airway? (4
marks)
Stridor/immediate airway compromise
Facial burns
Singeing of eyebrows/nose hairs
Carbon deposits on oropharynx
Carbonaceous sputum
Hoarse voice
Hx impaired mentation/confined in burning building
Explosion with burns to head and torso
COHb >10%

Primary survey does not reveal any injury. The patient has significant burns to his upper
limbs and face and head. You arrange for the patient to be intubated. ABG reveals a
COHb of 40%

What is a normal COHb? (1 mark)


Up to 15% in a smoker

What symptoms might be expected in a patient with raised COHb? (1 mark)


Headache and nausea, Confusion, Coma, Death

What is the half life of CO Hb On air and On 100% O2? (2 marks)


Air 4 hrs
100% O2 40 mins

He has partial thickness burns as shown. Estimate the area (2 marks)

Arm = 9%. Head = 9%. Whole head + 2x 2/3 arms =9+6+6+21%

PRAY FOR ME Dr. Ashraf Elshehry Page 190


A 25 year old male is brought into the ED by his friend. He has been unwell on the night
out and his friend state that “far a laugh they out something in his drink. He looks drunk
but doesn’t smell of alcohol. He is vomiting and slurring his speech. His friend says that
they work in a garage and have been using antifreeze today.

What treatment options are available to you? (2)


Ethanol 125-150ml as whisky, gin or vodka orally then IV (5% solution in dextrose)
initially at 12g/hr
Fomepizole

What investigations would you want to carry out and what would you be looking for? (4)
ABG for acidosis
U&E ATN and renal failure
Calcium for hypocalcaemia
Plasma ethylene glycol level for the obvious
ECG for arrhythmia

His ABG shows a raised anion gap metabolic acidosis with pH of 7.0
What would you give now? (1)
Sodium bicarbonate to get ph >7.2

Give 6 causes of metabolic acidosis with normal anion gap. (3)


Hyperalimentation, hyperventilation (chronic)
Addisons, acids e.g. HCl, acetazolamide
Renal tubular acidosis
Diarrhoea
Uterosigmoidosomy
Pancreatic fistula

Legal issues

A 13 year old girl attends for the morning after pill, 48 hrs after Unprotected Sexual
Intercourse

What issues are raised? 4 marks

Fraser rules
Parental involvement
GU disease
?Child abuse

What will you ask the girl? 2marks

Are parents aware?


Is sexual intercourse consensual?

PRAY FOR ME Dr. Ashraf Elshehry Page 191


How will you proceed? 4 marks

Give Morning after pill if pt competent


Arrange GU follow up
Arrange proper contraceptive follow up
Try to persuade pt to involve parents

A 54 year old man presents to the ED with abdominal pain and confusion. He is jaundiced
with obvious spider nevi and asterixis He looks unwell. He is agitated, restless and
hyperreflexic.His abdomen is distended and tense and generally tender. He has a fever of
38°C and his pulse is 110/min and BP 90/60.
He admits to drinking 1 litre of spirits per day, and has done since his wife died 4 years
ago. He is known to the Gastroenterologists at your hospital.

You take routine bloods and resuscitate him as appropriate

His results are as follows:


Hb 9.6 (macrocytic) Na 125 Bili 302
WBC 25.0 K 3.2 GGT 450
Platelets 56 Urea 1.3 AST 500
PT 75 secs Creat 94 ALT 560
INR 2.3
ABG normal
BM 2.5

What are the most likely diagnoses? (2)


Grade 3 Hepatic encephalopathy on background of alcoholic liver disease
Spontaneous bacterial peritonitis

Which one simple investigation would you like to carry out? (1)
Ascitic tap to rule out SBP

Excluding initial resuscitation and ABC, what treatment would you institute for this
patient? (4)

Lactulose 20g bd via NG tube


Oral non absorbable antibiotics e.g. neomycin 1g PO 6 hrly
IV dextrose 10% 1litre 12 hrly plus 50ml 50% dextrose IV if BM <3.5
IV antibiotics e.g. Cefuroxime 1.5g IV plus Metronidazole 500mg IV or 1-2g Ceftriaxone
Vitamin K 10mg IV

PRAY FOR ME Dr. Ashraf Elshehry Page 192


Give 6 causes of acute liver failure in an otherwise heathly individual presenting to the
ED. (3)
Paracetamol OD
Infection e.g Hep A, B, C, D, EBV, CMV
Drugs esp Halothane, TB treatment, valproate, Ciprofloxacin, aughmentin
Illicit drugs e.g. Cocaine, ecstasy
Magic Mushrooms (Amanita Pylloides)
Herbal Remedies e.g. ginseng
Salicylates e.g. reyes syndrome
Budd-Chiari and Portal vein thrombosis
Acute fatty liver of pregnancy
Carbon tetrachloride
Weils disease, Wilsons disease
Malignancy

Long QT

A 34 year old man attends your department after an RTC, complaining of chest pain. An
ECG is taken. The report of the ECG is that there is a prolonged QT interval.

How is the QT interval defined? How does it change with rate? (1 mark)

Start of QRS complex to end of T wave. Decreases with rate.

What is QTc, how is it calculated and why. What is the normal QTc? (1 mark)

QT over square root of RR interval. Corrects for change in QT with rate.


Normal is < 0.44

Give 4 classes of medication that prolong QT interval (4 marks)

Class 1 and 3 antiarrythmics


Macrilodes

PRAY FOR ME Dr. Ashraf Elshehry Page 193


Quinolones
Phenothiazines
TCA’s
SSRI’s
Methadone

Give three non inherited conditions that prolong QT interval (3 marks)

Congestive heart failure


Myocardial infarction
Hypocalcaemia
Hypomagnesaemia
Rheumatic Fever
Myocarditis

The patient is not on any medications and has no apparent reason to have a prolonged QT
interval. What difference could it make to the patient? (1 mark)

Inc risk of torsades de pointes and sudden death. Avoid drugs which inc QT

Look at the following lesions:

This lady attends the department with this rash on both shins.

What is the rash? (1)


Erythema Multiforme

List 4 associated causes. (2)


Drugs: Barbitutares, Sulphonamides
Infections: Herpes, mycoplasma, Orf
Collagen disorders
Idiopathic 50%

What is this condition called in its more severe form and what is the drug of choice in its
treatment? (2)
Stevens-Johnson syndrome: effects mouth, genitalia, bronchial tre and eyes with fever.
Treatment is high dose steroids

Another patient presents to minors with this painful rash on his shins

PRAY FOR ME Dr. Ashraf Elshehry Page 194


What is the rash? (1)
Erythema nodosum

Name 4 conditions which are associated with this condition?(2)


Sarcoid
Drugs: sulphonamides, OCP and dapsone
Bacterial Infectons: Streptococcus, mycobacterium-TB and Leprosy)
Also UC, Crohns, Leptospirosis

Name 4 other cutaneous manifestations of systemic disease. (2)

Pretibial myxoedema (Hyperthyroid)


Striae (Cushings)
Spider nevi, palmar erythema (liver disease)
Necrobiosis lipoidica. Granuloma annulare (DM)
Pyoderma Gangrenosum (UC, Crohns, RA)
Rheumatoid nodules (RA)
Acanthosis nigracans, dermatomyositis (Malignancy)
Dermatitis herpetiformis (Coeliac)
Erythema Chronicum Migrans (Lyme disease)
And many, many more…….

A 27 year old male has fallen onto his buttocks from a height of 4 m whilst skateboarding
at a local ramp. He complains of severe back pain. The lateral view of L3 is shown below

PRAY FOR ME Dr. Ashraf Elshehry Page 195


a)Describe the myotomes and dermatomes below (4 marks)

Dermatome Myotome
Anterior thigh Hip extension
L3 Knee flexion

Ant thigh, ant knee Knee extension


L4 Medial foot Foot dorsiflexion

b) What features of a similar injury would lead you to suspect it was unstable? (4 marks)
anterior margin < ½ posterior margin (posterior complex rupture)
avulsion fracture at tip of spinous process
wide separation of vertebral spines at level of injury
facet joint or pedicle fracture
facet joint dislocation
c) At what stage should he be removed from the Spine board? (1 Mark)
as soon as possible

d) What other injuries are associated with this injury? (1 Mark)


other spinal injuries, base of skull fracture

Major Incident

You are the most senior doctor in your department when a call comes through of a bus
crash and a major incident standby

PRAY FOR ME Dr. Ashraf Elshehry Page 196


Who will you inform (2 marks)

Consultant, shift leader, hospital manager

What will be your actions (1 mark)


See how many patients are in dept. Warn staff

The Standby is then altered to a Major incident declared. You are still the most senior
doctor in the department as the bus crash has created severe traffic congestion.

What will be your first actions? 1 mark


Initiate Major incident callout
Allocate roles from major incident cupboard

How will you deal with the patients in the waiting room? 1 mark
Tell them to go home or to a WIC

How will you deal with the patients in the Minors area? 1 mark
Same

How will you deal with the patients in the Majors area? 2 marks
Either one patient to each ward, or to eg day case surgery ward

When patients start to arrive, what should be your role while you are the senior doctor? (1
mark)
Triage at the door

Name types of triage used in a major incident. 1 mark


Sieve and Sort

March 2006 SAQ Paper

1. A 26 year old man attends the ED at 6pm on a Friday evening. He is complaining of


lower abdominal pain and dysuria. The GU clinic is closed for the next 3 days as it is a
long weekend.

His urinalysis shows

Blood ++
Protein +
Glucose –ve
Nitirites (nitrates were negative) ve
Leucocytes negative

i) What features in the history would suggest an STD rather than a UTI (2)

PRAY FOR ME Dr. Ashraf Elshehry Page 197


i)

ii)

You decide it is an STD.

ii) What 3 things would you like to do before commencing empirical therapy? (3)

i)

ii)

iii)

iii) What antibiotic regime would you commence and for how long? (3)

i)

What 4 things would you tell him before he leaves the department? (2)

i)

ii)

iii)

iv)

2. An 18 month old child is brought into the ED by his mother. He has been playing with
a few 10p coins and she thinks she saw one in his mouth.

i) What 3 questions would you ask in the history? (3)

i)

ii)

iii)

ii) What (two things would you include in your) examination would you make? (2)

i)

PRAY FOR ME Dr. Ashraf Elshehry Page 198


ii)

iii) What two methods of investigation could be carried out in the ED? (2)

i)

ii)

iv) describe 3 sites and the vertebral level at which foreign bodies are most likely to get
stuck (3)

site: level:

site: level:

site: level:

3. A 34 year old women returns to the ED. She was seen the day before with a history of
malaise, arthralgia, fever and dry cough. She was seen by one of the SHOs and sent home
as a viral illness. She returns to the ED SOB with sats of 94% on high flow oxygen.
This is her xray. (I am sure xray showed RML as well as bibasal shadowing)

PRAY FOR ME Dr. Ashraf Elshehry Page 199


Comment on her x-ray (2)

What tests in the ED could you perform to confirm your diagnosis? (2)

i)

ii)

What type of NIV would you use if indicated? (1)

i)

How could you improve compliance with NIV? (3)

i)

ii)

iii)

There was an ECG question for 2 marks but no ECG in the data for this question.

Q4. A 35 year old man comes to the ED with bloody diarrhoea and abdominal pain. He
has a 5 year history of Crohn’s disease. He has severe pain, looks unwell (he has
guarding) and has a temperature of 37.8°C. He is tachycardia at2 120/min

Comment on his X ray. (2)

PRAY FOR ME Dr. Ashraf Elshehry Page 200


i)

ii)

iii)

iv)

What antibiotics and by what route would you give? (2)

i)

ii)

Give 2 other drug treatments and routes (2)

i)

ii)
Give 4 skin and musculoskeletal manifestations of inflammatory bowel disease (4)

i)

ii)

PRAY FOR ME Dr. Ashraf Elshehry Page 201


iii)

iv)

Q5. A 46 year old woman attends the ED with pins and needles in her toes, she feels
clumsy and has (blurred) double vision. She had an “upset stomach” last week. She is
usually fit and well.

Examination reveals power 4/5 in her lower limbs with absent deep tendon reflexes and
diplopia on right lateral gaze.

What is the likely diagnosis? (1)

i)

What are 2 other possible differential diagnoses? (2)

i)

ii)

What (three) investigations would you do in the ED and why (3)

Give nerve root innervation of the following reflexes. (5)

Jaw

Supinator

Triceps

Biceps

Knee

Ankle

PRAY FOR ME Dr. Ashraf Elshehry Page 202


Q6. A mother brings her 10 year old Caucasian child to the ED with lethargy and malaise
for a few weeks. She is limping and complaining of hip pain. There is gross limitation of
movements of the left hip.

Bloods are: Hb 8.4


Plts 20
WBC 28
Retics 0.5%

Give 4 possible diagnoses (4)

i)

ii)

iii)

iv)

What investigations would you like to perform? (3)

i)

ii)

iii)

iv)

v)

vi)

Comment on the blood results (3)

i)

ii)

iii)

PRAY FOR ME Dr. Ashraf Elshehry Page 203


Q7. A 51 year old man comes to ED by ambulance. He has been up all night vomiting.
This morning he has suddenly got worse. He is SOB, his RR is 40. He has a pulse of 130
and he is pyrexia at 38.5° C
Hb 12.7 WBC 9.8 MCV 106 ALT 50
Plts 150 ALP 75 Bili 9 GGT 252
Comment on his Xray (2)

i)

ii)

What is this condition called? (2)

i)

What investigations/ therapeutic measures could confirm the diagnosis? (2)

i)

ii)
8
Comment on his blood results (2)

i)

ii)

In view of history and blood results, what is the likely underlying aetiology/precipitant (1)

i)

PRAY FOR ME Dr. Ashraf Elshehry Page 204


Q8. A 25 year old man comes to hospital complaining of a sore (red) left eye for 2 hours.
He was hammering metal yesterday and wears permanent contact lenses. His vision is
slightly reduced in the effected eye.

What questions might you want to ask in the history? (4)

i)

ii)

iii)

iv)

v)

vi)

What features might suggests penetrating eye injury on inspection of the anterior aspect
of the globe? (3)

i)

ii)

iii)

You decide this is a penetrating eye injury. What 3 drugs would you give? (3)

i)

ii)

iii)

Q9. A 9 year old boy (25kg weight) is brought into the Ed by his mother. He is
dehydrated and lethargic. He looks unwell.

Give 5 specific features to suggest 5% dehydration. (4)

i)

ii)

iii)

iv)

He has received resuscitation fluids and his deficit has been calculated.
PRAY FOR ME Dr. Ashraf Elshehry Page 205
Calculate his maintainance fluid requirements for the next 8 hours. (3)
Show your calculations

Give 3 other tests you would like to do in the ED. (3)

i)

ii)

iii)

PRAY FOR ME Dr. Ashraf Elshehry Page 206


Q10. A 37 year old male comes to the ED with a 3 month history of malaise, night sweats
and weight loss. His partner is HIV positive.

Comment on his chest x ray (1)

Give 3 possible diagnoses of the x ray findings. (4)

i)

ii)

iii)

iv)

List 3 common respiratory pathogens in HIV (2)

i)

ii)

iii)

Give three items in the BTS guidelines that can help assess this patient? (3)

i)

ii)

iii)

PRAY FOR ME Dr. Ashraf Elshehry Page 207


Q11. A 14 year old girl comes to the ED with lower abdominal pain. She is here without
her parents and pregnancy test is positive. She is requesting a termination. She is in a
relationship with a 19year old partner. (She does not want her parents notified)

How do you assess someone has capacity? (3)

i)

ii)

iii)

What things must you ensure regarding Gillick competence (3)

i)

ii)

iii)

On closer questioning she tells you she has had multiple sexual partners who are all adult
and provider by her 19 year old boyfriend.

What do you do now? (4)

i)

ii)

iii)

iv)

PRAY FOR ME Dr. Ashraf Elshehry Page 208


Q12. A 37 year old woman comes to ED. She has recently been diagnosed with SLE and
has been taking regular Ibuprofen for joint pains. She has come in because her ankles,
hands and eyelids are swollen and she feels very breathless. Her exercise tolerance in
greatly reduced. She was fit and well prior to this. Her bloods show
Na 145 K 6.7 Urea 16.7 Creat 135 FBC normal
Metabolic acidosis on ABG Total Protein 24 LFT normal

Comment on her ECG (2)

i)

ii)

She has runs of VT. Other than antiarrythmic medication what drug treatment would you
begin? (2)

i)

ii)

What other investigations (three things in the urinalysis) would you do? (4)

i)

ii)

iii)

Give 2 processes to explain her blood results (2)

i)

ii)

Q13. A 95 year old with advanced alzheimers is brought in from a nursing home. He has
a 3 day history of the rash seen below. He has a low grade fever, has a pulse of 100 and a
BP of 102/65
PRAY FOR ME Dr. Ashraf Elshehry Page 209
Give 4 causes of the picture below (4)

i)

ii)

iii)

iv)

What one question (in the standard medical history) would (help you) out?(1)

i)

What is Nikolsky’s sign? (1)

i)

What treatment would you give? (4)

i)

ii)

iii)

iv)

Q14. A 72 year old man has fallen from a horse. He is immobilised and brought into the
ED on a spinal board. His pulse is 80 per minute and his BP is 102/64. He is complaining

PRAY FOR ME Dr. Ashraf Elshehry Page 210


of abdominal pain and back pain. He has a past medical history of atrial fibrillation and
hypertension.

Name two medications other than warfarin that he may be taking which may effect his
physiological status. (2)

i)

ii)

You decide to do a FAST scan. Give 4 limitations to FAST scan. (4)

i)

ii)

iii)

iv)

The surgeons decide to take him to theatre – what two things would you do now with
doses (2)

i)

ii)

What are the complications of the medications in iii) (2)

i)

ii)

Q15. A 45 year old lady attends 1 week post abdominal hysterectomy for fibroids. She is
known to have hyperthyroidism, on treatment. She attends with a tachycardia of 140 bpm.
She is pyrexial with a temp of 39.1°C and is agitated. Her abdominal wound is well
healed and her abdomen is soft. She may have had a seizure too.
You decide the diagnosis is thyroid storm.

Give 4 other possible differential diagnoses that could account for her symptoms. (2)

i)

ii)

iii)

iv)

PRAY FOR ME Dr. Ashraf Elshehry Page 211


Give 4 non-drug interventions (management) in her management. (4)

i)

ii)

iii)

iv)

Give 4 drug treatments in this specific case and their purpose (4)

i)

ii)

iii)

iv)

Q16. A 27 (69kg) year old female attends the Ed with the butterfly from her ear ring stuck
in her lobe.

Draw the ear, its sensory innervation and the landmarks for nerve blocks that would
enable you to remove the butterfly. (5)

Calculate the dose (mls) of 1% plain lidocaine you could use for this procedure. Show
your calculations. (2)

Give 3 systems affected by local anaesthetic toxicity and how they are affected. (3)

i)

ii)

iii)

PRAY FOR ME Dr. Ashraf Elshehry Page 212


FFAEM OSCE May 1998

Question 1
A 68 year old man with a PMH of MI presents with chest pain and a BP of 86/53.
ECG shows broad complex tachycardia.

a. What is the diagnosis?


Ventricular Tachycardia

b. How would you confirm this on the monitor?


Print rhythm strip and identify presence of capture beats/ fusion beats

c. How would you treat the patient?


ABC with high concentration Oxygen facemask
Aspirin 300mg
Sedation (etomidate)
Synchronised cardioversion 100/200/360J

d. If the patient was asymptomatic how would you treat him?


ABC with high concentration Oxygen facemask
Amiodarone 150mg over 10 minutes. If this fails:-
Correct serum potassium/magnesium and synchronised cardioversion 100/200/360J

e. When would you use Magnesium and in what dosage?


Torsades de Pointes
5mls 50% Magnesium Sulphate over 30 mins.

Question 2
Photograph of foot with redness around 1st MTPJ. The patient is a hypertensive 46
year old female who has recently been to a dinner/dance. She has dyspepsia.

a. What is the diagnosis?


Acute gout

PRAY FOR ME Dr. Ashraf Elshehry Page 213


b. Give three possible precipitants in this patient.
Drugs (Thiazide diuretics or aspirin)
High purine content of meal (fish/meat/cheese)
Alcohol
Trauma (?someone stood on toe dancing)
c. What is the best method of confirming the diagnosis?
Arthrocentesis and polarised light microscopy for urate crystals
(strongly negatively birefringent needle shaped crystals)

d. How would you treat the patient?


Colchicine 1mg initially then 0.5mg every 3 hours until stools become loose or 6mg total
taken (NSAID’S relatively contraindicated in setting of dyspepsia and concurrent use of
aspirin)
Advise rest/ increased fluid intake/ avoid purine rich foods
Advise GP follow up re: allopurinol when acute episode resolved

Question 3
An 18 year old female is brought to A&E by her parents. She was well until the
previous evening and has no past medical history. She has a GCS of 10. Blood results
are as follows:-
Glucose 6
Urea 5

P02 10

PH 7.3
PC02 3.5
Bicarb 10

a. What is the diagnosis?


Metabolic Acidosis with partial respiratory compensation
Meningitis/Encephalitis
Salicylate overdose
Tricyclic Overdose

PRAY FOR ME Dr. Ashraf Elshehry Page 214


b. What other test would you do to confirm this?
Salicylate levels
ECG
CT head

c. How would this latter test affect your management?


May give sodium bicarbonate
May perform haemodialysis
Exclude intracranial haemorrhage

d. What is the prognosis?

Question 4
A young man in his 20’s has been shot in the right upper chest.

a. What does the chest X-ray show?


Fractured right 1st and 2nd ribs
Wide mediastinum

Pleural caps

Surgical emphysema
Right pulmonary contusion
MORE LIKELY HAEMOTHORAX!

b. Give 4 reasons for cardiothoracic referral


Traversing penetrating chest injury - Need to explore wound in theatre
Haemothorax draining>1500mls initially or >200mls per hour for 2 consecutive hours
Mediastinal vessel injury requiring repair
Open pneumothorax

PRAY FOR ME Dr. Ashraf Elshehry Page 215


c. What other radiographs should be performed and why?
Spinal series (exclude obvious bony spinal injury)
X-ray neck and abdomen/pelvis (always perform radiographs at least one body segment
above below body segment entered)
Spiral CT with contrast to identify aortic inj

Question 5
An 18 year old male presents with headache and fever. His GCS is 12. The casualty
SHO orders skull X-rays. There are the following abnormalities:-
Air in frontal lobe outside sinuses

a. What is the diagnosis?

Kippel Feil Syndrome

Short neck, decreased cervical ROM, and a low hairline (in 40-50% of patients)

Decreased ROM is the most frequent clinical finding. Rotational loss usually is more
pronounced than is the loss of flexion and extension.

Other patients present because of facial asymmetry. Neurological problems may develop
in 20%

Mandibular dysplasia

Associated with Still’s Disease (hepatosplenomegaly)

Question 6
A photo shows a young male with his head on a pillow. The patient has a right
periorbital haematoma and fluid dripping out of his right ear which is forming two
rings on the pillow.
a. What is this sign called?
Halo sign (Double ring sign)

b. Describe the physiological basis of it.


Blood and CSF have different densities and therefore they separate (CSF travels further)

PRAY FOR ME Dr. Ashraf Elshehry Page 216


c. What is the diagnosis?
Base of skull fracture (+/- right zygomatico-maxillary frontal complex fracture) with CSF
leak

d. What investigation should be performed?


C-spine series X-rays
CT head and facial bones if facial views suggest a fracture
Examination of his right ear/ ? glucose stix on fluid or beta-transferrin

e. How should the patient be treated?


C-spine immobilisation/ Analgesia/ Neuro-Observation
Ophthalmic/ ENT /neurosurgical review
Question 7
A 42 year old female with alcohol dependence has a three day history of abdominal
pain and a three month history of amenorrhoea. A photograph of her abdomen
shows that it is swollen with some flank and peri-umbilical discolouration and some
red spots.

a. What is the differential diagnosis?


Ruptured ectopic pregnancy (likely fundal)
Haemorrhage from ruptured ovarian cyst
Haemorrhagic pancreatitis

b. What does the photograph show?


Abdominal distension
Cullen’s sign
Spider naevi

c. What two tests should be performed?


Pregnancy Test
Ultrasound or CT Abdomen depending on if ectopic or pancreatitis more likely

d. Describe the initial management


ABC with high concentration oxygen by facemask

PRAY FOR ME Dr. Ashraf Elshehry Page 217


2 large iv cannula antecubital fossae
Fluids as necessary to maintain BP 90-100mm Hg systolic
Analgesia
X-match 6 units
(FBC/COAG/U&E/AMYLASE/LFT’s)
Inform O&G registrar urgently and inform anaesthetist/theatre
500 IU of Anti-D Imunoglobulin if indicated

Question 8
A 88 year old lady has fallen. What does the Pelvic X-ray show?
Displaced subcapital NOF
Fractured inferior pubic ramus
Lucent areas in the pelvis
Constipation

Differential Diagnosis of Solitary Lucent Bone Lesions

Solitary Bone Cyst

Aneurysmal Bone Cyst

Fibrous Dysplasia

Enchondroma / Ecchondroma

Osteoid osteoma

Eosinophilic Granuloma

Metastasis / Myeloma

Haemangioma

Osteomyelitis

Hyperparathyroidism (brown tumors)

PRAY FOR ME Dr. Ashraf Elshehry Page 218


Differential Diagnosis of Multiple Lucent Bone Lesions

Metastasis / Myeloma

Renal Osteodystrophy (Looser’ lines)/ Hyperparathyroidism (brown tumors)

Haemangioma

Fibrous Dysplasia

Osteomyelitis

Eosinophilic Granuloma

Differential Diagnosis of Focal or Multifocal Sclerotic Bone Lesions

Vascular Primary

Haemangiomas, Infarct Osteoma, Osteosarcoma


Infection Metastatic

Chronic osteomyelitis Prostate, Breast


Endocrine/Metabolic

Paget's disease

Question 9

A young male adult claims that he was punched by a policeman a week ago. He
presents with left cheek pain and swelling. (Facial X-rays show a teardrop sign).

a. Describe the X-ray appearances


Step in orbital floor
Opacification left maxillary sinus (blood)
Teardrop sign (herniation of soft tissues through orbital floor)

b. What points are of greatest importance in the physical examination?


Examine visual acuity and for diploplia
Examine range of eye movements (restricted upward gaze – inferior rectus trapping)
Palpate orbital floor for step/ zygoma/maxilla
Check sensation of cheek/side of nose (infra-orbital nerve)
Look for enophtalmus/ proptosis (retrobulbar haematoma)
Fundoscopy for retinal detachment

PRAY FOR ME Dr. Ashraf Elshehry Page 219


c. How should the patient be treated?
Prophylactic antibiotics on max-fax advice (e.g. Augmentin orally)
Warned not to blow nose until max-fax review
Analgesia
May need surgical repair of orbital floor when swelling sunsides
Check tetanus status

OSCE May 2001


Question 1
Young child with ?UTI.
a. List 3 methods for collecting a urine specimen, with their advantages and
disadvantages.
Clean catch MSU after perineal wiping (front to back) –
Contamination less likely than bag method. Non-invasive.
Requires older child and co-operation, may take some time to receive sample

Bag (eg Hollister U bag) or adsorbant pad after skin cleaned


Easy to use and apply. Non-invasive. Useful for infants.
May take a while for sample to arrive and likely to be contamination from skin flora.

Suprapubic aspiration
Fast, useful in sick infants. Least likely method to suffer from sample contamination.
Invasive. Theoretical possibility of bowel perforation.

b. Criteria for diagnosis of UTI in child?


Urine culture grows 100,000 CFU’s/ml
OR Organisms on microscopy if aspirated sample and performed by appropriately trained
person
NB dipstix cannot prove/exclude UTI in children

c. Does the child need follow up?


Yes. Antibiotics until Ultrasound of kidneys/ureters +/-DMSA scan +/- micturating
cystourethrogram.

PRAY FOR ME Dr. Ashraf Elshehry Page 220


d. If so why?
Significant proportion of children with UTI’s will have Vesico-ureteric reflux that in
young children can result in renal scarring. This can cause hypertension and renal failure
in later life. May also identify congenital abnormalities e.g. structural abnormalities
(posterior urethral valves/ polycystic kideys)

Question 2
Fundoscopy. Photo of proliferative diabetic retinopathy.
Annotate the photo.
Microaneurysms
Blot haemorrhages
Hard exudates (lipid)
Circinate above macula (oedema)
Cotton-wool spots (nerve infarction)
Venous irregularity and bleeding
Leashes of new vessels
Photocoagulation scars.

What's the diagnosis?


Proliferative diabetic retinopathy

Question 3
CXR of a right spontaneous pneumothorax and a massive left sided bullae.
a. Would you let your SHO put a chest drain in?
Yes, with supervision if not suitable for aspiration
Another Medical CXR
b. List 3 abnormalities

Question 4
ECG. Broad complex tacchycardia. VT.
a. Describe the rhythm?

PRAY FOR ME Dr. Ashraf Elshehry Page 221


Broad complex tachycardia. VT.
b. Management of VF?

c. What does the ECG now show?


Posterior MI.

Question 5
X-ray. Lunate dislocation
a. Method of reduction?
Open reduction and intercarpal ligament repair
Alternatively, GA closed reduction
Hyperflexion and dorsal translation of the lunate which converted the injury to a
perilunate dislocation. Subsequently, reduce the distal carpal row over the lunate.
X-ray Subtle # of 5th MC

Question 6
Eye photo
a. Diagnosis?
Subconjuntival haemorrhage.

b. Further examination required?


Check can see posterior aspect (and look for signs of base of skull fracture)
Blood pressure

Eye photo 2. Infected ulcer/wound on forehead.


c. Diagnosis
Ophthalmic Zoster (trigeminal nerve)

d. Complications?
Ulcerative keratitis
Corneal scarring
Trigeminal neuralgia
Secondary bacterial infection
PRAY FOR ME Dr. Ashraf Elshehry Page 222
Question 7
Childs Elbow X-ray
a. Describe abnormalities
Periosteal reaction on humerus (?Old injury)

Child AP and Lateral CXR


b. List 3 abnormalities

Dislocated hip in child


c. Complications?
Sciatic nerve injury
Avascular necrosis femoral head
Myositis Ossificans
Late OA Joint
Acetabular fracture

Question 8
Patient presents with ?PID
a. 4 symptoms
PV discharge
Lower abdominal Pain
Fever
Systemic upset eg vomiting

b. 4 signs
Lower abdominal tenderness
Cervical motion tenderness

PRAY FOR ME Dr. Ashraf Elshehry Page 223


Adnexal tenderness
Raised Temperature

c. 2 organisms
Neisseria gonorrhoea
Chlamydia Trachomatis

d. Definitive test
Laparoscopy

e. Management
Exclude Pregnancy
Analgesia
High vaginal swabs and endocervical swabs
Ciprofloxacin (single dose), Metronidazole (1 week) and Doxycycline(2 weeks)
Advise re: decreased OCP effect and alcohol interaction with Metronidazole.
GU follow up for contact tracing repeat testing/ HIV counselling etc.

Admit under gynae if


Peritonitis, systemic upset, requiring iv analgesia, predicted poor compliance

Question 9
Kid with purpuric rash
a. Diagnosis?
Henoch Schonlein Purpura
b. Acute and long term complications
Acute

Arthralgia/Arthritis - most commonly involving the knees and ankles

Gastrointestinal – Abdominal pain and vomiting (85%), Intesusseption, Bleed

Renal

Nephritic syndrome – Haematuria

PRAY FOR ME Dr. Ashraf Elshehry Page 224


Nephrotic syndrome - Oedema

Long term
Chronic HSP
Renal Failure/hypertension

c. Tests
Urinalysis
U&E
FBC
COAG
Ultrasound abdomen (if concerns re:intesusseption)
Renal or skin biopsy would prove diagnosis

d. Follow up
GP test urine periodically (blood or protein) and monitor blood pressurefor 6 months. If
normal at this stage then no further follow up necessary. Children with renal impairment
need longer follow up by the paediatric team.

e. Treatment
Supportive e.g. fluids and rest
NSAID’s for arthralgia (if no renal impairment)
Treat renal impairment/abdo pain/intesusseption as necessary
Question 10
Multi-trauma CXR
a. List 5 abnormalities?
Clavicle #
Rib #'s
Ruptured diaphragm
Pulm contusion
etc.
b. Management?

PRAY FOR ME Dr. Ashraf Elshehry Page 225


c. Investigations?

d. Interventions?

Question 11
CXR of Asian lady with TB
a. Describe X-ray
Primary pulmonary tuberculosis
Atelectasis, parenchymal consolidation, lymphadenopathy, pleural effusion and a miliary
pattern. Any lobe may be affected, although lower-lobe mostcommon.
Reactivation tuberculosis
Usually upper-lobe involvement with cavitation in 50 percent. Atypical radiographic
findings are extremely common in HIV-infected patients

CXR of Smoker with collapse/consolidation


b. Describe X-ray

Question 12
Paracetamol OD
a. Assessment of suicidal risk factors
SADPERSONS
b. Management of delayed presentation
N-acetyl cysteine if >150mg/kg taken or 12g
Supportive E.g. anti-emetics/analgesics/fluids as necessary
Admit medically
Paracetamol/Salicylate levels
Monitor INR/U&E(creatinine)/LFT’s +/- ABG
Involve liver team if failure predicted (INR>2 at 24 hours)
Psychiatric assessment as inpatient

Dear Michelle

Hope this is helpful

SAQ topics
PRAY FOR ME Dr. Ashraf Elshehry Page 226
1. Addisons-- asked about a woman who had collapsed and fallen down a flight of stairs,
possible DD, then said that she had an ABG, what could that ABG mean, then said she
was wearing a medicalert bracelet-- what did that make us this of, and what was the ONE
lifesaving treatment that we would give and what was the dose?

2. Asthma (paeds), I remember that they asked about PEFR and treatment of acute
exacerbation of asthma in children, and definitions of mild, moderate, severe and
lifethreatening types of paed astham

3. Bells Palsy-- I have no memory of this question!!

4. Ectopic-- 19 year old woman with lower abdominal pain and no PV bleeding, I think,
Then Risk factors for ectopics in any age group, etc

5. Glaucoma-- acutely painful eye, with vomiting, acute angle glaucoma, treatment in the
ED

6. Diarrhoea (baby)-- basically signs of dehydration in a 1 year old, I think and


management

7. STEMI-- showed us an ECG, and asked diagnosis and management

8. Supracondylar fracture-- showed us an X ray, asked diagnosis, management, possible


future complications

9. Angiodema-- Hereditary angioedma, C1 esterase inhibitor deficiency and ACE


inhibitor causing oedema, plus management

10. Head injury-- Basically NICE guidelines of a head injury

11. Crohns-- man whose crohns has been dormant for 6 years, not on any meds, presents
with a flare-- management

12. Psoriasis-- this was the worst one, as it was an awful picture, that was impossible to
tell what the rash actually was

13. Mumps-- I have no memory of this question!!

14. House fire-- I have no memory of this question!!

15. Aspirin Overdose-- I have no memory of this question!!

16. Retention-- I have no memory of this question!!

OSCES

1. Major Trauma-- I had a man who was stabbed in the left side of his chest, in the ant ax
line, with a huge haemothorax and white out on X ray, and then the management of that---
although they didn't actually ask you to put the chest drain in, but they did ask how to
secure it and what instructions you would give the nurses
PRAY FOR ME Dr. Ashraf Elshehry Page 227
2. Paediatric Trauma-- I actually had Paediatric Airway management-- child with grape
aspiration, then i had to look for the foreign body and remove it with a magills, then
further management, and handover to the ITU team

3. Needlestick-- a nurse from a ward sustained a needlestick from a patient upstairs,


discuss PEP, with it's side effects, and risk benefit ration

4. Airway-- this was actually to teach a student about different types of airway adjuncts,
so they had a table full of things like ET tubes, guedels, bag and mask, eve na
astethoscope, and you just had to stand there, the premise was that apparently a trauma
call is coming in with a GCS of 7, teach the student which airway adjuncts may be useful
in this case, the student gave not fedback, didn't really talk, so it was more like a lecture
really--- weird station

5. PID history-- communication, bacsically sexual history from a woman with lower
abdo pain and discharge- she did ask whether this would affect her fertility or not

6. Diabetes-- again a weird communication station, woman with 3+ glycosuria and a


diabetes sounding history, you had to talk through her diagnosis, v v weird

7. Drop foot-- man who had been in a below plaster and ended up with foot drop, but you
had to elucidate that he had a plaster on, he also wanted to know is the people who put the
plaster on were at fault, and whether he could sue them

8. Suturing- woman with a fake cut on her hand, put a couple of sutures in and give her
advice

9. Haematuria-- man with haematuria and a family history of bladder cancer, secondary
to dyes, take a history and you had to drag it out of him that he was worried about cancer
(PAtrick roberts told me that they had had this one before, but with PR bleeding)

10. ECG-- another weird one, you had a student and an ECG, the history of a woman with
a fast heart beat that had now reverted to sinus rhythm, and you had to teach the student
how to read an ecg, but you weren't allowed to ask the student any questions and she
didn' really talk.

11. Respiratory Exam-- man with a weird scar just at the top of his sternum (not a trach)
and L basal absent breath sounds-- I think he had a L lobectomy, anyway, Resp exam on
him

12. -----------

13. Triage station-- you were given the 5 triage cards in the rest station before, so that you
had time to read them, then you had to go and talk to anurse and explain why you wanted
to change the order that these cards had been triaged in, she was not the nurse who did the
triage

the cards were-- a 7 day old baby, mum says drowsy, vomiting an less responsive, no obs
done cos baby was "sleeping" triaged as a blue, an MP who presented with painful
bunions but was worried that he was getting hassled by the press, cos he was involved in a
PRAY FOR ME Dr. Ashraf Elshehry Page 228
scandal, who was to be seen within 10 minutes, a woman who had run out of her OCP,
seen within an hour, an two others that I can't remeber; you had three doctors and another
one coming on in an hour

14. ENT-- man with nasal fracture, that you had to explain how to use those weird forcep
type things to look up his nose

15. NAI-- actually communication again-- you had to try and get the Paeds A+E sister to
call the paeds reg down for you to see a child suspected of NAI-- had a weird fracture and
had a head injury some time ago, and she was resistant, plus she said she knew the family
and they would neve do anything like that.

16. Hand Exam-- woman with painful swollen knuckles, assess the hands and give
diagnosis

17. Femoral nerve block-- didn't have to do, just talk through on a dummy and show
position-- scenario was a man with fracture shaft of femur, they had a man lying there
with his leg out and you had to explain to him what we were going to do

18. Eye exam-- man working with metal yesterday, painful eye FB sensation, how would
you examine, mention penetrating FB

19. Mental health- i had a man who took an OD-- assess

20. THAT'S IT!!!!!!!!!!!!!!!!!!!!!!!!!!!!!!!!!!!!!!!!!!!!!!!!!!!!!!!!!!

hope this is helpful

NIV

A 67 yr old woman is brought by ambulance to your department with acute shortness of


breath. She is known to have COPD, and she has been gradually worsening over the past
3 days. She is on 100% oxygen. Her O2 saturations are 99%. One of the junior doctors
has taken her ABG’s which are as follows

pH 7.13
pO2 15.7
pCO2 10.3
BE -2.0

What would be your first action? 1 mark

Reduce Oxygen from 100%

What parameters would you tell the nurses for her monitoring? 1 marks

Maintain sats low 90’s %

PRAY FOR ME Dr. Ashraf Elshehry Page 229


What would be your initial treatment? 3 marks
Steroids
Nebulised B-agonists
Nebulised ipratropium- not mentioned in NIVCE but fair dincum I think
Theophyline should only be administered if failure of the above- NICE
Antibiotics only if H/O purulent sputum- NICE

An hour later her ABG are as follows

pH 7.10
pO2 8.5
pCO2 10.9
BE -2.5

What are the indications for NIV (BiPAP)? 2 marks

Acute exacerbation of COPD with resp acidosis pH <7.35 (this gets you 2marks)
also
Hypercapnic resp failure secondary to neuromuscular or chest wall deformities
Decompensated obstructive sleep apnoea

What are the contraindications for NIV? 3 marks

Recent facial/upper airway surgery


Pt vomiting
Fixed upper airway obstruction
Copious resp secretions, inability to protect airway, life threatening hypoxaemia, severe
co-morbidity, confusion/agitation, bowel obstruction.
Pneumothorax without drain.

Thorax 2002;57:192-211 NIV in acute respiratory failure

OSCEs

FFAEM Exam
8th November 2000

(Total 12 questions, although some multiple parts. 60 minutes)


1. An ECG, (12 lead).
Broad complex rhythm for half the ECG, rate about 90/min. Then sinus
rhythm. Not clear whether or not p waves present.
a. What is the rhythm. (I wasn’t sure:? CHB or nodal rhythm with low
escape. I think I opted for the latter. Other answers suggested
include idioventricular rhythm)
Second ECG was taken after the first above.
b. Describe the abnormalities.(I thought it was an inferior MI with
lateral extension and reciprocal anterior changes. There was
appropriate ST elevation, depression and Q-waves.)

PRAY FOR ME Dr. Ashraf Elshehry Page 230


2. A CXR from a paediatric victim of trauma (aged about 9).
Showed multiple L # ribs, I think from 1-8 visible posteriorly. Also # scapula
at neck, # clavicle. Patient was intubated and appeared to have a R jugular
line in situ. Pleural cap, loss of L costophrenic angle implying haemothorax).
a. What abnormalities do you see?
b. What further imaging would you choose and why?

3. Pelvic Xray in an elderly patient. Showed # of R acetabular floor. Also


showed calcified aortic bifurcation and common iliac vessels.
a. Describe 2 abnormalities. (Other possible abnormalities were
phleboliths and possibly something (? ? spina bifida occulta) at
about L5)
4. Lateral skull Xray of a patient with a gunshot wound to head.I saw a hole in
the L temporal region, with a white opacity beside it. I now suspect it was a
bullet hole and depressed skull fracture, with intracranial air and the
opacity was the flattened bullet or more likely, airgun pellet.
a. What abnormality(s) do you see?

5. 2 facial Xrays. Severe injuries: # R zygomatic complex (tripod #), # medial


orbital wall, inferior orbital margin, blood in the R maxillary sinus.
a. What abnormality(s) do you see?

6. Lateral skull Xray of patient with a head injury. (I think the patient had a
nosebleed). Fluid level (blood) in sphenoid sinus.
a. What abnormality(s) do you see?

7. CXR in trauma patient. I think it showed a tension haemopneumothorax.


a. What imaging do you next need to perform. (I wasn’t sure
whether this meant CXR after emergency thoracocentesis or chest
drain; or contrast CT chest, so I wrote both, in order).

8. Photograph of an (elderly?) torso with bruise L upper chest/clavicle, L


elbow and blue, swollen L forearm. There was said to be no bony injury, but
absent L radial pulse.
a. What urgent investigation(s) would you carry out?(eg coag, FBC.
Might have included angiography, with or without CT; or
measurement of compartment pressures).
b. What abnormality(s) can you see and what 2 causes can you
suggest.(? compartment syndrome in L forearm or
occlusion/dissection L brachial artery; L axillary artery
avulsion/dissection/occlusion)
c. What operative management may need to be performed?
(Emergency fasciotomy L forearm; emergency CT angiography of
the thorax with a view to repair).
d. Can you think of any pre-existing factor that may have influenced
the development of this complication? (Warfarinisation, previous
operation, such as harvest of radial artery for bypass graft;
cervical rib)

PRAY FOR ME Dr. Ashraf Elshehry Page 231


9. Trauma Xray of a foot and ankle with ‘lateral tenderness’.
a. What abnormality do you see? (Subluxation of the cuboid. Some
also saw a fracture line).

10. Biochemistry results of young patient with hypotension, confusion, vomiting.


Showed, amongst other things, markedly raised glucose, raised K, low Na,
low HCO3, low Cl. Normal Ca, slightly raised creat, urea.
a. Calculate the anion gap.(Raised).
b. How do you account for the result. (Diabetic ketoacidosis).
c. What other biochemistry test would you request? (ABG)
d. How would you initially treat the patient.(How much did they
want? Oxygen; treat for shock, then rehydration with 0.9% saline;
intravenous actrapid via sliding scale, checking BM every 10-15
minutes and regular electrolytes; accurate fluid balance; high
dependency nursing; search for focus of infection etc.)

11. Elbow Xray of 10 year old boy after trauma, (suboptimal views presumably
due to the nature of the injury).
a. What abnormality(s) do you see? (effusion, supraconduylar
fracture, ?dislocated capitellum. Some also wondered if medial
epicondyle was displaced).
b. What complications may occur?.(Neurovascular, particularly
median and ulna nerve lesions; vascular compromise – brachial
artery at elbow – check distal pulses/capillary return to reduce
risk of Volkmann’s ischaemic contracture;
osteoarthritis/stiffness/loss of function).
c. What ossification centres can you see? (Visible was capitellum,
radial head and ? medial epcondyle. The others weren’t. NB
ossification centres appear later in boys).

12. ECG rhythm strip. Showed broad complex tachycardia.


a. What makes you think it is VT? (Broad complex; tachycardia;
(likely to be lead II in which case) left axis deviation; fusion beat;
capture beat).
b. You elect to cardiovert under appropriate sedation. What energy
levels would you use for the first 3 shocks? (Different answers
between candidates – 100J, 200J, 360J is what the ALS manual
says).
c. This fails. What would you try next? (Answer not clear. ? optomise
oxygenation, electrolytes including K, Mg, Ca, pH. Consider drugs
eg lignocaine, amiodarone, magnesium iv).
d. This fails. What else would you try. (Answer unclear ? change
pads/paddles, paddles position, disopyramide. Try cardioverting
after loading with amiodarone. The best answer was probably
overdrive pacing)..

13. Patient transferred to your unit after trauma. CXR showed diffuse
pulmonary shadowing.
a. What is the most likely diagnosis? (? pulmonary oedema. Could be
secondary to fluid overload. Differential could have been diffuse

PRAY FOR ME Dr. Ashraf Elshehry Page 232


pulmonary contusion. Less likely to have been ARDS, since the
scenario they gave was of an acute injury).

14. 2 photos of a pair of hands, in patient with a L breast lump.


a. What abnormality of the hands do you see? (Strangely, some of
the changes seemed to be bilateral. Guttering, wasting of
hypothenar eminence, finger clubbing, ? slight yellowing of the
fingers (lymphatic involvement or smoking – depending on whether
you thought the nails were yellow, and how many of them.
Changes were more marked in the left hand)

Next picture was the patient’s CXR. Showed L apical collapse/consolidation,


but no obvious rib involvement.
b. What abnormality(s) do you see?
c. What is the explanation for the clinical findings? (Brachial plexus
involved; possibly lymphatics (depending on your answer to
above). In retrospect, I think the lung mass must have been the
primary, and the breast lump unrelated to the hand appearances,
since carcinoma of the breast does not cause finger clubbing).

15. Picture of the hand of a milkman who had fallen over onto broken glass, and
had been asked to make a fist. Laceration at base of thenar eminence. Little
and ring fingers flexed at DIP and PIPJs. Index: DIP and PIP remained
straight (? Flexor digitorum profundus and superficialis divided); middle: DIP
flexed, but PIP straight, (profundus intact but superficialis divided).? Thumb
also not flexed as much as would be expected.? Damage to thumb flexors as
well
a. What structures have been divided?
b. What other structures would you check? (Nerves, vascular,
thumb).

16. In a major incident:


a. What essential information is required to convey to the receiving
hospital? (Location, hazards, approx numbers dead, injured; which
emergency services required).
b. Who is responsible for removing the dead bodies at the scene? (If
definitely dead, I believe this is the responsibility of the police
acting on behalf of the coroner).
c. Under which 2 circumstances would you remove the dead
urgently? (If body presented a hazard to the rescuers – eg lying on
a power line, or a live underground rail; or to allow rescue of a
live casualty).

These questions are based on our memory of the exam, and the answers that
we (or others) gave. They may not be correct, but I hope they give you some
idea of the sort of questions that may be asked!

PRAY FOR ME Dr. Ashraf Elshehry Page 233


Robert Florance. SpR A&E, Addenbrookes, Cambridge.

Vicki Holloway. SpR A&E. Ealing Hospital, Middlesex.

OSCE November 2003


Question 1.
24 yr old gentleman. Binge drinking at weekend. Vomiting since.
Presented with fever. Had grand mal seizure for 2 mins in ambulance.
Na 124
K3
Ur & Cr essentially ok
Chloride 80 ish.

ABG’s
pH 7.2 ish
pCO2 3.4
pO2 26
HCO 10
BE -6
HCT 55

CXR
a. List 2 radiological abnormalities:
Pneumomediastinum
Subcut emphysema
Right middle lobe consol?

b. Causes of radiological abnormalities:


Oesophageal rupture (Boerhaave’s syndrome) – air has tracked into mediastinum and
patient has probably aspirated.

c. State the pathophysiological processes leading to the blood results.


Vomiting (loss of HCl)
Alcoholic Ketoacidosis

PRAY FOR ME Dr. Ashraf Elshehry Page 234


Sepsis

d. Why did he fit?


Possible secondary to hyponatraemia
Possible hypoglycaemia
Possibly alcohol withdrawal

Question 2
7 year old with several week history of pain in both knees. Now stopped playing
football. Seen GP on multiple occasions – no diagnosis found. Also history of
intermittent fever over last week. Early in the week he had a rash on his chest. He has
also been complaining of increasing tummy pains.
a. 6 things important to ask in history
Recent URTI
Haematuria
Abdominal pain/ diarrhoea/PR bleed
Eye pain
Other joint involvement
Recent travel

b. 4 differential diagnoses
HSP
Still’s disease
Acute Viral Illness
Inflammatory bowel disease

c. 2 tests in A&E
Urinalysis
U&E

PRAY FOR ME Dr. Ashraf Elshehry Page 235


Question 3
Trauma question. Young person. ? fall or RTA.
Blood from both ears. Decreased GCS
a. Comment on C spine XR
Inadequate
C7 spinous process fracture (Clay-shoveller’s)

b. List Chest X-ray abnormalities


Left upper rib fractures
Large contusion
Haemothorax

c. Comment on pelvis XR
?SI joint disruption
Rotation of left hemipelvis

d. Assuming primary and secondary surveys are fully completed and catheter in, 2
further management priorities:
Assuming airway protected:-
Pelvic stabilization (tie in sling) – before formal ortho. stabilisation
Chest drain on left

Question 4
Picture of fractured penis with history of sudden pain during intercourse.
a. Diagnosis
Fractured penis

b. 3 symptoms or signs of this


Discolouration
Swollen and deformed
Tender

c. 2 management options

Conservative therapy - cold compresses, penile splinting

PRAY FOR ME Dr. Ashraf Elshehry Page 236


Operative repair – haematoma evacuation and repair of tunica albuginea

d. 3 complications

Urethral stricture

Permanent penile curvature

Arterio-venous fistula +/- impotence

Question 5
Foot XR. History of crush between vehicle and post.
a. List the radiological abnormalities
Fracture dislocation tarso-metatarsal junction
Fractured base of 2nd metatarsal

b. Name of injury
Lisfrancs

c. Usual Associated radiological features


Soft tissue swelling
Fracture base of 2nd metatarsal

d. 2 complications
Acute neurovascular injury e.g. Acute ischaemic (non-viable) foot
Later - Compartment syndrome

Question 6
Unwell infant. Fitting pyrexial. Purpuric rash. Fitting stopped with lorazepam. Aiway,
breathing & oxygen and iv access sorted. Fluid bolus started. Very tachycardic and
hypotensive. Decreased cap refill.

PRAY FOR ME Dr. Ashraf Elshehry Page 237


a. Give the (one) next further immediate management:
Intravenous antibiotics (Cefotaxime 80mg/kg)

Treated with oxygen and fluid bolus. Still tachy and hypo with raised refill time
Blood results : Low glucose. High CO2 on gases.
b. Give 2 next management steps.
Protect airway and assist ventilation (and call PICU anaesthetist)
5mls/kg 10% glucose iv

Now stabilized. Paeds Reg is very busy and asks you to do LP in A&E.
c. What is your answer and why.
No. Contraindicated in patient with decreased GCS and in presence of sepsis and
purpuric rash..

Question 7
8 yr old girl with asthma brought in by teacher from school. Increasingly SOB.
Unable to talk in sentences. Pulse =140, RR >50/min. Sats 93% on high flow oxygen.
Slt end exp. wheeze on examination. Uses only inhalers.
a. List your first 2 treatments with doses.
Salbutamol 5mg neb
Hydrocortisone 4mg/kg iv or Prednisilone 2mg/kg po

After ?10 mins there is no improvement:


b. 4 further management steps:
Repeat nebuliser with ipratropium
Salbutamol/Aminophylline/ Magnesium according to drug history and local policy

PRAY FOR ME Dr. Ashraf Elshehry Page 238


CXR
Contact paed. / paed. anaesthetist

c. List 4 behavioural/ social factors that make severe asthma more likely.
Poor compliance with steroids/inhalers
Allergens in house eg dust mite/pets
Patient/parent smoke
Low socio-economic class

Question 8
20 ish gentleman. No foreign travel. No meds. No PMH. Suddenly unwell. Confused.
Liver flap. Spider naevi. Large palpable spleen
Abdo Ultrasound:
Small liver, big spleen
Bloods:
Low Hb
Low WCC
Low Platelets Retics 6%
Low Na
Normal Renal function.
Slightly raised ALT.
Alk Phos and Gamma GT normal
Massively raised Conj. and Unconj. Bilirubin
Albumin low
Ammonia rasied+++

a. State the pathophysiological processes seen above.


Portal Hypertension secondary to Cirrhosis
Hepatic encephalopathy as liver unable to clear ammonia produced from protein load
e.g. diet or GI bleed
Haemolytic anaemia/ Bone marrow suppression

b. Diagnosis

PRAY FOR ME Dr. Ashraf Elshehry Page 239


Wilson’s Disease (Hepatic encephalopathy secondary to cirrhosis)

c. 1 Clinical finding that could look for to confirm diagnosis.


Kayser-Fleischer rings

Question 9
Male brought in from house fire by paramedics and have filled in the burns chart for
you. ABC done.IV access in. Analgesia given. Soot in mouth and voice change.
a. Using the chart calculate the % burn

b. Stating which formula you are using calculate the fluid requirement for the first 4
hours.
Parklands 4mls/kg/% burn in 24 hours
Half in first 8 hours (half of this in first 4 hours)

Patient going to be transferred to burns unit:


c. 6 tests you would request
CXR
ABG
Carboxyhaemoglobin
FBC (haematocrit)
U&E (baseline)
G&S

d. 3 managements prior to transfer


Burn dressings (clingfilm)
Secure airway (RSI intubation by experienced anaesthetist)
Check Tetanus status

Question 10
PRAY FOR ME Dr. Ashraf Elshehry Page 240
Picture of trench foot/ chronic severe athletes foot.
a. Give 4 differential diagnoses
Trench Foot, Severe athletes foot, ?PVD
b. 2 tests
Doppler foot/ Ankle/brachial pressure index
c. 2 treatment steps.
Analgesia
Question 11
RTA. Young male. Diabetic alert bracelet. GCS decreased. Airway fine. Decreased air
entry on left and dull to percussion. Tachy and hypotensive .Given 100% Oxygen Iv
access and fluids running
Chest drain inserted 300 mls blood only. Chest X-ray to review.
a. 4 radiological abnormalities excluding cardiac monitoring wires.

b. 6 reasons for his depressed conscious level.


Hypotension (decreased cerebral perfusion)
Hypo/ hyperglycaemia
Hypoxia
Cerebral bleed
Cerebral contusion
Alcohol/drugs

Initially stablised but suddenly despite crystalloid resuscitation and blood:


Pulsed 140/min and BP 80 systolic.
c. What is likely cause of his destablisation?
Continued occult bleeding ilikely abdomen (spleen)

SpR surgery busy, asks you to send him for a CT of head and abdomen.
d. What is your answer and what do you want him to do instead?
Too unstable for scanner –dangerous
Laparotomy

Question 12
ECG – (Left Bundle Brach Block)
PRAY FOR ME Dr. Ashraf Elshehry Page 241
46 year old man with 2 hours crushing central chest pain, sweaty, vomiting. No
previous history. Otherwise well. On oxygen, ongoing pain
a. What does ECG show?
LBBB

b. Why is it that?
Acute anterior MI (involving ischaemia of left sided conduction pathways)

c. 4 management steps:
Morphine with anti-emetic (metoclopramide)
Buccal nitrate 3mg
Aspirin 300mg po
Thrombolyis or angioplasty (PCI)

Same ECG but patient 76 yr old, MI 3 months ago. Half hour of pain which now
resolved. Normal obs.
d. 4 Management steps.
Aspirin 300mg po
Enoxaparin 1mg/kg S/C injection
Request old ECG
Admit CCU +/- angioplasty

OSCE November 2002 exam


Question 1
Photo of fingertip avulsion injury with bone exposed.
a. How would you manage this injury?
Analgesia and digital nerve block
Clean, non-adherent dressing and elevate
Check tetanus status/ give anti-tetanus immunoglobulin
Antibiotics
Radiograph
Refer to hand surgeons for terminalisation or graft/flap

PRAY FOR ME Dr. Ashraf Elshehry Page 242


Question 2
Woman presents with lower abdominal pain. Has been seen by the GP who has
diagnosed PID and started the patient on doxycycline and metronidazole.
a) How would you confirm the diagnosis?
Examination:-
Lower abdominal tenderness
Cervical tenderness
Adnexal tenderness

b) What other tests would you do?


Pregnancy test
FBC
ESR/CRP/PV depending on local preference
Urinalysis
Blood cultures if pyrexial/peritonitic

Question 3
23 year old man found collapsed at 6am with a GCS of 3/15. Blood gas results:
pH 7.31
pCO2 and pO2 were normal
Base excess -6
K+ 7.1
a) What do these bloods indicate?
Mild metabolic acidosis with no respiratory compensation
Hyperkalaemia

b) What does the ECG show?


ST segments down in V2-4 (?posterior MI ?SAH)
Tall/tented T waves

c) Blood test results come back and show the following:


Ur 11

PRAY FOR ME Dr. Ashraf Elshehry Page 243


Cr 220
K+ 7.9
CK 85,000
Amylase 2000

d. Give 3 possible causes


Ecstasy/amphetamine/cocaine/PCP OD
Status Epilepticus
Rhabdomyolysis from prolonged immobility secondary to alcohol
Carbon Monoxide poisoning

Question 4
Picture of a 6 year old girl with red swollen eye.
a. Diagnosis?
Orbital cellulites
Pre-septal cellulites
Erysipelas

3 complications?
Cavernous sinus thrombosis
Meningitis
Cerebral abscess

2 possible causative organisms?


Strep pneumoniae
Haemophilus influenzae

Possible source of infection in this age?


Spread from paranasal sinus(ethmoid)
Spread from conjunctivitis

Question 5

PRAY FOR ME Dr. Ashraf Elshehry Page 244


Picture of an elderly lady with shingles over her forehead on the left side
a. Diagnosis?
Ophthalmic Zoster (trigeminal nerve)
b. What other areas would you examine?
Eye, side of nose and mandibular/maxillary branches of trigeminal nerve
c. Complications?
Ulcerative keratitis
Corneal scarring
Trigeminal neuralgia
Episleritis/Scleritis
d. How would you treat this?
Analgesia, Oral acyclovir +/- acyclovir eye drops, refer to ophthalmology follow up
Question 6
X-ray of a fractured pelvis
What injuries would you expect with this fracture?

Question 7
Fitting child with a temperature of 39.8 oC
a. What is your first line treatment?
ABC with high concentration oxygen followed by intravenous lorazepam
b. What can you give for subsequent treatment?
Further lorazepam, paraldehyde, phenytoin, then thiopentone if still fitting
c. What other treatment would you give?
PR paracetamol
Intravenous antibiotics
Intravenous acyclovir
Intravenous dexamethasone

Question 8
X-ray trauma series of a patient in an RTA
a. Report on the C-spine XR
Inadequate film
?C1 fracture / ?normal

PRAY FOR ME Dr. Ashraf Elshehry Page 245


Question 9
Chest Xray of an injured patient
a. Name three abnormalities on the CXR
Haemothorax
Flail segment

Question 10
Young man with chest pain. ECG showed ?pericarditis
a. Describe the ECG

Question 11
CXR of a 52 year old woman with a wedge shaped shadow in the midzone of the
right lung
a. Differential diagnosis
PE
Pneumonia
Bronchial Ca/collapse
b. Also given an ECG, what rhythm did the ECG show
AF
c. Explain why you think the ECG shows this
No p-waves and irreg irreg

Question 12
Interactive scenario: Given a written scenario and asked to explain to a mother (an
actress) about the pulled elbow which you think her son has sustained and which
has gone back spontaneously

OSCE November 2004


Question 1

PRAY FOR ME Dr. Ashraf Elshehry Page 246


A man presents with a red eye and no history of trauma. Shown a picture of
herpetic/ dendritic ulcer
List 4 facts in the history which may indicate a cause for the red eye
Cold sores/whitlow recently
Excessive sunlight
Trauma
Systemic illness/URTI/ / stress

What is the cause of this appearance.


Herpes Simplex 1 Infection

What is the treatment for this condition.


Referral to ophthalmology
Acyclovir drops 3% 2drops 5 times per day

Name 2 other causes of painful non traumatic red eye


Acute Glaucoma
Iritis

Question 2
A man presents with jaundice, hepatomegaly and a distended liver, he is passing
dark urine and admits to drinking a lot recently.
Results show a list of LFTs (raised ALT and normal Alk phosphatase)

What is the diagnosis


Acute hepatitis

List 4 causes of this picture


Alcoholic hepatitis
Viral hepatitis (Hepatitis A-E, EBV, CMV)
Malaria
Drugs (Fluconazole, paracetamol excess)

PRAY FOR ME Dr. Ashraf Elshehry Page 247


What 2 blood tests would confirm the diagnosis
Monospot
Viral titres

Question 3

76 year old man presents with back pain and no history of trauma, these are his
blood results
Hypercalaemia
Hyperkalaemia
renal failure
normal alk phosphatase

What is the diagnosis?

Myeloma

What 5 interventions would you consider and why?

♦ Intravenous N.Saline (dehydration)


♦ Furosemide/ N.Saline/ Pamidronate (Hypercalcaemia
♦ Insulin/glucose infusion, Salbutamol neb. (Hyperkalaemia)
♦ Codeine, paracetamol (pain)
♦ Blood transfusion (after renal failure treated)

What two further interventions would you consider?

♦ Plasmapheresis or dialysis (Acute renal failure)


♦ Corticosteroids (Dexamethasone) for spinal cord oedema

What 2 things would you tell the SHO to do next?


Refer to haematology registrar
Inform renal registrar

PRAY FOR ME Dr. Ashraf Elshehry Page 248


Question 4

Child brought in by father - obtunded, cyanosed after drinking some liquid


pH 7.25
pCO2 3.5
pO2 13.0
What did the child drink?
Amyl Nitrate

What is the diagnosis?


Methaemaglobinaemia

What 4 further investigations would you do?


Methaemoglobin levels (Co-oximeter)
Blood glucose
ECG
CXR

What four treatment options would you consider?


Protect airway and administer high concentration oxygen via facemask
Methylene blue 1mg/kg
Phone PICU for paediatric anaesthetist (requires intubation + gastric lavage)
Packed red cell transfusion exchange

Question 5
Long history of a 76 year old man with a collapse, in AF, and a list of neurological
findings, also with a list of abnormal blood results, raised blood sugar, pyrexial. CT
shows infarct
Which blood vessel has been occluded?
SEE PREVIOUS

List four interventions which have been recognised to improve outcome in this
patients?

PRAY FOR ME Dr. Ashraf Elshehry Page 249


List four criteria from national guidelines for thrombolysis to be administered to this
patient?
NINDS criteria met (more than slight deficit and not improving)
Haemorrhage excluded on CT scan
Less than 3 hours since onset
National registered centre for thrombolysis in stroke (SITS-MOST)

Question 6
Pregnant lady presents with a fit.
What is needed to make the diagnosis of eclampsia?
Seizure in a female over 20 weeks pregnant
BP >140mmHg systolic or >90 mmHg diastolic
Oedema
Proteinuria

List 4 treatments which will stop the fitting?


Magnesium Sulphate 4 g intravenously
Lorazepam
Phenytoin
Delivery of baby

Which treatment has been shown to reduce fitting long term?


Labetolol

List 4 haematological abnormalities associated with eclampsia?


Haemolysis (Low haemoglobin, raised reticulocytes)
Low Platelets
Elevated Liver enzymes
DIC
PRAY FOR ME Dr. Ashraf Elshehry Page 250
Question 7
A man presents to A&E with chest pain, BP-90/58
ECG showed inferior AMI with RV involvement.
What would you tell the SHO to do next?
Remove buccal nitrate/ lie flat and elevate legs

What else would you do for this man?


Intravenous fluid challenge (250 mls N.Saline)

List 5 treatment options


Give high flow oxygen
Carefully titrated morphine with anti-emetic
Aspirin
PCI Angioplasty (preferred) or Thrombolysis
Intra-aortic balloon pump

What is the diagnosis and which vessel is involved?


RV infarct
Right coronary artery

Question 8
Picture of a wound in a child, on extensor surface of arm, sustained with a piece of
glass in a green house.
What 4 complications would you consider?
Neurovascular injury
Muscle/tendon injury
Infection
Retained FB

What 4 interventions would you do to reduce the anxiety/ pain in this patient (Drug
doses) and to enable exploration and suturing of wound?
Experienced play therapist/ paediatric nurses
Analgesia (paracetamol 15mg/kg)

PRAY FOR ME Dr. Ashraf Elshehry Page 251


Procedural sedation (Ketamine 2mg/kg with atropine 0.01mg/kg intramuscularly)
Local anaesthetic (up to 0.3mls/kg of 1% lignocaine)

His mum phones as he is getting agitated, she demands the nanny brings the child
home. What 3 things would you do next?
Encourage mum to attend the Emergency Department with explanation of need for
treatment and possible complications
Check old cards and CPR
Contact father if he has consent rights and mum refuses (after informing mum)

Question 9
Xray of a supracondylar fracture in a child
What does the Xray show?

What 4 complications can occur?


Median nerve injury
Ulnar nerve injury
Brachial artery injury/ ischaemic limb
Compartment Syndrome

List 4 management options


Broad arm sling
Above elbow backslab
Closed reduction under GA
Open reduction and internal fixation under GA
(Nasal diamorphine in all cases)

List the ossification centres and when they appear.


C 3-6 months
R 4-5
I 4-7
T 9-10
0 9-11

PRAY FOR ME Dr. Ashraf Elshehry Page 252


L 9- 13

Earlier in girls and Chinese.

List two recognised complications with this condition


Myositis ossificans
Late Volkman’s ischaemic contracture

Question 10
A man has collapsed in Xray, this is his xray.
Tension pneumothorax with a chest drain but poor quality film
List 4 expected findings on chest examination

What would you do next?


High flow oxygen and needle thoracocentesis

List 4 abnormalities on the Xray.

What 2 other options would you consider?


Reposition drain (malposition) or replace drain (blocked0
Contact cardiothoracics re: ?large airway injury/bronchopleural fistula

Question 11
History of pyrexial fitting child with picture below
Picture of Koplicks spots
Fill in the boxes/doses and the times you would administer the drugs
What does the picture show and what is the condition?
List 6 other reportable infectious diseases.
MMR
DTP
PRAY FOR ME Dr. Ashraf Elshehry Page 253
Question 12
Elderly confused diabetic pyrexial male
Blood results
Hyperglycaemia
Dehydrated.
ABGs – acidotic
CXR (poor quality picture)
Interpret the CXR

What are the likely diagnoses


DKA + LLL pneumonia

List your management and investigations


Protect airway and administer high concentration oxygen
U&E/ Blood cultures
ECG
Intravenous normal saline (1l over 1 hr)
Sliding scale insulin (initially 6IU/hr)
Co-amoxiclav 1.2g iv and Clarythromicin 500mg iv
Ng tube
Urinary catheter (sample for C&S)
Enoxaparin/TED’s
PR paracetamol/ Refer to medics
If Hyperosmolar Hyperglycaemic Non-ketotic Syndrome (HHNS):-
Enoxaparin
½ N.Saline if sodium >150mmol/l
Initially start insulin at 3IU/hr.

AASAQ paed rash

The following 5 yr old child presents to the ED. His mother is worried he has measles.
The triage nurse thinks it might be Kawasakis disease and the GP thought it was Scarlet
Fever.

PRAY FOR ME Dr. Ashraf Elshehry Page 254


a)What features in the history and examination would support a diagnosis of measles? (2
marks)

Cough
Choryza
Conjunctivitis
Miserable child
Rash starting at neck and progressing
Koplik’s spots

b)What features in the history and examination would support a diagnosis of Scarlet
Fever? (2 marks)

2-4/7 sore throat


Fever, rigors, lymphadenopathy
Sandpaper rash, esp in flexures
Whire coated -> strawberry tongue

PRAY FOR ME Dr. Ashraf Elshehry Page 255


c)What features in the history and examination would support a diagnosis of Kawasakis
disease. (2 marks)

Fever > 5/7


Irritability disproportionate to fever
Peeling extremities(palms and soles)
Non-purulent conjunctivitis
Cervical lymphandenopathy
Strawberry tongue and cracked lips
Polymorphous rash

d)What is the diagnosis? (2 marks)

Scarlet Fever – the GP was right!

e)Give two ways to confirm this. (1 mark)

ASOT or Throat swabs

f) How would you treat him? (1 mark)

Penicillin

Pelvic fracture

PRAY FOR ME Dr. Ashraf Elshehry Page 256


A 27 year old motorcyclist has lost control of his machine and hit a tree. Lateral c-spine
and CXR are normal, but his pelvic view is as shown.

Describe this X-Ray (3 marks)

Fracture of L acetabulum
Fracture of R pubic rami ant and post

What injuries would be possible with this injury? (4 marks)


Bleeding of pelvic vessels
Urethral tear
Nerve injury
Rectal injury

Outline your management of this patient if they were in hypovolaemic shock (3 marks)
Fluid resuscitation
Stabilise pelvis with straps
Ortho for exfix
? embolisation

Asked Nov 2002

pleuritic

A tall, thin 27 yr old male presents with sudden onset of right pleuritic chest pain. He flew
back from Australia last week. A CXR has been performed. He has no PMH and is on no
medication.

1. What is the differential diagnosis? 1 mark

Pneumothorax, PE, infection, musculoskeletal 1 mark for >2

PRAY FOR ME Dr. Ashraf Elshehry Page 257


PART A: Assuming his CXR shows a pneumothorax

2. Under what circumstances, under BTS guidelines, would you

a)manage him conservatively

not SOB and rim of air < 2cm

b)Aspirate his pneumothorax

SOB or Rim of air > 2cm

c)Insert a chest drain 3 marks

2x aspiration unsuccessful

3. If you had attempted aspiration but a repeat film showed a persisting pneumothorax,
when would you re aspirate and when would you insert a chest tube?
2 marks

PART B: Assuming his CXR was normal,

4. What non-radiological investigations are available? 1 mark

D-Dimer, FBC

5. Name 3 radiological investigations available 2 marks

Q Scan, Spiral CT, Pulmonary angiography

A 32 year old builder attends the ED with a wound to his left thigh which he sustained on
some barbed wire yesterday. He is diabetic, on insulin and has no allergies He is fully
covered for tetanus vaccinations and noticed this morning that his wound, a 2cm
superficial wound had become painful and red. The triage nurse marks the 1cm x 2cm
area of redness and directs him back to the waiting room. You call him into a cubicle 2
hours later to find him looking unwell. He is pyrexial and looks toxic. The erythema has
spread and is now blistered although he says he can’t feel it as much now. He has a pulse
of 110/min and BP of 94/64.
You examine the leg and find it as you see it below.

PRAY FOR ME Dr. Ashraf Elshehry Page 258


What is the likely diagnosis? (1)
Necrotizing Fasciitis

Give 4 features in history and examination would help with the diagnosis? (2)
Rapidly spreading erythema
Dusky purplish discolouration at wound
Anaesthesia to affected area
Putrid discharge / bullae / tissue necrosis
Gas forming infection
Clinical signs of shock/ fever/toxic

What are the usual causative organisms? (2)


Group A haemolytic strep / Staph aureus / Bacteroides / Clostridium perfringens /
Peptostreptococcus / Coliforms / Proteus / Klebsiella
1 for first 4, ½ mark each for others

Outline your management of this patient.(3)


High flow Oxygen 15l via rebreather mask
Iv access and 1litre normal saline Stat
FBC/U&E/Blood Cultures/tissue cultures/ABG/urinalysis
Xray affected area for gas gangrene
Iv antibiotics: Benzylpenicillin 1.2g
Clindamycin 600-900mg
Metronidazole 500mg
3rd generation cephalosporin ceftriaxone 1-2g
Gentamicin if penicillin allergic
Consider hyperbaric
Refer surgeons immediately
½ mark each

Outine 2 complications. (1)


Renal failure
Circulatory collapse
Repeated and radical amputation leading to disfigurement

PRAY FOR ME Dr. Ashraf Elshehry Page 259


What is the overall mortality? (1)
70-80%

A 30-year-old man is brought to your Emergency Department complaining of a red


left eye. The right eye is normal.

Part A

Give 4 key areas in the history which are of importance. For each area give 2 specific
examples of important positive points you may elicit which help in the differential
diagnosis of red eye. (4 Marks)

Part Bi
What is the diagnosis and the underlying causative agent? (2 marks)
Diagnosis:

Causative agent:

Part Bii
Name 2 things you would do to manage this condition? (2 marks)

PRAY FOR ME Dr. Ashraf Elshehry Page 260


1)

2)

Part C
Name 2 non-traumatic causes of a painful red eye that are a threat to vision?
(2 Marks)

A 66 year old man is brought to the ED. He has been experiencing palpitations. He is
comfortable at rest. His heart rate is 160/min.

What are 4 signs of instability? (2)


Reduced conscious level
Systolic BP <90mmHG
Chest pain
Heart failure

If a patient displays these signs, what is the immediate course of action? One drug and
one non drug with doses as required? (1)
Synchronised DC shock up to 3 attempts
Then amiodarone 300mg IV over 10-20 mins followed by 900mg over 24 hours

The ECG shows a narrow complex tachycardia which is irregular. What key feature in the
history will determine your management? (1)
Onset of arrhythmia <48 hours. If yes, cardioversion. If no, rate control

Which drugs can be used in treatment of atrial fibrillation according to the 2005
resuscitation council guidelines? (3)
Digoxin 500mcg loading dose IV
Beta-blocker oral or IV (take your pick ? 25-50mg oral atenolol or 5mg IV)
Both for rate control
Or Amiodarone 300mg over 20-60 mins followed by 900mg/24hrs.

On the bradycardia algorithm, what features indicate increased risk of asysole? (2)
Heart rate < 40
Mobitz 2
Ventricular pauses > 3secs
Complete heart block with wide QRS

Outline initial treatment of bradycardia which has failed to respond to an initial dose of
atropine. Include doses. ( ¼ mark each) (1)
Atropine 500mcg to max of 3mg
Adrenaline 2-10mcg/min
Transcutaneous pacing
Alternative drugs including isoprenaline, aminophyline, dopamine, glucagons if beta-
blocker or calcium blocker, glycopyrollate can be used instead of atropine ? why bother
though

PRAY FOR ME Dr. Ashraf Elshehry Page 261


PRAY FOR ME Dr. Ashraf Elshehry Page 262
Describe the criteria to diagnose Rheumatic fever (10 marks)

Jones Criteria revised 1992.


Need Hx of antecedent sore throat
2 major or 1 major + 2 minor

MAJOR
Carditis
Polyarthritis
Erythema marginatum
Chorea
Subcutaneous nodules

MINOR
Fever
Arthralgia
Long PR interval
Raised ESR/CRP

RTC passenger
A 17 yr old boy is brought to your department. He was a rear seat passenger behind the
driver in a RTC in which there was extensive damage to you car. Also in your department
is the driver, who has sustained a pneumothorax. The patient was mobile at scene but has
been placed on a spine board. He complains of back pain.

What features would enable you to clear his c-spine without radiographs? (4 marks)

Awake, alert, sober, neurologically normal, no neck pain or midline tenderness


(ATLS)

On log roll he has tenderness and swelling at L2.


Would you leave him on the spine board or remove it? (1 mark)
Remove

PRAY FOR ME Dr. Ashraf Elshehry Page 263


What is the name of this fracture with this mechanism? (1 mark)
Chance fracture

What are the risks to the patient? (2 marks)


Potentially unstable spinal fracture
Risk of intra-abdominal injury(15%)

How would you dispose of the patient? (2 marks)


Ortho and surgical referral

1) A 5 month old boy presents with fever, irritability and vomiting. His
temperature at home was 38.0. He has vomited 5 times since
yesterday. Past medical history is unremarkable.

On examination he does not focus or interact well. Anterior fontanelle is full but
he is crying. PEARL. Normal muscle tone. No other abnormalities.

What is the differential diagnosis?

i) Meningitis.
ii) Septicaemia.
iii) Encephalitis.
iv) Shaken baby syndrome.

What initial investigations are appropriate?

PRAY FOR ME Dr. Ashraf Elshehry Page 264


i) BM
ii) Urinalysis
iii) FBC, U&E, CRP, clotting, blood cultures, serology.

What initial treatment should be given?

i) Resuscitate
ii) IV cefotaxime/ ceftriaxone

What subsequent tests should be arranged?

LP is required if meningitis is suspected but ensure CT scan is performed first if


there are focal neurological signs or evidence of elevated ICP. Do not perform LP
if there is coagulopathy.

The CSF was homogenously bloody and did not clear. CT scan without contrast
showed a posterior inter-hemispheric subdural haematoma. Diagnosis of NAI
was made.

2) A 54 year old man fell down some stairs whilst drunk. He awoke in the
morning complaining of pain in his back and that he was unable to
move his legs. He was still unable to stand later that day and came to
the ED by ambulance.

Examination of his legs revealed absent reflexes, flaccid paralysis, absent pain
and temperature sensation but normal touch, vibration and joint position sense.
Sensory level was T11. Bladder was distended and he had no control over
micturition. He was haemodynamically stable. There was no vertebral
tenderness. Urinalysis showed haematuria +++.

The answer given to this was a bit strange. My priorities would be;

A (with cspine assessment and control)/ B/ C/ D/ E


Check BM
There is no evidence of neurogenic shock but monitor BP/ pulse/ ECG
Send blood for FBC/ U&E/ clotting/ LFT/ G+S
Secondary survey
Keep patient warm
X-ray/ CT imaging of spine

This is an anterior cord syndrome; touch and proprioception run dorsally.

3) A 40 year old woman presents with tinnitus and deafness.

PRAY FOR ME Dr. Ashraf Elshehry Page 265


What does the scan show?

A well demarcated lesion in the left cerebello-pontine angle.

What is the most likely diagnosis?

Acoustic neuroma.

List 3 other lesions that occur in this area.

i) cholesteatoma
ii) meningioma
iii) neuromas of CN V, VII, X
iv) basilar artery aneurysm
v) medulloblastoma

What cranial nerves may be affected by such lesions?

V-XI

Vth nerve symptoms most common; depression of the corneal reflex occurs early.
Facial pain, paraesthesiae and numbness may develop.

Large tumours may compress cerebellum, pons and 4th ventricle.

4) Question about Nigerian man c/o falling down stairs recently.

Tuberculoid leprosy – localised form of disease caused by mycobacterium


leprae. Plaques may be erythematous or hypopigmented. Lesions are
anaesthetic and neural involvement is common; it leads to tender thickened

PRAY FOR ME Dr. Ashraf Elshehry Page 266


nerves with loss of function. The superificial peroneal nerve is commonly
affected.

5) A 69 year old lady is brought in complaining of abdominal pain and


generalised weakness. She has become increasingly unwell over the
last 10 days and now appears lethargic and disorientated. She has not
opened her bowels for 5 days and has started to vomit today. She
appears dehydrated and there is microscopic haematuria on urinalysis.

What does the CXR show?

Well circumscribed circular opacities in both lung fields with prominent hilar
markings. Normal heart shadow.

Give two investigations you would like to perform.

Biochemistry – U&E, calcium


CT chest and abdomen

What is your initial treatment plan?

Monitoring and ECG.


Fluid resuscitation.
Manage electrolyte abnormalities as found.

What is the underlying diagnosis?

PRAY FOR ME Dr. Ashraf Elshehry Page 267


The CXR shows cannonball metastases typical of metastatic renal carcinoma.

6) Question about malignant hypertension and retinopathy.

Retinal changes mirror the systemic circulation and their severity correlates well
with the development of systemic complications and survival.

Grade 1 generalised arteriolar narrowing


Grade 2 more marked narrowing with irregular points of focal
constriction
Grade 3 generalised and focal narrowing plus cotton wool spots,
flame haemorrhages, hard exudates
Grade 4 as grade 3 but with swelling of the optic disc, ‘silver wiring’

Diabetic retinopathy is the most frequent cause of new cases of blindness


amongst adults aged 20-74 years. Diabetic retinopathy is classified as:

Background dot and blot haemorrhages, microaneurysms, exudates


Maculopathy macular oedema
Pre-proliferative ↑ cotton wool spots, dot and blot haemorrhages
Proliferative new vessel formation

Remember there may be evidence of laser therapy.

7) This patient has recently started lamotrigine.

What is this rash?

PRAY FOR ME Dr. Ashraf Elshehry Page 268


Toxic epidermal necrolysis. This is an immunological disease most often
caused by adverse drug reactions (antibiotics, NSAIDs, anticonvulsants). Non-
drug causes include viral and bacterial infections, idiopathic and malignancy.

Thought to be related to Stevens-Johnson syndrome.

Needs to be distinguished from staphyloccocal scalded skin syndrome (skin


biopsy).

It is a potentially life-threatening emergency as the patient may develop


dehydration, eye problems, renal involvement leading to ARF, systemic
infections, shock and MOF.

Management consists of treating cause, supportive care, prevent complications.

8) Question about patient presenting with facial swelling, has happened


before, also family history of same.

Diagnosis – angioneurotic oedema (C1-esterase inhibitor deficiency).

Autosomal dominant inheritance, usually presents in 2nd decade.

May also be acquired as paraneoplastic syndrome.

Clinical features;

i) S/C oedema affecting face, limbs, buttocks, genitals (91%).


ii) laryngeal oedema (48% of attacks).
iii) abdominal symptoms.

Precipitating factors include stress, infection, OCP, ACE inhibitors

Management of severe attacks may require;

i) chlorpheniramine 10mg IV
ii) hydrocortisone 200mg IV

PRAY FOR ME Dr. Ashraf Elshehry Page 269


iii) adrenaline 0.5ml of 1:1000 S/C
iv) FFP or C1-esterase inhibitor plasma concentrate

Diagnosis made by low complement C4 levels.

9) Question about young man presenting with spontaneous


pneumothorax.

Know BTS guidelines or else.

10) Question about A-a gradient (Alveolar-arterial oxygen gradient).

In the EM setting the A-a gradient is usually used to evaluate patients with a
suspected pulmonary embolus. It is a fairly reliable measure of oxygen exchange.

Unfortunately there appear to be lots of different formulae, using different units,


calculating at sea level etc……

My formula, using kPa as units is:

FiO2-(PaO2+(PaCO2/0.8))

Where 0.8 is the respiratory quotient.

A normal A-a gradient is less than 2kPa when breathing air; it may reach 4kPa in
the elderly. An increased A-a gradient identifies decreased oxygen in the arterial
blood compared to the oxygen in the alveolus. This suggests a process that
interferes with gas transfer, or in general terms, suggests ventilation-perfusion
mismatch. A normal A-a gradient in the face of hypoxemia suggests the
hypoxemia is due to hypoventilation and not due to underlying lung disorders.

However neither measured PaO2 (18% normal) nor the A-a gradient (6% normal)
are reliably sensitive in predicting PE.

11) A 50 year old man is brought to the ED with a sudden onset of


central chest pain. He has had D&V for 3 days. No significant past
medical history. On examination he is tachypnoeic, tachycardic,
BP 90/40. ECG shows sinus tachycardia, no ischaemic changes.

PRAY FOR ME Dr. Ashraf Elshehry Page 270


What is the diagnosis?

Boerhaave’s Syndrome.

What is the management of this condition?

i) Oxygen.
ii) Fluid resuscitation.
ii) IV antibiotics.
iv) Refer cardiothoracics.

12) A 30 year old IV drug abuser attends the ED with a 2 month history
of widespread rash over his left arm, trunk and leg. The lesions are
similar to those shown in the picture.

PRAY FOR ME Dr. Ashraf Elshehry Page 271


What is the diagnosis?

Molluscum contagiosum.

What is the aetiology?

Pox virus.

What is the prognosis?

Will resolve spontaneously with time but may be removed with liquid nitrogen,
curettage.

What concerns do you have with this particular patient?

Possible HIV infection.

1) This 35 year old man fell onto his outstretched wrist today.

What does the X-ray show?


PRAY FOR ME Dr. Ashraf Elshehry Page 272
Dislocation of the lunate (remember perilunate dislocation is when the lunate stays in
position and the rest of the carpus dislocates).

How would you manage this patient?

Analgesia, assess neurovascular status, immobilise, refer T+O for reduction.

What are the possible complications?

Median nerve injury, avascular necrosis, Sudeck’s atrophy.

2) This 26 year old female presents with a 3 week history of a tender, red,
progressive rash on her legs. She has a history of exposure to TB which is
currently not active. The patient started taking a new OCP 4 weeks prior to
presentation. She is now on her second course of antibiotics for suspected cellulitis but
the rash is not improving.

What is the likely diagnosis?

Erythema nodosum.

What are the possible causes?

i) sarcoidosis
ii) infections: streptococci, TB, infectious mononucleosis, chlamydia, viral
iii) drugs: sulphonamides, OCP, salicylates
iv) inflammatory bowel disease
v) idiopathic

How would you manage this patient?

Advise NSAID, bed rest, elevate legs.


Remove underlying cause – OCP.
Most attacks settle within 2-12 weeks.
Systemic steroids may be used for severe cases.

PRAY FOR ME Dr. Ashraf Elshehry Page 273


3) This ECG is from a 55 year old with type II DM who states she feels weak all over
for the last few days.

What does the ECG show?

Broad QRS complexes


Tall peaked T waves
Small/ absent P waves

What is the likely underlying cause?

Hyperkalaemia secondary to CRF due to DM.

Hyperkalaemia notes

Mild (5.5-6.0)
Moderate (6.1-6.9)
Severe (>7.0)

Causes

i) Factitious e.g. haemolysed sample.


ii) Reduced renal excretion – ARF, CRF, K+ sparing diuretics e.g. spironolactone.
iii) Cell injury e.g. burns, rhabdomyolysis.
iv) Hyperaldosteronism – Addison’s disease, drug induced (NSAID, ACE
inhibitors).
v) K+ cellular shifts – acidosis from any cause (DKA), drugs (suxamethonium).

Clinical features include muscle weakness/ cramps, paraesthesiae, hypotonia.


PRAY FOR ME Dr. Ashraf Elshehry Page 274
Treatment

i) 10ml 10% calcium gluconate.


ii) 10u Actrapid with 50ml of 50% dextrose IV – helps cellular uptake of K+.
iii) Nebulised salbutamol.
iv) Careful fluid balance, correct acidosis with sodium bicarbonate.
v) ? calcium resonium.
vi) Correct underlying cause.

4) A 48 year old lady presents with syncope and hypotension.

What ECG abnormalities are there?

Atrial fibrillation
Right heart strain – RAD
S wave in lead I and T wave inversion in III

What is the diagnosis?

Pulmonary embolism.
Sinus tachycardia is the most common ECG finding. RBBB and right axis deviation are
usually only present in large PE. There may be non-specific T wave changes in the
anterior and inferior leads.

She is previously fit and has no risk factors for PE. What investigations would you
suggest to find the underlying cause?

Doppler USS of legs


CTPA
Inherited procoagulant screen (protein C, S, antithrombin III, Factor V Leiden)
Autoimmune screen (anticardiolipin antibodies, ANA)
USS or CT of abdomen and pelvis to look for occult masses

PRAY FOR ME Dr. Ashraf Elshehry Page 275


If a patient with suspected PE has arrested or is deteriorating, or in a stable patient with
confirmed PE, thrombolysis with alteplase 50mg IV is appropriate.

Five risk factors for PE?

i) Major abdominal/ pelvic surgery


ii) Puerperium
iii) Malignancy
iv) Previous proven PE
v) Lower limb fracture

5) This 48 year old Pakistani woman developed cough and haemoptysis with night
sweats and weight loss.

What is the diagnosis?

Pulmonary tuberculosis.

Which groups of patients are at risk from this?

Alcoholics, recent immigrants, HIV +ve, health care workers.

What test may be useful in determining the diagnosis of TB?

Mantoux test – Tuberculin is injected under the skin; if there is a strong reaction after 72
hours it means that there is a hypersensitivity to Tuberculin acquired by a previous BCG
vaccination or active infection.

What treatment is indicated?

Combination therapy with e.g. rifampicin, isoniazid, pyrazinamide, ethambutol.

PRAY FOR ME Dr. Ashraf Elshehry Page 276


6) This man was assaulted and placed on the observation ward overnight. His
GCS has deteriorated and he has been vomiting. You request a CT scan.

What abnormalities does the scan show?

Fracture of petrous part of temporal bone. There is blood in the sphenoid air sinus
indicating basal skull fracture.

7) A 30 year old woman presents with SOB and dizziness. Her blood pressure is
80/45. This is her ECG.

What is the diagnosis?

AV nodal re-entry tachycardia.

What is the management?

Resuscitation Council guidelines:

i) Support A/ B/ C
ii) Patient is unstable so give synchronised DC shock under sedation.
ii) If unsuccessful give amiodarone 300mg IV over 10 minutes and repeat shock;
followed by amiodarone 900mg over 24 hours.

8) A 12 year old boy presents with history of coin ingestion. An X-ray is


performed. Describe this.
PRAY FOR ME Dr. Ashraf Elshehry Page 277
X-ray shows radio-opaque oesophageal foreign body.

Objects that stick in the oesophagus do so at sites of anatomical narrowing;

i) Cricopharyngeus
ii) Aortic indentation
iii) Diaphragm

Neglected objects may result in oesophageal perforation and mediastinitis.

May require removal with rigid endoscopy. Foley catheters and magnets have also been
used but without much success.

9) A 60 year old lady presents with two hours of central chest pain not
relieved by GTN. Her ECG shows inferolateral ischaemia.

What are the TIMI risk factors?

TIMI= Thrombolysis in Myocardial Infarction trials.

i) Age ≥65 years


ii) ≥3 CAD risk factors
iii) Prior CAD (stenosis >50%)
iv) Aspirin in last 7 days
v) ≥2 anginal events in ≤24 hours
vi) ST deviation
vii) Elevated cardiac markers

The score (0-7) gives the risk of cardiac events (death, MI or urgent revascularisation)
within 14 days in TIMI IIB.

10) A patient attends with a history of general malaise over the past three weeks.
Some blood results are available to you;

PRAY FOR ME Dr. Ashraf Elshehry Page 278


Hb 9.0
WCC 6.9
Plt 190
Na 138
K 7.9
Urea 35.6
Creat 587
Ca 3.05

LFTs are normal.

Give two possible diagnoses?

Essentially this is a hypercalcaemia/ RF question; the RF may be caused by the


hypercalcaemia (e.g. multiple myeloma) or the hypercalcaemia may be caused by the RF
(e.g. CRF with tertiary hyperparathyroidism). A PTH level would be useful. Treatment
priority would be ↓ K+. The classic elderly patient with multiple myeloma has back pain
and presents with acute-on-chronic RF.

Causes of hypercalcaemia include:

i) Primary or tertiary hyperparathyroidism


ii) Malignancy (e.g. myeloma, metastases)
iii) Hyperthyroidism
iv) Sarcoidosis
v) Drug-related e.g. thiazide diuretics, lithium

Treatment of hypercalcaemia is fluids, careful fluid balance, calcitonin, diphosphonate.

11) Question about APLS fitting child protocol – know backwards.

12) List ten notifiable diseases.

i) Anthrax
ii) Cholera
iii) Dysentery
iv) Malaria
v) Measles
vi) Meningitis
vii) Plague
viii) Rabies
ix) Scarlet fever
x) Typhoid fever
1) A 30 year old woman attends the ED with a 2 month history of feeling weak.
She has generalised bone pain and is dyspnoeic. On examination she has a grade 4
generalised weakness and hyporeflexia. She is generally tender.

PRAY FOR ME Dr. Ashraf Elshehry Page 279


Blood results:

FBC normal
Na 137
K 2.8
Ur 4.2
Creat 97
Cl 115
HCO3 15

ABG show metabolic acidosis with respiratory compensation.

What is the anion gap?

(137+2.8)-(115+15) = 9.8

What is the diagnosis?

Hypokalaemic metabolic acidosis with normal anion gap.

Diagnosis is therefore renal tubular acidosis.

Not all cases of metabolic acidosis are caused by increased acid production.
In some cases the primary derangement is loss of bicarbonate as occurs with GI losses
e.g. profound diarrhoea or vomiting, or renal tubular acidosis. In these cases the serum
bicarbonate is low but the body is able to increase the chloride concentration to maintain
electroneutrality.

2) This eye has been stained with fluorescein.

What is the diagnosis?

Corneal dendritic ulcer due to herpes keratitis (HSV type 1).

List 4 things to ask for in the history.

PRAY FOR ME Dr. Ashraf Elshehry Page 280


i) Pain?
ii) Discharge?
iii) Previous herpes infections?
iv) Systemic disorders ~ ? immunocompromised.

Also contact lens use, reduced visual acuity etc.

Give 3 features of management.

i) Analgesia
ii) Topical acyclovir 3% X5/d
iii) Refer ophthalmology

Topical corticosteroids are contraindicated.

3) A 32 week pregnant lady presents with a headache and BP of 160/100.

List 6 symptoms/signs of pre-eclampsia.

i) Headache
ii) Visual disturbance
iii) Hyperreflexia
iv) Abdominal pain
v) Tremor
vi) Reduced urine output

List 4 risk factors for pre-eclampsia.

i) Primiparity.
ii) Maternal systemic disease – DM, renal disease, hypertension.
iii) Low socioeconomic status.
iv) Maternal age <20 or >35 years.

Give 3 immediate treatments.

i) Hydralazine 5mg IV over 20mins to max. 20mg.


ii) Careful fluid balance; preload of 500ml colloid prior to hydralazine may reduce
risk of hypotension and fetal distress.
ii) Magnesium sulphate 4g IV over 5-10 minutes followed by maintenance of
1g/hr for 24hrs.

What are the signs of magnesium toxicity?

Loss of deep tendon reflexes and respiratory depression.

PRAY FOR ME Dr. Ashraf Elshehry Page 281


4) A 75 year old man presents with a right hemiparesis and expressive
dysphasia. He is fully conscious, has a mild pyrexia of 38.0 and his blood
sugar is 10.5mmol/l.

What is the likely diagnosis?

CVA – infarct (80% of strokes).

The definition of stroke is an acute onset of focal neurological deficit of vascular origin
which lasts >24hrs.

Which arterial territory is affected?

Left middle cerebral.

Give 4 immediate managements that may reduce morbidity.

i) Aspirin 300mg after an urgent CT scan to exclude haemorrhage.


ii) Correct hypoglycaemia.
iii) Thrombolysis – rtPA 0.9mg/kg (if within 3hrs of symptom onset→10%
absolute reduction in death or disability).
iv) Admit to specialised stroke unit.

Also optimise airway/oxygenation, careful fluid balance, antipyretic to reduce fever,


correct metabolic disturbances, good nursing care etc.

Give 4 contraindications to thrombolysis.

Absolute:

i) Active internal bleeding (not menstruation).


ii) Known intracranial neoplasm.
iii) Severe uncontrolled hypertension.
iv) Suspected aortic dissection.

5) Give the chronology of ossification at the elbow.

1-2 Capitellum
4 Radial head
6 Internal (medial) epicondyle
8 Trochlea
10 Olecranon
12 Lateral epicondyle

6) A 6 year old boy suffers a laceration over the distal forearm whilst at school and is
brought in by a teacher.

What strategies can be used to facilitate suturing of the wound with minimal distress to
the child? Give 4.

PRAY FOR ME Dr. Ashraf Elshehry Page 282


i) Distraction/ play techniques.
ii) Suitable environment.
iii) Reassurance by good interaction with carers/ parents.
iv) Oral analgesia.

The child becomes very distressed and you are unable to suture the wound safely. You
telephone the child’s mother to explain that her son will need sedation or GA for suturing.
She refuses to come to the ED or give her consent. Give 4 things that have to be done
now.

i) Cover wound.
ii) Determine if she is legal guardian.
iii) Establish non-confrontational relationship and explain need for treatment.
iv) Consult with ED senior/ paediatrics team.

7) A 29 year old man presents with sore throat and left ear pain for the past 5 days.
On examination he is febrile, with cervical lymphadenopathy and a marked left facial
droop. Vesicles are noted in his left ear. The patient has vesicles and blisters on the left
side of the tongue. Examination is otherwise unremarkable.

What is the diagnosis?

Ramsay-Hunt syndrome. Facial nerve palsy caused by herpes zoster infection of the
geniculate ganglion. There may also be loss of taste on the anterior part of the tongue,
tinnitus, hearing loss and vertigo.

Give analgesia and refer to ENT for IV acyclovir and eye care.

8) A 30 year old woman with history of sickle cell disease presents to the ED
with severe pleuritic chest pain and breathlessness. She denies haemoptysis or leg
swelling. On examination she has a temperature of 38.0, RR 22/min, pulse 98/min,
BP 126/65, SaO2 93% on RA. Chest examination is unremarkable apart from a few
crackles in the right lower lung field.

Give 3 differential diagnoses.

PRAY FOR ME Dr. Ashraf Elshehry Page 283


i) Sickle cell ‘acute chest syndrome’
ii) Pneumonia
iii) Pulmonary embolism

Outline your initial treatment.

i) Keep patient warm, rested and give O2.


ii) Opioid analgesia is usually required for pain.
ii) Careful rehydration with fluids – crystalloid.
iv) Empirical antibiotic therapy e.g. cefuroxime 750mg IV tds.
v) Exchange transfusion may be required; aim for Hb between 7-9g/dl as any
higher can increase blood viscosity and precipitate further sickling.

List the most useful investigations

i) FBC
ii) Infection screen – blood culture, CXR, MSU
iii) U&E, ABG, ECG

Patients with sickle cell trait usually have no disability except at times of severe hypoxia.
Patients with sickle cell anaemia have chronic anaemia (8-10g/dl) and a small percentage
have recurrent crises. Later in life, chronic ill-health supervenes with renal failure, bone
necrosis, osteomyelitis (Salmonella), there is an increased susceptibility to infection, leg
ulcers.

Sickle cell crises can occur spontaneously or follow infection, cold, dehydration or any
situation where tissue hypoxia exists. Acute medical and surgical emergencies may be
mimicked.

i) Acute painful crises are the most common presentation; severe pain at one or
more sites associated with pyrexia, tenderness and local warmth and swelling.
Haemolysis may be increased – there is a fall in Hb and reticulocyte count is
increased. There are no reliable markers to indicate severity.

ii) Chest crisis is the most common cause of mortality. There is vaso- occlusion of
the pulmonary microvasculature resulting in local infarction. May be precipitated by
infection.

iii) Cerebral infarction: usually children <5yrs, rare in adults. Presents as acute
stroke.

iv) Splenic/ hepatic sequestration: usually children <5yrs, RBCs become trapped
in spleen and liver, causes severe anaemia and circulatory collapse.

v) Aplastic crisis: usually in children and young adults, mainly caused by


parvovirus infection. Reduced reticulocyte count.

vi) Priapism: Local vaso-occlusion causes prolonged, painful erections.


Urological emergency.

PRAY FOR ME Dr. Ashraf Elshehry Page 284


9) This is the CXR from a 25 year old man who admits to regular use of crack
cocaine. He has presented with SOB and pleuritic chest pain. Clinical examination
is unremarkable.

Describe the X-ray appearance.

Ill-defined nodular soft tissue shadowing throughout both lungs, predominantly of a


perihilar distribution.

What is the diagnosis?

Crack pneumonitis.

Give 3 differential diagnoses.

i) Atypical pneumonia.
ii) Sarcoidosis.
iii) Extrinsic allergic alveolitis.

Respiratory complications of crack use can include pneumonitis, infections, pulmonary


oedema, pulmonary fibrosis and ARDS.

10) Question about urethral injury.

Possible signs of urethral injury:

i) Perineal bruising
ii) Blood at external urethral meatus
iii) Abnormally high-riding prostate
iv) Inability to palpate prostate

Remember the parts of the urethra;

PRAY FOR ME Dr. Ashraf Elshehry Page 285


Prostatic urethra (3cm, receives prostatic ducts and ejaculatory ducts), membranous
urethra (2cm, pierces voluntary sphincter of bladder), anterior urethra (15cm, traverses
corpus spongiosum and made up of bulbar and penile sections).

Rupture of the membranous urethra occurs in about 30% of severe pelvic fractures~ the
prostatic urethra is fixed so the membranous part gives way. Blunt trauma to the perineal
area may result in bulbar trauma.

Investigation may involve retrograde urethrogram or antegrade cystography via an SPC.

The main complication of urethral trauma is recurrent stricture.

11) A 24 year old girl presents one week after a large partial thickness burn to her leg.
She has severe diarrhoea and is unwell. On examination she is drowsy, febrile and
shocked. No focal neurology. There is a widespread macular rash.

Investigations:

Hb 14.5 WCC 14 Plt 100


Urea 10 Creatinine 158 Potassium 7
CK 400

Give the most likely diagnosis.

Toxic shock syndrome ~ caused by exotoxins from Staph. aureus (occasionally


streptococci). Some cases during menstruation are related to tampons, others occur after
burns, surgery or other trauma.

There is high fever, generalized rash, confusion, diarrhoea, muscle pains, hypotension and
renal failure. There may be desquamation from hands and feet. Death may occur from
MOF.

Treat for shock with circulatory support and IV flucloxacillin. Remove tampons if
present. Send urine for urine MC+S. Take vaginal, throat and wound swabs.

12) Question about electrical injury to hand ~ patient has difficulty flexing MCPJ
and extending IPJ of ring and little fingers.

Which muscles are responsible for these movements?

3/4th lumbricals.

Remember ~ interossei abduct/ adduct fingers.

Nerve supply to 3/4th lumbricals (and interossei and adductor pollicis) is from the ulnar
nerve.

Describe the position of the fascial spaces of the hand.

i) Superficial pulp spaces of fingers


PRAY FOR ME Dr. Ashraf Elshehry Page 286
ii) Synovial tendon sheaths of the flexor tendons ~ that of the 2nd, 3rd and 4th
fingers are closed off proximally at the metacarpal head but the synovial sheaths of the
thumb and little finger extend into the palm (see diagram).
iii) Midpalmar space
iv) Thenar space

Diagram shows flexor tendon sheaths; midpalmar and thenar spaces lie deep.

1) What is the diagnosis?

Butterfly rash of SLE.

This is a chronic autoimmune disorder characterised by the production of a range of


autoantibodies, most commonly ANA. Commoner in young women.

Patients may present as a new diagnosis or with a flare up of the disease.

Clinical features (in descending order of frequency);

Constitutional fever, malaise, weight loss


Musculoskeletal athralgia, myalgia

PRAY FOR ME Dr. Ashraf Elshehry Page 287


Cutaneous butterfly rash, photosensitive rash, discoid lupus,
Raynaud’s
Haematological thrombocytopenia, anaemia, leucopenia
Neuropsychiatric depression, psychosis, fits, CN lesions, ataxia
Renal glomerulonephritis, nephritic syndrome
CVS or RS pleurisy, pericarditis, pericardial/ pleural effusions
Apthous ulcers

What 4 important emergency investigations would you now consider?

FBC, U&E, CRP, CXR, urinalysis, ECG

Other investigations include ANA, DNA, ENA, ACA, complement levels, viral serology,
24hr urine collection.

80% of patients are ANA +ve. Pneumococcal and meningococcal infections are more
common in patients with SLE as a consequence of deficiencies of the complement
pathway.

Treatment is with steroids, immunosuppresants e.g. azathioprine, antibiotics if infection


suspected.

2) A 35 year old woman presents with atraumatic pain and swelling in the left
calf. According to the Wells criteria she has a moderate risk of DVT.

What information is required to calculate a Wells score?

Wells et al 2001:

1) active cancer (treatment ongoing or within 6 months of palliative) 1


2) paralysis, paresis or immobilisation of lower limb 1
3) recently bedridden >3 days or major surgery within 4 weeks 1
4) localised tenderness along the deep veins 1
5) entire leg swollen 1
6) calf swelling 3cm more than asymptomatic side 1
7) pitting oedema confined to affected leg 1
8) dilated superficial veins 1
9) alternative diagnosis as likely or greater than DVT -2

Wells categorized patients into;

Low risk (score ≤0)


Moderate risk (score 1 or 2)
High risk (score ≥3)

The use of the Wells score is as a ‘rule out’ test in combination with D-dimer testing; i.e.
those patients who have a low risk and a -ve D-dimer do not require further investigation
for DVT. Anyone with a moderate risk should undergo duplex USS.
PRAY FOR ME Dr. Ashraf Elshehry Page 288
3) An obese 57 year old man presents with a history of sudden visual disturbance.
This affected his left eye, came on over a few seconds and obscured all vision in that
eye. He also noticed at the same time his handwriting deteriorated and he had
difficulty holding a pen.

Which artery has been affected?

Amaurosis fugax ~ left internal carotid. Other features of carotid TIA may be hemiparesis
or dysphasia. Most TIAs result from thrombo-embolic disease involving either the heart
or extra-cranial vessels.

Differential diagnosis includes cerebral tumour, focal migraine, Todd’s paresis,


hypoglycaemic episode and other causes of monocular visual loss e.g. retinal vessel
occlusion, temporal arteritis, vitreous haemorrhage etc.

Ask about risk factors e.g. hypertension, polycythaemia, anaemia, vascultits, sickle cell
disease. Look for AF, heart murmurs (mitral stenosis, artificial valves), carotid bruit,
evidence of AMI.

Check BM, send bloods and get ECG and CXR.

4) A 45 year old woman with a long-standing history of RA presents with a 6-


month history of worsening dyspnoea. She does not experience orthopnea. No raised
JVP, heart sounds normal. ECG is normal. Blood gases on air show type 1 respiratory
failure, no acidosis.

What is the probable diagnosis?

Pulmonary fibrosis 2° to RA. Other extra-articular features may include SC nodules,


vasculitis, splenomegaly, neuropathy, anaemia, pleurisy, pericarditis and eye problems.

What other physical signs would you look for?

RA is a symmetrical polyarthritis typically affecting the hands and feet of young women.
Remember cspine involvement. X-rays show soft tissue swelling, peri-articular erosions
and joint space narrowing, deformities.

She should undergo CXR, ? CT chest and spirometry.

5) A 65 year old woman presents c/o severe headaches for several weeks and of
now having lost vision in one eye. The eye is not red or painful.

Investigations show FBC normal, ESR 90.

What is the probable diagnosis?

PRAY FOR ME Dr. Ashraf Elshehry Page 289


Temporal arteritis. Beware in any patient >50yrs who presents with new headache or
change in headache, weight loss, night sweats and jaw claudication. There is an
association with polymyalgia.

What features would be important in the physical examination?

i) tenderness over temporal artery or loss of pulsation.


ii) fundoscopy – papilloedema may occur late in the disease.
iii) if the patient is in AF or has a carotid bruit then need to consider other causes
of painless monocular visual loss, e.g. central retinal artery occlusion, stroke.

What diagnostic test will confirm this diagnosis and what treatment is indicated in the
ED?

Temporal artery biopsy. Hydrocortisone 200mg IV.

What may be the SE of steroids in the elderly?

Loss of diabetic control, peptic ulceration, hypertension, thinning of skin (bruise easily),
osteoporosis.
6) A 45 year old man presents with a one day history of a painful, watering eye.
He has had similar symptoms before but never this badly; he can recall no trauma.

Give five differential diagnoses.

i.e. list 5 causes of a painful red eye ~ should know something about all of these.

i) conjunctivitis
ii) foreign body in eye/ corneal abrasion
iii) acute uveitis
iv) acute closed angle glaucoma
v) ulcerative keratitis

Which is most likely and why?

Acute uveitis. The pupil is irregular due to previous adhesions.

Give 5 associated diseases.

Ankylosing spondylitis, ulcerative colitis, sarcoid, AIDS, Behcet’s syndrome.

Outline your management plan.

PRAY FOR ME Dr. Ashraf Elshehry Page 290


Give analgesia. Check VA. Pain on accommodation as pupils react is called Talbot’s test.
Fundoscopy, slit lamp examination. Refer to ophthalmology for steroid eye drops.

7) A 76 year old woman presents following a three day history of polydipsia and
urinary frequency. On examination she is pyrexial, drowsy and severely dehydrated.

Investigations:

Hb 16.2
WCC 19.6
Plt 410

Na 160
K 5.2
Cl 128
HCO3 23
Urea 31
Creat 160

Urinalysis ~ glucose +++, ketones +

What is the likely diagnosis?

HONK. This usually occurs in elderly patients with NIDDM and can develop over days
or weeks; glucose levels are often >30mmol/l. It often occurs with intercurrent illness,
especially infection. Patients are usually severely dehydrated and there is impairment of
consciousness.

Diagnosis is made by:

i) hyperglycaemia with osmolality >350mmol/l (normal 280-305)


ii) no acidosis
ii) <++ ketones on urinalysis

There may be a coexistent lactic acidosis (which implies a poor prognosis).

Suggest 4 essential investigations.

ABG, blood glucose, septic screen, ECG ~ look for evidence of infection and AMI or
myocardial ischaemia.

Apart from ABC, what should the initial treatment be?

Mainstays of treatment are fluid resuscitation and insulin.

IV fluids:

1l in 1hr
1l in 2hrs
PRAY FOR ME Dr. Ashraf Elshehry Page 291
1l in 2 hrs
then continue with 1l every 4hrs.
if Na+ <160mmol/l use normal saline
if Na+ >160mmol/l use ½ normal saline

K+ is usually normal; no K+ in first litre of fluid, subsequent replacement depends on K+


level.

Insulin infusion commenced (50U Actrapid in 50ml N/saline – start at 3U/hr) to maintain
fall of about 3-6mmol/hr.

Full anticoagulation with heparin.

May need catheter and CVP line.

NG tube if consciousness impaired.

Treat underlying cause if found e.g. UTI.

ICU/ HDU admission.

8) Outline the current guidelines for tetanus prophylaxis.

Standard active immunisation involves an initial course of 3 doses of tetanus toxoid at 2,


3 and 4 months of age followed by booster doses at 4yrs and 14yrs. A full course of 5
doses is considered to give lifelong immunity. Inadequate immunity against tetanus is
likely in immigrants, the elderly, patients who are immunosuppressed and those who have
refused vaccination.

The following wounds are regarded as ‘tetanus prone’:

i) heavy contamination (esp. soil or faeces)


ii) devitalised tissue
iii) infected or wounds >6hrs old
iv) puncture wounds and animal bites

For fully immunised patients, a dose of human anti-tetanus immunoglobulin (HATI,


250U IM) is only necessary for very high-risk wounds. For other patients, continue/ begin
the standard schedule and give HATI for tetanus-prone wounds.

Standard immunisation schedule:

2 months D, T, P, polio, Hib, meningitis C


3 months D, T, P, polio, Hib, meningitis C
4 months D, T, P, polio, Hib, meningitis C
12-15 months MMR
3-5yrs D, T, P, polio, MMR
10-14yrs BCG
13-18yrs D, T, polio
9) WPW question.
PRAY FOR ME Dr. Ashraf Elshehry Page 292
Impulses are conducted from the atria via the AV node and an accessory pathway (bundle
of Kent). The accessory pathway conducts more quickly than the AV node so the PR
interval is short. The region of ventricle activated by the accessory pathway slowly
depolarises giving rise to a delta wave. Shortly afterwards the rest of the ventricular
muscle is depolarised by the arrival of the impulse from the AV node.

It is one of the commonest causes of tachyarrhythmias in children (may be accompanied


by palpitations, dizziness, faints, chest pain) but can be asymptomatic. In infancy 80% are
idiopathic but other causes include ASD and cardiomyopathy. It can present with AF
associated with WPW ~ consult cardiology as this a potentially dangerous rhythm.

Patients with WPW should not be given drugs that block the AV node (digoxin, calcium
channel blockers) as this can result in acceleration of conduction through the accessory
pathway leading to VF.

Treat tachyarrhythmias as per Resuscitation Council tachyarrhythmia guideline.


Adenosine is OK if the tachyarrhythmia is regular but if in doubt consult cardiology. Cure
may be achieved by radiofrequency ablation.

Adenosine acts by slowing conduction through the AV node. Maximum dose is 12mg. It
has a very short half-life. CI include 2nd or 3rd degree heart block, sick sinus syndrome,
AF and atrial flutter. Caution in patients with asthma as it may induce
bronchoconstriction.

10) Burn question: person with 36% burns. What are the fluid requirements?

4ml X (burn surface area) X (body weight (kg))


4 X 36 X 70 = 10,080ml

50% given in first 8 hours, 50% over next 16 hours. Object is to obtain urine output of
1ml/kg/hr.

Children receive maintenance requirements in addition to above amount.


11) A 3 year old child presents after 4 days of D&V. He is afebrile with a dry
mouth and a pulse rate of 150.

What is his maintenance fluid requirement?

PRAY FOR ME Dr. Ashraf Elshehry Page 293


Need to calculate percentage dehydration:

Mild (<5%)
Thirst
Dry mouth
Concentrated urine

Moderate (5-10%)
Sunken fontanelle/ sunken eyes
↓urinary output (<4 wet nappies/24hrs in a baby)
Tachypnoea
Tachycardia

Severe (>10%)
Hypotension (very late)
Skin turgor
Drowsiness/ irritability

Maintenance requirements are:

100ml/kg/day for first 10kg


50ml/kg/day for next 10kg
20ml/kg/day for each subsequent kg

So his maintenance requirements are:

Estimated weight (age+4) X 2 = 14kg


(10 X 100) + (4 X 50) = 1200ml

Assume deficit of 10%:

10 X 10(%) X 14 = 1400ml

So total daily requirement is 2,600ml. Use 0.45% saline/ 5% dextrose if not able to
tolerate oral rehydration or is deteriorating.

12) Neonatal resuscitation question ~ know the Resuscitation Council Guideline


(2005).

APGAR scores:

APGAR scores are done at 1 and 5 minutes post delivery.

2 1 0

Heart rate >100 <100 Absent

PRAY FOR ME Dr. Ashraf Elshehry Page 294


Respirations Good, crying Slow, irregular Absent

Muscle tone Active motion Some flexion Limp


Reflex Cough or
irritability grimace No response
(catheter in nares)
sneeze
Completely Pink body, blue
Colour Blue or pale
pink limbs

Remember if you need to cannulate the umbilical vein (fastest method of venous access in
newborn) it is the single large dilated vessel adjacent to the 2 constricted arteries. Insert a
5F catheter 5cm into the vein and secure with a tie.

There was another question about a 10 week old baby with feeding difficulty since birth ~
picture showed floppy baby.

Causes:

i) Muscle weakness – NMJ problem or spinal cord lesion.


ii) Normal muscle strength – cerebral palsy, Prader-Willi, Downs,
hypothyroidism.

One more question about chickenpox pneumonia in a child ~ remember this is usually
staphylococcal in children.

1) 3 pictures: Mallet finger


Boutonniere deformity
Swan neck deformity

Explain the mechanism for these deformities.

Mallet: avulsion fracture at dorsal aspect of base of terminal phalanx or


avulsion of terminal portion of extensor tendon.
Boutonniere: rupture or laceration of central slip of extensor tendon, remaining
lateral parts of extensor tendon slip along side of finger
producing characteristic deformity.
Swan neck: damage to volar plate of the PIP joint either by trauma or
degeneration as in RA.

2) Other possible rheumatology questions:

i) Ankylosing spondylitis.

Usually presents as chronic low back pain in men aged 15-30. There is progressive spinal
fusion and immobility. Other features include iritis, apical lung fibrosis and plantar

PRAY FOR ME Dr. Ashraf Elshehry Page 295


fasciitis. There may be a normochromic anaemia and ↑ ESR. X-ray shows bamboo spine,
obliterated SI joints.

ii) Reiter’s syndrome.

Triad of urethritis, conjunctivitis and seronegative arthritis. May cause monoarthritis,


typically of larger lower limb joint. Other features include psoriaform skin lesions
(keratoderma blenorrhagicum), circinate balanitis and plantar fasciitis. May progress to
give aortic incompetence, heart block, pericarditis.

iii) Behcet’s syndrome.

Polyarthritis (± erythema nodosum) with painful orogenital ulceration and iritis.

iv) Felty’s syndrome.

A variant of RA characterised by RA, splenomegaly, leucopenia and recurrent infections.


Splenectomy may improve the WCC.

v) Enteropathic arthropathies.

Associated with inflammatory bowel disease.

vi) Gonococcal arthritis.

Genital infection may be silent.

3) A 24 year old woman presents with this appearance 2 days after a minor
laceration of her right leg.

Give 2 differential diagnoses.

i) Necrotising fasciitis

ii) Gas gangrene

Which organisms are involved for each differential?

PRAY FOR ME Dr. Ashraf Elshehry Page 296


i) Group A haemolytic Strep. pyogenes, may also be Staph. aureus and
anaerobes.

ii) Clostridium perfringens (anaerobic Gram +ve bacillus, produces exotoxins).

Treatment involves analgesia, resuscitation, IV antibiotics (penicillin with


metronidazole), surgical debridement of affected tissues. Sometimes hyperbaric oxygen is
used and gas gangrene antitoxin may be useful if Clostridium is suspected.

4) An 18 year old patient collapsed at school whilst playing football. On arrival


in the ED GCS is 15. Haemodynamically stable. Physical examination is normal. A
sibling had died in similar circumstances 10 years earlier.

What are the differential diagnoses?

i) Vasovagal episode.

ii) HOCM

iii) Hereditary long QT interval (Lange-Nielsen (high-tone deafness) and


Romano-Ward syndromes). Both carry risk of ventricular arrhythmias and are
associated with torsades de pointes and sudden cardiac death.

The 12 lead ECG is shown.

List some causes of a prolonged QT interval.

i) Hereditary (described above).

ii) Hypocalcaemia: clinical features include paraesthesiae, tetany, fits and


psychiatric disturbance.

Look for Trousseau’s sign (carpal spasm when brachial artery occluded with BP cuff) and
Chvostek’s sign (twitching of facial muscles when tapping facial nerve) and
papilloedema.

iii) Drugs: antiarrhythmics (quinidine, amiodarone, sotalol), antihistamines,


antimalarials, organophosphates.

PRAY FOR ME Dr. Ashraf Elshehry Page 297


iv) Hypomagnesaemia

v) Hypokalaemia

vi) Intrinsic heart disease (IHD, myocarditis).

vii) SAH

viii) Hypothermia

Congenital long QT syndromes may be treated with long-term propranolol or an ICD.


Other family members should be screened for disease.

5) Question about infectious diseases of childhood.

Measles

Viral infection, droplet spread. Incubation 10-14 days.


Features ~ fever, malaise, coryza, conjunctivitis, cough. Koplik’s spots. Spreading
maculopapular rash.
Treatment is symptomatic unless complications ensue e.g. otitis media, bacterial
pneumonia, encephalitis. Mortality low in UK.

Mumps

Viral infection, saliva and droplet spread. Incubation 14-18 days.


Features ~ fever, pain and swelling of parotids, orchitis (10%). Aseptic meningitis may
occur. Treament is with analgesia and possibly steroids for orchitis.

Rubella

Viral infection, airborne spread. Incubation 2-3 weeks.


Usually a mild disease with rash, mild fever, occipital lymphadenopathy and arthralgia.
Infection during pregnancy may cause severe congenital disorders. If any concern take
blood for viral antibody levels.

Pertussis

Bordetella pertussis. Notifiable disease. Incubation 5-14 days.


Features ~ coryza wirth worsening cough, may persist for weeks. Risk of apnoeic
episodes in infants. Treat with erythromycin.

6) A 25 year old man attends after returning from a diving holiday that day. He
complains of mild headache, lower back pain and painful (non- tender) knees.

Give 4 important points in the history.


PRAY FOR ME Dr. Ashraf Elshehry Page 298
i) Time of onset of Sx related to the dive.

ii) Dive profile (depth, duration, activity, speed of ascent, water temp. etc).

iii) Previous medical history.

iv) Did he fly back? (decompression illness may be precipitated if insufficient


time is left between diving and flying).

If suspected, discuss with Duty Diving Doctor. Treatment is recompression, pending this
give high-flow O2, IV fluids and aspirin (to prevent sludging).

7) This girl is 5 years old. Capillary refill time is 5 seconds.

What is the diagnosis?

Meningococcaemia.

What organisms are responsible and what is appropriate initial management?

Meningitis may be bacterial, viral or rarely, fungal. Usual bacteria are Neisseria
meningitidis or pneumococcus. Other bacteria (e.g. TB, Listeria) may cause meningitis in
the elderly, the immunosuppressed and neonates.

Initial management consists of A/B/C and:

i) cefotaxime or ceftriaxone 80mg/kg.


ii) look for signs of shock or raised ICP (decreasing or fluctuating level of
consciousness, unequal or poorly reacting pupils, focal neurological signs,
abnormal posturing or seizures).
iii) if shocked give colloid bolus (20ml/kg 4.5% HAS) and repeat if necessary;
observe closely, may require inotropes, intubation etc. on PICU.

PRAY FOR ME Dr. Ashraf Elshehry Page 299


iv) if evidence of ↑ ICP, give mannitol (0.25g/kg bolus) followed by frusemide
(1mg/kg) and steroids (dexamethasone 0.4mg/kg bd). Treat seizures as usual. Will
require intubation and PICU.

What risks are there to healthcare workers?

Minimal; prophylaxis is unnecessary unless mouth-to-mouth resuscitation has occurred.


Household contacts should be given rifampicin (warn about orange discoloration of urine
and interaction with OCP).

Remember to inform the Public Health Department.

8) This patient was cleaning masonry when he received this burn.

What is the causative agent?

Hydrofluoric acid. HF acid rapidly crosses lipid membranes and penetrates tissues deeply
where it releases the highly toxic fluoride ion. These ions may gain access to the
circulation producing a variety of systemic problems, notably hypocalcaemia.

What is the immediate management?

i) Analgesia.
ii) Copious lavage.
iii) Calcium gluconate gel may be applied to the burn.
iv) Check serum Ca2+, U&E and Mg2+.
v) Record ECG and monitor.
vi) Treat hypocalcaemia.
vii) Call plastics team at an early stage.

9) This lady has a temperature of 38.5°.

PRAY FOR ME Dr. Ashraf Elshehry Page 300


What is the diagnosis?

Erysipelas. This is a skin infection typically caused by Group A β-haemolytic


streptococci. It is a more superficial infection than cellulitis. It is characterised by intense
erythema, induration and a sharply demarcated border. The lower limbs are commonly
affected, the face in 5-20%.

What is the appropriate management?

Analgesia and antibiotics; penicillin or erythromycin.

10) Picture of elderly female hand with inability to extend thumb.

Rupture of EPL may occur a few weeks after (usually undisplaced) fracture of the distal
radius. Tendon ruptures are also associated with RA, OA, CRF and SLE.

11) This patient has swallowed a button battery.

PRAY FOR ME Dr. Ashraf Elshehry Page 301


What is the appropriate course of action?

NPIS advice:

i) Batteries lodged in the oesophagus require immediate retrieval by endoscopy.


ii) Batteries in the stomach require review at 48 hours to ensure that they have
passed through the pylorus; if not then they require endoscopic removal.
ii) If the battery has passed through the pylorus and remains asymptomatic then
stools should be monitored for up to one week and the patient reviewed if the battery has
not passed.
iv) If at any time the patient develops symptoms or signs of GI bleeding or
obstruction then the battery should be retrieved.

12) This patient has been bitten on the hand during a fight.

What treatment is indicated?

i) Analgesia.
ii) History regarding tetanus status.
iii) X-ray.
iv) Wound irrigation/ exploration.
v) Augmentin.
vi) Counsel regarding HIV and hep. B transmission: if thought to be high- risk
then give prophylaxis.

1) A 74 year old man presents with a 7 day history of right sided chest pain.
He is a smoker and has NIDDM, otherwise previously well.

PRAY FOR ME Dr. Ashraf Elshehry Page 302


What is the diagnosis?

Lung abscess right upper lobe.

List 6 important symptoms to elicit.

i) Cough ? bloodstained/ offensive sputum.


ii) Fever.
iii) Night sweats.
iv) Nature of pain ? pleuritic.
v) SOB.
vi) Anorexia or weight loss.

The most common cause of lung abscess, or empyema (pus in the pleural cavity), is
aspiration. Patients at risk include the elderly, alcoholics, those with poor dentition or
primary lung disease. Other causes of empyema include penetrating chest trauma
(including chest drains) and oesophageal rupture.

The patient is usually elderly and the abscess is most commonly located in the dependent
part of the lung on the right side. Organisms are usually polymicrobial oral flora e.g.
Bacteroides and Fusobacterium.

Increasingly in the paediatric population S. aureus has become the predominant organism
because of the use of the pneumococcal conjugate vaccine.

Treatment is with broad-spectrum antibiotics and drainage by tube thoracostomy.

2) A 40 year old man presents with a 10 day history of a penile ulcer.

PRAY FOR ME Dr. Ashraf Elshehry Page 303


What is usually classical of this lesion to help diagnosis?

It is a painless ulcer – usually solitary, may be multiple. Usually associated with regional
lymphadenopathy.

What is it?

Primary chancre of syphilis.

What is the offending organism and how is it transmitted?

Treponema pallidum, a spirochaete. It is almost always transmitted by sexual contact with


infectious lesions but can be transmitted in utero and by blood transfusion.

List 4 other causes of genital ulceration.

i) Other infections e.g. herpes simplex, gonorrhoea.


ii) Neoplasms e.g. carcinoma of penis
iii) Behçet’s disease.
iv) Trauma (may be self-inflicted).

What investigations could confirm the diagnosis?

Direct visualisation by darkfield microscopy.


VDRL serology.

What is the treatment of choice?

Primary and secondary syphilis are highly responsive to penicillin and cure is likely.

How may secondary syphilis present?

There is often a localised or diffuse mucocutaneous rash and generalised


lymphadenopathy. Constitutional symptoms include malaise, sore throat, headache, fever,
arthralgia and myalgia. Other less common manifestations include hepatitis,
nephropathy, optic neuritis, proctitis.

PRAY FOR ME Dr. Ashraf Elshehry Page 304


Generalised rash of secondary syphilis.

3) This patient presents with a rash on the hand.

What is the diagnosis?

Bowen’s disease.

Usually results from chronic sun exposure. It may develop into an intraepidermal
squamous cell carcinoma. Early lesions may resemble fungal infections, dermatitis or
psoriasis. It is slow growing and metastasises rarely.

Diagnosis is by skin biopsy. Treatment is with cryotherapy, surgical excision,


photodynamic therapy or 5-FU cream.

PRAY FOR ME Dr. Ashraf Elshehry Page 305


4) This woman presents with a rash.

What is the diagnosis?

Necrobiosis lipoidica.

Commonly affects the shins, seen more often in women. More than fifty percent of
sufferers have DM. It is a chronic condition; ulceration may occur.

Flare-ups may respond to cortisone cream or UV light. Aspirin may also help.

5) You undertake fundoscopy on a patient; there is no history of visual


problems.

What is the diagnosis?

Congenital anomaly: myelinated retinal nerve fibres. Myelination does not normally
extend onto the retina; when it does the appearance is as shown. Vision is unaffected.

PRAY FOR ME Dr. Ashraf Elshehry Page 306


6) This patient suffered an MI 6 months ago and presents with chest pain.

What is the diagnosis?

Left ventricular aneurysm.

Occur after 2-15% of infarcts, usually on left ventricle.

Complications may include:

i) Heart failure (filling of the aneurysm during systole reduces the EF).
ii) Ventricular arrhythmias.
iii) Persistent angina.
iv) Systemic emboli.

Usually treated by limiting activity and close follow-up. Surgical removal is considered
for persistent arrhythmias or heart failure.

7) This Asian gentleman presents with abdominal pain.

What is the diagnosis?

PRAY FOR ME Dr. Ashraf Elshehry Page 307


Grey Turner’s sign. Acute pancreatitis.

List some common aetiologies of pancreatitis.

i) Idiopathic.
ii) Obstruction; gallstones, tumours.
iii) Toxins; alcohol, drugs e.g. salicylates.
iv) Trauma.
v) Infection.
vi) Pancreatic structural anomalies.

Complications?

Local

Necrosis ± infection.
Fluid collections.
Pseudocysts.
GI haemorrhage.

Systemic

Shock.
Coagulopathy.
Renal failure.
Respiratory failure.
Hyperglycaemia.
Hypocalcaemia.

8) What is the diagnosis?

Tigroid fundus.

If the retinal epithelium is not well pigmented, as in people with blonde or red hair, the
underlying choroidal vessels may become strikingly visible.

PRAY FOR ME Dr. Ashraf Elshehry Page 308


9) This patient tripped and fell injuring her wrist.

What is the diagnosis?

Smith’s fracture.

What is the treatment?

Analgesia, immobilise, refer T+O for buttress plate.

10) This 14 year old boy was hit by a car sustaining the following isolated injury.

Apart from initial A/B/C etc, how would you manage this?

Analgesia:

i) IV morphine 0.1mg/kg = 3.6mg.

PRAY FOR ME Dr. Ashraf Elshehry Page 309


ii) Femoral nerve block (max. dose is 3mg/kg, there are 10mg/ml in 1%
lignocaine = 108mg = 11ml).

Immobilise in Thomas splint or skin traction. Refer T+O.

11) This child presents with a painful right eye, pyrexia and inability to move
the eye on examination.

What is the diagnosis?

Orbital cellulitis.

Name 2 possible causative organisms.

Since invasive H. influenza infection has been all but eradicated by immunisation, S.
aureus and Strep. pneumoniae are the commonest pathogens.

What is the likely source of infection?

It is usually caused by spread from the sinuses (ethmoidal or para-nasal) but may arise
from local trauma (e.g. bites, foreign body) or haematological spread.

List 3 complications.

i) Cavernous sinus thrombosis.


ii) Cerebral abscess.
iii) Optic nerve compression leading to loss of vision.

Patients should be admitted under joint care of paeds./ ENT/ eyes.


Antibiotics – IV flucloxacillin and metronidazole. Essential investigation is CT scan of
orbit and sinuses.

PRAY FOR ME Dr. Ashraf Elshehry Page 310


12) A 78 year old lady is sent in from a nursing home. She has been previously
well apart from mild dementia but over the last few days she has become unwell, is
not feeding and is now drowsy. She is afebrile, PR 120 irregular, clinically
dehydrated. CXR shows a RLL pneumonia.

What initial investigations, apart from CXR, are appropriate?

All patients admitted to hospital: FBC, U&E, CRP, ABG, blood cultures,
sputum cultures.

For patients with severe CAP pneumococcal antigen, legionella urine antigen, chlamydial
antigen and mycoplasma CFT are appropriate.

What features may indicate an adverse prognosis?

Pre-existing

Age >50 years.


Presence of co-existing disease.

Core clinical adverse features (CURB 65)

Confusion: new confusion or defined as an AMT score of 8 or less.


Urea: raised >7mmol/l.
Respiratory rate: raised ≥30/min.
Blood pressure: low BP systolic <90 and or diastolic ≤60.
Age: >65 years.

A score of 2 or greater on CURB 65 means hospital treatment is usually necessary.

Additional adverse features

Hypoxaemia: SaO2 <92% or PaO2 <8kPa regardless of FiO2.


Bilateral or multilobe involvement on the CXR.

What antibiotic therapy is recommended?

For hospital-treated, not severe CAP: amoxicillin 500mg tds plus clarithromycin 500mg
bd.
For severe CAP: co-amoxiclav 1.2g tds IV plus clarithromycin 500mg bd.

See BTS Guideline for full details of recommended therapies.

PRAY FOR ME Dr. Ashraf Elshehry Page 311


13) A 56 year old man attends with a 2 day history of SOB. He is previously
well except for recent malaise, night sweats and weight loss. Vital signs are PR
130/min, BP 95/60, T37.5°C, SaO2 91%. He has extensive inspiratory crepitations and
a gallop rhythm with a new systolic murmur at the apex.

What is your working diagnosis?

Infectious endocarditis. Mitral valve is most commonly affected.

What signs will you look for on further examination?

i) Splenomegaly.
ii) Roth spots (retinal haemorrhages with central clearing).
iii) Splinter haemorrhages.
iv) Anaemia.
v) Janeway lesions (red skin spots on the palms and soles).
vi) Osler’s nodes (red, painful intradermal pads in the fingers and toes).
vii) Haematuria.

What investigations are appropriate?

ECG, CXR, FBC, blood cultures X 3, ECHO, ASO test.

Commonest organism is S. viridans (found in the mouth, 40%) but others are often
implicated, e.g. S. aureus (which presents with heart failure), enterococci and fungal e.g.
candida, aspergillus. Complications include valve destruction, heart block, LVF, embolic
events, lung abscesses (right-sided disease).

Which patients are at risk from this condition?

May develop on previously normal valves as well as diseased valves or prosthetic valves.
IV drug abusers are prone to staphyloccal infection of the tricuspid valve (i.e. right-sided),
with fever and pneumonia from septic PE.

What antibiotics would be appropriate for initial management?

Benzylpenicillin and gentamicin IV.

14) A 5 week old boy is brought in by his mother. He has a history of vomiting
shortly after feeds.

What aspects of the history are important?

i) Birth; term, weight, delivery, neonatal problems.


ii) Severity; duration of vomiting, wet nappies, diarrhoea.
iii) Other; thriving, rashes, fever, alertness.

Capillary gases show:


PRAY FOR ME Dr. Ashraf Elshehry Page 312
pH 7.5
pCO2 5.3
pO2 3.1
HCO3- 32
Na 131
K 3.6
Cl 67 (range 95-105)

Comment on these results.

Hypochloraemic, hypokalaemic alkalosis.

What is the diagnosis?

Pyloric stenosis.

Congenital pyloric stenosis is the most common cause of intestinal obstruction in infancy.
It is more prevalent in males, usually a first-born aged 3-6 months. Vomiting is projectile
and bile-free. Test feed may reveal a palpable tumour. USS may also be used in diagnosis.

Vomiting leads to the characteristic metabolic picture as the metabolic alkalosis leads to
K+ loss in the urine.

The child should be kept NBM and an NG tube passed. If rehydration is necessary use ½
Normal saline with dextrose. Refer for surgery – Ramstedt’s pyloromyotomy.

Pyloric stenosis in adults results from scarring, usually secondary to a chronic DU. It
presents with vomiting, dehydration, weight loss and malnutrition. There may be an
audible succession splash.

15) HIV notes.

RNA retrovirus. Binds to CD4 receptors on T-lymphocytes, monocytes and macrophages.


These CD4 cells normally play a crucial role in co-ordinating the immune response. CD4
cell counts provide an indication of disease progression.

Presentations to the ED:

i) Respiratory: PCP pneumonia, pulmonary TB, Aspergillus, Cryptococcus.

ii) Neurological: Cryptococcus meningitis, cerebral toxoplasmosis, cerebral


lymphoma, CMV encephalitis.

iii) Eye: CMV retinitis.

iv) GI problems: Nausea, vomiting, weight loss are common and may be drug
effects. Oesophageal candida or herpes simplex infection. CMV colitis and other
causes of infectious diarrhoea e.g. cryptosporidium, Giardia, Salmonella.

PRAY FOR ME Dr. Ashraf Elshehry Page 313


v) Mucocutaneous: Oral candidiasis, seborrheic dermatisis. Oral hairy
leukoplakia, herpes infections, molluscum contagiosum, Kaposi’s sarcoma.

SAQ PAPER October 2005

Q1.

A 38 year old man presents to the ED with shortness of breath but has no chest pain. His
chest is clear and his observations and investigations are as follows:

Pulse 110 BP 120/65 Sats (on Air) 91%

ABG (On Air) pH 7.41


pCO2 3.5
pO2 8.5
HCO3 26
BXS -1

1. What four factors in this patient’s history might suggest he has had a
Pulmonary Embolus? (4 marks)

Well’s Criteria for PE:


History of Immobilisation, Recent Surgery, Cancer, Lower Limb
Paralysis, Cardio-respiratory disorders, Thrombophilia, Strong
Family History, Previous proven or current PE or DVT

2. What three investigations would exclude other causes for his symptoms
according to BTS guidelines? (3 marks)

ECG, CXR, Echocardiogram

3. The D-Dimer is raised at 330 (Normal<273) what are your next two
management steps? (2 marks)

Oxygen, Administer Tinzaparin 1.75U/kg

4. The patient deteriorates, Pulse 130, BP 80/60, what would you consider
next (1 mark)

Give Fluids, Request Urgent Echocardiogram, Consider Thrombolysis

Q2.

A seven year old boy presents with 12 hours of headache, neck stiffness, nausea and
vomiting. His temperature is 38.9 C and his GCS is 12/15.

PRAY FOR ME Dr. Ashraf Elshehry Page 314


His investigations are as follows:

Na 115
Urea 6.3
Creatinine 100

CT Head Scan

1. Give two possible radiological diagnoses? (2 marks)

Meningits, Encephalitis, Raised Intracranial Pressure

2. Why is the sodium low? (1 mark)

Syndrome of Inappropriate ADH secretion

3. What ED investigations would do for the low sodium? (5 marks)

Serum and Urine Osmolalities, Serum Cortisol & Serum ACTH,


Glucose, CXR, FBC

4. What neurological complications could result from the low sodium?


(2 marks)

Fitting
Demyelination during correction of hyponatremia well recognised

Q3.

A sixty-five year old alcoholic is found fitting in the street, he smells of alcohol and is
brought in by paramedics, still actively fitting. His airway is controlled and he is on 15
litres of oxygen/ minute via non-rebreathing mask. His BM is 2.2.

1. What is the definition of status epilepticus? (1 mark)

PRAY FOR ME Dr. Ashraf Elshehry Page 315


A prolonged seizure (usually defined as lasting longer than 30 minutes) or
a series of repeated seizures; a continuous state of seizure activity; may
occur in almost any seizure type.

2. What in a chronic alcoholic predisposes them to fitting? (4 marks)

Hypoglycaemia/Excess Alcohol or Alcohol Withdrawal


Hyponatraemia
Falls & ICH
Poor compliance to anticonvulsants
Hepatic Encephalopathy
Overdose of TCAs

3. What are your three management priorities? (3 marks)

Maintain airway/oxygenation
Glucose 50mls of 50% intravenous
Thiamine 100mg intravenous,
Lorazepam 4mg intravenous

4. If the fitting does not stop what would your management be? (2 marks)

Lorazepam 4mg intravenously


Phenytoin infusion 18mg/kg over 30minutes

Q4.

A sixty-five year old man has an itchy generalised rash. Your new SHO thinks the rash is
scabies.

1. Describe the rash. (2 marks)

Erythematous papular rash, with excoriations and evidence of burrows


Likely palm of hand

PRAY FOR ME Dr. Ashraf Elshehry Page 316


2. What is the differential? (2 marks)

Scabies, Pompholyx

3. Why is this rash itchy? (1 mark)

Dermatitic reaction to faeces of scabies mite

4. What is the treatment of choice? (1 mark)

Permethrin, Malathion

5. What two features in the history would suggest the diagnosis? (2 marks)

Nightime itchy and after hot shower, Genital Itching

6. What would you tell the patient? (2 marks)

Apply at bedtime. Wash off in the morning. Repeat in 1 week.


Treat all household contacts, Launder all bedlinen/clothes/towels

Q5.

A thirty-five year old woman who is 38 weeks pregnant complains of headache and has a
GCS of 13/15. Her observations and investigations are as follows:

Pulse=110 BP=160/95

Hb 8 Poikilocytes Seen
WCC 9.3
Plat 35
Urea 10
Cr 130
Bili 15
AST 150
ALT 600
Alk P 45

Urine: Protein +++ Blood ++

1. What are the haematological abnormalities and what is the cause? (2 marks)

Anaemia secondary to haemolysis


Thrombocytopaenia secondary to haemolysis (?increased destruction)

2. What is the diagnosis? (1 mark)

Pre-eclampsia complicated by HELLP syndrome

PRAY FOR ME Dr. Ashraf Elshehry Page 317


3. How would you control the blood pressure? (2 marks)

Labetolol intravenously initially 10mg, then infusion

4. What are you next management steps? (5 marks)

Call Anaesthetist/ Obstetrician/ Neonatologist


Insert arterial line
Intravenous magnesium 4-6g over 5-10mins (seizure prophylaxis should be
given and BP controlled before delivery [Emery Med Secrets 3rd ed].)
Monitor baby by CTG
Deliver Baby

Q6.

Blood Results:

Na 138
K 6.2
Ur 43
Cr 612
Dig 16ng/ml

1. What do you see on this ECG? (4 marks)

Atrial Flutter, Bradycardia, T wave inversion in V4-6 and inferior lead.s


Type II Heart Block

PRAY FOR ME Dr. Ashraf Elshehry Page 318


2. What are your indications for digibind? (3 marks)
Digoxin level>10ng/ml, Life Threatening Arrhythmia,
Ingestion of >10mg of digoxin

3. Other than digibind what are your other non-arrhythmia management steps
and why are you taking these steps? (3 marks)
Intravenous Fluids- to address hypotension and ARF (renally excreted)
Salbutamol Nebs, ivi Insulin & Dextrose/ Sodium Bicarbonate- to address
Hyperkalaemia
Intubate & Give Charcoal-to reduce digoxin absorption
Refer to Nephrologist and Intensivist

Q7.

A young man has been exposed to a chemical at the train station and is short of breath and
has blurred vision. He collapses after a few minutes in the department. Many other
passengers are affected.

1. Other than calling the ED consultant, what four steps would you take?
(4 marks)

Isolate Patient and Isolate ED from hospital


Instruct staff to put on Personal Protective Equipment
Call Firebrigade to set up decontamination facility outside department
Declare Hospital MAJAX

2. What are the other muscarinic features of organophosphorus poisoning?


( 3 marks)

Salivation, Constricted Pupils, Diarrhoea & Vomiting, Bradycardia

3. Which three drugs would you use? (3 marks)

Atropine (up to 3mg), Pralidoxime(30mg/kg), Diazepam (0.5mg/kg),


Neostigime (50-70mcg/kg, max 5mg)

Q8.

A thirty year old solicitor is due to be in court for a difficult case tomorrow. She is found
collapsed by her husband and then fits for 60 seconds. Her observations and ECG are
shown:

Pulse 130 BP 90/60

PRAY FOR ME Dr. Ashraf Elshehry Page 319


1. What do you see on this ECG? (2 marks)

Broad Complex Tachycardia


Extreme Axis

2. What is your diagnosis? (1 mark)

Ventricular Tachycardia due to Tricyclic Overdose

3. What antiarrhythmic would you use? Give drug name, mechanism,


and state how you would monitor its effectiveness. (3 marks)

Drug=Sodium Bicarbonate 8.4% 50ml intravenously


Mechanism=Blocks fast sodium channels and protein binding
Effectiveness=Width of QRS complex

4. What is the further management of this patient? (2 marks)

Transfer to ICU
Psychiatric Assessment when medically fit
Q9.

A twenty year old man is kite surfing and is catapulted into a shallow water landing on his
head. He was RSI’d with etidominate and suxamethonium as his GCS was less than eight,
is fully immobilized, has had 2 litres of fluid, and is now on a propofol infusion.

His observations are: Pulse 55 BP 90/60

His ABG (on 15L/min) is: pO2 38, pCO2 4, Lactate 4.5

His CT Head shows Oedema and CT Abdomen is normal.

His C-Spine radiograph is below:

PRAY FOR ME Dr. Ashraf Elshehry Page 320


1. Explain his haemodynamic abnormalities. (2 marks)

Hypotension due to anaesthetic agents and possible hypovolaemia


Bradycadia due to C-Spine injury? (Neurogenic shock due to sympathetic
chain involvement in the neck)

2. Describe the C-Spine radiograph (3 marks)

C6 Anterior vertebral body fracture, Soft Tissue Swelling, Inadequate film,


Endotracheal Tube in situ

3. Explain four neurological abnormalities would you might find in this


patient? (4 marks)
Loss of anal tone, Priapism, Loss of ankle jerks- due to spinal cord injury
Horner’s Sydrome- Intrapment of sympathetic plexus

4. Why is he hypotensive? (1 mark)

Neurogenic Shock involving sympathetic plexus


Q10.

A 13 month old child has abdominal pain, bloody diarrhoea and a rash. She recently had
an URTI. She is dehydrated clinically.

Hb 16.3 WCC 9.7 Plat 412 Coagulation Screen- Normal

Radiograph:

PRAY FOR ME Dr. Ashraf Elshehry Page 321


1. What does the radiograph show? (1 mark)

Dilated bowel and stomach

2. Give three possible diagnoses? (3 marks)

Vovulus, Intussception due to Henoch Scholein Purpura,


Intussception due to Mesenteric Adenitis

3. What advice would you give the SHO regarding fluids? (3 marks)

Fluid Resuscitate until CRT<2 seconds, using boluses of 20ml/kg


of 0.9% NaCl. Titrate fluid to ensure Urine Output>1ml/kg/hr. Close
monitoring essential.

4. How do you treat this condition? ( 3 marks)

Air Enema, Gastrograffin Enema, Laparotomy all possible treatments

Q11.

A three year old has had a cough and stridor for 12 hours but no wheeze. His pulse is 160,
Respiratory Rate is 45 and has subcostal and intercostal recession and tracheal tug.

1. Apart from croup what is your differential diagnosis ? (4 marks)

Inhaled FB, Injury to larynx, Diptheria, Bacterial tracheitis,


Epiglottis, Anaphylaxis

2. Give four features of a recognized croup score? (2 marks)

PRAY FOR ME Dr. Ashraf Elshehry Page 322


Presence of: Stridor, Respiratory Distress, Cyanosis
Reduced Consciousness, Reduced Air Entry on Auscultation

3. Name three drugs (and doses) which may be beneficial? (3 marks)

Dexamethasone 0.15mg/kg (up to 0.6mg/kg)


Adrenaline Neb 5mg
Pulmicort Neb 2mg

4. What clinical signs which would prompt admission?


(1 marks)

If hypoxaemic (low sats) or in respiratory distress (raised RR, dyspnoea)

Q12.

A thirty-five year old homosexual man presents to the ED with 3 months of a persistent
dry unproductive cough. He is accompanied by his partner who is HIV positive and
taking medications for TB. The patient’s saturations are 93% on air. This is his chest
radiograph:

1. Describe the abnormality on CXR. (1 mark)

Cavitating mass in the Left Upper Lobe

2. Apart from TB, what are the other possible diagnoses? (3 marks)

Bronchogenic Carcinoma, Kaposi Sarcoma, Pneumonia/ Lung Abscess


, Aspergilloma

3. Name three organisms which cause pulmonary disease in HIV infection

PRAY FOR ME Dr. Ashraf Elshehry Page 323


(3 marks)

Pneumocytitis carnii, Streptococcus pneumoniae, Pseudomonas,


Cryptococcus, Aspergillosis

4. What six investigations in the ED, would help with your management?
(3 marks)

Lateral CXR, ABG, FBC(CD4),U&E, Atypical Serology, Sputum for AFB,


Blood Cultures, Urinanalysis, Mantoux Test

Q13.

A sixty eight year old woman presents with sudden loss of vision in her right eye.
This is her fundus.

1. Describe the appearance on this picture. (2 marks)

Extensive intra-retinal haemorrhages, dilated veins over fundus,


scattered cotton wool spots (“stormy sunset”)

2. What six questions would you ask in the history? (3 marks)

Is it painful, Any trauma involved, Speed of onset,


Previous eye disease, Headache, Systemic disease

3. What is the diagnosis? (1 mark)

Central Retinal Vein Occlusion

4. What medical conditions are associated with this condition?


(4 marks)

PRAY FOR ME Dr. Ashraf Elshehry Page 324


Diabetes Mellitus, Ischaemic Heart Disease, Polycythemia,
Coagulopathies, Hypertension, Hyperlipidaemia

Q14.

A sixty-five year old man is brought into the ED. He is depressed and wants to die.

1. What questions in the history help to assess his suicide risk? (3 marks)

This is the SADPERSONS Scale:


Sex(male), Age(<19 or >45), Depression or Hopelessness, Previous
suicide attempt or Psychiatric care, Excessive alcohol or drug usage,
Rational Thinking Lost, Separated, divorced or widowed, Organised
attempt, No social suppott, Stated future suicidal intent

2. What principles of the Mental Health Act are relevant to the Emergency
Department (4 marks)

Mental Capacity Act (October 2005): A presumption of every adult to have


capacity, right for individuals to be supported in making their own
decisions, right to make what might be seen as an unwise or eccentric
decision, act in patient’s best intests, should be least restrictive of basic
rights and freedom

3. What three fold test can be used to determine if the patient can make a
this decision? (3 marks)

He understands and believes the information that he is told, and he is able


to retain this information and weigh up and make a balanced judgment
about the consequences of refusing treatment.

Q15.

This 22 year old man has a cut above his left eye following an assault. These are his
radiographs.
PRAY FOR ME Dr. Ashraf Elshehry Page 325
1. What abnormalities do you see on his radiographs and what is their
significance? (3 marks)

Fluid level in left maxillary antrum-suggests antral wall fracture


Left Tear drop sign- prolapse of infraorbital extrocular muscles
Left eye brow sign- Free gas in orbit due to infraorbital floor fracture

2. What four features would you look for on clinical examination of the
orbital contents? (4 marks)

Visual acuity and visual field,, Different resting pupil level as well as
inability to look up, Proptosis or Enophthalmos, Fundoscopy

3. How would you manage the patient, if he can only be seen in specialty
clinic the next day? (3 marks)

Offer analgesia, Clean/Close laceration over left eye (after checking no


FB, if suspected), Give antibiotics, and advise not to blow nose.

Q16.

This 14 month old boy fell. He weighs 10.5kg. This is the radiograph of his right side.

PRAY FOR ME Dr. Ashraf Elshehry Page 326


1. Describe the radiograph. (1 mark)

Grossly displaced supracondylar fracture of right humerus

2. Give two neurological structures which might be damaged. (2 marks)

Median and Ulnar Nerves, both motor and sensory branches

3. What drug would you give for analgesia? State dose and route (2 marks)

Intravenous Morphine 1.05mg or Intranasal diamorphine same dose

4. Name three joint injuries and their associated nerve and vascular injury.
(3 marks)
Shoulder Dislocation-Axillary nerve and brachial artery
Hip Dislocation- Sciatic Nerve Injury and AVN Femoral Head
Knee Dislocation-Peroneal Nerve Injury and Injury to Popliteal Artery
Ankle Dislocation-Peroneal and Saphenous Nerve Injury and AVN of
talus.

5. What three clinical requirements are needed to clear a C-Spine? (2 marks)

No neurological signs or symptoms, Ability to turn head>45 degree


laterally with no pain, in a fully conscious sober patient with no other
distracting injuries and age less than 65.

Short Answer Questions Practice

1. A 26-year old man attends the ED at 6 pm on a Friday evening complaining of


lower abdominal pain and dysuria. The GU clinic is closed for the next 3 days as
it a long weekend.

His urinalysis showed:

PRAY FOR ME Dr. Ashraf Elshehry Page 327


• Blood ++
• Protein +
• Glucose –ve
• Nitrites (nitrates) –ve
• Leucocytes –ve

What features in the history would suggest STD rather than UTI (2)

a.

b.

c.

d.

You decide it STD. What 3 things would you like to do before commencing empirical
therapy? (3)

a.

b.

c.

What antibiotic regime would you commence and for how long? (2)

a.

b.

2. His mother brought an 18-months old child into the ED. He has been playing
with a few 10p coins and mum feels she saw one coin in his mouth. What
questions would you like to ask in the history? (3)

a.

b.

c.

What 2 thing you include in your examination? (2)

PRAY FOR ME Dr. Ashraf Elshehry Page 328


a.

b.

Name 2 investigations you would carry out in the ED. (2)

a.

b.

Describe three sites and the vertebral levels at which foreign bodies are most likely to get
stuck in the oesophagus? (3)

Anatomical site Vertebral level

3. A 34 year old woman returns to the ED after she was seen a senior house office
(SHO) a day before with a history of malaise, arthralgia, fever and dry cough.
She was treated for viral illness. On return, she complains of breathlessness and
her oxygen saturation in the ED on high flow oxygen was 94%. Her chest x-ray
showed bibasal shadowing.

Report on the CXR provided. (2)

a.

b.

What type of non-invasive ventilation (NIV) would you use if indicated? (1)

a.

Name 3 ways you can improve compliance with NIV. (3)

a.

PRAY FOR ME Dr. Ashraf Elshehry Page 329


b.

c.

Make 2 comments on the ECG tracing from this patient. (2)

a.

b.

4. A 35 year-old man comes to the ED with bloody diarrhoea and abdominal pain.
He has a 5-year history of Crohn’s diseases. He has severe pain, looks unwell (he
has guarding) and has a temperature of 37.8ºC and tachycardic at 120/min.

Make 4 comments on this patient’s abdominal x-ray shown. (2)

a.

b.

c.

d.

List two antibiotics including their routes of administration for this patient. (2)

a.

b.

Give the names of 2 other drug treatment and the routes of administration. (2)

a.

b.

PRAY FOR ME Dr. Ashraf Elshehry Page 330


Name 4 cutaneous and musculoskeletal manifestation of inflammatory bowel disease. (4)

a.

b.

c.

d.

5. A 46-year old woman attends the ED with pins and needles in her toes,
clumsiness and double vision. She had an ‘upset stomach’ last week. She is
usually fit and well. Clinical examination revealed grade 4/5 power in her lower
limbs with absent deep tendon reflexes and diplopia on right lateral gaze. What
is the likely diagnosis? (1)

a.

List 2 differential diagnoses. (2)

a.

b.

List 3 investigations you would carry out in the ED and indicate the reason. (3)

Investigation Reason

a.

b.

c.

Give the nerve root innervations of the following reflexes. (5)

PRAY FOR ME Dr. Ashraf Elshehry Page 331


a.

b.

c.

d.

e.

6. A mother brings her 10-year-old child to the ED with lethargy and malaise for a
few weeks. She is limping and complaining of hip pain. There is gross limitation
of movement of the left hip.

Blood test results are as follow:


a. Haemoglobin = 8.4
b. Platelets = 20
c. Leucocytes = 28
d. Reticulocytes = 0.5%

Give 4 differential diagnoses. (4)

a.

b.

c.

d.

List 6 investigations you would perform in the ED. (3)

a.

b.

c.

d.

e.

PRAY FOR ME Dr. Ashraf Elshehry Page 332


f.

Comment on the haematology result provided on this patient. (3)

a.

b.

c.

7. Ambulance crew brought a 51-year old man who vomited throughout the night
to the ED. He deteriorated in the early morning with breathlessness, fever and
generally unwell. His respiratory rate was 40, pulse 130 and temperature of
38.5ºC.

Haematology result Biochemistry


Hb = 12.7 Alt = 75
W BC = 9.8 Bil = 9
Plt 150 GGT 252
MCV 106

Comment on his chest x-ray shown. (2)

a.

b.

What is the diagnosis? (2)

a.

b.

What investigations or therapeutic measures would confirm the diagnosis? (2)

PRAY FOR ME Dr. Ashraf Elshehry Page 333


a.

b.

Comment on the blood results. (2)

a.

b.

In view of the history and blood results, what is likely underlying aetiology or precipitant
of the diagnosis? (1)

a.

8. A 25-year old man comes to hospital complaining of sore (red) left eye for 2
hours. He was hammering metal yesterday and wears permanent contact lenses.
His vision is slightly reduced in the affected eye. What questions might you want
to ask in the history? (4)

a.

b.

c.

d.

What features might suggest penetrating eye injury on inspection of the anterior aspect of
the globe? (3)

a.

b.

PRAY FOR ME Dr. Ashraf Elshehry Page 334


c.

You decide this is a penetrating eye injury. What 3 drugs would you give? (3)

a.

b.

c.

9. A 9-year-old boy (25 kg weight) was brought into the ED by his mother. He ws
dehydrated and lethargic looking unwell. Give 4 specific features to suggest
dehydration. (4)

a.

b.

c.

d.

He has received resuscitation fluids and his deficit has been calculated. Calculate his
maintenance for the next 8 hours. (Show your calculations) (3)

a.

b.

c.

Give 3 other tests you would like to do in the ED. (3)

a.

PRAY FOR ME Dr. Ashraf Elshehry Page 335


b.

c.

10. A 37-year old male comes to the ED with a 3 months history of malaise, night
sweats and weight loss. His partner is hiv positive. Comment on the CXR (1).

a.

Give 4 possible diagnoses of the x-ray findings

a.

b.

c.

d.

List 3 common respiratory pathogens in HIV (3).

a.

b.

c.

List 3 items of the British Thoracic Society grade severity of pneumonia (3).

a.

b.

c.

11. A 14-year old girl comes to the ED with lower abdominal pain. She is here
without her parents and pregnancy test is positive. She is requesting a
termination. She is in a relationship with a 19-year old partner. She does not
want the parents notified. How would you assess some capacity? (3)
PRAY FOR ME Dr. Ashraf Elshehry Page 336
a.

b.

c.

What things must you ensure regarding Gillick competence. (3)

a.

b.

c.

On closer questioning she tells you she has had multiple sexual partners who are all adults
and provided by her 19-year old boyfriend. What would you do now? (4)

a.

b.

c.

d.

12. A 37-year-old woman comes to ED. She has recently been diagnosed with SLE
and has been taking regular ibuprofen for joint pains. She has come in because
her ankles, hands and eyelids are swollen and she feels very breathless. Her
exercise tolerance is greatly reduced. She was fit and well prior tot his. Her
bloods show:
Sodium 145 FBC is normal
Potassium 6.7 ABG shows metabolic acidosis
Urea 16.7 Total protein 24
Creatinine 197 LFT normal

Her ECG is shown. Comment on the ECG changes. (2)

PRAY FOR ME Dr. Ashraf Elshehry Page 337


a.

b.

What other investigations (three things in the urinalysis) would you do? (4)

a.

b.

c.

d.

Give 2 likely causes for the deterioration. (2)

a.

b.

13. A 95-year old with advanced Alzheimer’s is brought in from a nursing home. He
has a 3-day history of the rash shown below. He has a low-grade fever, has a
pulse of 100 and a BP of 102/95. Give 4 cause of the picture shown below. (4)

a.

b.

c.

d.

What single question in the history would contribute toward diagnosis? (1)

a.

PRAY FOR ME Dr. Ashraf Elshehry Page 338


What is Nikolsky’s sign? (1)

a.

What treatment would you offer? (4)

a.

b.

c.

d.

14. A 72-year old man has fallen from a horse. He is immobilised and brought into
the ED on a spinal board. His pulse is 80 and BP 102/84. He is complaining of
abdominal pain and back pain. He has a past medical history of atrial fibrillation
and hypertension. Name 2 medications other than warfarin that he may be
taking which may affect his physiological status. (2)

a.

b.

You decide to do a fast scan. Give 4 limitations of fast scan. (4)

a.

b.

c.

d.

The surgeons decide to take him to theatre – what 2 things would you do now with doses?
(2)

a.

PRAY FOR ME Dr. Ashraf Elshehry Page 339


b.

What are the complications of the medication listed in the first question? (2)

a.

b.

15. A 45-year old lady attends a week post abdominal hysterectomy for fibroids. She
is known to have hyperthyroidism, which is being treated. She attends with a
tachycardia of 140/min. Her temperature was 39.1ºC and agitated. Abdominal
examination showed well-healed surgical wound and soft to palpation. You
decide she has thyroid storm. Give 4 possible differential diagnoses, which could
account for her symptoms. (2)

a.

b.

Give 4 non-drug interventions (management) in her therapy. (4)

a.

b.

c.

d.

16. A 27 (69 kg) year old female attends ed with butterfly earring stuck in her ear
lobule. Draw a diagram of the ear and show the landmarks of the sensory
innervations to facilitate nerve blocks for removing the retained FB from the ear.
(5)

a.

b.

c.

d.

PRAY FOR ME Dr. Ashraf Elshehry Page 340


e.

Calculate the dose (mls) of 1% plain lidocaine you would use for this procedure. Show
your calculation. (2)

a.

b.

Give 3 systems affected by local anaesthetic toxicity and how they are affected. (3)

System affected Effects

17. Photograph of fingertip avulsion injury with bone exposed.

Who would you treat this injury?

a.

b.

c.

d.

What are the complications associated with this kind of injury?

a.

b.

c.

d.

PRAY FOR ME Dr. Ashraf Elshehry Page 341


18. A woman presents with lower abdominal pain. She has been seen by her GP who
diagnosed PID and started her on doxycycline and metronidazole.

How would you confirm this diagnosis? (3)

a.

b.

c.

What other tests would you do? (3)

a.

b.

c.

19. A 23-year-old man found collapsed at 6:00 am with a GCS of 13/15. His blood
gases results were as follows: pH 7.31, pCO2 and pO2 were normal. The base
excess was –6 and potassium was 7.1.

What do these blood tests indicate? (4)

a.

b.

c.

d.

What does the ECG show? (ST segment don in V2-4, tall/tented T waves) (4)

a.

b.

c.

d.

PRAY FOR ME Dr. Ashraf Elshehry Page 342


Blood tests results come back and show the following: urea 11, creatinine 220, potassium
7.9, CK 85 000, amylase 2000.

Give 3 possible causes. (3)

a.

b.

c.

What are your treatment priorities?

a.

b.

c.

d.

e.

20. A picture of a 6-year-old girl with red swollen eye.

What is the diagnosis (orbital cellulitis, pre-septal cellulitis, erysipelas)?

a.

Name 3 complications associated with this condition.

a.

b.

c.

Name 2 possible causative agents.

a.

b.

PRAY FOR ME Dr. Ashraf Elshehry Page 343


What is the possible source of infection in this age group?

a.

b.

c.

d.

1) A 70 year old gentleman is brought to the ED having collapsed outside M&S.


He is very apologetic about wasting your time and says he feels fine, doesn’t know
what all the fuss is about and would like to go home.
On further questioning he says he has had a few dizzy spells over the last
couple of months but always with a full recovery.

This is his ECG on admission.

Give 3 abnormalities.

1° heart block
RBBB (not a great example)
Left anterior hemiblock (LAD)

What is this group of features also known as?

Trifascicular block

PRAY FOR ME Dr. Ashraf Elshehry Page 344


This may show:
1° heart block+RBBB+left anterior hemiblock
1° heart block and LBBB

Whilst in the department he complains of feeling dizzy and unwell. A 2nd ECG is
recorded. What does it show?

Complete heart block

What is the cause and underlying pathology?

Fibrosis of the conducting system pathways.

Block of the right bundle branch and either fascicle is bifascicular block. If this is
combined with 1st degree AV block then it is called trifascicular block.

Give three treatment options.

If the patient is in CHB and bradyarrhythmia is causing severe haemodynamic


compromise then temporary pacing is the best option. Atropine 1mg IV bolus repeated if
necessary up to 3mg. Isoprenaline 0.2mg IV if there is a delay in pacing and the patient
remains unstable.

PRAY FOR ME Dr. Ashraf Elshehry Page 345


2) A 22 year old Caucasian woman presents acutely unwell. She has been
previously fit and well and there is no history of IV drug abuse. On examination
she is pale, dyspnoeic and tachycardic rate 130, no organomegaly.

Blood results show:

Hb 5.2 Bilirubin 74
WCC 3.9 ALT 25
Plt 195 AST 29
MCV 94 Alk phos 235

U+Es and clotting were normal

What is the diagnosis?

Haemolytic anaemia (normal MCV, elevated bilirubin). Causes may be congenital (e.g.
G-6-PD deficiency, sickle cell disease, hereditary spherocytosis) or acquired. In this
patient it is likely to be an acquired cause as hereditary disorders usually present early in
life. Splenomegaly would have suggested an underlying systemic disorder such as CLL or
SLE.

Give 3 further investigations you would like to perform.

Coomb’s test (detects circulating antibodies against RBC’s – presence may indicate
autoimmune or drug-induced haemolytic anaemia).
Blood film
CXR

Give 2 further features in the history that you would like to know.

Foreign travel
Drug history

Give some possible causes.

i) Drugs e.g. penicillin, methyldopa.

ii) Infections; viral, bacterial (mycoplasma) or protozoal (malaria).

iii) Fava beans may precipitate haemolysis in patients with G-6PD deficiency.

iv) Idiopathic autoimmune disorder.

v) Underlying systemic disorder e.g. CLL, SLE.

PRAY FOR ME Dr. Ashraf Elshehry Page 346


3) A 72 year old lady is brought into the ED by concerned relatives. She has
recently become increasingly confused and agitated and today is very unwell. The only
medication that she is taking is thyroxine. You are concerned that she may be
suffering a thyroid storm.

This is a rare condition; look for evidence of thyroid disease e.g. goitre, exopthalmos. It
may be precipitated by inappropriate cessation of anti-thyroid therapy, infection, trauma,
DKA, iodine administration, recent surgery or thyroid hormone overdose.

What other clinical signs would help confirm your diagnosis?

Onset may be sudden. Fever, CVS and CNS symptoms are common;

CVS
Tachycardia, palpitations
AF
Cardiac failure
CNS
Agitation, anxiety
Tremor
Delirium, coma
Other
Sweating
Abdominal pain
Vomiting

What would be your first investigation?

? TFTs

TFTs do not discriminate between simple thyrotoxicosis and thyroid crisis but an urgent
TSH or free T4 may be useful if diagnosis is unclear.
Funny question – need FBC, U&Es, BM (may be low), Ca (hypercalcaemia may occur),
infection screen, CXR, ECG.

What 3 medications would you use as first-line treatment?

1) Steroids – hydrocortisone 200mg IV


2) Propranolol 1mg IV inhibits peripheral T4→T3 conversion. Remember that a
history of cardiac failure (rate-dependent failure not included) or asthma may be CI for
β-blockade; guanethidine may also be used.
3) Antithyroid drug; propylthiouracil is more effective than carbimazole.

Treat the precipitating factor if possible. Fluid balance is important and CVP monitoring
is usually necessary. Monitor BM. Sedation should be given if necessary. Broad spectrum
antibiotics are indicated if infection is suspected. Do not give aspirin as this may displace

PRAY FOR ME Dr. Ashraf Elshehry Page 347


thyroxine from thyroid binding globulin. Treat fever with paracetamol. Iodine will be
given on the ITU once antithyroid medication has commenced effect.

Differential diagnosis includes neuroleptic malignant syndrome, septic shock,


anticholinergic or sympathomimetic overdose, withdrawal states.

Another thyroid storm question was given:

A 35 year old woman is admitted confused, pyrexial and vomiting. Her flat-mate reports
that she has been unwell for the last three months and has lost weight. Three days
previously she was bed-bound with a severe cold. Her brother is diabetic.

On examination she is disorientated, pyrexial and tachycardic with an irregular pulse.


There is no neck stiffness and no focal neurological signs, no obvious focus of infection
Blood results were unremarkable, BM 5.2.

Clues in this question;

History
Weight loss
Recent viral illness
Family history of autoimmune disease (presumably she has Graves’ disease)

Examination
AF
Confusion
Pyrexial

4) A 35 year old male solicitor attends with a headache. He was diagnosed as


being hypertensive a year ago and despite drug treatment including β-blockers,
calcium antagonists and an ACE inhibitor his blood pressure remains elevated. He
drinks 4 pints of beer/day and smokes 2 cigars every evening. In the department,
supine blood pressure is 210/110mmHg. Fundal examination shows grade 3
retinopathy (flame haemorrhages and cotton wool exudates). The rest of the
examination is normal.

Investigations:
Sodium 148
PRAY FOR ME Dr. Ashraf Elshehry Page 348
Potassium 3.0
Bicarbonate 32
Urea 4
Glucose 4
Urinalysis NAD

What is the likely diagnosis?

Conn’s syndrome (primary hyperaldosteronism). High aldosterone levels increase renal


excretion of potassium, but this is not diagnostic for this disease. Essential hypertension
which is being treated with diuretics may mimic this.

High blood pressure is the main, and often only, symptom.

Excess secretion of aldosterone may be caused by an adrenal adenoma or adrenal


hyperplasia.

What further investigations are indicated?

CT abdomen.
Serum aldosterone (elevated) and renin (low or undetectable).
However these are specialist investigations as they need to be done under controlled
conditions. Refer to an endocrinologist for appropriate treatment – surgical adrenalectomy
(adenoma) or spironolactone (hyperplasia).

I gave the answer malignant hypertension (wrong) which presents with hypertensive
encephalopathy (headache, nausea, vomiting, visual symptoms, confusion, fits) and needs
careful blood pressure management after consultation with physicians. Cautious reduction
of the BP is necessary to avoid complication such as CVA and AMI; avoid sublingual
nifedipine.

Other conditions that may present with severe hypertension include CVA, renal artery
stenosis, CREST syndrome, renal failure, phaeochromocytoma, Cushing’s syndrome.

Phaeochromocytomas are catecholamine-producing tumours of the adrenal glands. They


may present with hypertension, hypertensive crises, cardiac arrhythmias, anxiety attacks,
tremor, sweating and cold extremities. Careful rehydration is necessary before α-blockade
and referral. Send urinary and plasma catecholamines.

5) A 64 year old Somali woman has a 6 month history of significant weight loss
and episodic colicky abdominal pain not associated with meals or posture and no change
in bowel habit.

Investigations:

Sodium 125
Potassium 6.0
Urea 14
Calcium 2.76 (2.2-2.6)
Glucose 3.3

PRAY FOR ME Dr. Ashraf Elshehry Page 349


What is the diagnosis?

Addison’s disease.

This frequently has an insidious onset with weakness, apathy and anorexia in addition to
the other symptoms described. 80% of cases in the UK are idiopathic (autoimmune); other
causes include TB, metastatic disease, drugs (e.g. rifampicin, phenytoin), adrenal
haemorrhage 2° anticoagulation and sepsis. The biochemical picture is typical. Treatment
consists of identifying the underlying cause and steroid replacement therapy.

Chronic features of Addison’s disease include areas of vitiligo and hyperpigmentation in


the palmar creases, buccal mucosae and axillae.

Addisonian crisis (acute adrenal cortical insufficiency) is rare and usually precipitated by
sudden steroid withdrawal. Other causes include trauma, infection or stress. Main features
are shock, confusion and hypoglycaemia.

Treatment of crisis involves

i) fluid resuscitation
ii) check BM and treat if hypoglycaemic
iii) take blood for cortisol and ACTH
iv) hydrocortisone 100mg IV
v) infection screen and IVAB if suspected infection

6) A 45 year old smoker complains of tiredness and weakness.

Investigations:

Blood pressure 180/110mmHg


Sodium 140
Potassium 2.8
Bicarbonate 32
Urea 5
Glucose 12

What is the probable diagnosis?

Cushing’s syndrome, caused by excess glucocorticoids.

The commonest cause is the use of steroid medications e.g. for asthma.

Classical signs and symptoms;

Moon face
Central obesity
Abdominal striae
Thinning skin
PRAY FOR ME Dr. Ashraf Elshehry Page 350
Weight gain
Osteoporosis
Diabetes
Hypertension
Infections esp. skin

In this case, the likely cause is an ACTH-producing oat cell lung carcinoma. Rarely
ACTH-producing tumours in the pituitary can cause Cushing’s.

What investigations would you perform to confirm your diagnosis?

CXR
Serum and urine cortisol
Dexamethasone suppression test

7) What is the diagnosis?

Acromegaly. Left 3rd nerve palsy due to tumour invasion of left cavernous sinus. Caused
by excess production of growth hormone by the pituitary, usually by a benign tumour.

Characteristic features:

Large hands and feet


Enlargment of facial bones
Prognathism
Macroglossia
Widening of interdental spaces
Skin tags
Weakness, fatigue
Carpal tunnel syndrome
Joint pains
Diabetes
Hypertension

PRAY FOR ME Dr. Ashraf Elshehry Page 351


Treatment by pituitary surgery gives 80% cure rate.

8) What endocrine problem would cause this?

Pretibial myxoedema. Can be present in either Graves’ disease or hypothyroidism. It is an


infiltrative dermopathy that most frequently appears symmetrically over the anterior tibia
and dorsum of feet. Can present in nodular or diffuse forms. It is likely that thyroid
hormones affect the synthesis and catabolism of mucopolysaccharides and collagen by
dermal fibroblasts. Treatment with steroids and/or immunoglobulins may give some
relief.

9) What is the diagnosis?

PRAY FOR ME Dr. Ashraf Elshehry Page 352


Proptosis due to hyperthyroidism – Graves’ disease. Other ophthalmic manifestations are
lid retraction, strabismus and optic nerve compression.

Graves’ is an autoimmune disorder and the soft tissues around the eyes are infiltrated by
lymphocytes. It is the most common type of hyperthyroidism and is commonest in young
people. It is also known as diffuse toxic goitre. TSH will be low and T4 high.

Other causes of hyperthyroidism are toxic nodular goitre (in elderly, not usually
associated with eye problems) and De Quervain’s thyroiditis (inflammation of the thyroid
gland due to a viral infection).

In the hyperthyroid patient look for a thin patient, restlessness, tachycardia, hot sweaty
skin, fine tremor and brisk reflexes.

10) This patient presented with a rash following an URTI.

What is the likely diagnosis?

Henoch-Schonlein purpura.

What is the aetiology of this condition?

IgA-mediated vasculitis of small blood vessels. The exact causative mechanism is


unknown but it usually follows bacterial (particularly strep.) or viral infection.

What is the likely sex and age of the patient?

M:F 2:1
4-11 years

PRAY FOR ME Dr. Ashraf Elshehry Page 353


What is the prognosis?

Generally good but need follow up because of the possibility of delayed renal
involvement and development of nephrotic syndrome which indicates severe disease.

What other symptoms may the patient present with?

i) Malaise, low grade fever


ii) Hepatosplenomegaly
iii) Lymphadenopathy
iv) Colicky abdominal pain (may develop bloody diarrhoea, intussusception).
v) Arthritis/ arthralgia
vi) Testicular pain and/ or swelling

Remember other causes of purpuric rashes in children:

i) meningococcaemia
ii) HSP
iii) thrombocytopenia 2° ITP, leukaemia, aplastic anaemia
iv) trauma, coughing or retching
11) This young woman tripped down a flight of stairs landing on her left
shoulder. She complains of localised pain around the left SCJ. Her extremities are
neurovascularly intact.

What is the diagnosis?

Anterior SCJ dislocation.

How common is this condition and how is it diagnosed?

It is rare. Primarily a clinical diagnosis but AP and cephalic tilt X-rays may be obtained.
The optimal mode of imaging is CT scan.

What is the treatment?

Analgesia, sling, refer T+O.

What would the main risk to the patient if this were to occur in the reverse direction?

PRAY FOR ME Dr. Ashraf Elshehry Page 354


Pressure on the trachea can cause airway obstruction.

12) This boy’s rash, first noticed yesterday, has worsened overnight. It had started
on his hands, including his palms, and quickly spread to his arms and head. He had
had URTI symptoms for 3 days. The lesions are not pruritic but the boy complains of a
‘burning’ sensation in the affected area. He has now developed blisters on his oral
mucosa.

What is the diagnosis?

Typical target lesions of erythema multiforme. This is a localised form of vasculitis.


Attacks may be idiopathic, triggered by drugs (e.g. sulphonamides, phenytoin,
barbiturates), infection (viral (esp. Herpes simplex) or Mycoplasma pneumonia), or
related to malignancy. It can occur at any age.

What is the eponymous name for the severe form of this condition?

Stevens-Johnson syndrome. There may be oral, ocular and genital lesions. This form
carries a significant morbidity and mortality; complications may include renal and
respiratory involvement.

Management is symptomatic. Steroids may reduce the severity of the attack. The
underlying cause should be treated where possible.

PRAY FOR ME Dr. Ashraf Elshehry Page 355


SAQs October 2006

1. CXR of pulmonary oedema Describe


Explain Starlings curve/pathophysiology
Treatment
What triggers the release of BNP and what is
its use?

2. Lady with COPD What type of NIV would you use? BiPap
guidelines
Explain to a medical student the principles of
NIV
Look at CXR – what has happened now? -
pneumothorax
How would you manage her now?

3. AXR Describe XR - ?thumb printing / dilated


large bowel/ prosthetic right hip. SBO –
biochemistry/fluids
Causes of

4. Hx of travellers diarrhoea Main cause


What in history would you ask about
What treatments could you consider in this
patient?

5. Elderly man with ARF on diuretics Suggest causes of the blood test results
What would you ask in history?
What else would you do?

6. Neonatal jaundice What would you investigate for in every


patient?
5 causes of neonatal jaundice
6 treatments you would commence in ED

7. 16 yr old - paracetamol OD Suggest 3 reason for being high risk


2 things you should do next
What in the hx suggests a high risk patient?

8. Cut on lip Lac - How would you anaesthetise it?


Tetanus – what makes a wound tet prone?
In a gentleman who has had one booster only
12 years ago what would you do?

9. Bloods suggestive of HONC Diagnosis/ fluids


Calculate the osmolarity
2 treatments
One other treatment

10. Eye – orbital cellulitis On examination what would you look for?

PRAY FOR ME Dr. Ashraf Elshehry Page 356


11. Epistaxis 3 groups or types of drugs you would ask
about
Explain to medical student how to insert a
nasal tampon
If continues bleeding what would you do?
How would you do it?

12. Gout Physiology of gout and pseudogout


What foods/drugs should be avoided?
What dietary advice should be given?
2 treatments/groups of drugs used in
management of gout and explain their
mechanisms of action.

13. Rheumatic fever What are the criteria for having RF? Post-
strep joint pain

14. 36 yr old painful periods – not preg She has pain bilaterally on pv – causes?
Nerve supply of ovaries/uterine fundus

15. Photograph of frostbite digit Hypothermia/Management of frostbite

16. Trauma – photo of mangled car, 5 other things you would like to know
patient had seatbelt on before treating patient ie anyone else in car,
was he walking at scene. Chest trauma.

17. Violent patient Puerperal psychosis. Organic causes. NICE


Guidelines/recommendations in management
of this patient
NICE suggestions for rapid tranquilisation

18. First trimester PV bleeding GMC question – recommendations for


intimate examination – chaperone guidelines
UTI in pregnancy

19. Elderly patient with vertigo Non-central causes of vertigo


Tests to differentiate between central and
peripheral causes
Drugs and families of drugs you would use
to treat vertigo

20. Urinary retention Management with antibiotic guidelines

Question on Sepsis/SIRS Definitions

1) Question about APLS fitting child protocol.

PRAY FOR ME Dr. Ashraf Elshehry Page 357


2) Question about strokes – remember blood sugar control improves outcome.
Know RCP stroke guidelines.

3) Typhoid question.

Occurs throughout the world, especially where hygiene is inadequate.

Organism responsible – Salmonella typhi.

Spread by faecal/ oral route.

Along with malaria, first disease to consider if fever develops after visit to affected areas.
Incubation period 8-14 days.

Symptoms

1st phase - fever, headache, sweating, dry cough, myalgia, arthralgia, abdominal
discomfort, anorexia, constipation. Children are prone to diarrhoea. There may be
splenomegaly.

‘Rose spots’ are pink macular spots on the lower chest or upper abdomen which blanch
on pressure.

2nd phase - fever, severe diarrhoea (may be bloody), unwell++, confusion.

Pneumonia and intestinal perforation are possible complications.

Treatment

Isolate and barrier nurse. Ciprofloxacin. Careful fluid balance. Notify communicable
disease control.

PRAY FOR ME Dr. Ashraf Elshehry Page 358


4) Right 3rd nerve palsy question

Note lid has been lifted implying ptosis.

Right eye has a divergent strabismus and pupil is dilated.

Differential diagnosis

Space occupying lesions, after surgery (e.g. for pituitary lesions), aneurysms of the
posterior communicating artery, infections e.g. meningitis, encephalitis, herpes, syphilis.

Worthwhile mentioning;

Isolated 4th nerve palsy (superior oblique moves eye downward) → diplopia on
downward gaze.

Isolated 6th nerve palsy (lateral rectus moves eye laterally) → failure of lateral movement
with diplopia on looking at the affected side.

PRAY FOR ME Dr. Ashraf Elshehry Page 359


5) Question about flexor tendon zones in the hand

Zone II is the ‘No Man’s Land’ where the flexor tendons are located in a narrow
fibro-osseous tunnel; injuries to the flexors in this region have a worse prognosis
as the finger tends to become stiff as adhesions form.

6) Question about zones of neck when describing penetrating neck


trauma.

PRAY FOR ME Dr. Ashraf Elshehry Page 360


7) Question about torsades de pointes

Also known as polymorphous ventricular tachycardia. May present as recurrent


syncope or dizziness.

Important to recognise as there are a number of potentially reversible causes,


e.g. hypokalaemia, hypocalcaemia, hypomagnesaemia, CHB, congenital long QT
interval or drug related e.g. sotalol, tricyclics, antihistamines.

Torsades may be self-limiting but may progress to VF.

Treat by:
i) Correcting underlying cause if possible.
ii) IV magnesium sulphate 2g over 10 min.
iii) May require temporary overdrive pacing.

8) Question about erythema multiforme

This is a form of vasculitis; the characteristic target lesions are symmetrically


distributed, particularly over the extensor surfaces of the limbs, hands and feet.

Causes include infection (herpes, mycoplasma) and drugs (sulphonamides, barbiturates).


Treat underlying cause. Steroids may help symptoms in severe cases.

The severe bullous form with involvement of mucous membranes, Stevens-Johnson


syndrome, may occur. There is pyrexia and malaise with oral, ocular and genital lesions.

PRAY FOR ME Dr. Ashraf Elshehry Page 361


9) Question featuring CT head

Shows acute right sided subdural haematoma.

What groups of patients are prone to this?

Alcoholics, the elderly, patients on anticoagulants.

List 5 features which may be present.

i) Headache
ii) Fluctuating GCS
iii) Confusion
iv) Memory loss
v) Focal neurological deficit

9) 58 year old female attends ED complaining of increasing weakness in right


hand.

PRAY FOR ME Dr. Ashraf Elshehry Page 362


This is an ulnar nerve palsy with wasting of the first dorsal interossei (ulnar n. supplies
all interossei but 1st is almost always the first to become noticeably affected) and ulnar
claw appearance of hand. The claw hand appearance is due to paralysis of the intrinsic
hand muscles (lumbricals) which normally flex the MCPJs and extend the PIPJs.
Unopposed action of the long extensors pulls the MCPJs into hyperextension and the long
flexors pull the PIPJs into flexion.

Examination of ulnar nerve

Supplies:

Flexor carpi ulnaris


Half of FDP
Hypothenar muscles
Interossei
Two medial lumbricals
Adductor pollicis

Look for:

Claw hand, wasting of interossei, hypothenar wasting.


Scars/ deformity around elbow, wrist and hand.
Trophic changes.

Motor:

Test flexor carpi ulnaris.


Test FDP in little finger.
Interossei: hold sheet of paper between ring and little fingers and withdraw.
First interosseus: attempt to adduct index against resistance, look and feel for
contraction in 1st webspace.
Froment’s test (adductor pollicis).

Sensation:

Over ulnar 1½ fingers and ulnar border of hand.

State 2 causes of this condition.

At wrist – lacerations, ganglia.


At elbow – occupational secondary to excessive leaning.

10) This 64 year old male presented to the ED after an increasing number of falls.

PRAY FOR ME Dr. Ashraf Elshehry Page 363


Stooped posture/ shuffling gait typical of Parkinson’s.

Name 3 features of this condition.

Tremor, rigidity, bradykinesia.

Also fixed facial expression, speech problems, poor balance, dementia

List 3 drugs used as treatment.

Levodopa, dopamine agonists (e.g. bromocriptine), selegiline

11) This 64 year old farmer complains of burning pain in both hands.

What feature is seen?

Wasting of thenar eminence muscles

What is the likely diagnosis?

PRAY FOR ME Dr. Ashraf Elshehry Page 364


Carpal tunnel syndrome secondary to arthritis.

How can the diagnosis be confirmed?

Nerve conduction studies.

List 6 possible underlying causes.

i) Arthritis
ii) Pregnancy
iii) Hypothyroidism
iv) Acromegaly
v) Trauma
vi) DM

Examination of median nerve

Supplies:

Muscles in forearm (through anterior interosseous branch)


FPL
½ of FDP
FDS
FCR
Palmaris longus
Pronator quadratus and teres

Muscles in hand
L lumbricals (lateral two)
O opponens pollicis
A abductor pollicis brevis
F flexor pollicis brevis

Look for:

Index finger held in extension (Benediction attitude).


Wasting of thenar muscles, deformity/ scars around wrist and elbow.
Trophic changes.

Motor:

Test APB by asking patient to lift thumb off flat surface against resistance.
Test FCR.
Test FPL and FDP in the index by flexing joint against resistance.
Test pronator quadratus by asking patient to pronate arm against resistance with elbow
extended.

Sensation:

In median nerve distribution.

PRAY FOR ME Dr. Ashraf Elshehry Page 365


12) A 45 year old man is referred to the ED by his GP. He had a two day history
of progressive right-sided facial weakness and lower limb weakness. He had been
previously fit apart from a mild URTI one week previously. Examination revealed a
right-sided LMN CN VII palsy, generalised weakness of the legs with reflexes
diminished in the upper limbs and absent in the lower limbs. No other findings of
note.

What is the diagnosis?

Guillan-Barré syndrome

What are the diagnostic features?

i) Ascending, usually symmetrical, progressive LMN weakness.


ii) Sensory loss is not usually profound but paraesthesiae may precede
weakness. If there is a sensory level then spinal cord compression should be the
diagnosis until proved otherwise.
iii) Reflexes are diminished.
iv) Autonomic dysfunction is common.
v) Ventilatory failure as disease progresses.
vi) Often involves the cranial nerves (7th commonest).

How would you investigate this patient?

GBS is a diagnosis of exclusion with an extensive differential (see below). The


management of the patient with GBS is that of any patient with neuromuscular failure but
specific measures include;

i) Autonomic instability is a common feature so pulse/BP/ECG monitoring is


essential.
ii) Check spirometry and ABGs; early ventilatory support may be required
iii) CT head if diagnosis unclear.
iv) CSF analysis may be required – CSF protein characteristically rises and
peaks at 4-6 weeks but may be normal initially.
v) Specific treatment with immunoglobulin.

What is the mortality of this condition?

Associated mortality is 10%. Poor prognostic features on presentation include rapid onset,
requirement for ventilation, age>40. Grading system from I (able to run) to V (ventilated).

GBS probably represents an immune-mediated attack on peripheral nerves.

Differential diagnosis of acute generalized weakness:

Myasthenia gravis Tetanus


Multiple sclerosis Spinal cord compression
Alcoholic myopathy Botulism
Poisoning (lead, organophosphates) Hypokalaemia

PRAY FOR ME Dr. Ashraf Elshehry Page 366


- CXR -LVF

Q- what are the 4 xray findings

Q What are 3 drug Rx, other than o2 with dosages and routes

Q decsribe/ draw starlings curve and state how the above condition will effect it.

Q BNP- what stimulates its production and what is its effect.

2-RTA- picture of car- restrained passenger hit steering wheel no airbags sats 93% rr
up, tachycardic bp 100/80

Q 4 pieces of info required? ( ? from scene or en route or from pt - not clear)

Q What diagnosis do you suspect- tension pneumo

Q What signs would support this diagnosis

3-torso rash- suspected Rh Fever, has joint pain and tenderness, temp PR
prolonged on ECG plus history of sore throat

what are the requirements of duckett jones criteria

What signs support this- erythema marginatum, athralgia, prolonged PR,


temp

Q pathophysiology- group a beta haem strep- immune cross reaction

4 treatments- bed rest until CRP normal, aspirin, joint splintage and
benzylpenicillin

4- African neaonate jaundiced and lethargic

Q causes- ? 4, haemolytic disease newborn, Torch, physiological, liver disease

Q invesigations in A+E ? 6 split bili, fbc, U+E, LFT, coag, BM, sickle cell/ thalasaemia-
prob first 6

Q What replacements will you give in A+E- Glucose, fluids, Vit k/ abidec? ?FFP

5 COPD exacerbation- apyrexial I think- sats 88% but hypercapnic- 10.5 o2 8.6 rr
28, PH 7.25

Q 2 immediate drug treaments- nebs ? on oxygen sats 86%

Q you decide to give NIV- which type

Q explain to a medical student how your NIV works- 4 points

Q patient becomes more unwell- CXR large >2cm pnemothorax 4 abnormalities on

PRAY FOR ME Dr. Ashraf Elshehry Page 367


Xray

Q what would you do (2)- stop NIV and chest drain (BTS guidelines)

6 45 year old female- abdo pain with period


Q- what is GMC guidance on intimate examinations ?

Q what is the dermatomes for cervix, perineum and vag: ovaries: body of
uterus ( your having a laugh)

Q - what is the likely differential ?3/4

7 Nice guidleines on violent patients- violent, agitated 35 year old female


plus partner and baby, has salbutamol on her

Q what do you do before seeing patient 4 marks

Q What are the likely organic causes in this pt- aminophylline od, sepsis,
drugs recreational ,withdrawl, hpoxia- (PE/ Pneumo-asthma), SLE vasculitis

8 Man acute retention urine, DM on diuretics and ACE

Q what are key points to urological history- 4

Q has raised U, Cr and K- explain possibilities why 3

Q decide to catherise what are the indications for antibiotics

9 Preg young female ? UTI

Q why do pregnant females get more UTIs 2 points

Q what findings would make you suspect pyelonephritis 4

Q what are the possible complications of pyelonephritis in pregnancy.4

10 Travellers diarrhoea

Q commonest cause- e coli

Q 2 others Shigella salmonella campylobacter- and others

Q what features in a patient with diarrhoea would make you concerned ( I read this
as not necesaarily a traveller)- bleeding/ constipation/ weight loss/ high
fever/ guarding

11 Picture- swollen closed red eye.

Q 3 predisposing factors- I put sinusitis URTI, and local trauma- but probalby insect
bite or something

PRAY FOR ME Dr. Ashraf Elshehry Page 368


Q Diff diagnosis- orbital/ periorbital cellulitis

Q 4 marks- differentiating features- proptosis, painful eye movements, tender eye,


limied rom eyes ? chemosis

Q organisms 3 - strep/ staph/ h infl

12 Xray small bowel obstruction- history- female 40's no appx vomiting 3 days no
flatus-

Q 2 abnormalities on xray- dilated loop small bowel, no gas in colon ( my answers)

Q Na low normal, K low, urea up, low normal chloride. 3 marks explain electrolyte
disturbance.

Q what are the possible causes of this presentation ( does this mean just the clinical
presentation or including investigations- I took it as just the presentation) - volvulus,
tumour, ileus, adhesions.

13 Collapse cardiac arrest and hypothermic

Q 4 things needed for resus other than standard resus rx- rectal therm- bear hugger,
warm fluids , wooly hat, overhead heater.

Q How would your resus protocol change 4

Q picture- frostbite hand- 2 things you would do 2 things you shouldn't

14 Confused old lady - bloods show HONK

Q diagnosis

Q causes of this presentation ie confused old lady

Q Whtat fluid resusitation and why

15Q pathophysiology of gout v's pseudo gout

Q drugs that can cause it 2

Q dietary advise and why

16 SIRS ill patient with physiology- which of the numbers infer SIRS
Q which infer septic shock

can't remeber rest of that one

17 paracetamol od

Q features suggesting serious From pathological view of OD

PRAY FOR ME Dr. Ashraf Elshehry Page 369


Q treatment if arrive 30mins

Q conditions that would make you treat on high risk line

18 sudden onset vertigo and vomiting

Q differential 2

Q how would you rule out central causes

Q once know it is not central differentiate ( i put BPV and labyrinthitis)- asks for test
name

and how-

Q 2 drug treaments and the drug group (4)

19 Cut lip in garden on spade

Q how would you anaesthetise and what would you use.

Qwhat makes a wound high tetanus risk

Q if only one tet jab 12 years ago what is the treatment

Q allourinol and colchicine mechanisms of action

A 2 year old child is brought into the ED at midnight with sudden onset os barking seal
like cough. He looks, and sounds like he has croup.

Name 4 drugs can be used for the treatment of croup? (2)


Budesonide (pulmicort) neb 1mg
Prednisolone 2mg/kg
Dexamethasone 0.15-0.6mg/kg
Epinephirine nebuliser 5 mls/1:1000

Which 5 signs make up the Westley score?(5)

Stridor none (0) when agitated (1) at rest (2)


Recession none (0) mild (1) moderate(2) severe (3)
Cyanosis none (0) when agitated (4) at rest (5)
Level of consciousness normal (0) Disoriented (5)
Air entry normal (0) moderate (1) severe (2)

Mild score 0-2 moderate 3-5

PRAY FOR ME Dr. Ashraf Elshehry Page 370


In a child presenting with acute epiglottitis, outline your management (2)

Disturb as little as possible. Do not assess airway if breathing.

Call for help: senior anaesthetist and Senior ENT surgein able to perform emergency
tracheostomy

Try humidified O2

Gaseous induction in controlled environment

-1 for any of Bloods, airway assessment, IV access, lateral neck x ray.

In airway obstruction, at what age would you consider surgical cricothyroidotomy? (1)

Age over 12 years

Seriously ill child

A 4 month old child is brought into your department. He has benn taking decreased feeds
for the past 48 hrs and now his Mum tells you he has stopped responding to her and is not
feeding at all. His breathing is “rattly” . He is maintaining his own airway. In your clinical
assessment (eg without monitoring)

Give 4 measures of the effort of breathing (2 marks)

Resp rate, recession, Grunting, accessory muscle use, flaring of alae nasae

Give 2 measures of the efficacy of breathing (1 marks)

Heart rate, skin colour, mental status

Give two measures of assessing circulatory status in this child (with normal values where
appropriate) (2 marks)

Cap refill <2 s


HR 110-160

The child has a small , dry cough and you can hear fine crepitations in all lung fields. The
child is clinically dehydrated.

What is the likely diagnosis, with causative organism (2 marks)

Bronchiolitis, RSV

PRAY FOR ME Dr. Ashraf Elshehry Page 371


Give three management steps you will take in the ED (3 marks)

Oxygen
IV fluids
admission

A 28 year old male attends the ED. He has been drinking heavily on a stag weekend. He
comes to hospital on the Monday morning with central abdominal pain, radiating to his
back.
He has been seen by an SHO who has given adequate analgesia and commenced an IVI.
He has done a FBC, U & E and an Amylase. He has also requested an erect CXR which is
normal. The SHO is due to do her MRCS part 1 next week and sees this as an opportunity
for revision. The amylase is raised at 1200.
She asks you about the Glasgow scoring system for severity in pancreatitis
Give her six points on the score. (3)
Age > 55
WCC > 15
Fasting glucose >10mmol/l
Urea > 16mmol/l
PO2 < 7.9kPA
Calcium < 2mmol/l
Albumin <32g/l
LDH >600
AST> 100
3 or more of the above constitutes
severe disease

She then asks what are main causes of pancreatitis. She can remember “scorpion bites, but
is sure that this is not the case in this patient.
Give 6 causes. (3)
Gallstones
Ethanol
Trauma
Steroids
Mumps, EBV, Coxackie
Autoimmune e.g. PAN
Scorpion venom
Hyperlipidaemia, hypercalcaemia, hypothermia
ERCP, embolism
Drugs e.g. thiazides, azathioprine,
Pancreatic cancer

What are the main early complications of pancreatitis? (2)


Acute renal failure
DIC
ARDS
Hypocalcaemia

PRAY FOR ME Dr. Ashraf Elshehry Page 372


Shock
½ each
What are the late complications? (2)
Pancreatic pseudocyst
Pancreatic abscess
Hyperglycaemia (May need insulin)
Thrombosis of vessels e.g SMA, Gastroduodenal, splenic
Bleeding: erosion of splenic artery (1 mark each)

Swollen eye

A 6 yr old girl present to your department complaining of a swollen eye:

Give three possible diagnoses (2marks)

Orbital cellulitis, pre-septal cellulites, erysipelas

How would you differentiate clinically between these? (2 marks)

Preseptal cellulitis: white eye, normal eye movts, no pain on movt, no proptosis

Give 3 possible complications (3 marks)

Cavernous sinus thrombosis, brain abscess, visual loss (20% in pre antibiotic era)

Give 2 possible causative organisms (2 marks)

Strep pneumoniae, Staph aureus, H influenzae

What is a possible source of infection at this age ? (1 mark)

Sinusitis

PRAY FOR ME Dr. Ashraf Elshehry Page 373


http://www.emedicine.com/oph/topic205.htm
Asked Nov 2002

Elderly o/d

A 67 yr old man presents to your department. He has been depressed for some time and
has been discovered at home having taken an overdose of theophylline. He has a history
of ischaemic heart disease and COPD.

What clinical features might you expect in theophylline overdose? 4 marks

Mild/moderate toxicity:
Nausea, vomiting (which is often severe and resistant to standard antiemetics,
epigastric pain, haematemesis and pancreatitis. Tachycardia, tremor, agitation,
restlessness, confusion and hallucinations may occur.

Severe toxicity:
Convulsions, hypotension and cardiac arrhythmias such as supraventricular or
ventricular tachycardia or ventricular fibrillation may occur. Coma may develop in
severe cases.

What metabolic features might you expect? 2 marks

Metabolic features are common and include hypokalaemia (which may be severe),
hyperglycaemia, hypophosphataemia, hypomagnesaemia and hypercalcaemia. Metabolic
acidosis and respiratory alkalosis may be seen.

The man develops Supraventricular tachycardia. Other than cardioversion, what treatment
options are available to you? 4 marks

.Sinus tachycardia or supraventricular tachycardia with an adequate cardiac


output is best left untreated. Beta-blockers may be given in severe cases but
extreme caution should be used if the patient has asthma or COPD and in these
cases a relatively beta-1 specific or short-acting beta-blocker such as metoprolol
or esmolol may be considered (see below). Give propranolol 1 mg intravenously
(25-50 microgrammes/kg in a child) repeated at 2 minute intervals as required to
a maximum of 10 mg. Propranolol may also be effective in treating the metabolic
complications (e.g. hypokalaemia) associated with theophylline overdose.

Alternatively metoprolol 5mg slowly intravenously repeated after 5 minutes to


maximum of 10-15 mg in an adult or esmolol 50 microgrammes/kg slowly

PRAY FOR ME Dr. Ashraf Elshehry Page 374


intravenously followed by an infusion of 50-200 microgrammes/kg/min in an adult
( 300-1000 microgrammes/kg in a child) may be given. These will not correct the
metabolic disturbances in the same way as propranolol.

Patients with asthma or COPD who, after correction of hypokalaemia, have


supraventricular tachycardias causing haemodynamic compromise should be
treated with intravenous verapamil. Alternatively DC cardioversion should be
considered.

Toxbase

Chest pain thrombolysis

A 58 yr old man attends your department with 50 mins of central crushing chest pain. He
was previously well. His ECG is this

Describe this ECG (3 marks)

Sinus Rhythm. Normal axis. ST elevation in I and V2 - V4 with reciprocal ST depression


in II and III

What is the diagnosis? 1 mark

Anterior Myocardial infarction

What are the criteria for thrombolysis? (1 marks)

1 mm ST elevation in 2 contiguous limb leads


2 mm ST elevation in 2 contiguous chest leads
New onset LBBB

What are the absolute contraindications to thrombolysis (5 marks)?

PRAY FOR ME Dr. Ashraf Elshehry Page 375


Thrombotic stroke 2/12
Intracranial bleed
Head injury 2/12
Surgery/Trauma 2/12
Other bleed 1/12
Pregnant/recent delivery
Aortic Dissection

A 45 year old male is brought into the ED with a sudden onset of occipital headache, the
worst he has ever had followed by collapse. He has a GCS of 7/15 with decerbrate
posturing. He is hypertensive and his pulse is 78/min. The SHO says he thinks it is a
Subarachnoid haemorrhage and has called the anaesthetist. When you arrive he is
intubated, invasively monitored and on his way to CT.

1) How sensitive is CT in the first 12 hours after headache? (1)


98% within 12 hours
93% up to 24 hours
80% at 72 hours
50% at 1 week

2) What is the commonest cause of SAH? (1)


Ruptured berry aneurysm-77% of spontaneous SAH

3) Outline 4 features from the history would you be interesting in finding out? (2)
Smoking -strong link
Alcohol -strong link
History of hypertension -increased risk of aneurysm formation
Family History of SAH, marfans, ehlers-danlos
Prodrome or warning bleed -30-50% of aneurismal SAHs

4) Comment on the CT below (2)

PRAY FOR ME Dr. Ashraf Elshehry Page 376


Widespread fresh subarachnoid blood
Right Anterior communicating/anterior cerebral artery haematoma or right frontal
intracerebral bleed.

4) What grade Hunt and Hess classification is he? (1)

Grade 5
Hunt and Hess grading system
o Grade 1 - Asymptomatic or mild headache
o Grade 2 - Moderate-to-severe headache, nuchal rigidity, and no
neurological deficit other than possible cranial nerve palsy
o Grade 3 - Mild alteration in mental status (confusion, lethargy), mild focal
neurological deficit
o Grade 4 - Stupor and/or hemiparesis
o Grade 5 - Comatose and/or decerebrate rigidity

5) Once back in resus you are asked by the anaesthetist what we should do next. What is
your next therapeutic intervention and why? (1)

Nimodipine PO/NG 60mg 4 hourly or 1mg/hr IV. Helps prevent vasospasm

6) Give 2 other Complications of SAH. (2)

Seizures Hydrocephalus Hyponatraemia Rebleeding

PRAY FOR ME Dr. Ashraf Elshehry Page 377


Endocrine SAQ.

A 27 year old female attends the emergency department. She is agitated and confused.
History reveals that she underwent radioiodine treatment 2 days previously and she has
felt unwell since. She has a pulse of 143/min, in atrial fibrillation, she is normotensive and
has a temperature of 38°C. She is complaining of central abdominal pain.

1) What is the likely diagnosis? Thyroid storm/crisis (1)

2) Outline which 4 investigations you would carry out. (4)

ECG.
T3/T4 levels/TSH.
Amylase.
FBC/UE/Glucose/Cultures.
CXR

3) Outline your immediate management (5)

A- may require intubation


B- supplemental O2 +/- ventilation
C- Propranolol 1mg IV over one minute or 40 mg PO or Esmolol 500mcg.kg
bolus then 50-200mcg/kg/min
IV fluids Normal Saline
Carbimazole 25mg PO/NG
Other Paracetamol/ tepid sponges/ fan
Consider IVAntibiotics
Consider sedation

MA/CO SAQ 2006

Topics 2 do

Sickle Cell √
Digoxin toxicity √
Hyperkalaemia √
Cocaine OD √
Hypothermia √
Electrocution √
Near Drowning √
Diving, bends √

PRAY FOR ME Dr. Ashraf Elshehry Page 378


Maxillary injuries
Mandibular injuries

Worrying rash

A mother brings a 5 yr old boy to your ED because he has developed a non blanching rash
and she is concerned it is meningitis. An FBC shows normal platelets. Clotting screen is
normal.

Give 4 Symptoms or signs which would support a diagnosis of meningitis. (4 marks)

Fever
Neck stiffness (+ve Kernigs sign)
Systemic collapse
Headache/irritable

The diagnosis is not meningitis. What is it? (1 mark)

Henoch-Schonlein purpura

Give 3 features in the history which would support this diagnosis (3 marks)
Abdominal pain
Joint pain
Subcutaneous oedema

Why was the platelet count and clotting screen important? (2 marks)

Rule out Idiopathic Thrombocytopaenic Purpura (ITP)

PRAY FOR ME Dr. Ashraf Elshehry Page 379


Rule out Thrombotic Thrombocytopaenic Purpura (TTP)

http://www.emedicine.com/emerg/topic845.htm
Young women

A 23 yr old woman attends your department. She normally takes her combined OCP at 8
am daily. She has missed her pill this am. It is now 2100. How would you advise her (3
marks)

Missed pill advice (BNF)


If <12 hrs late just take pill and carry on
If > 12 hrs late : take missed pill and take further pills as usual. Pill ineffective for next
7/7 use condoms.
If > 7 pills left in pack, leave usual 7 day break at end of pills
If <7 pills in pack, run directly on into next pack.

Beware 28 day packs with “dummy” pills.

A 23 yr old woman attends the ED complaining of increased PV bleeding running on


from her period. Her period is normally 3 days, she has had heavier bleeding for 5 days.
How would you manage her? (3 marks)

Exclude pregnancy
Tranexamic acid
GP follow up

A 23 yr old woman attends A&E with low abdominal pain and is pyrexial. Assuming she
odes not have a urinary tract infection, how would you manage her? (4 marks)

Exclude pregnancy
GU referral
Ofloxacin
Metronidazole

Short answer questions – examples and discussion

Question 1 – a paediatric resus question

The question with the ideal answer


A 6 yr old child is brought to your hospital fitting.
He has been fitting for 5 minutes. He is attached to monitors and is receiving
high flow O2. He has been unwell for 3 days with a cough and a runny nose. He
is not immunised
PRAY FOR ME Dr. Ashraf Elshehry Page 380
He has a temperature of 39.6 0C. His weight is 20 kg

Part A
Fill in the names, doses and timing of the drugs in the algorithm (7
marks)

Lorazepam 0.1mg/kg iv/io Diazepam 0.5mg/kg PR

10 minutes
Lorazepam 0.1mg/kg iv/io

10 minutes

Paraldehyde 0.4ml/kg PR in equal volume of olive oil


Onset of action 10-15 minutes

Phenytoin 18mg/kg IV/IO over 20 minutes under ECG monitoring


or Phenobarbitone 15-20mg/kg IV/IO over 10mins if already on
phenytoin

Call anaesthetist
Thiopentone 4mg/kg IV/IO

7 marks if completely correct


6 marks if one drug left not filled or incorrect or timing incorrect
5 marks if two drugs incorrect or not filled in or timing incorrect
4 marks if three drugs incorrect or not filled in or timing incorrect
0 marks if four or more drugs incorrect or not filled in or timing incorrect

in addition 1 mark removed for any unsafe dose suggested

Part B
Some lesions were noted in the child’s mouth (see picture)

PRAY FOR ME Dr. Ashraf Elshehry Page 381


What are the lesions called and what is the diagnosis?
Lesions Kopliks spots 1/2 mark

Diagnosis Measles 1/2 mark

Part C
Name eight other notifiable diseases
Any eight from:

Paratyphoid Acute encephalitis


plague Anthrax
polio Botulism
rabies Brucellosis
relapsing fever cholera
rubella diphtheria
SARS dysentery
scarlet fever Food poisoning
Smallpox HIV/AIDS
syphilis Legionella
TB Leptospirosis
tetanus leprosy
typhoid fever malaria
typhus Measles
viral haemorrhagic fever meningitis
viral hepatitis meningococcal septicaemia
whooping cough mumps
yellow fever. ophthalmia neonatorum

2-3 = ½ mark
4-5 = 1 mark
6-7 = 1.5 marks
8 = 2 marks

Discussion of the question


A 6 yr old child is brought to your hospital fitting.
He has been fitting for 5 minutes. He is attached to monitors and is receiving
high flow O2. He has been unwell for 3 days with a cough and a runny nose. He
is not immunised
He has a temperature of 39.6 0C. His weight is 26 kg

The stem attempts to describe a clinical scenario. The child’s age is set to
avoid problems with calculating doses for very low weights and to provide a
realistic clinical scenario.
Measles is a common childhood condition and recognising it is important.
There is a clue in that the child is not immunised. The candidate will not know
that at this stage though, although the picture is clearly visitble.

PRAY FOR ME Dr. Ashraf Elshehry Page 382


The examiners want to test if the candidate knows the algorithm for status
epilepticus in children. Therefore in the exam, the candidates are presented
with a blank algorithm to fill in. This is a useful technique as the algorithm
should be well known to candidates. It is important that candidates are familiar
with common algorithms, or with algorithms that have just been produced.
There is also then no discussion of what the right or wrong answer is.

Part A
Fill in the names, doses and timing of the drugs in the algorithm (7
marks)

Lorazepam 0.1mg/kg iv/io Diazepam 0.5mg/kg PR

10 minutes
Lorazepam 0.1mg/kg iv/io

10 minutes

Paraldehyde 0.4ml/kg PR in equal volume of olive oil


Onset of action 10-15 minutes

Phenytoin 18mg/kg IV/IO over 20 minutes under ECG monitoring


or Phenobarbitone 15-20mg/kg IV/IO over 10mins if already on
phenytoin

Call anaesthetist
Thiopentone 4mg/kg IV/IO

7 marks if completely correct


6 marks if one drug left not filled or incorrect or timing incorrect
5 marks if two drugs incorrect or not filled in or timing incorrect
4 marks if three drugs incorrect or not filled in or timing incorrect
0 marks if four or more drugs incorrect or not filled in or timing incorrect

Marks are given as above according to the accuracy of the answer. There is
no negative marking here, as if the wrong thing is written down, the candidate
fails to gain a mark anyway.

Part B
Some lesions were noted in the child’s mouth (see picture)

PRAY FOR ME Dr. Ashraf Elshehry Page 383


What are the lesions called and what is the diagnosis?
Lesions Kopliks spots 1/2 mark

Diagnosis Measles 1/2 mark

This section should be easy and all candidates score well. Therefore the
marking is weighted to reflect that with only 1 mark in total for this section

Part C
Name eight other notifiable diseases
Any eight from:
Paratyphoid Acute encephalitis
plague Anthrax
polio Botulism
rabies Brucellosis
relapsing fever cholera
rubella diphtheria
SARS dysentery
scarlet fever Food poisoning
Smallpox HIV/AIDS
syphilis Legionella
TB Leptospirosis
tetanus leprosy
typhoid fever malaria
typhus Measles
viral haemorrhagic fever meningitis
viral hepatitis meningococcal septicaemia
whooping cough mumps
yellow fever. ophthalmia neonatorum

2-3 = ½ mark
4-5 = 1 mark
6-7 = 1.5 marks
8 = 2 marks

The college is using this type of question less frequently. Since the list is so
long, it is possible for the candidate to gain marks without being very
discriminatory about it. However, occasionally we will still use a question that
requires a list of answers.

PRAY FOR ME Dr. Ashraf Elshehry Page 384


Question 2 – a question on respiratory medicine

A 38 year old male arrives in the Emergency Department. He woke in the


early hours of today with shortness of breath. He denies chest pain or
cough. Apart from a pulse rate of 110 beats per minute, the CVS
examination is normal. On chest auscultation he has normal breath
sounds. He has no fever, and has an oxygen saturation of 91% on air.

Q1 Give four risk factors for Pulmonary embolism that you would
want to exclude in this patient (4 marks)
• Recent immobilisation/prolonged travel
• Recent lower limb trauma and/or surgery
• Clinical DVT
• Previous proven DVT or PE
• Major medical illness/cancer
• IVDU
• Family history/known clotting disorder
1 mark each to maximum of 4, 1 mark only if more than
one on one line

Q2a Apart from a D Dimer level, give three other investigations that are
useful at this stage in excluding other causes of his symptoms as
recommended by the British Thoracic Society? (3 marks)

• White cell count


• Chest Xray
• Peak flow
• ECG
• ABG
1 mark each to a maximum of 3 marks

Q2b His D dimer level is returned at 300 ng/ml (normal < 224ng/ml).
Give two management steps you would now take (2 marks)

Start low molecular weight heparin


Arrange a CTPA or Isotope scan

PRAY FOR ME Dr. Ashraf Elshehry Page 5


Q3 The patient suddenly deteriorates while in the Emergency
Department. He becomes hypotensive and his GCS falls to 12. His
arterial blood gases are taken on air and shown in the data booklet.
Other than high flow oxygen, what single management step will you
consider now? (1 mark)

Arterial blood gases on


pH 7.2 (7.35-7.45)
PaO2 8.5 kPa (>10.6)
PaCO2 4.0 kPa (4.7-6)
HOC-3 18 mmol/L (24-30)
Base excess –6 (minus 6)

Thrombolysis
Urgent CT scan /ECHO
IVC filtration
Embolectomy – Cardiothoracic surgical review
Ref; Guidelines of the British Thoracic Society, 2003

PRAY FOR ME Dr. Ashraf Elshehry Page 6


Discussion of the answers

A 38 year old male arrives in the Emergency Department. He woke in the


early hours of today with shortness of breath. He denies chest pain or
cough. Apart from a pulse rate of 110 beats per minute, the CVS
examination is normal. On chest auscultation he has normal breath
sounds. He has no fever, and has an oxygen saturation of 91% on air.
The stem attempts to paint a typical picture which is easily
recognisable as a potential pulmonary embolism. PE is important to
recognise and has a fairly clear investigation strategy that is
recommended.

Q1 Give four risk factors for Pulmonary embolism that you would
want to exclude in this patient (4 marks)
• Recent immobilisation/prolonged travel
• Recent lower limb trauma and/or surgery
• Clinical DVT
• Previous proven DVT or PE
• Major medical illness/cancer
• IVDU
• Family history/known clotting disorder
1 mark each to maximum of 4, 1 mark only if more than
one on one line
giving the possible diagnosis and asking how you would want to
confirm or exclude the diagnosis helps the examiners explore
candidates critical thinking.

Q2a Apart from a D Dimer level, give three other investigations that are
useful at this stage in excluding other causes of his symptoms as
recommended by the British Thoracic Society? (3 marks)

•White cell count


•Chest Xray
•Peak flow
•ECG
•ABG
1 mark each to a maximum of 3 marks
increasingly the College is using national guidelines as the basis for
questions, and they will often be recently published guidelines. This not
only ensures the exam is up to date but also encourages the candidates
to keep up with recently published scientific evidence. If there are
multiple guidelines, or different guidelines used in different countries,
then allowans for this will be made in the marking scheme

Q2b His D dimer level is returned at 300 ng/ml (normal < 224ng/ml).
Give two management steps you would now take in the
emergency department (2 marks)

Start low molecular weight heparin

PRAY FOR ME Dr. Ashraf Elshehry Page 7


Arrange a CTPA or Isotope scan
A very generic answer in terms of no doses required and alternatives for
imaging, recognising practice is different throughout the country. Note that
management indicates not only treatment but management as well. “in the
emergency department” avoids the answer “arrange admission” . it is
essential to read the question carefully as the cues for the correct answer are
contained in the question

Q3 The patient suddenly deteriorates while in the Emergency


Department. He becomes hypotensive and his GCS falls to 12. His
arterial blood gases are taken on air and shown below. Other than
high flow oxygen, what single treatment step will you consider
now? (1 mark)

Arterial blood gases on


pH 7.2 (7.35-7.45)
PaO2 8.5 kPa (>10.6)
PaCO2 4.0 kPa (4.7-6)
HOC-3 18 mmol/L (24-30)
Base excess –6 (minus 6)

Thrombolysis
IVC filtration
Embolectomy – Cardiothoracic surgical review

This sudden change in the condition of the patient is included to ensure


candidates can assess risk and respond accordingly. Although the definitive
diagnosis has not been made, the balance of probability is that it is a massive
PE and life saving treatment must be started.

Ref; Guidelines of the British Thoracic Society, 2003

PRAY FOR ME Dr. Ashraf Elshehry Page 8


FFAEM OSCE, Oct 1999
Format
OSCE Section. 12 parts, 5 minutes each. If not finished when the bell goes, prepare to be physically ousted
from the station.

Questions
1. ECG showing delta wave with shortened PR interval and R wave in V1. Patient presented with BP 80
systolic and palpitations. SHO gave adenosine.

(a) What does the ECG show?


(b) Why (did the patient present so)?
(c) What is the dose of adenosine?
(d) How does it work?
(e) What is the next treatment according to ALS?

Answers

WPW (Type A)
AV Nodal Re-entrant Tachycardia due to an accessory pathway - in this case right sided activation of
the right ventricle prematurely (pre-excitation by the bundle of Kent)
6mg then 12mg bolus followed by flush
+
Stimulates A1 receptors on myocardial cells affecting K channel cyclic amp production. This produces
transient high grade AV block. Is rapidly broken down by rbc's to give a t1/2 of 2 to 6 seconds
Synch DC cardioversion

2. Pictures:

(i) Vescicular lesions of the face.

(a) What is the diagnosis?


(b) Explain this appearance.
(c) What are the complications?

Answers

Chicken Pox
?
Bacterial infection (skin, otitis media), scarring, pneumonia, encephalitis

(ii) Skin lesion on lower limb (extensive, ulcerated) followed by healed appearance in subsequent
photograph. Patient has ulcerative colitis.

(a) What is the diagnosis?


(b) What treatment has she had?

Pray to me Dr. Ashraf El Shehry


FFAEM OSCE, Oct 1999

Answers

Pyoderma Gangrenosum
Prednisolone

3. Abdominal x-ray showing distended large bowel in a young man just returned from Egypt with fever
and abdominal pain.

(a) What does the x-ray show?


(b) What investigations are
indicated? (c) What is the treatment?
(d) What is the worst outcome?

Answer

No idea what they were after - I wrote about infective causes and would probably not have ordered an
AXR

4. Two parents and their twin girls age 6 spent the day in the New Forest picking wild flowers and
mushrooms. They left the car tailgate open as it was a hot day, and drove home, on arriving at home the
parents found their girls unconscious in the back seat. They are brought to A&E.

(a) Give a differential diagnosis.


(b) What investigations are
indicated? (c) How would you
manage them?

Answers

Poisoning (plant / fungus, CO, drugs), Hyperthermia, Infection (cerebral / sepsis), Head Injury / NAI
Vitals (including temp, p, BP, RR, O2Sats, Cap refil, BM), Carboxy Hb, ABG. Others dependent on
findings.
ABCDE. Support / treat cause.

5. You are planning a retrospective audit on a condition in your A&E department.

a) When would you use sampling?


b) How could you be sure to get all the subjects?
c) What statistical test would you use to compare the data with another A&E department?
d) How would you present the statistics?

Any ideas?

Pray to me Dr. Ashraf El Shehry


FFAEM OSCE, Oct 1999
6. This boy fell off his bicycle. What injuries are shown?

a) Elbow x-ray.

Answer

An anterior fat pad and a probable undisplaced supracondylar fracture.

b) This boy fell over the handlebars on his BMX. CXR: List four abnormalities

Answer

right pneumothorax, right haemothorax /effusion, subcutaneous emphysema, # ribs 3-8 on right
(ATLS barn door Xray)

7. This young lady fell off her horse. Pelvis x-ray.

a) What are the injuries?


b) What are the complications?

Answer

Fractured left neck of femur and acetabular fracture.


NOF: avascular necrosis, non union, leg shortening, 2ndary OA,
Acetabulum: Obstetric difficulties, trauma to pelvic contents, myositis ossificans

8. This man presented with chest pain and was given intravenous analgesia with an antiemetic. Shortly
after he adopts a bizarre posture and is unable to let go of the blankets.
ECG is given showing hyperacute anterolateral myocardial infarction.

a) What is the problem?


b) What treatment is indicated?
c) What are the causes of ST elevation?

Pray to me Dr. Ashraf El Shehry


FFAEM OSCE, Oct 1999
Answer

Acute dystonic reaction 2ndary to metoclopramide


IV anticholinergic (benztropine 1-2 mg)
Myocardial infarction, myocardial injury, LV aneurism, normal variants (e.g. athletic heart, Edeiken
pattern, high-take off), acute pericarditis

9. A child has been in the garage where there was a whole bottle of white spirit which is now empty. She
comes in coughing.

a) What does this CXR show?


b) What is the diagnosis?
c) List other signs of hydrocarbon exposure.
d) List other poisons which cause respiratory signs and symptoms in children.

Answers

Hilar flare
Aspiration
Aspiration (related to low viscosity). Systemic toxicity (CNS and cardiotoxicity). Phenol, toluene and
camphor have high systemic toxicity. Low systemic toxicity agents include furniture polish, mineral
spirits (white spirit in the UK), kerosene, lighter fuel. Turpentine has variable systemic toxicity but is
rare in the UK.
Aspirin, opiates, Benzodiazepines, nearly all psychoactive agents, many others

10. 5 pictures given:

(i) mouth with ulceration / crusting of lip and a white exudate on tongue. What is the
diagnosis? - Candida

(ii) child with widespread erythematous maculopapular rash. Presents with this rash, fever, and
occipital lymphadenopathy. What vaccination has been missed? - MMR

(iii) Parallel linear contusions to side of child’s face. What is this appearance due to? - Slap

(iii) circular 5mm punched out burns to child’s hand. List two reasons to suspect NAI in this
case. - appearance (cigarette), old

(iv) Child with eczema and secondary infection. What organism is usually responcsible for
deterioration in this rash? - Staph Aureus

Pray to me Dr. Ashraf El Shehry


FFAEM OSCE, Oct 1999
11. A drunk lady fell downstairs. She is managed according to ATLS with c. spine
immobilisation and oxygen, and a 16G iv cannula is inserted. There is pain on springing her
chest but no flail segment. With an FiO2 of 85% her oxygen saturation is 85%, and she has a
sinus tachycardia. Her GCS is 6.

CXR & CT Head

What are the abnormalities on


this CXR? What are the
management priorities?
What 4 abnormalities are shown on the CT Head?

Answers

Right fractured ribs 5,6,7. Rim pneumothorax.


ABCDE. RSI and ventilate on arrival in the ED, right large chest tube (before or after RSI
depending on situation), if remains tachycardic IV fluids and hunt bleeding cause, CT
brain & neurosurgical assessment to consider surgery, give mannitol.
Cortical atrophy, midline shift, compression of lateral ventricles, subdural haematoma
(?acute/?acute- on-chronic).

12. Asked to demonstrate on ALS mannekin. A young man collapses out of hospital. Please
assess and manage as you would. (Patient in cardiorespiratory arrest). During BLS, examiner
hands over a bag-valve- mask device. "Show me how you would use this equipment."

Appropriate for an exit exam?

Pray to me Dr. Ashraf El Shehry


1- Question 1 of 5
A 36 yo lady presents to the ED with a short history of breathlessness a rash & feeling wheezy & SOB that came on
whilst out at a restaurant with friends. She is a known asthmatic & also has eczema

(a) Name 4 common causes of anaphylaxis:


Answer (Latex - Food (nuts, shellfish, wheat, strawberries) - Hymenoptera (bee stings) - Drugs & vaccines (many
including: NSAIDS, sux, Abx, Asprin, IV contrast etc)
(b) What is your initial management (assume airway compromise & a severe reaction)?
Answer (Assess ABC & treat accordingly -100% O2- IM adrenaline if needed (0.5mg 1:1000 IM) Salbutmaol nebs -
10mg IV chlorphemeramine- Fluid if not responding to adrenaline - Oral or IV steroid depending on if Pt can swallow
(c) After assessing ABC the Pt is stable. How long would you want to observe her for & why?
Answer (At least 4 hrs looking for delayed hypersensitivity, also Pt is asthmatic & they often have more severe react.
(d) Which medication is pertinent in the history of any Pt presenting with angio-oedema?
Answer (ACE inhibitor i.e. Ramipril)
(e) How do you treat Hereditary angio-oedema?
Answer (C1 esterase inhibitor)
‫ــــــــــــــــــــــــــــــــــــــــــــــــــــــــــــــــــــــــــــــــــــــــــــــــــــــــــــــــــــــــــــــــــــــــــــــــــــــــــــــــــــــــــــــــــــــــــــــــــــــــــــــــــــــــــــ‬
2- Question 2 of 5
A 65 yo gentleman presents to the department with a classic sounding history for cardiac chest pain which came on
suddenly across his chest 32 hrs ago.

(a) Describe the 2 important changes shown by the ECG (fig 1) & hence locate anatomically the pathology
Answer (1- ST elevation in II,III & aVF 2- Reciprocal changes in leads V1 & V2 3- This is therefore an acute inferior MI)
(b) What is it important to be vigilant for when a Pt presents with this type of picture (specific to the above
ECG changes) & what additional test could you do to access for this?
Answer (Rt ventricular infarct Do Rt ventricular leads (V4R))
(c) The Pt responds very quickly to GTN spray & the chest pain settles, also you notice that the repeat ECG
taken 5 minutes after the GTN now looks normal. What is a possible explanation for such quick resolution?
Answer (ECG changes resolving so quickly are more in keeping with Prinzmetal angina or variant angina)
(d) List 4 of the most important contraindications to thrombolysis
Answer (1- Severe HTN systolic >200 mmHg, diastolic >120mmHg
2- Head injury, CVA or recent TIA
3- Previous neurosurgery or cerebral tumour
4- Recent GI or GU bleed
5- Warfarin
6- Pregnancy 7- Recent major surgery
8- Puncture of non-compressible vessel )
(e) List 4 things that can cause a rise in Troponin levels
Answer (a. Acute myocardial infarction b. Sepsis c. Acute renal failure d. Pulmonary embolus)
‫ــــــــــــــــــــــــــــــــــــــــــــــــــــــــــــــــــــــــــــــــــــــــــــــــــــــــــــــــــــــــــــــــــــــــــــــــــــــــــــــــــــــــــــــــــــــــــــــــــــــــــــــــــــــــــــ‬
3- Question 3 of 5

A 39 yo man presents to the department after a fall from a 6 metre ladder onto his Rt side

1
(a) Describe the most important things that this CT chest shows (fig 1):
Answer (1- Pulm. contusion 2- Fractured ribs Also 3- Hameothorax 4- Pneumothorax (apical) 5- (Surgical emphysema )
(b) What's the most important initial management step? Where should this Pt be sent? What's your choice of
analgesia?
Answer (ABCD then as part of that ruling out a tension pneumothorax & insert. of a thoracostomy tube, Likely going to need
HDU/ITU care, Thoracic epidural but mark for mentioning that an intercostal nerve block may be a good intermediate choice)
(c) The Pt deteriorates in front of you with sats of 85% on 80% FiO2 you decide that he requires intubation &
IPPV. Describe the steps required for an RSI (including any drug doses)
Answer (Preoxygenate 3 mins if possible or ventilate with 100% O2, Ensure adequate monitoring ECG, Sats, BP &
secure IV access, Cricoid Pr., ensure not released until ET secured, Induction agent (Thiopentone 3-5mg/kg,
Etomidate 0.3mg/kg, Ketmaine 0.5-2mg/kg), Suxamethonium 1-1.5mg/kg, Confirm placement of tube (best by direct visualisat.
of tube passing through the cords), end tidal CO2. Listen to the chest both sides, inflate cuff & secure ET tube.)
(d) As part of the 1ry survey you notice that the Pt has a tender abd.. What do you do next?
Answer (FAST scan/DPL to rule out ? liver injury If stable CT Abd., if not laparotomy.)
(e) The CT is reported & the injuries are worse than you first thought, amongst other things there is a fracture
of the first rib. What is it essential to rule out & how will you do this?
Answer (Need to rule out an aortic injury, needs arch aortogram)
‫ــــــــــــــــــــــــــــــــــــــــــــــــــــــــــــــــــــــــــــــــــــــــــــــــــــــــــــــــــــــــــــــــــــــــــــــــــــــــــــــــــــــــــــــــــــــــــــــــــــــــــــــــــــــــــــــــــــــــــــــــــــــــــــــــــــــــــــــــ‬
4- Question id: 2003
A 26 yo male presents after having been hit around the head with a metal bar. His GCS on admission is 14 but after an
hr it has fallen to 13. His CT is shown (fig 1):

Fig 1
(a) Describe what it shows:
Answer (1- Rt frontal acute extradural haematoma 2- Skull fracture 3- Air bubble in the haematoma 4- Midline shift)
(b) Describe your initial management priorities, including the definitive management for this Pt
Answer (1- ABCDE 2- Intubation & controlled ventilation 3- Try to decrease ICP +/- mannitol 4- Arrange
appropriate neurosurgical referral/transfer 5- Definitive = Burr hole & evacuation of the haematoma.)

2
(c) List all indications for an immediate CT scan of the head following head trauma according to the NICE
guidelines.
Answer (1- GCS<13 when 1st assessed 2- GCS<15 when assessed 2 hrs after the injury 3- Suspected open or
depressed skull fracture 4- Sign of fracture at skull base (panda eyes, haemotypanum, Battle's sign, CSF leak from
nose or ears. 5- Post traumatic seizure 6- Focal neurological deficit 7- >1 episode of vomiting 8- Amnesia of events >
30 mins before impact)
(d) For a child under 16 how many episodes of vomiting after a head injury is acceptable before imaging is
required according to NICE?
Answer (3 or more = CT head)
(e) What is the role of hypothermia therapy after traumatic brain injury in children?
Answer (It does not improve outcome & may increase mortality)
‫ــــــــــــــــــــــــــــــــــــــــــــــــــــــــــــــــــــــــــــــــــــــــــــــــــــــــــــــــــــــــــــــــــــــــــــــــــــــــــــــــــــــــــــــــــــــــــــــــــــــــــــــــــــــــــــــــــــــــــــــــــــــــــــــــــــــــــــــــــــــــــــــ‬
5- Question id: 2004
A 68 yo lady is brought in to ED resus at 06:00 am. The crew say that she had a short history of waking up & being
unable to get her breath. O/E she has a RR of 36, sats of 89% on a re-breathe mask

(a) The ED in which you work has the resources to measure BNP or N-terminal- ProBNP. Is there any
diagnostic value in measuring this?
Answer (Yes, BNP> 500 pg/dL or NT- proBNP> 1000 pg/dL makes acute heart failure syndrome likely (approx +ve
likelihood ratio [LR+} = 6)
(b) The Pt deteriorates & you start her on CPAP 5 mmHg. Please explain briefly how CPAP works
Answer (1. CPAP splints the alveoli open, thereby preventing alveolar collapse & allowing unimpeded alveolar
ventilation (Recruits alveoli). 2. CPAP also ↓ preload & afterload, improves lung compliance,  FRC & ↓ work of
breathing. 3. NIV reduces trans-diaphragmatic Pr., Pr. time index of respiratory muscles & diaphragmatic EMG
activity. This leads to an  in tidal volume, ↓ in RR &  in minute ventilation. Also overcomes the effect of intrinsic
PEEP. CPAP reduces left ventricular transmural Pr. & therefore  CO. Hence it is a very effective for TTT of
pulmonary oedema. Causes increase in intrathoracic Pr. therefore improving cardiac output.)

(c) Her Bl. Pr. is 113/56 & decides to treat her with a nitrate infusion. Is there any evidence for or against
giving diuretics please discuss:
Answer: Yes but always in combination with nitrates. There is also some evidence to move away from diuretic
montherapy as it is unlikely to prevent the need for tracheal intubation & can worsen renal function which has been
shown to increase mortality.1 The advice is to use it in combination & to use them judiciously
(d) Explain the mechanism behind how diuretics work in the acute management of heart failure & how this fits
in with the pathogenesis of acute pulmonary oedema.
Answer: They work via venodilatat. The other acute HF syndromes (pulm. edema, HTN crisis & exacerbated HF) are
caused by a combinat. of progressive excessive vasoconstrict. superimposed on ↓ lt vent. functional reserve. The
impaired cardiac power & extreme vasoconstrict. induce a vicious cycle of afterload mismatch resulting in a dramatic
↓ of CO &  lt vent. end diastolic Pr., which is transferred backwards to the pulm. capillaries yielding pulm. oedema.
Therefore, the immediate TTT of these acute HF syndromes should be based on the administrat. of strong, fast-acting

3
IV vasodilators such as nitrates or nitroprusside. After initial stabilizat., therapy should be directed at ↓ recurrent episodes
of acute HF, by prevention of repeated episodes of excessive vasoconstrict. along with efforts to optimize cardiac funct.
(e) You decide to insert a central line as the Pt has very poor peripheral access. You insert a left sided internal
jugular line as there was some local scar tissue on the Rt. When you get the chest x-ray to confirm position you
see the film shown (see fig 1). Can the line be used? What would you want to do prior to using it?
Answer (It is actually venous, it's in a low brachiocephalic trunk but it doesn't look like it. You would want to check
the length of line inserted should be at least 14 cm. Aspirate all lumens & run it through a Bl. gas analyser to confirm
that it is venous Bl.., Attach it to a CVP monitor & transducer the line to look for a venous waveform. If further doubt
perform a venogram rarely the anatomy is unusual like in this case)
‫ـــــــــــــــــــــــــــــــــــــــــــــــــــــــــــــــــــــــــــــــــــــــــــــــــــــــــــــــــــــــــــــــــــــــــــــــــــــــــــــــــــــــــــــــــــــــــــــــــــــــــــــــــــــــــــــــــــــــــــــــــــــــــــــــــــــــــــــ‬
6- Question 1 of 50
A 50 yo man presented with an extremely painful lt ankle. There was no history of trauma & the pain had a gradual
onset over the previous 24 hrs. O/E there was minor redness & swelling.

(a) Besides acute gouty arthritis name 2 other possible diagnoses?


Answer (Pseudogout & septic arthritis.)
(b) Name three predisposing factors to acute gouty arthritis?
Answer (Trauma, surgery, starvation, dietary overindulgence, & ingestion of drugs affecting serum urate
concentrations (eg, allopurinol & uricosuric agents) may all promote gouty attacks. In men, increasing alcohol
consumption (beer & spirits, but not wine) is associated with proportionately a greater risk of developing gout.)
(c) What are the 2 most appropriate investigations to confirm acute gouty arthritis in this Pt?
Answer:  serum urate(however an appreciable No. of cases occur in Pts with normal or even ↓ serum urate
concentrate.) & aspirat. of synovial fluid from the affected joint & analysis of the fluid by Gram stain, culture &
polarized light microscopy.
(d) Name one feature on a plain radiograph suggestive of gouty arthritis?
Answer: Subcortical bone cysts. In chronic tophaceous gout bone erosions due to tophi may have delicate
"overhanging" edges & punctate to diffuse soft tissue calcification.
(e) Name 2 factors that predispose to chronic tophaceous gout?
Answer: Pts treated with cyclosporine (& often diuretics as well) are at  risk for the accelerated development of
chronic tophaceous gout. Other Pts at  risk for chronic tophaceous gout are those who have chronic kidney disease
that precludes full dose antihyperuricemic drug therapy & those who are allergic to or otherwise intolerant of
uricosuric agents & allopurinol.
‫ـــــــــــــــــــــــــــــــــــــــــــــــــــــــــــــــــــــــــــــــــــــــــــــــــــــــــــــــــــــــــــــــــــــــــــــــــــــــــــــــــــــــــــــــــــــــــــــــــــــــــــــــــــــــــــــــــــــــــــــــــــــــــــــــــــــــــــــ‬
7- Question 2 of 50
A 46 yo business man develops diarrhoea 3 days into his trip to SE Asia. He has 3 very important meetings over the
next three days & comes into the ED.

(a) What advice would you give him?


Answer (General advice re traveller s diarrhoea including strict h& hygiene not eating washed salad not drinking the
water. Could prescribe antibiotics, good evidence for early TTT. In SE Asia consider Campylobactor therefore
azithromycin rather than ciprofloxacin.1)
(b) What would you tell him about chemoprophylaxis for any further business trips?
Answer (Expert opinion supports the use of prophylactic antibiotics when the trip is vitally important or the
consequences of diarrhoea would be difficult to manage.)
(c) Why is chemoprophylaxis currently not recommended for most traveller s?
Answer (Potential complications i.e. predisposition to other conditions e.g. vaginal c&idiasis, or C. difficle diarrhoea,
development of bacterial resistance, cost, lack of data & efficacy of antibiotics given for 2 or 3 weeks.)
(d) Are probiotics any good at preventing traveller s diarrhoea?
Answer (Yes as much as 15% risk reduction)
(e) When he returns he finds that the diarrhoea has persisted for 2 weeks he comes to the ED out of hrs, as he
couldn't get a GP appointment. What do you suggest?
Answer (Stool sample should be sent for giardia, cryptosporidium & other parasities & GP should follow up.)
‫ـــــــــــــــــــــــــــــــــــــــــــــــــــــــــــــــــــــــــــــــــــــــــــــــــــــــــــــــــــــــــــــــــــــــــــــــــــــــــــــــــــــــــــــــــــــــــــــــــــــــــــــــــــــــــــــــــــــــــــــــــــــــــــــــــــــــــــــ‬
8- Question 3 of 50
A 24 yo woman presented with left flank pain which had come on gradually & difficulty passing urine. O/E her temp.
was 38.4 degrees. Urinalysis revealed nitrites, leukocytes & Bl. in her urine.

(a) What is the most likely diagnosis?


Answer (Pyelonephritis.)
4
(b) How would you further investigate this Pt?(Four steps)
Answer (MSU to laboratory for microscopy, culture & sensitivity. FBC, UE, Bl. cultures, renal ultrasound.)
(c) How would you manage this Pt?(Three steps)
Answer (Antibiotics, IV fluids, analgesia, organise admission or follow up)
(d) What is the most common pathogen to cause this condition?Name another 2 pathogens to cause this
condition?
Answer (Most common- E.Coli. Others include klebsiella, enterococcus, & staph saprophyticus.)
(e) Name three risk factors for this condition?
Answer (Frequency (≥3 times per week) of sexual intercourse in the previous 30 days, UTI within the previous 12
months (OR 4.4), diabetes (OR 4.1), stress incontinence within the previous 30 days (OR 3.9), a new sex partner in
the previous yr (OR 2.2), recent spermicide use (OR 1.7), UTI history in the participant's mother (OR 1.6).)
‫ـــــــــــــــــــــــــــــــــــــــــــــــــــــــــــــــــــــــــــــــــــــــــــــــــــــــــــــــــــــــــــــــــــــــــــــــــــــــــــــــــــــــــــــــــــــــــــــــــــــــــــــــــــــــــــــــــــــــــــــــــــــــــــــــــــــــــــــ‬
9- Question 4 of 50
A 60 yo woman presented to the ED complaining of lethargy & weakness. Her husband reported that she seemed
confused at times over the previous day. She self reported that she had a history of addisons disease. Her medical
records were unavailable as she lived overseas & was visiting her daughter in the area. O/E she had a postural drop in
her Bl. Pr. & her HR was 110bpm.

(a) What is the most likely diagnosis?


Answer (Addisonian crises.)
(b) What are the common causes of this condition?(Name 2)
Answer (Withdrawal of longterm steroid therapy, intercurrent injury, infection or stress.)
(c) How would you investigate this Pt?
Answer (FBC, UE, Bl. glucose, Ca, Bl. cultures, urine cultures, sputum culture, CXR.)
(d) What findings would you expect from the electrolyte profile?(Name 2)
Answer (Hyponatremia, hyperkalaemia, uraemia)
(e) How would you manage this Pt?
Answer (Hydrocortisone 100mg IV stat, IV fluids, treat hypoglycaemia if present, treat with broad spectrum
antibiotics if infection is believed to be the precipitant, specialist consultation.)
‫ـــــــــــــــــــــــــــــــــــــــــــــــــــــــــــــــــــــــــــــــــــــــــــــــــــــــــــــــــــــــــــــــــــــــــــــــــــــــــــــــــــــــــــــــــــــــــــــــــــــــــــــــــــــــــــــــــــــــــــــــــــــــــــــــــــــــــــــ‬
10- Question 5 of 50
A 75 yo man was found collapsed at home by his son. There was no available past medical history.He was living
independently & had last been seen 2 days previously by his son. O/E his GCS was 7/15. There was no nuchal
rigidity, pupil reflexes were sluggish but fundi exam. was noraml. Tone was slightly  in all 4 limbs. The peripheral
reflexes were present & plantars were downgoing. The HR was 39 bpm & the Bl. Pr. was 76/42 mmHg. Heart sounds
were normal & the chest was clear. Hypothermia was suspected.

(a) How is hypothermia defined?


Answer (A fall in core temp. below 35 degrees)
(b) How should the temp. be measured?
Answer (Rectal (core) temp. with a low reading thermometer.)
(c) List some clinical features of this condition?(four)
Answer (Impaired consciousness, cardiac embarrassment, bradycardia, hypotension, hypopnoea, sluggish pupillary &
peripheral reflexes, muscle rigidity, coma, ventricular fibrillation, & asystole)
(d) List some ECG features of this condition?(list four)
Answer (Bradycardia, tremor artefact, J-waves, prolonged QT interval, prolonged PR.)
(e) How would you treat this man if the diagnosis is confirmed?
Answer (Gradual rewarming using space blankets. If the Pt is unconsciousness or the temp. is less than 32 warm IV
fluids may be given. The aim is to increase the temp. by one degree/hr.)
‫ـــــــــــــــــــــــــــــــــــــــــــــــــــــــــــــــــــــــــــــــــــــــــــــــــــــــــــــــــــــــــــــــــــــــــــــــــــــــــــــــــــــــــــــــــــــــــــــــــــــــــــــــــــــــــــــــــــــــــــــــــــــــــــــــــــــــــــــ‬
11- Question 2151
An 80 yo man who has CKD 4 is sitting in the waiting area waiting to get a pre-tibial laceration dressed when he
develops palpitations & feels unwell. A nurse kindly puts him in a bay & records an ECG (fig 1)

5
(a) What do the rhythm strips in figure 1 show?
Answer Torsades de pointes
(b) Name 2 causes for this rhythm
Answer Hypomagnesaemia, hypokalaemia, prolonged QT interval (congenital or drug related)
(c) Name to drugs that could have caused this rhythm?
Answer Sotalol, antipsychotics, antihistamines, antidepressants,
(d) Why is it a concerning pattern?
Answer It may degenerate into VF
(e) How is it treated?
Answer IV magnesium 2g IV over 10 mins
f) It appears to be refractory what do you do?
Answer: call cardiologist may require over-drive pacing, can consider an isoprenaline infusion whilst awaiting pacing
‫ـــــــــــــــــــــــــــــــــــــــــــــــــــــــــــــــــــــــــــــــــــــــــــــــــــــــــــــــــــــــــــــــــــــــــــــــــــــــــــــــــــــــــــــــــــــــــــــــــــــــــــــــــــــــــــــــــــــــــــــــــــــــــــــــــــــــــــــ‬
12- Question 18 of 50
A 55 yo man is brought to the ED with haematemesis. His medical history is of alcohol abuse.

(a) Give a differential diagnosis?(give five)


Answer: Peptic ulceration, mucosal inflammation (oesophagitis, gastritis or duodenitis), oesophageal varices,
mallory-weiss tear, gastric carcinoma, coagulation disorders, tumors.
(b) What is the most common cause?
Answer: PUD.
(c) How would you investigate this Pt?(Give five)
Answer: FBC, Coag screen, UE, Bl. glucose, group & cross-match, LFTs.
(d) How would you manage this Pt?
Answer: ABCs, 2 large IV cannulae, IV fluids, consider PPI, keep fasted, gastroenterology or surgical consultation.
(e) If the cause is thought to be oesophageal varices another medication which may be indicated?
Answer: Vasopressin/terlipressin & octreotide/somatostatin
‫ـــــــــــــــــــــــــــــــــــــــــــــــــــــــــــــــــــــــــــــــــــــــــــــــــــــــــــــــــــــــــــــــــــــــــــــــــــــــــــــــــــــــــــــــــــــــــــــــــــــــــــــــــــــــــــــــــــــــــــــــــــــــــــــــــــــــــــــ‬
13- Question 19 of 50
A 26 yo samoan painter & decorator was admitted with acute colicky central abd. pain associated with vomiting. The
only past medical history was of a viral illness associated with a rash 2 wks previously. O/E he was pale, his HR was
120 bpm & his Bl. Pr. was 140/80 mmHg. The abd. was generally tender but there was no guarding, & bowel sounds
were present. Exam. of the CNS revealed reduced power & tone in the lower limbs & absent ankle & knee reflexes.
Investigat. were unremarkable apart from a microcytic anaemia. A diagnosis of lead poisoning is being considered.

(a) What investigation would confirm the diagnosis?


Answer: Serum lead concentration
(b) What is the TTT?
Answer: D-penicillamine
(c) Who is at risk for this condition?(Name four)
Answer: Scrap-metal workers, plumbers, individuals ingesting water from lead pipes, children ingesting old lead
based paint in the house, painters & decorators.
(d) What is the differential diagnosis for the above Pt?(Give 2)
Answer: AIP, arsenic poisoning, guillain-barre syndrome, PAN, sarcoidosis, alcohol abuse.
(e) A lead concentration above what value is considered toxic?
Answer: 4mmol/l
‫ـــــــــــــــــــــــــــــــــــــــــــــــــــــــــــــــــــــــــــــــــــــــــــــــــــــــــــــــــــــــــــــــــــــــــــــــــــــــــــــــــــــــــــــــــــــــــــــــــــــــــــــــــــــــــــــــــــــــــــــــــــــــــــــــــــــــــــــ‬
14- Question 21 of 50
6
A 30 yo woman was brought to the ED after an overdose of amitriptyline.

(a) Name 8 possible clinical features?


Answer: Tachycardia, dry skin, dry mouth, dilated pupils, urinary retention, ataxia, drowsiness, delerium,
hallucinations, dysarthria & jerky limb movements. If the Pt is unconscious they may have increased muscle tone,
increased reflexes, myoclonus, an extensor plantar response, convulsions & a divergent squint.
(b) What Bl. investigations should also be considered? (Give 2)
Answer: Fingerstick glucose, to rule out hypoglycemia as the cause of any alteration in mental status, paracetamol &
salicylate levels to rule out these common co-ingestions, & serum beta-HCG.
(c) What ECG changes might be expected in this condition?
Answer: Sinus tachycardia, increased PR interval, increased QRS complex, ventricular arrhythmias.
(d) How would you manage this Pt?
Answer: ABCs, monitoring, activated charcoal if within one hr, IV lorazepam for seizures, consider treating acidosis
with 8.4% Na bicarbonate, IV fluids & poisons information consultation.
(e) What are the possible complications of Na bicarbonate infusion?(Give 2)
Answer: Anaphylaxis, volume overload, hypernatremia, & metabolic alkalosis may result from prolonged
bicarbonate infusion. Other listed adverse effects include cerebral hemorrhage, CHF (aggravated), oedema, tetany,
gastric distension, hypernatremia, hyperosmolality, hypocalcemia, hypokalemia, intracranial acidosis & pulm. edema.
‫ـــــــــــــــــــــــــــــــــــــــــــــــــــــــــــــــــــــــــــــــــــــــــــــــــــــــــــــــــــــــــــــــــــــــــــــــــــــــــــــــــــــــــــــــــــــــــــــــــــــــــــــــــــــــــــــــــــــــــــــــــــــــــــــــــــــــــــــ‬
15- Question 22 of 50
A 28 yo gentleman from Poland attends after falling over in the garden & cutting his finger on a piece or metal that
was supporting a plant in his vegetable patch. The wound appears to be trivial.

(a) What important questions do you need to illicit in the history? (2 marks)
Answer: Essentially is it a tetanus prone wound? How old is the wound, was there any manure in the ground? Has he
had immunisations against tetanus?
(b) What are the signs of tetanus infection & at what stage after sustaining a wound do they present? (2 marks)
Answer: Presentat.: is 4-21 days (average 10) after infect., with agonising contract. superimposed on muscular rigidity
(c) What features constitute a tetanus prone wound? List 5 things.
Answer: a significant degree of devitalised tissue puncture type wound contact with soil or manure clinical evidence
of sepsis occurring more than 6 hrs before presentation
‫ـــــــــــــــــــــــــــــــــــــــــــــــــــــــــــــــــــــــــــــــــــــــــــــــــــــــــــــــــــــــــــــــــــــــــــــــــــــــــــــــــــــــــــــــــــــــــــــــــــــــــــــــــــــــــــــــــــــــــــــــــــــــــــــــــــــــــــــ‬
16- Question 23 of 50
A 69 yo smoker who lives alone is brought in acutely dyspnoeic by the crew. Initial observations show the she is
drowsy GCS 13. RR 33, HR 146, BP 78/47. They let you know that according to her next door neighbour she only
came home from hospital last week & hasn't left the house since. She has O2 at home & is on lots of medication.
Apparently she is awaiting placement in a nursing home as she can no longer manage with 3 calls a day. You
diagnose a severe exacerbation of COPD & are not concerned about sepsis.

(a) Your initial impression is that the Pt is peri-arrest. List 3 important things that you need to try to do in the
next 5 minutes in order of priority.
Answer: 1. Treat what you can treat i.e. ABCDE assessment gain IV access, take Bl. off etc. Try to ascertain a
diagnosis an ABG will be very helpful as will a CXR (may be too unstable?) 2. Get the arrest trolley out & organise
your team. 3. Try to speak with any family or the GP if possible, track down hospital notes; is there a plan in place for
this Pt if she should become very unwell?
(b) You notice that the O2 is flowing at 15 litres from the wall supply via a non-rebreathe mask. Is this of
relevance to the Pt's condition?
Answer: It might be, You need to access if the Pt is adequately O2ating & ventilating. The sats probe will help with
the former but not the later. An ABG will guide you. It is possible that she has been over oxygenated on route to
hospital & that the CO2 is raised causing the lowered GCS. Turning down the O2 may improve the Pt's condition.
(c) What amount of O2 should you give this type of Pt prior to obtaining an ABG?
Answer: This is clearly not an exact science but it is much better to start low & titrate up when the history points
towards COPD. If the Pt is known to have had hypercapnic respiratory failure in the past then give an FiO2 of 24%
via a venture mask. For all other Pts & when the diagnosis is unclear give 40% FiO2 until an ABG has been obtained.
(d) List 3 therapies that you gave the Pt on admission.
Answer: Salbutmaol nebs 2.5mg or 5 mg, Ipratropium nebs 500mcg & steroids, prednisolone 30mg if could swallow
(unlikely) therefore 200mg of IV hydrocortisone. Stat dose of doxycycline also given 200mg.
(e) The medical registrar dem&s that this Pt is put on NIV Rt now & sent straight to medical HDU. The Pt s
observations have now worsened. What is your response?
7
Answer: No. The Pt is clearly unstable is peri-arrest & would not tolerate NIV at present. Moving the Pt would be
catatrophic. ITU need to be involved with this Pt. If a decision is made that invasive ventilation is not appropriate then
a trial of NIV is an option although it may not be successful.
‫ـــــــــــــــــــــــــــــــــــــــــــــــــــــــــــــــــــــــــــــــــــــــــــــــــــــــــــــــــــــــــــــــــــــــــــــــــــــــــــــــــــــــــــــــــــــــــــــــــــــــــــــــــــــــــــــــــــــــــــــــــــــــــــــــــــــــــــــ‬
17- Question 24 of 50

Fig 1 Fig 2:
(a) Describe how the QT interval is measured
Answer: From the start of the Q wave to the end of the T wave. See fig 1
(b) What dangerous arrhythmias can be precipitated from long QT intervals?
Answer: Torsades de Pointes ,VF & hence sudden cardiac death
(c) The QT interval gets shorter as the HR speeds up, & longer as it slows down. What is the QTc & why's it
important?
Answer: It is the corrected QT interval i.e. it takes the rate out of the equation. Normal range is <440 ms. See fig 2
(d) There are 2 types of LQTS congenital & acquired. Name 2 causes of acquired LQTS
Answer: 1- Antiarrhythmics: Quninidine, procainamide, disopyramidine, flecanide, propafenone, sotalol, ibutilide,
dofetilide, amaiodarone (rare) 2- Antimicrobials: Erythromycin, clarithromycin, trimethoprim, ketoconazole,
itraconazole, choloroquine. 3- Antihistamines: terfenadine Electrolyte imbalances Severe bradycardia
(e) Name another cause of sudden cardiac death (SCD)?
Answer: 1- Hyperthrophic cardiomyopathy (HCM), risk of SCD is  with early age of diagnosis, family hx of SCD,
Non-sustained VT on 24hy tape, Abnormal BP in response to exercise, certain genetic mutations. 2- Arrhythmogenic
Rt Ventricular Cardiomyopathy (ARVC) is probably the 2nd most common cause of unexpected sudden death in the
young. 3- DCM 4- Restrictive Cardiomyopathy is the rarest of the cardiomyopathies. 5- Myocarditis 6- Brugada
Syndrome 7- Progressive Cardiac Conduction Defect (Lev-Lenegre's Syndrome) 8- Idiopathic VF (without Brugada
ECG changes) Catecholaminergic Polymorphic VT
‫ـــــــــــــــــــــــــــــــــــــــــــــــــــــــــــــــــــــــــــــــــــــــــــــــــــــــــــــــــــــــــــــــــــــــــــــــــــــــــــــــــــــــــــــــــــــــــــــــــــــــــــــــــــــــــــــــــــــــــــــــــــــــــــــــــــــــــــــ‬
18- Question 25 of 50
A 39 yo woman is brought into the department acutely SOB c/o of chest pain (worse on inspiration) She has a RR of
45 & her sats are 89% in air. She was completely well 30 minutes ago but collapsed suddenly at work

(a) Give 3 differential diagnoses?


Answer: 1. Massive PE 2. Pneumonia 3. Cardiac collapse, M.I.; now with arrhythmia? 4. Sepsis
(b) Name 3 crucial tests that need to be done.
Answer: 1. ABG 2. CXR 3. ECG 4. Bedside ECHO
(c) Are d-dimers useful (comment on specificity & sensitivity?
Answer: It depends; they need to be interpreted within the realms of clinical probability. Current BTS guidelines are
as follows: If Pt has features consistent with PE namely a): Raised RR +/- haemoptysis +/- pleuritic chest pain Plus 2
other factors: 1. Absence of another reasonable clinical explanation. 2. Presence of a major risk factor. If Pt has: a)
plus 1&2: HIGH pre-test probability b) plus 1 or 2: INTERMEDIATE pre-test probability c) alone: LOW pre-test clinical
probability There are 2 assays simpliRED & VIDAS. They vary in their sensitivity simpliRED 99% VIDAS 87%. Specificity is
however poor 60-70% so both = high false +ve rates. Basically a -ve d-dimer in Pts who were low (simpliRED) or
intermediate (VIDAS) don't need further imaging. A -ve test isn't useful where the pre-test probability is high (so shouldn't be
done)These pts will need imaging anyway! D-dimer becomes less reliable the longer a pt has been in hospital.
(d) The Pt arrests in front of you. Rhythm is PEA. You start the resuscitate according to current ALS guidelines.What will
you do?
Answer: Need to consider thrombolysis in this young Pt you has likely had a massive P.E. BTS guidelines would
support thrombolysis where clinical probability suggests massive P.E. causing cardiovascular collapse.

8
(e) List the 4 H's of cardiac arrest
Answer: Hypoxia, Hypovolaemia, Hypothermia, Hyperkalaemia/Hypokalaemia, Hypomagnesaemia (metabolic etc)
‫ـــــــــــــــــــــــــــــــــــــــــــــــــــــــــــــــــــــــــــــــــــــــــــــــــــــــــــــــــــــــــــــــــــــــــــــــــــــــــــــــــــــــــــــــــــــــــــــــــــــــــــــــــــــــــــــــــــــــــــــــــــــــــــــــــــــــــــــ‬
19- Question 26 of 50
A 36 yo preg. lady presents suicidal stating that she took an overdose 5 hrs ago. She wants to kill herself & her unborn child

(a) What important things do you need to establish from the Pt?
Answer: The amount taken Is she high risk for toxicity i.e. on any liver enzyme inducing drugs Is she anorexic,
alcoholic, HIV +ve, malnourished (as all of these things lead to a decreased glutathione store).
(b) How will the fact that she is pregnant affect your choice of antidotes?
Answer: It won t; still treat the same. Parvolex & Methionine have no harmful effects to the fetus.
(c) How do you approach the Pt that has taken a staggered overdose of paracetamol?
Answer: You need to base it on the time since the 1st OD, its easy if the Pt has taken >150mg/kg they need to be
treated. If not then you need to take Bl. for paracetamol levels & check U&E, LFTs, clotting, & paracetamol levels. This proves if
any paracetamol has actually been taken, if it has then you need to treat. If in any doubt start TTT & obtain expert advice
(d) List the symptoms of late overdose
Answer: Late presenters are more likely to have taken larger significant overdoses, they may have severe abd. pain &
vomiting which are symptoms of acute liver failure. Take caution when using the normogram as there is insufficient
data on Pts who present >15 hrs after ingestion
(e) Using the modified sad persons scale. what score would mandate a psychiatric opinion?
Answer: 6 - 8
‫ـــــــــــــــــــــــــــــــــــــــــــــــــــــــــــــــــــــــــــــــــــــــــــــــــــــــــــــــــــــــــــــــــــــــــــــــــــــــــــــــــــــــــــــــــــــــــــــــــــــــــــــــــــــــــــــــــــــــــــــــــــــــــــــــــــــــــــــ‬
20- Question 27 of 50
You perform a CXR on a 59 yo man you presents with SOB

(a) Describe what it shows: (fig 1)


Answer: Large Rt sided pleural effusion
(b) He doesn't have heart failure, you decide to do a pleural tap for diagnostic purposes, what do you need to
send samples for?
Answer: Check pH (can use Bl. gas syringe), Send to micro in culture bottles & plain tube for gram stain MC+S also
for AFB cultures. Send plain sample pot to biochemistry for LDH & protein. Send as much as you can in a sterile pot for cytology
(c) The protein is 29g/L, how do you work out if it is a transudate or an exudate?
Answer: Using Light's criteria; effusion is an exudates if it meets 1 of the following criteria: If the fluid protein/serum
protein ratio >0.5 Pleural fluid LDH/serum LDH ratio >0.6 Pleural fluid LDH> 2/3 the upper limit of normal serum LDH
(d) List the 4 most common causes for a pleural effusion in the Uk
Answer: Cardiac failure, pneumonia, malignancy, pulmonary embolus
(e) The pH comes back at 7.16 what will you do?
Answer: Needs an intercostal drain, empyema unlikely to resolve without drainage
‫ـــــــــــــــــــــــــــــــــــــــــــــــــــــــــــــــــــــــــــــــــــــــــــــــــــــــــــــــــــــــــــــــــــــــــــــــــــــــــــــــــــــــــــــــــــــــــــــــــــــــــــــــــــــــــــــــــــــــــــــــــــــــــــــــــــــــــــــ‬
21- Question 28 of 50
A man is brought in who has been involved in a , which makes mattresses. He looks drowsy but is rousable he is
complaining of a headache & feeling dizzy.

(a) You do a Bl. gas, which reveals a CO level of 17%. You are concerned about CO poisoning. What do you
do?
Answer: This represents possible severe poisoning Give high flow O2 Do an ECG check for arrhythmias & or MI If
becomes unconscious the consider IPPV Consider hybebaric O2 if there is a centre nearby
(b) The ABG also showed a profound metabolic acidosis a nurse thinks that is breath smells sweet what do you
consider?
Answer: Although the detection of almond smelling breath is not reliable 50% of people cannot smell it this may
represent cyanide poisoning
(c) What antidotes could you use if your suspicions above are correct?
Answer: Dicolbat edetate or Na thiosulphate
(d) What level of CO can smokers have in normality?
Answer: Around 8% would be a normal level
(e) What must you be careful of when treating this Pt?
Answer: Avoid getting contaminated yourself. Ensure that Protective clothing is worn!
‫ـــــــــــــــــــــــــــــــــــــــــــــــــــــــــــــــــــــــــــــــــــــــــــــــــــــــــــــــــــــــــــــــــــــــــــــــــــــــــــــــــــــــــــــــــــــــــــــــــــــــــــــــــــــــــــــــــــــــــــــــــــــــــــــــــــــــــــــ‬
22- Question 29 of 50
You are asked to see a 44 yo immediately who has a GCS of 7/15.
9
(a) You clear ABC & move to assessing D. His pupils are equal, normal sized & reactive. What do you do? You
have no history, he was found like this
Answer: Need to establish why GCS is 7, need to assess the need for airway protection Check BMG Look for
evidence of opiate use Look for medi alert bracelet Look for signs of head injury Look for any focal neurological
signs suggestive of CVA or SAH Evidence of ETOH? Evidence of any other overdose? Insulin?
(b) You decide to do a Bl. gas. It is normal apart form the glucose reads 1mmol/Litre. What is your
management?
Answer: Due to low GCS likely will not be able to give oral glucose therefore needs IV glucose, current
recommendations are 50mls of 10% glucose (previously 50mls of 50%) Different in different hospitals, author
advocates using 20% glucose. Glucagon 1mg IM/IV or SC Reassess BM after 5 minutes constantly reassess GCS
(c) What risk factors are there giving IV glucose & how can they be minimised?
Answer: Risk of thrombophlebitis, extravasation can cause severe tissue necrosis, can result in loss of limb in
extreme cases. Reduce the risk by using lower concentration of IV glucose.
(d) You find out from Pt s wife that she thinks that he deliberately took an insulin overdose. What will you
do?
Answer: Needs to be managed on ITU/HDU May need to be on a sliding scale for 24 hrs. Hypokalaemia can be
problematic Block excision of the injection site has been used as successful TTT for insulin OD but there is no clear
cut evidence that it works
(e) You reassess but after 15 minutes the GCS is only 8/15. What do you need to consider now?
Answer: Could there be another cause? CVA etc Or might represent development of cerebral oedema due to
hypoglycaemia, which has a high mortality. Will need urgent imaging of the brain
‫ـــــــــــــــــــــــــــــــــــــــــــــــــــــــــــــــــــــــــــــــــــــــــــــــــــــــــــــــــــــــــــــــــــــــــــــــــــــــــــــــــــــــــــــــــــــــــــــــــــــــــــــــــــــــــــــــــــــــــــــــــــــــــــــــــــــــــــــ‬
23- Question 30 of 50
A 72 yo man is phoned through presenting with chest pain, the crew have thrombolysed him as he had ST elevation

(a) Name some different thrombolytics


Answer: Streptokinase, alteplase (rtPA), retaplase (modified rtPA), tenecteplase (mosified rtPA).
(b) Name a some side effects of the first thrombolytic agent
Answer: Allergenic reaction to streptokinase. Causes hypotension. Also can r be used again as antibodies are
produced against it
(c) What are the requirements for thrombolysis?
Answer: >1 mm ST elevation in the limb leads or >2mm in 2 contiguous chest leads or LBBB (with typical M.I.
history NB DOES NOT HAVE TO BE NEW!)
(d) What are the anterior leads?
Answer: V1-V3 = anteroseptal, V2-V4 = anterior, V5-V6 = anterolateral
(e) What is a Rt ventricular infarct? What type of M.I is it likely to occur with? 6. f) How do you diagnose it &
what is it important to treat it with?
Answer: When the Rt ventricle is taken out by an inferior M.I. ST elevation in V1 with inferior M.I. suggest it,
especially if it is greater then in V2 & V3. Answer to f) Answer: Perform ECG with V4R. Ensure that IV fluid is
given to maintain adequate filling Pr. in Rt ventricular failure. 40% of Pts with inferior wall infarctions have Rt
ventricular &/or posterior wall involvement, which predisposes them to more complications & increased mortality
‫ـــــــــــــــــــــــــــــــــــــــــــــــــــــــــــــــــــــــــــــــــــــــــــــــــــــــــــــــــــــــــــــــــــــــــــــــــــــــــــــــــــــــــــــــــــــــــــــــــــــــــــــــــــــــــــــــــــــــــــــــــــــــــــــــــــــــــــــ‬
24- Question 2130
35 yo presents with tingling of the hands & feet, she has been feeling very weak over the last 2/7. Knee reflexes are
absent. There is blurring of the vision & diplopia on lateral gaze. She admits to a recent URTI

(a) Give the nerve roots for the reflexes:


Answer: Jaw: trigeminal V Biceps: The biceps & brachioradialis reflexes are mediated by the C5 & C6 nerve roots
Triceps The triceps reflex is mediated by the C6 & C7 nerve roots, predominantly by C7. Supinator C5-6 Knee: The knee
jerk reflex is mediated by the L3 & L4 nerve roots, mainly L4. Ankle: The ankle jerk reflex is mediated by the S1 nerve root.
(b) What is the most likely diagnosis
Answer: Guillain Barre Syndrome Antecedent illness Up to 2/3 of Pts with GBS report an antecedent illness or event
1-3 wks prior to the onset of weakness. Upper resp. & GIT illnesses are the most commonly reported condit.
Symptoms generally have resolved by the time of medical presentat. for the neurologic condit. Ophthalmoparesis may
be observed in up to 25% of Pts with GBS. The most common limitation of eye movement is from a symmetric palsy
associated with cranial nerve VI Lower extremity weakness usually begins 1st & ascends symmetrically &
progressively over the 1st several days. Upper extremity, trunk, facial & oropharyngeal weakness is observed to a
variable extent. Marked asymmetric weakness calls the diagnosis of GBS into question. Despite frequent complaints

10
of paresthesias, objective sensory changes are minimal. Reflexes are absent or hyporeflexic early in the disease course
& represent a major clinical finding O/E of the Pt with GBS
(c) Give 2 differential diagnoses
Answer: Poliomyelitis, Nutritional neuropathies, Toxic neuropathies (eg, arsenic, thallium, organophosphates, lead),
Multifocal motor neuropathy, Mononeuritis multiplex, Critical illness polyneuropathy, Botulism, Vasculitic
neuropathies, Diphtheritic polyneuritis, Acute myasthenia gravis.
(d) Name three things to do in the ED to get the diagnosis
Answer: Speak to neurologist Spirometry Frequent evaluations of these parameters should be performed at bedside to
monitor respiratory status & the need for ventilatory assistance LP The  in CSF protein is thought to reflect the
widespread inflammatory disease of the nerve roots MRI brain Imaging studies such as MRI or CT scan of the spine
may be more helpful in excluding other diagnoses, such as mechanical causes of myelopathy, than in assisting in the
diagnosis of GBS Nerve conduction studies: EMG studies can be very helpful in the diagnostic workup of Pts with
suspected GBS. Abnormalities in the NCS consistent with demyelination are sensitive & represent specific findings
for classic GBS Basic laboratory studies, such as complete Bl. counts & metabolic panels, are of limited value in the
diagnosis of GBS. They often are ordered, although, to exclude other infectious or metabolic causes of the weakness
(e) What needs to be monitored whet the Pt is admitted?
Answer: FVC to see any deterioration in respiratory function
‫ـــــــــــــــــــــــــــــــــــــــــــــــــــــــــــــــــــــــــــــــــــــــــــــــــــــــــــــــــــــــــــــــــــــــــــــــــــــــــــــــــــــــــــــــــــــــــــــــــــــــــــــــــــــــــــــــــــــــــــــــــــــــــــــــــــــــــــــ‬
25- Question 32 of 50
A 29 yo Nepalese man presented with haemoptysis. He had moved to the UK 2 months previously to train in hotel
management.There was no history of trauma

(a) How could you confirm with that the Bl. is from the lungs & not the stomach?(Give 2 methods)
Answer: Alkaline pH, foaminess, or the presence of pus may sometimes suggest the lungs as the 1ry source of
bleeding rather than the stomach
(b) What is the differential diagnosis?(Give eight)
Answer: Infection(URTI,pneumonia,TB,lung abscess), carcinoma, bronchiectasis, pulmonary oedema, PE, inherited
or acquired coagulation disorder, wegener's granulomatosis, goodpastures syndrome.
(c) How would you investigate this Pt?(Give eight)
Answer: FBC, Coag screen, UE, LFTs, Bl. group & crossmatch, ABG, SpO2, CXR, ECG, Sputum M/C/S & dipstick urine
(d) How would you manage this Pt?(Give four)
Answer: ABCs, O2, suction, face mask, 2 large bore IV cannulae, IV fluids, Bl. transfusion if indicated,correct
coagulopathy, respiratory consultation
(e) How would you define massive haemoptysis & what is its significance?
Answer: Massive hemoptysis is variably defined as expectoration of Bl. exceeding 100 to 600 mL over a 24-hr
period. Although only 5% of haemoptysis is massive some studies report a mortality rate of up to 80% in this group.
Cahill, BC, Ingbar, DH. Massive hemoptysis. Assessment & management. Clin Chest Med 1994; 15:147
‫ـــــــــــــــــــــــــــــــــــــــــــــــــــــــــــــــــــــــــــــــــــــــــــــــــــــــــــــــــــــــــــــــــــــــــــــــــــــــــــــــــــــــــــــــــــــــــــــــــــــــــــــــــــــــــــــــــــــــــــــــــــــــــــــــــــــــــــــ‬
26- Question 33 of 50
A 70 yo man was brought in by his family as he was feeling generally unwell. Bl. investigations revealed a potassium
level of 7.1 mmol/litre.His only medical history was of HTN for which he had recently been started on a medication.
He had no known history of hyperkalaemia

(a) Give a differential diagnosis for hyperkalaemia(Give four)?


Answer: ARF, CRF, potassium sparing diuretics, crush injury, burns, tumor cell necrosis, acidosis from any
cause(potassium cellular shift), drugs such as suxamethonium & beta-blockers, addisons disease, haemolysis of the
sample or a sample taken from a limb with a potassium infusion
(b) What medication for his HTN might this man have been started on which may exacerbate his
hyperkalaemia
Answer: Amiloride or spironolactone
(c) What are the clinical features of this condition?(Name three)?
Answer: Muscle weakness, muscle cramps, paraesthesiae, hypotonia & may cause focal neurological deficits
(d) What ECG features may be present?(Give four)
Answer: Peaked T waves,small broad or absent P waves, widening QRS complex, sine wave pattern QRST,
ventricular tachycardia/ventricular fibrillation.
(e) How would you manage this Pt?
Answer: Confirm result, monitored ECG, 10ml 10% Ca chloride, 10units Actrapid with 50ml of 50% dextrose, 5mg
nebulised salbutamol, IV fluids, treat underlying cause, specialist consultation
‫ـــــــــــــــــــــــــــــــــــــــــــــــــــــــــــــــــــــــــــــــــــــــــــــــــــــــــــــــــــــــــــــــــــــــــــــــــــــــــــــــــــــــــــــــــــــــــــــــــــــــــــــــــــــــــــــــــــــــــــــــــــــــــــــــــــــــــــــ‬

11
27- Question 34 of 50
A 21 yo girl is brought in to the department looking desperately unwell, she has been at an all night rave & has taken
7 ecstasy tablets she is pale & hot but is lucid

(a) She then becomes unresponsive, you fluid resuscitate her & check her BM which is normal. You send off a
full set of investigations, which show the following: INR 3.4, WBC 18.3, Ur 14 Cr 312, temp 38.2, urine dip +ve
for Bl., CK 1203. pH 6.31 What is going on?
Answer: Rhabdodyloysis from raving all night & ecstasy. She has also developed DIC. Acute renal failure
(b) Where should this Pt go & what should be done?
Answer: ITU, Prompt correction of fluid deficits & acidosis are crucial. Will likely need renal support
(c) What electrolytes can easily become deranged & need to be corrected in this condition?
Answer: K+ & Ca2+
(d) Give 3 other causes of the conditon:
Answer: compartment syndrome direct injuries & severe burns exertional: raving, fitting, metabolic disorders:
myxodema, neuroleptic malignant syndrome, myositis due to infection
‫ـــــــــــــــــــــــــــــــــــــــــــــــــــــــــــــــــــــــــــــــــــــــــــــــــــــــــــــــــــــــــــــــــــــــــــــــــــــــــــــــــــــــــــــــــــــــــــــــــــــــــــــــــــــــــــــــــــــــــــــــــــــــــــــــــــــــــــــ‬
28- Question 35 of 50
A 15 yo male presented to the ED with sudden onset of rapid palpitations which were not associated with chest pain
or dizziness.O/E he was well perfused & his Bl. Pr. was 120/80 mmHg. His ECG revealed an SVT.

(a) List 2 forms of TTT which may be attempted?


Answer: Vagotonic manoeuvres, IV adenosine
(b) After TTT the Pt's rest ECG revealed WPW. What are the ECG features of this condition?
Answer: Short PR interval, a wide QRS complex with a slurred upstroke(delta wave).
(c) What is the underlying pathology of this condition?
Answer: An accessort conduction pathway between th atria & ventricles.
(d) What is the definitive TTT in a young Pt with this disorder?
Answer: Radiofrequency ablation of the accessory pathway.
(e) What medication should be avoided in WPW?
Answer: Both digoxin & verapamil are contraindicated in WPW as they  the risk of malignant ventricular agents
‫ـــــــــــــــــــــــــــــــــــــــــــــــــــــــــــــــــــــــــــــــــــــــــــــــــــــــــــــــــــــــــــــــــــــــــــــــــــــــــــــــــــــــــــــــــــــــــــــــــــــــــــــــــــــــــــــــــــــــــــــــــــــــــــــــــــــــــــــ‬
29- Question 36 of 50
You see an 84 yo lady who lives alone at home who is acutely SOB. You suspect that she's in failure The paramedics
have surprisingly good notes on this lady as she was only discharged 2 days ago from a health care for the elderly
ward following a UTI. She was found to be in AF & had an echo, which showed normal lt vent. Funct. with a good EF

(a) On clinical exam. you here basal crepitations, she has a RR of 38 & sats of 92%. The chest x-ray has
widespread air space shadowing with upper lobe diversion. What is going on?
Answer: 2 main options: 1 either has non-cardiogenic pulmonary oedema or 2 has diastolic heart failure
(b) Name 4 causes of acute pulmonary oedema other than heart failure
Answer:  pulm. capillary Pr. (hydrostatic):  lt atrial Pr.: Mitral valve disease, atrial myxoma, arrhythmias.  lt vent.
end diastolic Pr.: Ischaemia, aortic valve disease, cardiomyopathy, uncontrolled HTN, fluid overload, high output
states Neurogenic: IC hge, cerebral oedema, post-ictal HAPE (rare obviously unless been up Everest recently)
Increased pulmonary capillary permeability ARDS Hypoalbuminaemia
(c) Explain the pathophysiology of diastolic heart failure
Answer: Essentially it occurs in the elderly who are hypertensive with LV hypertrophy, the ventricle has impaired
relaxation in diastole this leads to pulmonary oedema. With tachycardia diastolic filling time shortens & as the
ventricle is stiff in diastole left atrial Pr. is increased & pulmonary oedema occurs
(d) How would you manage a Pt in pulm. oedema who you new had a prosthetic mitral valve if they didn't
respond to initial therapy?
Answer: Need to involve cardiologist & cardiothoracic surgeon. Emergency thransthroacic or TOE to confirm
diagnosis of presumed prosthetic valve failure
‫ـــــــــــــــــــــــــــــــــــــــــــــــــــــــــــــــــــــــــــــــــــــــــــــــــــــــــــــــــــــــــــــــــــــــــــــــــــــــــــــــــــــــــــــــــــــــــــــــــــــــــــــــــــــــــــــــــــــــــــــــــــــــــــــــــــــــــــــ‬
30- Question id: 2050
A 66 yo woman presented to the ED with a 3 week history of progressive SOB & purulent cough. She had
tuberculosis treated in East Timor 24 yrs previously. O/E her O2 saturation was 92% on room air.

(a) Describe the main finding in the the CXR shown?Name 2 other findings which are seen in radiographs of Pt
with TB?
12
Answer: There is a Rt upper zone opacity with cavitation. Other findings include hilar adenopathy, sometimes associated with
Rt middle lobe collapse, infiltrates or cavities in the middle or lower lung zones, pleural effusions, solitary nodules.
(b) Besides tuberculosis give a differential diagnosis of 2 other conditions?
Answer: Neoplasm, pneumonia.
(c) What further investigations should be carried out in the ED?(Name three)
Answer: Full Bl. count, urea & electrolytes, r&om sample sputum staining for acid-fast bacilli, & Bl. cultures
(d) How should this lady be managed in the ED?(three steps)
Answer: Isolation with barrier nursing, -ve Pr. room if available, supplemental O2 therapy, & respiratory consultation

(e) Name 2 complications of this condition?


Answer: Haemoptysis, pneumothorax, bronchiectasis, & extensive pulmonary destruction.
‫ـــــــــــــــــــــــــــــــــــــــــــــــــــــــــــــــــــــــــــــــــــــــــــــــــــــــــــــــــــــــــــــــــــــــــــــــــــــــــــــــــــــــــــــــــــــــــــــــــــــــــــــــــــــــــــــــــــــــــــــــــــــــــــــــــــــــــــــ‬
31- Question 38 of 50
A 74 yo man deliberately ingested 20 sulphonylurea tablets

(a) What are the clinical features of sulphonylurea overdose?(Name four)


Answer: Hypoglycaemia(confusion, difficulty speaking, dizziness, hemiparesis, seizures, or coma, anxiety, nausea,
sweating, & palpitations) hypokalaemia
(b) How would you investigate this Pt?(Give four)
Answer: UE, Bl. glucose, check for paracetamol & salicylate level, ECG
(c) How would you manage this Pt?(Give four)
Answer: Observe for at least 24 hrs, oral or IV glucose as needed, correct hypokalaemia, consider octreotide, expert
advice in severe poisoning, & mental health consultation.
(d) How would the management of this Pt be different if he had renal failure?
Answer: Renal failure results in impaired drug clearance & this increases the risk of hypoglycaemia
(e) Why may octreotide be indicated?
Answer: Octreotide blocks pancreatic insulin release.
‫ـــــــــــــــــــــــــــــــــــــــــــــــــــــــــــــــــــــــــــــــــــــــــــــــــــــــــــــــــــــــــــــــــــــــــــــــــــــــــــــــــــــــــــــــــــــــــــــــــــــــــــــــــــــــــــــــــــــــــــــــــــــــــــــــــــــــــــــ‬
32- Question 39 of 50
A 45 yo woman presented with pleuritic chest pain.

(a) What is the differential diagnosis of pleuritic chest pain?(Give four)


Answer: PE, pericarditis, viral pleurisy, Pneumonia, Pneumothorax, Collagen vascular diseases, including systemic
lupus erythematosus, mixed connective tissue disease, & rheumatoid arthritis, drug-induced lupus, inflammatory
bowel disease, familial Mediterranean fever, & radiation pneumonitis
(b) How would you investigate this Pt?(Give four)
Answer: ECG, D-Dimer, CXR, FBC, ABG, BNP, troponin, CTPA, V/Q Scan
(c) What %age of Pts with a PE have abnormal D-Dimer value?(not including sub-segmental PE)
Answer: D-dimer levels are abnormal in 95% of Pts with PE.They are abnormal in only 50% of Pts with subsegmental PE.
(d) If PE is confirmed how would you manage this Pt?(three steps)
Answer: O2, hemodynamic support, analgesia, & anticoagulation
(e) What is the prognosis of untreated PE?
13
Answer: PE is associated with a mortality rate of approximately 30% without TTT, primarily the result of recurrent embolism
‫ـــــــــــــــــــــــــــــــــــــــــــــــــــــــــــــــــــــــــــــــــــــــــــــــــــــــــــــــــــــــــــــــــــــــــــــــــــــــــــــــــــــــــــــــــــــــــــــــــــــــــــــــــــــــــــــــــــــــــــــــــــــــــــــــــــــــــــــ‬
33- Question 40 of 50
A 34 yo gentleman attends with sudden onset of left sided chest pain worse on inspiration. His sats are 94% in room
air. You consider the diagnosis of a spontaneous pneumothorax

(a) What element of the social history that you don't have would support the likelihood of a spontaneous
pneumothorax?
Answer: Smoking the lifetime risk of developing a pneumothorax in healthy smoking men may be as much as 12%
compared with 0.1% in non-smoking men.1
(b) What is a 1ry pneumothrax compared to a 2ry one?
Answer: 1ry pneumothoraces arise in otherwise healthy people without any lung disease. 2ry pneumothoraces arise in
subjects with underlying lung disease.
(c) You perform a CXR which confirms your diagnosis, which 2 features would lead you to considering
aspiration according to current BTS guidelines?
Answer: If the rim was >2cm from the chest wall or if the Pt was breathless as a result of it.
(d) You attempt aspiration but it is unsuccessful, what would you do next?
Answer: Could consider repeat aspiration or if that fails again insert an intercostal drain
(e) Explain why it is crucial to obtain an erect chest x-ray in Pts with a suspected pneumothorax
Answer: On supine CXR lung markings will extend to the chest wall as air in the pleural cavity moves anteriorly. A lateral
or lateral decubitus CXR should be performed if the clinical suspicion of pneumothorax is high, but a PA CXR is normal
(f) What type of x-ray may be of benefit in these cases?
Answer: lateral decubitus film.
(g) In an older Pt with underlying COPD who develops a spontaneous pneumothorax that is 4 cm in size what
condition must you be wary of post aspiration?
Answer: re-explansion pulmonary oedema, this is especially important if the Pt has waited a few days before
seekingmedical attention as the incidence is higher the longer the lung has been collapsed.
‫ـــــــــــــــــــــــــــــــــــــــــــــــــــــــــــــــــــــــــــــــــــــــــــــــــــــــــــــــــــــــــــــــــــــــــــــــــــــــــــــــــــــــــــــــــــــــــــــــــــــــــــــــــــــــــــــــــــــــــــــــــــــــــــــــــــــــــــــ‬
34- Question 41 of 50
A 61 yo lady is sitting in the cubicle area on a trolley, you go to see her & think that she looks unwell, she is sweaty
clammy & tachycardic. You re-check her observations: she has a pulse of 105 sats of 96% in air & BP 0f 145/70. She
is with her partner who says that she fitted earlier & that's why they have come in. You find out that she is an
alcoholic & hasn't drunk for 2 days now

(a) What is going on?


Answer: She appears to be withdrawing from alcohol. Although the differential diagnosis is wide & one needs to
constantly assess & monitor this Pt
(b) How would you treat this situation?
Answer: benzodiazepines normally chlordiazepoxide 20mg but higher doses may be required to control symptoms.
Maximum does of 200mg in 24 hrs. Also IV vitamin complexes (Pabrinex) will need vitamin B co-strong & thiamine.
(c) What is delirium tremens?
Answer: It is a medical emergency occurring in alcohol withdrawal. The Pt may have all the signs of withdrawal but
in addition have hallucinations, sinister delusions, confusion & disorientation. Deaths occur from arrhythmias (2ry to
acidosis, electrolyte disturbances or alcohol related cardiomyopathy).
(d) Explain the type of picture you might see on an ABG of a Pt that you suspect has alcoholic ketoacidosis
Answer: It develops from an alcoholic withdrawing, not eating & vomiting repeatedly. A metabolic acidosis is
normally seen with a high anion gap but the pH is actually variable as vomiting could lead to an alkalosis & they
could have an element of respiratory alkalosis
(e) The magnesium you sent earlier comes back at 0.017 what will you do?
Answer: IV replacement but not too fast (likely chronically low) cannot do too much harm by giving high doses.
good place to start is to give 8mmol of Mg sulphate over 20 min then to give another 8 mmols over 4-6 hrs then re-
checking the level. Ensure that all other electrolytes are checked carefully this Pt should be managed on HDU
‫ـــــــــــــــــــــــــــــــــــــــــــــــــــــــــــــــــــــــــــــــــــــــــــــــــــــــــــــــــــــــــــــــــــــــــــــــــــــــــــــــــــــــــــــــــــــــــــــــــــــــــــــــــــــــــــــــــــــــــــــــــــــــــــــــــــــــــــــ‬
35- Question 42 of 50
A 56 yo lady has taken 56 of her amitriptyline tablets

(a) List features that would commonly be apparent in a Pt who had done this?
Answer: Dry mouth Tachycardia Dry skin Dilated pupils Ataxia Urinary retention Jerky limb movements Coma
(b) What dose is toxic?
14
Answer: When >10mg/kg is taken
(c) What ECG changes can be seen?
Answer: Sinus tachycardia is common, with severe poisoning PR & ORS duration increase. The rhythm can look like
VT as the P waves are superimposed on the preceding T wave & the QRS duration is prolonged. Any arrhythmia can
occur & bradycardia normally indicates a per-arrest scenario
(d) She becomes unconscious & requires mechanical ventilation, whilst being ventilated develops a bizarre
tachyarrhythmia, what do you do?
Answer: Don't treat with antiarrhythmics, instead treat the acidosis & correct hypoxia, use NaHCo3 8.4% (adult 50-100mL IV)
(e) Is there a role for activated charcoal in an amitriptyline overdose?
Answer: Yes it binds it but must ensure that the Pt can protect their own airway & that it's given within an hr or so of presentat.
‫ـــــــــــــــــــــــــــــــــــــــــــــــــــــــــــــــــــــــــــــــــــــــــــــــــــــــــــــــــــــــــــــــــــــــــــــــــــــــــــــــــــــــــــــــــــــــــــــــــــــــــــــــــــــــــــــــــــــــــــــــــــــــــــــــــــــــــــــ‬
36- Question 43 of 50
A 60 yo man presented with SOB. He complained that he was waking at night with SOB & couldn't lie flat.His
previous history was of MI.O/E there was pulmonary rales & mild lower extremity edema

(a) What is the diagnosis?


Answer: Symptomatic left ventricular systolic dysfunction
(b) Name 2 other possible findings O/E in this condition?
Answer: S3 gallop, a decrease in tissue perfusion,pulsus alternans & elevated jugular venous Pr
(c) How would you investigate this Pt?(Name six)
Answer: SpO2, FBC, UE, LFTs, Bl. glucose, thyroid function tests, iron studies , plasma BNP , CXR, ECG
(d) How would you manage this Pt in the ED?(Name four)
Answer: O2 +/- assisted ventilat., IV loop diuretic , vasodilator therapy(NTG), morphine sulfate, +ve inotropic agents
(e) What are the four most common cause of this condition?
Answer: The most common causes of systolic dysfunction are IHD, idiopathic DCM, HTN & valvular disease
‫ـ‬KK‫ ــــــــــــــــــــــــــــــــــــــــــــــــــــــــــــــــــــــــــــــــــــــــــــــــــــــــــــــــــــــــــــــــــــــــــــــــــــــــــــــــــــــــــــــــــــــــــــــــــــــــــــــــــــــــــــــــــــــــــــــــــــــــــــــــــــــــــــ‬37- Question 44 of
50
A 23 yo man presented with a 1 day history of a painful itchy Rt eye. O/E the eye was red & there was a small amount
of white discharge

(a) What pathogens commonly cause bacterial conjunctivitis?


Answer: Staphylococcus aureus, Streptococcus pneumoniae, Haemophilus influenzae, & Moraxella catarrhalis
(b) What virus typically causes viral conjunctivitis?
Answer: Viral conjunctivitis is typically caused by adenovirus
(c) What is the cardinal symptom distinguishing allergic conjunctivitis from a viral aetiology?
Answer: Itching is the cardinal symptom of allergy, distinguishing it from a viral etiology, which is more typically
described as grittiness, burning, or irritation. Eye rubbing can worsen symptoms. Pts with allergic conjunctivitis often
have a history of atopy, seasonal allergy, or specific allergy (eg, to cats).
(d) How is the vision affected in conjunctivitis?
Answer: The diagnosis can be made in a Pt with a red eye & discharge only if the vision is normal & there is no
evidence of keratitis, iritis, or angle closure glaucoma.
(e) Name 4 'red flags' which should alert the clinician that there may be a more serious underlying condition
than simple conjunctivitis?
Answer: Reduction of visual acuity, ciliary flush: A pattern of injection in which the redness is most pronounced in a
ring at the limbus (the limbus is the transition zone between the cornea & the sclera), photophobia, severe FB sensate.
that prevents the Pt from keeping the eye open, corneal opacity, a fixed pupil & severe headache with nausea.
‫ـــــــــــــــــــــــــــــــــــــــــــــــــــــــــــــــــــــــــــــــــــــــــــــــــــــــــــــــــــــــــــــــــــــــــــــــــــــــــــــــــــــــــــــــــــــــــــــــــــــــــــــــــــــــــــــــــــــــــــــــــــــــــــــــــــــــــــــ‬
38- Question id: 2128
A 55 yo man is on the observation ward/ CDU after a minor head injury. You are urgently called to see him as the
nurses feel he is acutely withdrawing from alcohol

(a) List 6 features that would support this diagnosis


Answer: Tremor, anxiety, hallucinations, sweaty, agitation, confusion
(b) What are the risk factors for severe withdrawal to occur?
Answer: Fitting, electrolyte disturbances, arrythmias, infection, CVS collapse, hpoglycaemia, alcoholic ketoacisosis
(c) List the medical management of severe alcohol withdrawal
Answer: See figure 1 next page
(d) What are the diagnostic features of Wernicke-Korsakoff syndrome & how might it be precipitated?

15
Answer: Ophthalmoplegia Ataxia Mental status changes nystagmus Mental status changes Apathy, indifference,
paucity of speech Hallucination, agitation Confabulation: Pt fills in gaps of memory with data that can be recalled at
that moment. Debate remains as to whether this action represents a deliberate attempt by the Pt to hide his memory
deficits or if it is an unconscious mechanism
‫ـ‬KK‫ــــــــــــــــــــــــــــــــــــــــــــــــــــــــــــــــــــــــــــــــــــــــــــــــــــــــــــــــــــــــــــــــــــــــــــــــــــــــــــــــــــــــــــــــــــــــــــــــــــــــــــــــــــــــــــــــــــــــــــــــــــــــــــــــــــــــــــ‬39- Question 46 of
50
A 36 yo man was brought to the ED because his mother had found him that morning in his bedroom confused &
drowsy. She had heard him vomiting during the night.She had found an empty packet of anti-histamines by his
bedside. He had a medical history of alcoholism & IV drug abuse. O/E he was febrile at 38 degrees , his HR was 112,
his SpO2 was 99% on RA & his BP was 140/90. His pupils were dilated & his skin was hot to touch. He was
disorientated & answering questions inappropriately with incoherent speech

(a) What is the most likely diagnosis? What other important diagnosis should be considered?
Answer: Most likely diagnosis-Cholinergic toxicity. Other important diagnosis to consider is sepsis or meningitis
given fever,tachycardia & disorientation
(b) What other points may be found O/E of the Pt?(Three points)
Answer: Decreased or absent bowel sounds, "Red as a beet" (cutaneous vasodilation), "Dry as a bone" (anhidrosis),
"Hot as a hare" (anhydrotic hyperthermia), "Blind as a bat" (nonreactive mydriasis), "Mad as a hatter" (delirium;
hallucinations),& "Full as a flask" (urinary retention)
(c) How would you investigate this Pt?(Give four points)
Answer: Bl. glucose, FBC, UE, ECG, Paracetamol level
(d) How would you manage this Pt?(Give five points)
Answer: Stabilization of the ABC. Pts should have IV access, O2, cardiac monitoring, & continuous pulse oximetry.
Consultat. with a medical toxicologist or regional poison center.Agitation & seizures may be treated with benzodiazepines.
Hyperthermia should be treated in typical fashion. Charcoal should be withheld in Pts who are sedated
(e) What is the anidote for this condition?
Answer: Physostigmine.

16
Fig. 1 of Q 38
‫ـ‬KK‫ــــــــــــــــــــــــــــــــــــــــــــــــــــــــــــــــــــــــــــــــــــــــــــــــــــــــــــــــــــــــــــــــــــــــــــــــــــــــــــــــــــــــــــــــــــــــــــــــــــــــــــــــــــــــــــــــــــــــــــــــــــــــــــــــــــــــــــ‬ 40- Question 47 of
50
A-35 yo man who has recently come to the UK from Liberia (West Africa) Presents with a 2 day history of worsening
symptoms of joint pains, fever, chills, rigors & waking up drenched in sweat. O/E he is febrile at 40 C & he is
complaining of retrosternal pain

(a) What initial investigations should you carry out?


Answer: Bl. cultures (all febrile Pts)FBC,U&E Bl. films, malaria antigen dip stick testing LFTs Clotting Urinalysis CXR
(b) The malaria screen is -ve what do you do?
Answer: Contact an expert centre in infections/tropical diseases & arrange transfer. Consider the possibility of viral
haemorrhagic fevers i.e. Lassa fever which is endemic to Liberia. 1 malaria screen that is -ve doesn t necessarily
mean that malaria is not present
(c) Name at least 2 subtypes of malaria & describe which one is potentially fatal?
Answer: P. Falciparum (malignant) potentially fatal P. Ovale P. Vivax P. malariae
(d) Describe features that would make this a severe case of falciparum malaria?
Answer: Renal failure Acidosis Coagulopathy Hypoglycaemia Coma WBC >12 Hb <7 Pulmonary oedema Retinal
haemorrhages > than 2% Schizonts on Bl. Film
(e) What is the first line TTT of falciparum malaria?
Answer: Parenteral. If the Pt is seriously ill or unable to take tablets, quinine should be given by IVI. The adult
dosage regimen for quinine by infusion is:loading dose(2) of 20 mg/kg(3) (up to maximum 1.4 g) of quinine salt(1)
infused over 4 hrs then 8 hrs after the start of the loading dose, maintenance dose of 10 mg/kg(4) (up to maximum 700
mg) of quinine salt(1) infused over 4 hrs every 8 hrs (until Pt can swallow tablets to complete the 7-day course
together with or followed by either doxycycline or clindamycin as above). Specialist advice should be sought in

17
difficult cases (e.g. very high parasite count, deterioration on optimal doses of quinine, infection acquired in quinine-
resistant areas of south east Asia) because IV artesunate may be available for named-Pt use
‫ـ‬KK‫ ــــــــــــــــــــــــــــــــــــــــــــــــــــــــــــــــــــــــــــــــــــــــــــــــــــــــــــــــــــــــــــــــــــــــــــــــــــــــــــــــــــــــــــــــــــــــــــــــــــــــــــــــــــــــــــــــــــــــــــــــــــــــــــــــــــــــــــ‬41- Question 48 of
50
A 39 yo woman presented with a generalised macular erythematous rash. She had been feeling unwell for the previous 2 days but
the rash had relatively quickly on the day of presentat.The wk before presentat. she had been started on a new medicat. for a
chronic medical condit. She also complained of difficulty eating 2ry to oral pain. O/E she was febrile & had a generalised
erythematous macular rash. She had multiple oral ulcerated areas. A diagnosis of stevens-johnsons syndrome was made

(a) What is the differential diagnosis of this condition?(Name three)


Answer: Erythematous drug eruptions, pustular drug eruptions, phototoxic eruptions, staphylococcal scalded skin
syndrome (SSSS) & toxic shock syndrome (TSS)
(b) Name 2 medications which commonly cause this condition
Answer: Anti-gout agents (especially allopurinol), antibiotics (sulfonamides > penicillins > cephalosporins), antipsychotics
& anti-epileptics (including carbamazepine, dilantin, lamotrigine & phenobarbital) & analgesics & NSAID
(c) How would you investigate this Pt?(Name three appropriate investigations)
Answer: FBC with differential cell count, LFTs (Mild elevations in serum aminotransferase levels (2 to 3 times
normal) are present in about 1/2 of Pts), skin biopsy,CXR (condition may be caused by infect. as well as medication).
(d) How would you manage this Pt? (Five management steps)
Answer: Prompt removal of offending agent. Supportive care includes wound care, fluid & electrolyte management,
nutritional support, ocular care,oral care, temp. management, & monitoring for & TTT of superinfections. Consider
transfer to a burn unit as massive loss of the epidermis may occur
(e) What is the prognosis of this condition?
Answer: Mortality of 1 to 3%.
‫ـ‬KK‫ ــــــــــــــــــــــــــــــــــــــــــــــــــــــــــــــــــــــــــــــــــــــــــــــــــــــــــــــــــــــــــــــــــــــــــــــــــــــــــــــــــــــــــــــــــــــــــــــــــــــــــــــــــــــــــــــــــــــــــــــــــــــــــــــــــــــــــــ‬42- Question 49 of
50
A 18 yo girl was brought to the ED after overdosing on her mothers iron tablets

(a) What are the clinical features of iron toxicity?


Answer: Nausea, vomiting, diarrhoea, abd. pain, black stools, hyperglycaemia, shock, hypoglycaemia, jaundice,
metabolic acidosis, hepatic encephalopathy, & renal failure & coma.
(b) How would you investigate this Pt?(Give four)
Answer: Check serum iron, FBC, UE, glucose, ABG
(c) How would you manage this Pt?
Answer: Charcoal doesn't absorb iron, gastric lavage if within 1 hr, expert advice, supportive measures, desferrioxamine
(d) What are the complications of desferrioxamine?(Give four)
Answer: Iron-desferrioxamine complex makes the urine orange or red, desferrioxamine causes hypotension if infused
too rapidly, rashes, anaphylaxis, pulmonary oedema, ARDS
(e) What gastrointestinal sequelae may survivors exhibit? (Give one)
Answer: Gastric strictures & pyloric obstruction
‫ـ‬KK‫ ــــــــــــــــــــــــــــــــــــــــــــــــــــــــــــــــــــــــــــــــــــــــــــــــــــــــــــــــــــــــــــــــــــــــــــــــــــــــــــــــــــــــــــــــــــــــــــــــــــــــــــــــــــــــــــــــــــــــــــــــــــــــــــــــــــــــــــ‬43- Question 50 of
50
A 46 yo man was came into his house after being in the garden playing with his dog. Over the course of an hr he
developed facial swelling & SOB resulting in his wife calling the ambulance

(a) Name three common causes of anaphylaxis?


Answer: Drugs, vaccines, bee/wasp stings, nuts, shellfish, strawberries, wheat, latex
(b) Pts taking what medication may have particularly severe symptoms?
Answer: B-Blockers.
(c) What are the basic initial management steps with this man?
Answer: Cardiac monitoring, SpO2 monitoring, IV access, supplemental O2, monitor BP.
(d) What dose of adrenaline should be given if there is shock, airway swelling or respiratory difficulty?
Answer: 0.5mg(0.5ml of 1:1000) IM. Repeat in five minutes if there is no improvement
(e) Caution should be observed in administering adrenaline if the Pt is taking what medications?(Name 2)
Answer: TCAs, MAOIs, B-Blockers.
‫ـــــــــــــــــــــــــــــــــــــــــــــــــــــــــــــــــــــــــــــــــــــــــــــــــــــــــــــــــــــــــــــــــــــــــــــــــــــــــــــــــــــــــــــــــــــــــــــــــــــــــــــــــــــــــــــــــــــــــــــــــــــــــــــــــــــــــــــ‬
44- Question 3 of 20
A 73 yo man presents to the ED acutely SOB. He is found to be in fast AF with a PR of 153. He suffered 3 M.I.s last yr & has a
10 yr history of HTN. His BP was 100/53 & auscultat. of his chest revealed insp. crackles & exp. wheeze with a  JVP
18
(a) What should happen to this Pt according to resus council guidelines?
Answer: Attempt synchronised shocks up to 3 attempts. According to tachyarrhythmia guidelines the Pt is unstable &
requires electrical cardioversion
(b) Which of the following drugs would be the best for restoring sinus rhythm? 1. IV digoxin 2. IV amiodarone
3. IV flecanide 4. IVesmolol 5. IVdofetolide
Answer: IV amiodarone. - digoxin & esmolol wouldn�t restore sinus ryhtm, flecanide is contraindicated in heart
failure & amiodarone is the least -vely ionotropic Dofetolide = good option but amiodarone = better
(c) What type of Pt should one avoid digoxin in?
Answer: Renal Pts/ Pts with known renal failure.
(d) List the investigations that you would want to help you work out why the Pt was in fast AF.
Answer: CXR, MSU, Bl. cultures, ECHO,
‫ـ‬KK‫ ــــــــــــــــــــــــــــــــــــــــــــــــــــــــــــــــــــــــــــــــــــــــــــــــــــــــــــــــــــــــــــــــــــــــــــــــــــــــــــــــــــــــــــــــــــــــــــــــــــــــــــــــــــــــــــــــــــــــــــــــــــــــــــــــــــــــــــ‬45- Question 4 of
20
A 68 yo lady is brought in to ED resus at 06:00 am. The crew say that she had a short history of waking up & being
unable to get her breath�. O/E she has a RR of 36, sats of 89% on a re-breathe mask

(a) The ED in which you work has the resources to measure BNP or N-terminal- ProBNP. Is there any
diagnostic value in measuring this?
Answer: Yes, BNP> 500 pg/dL or NT- proBNP> 1000 pg/dL makes acute heart failure syndrome likely (approx +ve
likelihood ratio [LR+} = 6)
(b) The Pt deteriorates & you start her on CPAP 5 mmHg. Please explain briefly how CPAP works
Answer: 1. CPAP splints the alveoli open, thereby preventing alveolar collapse & allowing unimpeded alveolar ventilat.
(Recruits alveoli). 2. 2. CPAP also ↓ preload & afterload, improves lung compliance,  FRC & ↓ work of breathing.
3. NIV ↓ trans-diaphragmatic Pr., Pr. time index of resp. muscles & diaphragmatic EMG activity. This leads to an  in TV,
↓ in RR &  in minute ventilat. Also overcomes the effect of intrinsic PEEP. CPAP ↓ lt vent. transmural Pr. & therefore 
CO. Hence it is a very effective for TTT of pulm. oedema. Causes  in intrathoracic Pr. therefore improving CO
(c) Her Bl. Pr. is 113/56 & decides to treat her with a nitrate infusion. Is there any evidence for or against
giving diuretics please discuss:
Answer: Yes but always in combination with nitrates. There is also some evidence to move away from diuretic
montherapy as it is unlikely to prevent the need for tracheal intubation & can worsen renal function which has been
shown to increase mortality.1 The advice is to use it in combination & to use them �judiciously
(d) Explain the mechanism behind how diuretics work in the acute management of heart failure & how this fits
in with the pathogenesis of acute pulmonary oedema
Answer: They work via venodilatat. The other acute HF syndromes (pulm. edema, HTN crisis & exacerbated HF) are
caused by a combinat. of progressive excessive vasoconstrict. superimposed on ↓ lt vent. functional reserve. The impaired
cardiac power & extreme vasoconstrict. induce a vicious cycle of afterload mismatch resulting in a dramatic ↓ of CO &
 lt vent. end diastolic Pr., which is transferred backwards to the pulm. capillaries yielding pulm. oedema. Therefore,
the immediate TTT of these acute HF syndromes should be based on the administrat. of strong, fast-acting IV
vasodilators such as nitrates or nitroprusside. After initial stabilizat., therapy should be directed at ↓ recurrent episodes
of acute HF, by prevention of repeated episodes of excessive vasoconstrict. along with efforts to optimize cardiac funct.
(e) You decide to insert a central line as the Pt has very poor peripheral access. You insert a left sided internal
jugular line as there was some local scar tissue on the Rt. When you get the chest x-ray to confirm position you
see the film shown (see fig 1). Can the line be used? What would you want to do prior to using it?
Answer: It is actually venous, it's in a low brachiocephalic trunk but it doesn't look like it. You would want to check
the length of line inserted should be at least 14 cm. Aspirate all lumens & run it through a Bl. gas analyser to confirm
that it is venous Bl.. Attach it to a CVP monitor & transducer the line to look for a venous waveform. If further doubt
perform a venogram rarely the anatomy is unusual like in this case
‫ـ‬KKKK‫ ــــــــــــــــــــــــــــــــــــــــــــــــــــــــــــــــــــــــــــــــــــــــــــــــــــــــــــــــــــــــــــــــــــــــــــــــــــــــــــــــــــــــــــــــــــــــــــــــــــــــــــــــــــــــــــــــــــــــــــــــــــــــــــــــــــــــــــ‬46- Question id:
2110
The following ECG was recorded on a 67 yo male in the ED. He was sweaty & clammy & felt SOB but said that he
had no chest pain although he described discomfort in his mouth & neck

(a) What changes are shown in yellow & blue, what is the diagnosis?
Answer: Anterior M.I. yellow = ST segment elevation most pronounced in the anterior leads V1-V4. Blue =
reciprocal changes in the inferior leads.

19
(b) Which coronary vessel is likely to have been occluded?
Answer: Likely left LAD to be exact.
(c) Name 3 conditions that could mimic the picture shown above
Answer: Pericarditis, trauma to the myocardium, WPW, hyperkalaemia, pneumothorax, cardiac amyloid/sarcoid,
cardiac tumours, cardiomyopathy, LBBB, LVH or RVH, pancreatitis
(d) When do troponin levels rise post M.I.? How long do they remain elevated for?
Answer: They start to rise 3 hrs post M.I. but peak at 24-48 hrs they can remain elevated for 7-14 days
(e) Give 3 contraindications to thrombolysis, (appreciating that most are relative, choose ones where you would
be very hesitant to administer thrombolytic agents)
Answer: Arterial or major surgery within 4 wks Previous hgic stroke Prolonged CPR Pregn. Possible aortic dissect. Severe HTN
‫ـــــــــــــــــــــــــــــــــــــــــــــــــــــــــــــــــــــــــــــــــــــــــــــــــــــــــــــــــــــــــــــــــــــــــــــــــــــــــــــــــــــــــــــــــــــــــــــــــــــــــــــــــــــــــــــــــــــــــــــــــــــــــــــــــــــــــــــ‬
47- Question 7 of 20
A 69 yo man presented with a painful swollen Rt knee which had come on insidiously over the course of the previous
48 hrs. He had no history of joint disease. There was no history of trauma. His backgound was of type II DM & was
on warfarin for a prosthetic heart valve.O/E there was an effusion with restricted range of movement

(a) Give a differential diagnosis of four conditions?


Answer: Infective arthritis, hemorrhagic effusions, noninflammatory effusions (osteoarthritis), & inflammatory
effusions (RA, gout, pseudogout)
(b) Name three characteristics of normal synovial fluid?
Answer: Highly viscous, clear, essentially acellular, protein concentration approximately one-third that of plasma , &
glucose concentration similar to that in plasma
(c) Approximately what is the cell count in bacterial joint infections?
Answer: Bacterial joint infections typically are purulent with leukocyte counts (most of which are neutrophils) of
50,000 to 150,000 cells/mm3
(d) What is seen in the synovial fluid in acute crystal-induced synovitis?
Answer: MonoNa urate (MSU) crystals(Gout) & Ca pyrophosphate dihydrate (CPPD) crystals(pseudogout)
(e) Name 2 conditions in which an eosinophilia may be prominent in the synovial fluid?
Answer: Eosinophilia in the synovial fluid suggests parasitic infection, allergy, neoplasm, or Lyme disease.
‫ـــــــــــــــــــــــــــــــــــــــــــــــــــــــــــــــــــــــــــــــــــــــــــــــــــــــــــــــــــــــــــــــــــــــــــــــــــــــــــــــــــــــــــــــــــــــــــــــــــــــــــــــــــــــــــــــــــــــــــــــــــــــــــــــــــــــــــــ‬
48- Question 10 of 20
A 21 yo university student comes in having ingested is Aunties complete supply of digoxin tablets. He is fully
conscious but has a rate of 38 b.p.m

(a) What is your initial management?


Answer: ABCDE, ECG- IV access & atropine- in increments start with 0.5mg
(b) Is digoxin +vely or -vely ionotropic?
Answer: +vely
(c) Is there a role for charcoal in this Pt?
Answer: Yes- give it down an NG tube
(d) Assuming your initial TTT works where should he be admitted?
Answer: CCU
(e) After 20 minutes he starts to feel dizzy & his HR drops to 36 b.p.m what is the next step in management?
Answer: Digoxin specific fab fragment (Digibind)
(f) The Pt gets worse & the monitor shows VT, he has a pulse but his BP is 85/49. What will you do now?

20
Answer: He is haemodynamically unstable & has a VT due to digoxon toxicity. DC cardioversion is relatively
contraindicated here unless all other measures have been exhausted. The most useful drugs in this setting are
lignociane & phenytoin. Amiodarone would increase digoxin levels & is contraindicated
‫ـــــــــــــــــــــــــــــــــــــــــــــــــــــــــــــــــــــــــــــــــــــــــــــــــــــــــــــــــــــــــــــــــــــــــــــــــــــــــــــــــــــــــــــــــــــــــــــــــــــــــــــــــــــــــــــــــــــــــــــــــــــــــــــــــــــــــــــ‬
49- Question id: 2092
A 76 yo gentleman with prostate cancer comes in with severe back & lt leg pain. He also cannot pass urine. & is off
legs & confused

(a) What systems must you examine carefully?


Answer: He needs everything examining but careful attention should be paid to lower limb neurological exam..
(cauda equina) PR is absolutely crucial (cadua equina & is the rectum full of hard stool?) Mini mental state exam.
(level of confusion & cause?)
(b) What 2 investigations are critical?
Answer: Serum Ca Radiographs of the left leg Bladder scan
(c) How do you manage his pain? At home he takes MST 60mg BD & oramorph 10mg PRN?
Answer: In the acute setting IV morphine but consider the dose carefully, will need to be titrated. He may well be
fairly opiate tolerant.
(d) Some lab results become available Hb- 9.5 WBC 14.8 Plt 93 Na 146 K+ 3.6 Ur 12 Cr 345 Ca2+ 3.9 What
concerns you & how would you manage these results?
Answer: WBC increasing Ca2 increasing & ARF! All concerning. Initial therapy is going to be around rehydration &
careful monitoring of ARF. Best managed on medical HDU. Decisions about filtration will need to be had.
(e) The radiograph of the left femur is shown (fig 1). Describe what it shows
Answer: A pathological fracture, mid-shaft with angulation & minimal displacement.
‫ـــــــــــــــــــــــــــــــــــــــــــــــــــــــــــــــــــــــــــــــــــــــــــــــــــــــــــــــــــــــــــــــــــــــــــــــــــــــــــــــــــــــــــــــــــــــــــــــــــــــــــــــــــــــــــــــــــــــــــــــــــــــــــــــــــــــــــــ‬
50- Question 12 of 20
A 58 yo man presents to the ED with retrosternal chest pain for the last 2 hrs. He appears unwell & is sweaty. He has
smoked for the last 20 yrs but has no significant medical history of note. HR is 110 & Bp is 82/45. His JVP is raised & he
has an audible S3, he also has creps to the midzones of the lungs. ECG reveals St elevat. in V1-V4 & depression in II,III & aVF

(a) What 2 oral medications should he be given immediately?


Answer: Asprin & Clopidogrel 300mg each.
(b) What has occurred?
Answer: Large anterior M.I. complicated by cardiogenic shock
(c) What investigations would be very helpful in this case?
Answer: CXR- To look for widening of the mediastinum to help rule out dissection (although NB that it doesn�t rule
it out) Also bedside ECHO to look for pericardial tamponade.
(d) How should this Pt be managed?
Answer: IV vasopressors to treat the shock & intra-aortic balloon pump also reperfusion of the coronary arteries via
PCI. The SHOCK trial showed that emergently revascularisat. was better than thrombolysis with regard to improving mortality
‫ـــــــــــــــــــــــــــــــــــــــــــــــــــــــــــــــــــــــــــــــــــــــــــــــــــــــــــــــــــــــــــــــــــــــــــــــــــــــــــــــــــــــــــــــــــــــــــــــــــــــــــــــــــــــــــــــــــــــــــــــــــــــــــــــــــــــــــــ‬
51- Question id: 4519
A 68 yo man presents to the department with a feeling of being generally unwell & weak. His wife tells you that he
collapsed earlier this morning but he is denying any such thing. He has lost 6kg in weight over the last 3 months. Abd.
exam. reveals some left sided loin tenderness. Urinalysis reveals Bl. 4+, protein 2+.

(a) You do a chest x-ray as part of your collapse? Cause work up & find the following findings (see figure 1).
What does the radiograph show? List a few differential diagnoses for this picture.
Answer: Cannonball metastasis- could be from renal, testicular, colon, osteosarcoma.
21
(b) The Pt's haemoglobin is 19g/dL what could be the cause of this?
Answer: With a diagnosis of renal cell carcinoma from the history given the relative polycythemia could be due to an 
amount of circulating rennin. Renal tumours often secrete rennin & ertythropoetin along with other peptide hormones
(c) What lab tests are especially important in this case?
Answer: Ca level- could be raised & need treating. U&E- again could show decreased renal function. LFTs; any
evidence of liver involvement
(d) Given the likely diagnosis what is the management & prognosis for this Pt?
Answer: Very poor. If it has spread metastatically to other organs, the 5-yr survival rate is<5 %. Management would still be
surgical to remove the tumour from the lt kidney if the Pt was fit enough for surgery as removal of the 1ry tumour has been
shown to improve survival & cause regression of the metastasis. Also likely to go on to have palliative chemo & radiotherapy

‫ـــــــــــــــــــــــــــــــــــــــــــــــــــــــــــــــــــــــــــــــــــــــــــــــــــــــــــــــــــــــــــــــــــــــــــــــــــــــــــــــــــــــــــــــــــــــــــــــــــــــــــــــــــــــــــــــــــــــــــــــــــــــــــــــــــــــــــــ‬
52- Question 14 of 20
A 46 yo factory worker comes in with chest pain that started yesterday after some heavy lifting. His ECG shows T
wave inversion in the lateral leads & his 12 hr troponin came back at 0.08. He is pain free when you see him

(a) What does this represent?


Answer: troponin <0.1 is not an NSTEMI but > 0.03 may well represent unstable angina
(b) What does this Pt need?
Answer: Needs to be admitted. He needs an aniogram. In Pt exercise stress testing should be avoided in view of the
abnormal troponin
(c) List 3 causes of a raised troponin other than myocardial ischaemia?
Answer: Renal failure, infection- sepsis, pulmonary embolism, myocarditis/Pericarditis, prolonged tachycardia,
(d) The Pt was sent home on asprin, clopidogrel & sivastatin. What other medicat. should the Pt receive to improve his
prognosis?
Answer: ACEi/angiotensin receptor blocker. The HOPE study looked at the role of ACEi in high risk populations
without any evidence of left ventricular dysfunction
(e) How long does he effect of clopidogrel last for after it is discontinued?
Answer: The life of the platelet as it irreversibly changes the platelets ability to aggregate- 10-14 days. It stops ADP
from binding to its receptor on the platelet surface
‫ـــــــــــــــــــــــــــــــــــــــــــــــــــــــــــــــــــــــــــــــــــــــــــــــــــــــــــــــــــــــــــــــــــــــــــــــــــــــــــــــــــــــــــــــــــــــــــــــــــــــــــــــــــــــــــــــــــــــــــــــــــــــــــــــــــــــــــــ‬
53- Question 15 of 20
A 26 yo Pt was admitted via ED with acute asthma for the 5th time in the last 2 yrs. The Pt had recently had
aminophyline added to her inhaled therapy, which consisted of seretide 500 & salbutamol. On admission she was
unwell with sats of 88% in air & a RR of 44. CXR revealed consolidate. at the Rt base. She was started on erythromycin as
she had a penicillin allergy. She improved but 2 days later suffered 3 grand mal seizures & needed to be ventilated on ITU

(a) What step is this Pt on with regard to her asthma management according to BTS guidelines?
Answer: 4
(b) What is Seretide a combination of?
Answer: Salmeterol (LABA) & Fluticasone (steroid)
(c) What do you think might be the major problem with this Pt's asthma?
22
Answer: Poor compliance
(d) The Pt had no previous history of fitting from the following options which do you think was the cause of the
seizures & why? 1. Hypoxia 2. Meningitis 3. Benign IC HTN 4. React. to erythromycin 5. Theophylline toxicity 6.
Herpes encephalitis
Answer: Theophylline toxicity: the erythromycin inhibits the metabolism of theophylline therefore potentiating its effects
(e) What is the cross over for penicillin allergic Pts when considering giving cephalosporins?
Answer: Quoted as 10%
‫ـــــــــــــــــــــــــــــــــــــــــــــــــــــــــــــــــــــــــــــــــــــــــــــــــــــــــــــــــــــــــــــــــــــــــــــــــــــــــــــــــــــــــــــــــــــــــــــــــــــــــــــــــــــــــــــــــــــــــــــــــــــــــــــــــــــــــــــ‬
54- Question 16 of 20
A 38 yo samoan lady presented to the ED with an ulcer on the lateral aspect of her Rt small toe & a surrounding
cellulitis. She had a one yr history of NIDDM & was prescribed metformin but had not been taking this medication.

(a) How would you investigate this Pt?(Four points)


Answer: FBC, UE, Bl. cultures, swab from lesion, X Ray foot
(b) How would you manage this Pt?(Four points)
Answer: IV antibiotics, analgesia, subcutaneous insulin sliding scale, IV fluids, endocrinology consultation
(c) Name three categories of bacteria which may be causing infection in this Pt?
Answer: Aerobic gram-+ve organisms , gram--ve organisms , & anaerobic organisms.
(d) Name four factors which make Pts with diabetes at high risk for foot infections?
Answer: Sensory neuropathy(which causes a decreased appreciation of temp. & pain) motor neuropathy(which can
cause foot deformities), autonomic neuropathy(which can cause decreased sweat & sebaceous gl& secretion resulting
in dry, cracked skin), peripheral arterial disease(which can reduce the Bl. supply needed for healing of ulcers &
infections) & hyperglycemia(which impairs neutrophil function & defects in host defenses)
(e) Name a complication of this condition?
Answer: Osteomyelitis, & systemic infection
‫ـــــــــــــــــــــــــــــــــــــــــــــــــــــــــــــــــــــــــــــــــــــــــــــــــــــــــــــــــــــــــــــــــــــــــــــــــــــــــــــــــــــــــــــــــــــــــــــــــــــــــــــــــــــــــــــــــــــــــــــــــــــــــــــــــــــــــــــ‬
55- Question 17 of 20
A 45 yo man has now been in the ED for 2 hrs he is profoundly unwell. He presented in what appeared to be septic
shock & has been treated as such. He was febrile 39.2, tachycardic with a BP of 67/40. There is no history available
about what happened

(a) As yet no source for sepsis has been found. You do a tox screen which is also -ve. List the DD apart from
sepsis
Answer: MDMA(ecstasy), Thyrotoxic storm, Malignant hyperthermia, Heat stroke (malignant hyperpyrexia), EBV,
Serotonin syndrome.
(b) TSH is 8 & T4 is 20. What will you do now?
Answer: Treat as a thyrotoxic crisis
(c) List 4 precipitants of a crisis such as this?
Answer: Thyroid surgery Withdrawal of antithyroid drugs Iodinated contrast dyes Thyroid palpation Sepsis P.E.
DKA Trauma or emotional stress
(d) What would giving salicylates do?
Answer: Make it worse by displacing the T4 from thyroid binding globulin (TBG)
(e) management steps?
Answer: CVP & accurate fluid resuscitation B-blockers if no contraindicate. Active cooling techniques Treat any
infect. High dose antithyroid drugs Propylthiouricil is better than carbimazole Hydrocortisone inhibits the conversion
of T4-T3 Monior glucose levels
‫ـــــــــــــــــــــــــــــــــــــــــــــــــــــــــــــــــــــــــــــــــــــــــــــــــــــــــــــــــــــــــــــــــــــــــــــــــــــــــــــــــــــــــــــــــــــــــــــــــــــــــــــــــــــــــــــــــــــــــــــــــــــــــــــــــــــــــــــ‬
56- Question 18 of 20
The nurse in charge takes a phone call from the ambulance staff who are en route to the ED with a 60 yo woman who
has arrested. She had called the ambulance as she had chest pain but had arrested soon after the ambulance staff got to
her home.They arrive in the ED. The Pt had received CPR & has IV access but hasn't received any medicat. thus far.

(a) What is the first step in the management if this Pt is in VF?


Answer: Give one shock. Resume CPR immediately after the shock.
(b) After this step how many cycles of CPR should be given before the next rhythm check?
Answer: Give five cycles of CPR & then check rhythm.
(c) After the cycles of CPR there is another rhythm check & the Pt is still in VF. What are the 3 next
management steps?
Answer: Give 1 shock. Resume CPR. Give adrenaline 1mg IV/IO,repeat every 3-5 minutes.

23
(d) What anti-arrhythmic medication should be given during CPR?
Answer: Amiodarone 300mg IV/IO once, then consider additional 150mg IV/IO.
(e) If the Pt is in torsades de pointes what medication should be given & at what dose?
Answer: Magnesium, loading dose 1 to 2g IV/IO.
‫ـــــــــــــــــــــــــــــــــــــــــــــــــــــــــــــــــــــــــــــــــــــــــــــــــــــــــــــــــــــــــــــــــــــــــــــــــــــــــــــــــــــــــــــــــــــــــــــــــــــــــــــــــــــــــــــــــــــــــــــــــــــــــــــــــــــــــــــ‬
57- Question 2080
A 35 yo man who has a known personality disorder says that he has taken 45 300mg asprin tablets. He is sweating
profusely & is agitated, he has been vomiting & says that his ears will not stop ringing

(a) Explain the results of the ABG: pH 7.36 paO2 12.3 paCO2 2.8 BE -16 HCO3- 17
Answer: Shows a mixed picture, shows a mixed metabolic acidosis with respiratory alkalosis which is typical in
salicylate poisoning, the danger is that the acidosis will worsen

(b) What does could be fatal?


Answer: >500mg/kg could cause fatal poisoning.
(c) Is there a role for activated charcoal with asprin od?
Answer: Yes, if 2 levels are taken & the 2nd 1 ↑ you could consider giving another dose of 50g of activated charcoal.
(d) A CXR is taken (fig 1): What is shown & how would you manage this?
Answer: ARDS needs ITU for ventilation & supportive care, likely to need renal support.
(e) What is the definitive management in severe cases?
Answer: ITU for haemodialysis Paralysis & ventilation often helpful IV glucose as brain levels can get very low
‫ـــــــــــــــــــــــــــــــــــــــــــــــــــــــــــــــــــــــــــــــــــــــــــــــــــــــــــــــــــــــــــــــــــــــــــــــــــــــــــــــــــــــــــــــــــــــــــــــــــــــــــــــــــــــــــــــــــــــــــــــــــــــــــــــــــــــــــــ‬
58- Question 3 of 50
A 44 yo male presents with retrosternal chest pain & a mild fever. He says that the pain is sharp & worse on
inspiration. It gets better when he leans forward. His ECG is shown in fig 1

(a) What is the diagnosis?


Answer: Pericarditis
(b) Can you name 4 causes?
Answer: Viral (coxsackie B, HIV), Post MI (Dressler’s syndrome). Bacterial pneumonia or septicaemia, TB,
Caner, Rheumatic fever, Uraemia, Collagen vascular disease ( SLE, PAN, Rheumatoid arthritis), After cardiac
surgery, Drugs ( hydralazine, procainamide, methyldopa, minoxidil)
(c) He subsequently tells you that he has had unprotected homosexual intercourse & is concerned about HIV.
What other important test does this Pt need?
Answer: TB, as this is a cause of pericarditis & he may well have TB due to being HIV +ve.
(d) Should this Pt have troponins measured?
Answer: There is no evidence that it guides management or provides prognostic information.
(e) The Pt responds well to simple TTT & you decide that he can be discharged however he returns to see his
GP a week later with symptoms of tiredness & further chest pain. The GP runs some routine tests including an
ESR which is found to be elevated. What further tests would you like to do & what diagnosis needs to be ruled
out?

24
Answer: diagnosis to rule out is myocarditis, test: ESR is ↑ in 60% of cases, will need cardiology work up & possibly
an endomyocardial biopsy which continues to be of use in diagnosing myocarditis. Also an echo will be useful.
‫ـــــــــــــــــــــــــــــــــــــــــــــــــــــــــــــــــــــــــــــــــــــــــــــــــــــــــــــــــــــــــــــــــــــــــــــــــــــــــــــــــــــــــــــــــــــــــــــــــــــــــــــــــــــــــــــــــــــــــــــــــــــــــــــــــــــــــــــ‬
59- Question 5 of 50
A 74 yo gentleman is called through to the department by the ambulance crew. He was found collapsed at home &
was unresponsive when they arrived. He had an aneurysm repair 2 yrs ago.

(a) In the 1ry survey you establish that he is shocked & has a GCS of 14 as he is confused but he is breathing
spontaneously & is maintaining his own airway. You can see a laparotomy scar. You gain IV access & attach
fluids aiming to maintain a BP of around�.. what?
Answer: The Pt is shocked, you have heard that he had an aneurysm repair 2 yrs ago. You want to aim for a MAP of
around 70 or a systolic of around 90 or less.
(b) How do you calculate MAP?
Answer: Diastolic Pr. + 1/3 of pulse Pr. (systolic-diastolic)= MAP approx.
(c) In what other circumstances should Pts be managed in this way & what is the underlying principle known
as?
Answer: Permissive hypotension; trauma is the other situation. If someone is shocked in trauma the principle should
be to maintain a similar MAP whilst aiming to prevent the dilution of clotting factors.
(d) What fairly new agents are you aware of that can help to stem bleeding in the shocked trauma Pt?
Answer: Activated factor VIIa
(e) He then proceeds to have a large PR bleed; the Bl. appears to be fresh. Name 2 DD for what is happening?
Answer: 1 large fresh rectal/lower GI bleed could be from numerous causes including diverticular disease,
angiodysplasia etc 2 aortoenteric fistula, rare but fits with the history.
(f) You alert the surgeons & arrange imaging. You manage to stabilise the Pt with resuscitation. What is the
best way to ensure that this Pt is adequately monitored at this time?
Answer: an arterial line will be very helpful to detect the beat to beat variation in Bl. Pr., easy & quick to insert.
(g) What is octaplas & in what situations is it used?
Answer: It is Fresh Frozen Plasma used to reverse the effects of warfarin very quickly
‫ـــــــــــــــــــــــــــــــــــــــــــــــــــــــــــــــــــــــــــــــــــــــــــــــــــــــــــــــــــــــــــــــــــــــــــــــــــــــــــــــــــــــــــــــــــــــــــــــــــــــــــــــــــــــــــــــــــــــــــــــــــــــــــــــــــــــــــــ‬
60- Question 10 of 50
A 75 yo man was found collapsed at home by his son. There was no available past history.He was living independently &
had last been seen 2 days previously by his son. O/E his GCS was 7/15. There was no nuchal rigidity, pupil reflexes
were sluggish but fundi exam. was noraml. Tone was slightly  in all 4 limbs. The peripheral reflexes were present &
plantars were downgoing. The HR was 39 bpm & the Bl. Pr. was 76/42 mmHg. Heart sounds were normal & the chest
was clear. Hypothermia was suspected

(a) How is hypothermia defined?


Answer: A fall in core temp. below 35 degrees
(b) How should the temp. be measured?
Answer: Rectal (core) temp. with a low reading thermometer
(c) List some clinical features of this condition?(four)
Answer: Impaired consciousness, cardiac embarrassment, bradycardia, hypotension, hypopnoea, sluggish pupillary &
peripheral reflexes, muscle rigidity, coma, ventricular fibrillation, & asystole
(d) List some ECG features of this condition?(list four)
Answer: Bradycardia, tremor artefact, J-waves, prolonged QT interval, prolonged PR
(e) How would you treat this man if the diagnosis is confirmed?
Answer: Gradual rewarming using space blankets. If the Pt is unconsciousness or the temp. is less than 32 warm IV
fluids may be given. The aim is to increase the temp. by one degree/hr.
‫ـــــــــــــــــــــــــــــــــــــــــــــــــــــــــــــــــــــــــــــــــــــــــــــــــــــــــــــــــــــــــــــــــــــــــــــــــــــــــــــــــــــــــــــــــــــــــــــــــــــــــــــــــــــــــــــــــــــــــــــــــــــــــــــــــــــــــــــ‬
61- Question 2165
An 80 yo man presented feeling generally unwell. O/E his HR was 30 bpm

(a) Name three basic initial steps in the management of this man?
Answer: O2, ABCs, monitor ECG, monitor BP, SpO2, establish IV access.
(b) What are the signs & symptoms suggesting poor perfusion caused by bradcardia?(Name three)
Answer: Acute altered mental status, ongoing chest pain, hypotension or other signs of shock.
(c) If the Pt has poor perfusion what medication should be considered?What is the dose?
Answer: Atropine 0.5mg IV. May repeat to a total of 3mg.
(d) If this is ineffective & no specialist consultation is available what is the next step?
25
Answer: Transcutaneous pacing.
(e) If the therapeutic modality of step 4 is ineffective what medication may be added to try & ↑ its
effectiveness?
Answer: Adrenaline(2-10ug/min) or dopamine(2-10 ug/kg per minute) infusion.
‫ـــــــــــــــــــــــــــــــــــــــــــــــــــــــــــــــــــــــــــــــــــــــــــــــــــــــــــــــــــــــــــــــــــــــــــــــــــــــــــــــــــــــــــــــــــــــــــــــــــــــــــــــــــــــــــــــــــــــــــــــــــــــــــــــــــــــــــــ‬
62- Question 17 of 50
A 45 yo woman presented with pleuritic chest pain.

(a) What is the differential diagnosis of pleuritic chest pain?(Give four)


Answer: PE, pericarditis, viral pleurisy, Pneumonia, Pneumothorax, Collagen vascular diseases, including systemic
lupus erythematosus, mixed connective tissue disease, & rheumatoid arthritis, drug-induced lupus, inflammatory
bowel disease, familial Mediterranean fever, & radiation pneumonitis
(b) How would you investigate this Pt?(Give four)
Answer: ECG, D-Dimer, CXR, FBC, ABG, BNP, troponin, CTPA, V/Q Scan.
(c) What %age of Pts with a PE have abnormal D-Dimer value?(not including sub-segmental PE)
Answer: D-dimer levels are abnormal in 95% of Pts with PE.They are abnormal in only 50% of Pts with subsegmental PE
(d) If PE is confirmed how would you manage this Pt?(three steps)
Answer: O2, hemodynamic support, analgesia, & anticoagulation
(e) What is the prognosis of untreated PE?
Answer: PE is associated with a mortality rate of approximately 30% without TTT, primarily the result of recurrent embolism
‫ـــــــــــــــــــــــــــــــــــــــــــــــــــــــــــــــــــــــــــــــــــــــــــــــــــــــــــــــــــــــــــــــــــــــــــــــــــــــــــــــــــــــــــــــــــــــــــــــــــــــــــــــــــــــــــــــــــــــــــــــــــــــــــــــــــــــــــــ‬
63- Question 20 of 50
A 73 yo woman presented to the ED complaining of severe pain. Her vital signs were within the normal range. Exam.
revealed the rash shown in the picture.

Fig 1
(a) What is the causative organism?
Answer: Herpes zoster(shingles) results from reactivate. of endogenous latent VZV infect. within the sensory ganglia.
(b) How does the rash begin?
Answer: The rash of herpes zoster starts as erythematous papules, which quickly evolve into grouped vesicles or bullae
(c) How long is the Pt infective?
Answer: In immunocompetent hosts, the lesions crust by 7 to 10 days & are no longer infectious
(d) Which 2 dermatomal regions are most commonly involved?
Answer: The thoracic & lumbar dermatomes are the most commonly involved sites of herpes zoster. Zoster is
generally limited to one dermatome in previously healthy hosts.
(e) What percentage of Pts develop systemic symptoms & name three systemic symptoms?
Answer: Fewer than 20% of Pts have significant systemic symptoms, such as headache, fever, malaise, or fatigue.
‫ـــــــــــــــــــــــــــــــــــــــــــــــــــــــــــــــــــــــــــــــــــــــــــــــــــــــــــــــــــــــــــــــــــــــــــــــــــــــــــــــــــــــــــــــــــــــــــــــــــــــــــــــــــــــــــــــــــــــــــــــــــــــــــــــــــــــــــــ‬
64- Question 25 of 50
Eligibility criteria for the TTT of acute ischemic stroke with recombinant tissue plasminogen activator (rt-PA) include:

(a) Within how long a period of time can TTT be given after a clearly defined symptom onset?
Answer: Thrombolytic TTT should be initiated within 3 hrs of a clearly defined symptom onset.
(b) Name three features of the history which may exclude the use of thrombolysis
Answer: Features of the history which exclude the use of thrombolysis include stroke or head trauma within the prior
3 months, any prior history of intracranial hemorrhage, major surgery within 14 days, gastrointestinal or

26
gentitourinary bleeding within the previous 21 days, myocardial infarction in the prior 3 months, arterial puncture at a
noncompressible site within 7 days, & lumbar puncture within 7 days.
(c) Name three clinical features which exclude the use of thrombolysis.
Answer: Clinical features which may exclude the use of thrombolysis include rapidly improving stroke symptoms,
only minor & isolated neurologic signs, seizure at the onset of stroke is an exclusion if the residual impairments are
due to postictal phenomenon(Seizure isn't an exclusion if the clinician is convinced that residual impairments are due
to stroke & not to postictal phenomenon), symptoms suggestive of subarachnoid hemorrhage, even if the CT is
normal, clinical presentat. consistent with acute MI or post-MI pericarditis, persistent systolic BP>185, diastolic BP>110
mmHg, or requiring aggressive therapy to control BP, preg. or lactation, active bleeding or acute trauma (fracture).
(d) Name three laboratory features which exclude the use of thrombolysis.
Answer: Lab. features which exclude the use of thrombolysis include platelets <100,000/mm3, serum glucose <50 mg/dL
(2.8 mmol/L) or >400 mg/dL (22.2 mmol/L), INR >1.7 if on warfarin & an elevated PTT if on heparin
(e) Name 2 head CT scan features which exclude the use of thrombolysis.
Answer: Head CT scan which exclude the use of thrombolysis include evidence of hemorrhage & evidence major
early infarct signs, such as diffuse swelling of the affected hemisphere, parenchymal hypodensity, &/or effacement of
>33 % of the middle cerebral artery territory
‫ـــــــــــــــــــــــــــــــــــــــــــــــــــــــــــــــــــــــــــــــــــــــــــــــــــــــــــــــــــــــــــــــــــــــــــــــــــــــــــــــــــــــــــــــــــــــــــــــــــــــــــــــــــــــــــــــــــــــــــــــــــــــــــــــــــــــــــــ‬
65- Question id: 2146
A 65 yo woman presents with chest pain. Her ECG is shown in figure 1.

(a) What are the 4 criteria according to the UK Resuscitat. Council 2005 guidelines that constitute an unstable
tachyarrhythmia?
Answer: 1. Presence of chest pain 2. Systolic BP <90 3. Evidence of heart failure 4. Decrease in conscious level
(b) What is the TTT of choice? What is it crucial to appreciate from an anaesthetic viewpoint?
Answer: DC cardioversion is the TTT of choice. Must be done in synchronised mode so that the shock is delivered on
the R wave to avoid precipitating VF. Need to appreciate that the circulation time & cardiac output are obviously
markedly reduced therefore a gentle anaesthetic is required, also high risk of aspiration as not starved
(c) How many joules would you select for the above rhythm?
Answer: 200 monophasic, 120-150 biphasic for starters

Fig1
(d) How many shocks would you deliver if your first were not successful?
Answer: 3
(e) What would you do after the No. of shocks you stated in part d?
Answer: Give 300mg of amiodarone over 10-20 mins & rpt the shock.
‫ـــــــــــــــــــــــــــــــــــــــــــــــــــــــــــــــــــــــــــــــــــــــــــــــــــــــــــــــــــــــــــــــــــــــــــــــــــــــــــــــــــــــــــــــــــــــــــــــــــــــــــــــــــــــــــــــــــــــــــــــــــــــــــــــــــــــــــــ‬
66- Question 27 of 50
A 29 yo woman who was 38 weeks pregnant called an ambulance because she felt palpitations. The ambulance staff
called in that the Pt had a narrow complex tachycardia.

(a) What are the symptoms & signs that suggest that this Pt may be unstable?(Give three)
Answer: Altered mental status, ongoing chest pain,& hypotension
(b) If it is decided that the Pt is stable give four basic steps prior to TTT?
Answer: O2, monitor, IV access, 12 lead ECG.
(c) If the rhythm is regular & QRS complex is narrow how would you procede prior to administering any
medication?
Answer: Vagal maneuvers.

27
(d) If this fails, with what medication would you treat the Pt?What is the dose of the medication?
Answer: Adenosine 6mg IV push. If no conversion give 12mg rapid IV push;may repeat 12mg dose once.
(e) If the rhythm fails to convert after this medication what other diagnoses should be considered?(Give 2)
Answer: Atrial flutter, ectopic atrial tachycardia or junctional tachycardia. The rate should be controlled with a Ca
channel blocker or a beta blocker, treat the underlying cause & consider expert consultation
‫ـــــــــــــــــــــــــــــــــــــــــــــــــــــــــــــــــــــــــــــــــــــــــــــــــــــــــــــــــــــــــــــــــــــــــــــــــــــــــــــــــــــــــــــــــــــــــــــــــــــــــــــــــــــــــــــــــــــــــــــــــــــــــــــــــــــــــــــ‬
67- Question 28 of 50
A 20 yo male presented to the ED with intermittent headaches & malaise since a head injury at work the previous
week. The head injury had caused a LOC & he had been brought to another hospital immediately afterwards & a CT
brain scan had not revealed any intracranial pathology. Three days later he had re-attended the same hospital as he
still had headaches & malaise. He had undergone a 2nd CT brain scan, which again was unremarkable

(a) What are the common post concussion symptoms? (Give four)
Answer: Headache, lethargy, low mood, poor concentrating ability, dizziness
(b) What are the characteristics of post concussion headaches? (Give 2)
Answer: May last for several months, intermittent, become worse during the day, become worse on exercise
(c) What factors may contribute to dizziness caused after a concussion? (Give one)
Answer: Codeine based analgesia, Pts are more sensitive to the effects of alcohol
(d) Name 2 categories of Pts who are prone to developing a chronic subdural haematoma?
Answer: Elderly, Pts with bleeding disorders, alcoholics.
(e) How would you manage this Pt? (Give four)
Answer: History should cover symptoms of other types of headache e.g. photophobia, meningismus, full neurological
exam, investigations to out rule other causes of headache if appropriate, check the reports of the CT Brain radiologist
report from the initial hospital, explanation of symptoms to Pt, arrange follow up with GP.
‫ـــــــــــــــــــــــــــــــــــــــــــــــــــــــــــــــــــــــــــــــــــــــــــــــــــــــــــــــــــــــــــــــــــــــــــــــــــــــــــــــــــــــــــــــــــــــــــــــــــــــــــــــــــــــــــــــــــــــــــــــــــــــــــــــــــــــــــــ‬
68- Question 29 of 50
A 50 yo truck driver presented with dysuria & painful wrists, shoulders, knees & ankles.He also complained of purulent eye
discharge. O/E he was febrile (38.5)&had a small joint effusion in his Rt knee. His dipstick urine revealed nitrites,leukocytes&Bl.

(a) What is the diagnosis?


Answer: Reiters syndrome.
(b) Name five investigations which should be carried out?
Answer: FBC, UE, MSU, Bl. cultures, knee synovial fluid aspiration, stool culture (as enteric infections can cause a
reactive arthritis), CRP, ESR, & plain radiographs to exclude other diagnoses.
(c) Name three pathogens which can cause a reactive arthritis?
Answer: The classical pathogens for reactive arthritis are: Chlamydia trachomatis, Yersinia, Salmonella, Shigella &
Campylobacter, & perhaps Clostridium difficile & Chlamydia pneumoniae.
(d) Name three management steps in the ED?
Answer: NSAIDS, Rheumatology consultation, Infectious diseases consultation to discuss appropriate additional tests
& medications for symptomatic relief or microbiologic cure & to ensure follow-up TTT
(e) What is the prognosis of this condition?
Answer: Most Pts remit completely or have little active disease six months after presentation. Chronic persistent
arthritis, lasting more than six months, occurs in only a small proportion of Pts.
‫ـــــــــــــــــــــــــــــــــــــــــــــــــــــــــــــــــــــــــــــــــــــــــــــــــــــــــــــــــــــــــــــــــــــــــــــــــــــــــــــــــــــــــــــــــــــــــــــــــــــــــــــــــــــــــــــــــــــــــــــــــــــــــــــــــــــــــــــ‬
69- Question 30 of 50
A 30 yo women presented with redness & pain around her Rt eye.

(a) What is the most likely diagnosis?What is the most serious differential diagnosis?
Answer: Preseptal(periorbital) cellulitis & orbital cellulitis.(Preseptal cellulitis is much more common than orbital cellulitis)
(b) What are the most common pathogens to cause this condition?(Name 2)
Answer: The most common inciting organisms of preseptal cellulitis include St. pneumoniae, Staph.aureus, other St.
species & anaerobes.
(c) Name 2 indications for CT scanning?
Answer: Inability to accurately assess vision, gross proptosis, ophthalmoplegia, bilateral edema, or deteriorating
visual acuity, & signs or symptoms CNS involvement.
(d) How would you manage this Pt?(2 points)
Answer: Broad-spectrum oral antibiotics, consider anaerobic cover, opthalmology consultation, close observation.
(e) Name some complications of this condition?(Name three)
Answer: Recurrent preseptal cellulitis, orbital cellulitis, vision loss, death
‫ـــــــــــــــــــــــــــــــــــــــــــــــــــــــــــــــــــــــــــــــــــــــــــــــــــــــــــــــــــــــــــــــــــــــــــــــــــــــــــــــــــــــــــــــــــــــــــــــــــــــــــــــــــــــــــــــــــــــــــــــــــــــــــــــــــــــــــــ‬
28
70- Question 31 of 50
A 69 yo man developed a sudden onset painful left upper limb while at rest. He also complained of paraesthesia. O/E
he was in severe distress & his left upper limb was pulseless distal to the brachial pulse & extremely pale.

(a) What is the most likely diagnosis?


Answer: Ischaemic limb
(b) Name three other medical conditions that frequently co-exist with this condition?
Answer: Other maladies that often coexist with PVD are coronary artery disease, myocardial infarction (MI), atrial
fibrillation, transient ischemic attack, stroke, & renal disease
(c) Name 2 other points that should be noted in the exam. of the Pt?
Answer: Presence or absence of heart murmurs. Investigate all peripheral vessels, including carotid, abd., & femoral,
for pulse quality & bruit. Note that the dorsalis pedis artery is absent in 5-8% of normal subjects, but the posterior
tibial artery usually is present. Both pulses are absent in only about 0.5% of Pts.
(d) What are the mangement steps in the emergency deparment?
Answer: Analgesia.Urgent vascular surgery consultation.+/- heparin infusion.
(e) Name three risk factors for this condition?
Answer: Smoking, hyperlipidemia, DM, & hyperviscosity.
‫ـــــــــــــــــــــــــــــــــــــــــــــــــــــــــــــــــــــــــــــــــــــــــــــــــــــــــــــــــــــــــــــــــــــــــــــــــــــــــــــــــــــــــــــــــــــــــــــــــــــــــــــــــــــــــــــــــــــــــــــــــــــــــــــــــــــــــــــ‬
71- Question 32 of 50
A 65 yo man with lung cancer comes in as he has noticed a swelling to his Rt arm & has had worsening SOB over the
course of the day. He looks stressed & a bit red in the face. His arm is indeed swollen & you can see his ext. JV easily

(a) What do you suspect might be causing his symptoms?


Answer: Need to think about SVC obstruction.
(b) How would you investigate him?
Answer: Urgent admission & CT chest to see what is causing the obstruction
(c) Is there any particular therapy that could be started prior to any investigations?
Answer: Dexamethosone 8mg BD (high dose)
(d) What is the definitive TTT?
Answer: Radiotherapy (within 24hrs) +/- stenting.
(e) What is tumour lysis syndrome & when does it occur?
Answer: Occurs after starting chemotherapy, more common bulky chemosensative disease (lymphoma, germ-cell
tumours, high blast-count leukaemias) can occur within hrs or days. It is caused by rapid lysis of tumour cells by the
chemotherapeutic agents. The cell death leads to leak of metabolites & can lead to ARF.
‫ـــــــــــــــــــــــــــــــــــــــــــــــــــــــــــــــــــــــــــــــــــــــــــــــــــــــــــــــــــــــــــــــــــــــــــــــــــــــــــــــــــــــــــــــــــــــــــــــــــــــــــــــــــــــــــــــــــــــــــــــــــــــــــــــــــــــــــــ‬
72- Question 35 of 50
A 23 yo lady is wheeled into the resus department complaining of chest pain & dizziness. She also says that she can’t
feel her fingers. The crew report that she has no medical history, she’s a non-smoker & is normally very fit & well.
Her observat. in the ambulance show a RR of 45 but are otherwise unremarkable apart from her seeming very anxious

(a) What is the most likely diagnosis from the history given above?
Answer: 1ry hyperventilation, pyschongenic (panic attack)
(b) What tests must you do to confirm your initial thoughts?
Answer: Need to rule out 2ry causes for hyperventilation i.e. DKA Kussmal's breathing therefore to a BM,
Saturations: pneumothroax/PE ECG: cardiac cause
(c) What will you do with this Pt?
Answer: Reassure her that there is nothing serious going on & encourage her to take control of her respirations
perhaps counting breathe in through the nose, count for 6, breathe out through the mouth count for 6, hold for 3 etc.
(d) The RR doesn’t come down & despite your efforts the Pt isn’t changing or improving. What tests would you
do now?
Answer: ABG, CXR, U&E, Bl. glucose consider tox screen
(e) Name a group of presentations common to the ED which could present in this way.
Answer: Overdose of: Aspirin/CO/ Methanol/ cyanide/ ethylene glycol
‫ـــــــــــــــــــــــــــــــــــــــــــــــــــــــــــــــــــــــــــــــــــــــــــــــــــــــــــــــــــــــــــــــــــــــــــــــــــــــــــــــــــــــــــــــــــــــــــــــــــــــــــــــــــــــــــــــــــــــــــــــــــــــــــــــــــــــــــــ‬
73- Question 36 of 50
The same 46-yo septic Pt that you met in a previous question is now on the ITU. He has sepsis from pneumonia.

(a) He has a haemoglobin of 8 g/dL & you consider giving a Bl. transfusion. What is the current best evidence around
this?

29
Answer: RBC transfusions for adults should occur only when hemoglobin is < 7.0 g/dL to a target hemoglobin
between 7 & 9 g/dL
(b) His platelets have been falling & are currently 20,000mm/3 Should you give a platelet transfusion?
Answer: No unless there's a very high bleeding risk. When drops below 5000/mm3 then they should be given regardless
(c) What platelet level is normally considered minimum when considering surgery or other invasive
procedures?
Answer: 50,000mm/3
(d) When considering how a ventilator should be set with this Pt what are the important things to consider to
reduce the chances of ALI/ARDS?
Answer: Lower tidal volume mechanical ventilation (6 mL/kg based on ideal body weight) can reduce mortality rates
to 22.1% from 39.8% compared with conventional methods (12 mL/kg based on ideal body weight) Tidal volumes
should be ↓ over 1 to 2 hrs to a low TV (6mL/kg predicted body weight) as a goal (grade 1B recommendation) in
conjunction with the goal of maintaining peak airway Pr.s below 30 cm H2O (grade 1C recommendation).
(e) What other therapies need to be considered in this Pt?
Answer: Stress ulcer prophylaxis, DVT prophylaxis
‫ـــــــــــــــــــــــــــــــــــــــــــــــــــــــــــــــــــــــــــــــــــــــــــــــــــــــــــــــــــــــــــــــــــــــــــــــــــــــــــــــــــــــــــــــــــــــــــــــــــــــــــــــــــــــــــــــــــــــــــــــــــــــــــــــــــــــــــــ‬
74- Question 7 of 10
A 22 yo gentleman presents to the ED at 03:00 am, he has been out at a party. He is accompanied by a friend who
tells you that he was previously completely well & has no medical history. When you examine him he is only
responding to pain & has a GCS of 10 (E2, V3, M5). He has a temp. of 38.8 �C, pulse of 120 bpm & Systolic BP of
85. You make a diagnosis of septic shock

(a) What are the 4 most important initial management nterventions?


Answer: O2 & airway protection (+/- intubation & IPPV) Obtaining IV access, taking Bl. cultures & starting broad
spectrum antibiotics early Aggressive fluid therapy, Early involvement of ITU
(b) Define SIRS
Answer: Body temp. of >38 C or <36 C HR >90 bpm RR> 20 or PaCO2 <4.3 KPa WCC >12x 109/litre or <4 x
109/litre or > 10% immature b& forms
(c) What investigations would you like to carry out?
Answer: ABG, CXR, MSU, Bl. Cultures, Bl. Glucose, CT head
(d) Assuming a working diagnosis of septic shock what parameters would be sensible TTT targets in this
previously fit young man? Where should this Pt be managed?
Answer: MAP >65 mmHg, CVP 8-12 mmHG (12-15mmHg if ventilated), ITU with arterial & central venous lines
(e) Describe the technique of inserting an internal jugular central line using ultrasound guidance:
Answer: Full aseptic technique, Describe landmarks & how to use probe to identify the vein, Description of Sell-
dinger technique, Ensure all ports aspirate freely, 4 point fixation with skin sutures, Ensure CXR is ordered prior to use
‫ـــــــــــــــــــــــــــــــــــــــــــــــــــــــــــــــــــــــــــــــــــــــــــــــــــــــــــــــــــــــــــــــــــــــــــــــــــــــــــــــــــــــــــــــــــــــــــــــــــــــــــــــــــــــــــــــــــــــــــــــــــــــــــــــــــــــــــــ‬
75- Question 1 of 5
A 53 yo man presents with a swollen hot lt knee. He has also noticed that his calf muscle is hurting & there is
erythema over the calf. He has a background of gout, angina & HTN, he is allergic to penicillin. He is sure that there
has been no trauma to the knee. His temp. is 38.2

(a) Give three differential diagnoses in this scenario


Answer: 1: Septic arthritis 2: Gout/CPPD with local concurrent cellulitis of the leg 3: Reactive arthritis/Rieter's
syndrome 4: Gonococcal arthritis
(b) What are the 2 key investigations that you must perform in the ED?
Answer: 1: Bl. cultures� 2: Joint aspiration, gram stain, microscopy & polarized light microscopy.
(c) What test is used to determine the presences of crystal disease within a joint?
Answer: The use of polarized light microscopy. Gout -vely birefringent crystals that are needle shaped.
‫ـــــــــــــــــــــــــــــــــــــــــــــــــــــــــــــــــــــــــــــــــــــــــــــــــــــــــــــــــــــــــــــــــــــــــــــــــــــــــــــــــــــــــــــــــــــــــــــــــــــــــــــــــــــــــــــــــــــــــــــــــــــــــــــــــــــــــــــ‬
76- Question 4 of 5
A 46 yo gentleman is brought in by the ambulance crew. He has been unwell at home for the last 3 days with a cough.
His wife found him drowsy & unresponsive when she arrived home from work & dialled 999. His initial observations
are as follows, pulse 125 regular, Bp un-recordable, temp 38.7, RR28, GCS 13

(a) You presume a working diagnosis of sepsis. What are the 4 parameters that need to be aggressively
achieved within the golden hr- first 6 hrs of TTT?
Answer: Maintain strict parameters of normal values of CVP between 8 & 12 mm Hg (12 to15 mm Hg in the
mechanically vented Pt) with crystalloid or colloid infusions, MAP≥ 65, ScvO2≥ 70% or SvO2≥ 65%, UO≥ 0.5 mL/kg/hr

30
(b) You instigate aggressive fluid resuscitation with Hartman's & place a central line. The Pt initially responds
& now has a MAP of 50. You have infused 4 litres but the MAP remains 50 & his GCS is 14, UO is <0.5ml/hr,
SvcO2 is 60%. What are the next crucial steps for this Pt?
Answer: Needs urgent Bl. cultures take 20mls from 2 sites. Needs early antibiotic therapy. As he is not meeting the
targets despite fluid resuscitation you need to instigate the following. As the SvcO2 has not improved need to consider
the following, Additional fluid, Transfusion of red Bl. cells (RBCs) as needed to hematocrit ≥ 30%, Inotropic agents
(dobutamine 2.5 to 20 micrograms (mcg)/kg/min)
(c) Considering all comers what is the most likely cause of sepsis?
Answer: Lung: 35%; Abd.: 21%; Urinary tract: 13%; Skin & soft tissue: 7%; Other site: 8%; & Unknown: 16%..
(d) What are the recommended 1st line vasopressors?
Answer: Dopamine & norepinephrine
(e) What is the current recommendation for the use of steroids in septic shock?
Answer: Steroids. IV steroids (hydrocortisone 200 to 300 mg/day) for 7 days or 4 divided doses or by continuous
infusion is suggested only for Pts who, despite adequate fluid replacement, require vasopressor therapy to maintain
Bl. Pr. (grade 2C recommendation). This approach has only demonstrated ↓ mortality in those with relative adrenal
insufficiency (defined as postadrenocorticotropic hormone [ACTH] cortisol ≤9 mcg/dL).[10,11,35] Despite the long
-standing recommendat. to limit use of steroids to Pts with sepsis who (a) remain hypotensive despite adequate fluid replacement
& vasopressor therapy & (b) have insufficient rise in cortisol level from corticotropin challenge, steroids continue to
be widely used for those with septic shock.[36] For this reason, Sprung & colleagues[36] of the Corticotherapy for
Septic Shock (CORTICUS) study put this question to the test: Does the use of steroids for septic shock improve
mortality in a broader range of Pts with septic shock? Results from this landmark trial showed that hydrocortisone did
not reduce mortality Pts with sepsis at large & did  the risk for superinfection. Coupling this data (available but not
yet published at the time of the phase 2 SCC clinical guideline update) with the results of the study by Annane &
associates[35] published in 2002, the phase 2 SCC guidelines reiterated the restricted use of steroids to the population
described, & the strength of the rating was downgraded from the original guidelines published in 2004. The experts
who participated in SCC phase 2 debated about how best to communicate this recommendat. to clinicians, put
different wording options to a vote & the result was the following statement: "We suggest that IV hydrocortisone be
given only to adult septic shock Pts with Bl. Pr. poorly responsive to fluid resuscitat. & vasopressor therapy"(grade 2C)
(f) Who should receive Recombinant human activated protein C (rhAPC)?
Answer: An APACHE II score of 25 or greater; Sepsis-induced multiple organ failure; & No absolute
contraindications, related to bleeding risks. No mortality benefit in Pts with single-organ dysfunct. or APACHE ll < 25
‫ـــــــــــــــــــــــــــــــــــــــــــــــــــــــــــــــــــــــــــــــــــــــــــــــــــــــــــــــــــــــــــــــــــــــــــــــــــــــــــــــــــــــــــــــــــــــــــــــــــــــــــــــــــــــــــــــــــــــــــــــــــــــــــــــــــــــــــــ‬
77- Question 5 of 5
A 66 yo man presents with sudden severe ripping chest pain radiating to his back. He has a history of HTN. O/E there
is a diastolic murmur. You suspect an aortic dissection.

(a) What are the risk factors for this condition?(Name four)
Answer: The most important predisposing factor for acute aortic dissection is systemic HTN. Inflammatory diseases
that cause a vasculitis (giant cell arteritis, takayasu arteritis, rheumatoid arthritis, syphilitic aortitis) disorders of
collagen (eg, marfan syndrome, ehlers-danlos syndrome, annuloaortic ectasia) a bicuspid aortic valve, aortic
coarctation, turner syndrome, crack cocaine, previous aortic valve replacement, cardiac catheterization, trauma, high-
intensity weight lifting or other strenuous resistance & a history of CABG surgery are other associations.
(b) What other features(besides a diastolic murmur) in the exam. of this Pt may indicate an aortic
dissection?(Name 2)
Answer: Assymetry or absence of peripheral pulses or a pulse deficit, hypotension with features of tamponade, HTN,
neurological signs 2ry to carotid or spinal artery involvement.
(c) How would you investigate this Pt?(Name four)
Answer: ECG, CXR, FBC, UE, Glucose, Coag, Bl. group & crossmatch, TOE , CT Angiography.
(d) What features on the CXR give additional evidence to the suspected diagnosis?(Give three)
Answer: A widened mediastinum, a left sided pleural effusion, deviation of the trachea or NG tube to the Rt,
separation of 2 parts of the wall of a calcified aorta by >5mm (the Ca sign), & a double knuckle aorta.
(e) How would you manage this Pt if you suspected an aortic dissection?(Give four)
Answer: O2, 2 large bore IV cannulae, cross match Bl., IV opioid, specialist consultation, arterial line & BP control.
‫ـــــــــــــــــــــــــــــــــــــــــــــــــــــــــــــــــــــــــــــــــــــــــــــــــــــــــــــــــــــــــــــــــــــــــــــــــــــــــــــــــــــــــــــــــــــــــــــــــــــــــــــــــــــــــــــــــــــــــــــــــــــــــــــــــــــــــــــ‬
78- Question 2 of 20
A 24 yo woman has taken 37 paracetamol tablets & downed them with half a bottle of white rum. She said that she
wanted to end it all� She presents 3 hrs after taking the tablets. She weighs 55kg.

(a) What do you do?


31
Answer: Treat initially as she has taken> 150mg/kg. This is generally the threshold that will receive N-acetylcysteine.
(b) Give some risk factors that would make you want to treat Pts you had taken small overdoses
Answer: Taking enzyme inducing drugs such as Rifampicin, St John�s Wort, Phenytoin, Carbamazepine,
Barbiturates, Sulphonylureas, Alcohol (chronic excess) these increase the rate of paracetamol breakdown & hence
increase the amount of NAPQI which is toxic!
(c) What does the toxic metabolite of paracetamol bind to in the liver?
Answer: Glutathione
(d) Which test is the first to change if there has been liver damage?
Answer: Coagulopathies therefore the INR is the first to change & is sensitive for acute liver damage.
(e) Name an alternative to Parvolex�?
Answer: Methionine
‫ـــــــــــــــــــــــــــــــــــــــــــــــــــــــــــــــــــــــــــــــــــــــــــــــــــــــــــــــــــــــــــــــــــــــــــــــــــــــــــــــــــــــــــــــــــــــــــــــــــــــــــــــــــــــــــــــــــــــــــــــــــــــــــــــــــــــــــــ‬
79- Question 3 of 20
A 26 yo male presented to the ED with a stiff jaw & being unable to open his mouth. 3 days previously he was
immunized with tetanus toxin after lacerating his finger. O/E he had evidence of 'lock jaw'. His injured finger was
swollen , painful & exuding pus.

(a) What is the most likely diagnosis?


Answer: Tetanus.
(b) What is the organism?
Answer: Clostridium tetani.
(c) Where is the organism commonly found?
Answer: Soil , & faeces of domestic animals.
(d) What are the features of this condition?(Name four)
Answer: Rigidity, stiffness, reflex spasms,tachycardia, Bl. Pr. instability dysphagia,laryngeal spasm & opisthotonus.
(e) What is the immediate management?
Answer: Intramuscular human tetanus immunoglobulin, cleaning & if necessary surgical debridement of the wound
‫ـــــــــــــــــــــــــــــــــــــــــــــــــــــــــــــــــــــــــــــــــــــــــــــــــــــــــــــــــــــــــــــــــــــــــــــــــــــــــــــــــــــــــــــــــــــــــــــــــــــــــــــــــــــــــــــــــــــــــــــــــــــــــــــــــــــــــــــ‬
80- Question 4542
A 62 yo man presents acutely SOB. His vital signs are as follows: BP 78/49 pulse 110, he has a raised JVP, heart
sounds are muffled & inspiratory crackles are heard throughout the chest. He has a metal mitral valve & he admits to
having forgotten to take is warfarin the last week as he was on holiday in Spain.

(a) You perform a quick transthoracic ECHO which shows a mitral valve thrombus, what will you do?
Answer: Thrombolysis. He has an acute valve thrombosis resulting in cardiogenic shock. If he was stable then
surgery would be a better option.
(b) How do you measure the effect of your TTT?
Answer: Serial ECHO
(c) What is the best way to investigate/image a pt like this?
Answer: TOE gives much better views.
(d) List the differential diagnosis for the above Pt if you didn�t have access to ECHO
Answer: CCF, Cardiac tamponade, Tension pneumothorax? Cardiogenic shock post M.I.
‫ــــــــــــــــــــــــــــــــــــــــــــــــــــــــــــــــــــــــــــــــــــــــــــــــــــــــــــــــــــــــــــــــــــــــــــــــــــــــــــــــــــــــــــــــــــــــــــــــــــــــــــــــــــــــــــــــــــــــــــــــــــــــــــــــــــــــــــ‬
81- Question id: 2109
A 55 yo man presents with a 6 hr history of palpitations that woke him at 05:00am. His BMI is 29 but he is otherwise
well & takes no medication.

32
Fig1
(a) An ECG (fig 1) reveals the following rhythm: what is it?
Answer: Atrial fibrillation with rapid ventricular response
(b) What do you need to establish quickly?
Answer: Whether this rhythm is compromising the Pt or not i.e. are they stable? Reduced conscious level Systolic BP
<90 Chest pain Signs of heart failure
(c) What questions need to be asked in the history to try to establish a cause?
Answer: Any history of IHD or family Hx of structural (HOCM) or coronary disease, HTN, alcohol binge, caffeine
intake, hyperthyroidism, recent PE, acute pericarditis, acute pulmonary disease etc.
(d) You consider this Pt to be stable & he seems otherwise well. Would he be a c&idate for pharmacologic
cardioversion? What would contraindicate this?
Answer: Probably yes, if there is any suspicion of cardiac failure LVF then it is contraindicated. Many drugs that
could be used including sotalol, flecanide, quinidine, propafenone, disopyramide.
(e) Later on that day another Pt comes in who is in what seems to be the same rhythm shown in the ECG in
part a, she is 78 & has a history of palpitations on & off over the yrs. She takes digoxin & aspirin. She is
haemodynamically stable. Where does your management focus lie?
Answer: The cornerstones of AF management are controlling Pts symptoms & preventing thromboembolic
complications, not restoration of sinus rhythm. 1st line TTT would be beta blockers or dihydrpyridine Ca channel
blockers (verapamil or diltiazem) which are effective during exercise & at rest, digoxin is only effective at rest &
should be considered a second line agent.
‫ـــــــــــــــــــــــــــــــــــــــــــــــــــــــــــــــــــــــــــــــــــــــــــــــــــــــــــــــــــــــــــــــــــــــــــــــــــــــــــــــــــــــــــــــــــــــــــــــــــــــــــــــــــــــــــــــــــــــــــــــــــــــــــــــــــــــــــــ‬
82- Question 10 of 20
You are alerted that a man (looks about 50) has collapsed outside the department after leaving the hospital from a
renal out Pt appointment. You rush outside with a portable defibrillator & some equipment. When you arrive at the
scene there is a crowd & the Pt appears to have arrested. You decide that it is too far to try to move him to the ED &
you don't have a trolley so you shout for help & start the resuscitation

(a) You have a good team & you quickly intubate the Pt. The rhythm is VF & you deliver a shock. What size
ET tube did you use? What is the ratio of ventilations to compressions now?
Answer: 8 or 9 normally for an adult male. 7 or 8 for an adult female. When intubated the compressions are
continuous as are the ventilations.
(b) You get a pulse back after the third shock with one dose of adrenaline given. You quickly transport the Pt
to the resus room. What do you do now?
Answer: The Pt is intubated so you need to assess for signs of life & check if he is making any respiratory effort it is
likely that you will need to continue ventilating him. Check an ABG & send off Bl.s, get an ECG
(c) The potassium is 7.2mmol/L. What do you do?
Answer: Consider that this has caused the VF arrest, needs to be treated. Give 10mls of 10% Ca gluconate. Consider
Na bicarbonate particularly if there is severe acidosis/renal failure, which there clearly will be in this case. Give
insulin & glucose Consider haemodyalysis on ITU
(d) What ECG changes are seen in hypokalaemia?
Answer: Prominent U waves & flattened T waves.
(e) What is the recommended maximum infusion rate for potassium? What is essential for giving IV
potassium?
33
Answer: 20mmols/hr is the recommended maximum infusion rate but sometimes i.e. peri-arrest arrhythmias/cardiac
arrest due to hypokalaemia can be given faster but ideally this should be through a central line. Must have cardiac
monitoring to give IV replacement especially at the rates described.
‫ـــــــــــــــــــــــــــــــــــــــــــــــــــــــــــــــــــــــــــــــــــــــــــــــــــــــــــــــــــــــــــــــــــــــــــــــــــــــــــــــــــــــــــــــــــــــــــــــــــــــــــــــــــــــــــــــــــــــــــــــــــــــــــــــــــــــــــــ‬
83- Question 11 of 20
A 27 yo female presents with palpitations of sudden onset which she has had before.

(a) What does the ECG show (fig 1)?


Answer: Supraventricular tachycardia (SVT)
(b) In a young healthy individual with a normal resting ECG what is this type of rhythm likely due to?
Answer: It is likely to be an atrioventricular nodal re-entrant tachycardia (AVNRT) i.e. the most common cause of
narrow complex tachycardia in Pts with normal hearts.
(c) How do you treat it in the ED providing that she is stable with the rhythm? (Include exact drug doses &
sequences)
Answer: Attempt vagal manoeuvres, valsalva with 50 ml syringe, carotid sinus massage etc. If fails try adenosine
6mg, 12mg, 12mg
(d) Describe what you might need to explain to a Pt prior to the interventions you gave in part c.
Answer: Warn of the side effects of adenosine feeling of sudden chest discomfort & flushing Pts say that it feels terrible!
(e) What will the definitive management options?
Answer: Radio frequency ablation is the 1st line TTT for recurrent symptomatic episodes, which is curative. Or AV
blocking drugs B-blockers, diltiazem or verapamil can be used as a pill in the pocket, type approach to terminate an event.
‫ـــــــــــــــــــــــــــــــــــــــــــــــــــــــــــــــــــــــــــــــــــــــــــــــــــــــــــــــــــــــــــــــــــــــــــــــــــــــــــــــــــــــــــــــــــــــــــــــــــــــــــــــــــــــــــــــــــــــــــــــــــــــــــــــــــــــــــــ‬
84- Question 12 of 20
A 55 yo lady is brought in complaining of severe abd. pain. She is an epileptic & takes Carbamazepine for this; she's
normally fit & well her only other medicines are the OCP. You examine her abd. which is soft with no signs of
peritonism. She is tachycardic but hypertensive. She also complains of loss of sensation in her lower limbs. She is also
agitated. Her Bl. reveal a Na of 125mmmols/L. Her urine sample that was taken 30 minutes ago looks brown/red.

(a) What could be going on here? What would you do to confirm your suspicions?
Answer: Although the differential is wide the history is suggestive of acute porphyria Other differentials include
acute abdo pain (any cause of) Guillain-Barr syndrome, Systemic lupus erythematosus Test the urine for
porphobilinogen (PBG) (send a urine sample that is protected from light)
(b) Urinary porphobilinogen is markedly  which confirms your suspicions about what is going on. What will
you do?
Answer: Manage pain! Normally requires opiod analgesia In severe attacks, a glucose 10% infusion is commenced,
which may aid in recovery. Supportive TTT ensure that high carbohydrate feed is given. Haem arginate are the drugs
of choice in acute porphyria Consider propanolol to treat HTN
(c) What are the causes of the condition described?
Answer: Abnormalities of haem-biosynthesis, They are broadly classified as hepatic porphyrias or erythropoietic
porphyrias, based on the site of the overproduction & mainly accumulation of the porphyrins (or their chemical
precursors). They manifest with either skin problems or with neurological complications (or occasionally both).
(d) List things that can precipitate an attack of the condition described?
Answer: ETOH, lead poisoning, iron deficiency, drugs(carbamazipine, OCP, sulphonamides, methyldopa,
barbiturates, danazol, chloramphenicol, tetracyclines, some antihistamines,) smoking, sudden dieting, emotional &
physical stress, pregnancy etc .
‫ـــــــــــــــــــــــــــــــــــــــــــــــــــــــــــــــــــــــــــــــــــــــــــــــــــــــــــــــــــــــــــــــــــــــــــــــــــــــــــــــــــــــــــــــــــــــــــــــــــــــــــــــــــــــــــــــــــــــــــــــــــــــــــــــــــــــــــــ‬
85- Question 14 of 20
1 of your staff nurses (aged 28) asks your advice because she has had loose bowel motions for 2 wks since returning
from India. She is worried she may have dysentry. She has 8 loose stools per day with abd. cramps & for 3 days has
noticed some Bl. in the stool. She is previously healthy.

(a) Give 3 possible differential diagnosis.


Answer: Bacterial GE, IBD, Viral GE, Parasitic disease, Coeliac disease, Tropical sprue, Amoebic dysentry
(b) Give one indication for antibiotics in a Pt who presents with diarrhoea.
Answer: Severe invasive disease Bl. / refractory, prolonged diarrhoea.
(c) Other than antibiotics, what 2 other medicat. might you consider & give the rationale for their use in Pts with
diarrhoea
Answer: Simple analgesia e.g paracetamol for cramping pain Consider immodium to ↓ motility Fluid & electrolyte
replacement e.g diaralyte

34
(d) After discussion with microbiology, you decide to prescribe a course of antibiotics for her. What other 4
pieces of advice would you give her?
Answer: Hydration Hand washing / hygiene Occupational health clearance prior to return to work Caution with local
contacts (family/friends/food preparation etc) Follow up stool culture Avoid lactose containing foods until diarrhoea stops
‫ـــــــــــــــــــــــــــــــــــــــــــــــــــــــــــــــــــــــــــــــــــــــــــــــــــــــــــــــــــــــــــــــــــــــــــــــــــــــــــــــــــــــــــــــــــــــــــــــــــــــــــــــــــــــــــــــــــــــــــــــــــــــــــــــــــــــــــــ‬
86- Question id: 4536
A 44 yo woman comes in to the ED c/o headache & visual disturbance. She has essential HTN. Her BP is 235/119.

Fig 1
(a) What is occurring?
Answer: You don't know yet until full exam is performed. This is HTN urgency which differs from a HTN crisis/
malignant HTN. A HTN emergency is a condition in which  Bl. Pr. results in target organ damage. HTN urgency
must be distinguished from emerg. Urgency is defined as severely  Bl. Pr. (ie, systolic >220 mm Hg or diastolic>120
mm Hg) with no evidence of target organ damage. For malignant HTN to be diagnosed papiloedmea must be present.
(b) What exam. is critical here?
Answer: Need to look at the fundi for papiloedema or other changes associated with vascular damage such as flame-
shaped haemorrhages or soft exudates, but without papilloedema.
(C) How would you treat her?
Answer: Depends if this turns out to be a HTN emergency or not, if not then aim to reduce the BP slowly if no
contraindications for a B-blocker then this is a good option i.e. Atenolol 25mg HTN emergencies require immediate
therapy to ↓ Bl. Pr. within minutes to hrs. In contrast, no evidence suggests a benefit from rapidly ↓ Bl. Pr. in Pts with
HTN urgency. In fact, such aggressive therapy may harm the Pt, resulting in cardiac, renal, or cerebral hypoperfusion.
(d) The funoscopic picture reveals the following (see figure 1). What do you do?
Answer: Once the diagnosis of HTN emergency is made, the most commonly used IV drug is nitroprusside. An
alternative for Pts with renal insufficiency is IV fenoldopam. Labetalol is another common alternative, providing easy
transition from IV to oral dosing. B-blockade can be accomplished IV with esmolol or metoprolol. Also available
parenterally are diltiazem, verapamil & enalapril. Hydralazine is reserved for use in pregnant Pts, while phentolamine
is the drug of choice for a pheochromocytoma crisis. Pt should be managed with an arterial line on ITU.
‫ـــــــــــــــــــــــــــــــــــــــــــــــــــــــــــــــــــــــــــــــــــــــــــــــــــــــــــــــــــــــــــــــــــــــــــــــــــــــــــــــــــــــــــــــــــــــــــــــــــــــــــــــــــــــــــــــــــــــــــــــــــــــــــــــــــــــــــــ‬
87- Question 20 of 20
Its 03:00 am & you are asked to see a 24 yo girl with chest pain who has been out clubbing. A friend accompanies her &
they are both very talkative although the girl does indeed appear to be in severe pain. They admit to occasional substance misuse.

(a) The ECG seems to show widespread changes that look like an ST elevation M.I. What do you need to do?
Answer: Need to get a through history very quickly. Ask about risk factors for coronary disease. Need to establish if
she has taken illicit substances particularly cocaine, how much & when exactly she took it.
(b) What important & potentially life threatening condition should be ruled out in this Pt at this stage?
Answer: Could the pain be related to aortic dissection? Unlikely given the nature of pain described but increased risk
with cocaine use. Would be pertinent to at least do CXR before treating anything else.
(c) Yours suspicions were correct; she is having a myocardial infarction. How do you manage this Pt?
Answer: General measures are the same as anyone presenting with acute M.I.: MONA. In addition IV GTN to be
given at higher doses titrate but aim for high dose > 10mg/hr final level. Benzodiazepines to reduce anxiety
(d) You instigate initial measures as described above, what second line pharmacological agents could you use?
Answer: Verapamil: in high doses reduces cardiac work load & hence restores O2 supply & dem& as well as
reversing coronary vasoconstriction. Phentolamine: α-adrenergic antagonist & reverses vasoconstriction. Labetalol:
both α& βadrenergic effects it can be used after verapamil & phentolamine if Pt remains hypertensive.
(e) The Pt fails to improve what should happen next?
Answer: PCI. Evidence for thrombolysis is weak & generally associated with poor outcome 2ry to HTN induced
haemorrhagic complicat.
‫ـــــــــــــــــــــــــــــــــــــــــــــــــــــــــــــــــــــــــــــــــــــــــــــــــــــــــــــــــــــــــــــــــــــــــــــــــــــــــــــــــــــــــــــــــــــــــــــــــــــــــــــــــــــــــــــــــــــــــــــــــــــــــــــــــــــــــــــ‬
35
88- Question id: 2029
A 45 yo man presented to the ED with a 6 hr history of progressive neck swelling & fever.

(a) What is the most common cause of the condition in the picture?
Answer: The most common cause of Ludwig's angina is dental infection especially of the second & third lower
molars5. Predisposing factors include dental carries, recent dental TTT, systemic illnesses such as DM, malnutrition,
alcoholism & immunosuppression & immunocompromise.

(b) What are the common causative organisms?


Answer: The most common causative organisms result from polymicrobial odontogenic infections extending into the
deep facial spaces. F . nucleatum, Bacteroides & Prevotella spp, Peptostreptococcus spp., Actinomyces spp.
,Streptococcus spp & Staphylococci are the most common isolates
(c) What is appropriate antibiotic cover?
Answer: Appropriate antibiotics include high dose penicillin(+/- metronidazole), & clindamycin or more broad
spectrum antibiotics such as ticarcillin-clavulanate, piperacillin-tazobactam or ampicillin-sulbactam.
(d) Discuss some non-antibiotic TTTs for this condition.
Answer: Dexamethasone reduces oedema & cellulitis & provides an initial chemical decompression protecting the
airway as well as allowing improved antibiotic penetration into the area. Nebulised adrenaline (1ml of 1:1000 diluted
to 5ml of 0.9% normal saline) is also safe & effective in reducing upper airway obstruction.
(e) What is the mortality for this condition?
Answer: It had a 50% mortality before the common use of antibiotics due to upper airway oedema but therapeutic
advances including antibiotics & supportive therapy have reduced the mortality to 8%.
‫ـــــــــــــــــــــــــــــــــــــــــــــــــــــــــــــــــــــــــــــــــــــــــــــــــــــــــــــــــــــــــــــــــــــــــــــــــــــــــــــــــــــــــــــــــــــــــــــــــــــــــــــــــــــــــــــــــــــــــــــــــــــــــــــــــــــــــــــ‬
89- Question 7 of 20
A 74 yo lady presents with severe left sided loin pain that radiates to the left side of the abd.. She seems distressed &
cannot keep still on the bed. You consider a diagnosis of renal colic.

(a) What are the initial priorities?


Answer: Clearly ABCD as ever- analgesia is key! Also hydrat. as Pts are often dehydrated from prolonged vomiting.
(b) What is the investigation of choice?
Answer: CT of the abd. 95% specific & 95% sensitive- also has the added advantage of picking up other things i.e. AAA
(c) What %age of Pts do not have haematuria?
Answer: Around 15% (e Medicine)
(d) What is the sex ratio for renal calculi?
Answer: The overall lifetime rate of kidney stones in the general population is approximately 12% for men & 4% for
women. (Emedicine)
(e) When can a Pt be discharged with renal colic?

36
Answer: If the pain has settled & the imaging has shown no signs of obstruction (NB that sometimes the pain going
can represent obstruction) As long as the Pt is otherwise well.

90- Question id: 4518


An 88 yo man is brought in from a nursing home with a GCS of 12. He had been a little sleepy over the last few days.
You find out that he was found slumped in a chair with a left sided facial droop.

(a) What are your first priorities?


Answer: You need a full history of events from the home. Full ABC assessment, rule out the simple correctable
causes for a lowered GCS for example Bl. glucose, any evidence of poisoning etc, Full neurological assessment.

Fig 1
(b) You organise a CT head, which is shown in figure 1. Describe what it shows
Answer: Large Rt sided subdural haematoma with evidence of an acute on chronic bleed. There is also global
cerebral atrophy & midline shift.
(c) What must have happened to this gentleman?
Answer: Must have had head trauma at some stage.
(d) What is the TTT?
Answer: Consideration of burr hole evacuation to improve symptoms. Generally, because the lesion represents
clotted Bl., the burr hole is not curative, & emergent craniotomy is necessary.
(e) Why are alcoholics especially susceptible to the above problem?
Answer: Often they have coagulopathies, which puts them at high risk, also prone to falling over when intoxicated.
‫ـــــــــــــــــــــــــــــــــــــــــــــــــــــــــــــــــــــــــــــــــــــــــــــــــــــــــــــــــــــــــــــــــــــــــــــــــــــــــــــــــــــــــــــــــــــــــــــــــــــــــــــــــــــــــــــــــــــــــــــــــــــــــــــــــــــــــــــ‬
91- Question id: 2056
A 76 yo man presented with a sudden onset tearing chest pain radiating to his back. His CXR is shown.

Fig 1
(a) What is the differential diagnosis?(Name five)
Answer: Myocardial ischemia due to an ACS with or without ST segment elevat., pericarditis, PE, aortic regurgitat.
without dissection, aortic aneurysm without dissection, musculoskeletal pain, mediastinal tumors, pleuritis,
cholecystitis, atherosclerotic or cholesterol embolism, PUD or perforating ulcer, acute pancreatitis.
(b) Name four findings on a CXR which are consistent with aortic dissection?
Answer: Widening of the aorta, pleural effusion , widening of the aortic contour, displaced calcification, aortic
kinking,a pleural cap & opacification of the aorticopulmonary window.
(c) What are the risk factors for aortic dissection(Name five)?

37
Answer: HTN, preexisting aortic aneurysm, inflammatory diseases that cause a vasculitis , disorders of collagen , a
+ve family history, bicuspid aortic valve, aortic coarctation, turner syndrome, coronary artery bypass graft surgery
(CABG), previous aortic valve replacement, & crack cocaine.
(d) What are the potential complications of an ascending aortic aneurysm?(Name five)
Answer: Acute aortic insufficiency, acute myocardial ischemia or MI, cardiac tamponade & sudden death ,
hemothorax & exsanguination , neurologic deficits, horner syndrome , & vocal cord paralysis.
(e) How are aortic dissections classified?
Answer: The Daily system classifies dissections that involve the ascending aorta as type A, regardless of the site of
the 1ry intimal tear, & all other dissections as type B. In comparison, the DeBakey system is based upon the site of
origin with type 1 originating in the ascending aorta & propagating to at least the aortic arch, type 2 originating in &
confined to the ascending aorta, & type 3 originating in the descending aorta & extending distally or proximally.
‫ـــــــــــــــــــــــــــــــــــــــــــــــــــــــــــــــــــــــــــــــــــــــــــــــــــــــــــــــــــــــــــــــــــــــــــــــــــــــــــــــــــــــــــــــــــــــــــــــــــــــــــــــــــــــــــــــــــــــــــــــــــــــــــــــــــــــــــــ‬
92- Question 4513
An 18 yo male presented to the ED following a collapse at a local night club. O/E he was drowsy. His temp. was 40
degrees & he was sweating profusely. His HR was 120 bpm & regular. His Bl. Pr. was 170/100 mmHg. His pupils
were dilated & reacted poorly to light. His Bl. investigations revealed a Na of 124 mmol/l.

(a) What is the most likely cause of his presentation?


Answer: Ecstasy(MDMA) abuse.
(b) What are the possible complications of this presentation?(list four)
Answer: Rhabdomyolysis, ARF, DIC, acute hepatitis, MI, & CVA.
(c) Give 2 therapeutic steps in the management of his temp..
Answer: Cooling/tepid sponging, paracetamol & IV dantrolene.
(d) Give 2 explanations for the hyponatremia?
Answer: SIADH, & excessive Na loss from skin during profuse perspiration.
(e) Name some other causes of hyperpyrexia? (Name four)
Answer: Septicaemia, malaria, viral infections, neuroleptic malignant syndrome, malignant hyperpyrexia, cocaine
abuse, malignancy, & aspirin toxicity.
‫ـــــــــــــــــــــــــــــــــــــــــــــــــــــــــــــــــــــــــــــــــــــــــــــــــــــــــــــــــــــــــــــــــــــــــــــــــــــــــــــــــــــــــــــــــــــــــــــــــــــــــــــــــــــــــــــــــــــــــــــــــــــــــــــــــــــــــــــ‬
93- Question id: 2116
A 59 yr old lady with a history of IHD presents with tiredness & SOB. Her initial observations show a HR of 59
b.p.m. Bp of 140/84 & sats of 100%.

Fig 1
(a) What part of the history is key to making any diagnosis here?
Answer: Is she on beta-blockers? If yes then these may be normal observations, also if she was an especially fit 59 yo
it is possible that this represents a normal HR.
(b) What is first degree AV block?
Answer: Prolonged PR interval i.e. > than 0.2 seconds (5 small squares on st&ard ECG) in itself it is benign but it
may represent IHD, digoxin toxicity, electrolyte disturbances, acute rheumatic carditis
(c) Mobitz type 2, & mobitz type1 (Wenkebach type) are both types of 2ry degree heart block. Which one is
benign & which can lead to complete heart block?
Answer: Wenchebach is normally benign. Mobitz 2 & 2:1 block can lead to third degree �complete heart block.
(d) What does this ECG show (fig 1)?
Answer: Complete heart block (CHB)/third degree block
(e) How would you treat it in the ED if the Pt were unstable?
Answer: Atropine, adrenaline then transcutaneous pacing. Temporary measures before transvenous pacing can be arranged.
‫ـــــــــــــــــــــــــــــــــــــــــــــــــــــــــــــــــــــــــــــــــــــــــــــــــــــــــــــــــــــــــــــــــــــــــــــــــــــــــــــــــــــــــــــــــــــــــــــــــــــــــــــــــــــــــــــــــــــــــــــــــــــــــــــــــــــــــــــ‬
94- Question 3 of 10
A 60 yo man with diet controlled type II DM & HTN was found collapsed at the bottom of the stairs in his home by
his son. He was on a thiazide diuretic. On exam he was drowsy, his HR was 40 bpm, his BP was 150/95mmHg. His
temp. was 36.4 degrees & his JVP was not raised. The heart sounds were normal & his chest was clear.His Rt lower
limb was externally rotated & painful to move, there was extensive bruising on his Rt buttock & thigh.
38
(a) How would you investigate this Pt?(List 6)
Answer: FBC, UE, LFTs, Glucose, CK, urinalysis, Hip X Ray, ECG,CXR & CT Brain.
(b) His renal profile revealed that his urea was 15 mmol/l & his creatinine was 700 ummol/l.His CK was also
grossly elevated. What is the diagnosis?
Answer: Rhabdomyolysis
(c) What causes the renal failure in this condition?
Answer: Skeletal muscle trauma, inflammat. or infarct. causes  myoglobin levels in the Bl. which is toxic to the renal tubules.
(d) Name four other causes of this condition?
Answer: Electrocution, hypothermia, status epilepticus, ecstasy/amphetamine abuse, burns, septicaemia, statins,
strenuous exercise, neuroleptic malignant syndrome.
(e) How should this Pt be treated?
Answer: Hydration with alkalinization of the urine.
‫ـــــــــــــــــــــــــــــــــــــــــــــــــــــــــــــــــــــــــــــــــــــــــــــــــــــــــــــــــــــــــــــــــــــــــــــــــــــــــــــــــــــــــــــــــــــــــــــــــــــــــــــــــــــــــــــــــــــــــــــــــــــــــــــــــــــــــــــ‬
95- Question 4 of 10
A 26 yo male presents to the ED with abd. pain & diarrhoea. He tells you that he hasn't felt well for about 6 months but in
the last few wks he has noticed that he has been losing weight & opening is bowels up to 8 times a day, since yesterday
he has had worse abd. pain. O/E he was pale & slim, is abd. was soft & mildly tender throughout with no guarding.

(a) What are the likely differentials in this case?


Answer: 1. IBD- UC/Crohn�s 2. Coeliac disease 3. Any other cause of abd. pain is of course is a possibility here i.e.
appendicitis (diarrhoea can be associated!) 4. Gastroenteritis (unlikely due to chronic Sx but need to ask about foreign
travel for things like giardia & tapeworms etc) 5. Intestinal lynphangiectasia 6. Hypogammaglobulinaemia 7. Small
bowel lymphoma 8. C.diff?? ask re Abx (despite age being young; it is known!)
(b) What is the current laboratory test for celiac disease?
Answer: IgA anti-endomyosial antibodies.
(c) You take some Bl.s what other important investigation needs to be done in the ED?
Answer: Plain abd. film (1 of the few reasons that 1 still needs to do a plain abd. film) to rule out a toxic megacolon.
(d) How do you treat confirmed case of c.difficle?
Answer: Oral metronidazole in the first instance- vancomycin orally if not working.
(e) What group of Pts get caecal volvulus?
Answer: 20-40 presenting with an acute onset of small bowel obstruction.
‫ـــــــــــــــــــــــــــــــــــــــــــــــــــــــــــــــــــــــــــــــــــــــــــــــــــــــــــــــــــــــــــــــــــــــــــــــــــــــــــــــــــــــــــــــــــــــــــــــــــــــــــــــــــــــــــــــــــــــــــــــــــــــــــــــــــــــــــــ‬
96- Question 5 of 10
A 75 yo lady presents with acute pulm. oedema. Your system tells you that she has presented 7 times in the last 5
months with pulm. oedema. An ECHO last time showed a near normal LV function with a gradient across the aortic
valve of 69mmHg. O/E she is SOB has an ESM & a slow rising pulse. She is normally SOB on minimal exertion.

(a) What is the diagnosis here?


Answer: Severe aortic stenosis
(b) What does this Pt need to definitively treat her condition?
Answer: An aortic valve replacement, she is symptomatic with severe AS.
(c) Is this Pt more or less likely to develop Torsades de pointes than a gentleman with a similar presentation?
Answer: More likely, this sex difference is unexplained.
(d) Can Pts with a near normal LV function be classed as having heart failure?
Answer: Yes population based studies report that around a third of Pts with CCF have normal or near normal EF. The
concept of diastolic heart failure is perhaps under recognised in clinical practise. It can only be separated from systolic
heart failure by Doppler echocardiography.
(e) Which of the following medications have been shown to improve prognosis post M.I.? Nicorandil, flecanide,
nitrates, clopidogrel, Ca antagonists
Answer: Clopidogrel
‫ـــــــــــــــــــــــــــــــــــــــــــــــــــــــــــــــــــــــــــــــــــــــــــــــــــــــــــــــــــــــــــــــــــــــــــــــــــــــــــــــــــــــــــــــــــــــــــــــــــــــــــــــــــــــــــــــــــــــــــــــــــــــــــــــــــــــــــــ‬
97- Question id: 4538
A 56 yo male attends the ED with an episode of syncope. It occurred whilst he was out walking his dogs. He smokes
15 a day & has done for yrs; he has controlled HTN & is on a statin. O/E he has an ESM & a soft second heart sound.

39
Fig 1
(a) He feels completely fine now & wants to go home, what do you need to do in the ED?
Answer: The Pt likely has cardiac syncope related to severe AS. He needs to be admitted & needs a fairly urgent
echocardiogram to assess the aortic valve. He will likely need to have it replaced & will therefore also need
angiography prior to this to guide the cardiac surgeons
(b) You perform an ECG (figure 1). Describe what it shows:
Answer: ECG showing gross left ventricular hypertrophy (LVH) with strain in case with severe aortic stenosis. The R
waves in V5 & V6 are so tall that they are overlapping with the tracing in the channel above. ST segment depression
& T wave inversion are seen in inferior & lateral leads. This is a Pr. overload pattern which can be seen also in severe
systemic HTN & hypertrophic obstructive cardiomyopathy.
(c) What is the next step in this Pts' management? 1. Start an ACEi? 2. Tredmill test 3. Percutaneous aortic
balloon valvulotomy 4. Give flecanide 5. Amiodarone 300mg IV over 30 minutes 6. All of the above 7. None of
the above
Answer: Answer= 7. This Pt if shown to have what you believe clinically to be severe aortic stenosis will need his
valve replacing, valvulotomy is only really used as a bridge to surgery in unstable Pts.
(d) What is the current guidance regarding antibiotic prophylaxis for Pt undergoing dental procedures who have valvular
HD?
Answer: That it is NOT REQUIRED. New guidance in 2006 from British Society for Antimicrobial Chemotherapy
(BSAC) states that it is no longer required as there is no evidence that it leads to BE. HOWEVER THIS IS
CONTROVERSIAL & a lot of cardiologists do not agree- we await NICE guidance on this.
‫ـــــــــــــــــــــــــــــــــــــــــــــــــــــــــــــــــــــــــــــــــــــــــــــــــــــــــــــــــــــــــــــــــــــــــــــــــــــــــــــــــــــــــــــــــــــــــــــــــــــــــــــــــــــــــــــــــــــــــــــــــــــــــــــــــــــــــــــ‬
98- Question id: 4527
An 88-yo lady is brought in by ambulance. They were on route to the medical admissions unit but felt that the Pt was
too unwell. She is from a nursing home & has long standing dementia she was sent in by the GP due to a general deteriorate.
& possible dehydrate. The reason the crew became concerned was due to brief periods of unresponsiveness that
seemed to be occurring quite frequently. She has a past history of CVA, IHD, HTN, NIDDM & dementia. She is on asprin,
clopidogrel, simvastatin, ramipril, digoxin, bisoprolol & metformin. She is normally bed bound & fully dependant for all ADLs.

(a) List 4 investigations that are important in the initial care of this Pt.
Answer: BP, pulse, sats, ECG, CXR, Bl. gases (for electrolytes & Bl. sugar) & to see if acidotic. ECG is crucial.
(b) What does the ECG in figure 1 show?
Answer: Complete heart block. The ventricular pacing rate has taken over as there is complete dissociation between
the atria & the ventricles.
(c) What is occurring & how would you manage this Pt initially?
Answer: She is having syncopal episodes related to runs of asystole. She needs to be fully monitored using a
defibrillator. Try atropine in 500mcg increments to a max of 3mg.
(d) What are the adverse signs according to the resus council UK that you need to treat when considering
bradyarrhythmias?
40
Answer: Systolic <90, HR <40, Ventricular arrhythmias compromising BP, Heart failure
(e) Thinking of possible causes of the picture described in this Pt what potential reversible causes can be
identified from the history given?
Answer: Drugs! Digi-toxic or B-blocker overdose? Consider addressing these 2 issues need to check to digoxin level
also consider glucagon for reversing B-blocker effect. Electrolyte abnormalities are also potential reversible causes to
be considered ion complete heart block.
(f) The Pt's HR appears to drop to around 20 b.p.m & she continues to have runs of asystole associated with no
output. What will you do?
Answer: On the one h& the Pt needs to be paced urgently- this could be done by transcutaneous pacing until trans-
venous pacing can be established. However in the above Pt the entire picture needs to be considered. She is very
unlikely to do well in this scenario & there are significant risks involved with placing a trans venous pacing wire. It
may be better to simply monitor the Pt & aim to keep her comfortable.
‫ـــــــــــــــــــــــــــــــــــــــــــــــــــــــــــــــــــــــــــــــــــــــــــــــــــــــــــــــــــــــــــــــــــــــــــــــــــــــــــــــــــــــــــــــــــــــــــــــــــــــــــــــــــــــــــــــــــــــــــــــــــــــــــــــــــــــــــــ‬
99- Question 8 of 10
A 19 yo student presented to the ED with a headache. He lived with 2 other students who found him after he failed to
answer a wake up call. O/E he was flushed & drowsy.There was a cherry red discoloration to his lips. He was afebrile,
he didn't have a skin rash. His HR was 95 b/m & his Bl. Pr. was 130/90 mmHg. His GCS was 11/15. There was no
nuchal rigidity. The CNS & PNS exam. were normal. His investigations revealed a normal CBC, renal profile & electrolyte
profile. His ABG  a pH in the normal range, a ↓ PaO2(7.8 kPa) & a ↓ PaCO2 (3.6 kPa). His SpO2 was 98% on room air

(a) What is the most likely diagnosis?


Answer: Carbon monoxide poisoning.
(b) Explain the arterial Bl. gas results?
Answer: Carbon monoxide displaces O2 from Hb.
(c) Explain why the pulse oximeter reading is normal?
Answer: Pulse oximeter analysers cannot differentiate between oxyHb & carboxyHb. PaO2 is low when there is
significant carbon monoxide poisoning.
(d) Name some common sources of this condition?(Name 2)
Answer: Combustion engines, faulty stoves, paraffin heaters with poor ventilation facilities.
(e) What is the TTT for this condition?
Answer: Administration of 100% O2. Pts with neurological signs & symptoms, ECG abnormalities, myocardial
ischaemia, pulmonary oedema & shock require hyperbaric O2 at a specialised centre.
‫ـــــــــــــــــــــــــــــــــــــــــــــــــــــــــــــــــــــــــــــــــــــــــــــــــــــــــــــــــــــــــــــــــــــــــــــــــــــــــــــــــــــــــــــــــــــــــــــــــــــــــــــــــــــــــــــــــــــــــــــــــــــــــــــــــــــــــــــ‬
100- Question id: 2084
This ECG is from a 76 yo man who presented with central chest pain & nausea.

Fig 1

Fig 2
41
(a) What does the ECG in figure 1 show?
Answer: Infero-posterior M.I., would accept inferior M.I., with lateral reciprocal changes.
(b) The Pt seems to deteriorate & a repeat ECG (fig 2) shows the following: Explain why this has occurred
referring to the anatomy of the coronary arteries.
Answer: When The Pt has suffered an occlusion of the Rt coronary artery (RCA) the infero-posterior ischaemic
changes in the first ECH demonstrate this.The RCA supplies the SA node, the AV node & the entire posterior surface
of the heart. They can therefore lead to dangerous arrhythmias.
(c) When faced with the ECG in (figure 1) what additional investigations would you like to perform?
Answer: Posterior leads. To do true posterior leads, here�s what you do: take all the chest lead wires off. Now stick
on three more chest electrodes along the same line of V5 & V6, along the fifth intercostal space, using the same
spacing that you used for the chest leads, ending up under the scapula: V7, V8, & V9. Now start reattaching the wires:
put the V1 lead wire on the V4 electrode. See? The V2 lead goes on the V5 electrode. & so on around the chest. Now
when you do your 12-lead, you�ll get a clear picture of what the entire RV is doing: inferiorly & posteriorly.
(d) Name 3 acute complications of STEMI
Answer: Continuing chest pain, fever, new systolic murmur (VSD, MR or Pericarditis), dysrrhythmia (VT, AV block
ectopics & bradycardia), cardiogenic shock.
‫ـــــــــــــــــــــــــــــــــــــــــــــــــــــــــــــــــــــــــــــــــــــــــــــــــــــــــــــــــــــــــــــــــــــــــــــــــــــــــــــــــــــــــــــــــــــــــــــــــــــــــــــــــــــــــــــــــــــــــــــــــــــــــــــــــــــــــــــ‬
101- Question id: 2115
A 76 yo gentleman presents with the ECG above, his HR is as shown & he is symptomatic.

(a) What is the rate?


Answer: Accept 35-39 b.p.m. 300/8 = 37.5, 8-8.5 large squares.
(b) What is the rhythm called?
Answer: Its 2ry degree heart block, mobitz type 2.

(c) What is your initial pharmacological management including dose?


Answer: Atropine 500 mcg or glycopyrolate 200-600 mcg.
(d) After the drug you gave there was no response. What do you do next?
Answer: Repeat atropine up to 3mg, rpt glycopyrolate as necessary.
(e) You have an external pacing device available, explain exactly what you would do & how it works to set it up
including anything you would do to the Pt. e) When do ventricular pauses become concerning?
Answer: Consider sedative & analgesia as can be uncomfortable, if clinical state will allow then give morphine &
midazolam (cautious in elderly) Explain to the Pt that will feel uncomfortable Apply sticky pads to the chest & to the
back (AP paddles) Select external demand pacing mode on the defibrillator & set the rate to 70 b.p.m Then start to
dial up the pacing current from zero until you see that a beat had been captured on the monitor. Clinically a capture
beat results in a peripheral pulse & an improvement in the Pts condition. Ensure that this occurs despite the monitor
showing a captured beat. Answer; always of concern but generally if pauses are lasting > 3 seconds then something
needs to be done sooner rather than later.
‫ـــــــــــــــــــــــــــــــــــــــــــــــــــــــــــــــــــــــــــــــــــــــــــــــــــــــــــــــــــــــــــــــــــــــــــــــــــــــــــــــــــــــــــــــــــــــــــــــــــــــــــــــــــــــــــــــــــــــــــــــــــــــــــــــــــــــــــــ‬
102- Question id: 2111
A known alcoholic lady downs an entire bottle of antifreeze she wrote a suicide note & was found
unconscious by her neighbour

(a) Explain why it is toxic, what does it contain?


Answer: The substance is ethylene-glycol, it is toxic due to its metabolites.
(b) Give 2 antidotes & explain why they work?
Answer: Alcohol & fomepizole. They work by inhibiting the metabolism of ethylene glycol & hence preventing the
formation of metabolites.
(c) What are the presenting features assuming that you have no history of antifreeze ingestion?
Answer: Pt appears drunk (but doesn't smell of ETOH) Ataxia Dysarthria Nausea/vomiting Haematemesis

42
(d) In severe poisoning like the lady described above what is going to be the likely course of management &
where will she be managed?
Answer: On ITU Haemodyalysis Correction of acidosis
(e) What metabolic disturbance must one be especially vigilant for? & how is it treated?
Answer: Hypocalcaemia, which can be severe, treated with Ca gluconate.
‫ـــــــــــــــــــــــــــــــــــــــــــــــــــــــــــــــــــــــــــــــــــــــــــــــــــــــــــــــــــــــــــــــــــــــــــــــــــــــــــــــــــــــــــــــــــــــــــــــــــــــــــــــــــــــــــــــــــــــ‬
103- Question id: 4549
A 68 yo gentleman is brought in by ambulance after being found collapsed at his home. He is covered in faeces &
appears very thin & unkempt. He has had some malaena. The paramedics do not know what happened today & had to
break into his house. The Pt's daughter called them, as she hadn't heard from him in 2 days. He is an alcoholic who
drinks a litre of vodka a day.

(a) O/E he had following observat.: A- own B- Clear sats 98% in air C- PR 120 BP 81/40 D- GCS 13 E- abd.
soft - maleana present on the sheets. You discover from the notes that he has had 2 previous GI bleeds & on the
last attendance he refused an OGD & was treated as a presumed bleed. You instigate initial resuscitat.- list 8 things that
you do?
Answer: 1- IV fluids 2- IV pabrinex 3- Check Bl. glucose 4- ECG 5- CXR 6- Bl. test including cultures 7- Bl. gas 8-
Speak to any family, try to gain as much pre morbid functional status as possible
(b) He stabilises a little & his Bl. Pr. improves. His GCS remains 13. He stabilises a little & his Bl. Pr. improves.
His GCS remains 13. Pending Bl. test results you speak to the on-call enoscopist. What is the next most
important investigation/intervention? Bl. glucose is 6.1
Answer: Need to work out why GCS is 13. Look for signs of trauma will likely need a CT head if hasn't improved
after initial resuscitation in the ED. ? has had a sub-dural etc
(c) Bl. tests come back as follows ALT- 112 GGT- 980 Bili- 73 ALKP- 442 Alb- 38 Lactate- 10.3 Na+ 149 K+ 4.8
Ur 3.2 Cr 172 Hb 12.6 Plts 263 WBC 12.3 Clotting normal Lipase- 5479 Amylase 332 How does this affect the
DD? What could be going on? Which tests results are most concerning?
Answer: Pt is clearly unwell with a lactate of 10.3, liver function is grossly deranged but renal function is not too far
abnormal- the urea is normal & the haemoglobin is also normal meaning that any GI Bl. loss is likely not to be the
most significant thing occurring here. The lactate is the most concerning Bl. test. Differential- diagnosis: pancreatitis?
Ischaemic gut? Alcoholic ketoacidosis? GI bleed with perforation? ?
(d) His CXR is normal as is the ECG. There is no sign of ascites & he is not septic, abdominal exam. is
unremarkable. Urinalysis reveals 4+ ketones. What is the diagnosis?
Answer: This case is alcoholic ketoacidosis
‫ـــــــــــــــــــــــــــــــــــــــــــــــــــــــــــــــــــــــــــــــــــــــــــــــــــــــــــــــــــــــــــــــــــــــــــــــــــــــــــــــــــــــــــــــــــــــــــــــــــــــــــــــــــــــــــــــــــــــ‬
104- Question id: 2097
A 69 yo male attends with a history of general malaise over the past 3 weeks. On further questioning he has had bony
pains in his back & in his ribs for several wks that he attributed to ?old age?. A CXR is normal. His vision has been a
bit ?blurry? over the last few days. Bl.s show: Hb 9.0, MCV 83 fL, MCH 29pg, MCHC 34g/dl WCC 8.4, Plts 334
Urea 35.6; Creat 587; Na 138; K 7.9 Ca 3.05; Alk P 220u/L

(a) Give two possible diagnoses


Answer: Multiple myeloma: Hyperparathyroidism (& renal failure) Vit D excess (sarcoidosis & thyrotoxicosis)
Hypercalcaemia of malignancy (expect elevated Alk P)
(b) Give 5 TTT options available (multiple TTTs for the same abnormality not accepted)
Answer: Re-hydraton is fairly crucial, Rx hyperkalaemia: nebs, Insulin/Glc. Not Ca (?) ARF: Dialysis,
haemofiltration? Hypercalcaemia: Fluids & steroids- hydrocortisone Pamidronate (malignancy) if the hypercalcaemia
persists: For myeloma also consider prednislone 30-60mg Calcitonin
(c) How could you confirm the diagnosis?
Answer: BJ protein, serum/urine electrophoresis, bone marrow,
‫ـــــــــــــــــــــــــــــــــــــــــــــــــــــــــــــــــــــــــــــــــــــــــــــــــــــــــــــــــــــــــــــــــــــــــــــــــــــــــــــــــــــــــــــــــــــــــــــــــــــــــــــــــــــــــــــــــــــــ‬
105- Question id: 4510
A 65 yo male presented to the ED with a 3 hr history of chest pain, palpitations & breathlessness. His P/H was of
HTN & a MI His regular medicat. include aspirin 75mg , frusemide 40mg & atorvastatin 20mg.

43
(a) What is shown on the ECG?
Answer: Ventricular Tachycardia
(b) List the ECG features of this condition(List four)
Answer: Broad complex QRS, Extreme axis deviation, positive or negative concordance in the precordial leads, RSr
pattern in V1, Deep S-wave in V6, Fusion & Capture beats, Dissociated p-waves
(c) What drug may be used to treat this condition?
Answer: Amiodarone
(d) Shortly after administering the drug the Pt becomes clammy & cyanosed. His conscious level deteriorates &
his Bl. Pr. is low. What is the next step in management?
Answer: DC Cardioversion
(e) From which ventricle does the above rhythm usually arise?
Answer: Left ventricle.
‫ـــــــــــــــــــــــــــــــــــــــــــــــــــــــــــــــــــــــــــــــــــــــــــــــــــــــــــــــــــــــــــــــــــــــــــــــــــــــــــــــــــــــــــــــــــــــــــــــــــــــــــــــــــــــــــــــــــــــ‬
106- Question id: 2019
A 64 yo lady is brought in vomiting she has had haematemesis for the last 2 hrs & has just passed a large volume of PR Bl..

(a) What anatomical point differentiates an upper from a lower gastronintestinal bleed?
Answer: Ligament of Treitz it inserts as nonstriated muscle commonly into the third & fourth portions of the
duodenum & frequently into the duodenojejunal flexure as well.
(b) Is it safe to assume that the above Pt is having an upper or a lower gastrointestinal bleed (LGIB)?
Answer: It is unclear. You need to know if the Bl. is classical malaena or fresh Bl. but be warned LGIB can present
with fresh PR Bl. loss. 15% of LGIB present as UGIB.
(c) List 5 potential causes of an upper GI bleed.
Answer: ? Ulceration stomach or duodenum ? Inflammation: oesophagitis/gastritis/duodenitis ? Mallory Weiss tear ?
Warfarin or clotting disorders ? Gastric or oesophageal malignancy ? Oesophageal varices
(d) Name a scoring system for risk stratifying upper GI bleeds & list 4 criteria that it focuses on.
Answer: The Rockall score 1 Variable Score 0 Score 1 Score 2 Score 3 Age <60, 60- 79, >80 Shock No shock PR
>100 SBP <100 Comorbidity Nil major CCF, IHD, major morbidity Renal failure, liver failure, metastatic cancer
Diagnosis Mallory-weiss All other diagnoses GI malignancy Evidence of bleeding None Bl., adherent clot, spurting vessel
(e) What is the commonest type of LGIB?
Answer: UGIB followed by diverticular bleeds.
(f) The Pts Hb comes back at 4.4 how many units of Bl.s will you give?
Answer: The aim of transfusion should be to minimize risk whilst improving the clinical situation. Therefore the
smallest volume of Bl. should be given aiming for a Hb> 7. 3-4 units initially but if ongoing bleeding then more Bl.
will be needed as will clotting factors & potentially platelets.
(g) What methods can be used to control a variceal UGIB prior to endoscopy?
Answer: Telipressin 2mg IV 4-6 hrly & possible insertion of a Sengstaken/Minnesota tube. 1.
‫ـــــــــــــــــــــــــــــــــــــــــــــــــــــــــــــــــــــــــــــــــــــــــــــــــــــــــــــــــــــــــــــــــــــــــــــــــــــــــــــــــــــــــــــــــــــــــــــــــــــــــــــــــــــــــــــــــــــــ‬
108- Question id: 2153
You see a 94 yo nursing home resident who has been brought in as she is confused. The nurse with her states that she
is not normally confused.

44
(a) List 3 simple ED tests that are crucial in this case.
Answer: Urinalysis Baseline observations Temperature BM AMMT Breath alcohol
(b) You take a very detailed history & perform a through exam.. All base line observations are normal apart
from a temp of 38.3?C. The nurse tells you that she had been cold this morning & they had out on the gas fire
in her room for the first time this yr. Would you do any further tests in light of the above information?
Answer: Need to perform an ABG for CO. Possible that she could have carbon monoxide poisoning.
(c) What other investigations would you like?
Answer: CXR, ECG, FBC, U&E Bl. glucose.
(d) The urinalysis is grossly positive & on repeat questioning the nurse had noticed that her urine had been
rather pungent over the last 48 hrs. What will you do now?
Answer: Pt needs admission Bl.s cultures Send MSU to the lab Empirically treat likely with oral antibiotics initially if
Pt can swallow them. Trimethoprim 200mg/BD
(e) Name 2 pathogens that commonly cause UTIs
Answer: E coli spec Enterococcus faecalis Klebsiella pneumoniae Proteus mirabilis Bacteriodes Pseudomonas aeruginosa
‫ـــــــــــــــــــــــــــــــــــــــــــــــــــــــــــــــــــــــــــــــــــــــــــــــــــــــــــــــــــــــــــــــــــــــــــــــــــــــــــــــــــــــــــــــــــــــــــــــــــــــــــــــــــــــــــــــــــــــ‬
109- Question id: 2120
A 71 yo lady presents "off legs" she has been in a nursing home for the last few months as she could no longer cope at
home due to her metastatic breast malignancy. She appears dehydrated & a little confused. The nursing home staff
state that she has mobile yesterday, they also tell you that she was doubly incontinent today which is unusual for her.

(a) If you could only perform 2 aspects of clinical exam. in this case to ascertain the main problem which 2
would you chose? (e.g. cardiovascular exam & exam. of the fundi)
Answer: A PR (to check for anal tone & sensation) A complete lower limb neurological exam.. Looking for evidence
of spinal chord compression.
(b) What investigation do you try to organise?
Answer: MRI to image the spinal chord
(c) Which Bl. tests are you especially interested in?
Answer: U&E & Ca are of particular interest, hypercalcaemia is a very common cause of confusion in these Pts.
(d) How will the primary problem described in a) be managed?
Answer: Normally radiotherapy but sometimes it may be appropriate for no TTT to occur & analgesia might be the
mainstay of TTT.
‫ـــــــــــــــــــــــــــــــــــــــــــــــــــــــــــــــــــــــــــــــــــــــــــــــــــــــــــــــــــــــــــــــــــــــــــــــــــــــــــــــــــــــــــــــــــــــــــــــــــــــــــــــــــــــــــــــــــــــ‬
110- Question id: 4525
A 57 Yo with known COPD comes in- she appears to be having an exacerbat, her initial observat. are as follows- a-
talking in broken sentences but drowsy, B- sats 83% widespread wheeze poor AE, RR 43, c- PR 115, d GCS 14 (drowsy)

(a) In general who should receive O2?


Answer: Only hypoxaemic pts should get O2 therapy- there is no evidence at all that O2 benefits pts who are
dyspnoeic but not hypoxic. There is increasing evidence that too much O2 may actually do harm.
(b) What are the first & second things that you should do?
Answer: Firstly: ensure that the pt is on O2- BTS guidelines (see reference) would advice checking for an O2-
warning card (i.e. to find out if known to retain & are they on home O2 etc, in a perfect world they would come in
with their own venturi!), This pt would be treated in accordance with table 1 in the guideline as she is crtically ill so
would be put on a non-rebreathe mask pending gases then maintained at 94-98% sats if normal CO2; if CO2  then
aim for 88-92%. Then on this basis go for either 28% via venturi or 24% via venturi. Of course if this doesn't work
(aiming for sats 88-92%) then put on higher flow O2/consider NIV etc. Also note that the guidelines suggest  the
flow to 50% above the minimum flow rate suggested on the venturi if the pts R is >30, The 2nd step needs to be an ABG.
(c) What is now encouraged for the crews to be doing pre-hospital?
Answer: ? Answer to start these pts on 28% O2 via venturi! ?
(d) What should happen if sats go above 92%?
Answer: O2 concentration should be turned down?
(e) What are the 2 sats ranges to aim for hypoxic Pts suggested in the BTS guideline on emergency O2 use?
Answer: 88-92% for known COPD, or others at risk of hypercapnic respiratory failure (pending ABG result) Adjust
to the higher range if the pCO2 is normal & no history of hypercapnic respiratory failure. 94-98% for anyone else. ?
(f) How should nebulised therapy be given for the above Pt?
Answer: Should be driven in air, any O2 required should be given via nasal cannulae. If this is not available then
drive by O2 but limit this to 6 minutes.
‫ــــــــــــــــــــــــــــــــــــــــــــــــــــــــــــــــــــــــــــــــــــــــــــــــــــــــــــــــــــــــــــــــــــــــــــــــــــــــــــــــــــــــــــــــــــــــــــــــــــــــــــــــــــــــــــ‬
111- Question id: 2177
45
A 46 yo woman with bipolar disorder presented as she had mistakenly taken too many of her lithium tablets. Her
previous medicat. had been discontinued & she had been started on lithium the previous wk. She was taking the lithium
tablets according to her previous medication?s dosing regime. This had resulted in her taking excess lithium tablets
over the course of the week , a fact which she had only discovered on the day of presentation.

(a) What are the clinical features of lithium poisoning?


Answer: Nausea, vomiting, diarrhoea, tremor, ataxia, confusion,  in muscle tone, clonus, convulsions, coma & renal failure.
(b) What are the two most important Bl. investiagtions?
Answer: UE & lithium level.
(c) What is the normal range for therapeutic lithium levels?
Answer: <1.2mmol/L
(d) What is the role of activated charcoal in this Pt's TTT?
Answer: Activated charcoal does not absorb lithium.
(e) How would you manage this Pt?(Give four)
Answer: Gastric lavage is indicated if within 1 hr of a single large overdose,consider poisons consultation, control
convulsions with benzodiazepines, haemodialysis for severe poisoning.
‫ــــــــــــــــــــــــــــــــــــــــــــــــــــــــــــــــــــــــــــــــــــــــــــــــــــــــــــــــــــــــــــــــــــــــــــــــــــــــــــــــــــــــــــــــــــــــــــــــــــــــــــــــــــــــــــ‬
112- Question id: 4553
A 27 yo Turkish man presents to the ED at the weekend sent in from the walk in centre, he has painful legs & has
noticed a rash. He lives in a poor area of the inner city.

Fig 1
(a) What is the rash shown in figure 1?
Answer: Erythema nodusom?
(b) List 5 causes of this rash?
Answer: 1. Crohn?s/colitis 2. TB 3. HIV 4. Drug induced 5. St. infect. (beta haemolytic) 6. Sarcoidosis 7. Leprosy &
other infections( Yersina, toxoplasmosis, histomplasmosis, Chlamidya) 8. SLE 9. Beh's disease ?
(c) What important questions do you ask in the history to try to help you with the diagnosis? Limit the answer
to the 4 most important questions. (Remembering that common things are common)
Answer: ? Ask about bowel habit/abdo pain & rectal bleeding/ features of IBD ? Ask about recent travel history &
possible TB contact ? Take a complete drug history ? Ask about recent infection (anything to suggest strep sore throat
etc) This probably covers the most common causes of erythema nodusum ?
(d) What base line investigations would be useful & why?
Answer: CXR: look for evidence of TB & sarcoid, FBC- looking for anaemia (IBD), ESR & CRP looking for
inflammation (vadculitis/IBD), ASO titer, Urinalysis, Throat culture, Intradermal tuberculin test
(e) He is s smoker & tells you that he has a cough from time to time. From your screening questions that you
chose above you decide that he doesn't seem to have any of the risk factors for common causes of this type of
rash. You decide to investigate further. You find out that he has had some urthethritis but denies sexual
intercourse in the last 6 months he has also noticed that he has intermittently painful joints. You notice some
mouth ulcers O/E. What could the diagnosis be?
Answer: Beh's disease?
‫ــــــــــــــــــــــــــــــــــــــــــــــــــــــــــــــــــــــــــــــــــــــــــــــــــــــــــــــــــــــــــــــــــــــــــــــــــــــــــــــــــــــــــــــــــــــــــــــــــــــــــــــــــــــــــــ‬
113- Question id: 4562

46
A 56 yo manager comes in after experiencing some palpitations. He tells you that he has experienced palpitations off
& on for a No. of yrs but has never worried about them. Today he felt as if they lasted longer than previous episodes.
He is found to be in atrial fibrillation with a rate of 76 b.p.m

(a) What information is useful to know about this gentleman?


Answer: Does he have any structural HD (ideally has he had an ECHO), Is he on any medicat. that could precipitate AF, Does he
have a possible driving factor for AF i.e. infect./ETOH/thyroid disease etc, Are there any old ECG tracings showing NSR?
(b) According to NICE guidance what will determine if the Pt needs to be anti-coagulated prior to attempting rhythm
control?
Answer: The duration of the AF. If it is less than 48 hrs then no need for anticoagulation. Most parties would give
LMWH to Pts who they thought might be cardioverted acutely.
(c) What drug therapies are available in the above scenario?
Answer: Flecanide would be a good option provided that he doesn't have any structural heart disease. If he does then
amiodarone would be the preferred agent.
(d) Before you decide what you are going to do the Pt becomes unresponsive & drops his BP to 76/43. The
anaesthetist is unavailable for the next 30 minutes as there is a trauma in the next bay what do you do?
Answer: Options here are variable of course- could give amiodarone IV to attempt to revert the arrhythmia. Ideally
needs electrical cardioversion.
(e) List 4 scenarios where rhythm control is felt to be superior to rate control when faced with a Pt in AF.
Answer: Age, < 65 aim for rhythm control, When the Pt is very symptomatic, 1st presentation of lone AF, When the
AF is 2ry to a treated or corrected precipitant, With congestive cardiac failure.
‫ــــــــــــــــــــــــــــــــــــــــــــــــــــــــــــــــــــــــــــــــــــــــــــــــــــــــــــــــــــــــــــــــــــــــــــــــــــــــــــــــــــــــــــــــــــــــــــــــــــــــــــــــــــــــــــ‬
114- Question id: 4560
An 87 yo lady is brought in from a nursing home after having had a few episodes of collapse over the last wk. She has
clearly had a fall as she has a bruised face. She seems to be well on initial assessment. She is on a plethora of medicat.

(a) List the 4 most important bits of history you want from this lady
Answer: 1. Medicat.- particularly warfarin/anticoagulants & any medications that cause bradycardia/hypotension etc
2. Normal functional status i.e. is she independent etc 3. What is her mental status today & what is normal for her. 4.
Any pre-syncopal features, i.e. is she aware that she is going to collapse
(b) What investigations do you want immediately in the ED?
Answer: 1. ECG 2. Bl. glucose 3. Postural BP recordings 4. routine Bl. tests, FBC,UE, Ca.
(c) ECG revealed: see figure 1: What does it show?
Answer: Mobitz type 2- this is mobitz 2 with 3:1 block. Mobitz Type 2 2nd degree Heart Block is considered an
important warning signal of the potential progression to 3rd degree Heart Block, which requires prompt attention.
(d) What will you do about it?
Answer: Depends if the pt is stable or unstable- If stable then can prepare for a pacemaker at the next available
opportunity If unstable then requires a temporary pacing wire to be inserted.
(e) Her heart rate drops to 38 b.p.m, what measures do you take?
Answer: Measure the BP & re-assess the Pt, if unstable then may need to instigate immediate pacing- could use
transcutaneous pacing If BP is relatively maintained could consider giving atropine (best titrated in this scenario)
‫ــــــــــــــــــــــــــــــــــــــــــــــــــــــــــــــــــــــــــــــــــــــــــــــــــــــــــــــــــــــــــــــــــــــــــــــــــــــــــــــــــــــــــــــــــــــــــــــــــــــــــــــــــــــــــــ‬
115- Question id: 4546
A 26 yo man attends the ED with haemoptysis & SOB- he appears very unwell & has had large amounts of haemoptysis for
the last hr. He says that he has felt sick & has had aching joints for the last 2 days he also mentions that he hasn�t been to
the toilet today. You do an ABG & order a CXR

47
Fig 1
(a) Gas on 15 litres of O2 pH 7.29 pO2 9.7 pCO2 4.3 HCO3 16 BE -5.3 Describe the Bl. gas picture shown
Describe the CXR (figure 1) & state 4 potential causes of the appearances.
Answer: The x-ray shows bilateral airspace shadowing which in this case is Bl. from pulmonary haemorrhage- but it
could be fluid (pulmonary oedema), lymph, or consolidation from infection. The gas shows a metabolic acidosis.
(b) What important investigations do you want to do immediately?
Answer: Clotting, renal function, complete biochemical profile, CBC, vasculitic screen- i.e. ANA, ANCA etc
(c) Some of his initial results come back- Hb 7.3 Plt 98 WBC 10.9 Na 134 K+ 6.1 Ur 25.6 Cr 435 What is the
most likely cause of the Bl. results above?
Answer: Most likely renal failure due to ATN as part of the vasculitic illness that is underlying this presentation.�
(d) What important step needs to be taken in light of the Bl. results?
Answer: ECG- look for signs of potassium toxicity i.e. tented T waves, widening of the QRS complex, slurred ST
segments, arrythmias, can lead to VF. Then if present treat with Ca gluconate to protect the myocardium.
(e) His initial observations are as follows: Pulse 100, BP 120/67, sats 94% in O2, RR 26, temp 37.4, GCS 15
Considering the Bl. gas & x-ray findings what is the DD?
Answer: Vasculitis- Goodpasture's, PAN, Wegnener's, microscopic polyangititis, Churg-Struass.
(f) The Pt is ANCA negative what is the most likely diagnosis?
Answer: Churg-Strauss
‫ــــــــــــــــــــــــــــــــــــــــــــــــــــــــــــــــــــــــــــــــــــــــــــــــــــــــــــــــــــــــــــــــــــــــــــــــــــــــــــــــــــــــــــــــــــــــــــــــــــــــــــــــــــــــــــ‬
116- Question id: 2101
A 54 yo lady with bipolar affective disorder tells you that she took a months worth of her lithium tablets you have the
boxes that she has brought in (they are slow release tablets)

(a) Would you consider using activated charcoal for this lady as she has presented within an hr of having taken the
tablets?
Answer: No as it doesn't absorb lithium
(b) Could you perform gastric lavage?
Answer: No as the slow release tablets are too large to pass up the nasogastric tube.
(c) What do you do?
Answer: In contact with a poison's specialist could consider whole bowel irrigation when slow release tablets have been taken.
(d) What are the symptoms of lithium overdose?
Answer: Nausea, vomiting, diarrhoea, are followed by tremor, ataxia & confusion. In severe cases there may be renal
failure, convulsions & coma.
(e) How would you control seizures if they occurred?
Answer: benzodiazepines: lorazepam, diazepam
‫ــــــــــــــــــــــــــــــــــــــــــــــــــــــــــــــــــــــــــــــــــــــــــــــــــــــــــــــــــــــــــــــــــــــــــــــــــــــــــــــــــــــــــــــــــــــــــــــــــــــــــــــــــــــــــــ‬
117- Question id: 4533
A 31 yo business man developed a sudden onset of sore throat, fever, diarrhoea & lethargy. He developed a rash over the
next few days affecting the face/trunk/palms & soles. He had been in Singapore 2 months previously. O/E he had cervical
lymphadenopahy a widespread rash, temp 38.4 & an erythematous pharynx. He was also c/o a non-productive cough.

(a) List some DD:


Answer: HIV seroconversion, EBV, MV infection, Acute hepatitis, TB.
48
(b) If he had glandular fever what would the most likely cause of the rash be?
Answer: Amoxicillin administration.
(c) What is common cause of diarrhoea in a Pt infected with HIV?
Answer: Cryptosporidium (supportive TTT)
(d) A CXR taken in the above Pt showed bilateral diffuse interstitial shadowing. What is the likely diagnosis?
Answer: HIV/AIDS- pneumocystis jiroveci pneumonia (formerly PCP)
(e) What is the TTT?
Answer: IV co-trimoxazole
‫ــــــــــــــــــــــــــــــــــــــــــــــــــــــــــــــــــــــــــــــــــــــــــــــــــــــــــــــــــــــــــــــــــــــــــــــــــــــــــــــــــــــــــــــــــــــــــــــــــــــــــــــــــــــــــــ‬
118- Question id: 2052
A 68 yo man presented with severe Rt sided eye pain which had come on over the course of two hrs. He had no
previous history of eye disease.A diagnosis of acute angle closure glaucoma has made.

(a) What are the risk factors for this condition?(Name four)
Answer: Family history of angle closure, age older than 40 to 50 yrs, female, history of symptoms suggesting angle-
closure, hyperopia (farsightedness), pseudoexfoliation (a condition in which abnormal flaky deposits on eye surfaces
can weaken the zonules that support the lens & cause it to shift forward), & race(the highest rates of angle closure are
reported in Asian populations).
(b) Besides pain what are the other symptoms a Pt may complain of?(Name four)
Answer: Decreased vision, halos around lights, headache, severe eye pain,nausea & vomiting.
(c) What signs may be found on exam?(Name four)
Answer: Conjunctival redness, corneal edema or cloudiness,a shallow anterior chamber & a mid-dilated pupil (4 to 6
mm) that reacts poorly to light.
(d) What time of the day is this condition most likely to occur?
Answer: Signs & symptoms of acute glaucoma often occur in the evening, when lower light levels cause mydriasis,
& folds of the peripheral iris block the narrow angle.
(e) What are the management steps in the ED?
Answer: Name two eye drops which may be of benefit?
‫ــــــــــــــــــــــــــــــــــــــــــــــــــــــــــــــــــــــــــــــــــــــــــــــــــــــــــــــــــــــــــــــــــــــــــــــــــــــــــــــــــــــــــــــــــــــــــــــــــــــــــــــــــــــــــــ‬
119- Question id: 2103

Fig 1
(a) What type of M.I. is shown in the ECG (fig 1), describe what is shown.
Answer: Lateral M.I. ST elevation in leads 1 & aVL can't see elevation in 2 & v6 but you don't always get a complete
set. Also note the inferior reciprocal changes 2,3 & aVF.
(b) Which coronary vessel is likely to be occluded?
Answer: Left circumflex. When the picture shows antero-lateral changes i.e. ST elevation in all the precordial &
lateral leads the occlusion is higher up in the left coronary artery before it splits into the LAD & LCx.
(c) You consider thrombolysis for this Pt, what 5 medications have you already given?
Answer: O2, morphine, aspirin, clopidogrel, LMWH.
(d) Name 2 agents that you could use for thrombolysis & describe how they are given.
Answer: Streptokinase give 1.5 mega units in a continuous infusion over 1 hr. Alteplase: 15mg bolus followed by
0.75mg/kg (max 50mg) IVI for 30 mins, then 0.5mg/kg (max 35mg) over 60 mins. Give heparin or s/c LMWH.
Reteplase: 2 IV boluses of 10 units each 30 mins apart (give heparin as above). Tenecteplase: single IV bolus over 10
seconds, Dose according to weight (also give heparin as above)
49
(e) After you give thrombolysis the Pt seems to still be in pain & after 30 minutes there is no resolution of the
ST segments. What will you do?
Answer: Will need to transfer to an interventional centre for rescue PCI
‫ــــــــــــــــــــــــــــــــــــــــــــــــــــــــــــــــــــــــــــــــــــــــــــــــــــــــــــــــــــــــــــــــــــــــــــــــــــــــــــــــــــــــــــــــــــــــــــــــــــــــــــــــــــــــــــ‬
120- Question id: 4537
A 71 yo man presents with central crushing chest pain. An ECG shows ST elevation in leads V1-V4. He receives
thrombolysis. 3 hrs later his ECG shows (see fig 1).

Fig 1
(a) What does the ECG show?
Answer: Second degree heart block- Mobitz type II.
(b) What size does the ST elevation need to be in the chest leads for thrombolysis?
Answer: Answer: >2 mm. In 2 anatomically contiguous leads (>1mm in limb leads, >2mm in V leads)�
(c) What has occurred with the above Pt?
Answer: Pt has had an anterior MI which has led to Mobitz type II which could lead to complete heart block.�
(d) With regard to the changes seen in figure 1 what does this Pt need, please chose the best option? 1.
temporary venous pacing wire 2. Atropine 3. Angiography 4. No TTT 5. temporary transcutaneous pacing
Answer: Answer: 1tempory venous pacing wire - Mobitz type 2 in this setting is very dangerous; the rhythm could
quickly turn into complete heart block.
(e) List the reasons that one would need to instigate urgent pacing after an M.I.
Answer: Complete HB, Asystole, Symptomatic bradycardia or Mobitz type 1 that isn't responding to atropine, New
BBB with 1st degree heart block, Old RBBB with 1st degree AV block & a new fasicular block.
‫ــــــــــــــــــــــــــــــــــــــــــــــــــــــــــــــــــــــــــــــــــــــــــــــــــــــــــــــــــــــــــــــــــــــــــــــــــــــــــــــــــــــــــــــــــــــــــــــــــــــــــــــــــــــــــــ‬
121- Question id: 4541
A 68 yrs old man presents with central chest pain. ECG shows an anterior STEMI.

(a) What is the best TTT?


Answer: Primary coronary angioplasty, lower mortality & lower re-infarction rates.
(b) What is the advantage of tenectaplase & reteplase as thrombolytic agents?
Answer: Answer: ease of administration as they are given as single bolus doses.
(c) What is the risk of major bleeding with thrombolysis?
Answer: 2-3%
(d) List 4 absolute contraindications to thrombolysis:
Answer: Active internal bleeding, suspected aortic dissection, recent head trauma/intracranial tumour, haemorrhagic
stroke at any time, ischaemic stroke within the last yr, previous allergic reaction to fibrinolytic agent, trauma or
surgery within the last 2 weeks at risk of bleeding.
(e) What drug is give along side thrombolytic agents?
Answer: IV heparin
‫ــــــــــــــــــــــــــــــــــــــــــــــــــــــــــــــــــــــــــــــــــــــــــــــــــــــــــــــــــــــــــــــــــــــــــــــــــــــــــــــــــــــــــــــــــــــــــــــــــــــــــــــــــــــــــــ‬
122- Question id: 4547
A 62 yo obese lady attends complaining of difficulty in getting her breath- she has insp. stridor & a grossly swollen tongue

(a) She is on some medication for HTN, angina & asthma. Which of the following medications most likely
caused this presentation?: Nicorandil, asprin, ramipril, simvastatin, monteleukast
Answer: Ramirpil- well described angioedematous reaction can occur yrs after stating an ACEi. Also can occur with
angiotensin 2 receptor blockers.
(b) How would you treat the Pt?
Answer: Ensure that the airway is protected- consider ENT & anaesthetic input if required- nurse in an area where pt
is monitored closely, be alert to any changes in pt condition. O2, consider IM adrenaline, chlorpenamine,
hydrocortisone. (treat as for anaphylaxis) These Pts need 24 hrs in hospital as there have been reported cases of
airway obstruction after early discharge.
(c) The Pt doesn't appear to improve & also complains of some abdominal pain. O/E you notice that she has
prominent cervical lyphadenopathy. What could explain her lack of improvement?
Answer: This could be acquired C1 esterase deficiency- seen in lymphoma. C1 estersae deficiency can be congenital
or acquired It can be treated with synthetic preparations of C1 esterase.
50
(d) When you look through her Bl. tests you note that the GP did a recent fasting glucose which came back as
8.4. What does this mean & which of the following should be instigated?: Rosiglitazone, metformin, insulin,
diet modification, gliclazide, pioglitazone.
Answer: She is diabetic & obese- likely to need drug therapy but start with diet modification & then first option
would be metformin as obese. Can't have glitazones as has IHD.
‫ــــــــــــــــــــــــــــــــــــــــــــــــــــــــــــــــــــــــــــــــــــــــــــــــــــــــــــــــــــــــــــــــــــــــــــــــــــــــــــــــــــــــــــــــــــــــــــــــــــــــــــــــــــــــــــ‬
123- Question id: 2131
A 49 yo man presents to the ED with an acutely painful left knee. The knee is swollen & painful. He felt fine 3 hrs
ago. His only medical history is of mild HTN for which he takes bendroflumethiazide. There is no history of trauma

(a) Give your top 3 DD:


Answer: You need to consider the DD for an acute monoarthritis, you must consider a septic joint until proven
otherwise. (must include 1&2 to score marks) 1. Septic arthritis 2. Acute gout/other crystal arthropathy 3. Ist
presentation of RA, SLE, seroneagtive arthritis (psoriatic/Rieter's) 4. Tramuatic causes: traumatic synovitis,
haemarhtrosis, fracture, ruptured ACL.
(b) Give 4 things can precipitate acute gout:
Answer: Diet, ETOH, diuretics, renal failure (ask about renal stones), trauma, cytotoxic drugs, myeloproliferative
disease, high cell turn over states i.e. lymphoma, psoriasis, Polycythaemia.
(c) What do you do?
Answer: The joint needs to be aspirated & the fluid sent for gram stain & culture but also for polarized light
microscopy to look for crystals. Also need to do Bl. cultures & send samples for WBC/CRP etc.
(d) List 2 contraindications to your suggestion in c)
Answer: Presumably answered aspirate the joint in c) 2 contraindications would be overlying sepsis i.e.
cellulitis/abscess, & bleeding diathesis.
(e) How would you treat septic arthritis?
Answer: Admit to orthopaedics Cultures as above Aspirate the joint to dryness check that there are not also crystals
as can be present in septic arthritis IV antibiotics flucloxacillin & benzypenicilin, in children use cefotaxime to cover
H. influenzae Strict bed rest Analgesics NSAIDS
‫ــــــــــــــــــــــــــــــــــــــــــــــــــــــــــــــــــــــــــــــــــــــــــــــــــــــــــــــــــــــــــــــــــــــــــــــــــــــــــــــــــــــــــــــــــــــــــــــــــــــــــــــــــــــــــــ‬
124- Question id: 2036
A 45 yo man presented with cough, & SOB. O/E he was febrile & had bronchial breathing in the Rt lower zone.

Fig 1
(a) What is the condition shown in the radiograph?
Answer: Pneumonia.
(b) Name 3 common microbes which cause this condition?
Answer: St. pneumoniae, resp. viruses, mycoplasma pneumoniae, chlamydia pneumoniae&haemophilus influenzae.
(c) Name four risk factors?
Answer: Alcoholism, COPD, smoking, structural lung disease aspiration, lung abscess, HIV infection, age, &
exposure to birds droppings.
(d) What is an initial appropriate anti-biotic regime for a Pt not admitted to ICU?
Answer: Combination therapy with ceftriaxone (1 to 2 g IV daily) or cefotaxime (1 to 2 g IV every 8 hrs) plus
azithromycin (500 mg IV or orally daily). Alternatively monotherapy with a respiratory fluoroquinolone given either
IV or orally except as noted (levofloxacin 750 mg daily or moxifloxacin 400 mg daily or gemifloxacin 320 mg daily
[only available in oral formulation]).
51
(e) How long after discharge should the radiographic abnormalities be resolved?
Answer: CXR at 7 to 12 weeks after TTT is recommended for selected Pts who are over age 40 yrs or are smokers to
document resolution of the pneumonia & exclude underlying diseases, such as malignancy.
‫ــــــــــــــــــــــــــــــــــــــــــــــــــــــــــــــــــــــــــــــــــــــــــــــــــــــــــــــــــــــــــــــــــــــــــــــــــــــــــــــــــــــــــــــــــــــــــــــــــــــــــــــــــــــــــــ‬
125- Question 1 of 20
A 10 month old boy was sent to the ED by his GP with a 1st episode of wheezing. A diagnosis of bronchiolitis was made

(a) Give a differential diagnosis of 4 conditions?


Answer: Viral-triggered asthma or wheezing, pneumonia, chronic lung disease, foreign body aspiration, GERD &/or
dysphagia leading to aspiration, congenital heart disease, heart failure, & vascular rings.
(b) What is the most common etiological agent to cause this condition?
Answer: Respiratory syncytial virus (RSV) is the most common cause.
(c) Name 4 risk factors for this condition?
Answer: Prematurity (gestat. age <37 wks), low birth weight, age<6 to 12 wks, chronic pulm. disease (bronchopulmonary
dysplasia, cystic fibrosis, congenital anomaly), hemodynamically significant congenital HD (eg, moderate to severe
pulm. HTN, cyanotic HD or congenital HD that requires medicat. to control heart failure), immunodeficiency,
neurologic disease & congenital or anatomical defects of the airways. Having older siblings, concurrent birth siblings,
passive smoke, household crowding, child care attendance & high altitude.
d)Name 3 factors associated with increase illness severity?
Answer: Toxic or ill appearance, O2 saturation <95% by pulse oximetry while breathing room air, age younger than 3
months, RR ≥70 breaths per minute & atelectasis on chest radiograph.
(e) How would you manage this child?(Name 3)
Answer: Supportive measures to ensure that the child is clinically stable, well hydrated, & well O2ated. Consider a
trial of inhaled bronchodilators.
‫ــــــــــــــــــــــــــــــــــــــــــــــــــــــــــــــــــــــــــــــــــــــــــــــــــــــــــــــــــــــــــــــــــــــــــــــــــــــــــــــــــــــــــــــــــــــــــــــــــــــــــــــــــــــــــــ‬
126- Question 2 of 20
A 6-yr-old boy brought in acutely SOB. The ambulance crew let you know that he is known to have asthma & that he
has been very wheezy over the course of the last 6 hrs. They have given him 1 dose of nebulised salbutamol.

(a) What important features do you want to elicit to differentiate moderate severe asthma from life-threatening
asthma? Please state 3 things that would guide your decision. (3 marks)
Answer: Any 3: Cyanosis, PEFR<33%, Silent chest, Agitation or reduced GCS, Exhaustion
(b) Why is heart rate sometimes not a useful guide of severity?
Answer: Tachycardia produced by salbutmaol
(c) The initial observations show that he has moderate-severe asthma. The sats are 90% in air what is your
initial management please include drug doses.
Answer: Give high flow O2, Nebulised salbutamol 5mg (driven by O2)/as sats<92% if were above could give inhaled
salbutamol via a spacer. Prednisolone 40mg (as over 5 yrs)
(d) The child deteriorates & his sats drop to 86% his RR is now 65 & he appears to be tiring. What do you do
know?
Answer: (5 marks drop marks for incorrect dosing), Continue with back to back neds driven by high flow O2, Give
nebulised ipratropium 500 mcg via neb, Give IV salbutamol loading dose of 15mcg/kg, IV aminophylline 5mg/kg
over 20 mins then loading dose then maintenance of 500mcg/kg/hr (if already on theophylline omit loading dose),
Importantly call anaesthetist & set up kit for tracheal intubation, alert PICU
(e) Discuss the role of Mg sulphate in the management of life threatening asthma in children.
Answer: No marks for mentioning that used in adults.(2 marks 1 for stating that it might be of benefit another for
stating that it is still undergoing trials) Answer: IV Mg does work & there is good evidence for it. Nebulised Mg may
work but there are a No. of ongoing clinical trials. So you can give it but ongoing research is needed for nebulised
route. Despite a suggestion of benefit in the sub-group of Pts with acute severe asthma this TTT isn't advocated at this
time by the current BTS/SIGN national asthma guidelines (2004). It is mentioned in the most recent edition of the
BNF as an unlicensed indication for Pts with acute severe asthma.
‫ــــــــــــــــــــــــــــــــــــــــــــــــــــــــــــــــــــــــــــــــــــــــــــــــــــــــــــــــــــــــــــــــــــــــــــــــــــــــــــــــــــــــــــــــــــــــــــــــــــــــــــــــــــــــــــ‬
127- Question 3 of 20
A 6 yo boy is brought to the ED by his parents as he was drowsy & poorly communicative. O/E his heart rate was 40
& his extremities appeared poorly perfused.

(a) Name 2 basic initial management steps with this child?


Answer: O2, Attach monitor/defibrillator, support ABC's.

52
(b) If despite the above steps the child is still bradycardic with poor perfusion what is the next management
step?
Answer: Perform CPR if depite O2ation & ventilation HR <60/min with poor perfusion.
(c) If the bradycardia is persistent & symptomatic what medication is indicated? (Assume the child does not
have increased vagal tone or 1ry AV block)
Answer: Adrenaline(IV/IO) 0.01mg/kg(1:10000; 0.1mL/kg) or 0.1mg/kg (1:1000: 0.1mL/kg) via ETT. This can be
repeated every 3-5 minutes.
(d) If the bradycardia is persistent & symptomatic & the child has  vagal tone or 1ry AV block what medicat. is
indicated?
Answer: Atropine 0.02mg/kg & may be repeated.
(e) What other TTT modality should be considered?
Answer: Cardiac Pacing.
‫ــــــــــــــــــــــــــــــــــــــــــــــــــــــــــــــــــــــــــــــــــــــــــــــــــــــــــــــــــــــــــــــــــــــــــــــــــــــــــــــــــــــــــــــــــــــــــــــــــــــــــــــــــــــــــــ‬
128- Question 4 of 20
A 2 yo boy presents with a 3 day history of intermittent fever & tummy ache. No diarrhoea. He vomited once
yesterday. O/E his temp. is 37.5C & exam. of his ear, nose throat & chest are normal. His abd. is soft & non-tender.

(a) You suspect a UTI. Give 3 other possible diagnosis that are important to rule out in a boy of this age (3)
Answer: Appendicits Mesenteric adenitis Orchitis Intussusception
(b) The urine dipstick is positve for nitirites & leucocytes. Name 3 of the most likely organisms. (3)
Answer: Escheria Coli Strep B Klebseiella Proteus Enterobacter Staph
(c) Give 4 indications for admission in a child with UTI? (4)
Answer: Dehydrat. / inability to tolerate oral fluids / repeated vomiting Toxic child requiring IV antibiotics Co-morbidities
Parental concerns / inability to cope Age < 3/12 ( some guidelines < 6/12) Pyelonephritis / renal angle tenderness clinically
(d) How should you obtain a urine sample?
Answer: Not with a bag, should be clean catch MSU
(e) What would you treat this child with?
Answer: More than 3 months of age with signs of pyelonephritis Treat with oral antibiotics for 10 days if sufficiently
well5 <1 yo, Cephradine or Co-amoxiclav >1 yo, Cephradine or Trimethoprim If IV antibiotics required Cefuroxime
is the drug of choice. IV antibiotics should be continued until the pyrexia has settled & culture is available from which
an appropriate oral antibiotic can be given (total duration of TTT 10 days) More than 3 months of age with signs of
cystitis Treat with oral antibiotics for 3 days if sufficiently well but review if no improvement after 24-48 hrs <1 yo,
Cephradine or Co-amoxiclav (Augmentin) >1 yo, Cephradine or Trimethoprim
‫ــــــــــــــــــــــــــــــــــــــــــــــــــــــــــــــــــــــــــــــــــــــــــــــــــــــــــــــــــــــــــــــــــــــــــــــــــــــــــــــــــــــــــــــــــــــــــــــــــــــــــــــــــــــــــــ‬
129- Question 5 of 20
A family present to the ED with 4 children who have all developed a widespread rash as seen below. They have high
temp.s & have generalised coryzal symptoms including sore throat, conjunctivitis. They tell you that none of the
children have had their immunisations as they don't agree with it.
(a) What is the diagnosis? (1 mark)
Answer: Measles
(b) What are the 2 life threatening complications that you need to be vigilant for? (2 marks)
Answer: Pneumonia & encephalitis
(c) What actions do you take? (3 marks)
Answer: Advice family of the condit., Inform the HPA as measles is a notifiable illness, Look for 2ry bacterial infect.
(d) The mother is very concerned about her youngest child aged 4 yo & dem&s that she is admitted to hospital.
You think that she is relatively well with normal vital signs. What do you tell her? (1 mark)
Answer: Explain that it is self limiting disease & that if things were not improving in 3 days then she needs to seek
medical attention. Or if the child becomes more unwell i.e. any features of pneumonia or encephalitis.
(e) How long will the children be infective for? (1 mark)
Answer: From onset of symptoms until 5 days after the rash disappears.
‫ــــــــــــــــــــــــــــــــــــــــــــــــــــــــــــــــــــــــــــــــــــــــــــــــــــــــــــــــــــــــــــــــــــــــــــــــــــــــــــــــــــــــــــــــــــــــــــــــــــــــــــــــــــــــــــ‬
130- Question 6 of 20
A 4 yo boy was brought to the ED by a parent with a painful ear.
(a) What is the diagnosis?
Answer: Otitis media.
(b) What are the risk factors for this condition?(Name 4)
Answer: The peak age-specific attack rate occurs between 6 & 18 months of age, the spread of bacterial & viral
pathogens is common in daycare centers, non-breast fed babies, Exposure to tobacco smoke & ambient air pollution 
the risk of OM, children who use a pacifier, children in developing areas, family history, social & economic condit.,

53
sleep position, season ( incidence during the fall & winter months), altered host defenses& underlying disease (eg,
cLt palate, Down syndrome, allergic rhinitis).
(c) What are the common species of bacteria accounting for most of the bacterial isolates from middle ear
fluid?(Name 2)
Answer: St. pneumoniae, Haemophilus influenzae, & Moraxella catarrhalis.
(d) How would you manage this Pt?(2 points)
Answer: Analgesia(paracetamol or ibuprofen),antibiotics(amoxicillin) & organise follow up to ensure resolution.
(e) What are the complications of this condition?(Name 4)
Answer: Mild conductive hearing loss, vestibular, balance & motor dysfunct., tympanic membrane perforat., inflammat.
of the mastoid &/or mastoiditis, petrositis & labyrinthitis. IC complications are rare in developed countries; they
include meningitis, epidural abscess, brain abscess, lateral sinus thrombosis, cavernous sinus thrombosis, subdural
empyema & carotid artery thrombosis.
‫ــــــــــــــــــــــــــــــــــــــــــــــــــــــــــــــــــــــــــــــــــــــــــــــــــــــــــــــــــــــــــــــــــــــــــــــــــــــــــــــــــــــــــــــــــــــــــــــــــــــــــــــــــــــــــــ‬
131- Question 7 of 20
A 2 yo boy presented with inspiratory stridor & a barking cough. O/E he was febrile & mildly tachycardic.

(a) What is the most likely diagnosis?


Answer: Croup.
(b) Give a differential diagnosis?(3 conditions)
Answer: Acute epiglottitis, peritonsillar & retropharyngeal abscesses, foreign body aspirat. or ingest., allergic react.,
acute angioneurotic edema, upper airway injury, congenital anomalies of the upper airway & laryngeal diphtheria
(c) What is the most common etiological agent?
Answer: Parainfluenza virus type 1 is the most common cause of acute laryngotracheitis, especially the fall & winter epidemics.
(d) Name 4 aspects of the exam. which are helpful in assessing the degree of upper airway obstruction & severity of
illness?
Answer: Overall appearance, quality of the voice, degree of resp. distress, TV, lung exam. & assessment of hydration status
(e) How would you manage this child?(Give five steps)
Answer: Administration of humidified air or humidified O2, antipyretics, encouragement of fluid intake, a single
dose of oral dexamethasone (0.6 mg/kg) (if fit for discharge), nebulized epinephrine, pulse oximetry , observation.
‫ــــــــــــــــــــــــــــــــــــــــــــــــــــــــــــــــــــــــــــــــــــــــــــــــــــــــــــــــــــــــــــــــــــــــــــــــــــــــــــــــــــــــــــــــــــــــــــــــــــــــــــــــــــــــــــ‬
132- Question 8 of 20
A 3 month old baby was brought to the ED as his mother had come upon the child in his cot apnoeic & off colour.
This was thought to be an apparent life threatening event (ALTE).

(a) How would you manage this Pt?


Answer: Take Bl. (FBC, UE, glucose, calcium, Mg, phosphate), admit for monitoring.
(b) What are the risk factors for SIDS(Name 4)?
Answer: Passive smoking, males, winter months, sleeping prone, premature babies, twins, apnoeic spells in first week
of life, lower socioeconomic groups, sibling with SIDS, maternal illicit drug abuse.
(c) What advice can you give the above Pt to prevent SIDS?(Give 4)
Answer: Avoid overheating, avoid duvets & excess bedding, sleep supine, consider apnoea alarm, avoid infant
sharing bed with parent.
(d) What is the definition of SIDS?
Answer: Sudden infant death in infancy with no cause identified after autopsy.
(e) What is the aetiology of SIDS?
Answer: Aetiology is unknown.
‫ــــــــــــــــــــــــــــــــــــــــــــــــــــــــــــــــــــــــــــــــــــــــــــــــــــــــــــــــــــــــــــــــــــــــــــــــــــــــــــــــــــــــــــــــــــــــــــــــــــــــــــــــــــــــــــ‬
133- Question 2005
A 6 yr old child is brought in fitting; he has been fitting for 5 min., is attached to monitors & is receiving O2. He has
been unwell for the last 3 days with a runny nose. He hasn't had his immunisat. His temp. is 39.6C & he weighs 20kg.

Fig 2

54
(a) Fill in the names, doses & timings of the drugs in the algorithm (see fig 1 for answers) for treating the fitting
child: There should be 5 boxes in the algorithm that you draw.
Answer: See fig 1
(b) Some lesions were noted in the child s mouth (fig 2): What are the lesions called & what if the diagnosis?
Answer: Kopliks spots & Measles
(c) Name 8 other notifiable diseases:
Answer: Any 8 from Acute encephalitis, Anthrax, Botulism, Bruscellosis, Cholera, Diphtheria, Dysentery, Food
poisoning, HIV/AIDS, Legionella, Leptospirosis, Leprosy, Malaria, Measles, Meningitis, Meningococcal
Septicaemia, Mumps, Opthalima neonatorum, Paratyphoid, Plague, Polio, Rabies, Relapsing fever, Rubella, SARS,
Scarlet fever, Small pox, Syphilis, TB, Tetanus, Typhoid fever, Typhus, Viral haemorrhagic fever, Viral hepatitis,
Whooping cough, Yellow fever.
‫ــــــــــــــــــــــــــــــــــــــــــــــــــــــــــــــــــــــــــــــــــــــــــــــــــــــــــــــــــــــــــــــــــــــــــــــــــــــــــــــــــــــــــــــــــــــــــــــــــــــــــــــــــــــــــــ‬
134- Question 10 of 20
A 4 yo girl with leukaemia is brought in by her mother, they are on holiday in the area & normally would have gone
straight into their local chemotherapy suite as she has not been well & has a temp.

(a) What defines febrile neutropaneia?


Answer: Temp>38.5C on 1 occasion Temp>38 on 2 or more occasions recorded >1 hr apart Neutropaenia: Absolute
neutrophil count < 500/ml (< 0.5 x 109) ANC <1.0 & rapidly falling count after chemotherapy
(b) You assess the child, her observations are as follows: Pulse 160, cap refill 4secs, RR 52, sats 99% in air,
what do you do (include any calculations)?
Answer: Pt is shocked, needs urgent IV fluid bolus. Ages 4 yrs so (4+4) x2= 16kg therefore 20mls pre kg = 230mls of crystalloid.
(c) Which investigations do you send?
Answer: Relevant immediate investigations: Bl. cultures, systemic & waste Urine dipstix & culture Throat swab,
bacterial & viral Biochemistry, UE, LFT, CRP Haematology, DIC screen if septic
(d) Which areas should you pay particular attention to O/E?
Answer: 1. Mouth, teeth, gums, pharynx. 2. ENT, especially examining for tenderness over the sinuses & mastoid
sites. Consider NPA for Pts with coryzal symptoms 3. Respiratory, RR & O2 saturations & requirements must be
recorded & documented. Hypoxaemia & normal auscultation may be associated with Pneumocystis pneumonia
(PCP). 4. CVS, Bl. Pr. must be documented. 5. Upper GIT painful swallowing may be suggestive of herpetic or
candidal oesophagitis. 6. Abd. tenderness Rt lower quadrant pain may suggest typhilitis (neutropaenic caecal
inflammat.), as well as appendicitis discuss with senior member of staff. 7.Perineum symptoms of perianal discomfort
or pain should always be asked about. If there are symptoms, the perineum should be inspected. 8. Skin lesions look
for petechiae & purpura (evidence of thrombocytopaenia or DIC), consider Pseudomonas, herpetic, fungal aetiology
9. Central venous line sites erythema, swelling, tenderness are suggestive of infect. tracking along the line 10.
Procedure sites e.g. Gastrostomy sites, lumbar puncture, posterior superior iliac crests
(e) What do you need to do prior to completing a full history & exam.?
Answer: Treat the shock aggressively, need to cover with powerful antibiotics Discuss with senior paeds oncologist if
available But don't delay giving antibiotics: 1st line antimicrobials: 1. IV Ceftazidime 50mg/kg every 8 hrs (max 2 gm
tds) 2. IV Gentamicin* (see below for exceptions) a. <12 yrs 2.5mg/kg every 8 hrs b. >12 yrs 1.5mg/kg every 8 hrs
(max 120 mg tds) c. Levels must be taken after the 3rd gentamicin dose (pre + post) d. Adjust initial dose if Pt requires
gentamicin but has renal impairment & take earlier levels usually just give normal dose & take levels with 2nd dose
(must be taken). Wait for level before giving dose. 3. Oral fluconazole prophylaxis 3mg/kg once daily continue while
neutropaenic IV Flucloxacillin should be used instead of gentamicin in the following cases: Renal impairment discuss
with senior staff member Renally toxic chemotherapy protocols, i.e. those containing cisplatin or ifosfamide Dose of
flucloxacillin 25mg/kg every 6 hrs (max 1 gram qds) If Pt shows signs of septic shock contact a Senior member of
staff as they may require gentamicin irrespective of renal impairment or TTT with renally toxic chemotherapy
Additional antibiotics 1. Consider adding glycopeptide as 1st line agent if: CVL related infect. suspected Severe
mucositis Previous MRSA isolate IV Teicoplanin 10mg/kg (max 400mg) every 12 hrs for 3 doses, then once daily 2.
If significant perianal inflammation or possible typhilitis (Neutropaenic colitis) add: IV Metronidazole 7.5mg/kg (max
500mg) every 8 hrs Febrile at 48 hrs Discuss possible 2nd line antibiotics with Consultant: If Pt is unwell add: IV
Teicoplanin 10mg/kg (max 400mg) every 12 hrs for 3 doses, then once daily Febrile at 96 hrs Discuss possible 3rd line
antibiotics with Consultant: Consider empirical TTT for possible fungal infection (Consultant decision only): IV
Liposomal amphotericin (Ambisome) Dose 3mg/kg od (remember to prescribe test dose as per cBNF) Discuss change
of antibiotic with Consultant on call: IV Imipenem* a. <12 yrs 15mg/kg qds (max 500mg qds) b. >12 yrs 12.5mg/kg
qds (max1g qds) *Use Meropenem if evidence of renal impairment or history of CNS disorders
‫ــــــــــــــــــــــــــــــــــــــــــــــــــــــــــــــــــــــــــــــــــــــــــــــــــــــــــــــــــــــــــــــــــــــــــــــــــــــــــــــــــــــــــــــــــــــــــــــــــــــــــــــــــــــــــــ‬
135- Question 11 of 20

55
A 6 month old boy is brought in by his Mum, he was fine yesterday but this morning she noticed that he was crying >
normal & that he wasn’t moving his Lt leg as normal. There was no story of trauma. His x-ray is shown below:

(a) Describe what is shown in the x-ray


Answer: Transverse femur fracture of the diaphysis that is angulated but not shortened.
(b) What will you do?
Answer: Need to complete a full history & exam.. Then need to contact senior paediatrician. Social services will need
to be contacted & the child will need to be admitted not only for TTT of the fracture but for full NAI investigation.
(c) What is Munchausen by proxy (factitious ilness?)
Answer: Severe form of child abuse where adult (usually mother) fakes illness in their child, often they will induce
physical symptoms by giving medications i.e. headaches (GTN) & they will dem& investigations.
(d) What else could have caused the appearances seen above?
Answer: Osteogenesis imperfecta,
‫ــــــــــــــــــــــــــــــــــــــــــــــــــــــــــــــــــــــــــــــــــــــــــــــــــــــــــــــــــــــــــــــــــــــــــــــــــــــــــــــــــــــــــــــــــــــــــــــــــــــــــــــــــــــــــــ‬
136- Question 13 of 20
A 9 yo boy was brought to the ED because after a prodrome of fever, malaise, & pharyngitis he had developed a
pruritic rash shown in the picture.
(a) What is the diagnosis?
Answer: Chicken pox.
(b) What is the incubation period & how is this disease transmitted?
Answer: The average incubation period for varicella infection is 14 to 16 days although this interval can range from
10 to 21 days. Transmission occurs in susceptible hosts via contact with aerosolized droplets from nasopharyngeal
secretions of an infected individual or by direct cutaneous contact with vesicle fluid from skin lesions.
(c) What areas do the lesions cover?
Answer: The Pt with varicella typically has lesions in different stages of development on the face, trunk & extremities.
(d) Name 3 complications of this disorder?
Answer: The most frequent complication among healthy children is bacterial skin superinfection. Others include
encephalitis, reye syndrome, hepatitis & pneumonia.
(e) Name 3 general measures to treat this condition?
Answer: Antihistamines, fingernails should be closely cropped & paracetamol. Acyclovir may be used in selected cases.
‫ــــــــــــــــــــــــــــــــــــــــــــــــــــــــــــــــــــــــــــــــــــــــــــــــــــــــــــــــــــــــــــــــــــــــــــــــــــــــــــــــــــــــــــــــــــــــــــــــــــــــــــــــــــــــــــ‬
137- Question 14 of 20
A 13 yo national standard ice skater is sent in by her podiatrist who has recently made her some new inserts for her
skates. She is very distressed as she has an important competition in 2 wks time. She is complaining of pain in the Rt forefoot &
says that its gets progressively worse when she walks & has been unable to skate for the last 2 days. There is no history of trauma

(a) What could be going on & what do you do to investigate?


Answer: Frieberg’s disease. Foot x-ray to start. Might show flattening, widening or fragmentation of the metatarsal
head, or narrowing of the MTPJ.
(b) What would you advise?
Answer: NSAIDS & rest in the first instance. For this girl I would arrange orthopaedic referral. Persistent cases can
be treated with excision of the MT head or osteotomy.
(c) What is Osgood-Schlatter’s disease?
Answer: It is another osteochondritis affecting the tibial tuberosity. It is a traction apophysitis of the tibial attachment
of the patellar tendon normally seen in teenagers boys >girls. The tuberosity is prominent & tender.
(d) How should you treat plantar fasciitis?
Answer: NSAIDS & rest, elevate the foot. Heel pads can help. Severe persistent cases can be treated with local
steroid injections & sometimes surgical division of the plantar fascia.
‫ــــــــــــــــــــــــــــــــــــــــــــــــــــــــــــــــــــــــــــــــــــــــــــــــــــــــــــــــــــــــــــــــــــــــــــــــــــــــــــــــــــــــــــــــــــــــــــــــــــــــــــــــــــــــــــ‬
138- Question 15 of 20
A 10 yo boy was in a cubicle in the ED waiting to be seen & suddenly his mom ran out of the cubicle shouting for
help as her son had become unresponsive. The nursing staff confirm that the boy has no pulse & begin CPR. Other
staff members arrive & attach a monitor which shows asystole.

(a) After resuming CPR what medication should be given?


Answer: Adrenaline IV/IO: 0.01mg/kg(1:10000: 0.1mL/kg). This may be repeated every 3-5 minutes.
(b) After how many cycles of CPR should the rhythm be checked again?
Answer: 5 cycles.
(c) What is the compression to breath ratio for one cycle of CPR?

56
Answer: 15:2
(d) Name eight possible contributing causes to asystole?
Answer: Hypovolaemia, hypoxia, hydrogen ion(acidosis), hypokalaemia/hyperkalaemia, hypoglycaemia, hypothermia,
toxins, tamponade(cardiac), tension pneumothorax, thrombosis(coronary or pulmonary),& trauma.
(e) If after the 1st rhythm check the monitor shows VF what is the energy level(J/kg) the child will be shocked at
initially?
Answer: 2J/kg.
‫ــــــــــــــــــــــــــــــــــــــــــــــــــــــــــــــــــــــــــــــــــــــــــــــــــــــــــــــــــــــــــــــــــــــــــــــــــــــــــــــــــــــــــــــــــــــــــــــــــــــــــــــــــــــــــــ‬
139- Question 16 of 20
A 1 yo boy is brought in by his parents after hitting his face on a wooden bar in the park, his mouth bled profusely
after the event & he appears to have lost his front tooth. By the time you see him the bleeding has settled

(a) What piece of information is crucial in this case?


Answer: Did the parents see the tooth; do they have it with them? If they didn’t there is a chance that it could have
been aspirated & go on to cause a lung abscess.
(b) What would you do?
Answer: Chest x-ray including lateral to look for foreign body.
(c) Another child presents with a laceration to the lip. What important features must you look for during the
exam.?
Answer: Must ensure that the laceration doesn’t cross the vermilion border. If it does it will need suturing as even a
1mm discrepancy will leave a scar.
(d) Unlike the cosmetically important facial lacerations that are almost always closed primarily, certain small
intraoral lacerations may be Lt open without repair. What are the indications for closure?
Answer: Indications for intraoral closure Mucosal laceration that creates a flap that interferes with chewing Mucosal
laceration that is large enough to trap food particles Wounds longer than 2 cm
‫ــــــــــــــــــــــــــــــــــــــــــــــــــــــــــــــــــــــــــــــــــــــــــــــــــــــــــــــــــــــــــــــــــــــــــــــــــــــــــــــــــــــــــــــــــــــــــــــــــــــــــــــــــــــــــــ‬
140- Question 17 of 20
A 10 yo boy was in a cubicle in the ED waiting to be seen & suddenly his mom ran out of the cubicle shouting for
help as her son had become unresponsive. The nursing staff confirm that the boy has no pulse & begin CPR. Other
staff members arrive & attach a monitor which shows VF.

(a) At what energy level(J/kg) should the child be shocked initially?


Answer: 2J/kg.
(b) After the shock is delivered what is the next step?
Answer: Give 5 cycles of CPR & recheck rhythm after 5 cycles.
(c) At what energy level should the second shock be delivered at?
Answer: 4J/kg.
(d) What medication should be given at this stage & at what dose?
Answer: Adrenaline IV/IO: 0.01mg/kg(1:10000: 0.1mL/kg). This may be repeated every 3-5 minutes.
(e) If CPR is ongoing what medication should be considered & at what dose?
Answer: Amiodarone 5mg/kg IV/IO (or lidocaine 1mg/kg IV/IO).
‫ــــــــــــــــــــــــــــــــــــــــــــــــــــــــــــــــــــــــــــــــــــــــــــــــــــــــــــــــــــــــــــــــــــــــــــــــــــــــــــــــــــــــــــــــــــــــــــــــــــــــــــــــــــــــــــ‬
141- Question 18 of 20
The above film is of a 4 yo girl who presents 5 days after a fall onto her Lt arm. Mum is concerned as she has noticed
a swelling over the outer aspect of the arm.

(a) What is the abnormality on the film? How would you describe it?
Answer: Mid-shaft fracture of the ulna, this is a plastic deformity with clear bowing of the ulna.
(b) 1 of your consultants happens to be a round & casually lets you know that the films are inadequate. What
do they mean & what do you need to do now? What abnormality do you not want to miss here?
Answer: You need a true lateral at the elbow joint as you don’t have one. You don’t want to miss a dislocation of the
radial head & hence a Monteggia fracture dislocation. This is a common pitfall if you don’t request the correct films.
(c) What is a greenstick fracture?
Answer: Almost exclusively occurs during infancy & childhood. The bending of a bone with incomplete fracture,
involving the convex side only. Green stick fractures are characterized by a break in the bone which partially extends
across & then along the length of the bone forming the characteristic fracture pattern for which it is named.
‫ــــــــــــــــــــــــــــــــــــــــــــــــــــــــــــــــــــــــــــــــــــــــــــــــــــــــــــــــــــــــــــــــــــــــــــــــــــــــــــــــــــــــــــــــــــــــــــــــــــــــــــــــــــــــــــ‬
142- Question 19 of 20
A neonate is brought to the ED by his parents as he had a fever, cough & wasn�t feeding well. The treating
emergency clinician suspected pneumonia.
57
(a) What are the common pathogens involved in neonatal pneumonia?(Name 2)
Answer: E.Coli, beta-haemolytic strep, chlamydia trachomatis, listeria monocytogenes, CMV.
(b) How would you investigate this Pt?(Give 4)
Answer: Throat swabs, FBC, cultures, viral titres, mycoplasma antibodies, SpO2, urine cultre, CXR.
(c) How would you treat this Pt?
Answer: O2, IV fluids, specialist referral, benzylpenicillin & gentamicin alternatively cefuroxime or co-amoxyclav.
(d) What are the risk factors for neonatal pneumonia?(Give 4)
Answer: Prolonged rupture of the fetal membranes (>18 hrs), maternal amnionitis, premature delivery, fetal
tachycardia, maternal intrapartum fever, anomalies of the airway (eg, choanal atresia, tracheoesophageal fistula, &
cystic adenomatoid malformations), severe underlying disease, prolonged hospitalization, neurologic impairment
resulting in aspiration of gastrointestinal contents.
(e) What are the factors which determine outcome? (Give 4)
Answer: Increased mortality is associated with preterm birth, pre-existing chronic lung disease, or immune
deficiencies. Severity of the disease, the gestational age of the Pt, underlying medical conditions, & the infecting
organism affect the prognosis of the disease.
‫ــــــــــــــــــــــــــــــــــــــــــــــــــــــــــــــــــــــــــــــــــــــــــــــــــــــــــــــــــــــــــــــــــــــــــــــــــــــــــــــــــــــــــــــــــــــــــــــــــــــــــــــــــــــــــــ‬
143- Question 20 of 20
A 10 yo girl presents with an earring embedded in the earlobe with an associated local infect. You decide to do a nerve block.

(a) draw a diagram to indicate the site of injection & the nerve involved
Answer: Great auricular nerve block Subcutaneous injection infiltrate 1cm below the ear lobe from the posterior
border of the SCM to the angle of the mandible.
(b) Calculate the dose of Lidocaine 1% for this girl, show calculation
Answer: 10 yrs = 28kg (age+4) x2 Max dose = 3mg/kg, ie 84 mg Max dose of 1% lidocaine is 8.4mls
(c) What systems & symptoms does LA affect in overdose
Answer: Perioral & lingular paraesthesia & numbness CNS: Lightheaded, dizzy, LOC, Seizure CVS: Arrhythmia, Cardiac arrest
‫ــــــــــــــــــــــــــــــــــــــــــــــــــــــــــــــــــــــــــــــــــــــــــــــــــــــــــــــــــــــــــــــــــــــــــــــــــــــــــــــــــــــــــــــــــــــــــــــــــــــــــــــــــــــــــــ‬
144- Question 1 of 30
A 3 yo child is sent in by their GP as having a non-blanching rash & the GP wonders if it might be meningococcal
disease. He gives the child IM penicillin & sends them straight in to see you.

(a) Please give 4 differential diagnoses for a true non-blanching rash. Not including ITPP, HUS, HSP or acute
leukaemias which are all distinct & usually not difficult to diagnose.
Answer: Meningococcal disease (MCD), Sepsis with other bacteria, Viral illness, Trauma/NAI
(b) Describe how a child with ITP normally presents.
Answer: Usually well children with multiple bruises & ppetechiae noticed over severall days. Often seen after a viral
illness. Can get conjunctival haemorrhage, nose bleeds & bleeding gums.
(c) Does the fact that the child has been treated with penicillin affect the management principles that you will
follow?
Answer: No, you would treat as you would another child but these children do require a senior paediatric review prior
to discharge.
(d) Define a purpuric rash.
Answer: Lesions >2mm in diameter that are non-blanching. Spontaneous bleeding into the skin usually appears as a
rash known as purpura
(e) If the lesions were purpuric & the child had a mild temp. what would be your initial management?
Answer: To give IV broad spectrum antibiotics; a third generation cephalosporin. Ceftriaxone 80mg/kg (od) or
cefotaxime 50mg/kg (tds)
(f) The lesions are confined to the area above the nipple line & you think that the child is otherwise quite well.
Explain the thoutht process that you will use to decide whether or not to admit him to hospital.
Answer: If the lesions are not purpuric i.e. they are less than 2mm & the child is well, i.e. not irritable, lethargic &
haemodynamically stable then you can look for the distribution of the rash if it is confined to the SVC distribution
then the child can be discharged as long as there is a focus of infection & there are no concerns over NAI.
‫ــــــــــــــــــــــــــــــــــــــــــــــــــــــــــــــــــــــــــــــــــــــــــــــــــــــــــــــــــــــــــــــــــــــــــــــــــــــــــــــــــــــــــــــــــــــــــــــــــــــــــــــــــــــــــــ‬
145- Question 3 of 30
A red phone call tells you that a 6 yo girl is on the way who is shocked. She is a type 1 DM & has been well over the
last few days; today she had some vomiting & Abd. pain. You assess her & begin to treat her gaining IV access &
instigating a fluid bolus. Her BM is 1.4. Mother tells you that she has been getting recurrent low BM readings over the
last few weeks that they haven't been able to explain.

58
(a) What will you do?
Answer: Give 5ml/kg 10% dextrose bolus followed by maintenance fluidsi. If unable to gain IV access & not drowsy
or unresponsive give sugar orally (eg.100ml coke, lemonade, orange juice, 2-3 dextrose tablets, milk feed, Glucogel)
If drowsy or unresponsive give IM Glucagon 0.5 mg < 25 kg, 1 mg > 25 kg
(b) You sent off some routine Bl.s initially & they come back, WBC normal, K+ 6.1, Na+ 128. You are
concerned as the child has not responded to your initial fluid bolus; you give another bolus & seek advice from
the consultant paediatrician. What is the possible diagnosis?
Answer: Undiagnosed 1ry adrenal insufficiency with acute adrenal crisis. Other autoimmune diseases may be a clue
to the presence of Addison�s disease. E.g. recurrent hypoglycaemia in a child with type 1 diabetes mellitus
(c) What is the management? (include any drug doses)
Answer: IV hydrocortisone 25mg (<10 kg), 50 mg (10-25 kg), 100mg (> 25kg) & continue 6 hrly until well with no
diarrhoea/vomiting & stable Bl. sugar & electrolytes. If unable to gain IV access give IM hydrocortisone
(d) Explain the pathophysiology of diabetes insipidus.
Answer: Diabetes insipidus is a condition characterized by excretion of large amounts of severely diluted urine,
which can't be ↓ when fluid intake is ↓. It denotes inability of the kidney to concentrate urine. DI is caused by a
deficiency of ADH, also known as vasopressin, due to the destruction of the back or "posterior" part of the pituitary
gland where vasopressin is normally released from, or by an insensitivity of the kidneys to that hormone. It can also
be induced iatrogenically by various drugs.
‫ــــــــــــــــــــــــــــــــــــــــــــــــــــــــــــــــــــــــــــــــــــــــــــــــــــــــــــــــــــــــــــــــــــــــــــــــــــــــــــــــــــــــــــــــــــــــــــــــــــــــــــــــــــــــــــ‬
146- Question 5 of 30
A 4 yo boy was playing with his brother magnetic set. He swallowed 4 magnetic balls yesterday but Mum has only
just found out. He is completely well in himself but she wanted to get him checked over.

(a) What do you tell Mother?


Answer: That you need to confirm that they are in his gut by taking an Abd. x-ray.
(b) What signs do you ask Mum to look out for?
Answer: Look out for any Abd. pain or vomiting, ensure that he is having normal bowel movements. If he appears at
all unwell then needs to be seen immediately.
(c) What do you do with this child? Do you admit them or send them home?
Answer: Needs surgical referral, may not need to come in but some authors propose that if multiple magnets are
found on imaging then they should be removed. Probably should come in for close observation.
(d) What complications could potentially ensue?
Answer: There are case reports of volvulus, perforation & obstruction when 1 or more magnets has been ingested. Or
when 1 magnet & a further metallic object has been ingested.
‫ــــــــــــــــــــــــــــــــــــــــــــــــــــــــــــــــــــــــــــــــــــــــــــــــــــــــــــــــــــــــــــــــــــــــــــــــــــــــــــــــــــــــــــــــــــــــــــــــــــــــــــــــــــــــــــ‬
147- Question 8 of 30
A 9 month baby girl is brought in with difficulty breathing & short history of being generally unwell & poor feeding.
She has a temp. of 38.2◦C & has saturations of 93% in room air.

(a) What is the normal RR in a child of 9 months? (1 mark)


Answer: Answer: 30-40 breaths a minute.
(b) When is the bronchiolitis season? (1 mark)
Answer: Answer: Late autumn to early spring
(c) Name 3 viruses that cause bronchiolitis stating which one is most common. (3 marks)
Answer: Respiratory syncitial virus (RSV) is the most common (70-80%). Others include, adenovirus, influenza,
parainfluenza, metapneumo-virus.
(d) Some children are at increased risk of severe illness & would almost always be admitted. List 3
circumstances where this would be the case. (3 marks)
Answer: Any of the following: Infants <6 wks, Ex-Preterm Infants, CLD, Congenital HD, Immunodeficiency
(e) On the history & information above what would you do with this child, list 3 criteria on which you would
base your decision please include some specific objective measurements? (3 marks)
Answer: It depends but answer should include a senior paeds review. Any one of these features may be sufficient to
prompt admission & not all are required. Marked recession/resp. distress or grunting resp. O2 saturations <92% * RR
>70/m Taking <50% usual feeds or concerns regarding hydration status History of apnoea Appears unwell or lethargic
(f) Describe the role of drug TTTs in bronchiolitis (2 marks)
Answer: Nebulised Ribavirin: minimal evidence shows it to be effective in reducing length of hospital stay &
ventilatory support in severely affected Pts. Consultant approval required before commencing ribavarin therapy. 1
mark awarded for stating that the mainstay of TTT is supportive rather than medication based.

59
(g) What is the most important thing that needs to be considered when admitting & nursing children with bronchiolitis?
(1 mark)
Answer: Limiting cross infection by any sensible means described in the answer.
‫ــــــــــــــــــــــــــــــــــــــــــــــــــــــــــــــــــــــــــــــــــــــــــــــــــــــــــــــــــــــــــــــــــــــــــــــــــــــــــــــــــــــــــــــــــــــــــــــــــــــــــــــــــــــــــــ‬
148- Question 9 of 30
A 5 yo boy has eaten some mushrooms & is brought in vomiting he is haemodynamically stable & only ate them 30
minutes ago he found them in his 17 yo brother's room. Parents are concerned that they may be magic mushrooms

(a) What is the dose of activated charcoal in children?


Answer: 1mg/kg
(b) What 2 conditions are essential to be met prior to considering its administration?
Answer: That it is within one hr of ingestion (unless slow release preparations are ingested could consider giving
later) That there is adequate airway protection either via an ET tube or a fully conscious Pt. (Aspiration can be fatal)
(c) Is there any place for giving it in this scenario?
Answer: Firstly the child is vomiting so is unlikely to be able to take the charcoal.Activated charcoal can be used if
death cap mushroom (Amanita phalloides) are ingested but there is no role for it in this scenario.
(d) Name to TTTs for ingestion of antifreeze.
Answer: Ethanol or alcohol dehydrogenase inhibitor (fomepizole)
(e) What is the dose of naloxone in children?
Answer: Neonate 5-10 micrograms/kg, repeated every 2-3 minutes if required Child 1 month-12 yrs 5-10
micrograms/kg; if response inadequate, give a subsequent dose of 100 micrograms/kg (max. 2 mg) Child 12-18 yrs
1.5-3 micrograms/kg; if response inadequate, give subsequent doses of 100 micrograms every 2 minutes
‫ــــــــــــــــــــــــــــــــــــــــــــــــــــــــــــــــــــــــــــــــــــــــــــــــــــــــــــــــــــــــــــــــــــــــــــــــــــــــــــــــــــــــــــــــــــــــــــــــــــــــــــــــــــــــــــ‬
149- Question 16 of 30
A 4 wk old baby boy is brought in by his parents who say that he has been vomiting after every feed they say that the
vomiting is projectile in nature. You wonder about pyloric stenosis. You establish venous access & give a fluid bolus of 10ml/kg

(a) What age group are affected by pyloric stenosis & what exactly is it?
Answer: Pyloric stenosis is hypertrophy of the muscles surrounding the pylorus of the stomach. It is uncertain
whether there is a real congenital narrowing or whether there is a functional hypertrophy of the muscle that develops
in the 1st few wks of life. Age affected: Usually presents between 3 & 6 wks of age Late presentat. up to 6 months can occur1
(b) What is helpful when making the diagnosis?
Answer: Palpable 'tumour' in Rt upper quadrant best felt from Lt during test feed Visible peristalsis often seen
Diagnosis can be confirmed by Abd. ultrasound Needs assessment of length, diameter & thickness of the pylorus A
wall thickness of great than 3mm supports the diagnosis Biochemically a hypochloraemic alkalosis exists
(c) How is it treated?
Answer: Correct dehydrate. over a 24 - 72 hr period, NGT is often required Ramstedt's pyloromyotomy 1st described
in 1911 Transverse Rt upper quadrant or circumumbilical incision Longitudinal incision in pylorus down to mucosa
Incision extend from duodenum onto the gastric antrum Need to try & avoid mucosal perforation pyloromyotomy
(d) Another child comes in with similar symptoms but doesn�t appear too dehydrated & the vomiting isn't
really projectile. What do you need to do to try to establish the diagnosis?
Answer: Do a test feed to assess the nature of the vomiting Also establish the total amount that they are feeding,
should be about 150mls per kg if they are massively overfeeding then this may represent the main problem.
‫ــــــــــــــــــــــــــــــــــــــــــــــــــــــــــــــــــــــــــــــــــــــــــــــــــــــــــــــــــــــــــــــــــــــــــــــــــــــــــــــــــــــــــــــــــــــــــــــــــــــــــــــــــــــــــــ‬
150- Question 17 of 30
A 6 yo boy is brought to the ED after a fall on his Rt wrist. X Ray reveals a colles fracture. The decision is made to
manipulate the boy's wrist using ketamine for procedural sedation

(a) What are the advantages of using ketamine for procedural sedation? (Give 2)
Answer: Ketamine provides sedation, analgesia, amnesia, & immobilization, while usually preserving upper airway
muscle tone, airway protective reflexes, & spontaneous breathing.
(b) What is the dose range when using ketamine for procedural sedation intravenously?
Answer: 0.5mg to 2 mg/kg.
(c) What is the duration of action of ketamine?
Answer: 10 to 20 minutes.(though typical duration of effective dissociation is 5-10 min)
(d) Name 3 side effects?
Answer: Side effects of ketamine include  salivat., vomiting, unpleasant hallucinations, laryngospasm rarely occurs.
(e) What are the disadvantages of giving ketamine via the IM route?(Give 2)
Answer: Longer recovery times & more vomiting.
‫ــــــــــــــــــــــــــــــــــــــــــــــــــــــــــــــــــــــــــــــــــــــــــــــــــــــــــــــــــــــــــــــــــــــــــــــــــــــــــــــــــــــــــــــــــــــــــــــــــــــــــــــــــــــــــــ‬

60
151- Question 2 of 20
A 3 yo child attends the department with worried parents who tell you that she refuses to use her Lt arm. There is no
history of trauma. O/E the arm appears to move normally with out discomfort but the child cries when you palpate the
arm. There is no swelling or deformity.

(a) What is the next appropriate course of action?


Answer: With no accurate history a fracture or other soft tissue injury can't be confidently ruled out. Therefore an x-
ray is the next step. If reasonable doubt surrounds the diagnosis, performing radiography of the extremity before
attempting reduct. is prudent to avoid manipulat. of an extremity with an elbow fracture this is a medico-legal pitfall.
(b) Explain how you would manipulate a pulled elbow
Answer: TTT consists of manipulating the child's arm so that the annular ligament & radial head return to their
normal anatomic positions. a. This is accomplished by immobilizing the elbow & palpating the region of the radial
head with 1 hand. b. The other hand applies axial compression at the wrist while supinating the forearm & flexing the
elbow. c. As the arm is manipulated, a click or snap can be felt at the radial head. A click noted by the examiner has a
+ve predictive value of more than 90% in 2 published case series & a -ve predictive value of 76% in one case series.
Some authors believe the likelihood of successful reduction is  if pr. is applied over the radial head. Nursemaid's
elbow can be reduced by extension of the forearm instead of flexion; however, extension was less effective in
achieving reduction in 1 case series. A recent abstract reports that pronation may be more effective than supination.1
(c) What is the age range for a pulled elbow, which arm is more commonly affected & is there any sex
preponderance?
Answer: Normally 1-4 yrs but 4 months to 15yrs have been reported, Lt arm more common as more care givers are
Rt h&ed. Girls more common than boys.
(d) How would you manage a failed attempt at manipulation of a pulled elbow?
Answer: Attempt again up to 3 times but must x-ray if still unsuccessful (if not already x-rayed) If radiographic
findings demonstrate no fracture, repeat attempts at reduction are unsuccessful, & the child doesn't regain normal
function after 30-40 min., the safest management is to support the arm in a sling (or splint & sling) & have the child
re-evaluated by a physician (usually a 1ry care physician, not an orthopedist) in 1-2 days. 1 case series reported 7 Pts
meeting these criteria had either spontaneous return of funct. or successful reduct. at follow-up evaluat. by day 4.1
‫ــــــــــــــــــــــــــــــــــــــــــــــــــــــــــــــــــــــــــــــــــــــــــــــــــــــــــــــــــــــــــــــــــــــــــــــــــــــــــــــــــــــــــــــــــــــــــــــــــــــــــــــــــــــــــــ‬
152- Question 3 of 20
A 4 yo boy was brought to the ED by his mother as he had a 2 week history of cough.

(a) What are the clinical features of whooping cough?(Give 4)


Answer: Cough may persist for several weeks, an inspiratory noise (�whoop�) after a bout of coughing, coughing
may culminate in vomiting, cough typically worse a night, conjunctival haemorrhage 2ry to severe coughing.
(b) What is the pathogen involved?
Answer: Bordetella pertusis.
(c) How would you investigate this Pt?(Give 4)
Answer: Bl. for viral titres, mycoplasma antibodies, FBC, CXR.
(d) How would you treat this Pt?(Name 3)
Answer: Erythromycin, avoidance of other children, arrange GP follow up, prophylaxis for unimmunised infant
siblings, notifiable disease.
(e) What are the complications of this condition?(Name 2)
Answer: Prolonged illness, neurological damage, bronchiectasis.
‫ــــــــــــــــــــــــــــــــــــــــــــــــــــــــــــــــــــــــــــــــــــــــــــــــــــــــــــــــــــــــــــــــــــــــــــــــــــــــــــــــــــــــــــــــــــــــــــــــــــــــــــــــــــــــــــ‬
153- Question 10 of 20
A 4 yo boy is brought to the ED by his parents. They complained that he had been distressed that morning but could
not explain why. He had no history of trauma or fever but had become abruptly distressed. O/E his heart rate was
200/min. There was also evidence of poor perfusion.

(a) Before ordering an ECG give 3 basic management steps?


Answer: ABC's, O2, attach monitor/defibrillator.
(b) His ECG revealed narrow QRS complexs & was thought to be a probable SVT. What is the next step?
Answer: Vagal maneuvers.
(c) If this fails & but IV access is readily obtained what medication should be given & at what dose?
Answer: Adenosine. 0.1mg/kg by rapid bolus.
(d) What other TTT modality should be considered?
Answer: Synchronised cardioversion. 0.5 to 1J/kg. If this isn't effective increase to 2 J/kg.
(e) Name eight possible contributing causes to paediatric tachycardias with poor perfusion(5H's & 5T's)?
61
Answer: Hypovolaemia, hypoxia, hydrogen ion(acidosis), hypokalaemia/hyperkalaemia, hypoglycaemia,
hypothermia, toxins, tamponade(cardiac), tension pneumothorax, thrombosis(coronary or pulmonary),& trauma.
‫ــــــــــــــــــــــــــــــــــــــــــــــــــــــــــــــــــــــــــــــــــــــــــــــــــــــــــــــــــــــــــــــــــــــــــــــــــــــــــــــــــــــــــــــــــــــــــــــــــــــــــــــــــــــــــــ‬
154- Question id: 2093
A 13 day baby boy is brought in, he Lt the neonatal unit 3 days ago. Mum says that he has a bowel disorder that needs
an operation, they are currently doing washouts of his rectum as he cannot pass stool himself. Mum is a little
concerned as he has vomited his feeds this afternoon & wonders if his abd. is a little distended.

Fig 1
(a) You ask if the baby had a rectal biopsy whilst on the NICU, they says yes & you also find out that he is
otherwise completely healthy & was not ventilated whilst on the NICU. What is the diagnosis?
Answer: Hirschprung's disease
(b) You examine the child carefully. What important signs are you looking for? Do you order any tests?
Answer: General ABCDE approach need to assess if the baby is septic, need to think about intestinal obstruction.
Looking for distended bowel loops. Check BM, temp. full set of observations cap refill etc. Order a plain Abd. film
looking for obstruction & signs of necrotising enterocolitis NEC. Full set of Bl.s including cultures/CRP/WBC
(c) The AXR (fig 1)was taken in the ED what does it show & what will you do?
Answer: It shows Pneumatosis intestinalis, which is pathognomonic for NEC. An urgent surgical consultation is
needed & further imaging ultrasound if a skilled provider is available or Lt lateral decubitus imaging to rule out a
pneumoperitoneum. If this is the case then surgery will be indicated. Baby needs to be NBM, have an NGT inserted &
have IV antibiotic started. Needs to be managed on PICU. If free gas was shown then needs an urgent laparotomy.
(d) What is the approximate mortality of this condition?
Answer: 50% mortality but higher in severe NEC
‫ــــــــــــــــــــــــــــــــــــــــــــــــــــــــــــــــــــــــــــــــــــــــــــــــــــــــــــــــــــــــــــــــــــــــــــــــــــــــــــــــــــــــــــــــــــــــــــــــــــــــــــــــــــــــــــ‬
155- Question 8 of 20
8 yr old presents lethargic & dehydrated. Weighs 22 kg. Looks unwell. RR 40, Sats 98% on O2. Started on re-hydrat. fluids

(a) You decide to give a bolus what would you give?


Answer: 20mls/kg or 10mls/kg = appropriate so either 440mls or 220mls depending on the level of dehydration they
thought was appropriate.
(b) Give 4 features of 5% dehydration
Answer: Clinically useful signs for detecting 5% dehydration were capillary refill time, abnormal skin turgor &
abnormal resp. pattern (table ). Dry MM, sunken eyes, & poor overall appearance are moderately useful
(c) Calculate the maintenance fluid required in the first 8 hrs
Answer: first 10 kg = 100ml/kg/24 hrs second 10 kg = 50ml/kg/24 hrs subsequent kgs = 20ml/kg/24 hrs i.e for 22kg
child = 1540 mls/24 hrs = 513 mls / 8 hrs
(d) Give 3 investigations to establish the underlying diagnosis in the ED with the diagnosis considered for each
test
Answer: Stool spec for GE. BM for new presentation of DM Urinalysis for Urinary sepsis CXR for pneumonia
‫ــــــــــــــــــــــــــــــــــــــــــــــــــــــــــــــــــــــــــــــــــــــــــــــــــــــــــــــــــــــــــــــــــــــــــــــــــــــــــــــــــــــــــــــــــــــــــــــــــــــــــــــــــــــــــــ‬
156- Question id: 2025
A 5 yo child presents with a rash & some Abd. pain. He also has pains in his knees & ankles. Mother tells you that he
had a cold 2 weeks ago & hasn't been Rt since the rash appeared within the last 24 hrs. The rash looks similar to the
one seen in this photo (see fig 1): It is present on the extensor surface of the lower limbs as well as the buttocks.

62
Fig 1
(a) What is the diagnosis? (1 mark)
Answer: Henoch-Schonlein purpura
(b) Which are the 3 main areas that are affected by this condition? (3 marks)
Answer: GI, renal & skin
(c) What investigations must be performed? (3 marks)
Answer: Urinalysis, BP & FBC/UE
(d) The child is admitted under the paediatricians but later in the day develops some Bl.y diarrhoea. What
investigation will you do & what condition is important to rule out? (2 marks)
Answer: Intussussception, Abd. ultrasound scan
(e) What information will you give the parents with regard to prognosis of the condition?
Answer: HSP is an acute self-limited illness & usually resolves without TTT, but may rarely lead to complications.
Initial attacks of Henoch-Schonlein purpura can last several months. One third of Pts have one or more recurrences.
Children younger than 3 yrs have a shorter, milder course & fewer recurrences. The long-term prognosis of Henoch-
Sch�nlein purpura is directly dependent on the severity of renal involvement.1
‫ــــــــــــــــــــــــــــــــــــــــــــــــــــــــــــــــــــــــــــــــــــــــــــــــــــــــــــــــــــــــــــــــــــــــــــــــــــــــــــــــــــــــــــــــــــــــــــــــــــــــــــــــــــــــــــ‬
157- Question id: 2016
A 5 yo girl is brought in by her parents who say that she isn't Rt but they cannot identify exactly why. When you ask
the girl if she is ok she says that she feels funny

Fig 1 Fig 2
(a) Her initial observation show that she is tachycardic at 260 b.p.m, what will you do? (2 marks)
Answer: ABCD, apply O2 Attach 3 lead monitoring & get a 12 lead ECG Measure BP & cap refill time essentially
assess if hameodynamically stable Obtain IV access in a large proximal vein.
(b) Her ECG is shown (see fig 1): What does it show (1 mark)
Answer: Narrow complex tachycardia. (SVT)
(c) Her BP is 90/50, but her heart rate is still 260. What will you do? (2 marks)
Answer: As is haemodynamically stable can try Vagal techniques: Try valsalva but in 5 yr old better to elicit diving
reflex, Facial cooling with ice for 15 seconds Immersion wrap the child in a towel & immerse the whole head in a
bucket of ice water for 5 seconds (no need to obstruct mouth or nose).
(d) Name a drug that could be used for this child & give the correct dose based on her age. (2 marks)
Answer: Adenosine dose (5+4=9) x2 = 18kg (estimated weight) therefore giving 0.05mg/kg= 0.9mg or 900mcg.
(e) Are there any drug interactions that you need to know about with your chosen drug? (1 mark)

63
Answer: Yes; adenosine's action is prolonged by a factor of 4 by dipyridamole!
(f) The drug that you chose failed to work what will you do next?
Answer: reassess check that still haemodynamically stable then give further adenosine at doses of 0.1mg/kg then 0.2mg/kg g)
(g) The child fails to respond & seems to be drowsy now you repeat the BP which is now not reading what will
you do? (3 marks) Must have dose for 1st shock to gain any marks
Answer: Get someone to urgently call the paeds on call anaesthetist. Draw up some drugs that they may need. Get the
defibrillator attached in sync mode & dial up 0.5joules/kg in this case 10 joules. Give synchronised DC shock.
‫ــــــــــــــــــــــــــــــــــــــــــــــــــــــــــــــــــــــــــــــــــــــــــــــــــــــــــــــــــــــــــــــــــــــــــــــــــــــــــــــــــــــــــــــــــــــــــــــــــــــــــــــــــــــــــــ‬
158- Question 4 of 50
A 34 yo man is involved in motor cross accident- he was partially impaled on a wooden stake at the side of the course
which penetrated the Rt side of his back. He is flown in. When he arrives his observations are as follows. GCS 14, RR
35 sats 88%, BP 145/70 pulse 110. He is screaming in pain intermittently. You can see a large open wound with a
wooden stake sticking through it on the Rt side of the back between T5-L3.

(a) What are you most concerned about? & what would you do about it?
Answer: A pneumothorax! The low sats & high RR rate with tachycardia along with the site of the injury must raise
the possibility of a haemopneumothorax. Examine the chest- if not tensioning get a CXR. Insert a chest drain.
(b) With fluid resuscitation & good analgesia the BP remains 145/68 & the tachycardia comes down to 85.
What analgesia is best in this situation? please give doses.
Answer: Why not use a fast acting opioid like fentanyl- 50-100 mcg in increments- start with 50 mcg then titrate the
rest Morphine takes too long to work in this situation.
(c) What would you do now?
Answer: FAST scan for free fluid would be good idea: if present then should go for a laparotomy but as is
haemodynamically stable CT chest/abd./pelvis would be the investigation of choice. May need MRI of the spine later
but that can wait.
(d) You perform a quick secondary survey: on neurological assessment you discover that the Rt leg is
hyperreflexic & has decreased power but the Lt leg seems normal. Light touch is normal both sides but the Lt
leg there is no sensation to painful stimulus. What could explain these findings?
Answer: Brown- Sequard syndrome
‫ــــــــــــــــــــــــــــــــــــــــــــــــــــــــــــــــــــــــــــــــــــــــــــــــــــــــــــــــــــــــــــــــــــــــــــــــــــــــــــــــــــــــــــــــــــــــــــــــــــــــــــــــــــــــــــ‬
159- Question 7 of 50
A 30 yo man presented to the ED with a lacerat. to his Rt middle finger over the middle phalanx. There was no associated
sensory symptoms or tendon damage. The treating clinician decided to repair the lacerat. using a digital nerve block.

(a) What anaesthetic agents are suitable for this procedure? What additional agent should be avoided?
Answer: Lignocaine, bupivacaine. Avoid adrenaline.
(b) What volume of fluid should be used on each side of the finger?
Answer: 1-2ml on each side of the finger.
(c) What alterat. should be made to the procedure if the lacerat. was over the proximal portion of the middle
phalanx?
Answer: An additional injection of LA should be given across the dorsum of the base of the proximal phalanx.
(d) How long does it take anaesthesia to develop?
Answer: About 5 minutes.
(e) How does the skin feel if the block is working?
Answer: Warm & dry as the autonomic nerves are blocked also.
‫ــــــــــــــــــــــــــــــــــــــــــــــــــــــــــــــــــــــــــــــــــــــــــــــــــــــــــــــــــــــــــــــــــــــــــــــــــــــــــــــــــــــــــــــــــــــــــــــــــــــــــــــــــــــــــــ‬
160- Question 15 of 50
A 42 yo man presented with sudden onset pain in his Lt ankle during a game of squash. The Pt had heard a snap &
reported that it had felt like a baseball bat had hit the back of his ankle. The treating clinician thought that an achilles
tendon rupture was likely

(a) Name 3 risk factors for this condition?


Answer: This typically occurs in men over the age of 30 who sporadically engage in sports & do not do a regular leg
conditioning program, fluoroquinolone antibiotic use, corticosteroid use & genetic predisposition.
(b) What features may be present O/E if the Pt has an achilles tendon rupture?(Name 3)
Answer: The Pt may be unable to st& up on the toes, thompson's test may be positive, & there may be a palpable gap
in the area of the achilles tendon.
(c) How would you investigate this Pt?(Give one investigation)
Answer: Ultrasound.

64
(d) How would you manage this Pt?
Answer: Orthopedic consultation for immobilization or repair is necessary for Pts with tendon rupture.
(e) What is the risk of non-operative TTT?(Give one)
Answer: Nonoperative TTT appears to be associated with a higher risk of rerupture.
‫ــــــــــــــــــــــــــــــــــــــــــــــــــــــــــــــــــــــــــــــــــــــــــــــــــــــــــــــــــــــــــــــــــــــــــــــــــــــــــــــــــــــــــــــــــــــــــــــــــــــــــــــــــــــــــــ‬
161- Question 24 of 50
A 15 yo boy comes in to see you with a swollen Rt knee. He was playing football yesterday. He went in for a tackle,
his studs were planted in the ground & he inwardly rotated on the knee joint causing severe pain. He heard a popping
sound as it happened

(a) He has a large swelling & can’t flex the knee >10 degrees, he doesn’t respond to pain killers. What do you need
to do?
Answer: Need to examine the knee fully. Rule out a bony injury. Need to x-ray the joint to look for haemarthrosis or
lipohaemarthrosis. If the effusion is so large that it is causing severe pain that is unresponsive to analgesics then 1
could consider aspirat.
(b) What are the indications for aspiration of a knee after trauma?
Answer: As above; if the pain cannot be relieved then can aspirate a large effusion. If you are concerned about
compartment syndrome.
(c) What would differentiate an effusion from a haemarthrosis?
Answer: Predominantly the history, if it occurs soon after injury more likely to be haemarthrosis if later more likely
to be an effusion.
(d) How would you manage this Pt?
Answer: As above consider aspiration for pain relief. Need to put in a Richard s splint & give crutches for comfort.
RICE. Fracture clinic follow up.
(e) If you clinically suspected a fracture of the patella but the x-rays appeared normal what could you do? f)
How do you treat infrapatellar bursitis?
Answer: Obtain skyline or oblique views. Answer: avoid the causative activity NSAIDS & rest Persistent symptoms
can lead to elective excision of the bursa If any systemic symptoms then watch for infective bursitis
‫ــــــــــــــــــــــــــــــــــــــــــــــــــــــــــــــــــــــــــــــــــــــــــــــــــــــــــــــــــــــــــــــــــــــــــــــــــــــــــــــــــــــــــــــــــــــــــــــــــــــــــــــــــــــــــــ‬
162- Question id: 2041
A 15 yo girl slipped rushing for a bus & fell on her outstretched hand. She complained of a painful wrist.

Fig 1
(a) What is name of the fracture shown?
Answer: Colles' fracture. Colles' fractures involve dorsal displacement of the distal radius fragment.
(b) What is the name of the characteristic deformity associated with this fracture?
Answer: Dinner fork deformity.
(c) What nerve can be compressed by severely displaced fractures of this kind? Where should sensation be
tested?
Answer: Median nerve. Sensation should be tested over the thumb & index fingers.
(d) If the distal radius fragment was displaced towards the palmer aspect what is the fracture called?
Answer: Smith's fractures involve palmar displacement of the distal radius fragment.
(e) What is the name of the classification system used for distal radius fractures?
Answer: The Frykman classification system.
65
‫ــــــــــــــــــــــــــــــــــــــــــــــــــــــــــــــــــــــــــــــــــــــــــــــــــــــــــــــــــــــــــــــــــــــــــــــــــــــــــــــــــــــــــــــــــــــــــــــــــــــــــــــــــــــــــــ‬
163- Question 41 of 50
A 18 yo woman presented with severe Rt iliac fossa pain. O/E she had a low grade fever.

(a) List 3 possible causes?


Answer: Appendicitis, cecal diverticulitis, meckel's diverticulitis ,acute ileitis,UTI & pelvic inflammatory disease.
(b) Name 3 non-imaging investigations which may be helpful?
Answer: A urinalysis is frequently obtained to rule out a UTI. The inflamed appendix is often in close proximity to
the bladder & ureter & as a result, microscopic hematuria & pyuria are found in up to 1/3 of Pts with acute
appendicitis. Pelvic cultures may be useful in sexually active, menstruating women. A beta-HCG is mandatory to rule
out an ectopic preg. Although leukocytosis is common 30% of Pts with acute appendicitis have a normal WBC count.
(c) Name 3 possible findings on plain radiograph in acute appendicitis?
Answer: Radiographic findings associated with acute appendicitis include, Rt lower quadrant appendicolith, localized
Rt lower quadrant ileus, loss of the psoas shadow, free air (occasionally), deformity of cecal outline & Rt lower
quadrant soft tissue density.
(d) Name 3 possible findings on st&ard CT scan in acute appendicitis?
Answer: Findings on st&ard CT scan which are consistent with a diagnosis of acute appendicitis include a thick wall
(>2 mm),appendicolith(seen in approximately 25% of Pts),target structure (concentric thickening of the inflamed
appendiceal wall),phlegmon,abscess,free fluid & fat str&ing/Rt lower quadrant inflammation.
(e) Name two bacterial groups which should be covered if a perforated appendix is suspected?
Answer: In Pts with perforated appendicitis, the antibiotic regimen should cover enteric gram -ve rods & anaerobes.
‫ــــــــــــــــــــــــــــــــــــــــــــــــــــــــــــــــــــــــــــــــــــــــــــــــــــــــــــــــــــــــــــــــــــــــــــــــــــــــــــــــــــــــــــــــــــــــــــــــــــــــــــــــــــــــــــ‬
164- Question id: 2042
A lactating 38 yo woman (G1 P1) presented with a painful red area on her Rt breast. On her initial presentation there
was no fluctuant mass palpable.

Fig 1
(a) What is the diagnosis? What is the most common differential diagnosis?
Answer: Mastitis. The most common differential diagnosis is plugged ducts. Plugged ducts usually present as
palpable lumps with tenderness without associated shooting pains or fever.
(b) Name two common aetiological agents?
Answer: Staphylococcus aureus, streptococcus, & Escherichia coli.
(c) Name 3 supportive measures used in the TTT of this condition?
Answer: Supportive measures include continued nursing, bed rest, NSAID such as ibuprofen for pain control.
(d) What is the initial antibiotic of choice for this condition? How long should antibiotic therapy continue for?
Answer: Antibiotic TTT should be started with flucloxacillin for 10 to 14 days.
(e) Despite antibiotic therapy the above lady represented 4 days later with a breast abscess. Name two risk
factors this lady has for the development of a breast abscess?
Answer: Risk factors for breast abscess formation include maternal age over 30 yrs of age, primiparity, gestational
age ≥ 41 weeks gestation, & mastitis.
‫ــــــــــــــــــــــــــــــــــــــــــــــــــــــــــــــــــــــــــــــــــــــــــــــــــــــــــــــــــــــــــــــــــــــــــــــــــــــــــــــــــــــــــــــــــــــــــــــــــــــــــــــــــــــــــــ‬
165- Question 47 of 50
A 46 yo man injured his Rt knee while pivoting during a Tae-kwon-do kick. There was a valgus, abducting stress on the knee. He
was tender over the medial aspect of the knee. There was pain with opening of the joint on valgus stress test of the knee.

66
(a) What is the diagnosis?
Answer: Medial collateral ligament injury.
(b) Name two other ligaments which provide support with valgus stress?
Answer: The ant. & post. cruciate ligaments also provide support with valgus stress, & injuries to these ligaments
may accompany a MCL injury.
(c) How are these injuries classified?
Answer: First degree, second degree & third degree separations.
(d) Name four management steps?
Answer: Ice, elevation, crutches, & activity limitation are advised during the first 7 to 14 days of therapy.
(e) What is the role of surgery in this condition?
Answer: In contrast to tears of the ant. or post. cruciate ligaments, surgery is rarely necessary for MCL repair, even in
Pts with third degree tears.
‫ــــــــــــــــــــــــــــــــــــــــــــــــــــــــــــــــــــــــــــــــــــــــــــــــــــــــــــــــــــــــــــــــــــــــــــــــــــــــــــــــــــــــــــــــــــــــــــــــــــــــــــــــــــــــــــ‬
166- Question 48 of 50
A 29 yo man presented with severe Rt sided flank pain which had occurred suddenly while driving to work on the
morning of presentation. He had no urinary symptoms & his only medical history was of depression. O/E he was
afebrile & his abd. was soft & non tender. Dipstick urine test revealed microcopic Bl. & a trace of protein.

(a) What is the most likely diagnosis?


Answer: Nephrolithiasis.
(b) What is the differential diagnosis for this condition? (Name four)
Answer: Renal cell carcinoma, renal cell carcinoma, aortic aneurysm , acute intestinal obstruction or appendicitis, &
drug seeking behaviour.
(c) How would you investigate this Pt? (Name 3)
Answer: CT KUB(if unavailable IVP or plain film of the abd.)U/S if it's necessary to avoid radiat., UE, FBC & MSU.
(d) How would you manage this Pt?(Name four)
Answer: NSAIDs & opioids, consider calcium channel blockers or alpha blockers, treat infection if present, &
urology consultation for pain that is not controllable in the ED or large stone size.
(e) What is the major determinant as to if this man will need a further procedure or can be managed
conservatively?
Answer: Stone size is the major determinant of the likelihood of spontaneous stone passage, although stone location
is also important. Most stones ≤4 mm in diameter pass spontaneously. For stones > 4 mm in diameter, there is a
progressive decrease in the spontaneous passage rate, which is unlikely with stones ≥10 mm in diameter.
‫ــــــــــــــــــــــــــــــــــــــــــــــــــــــــــــــــــــــــــــــــــــــــــــــــــــــــــــــــــــــــــــــــــــــــــــــــــــــــــــــــــــــــــــــــــــــــــــــــــــــــــــــــــــــــــــ‬
167- Question 49 of 50
A 14 yo girl has been assaulted by another girl in the park she comes in with epistaxis & a swollen nose. You suspect
a nasal fracture.

(a) What do you need to look for & document O/E?


Answer: Clinical diagnosis, no need to x-ray Ensure that can breathe through both nostrils Look for septal
haematoma as will need I&D (especially in children) Assess deviation (ask Pt to look in the mirror) NB is a head
injury, ensure that you check carefully for other facial/head injuries i.e. zygoma fracture.
(b) The nose is deviated, what do you do?
Answer: Give them an ENT follow up appointment for 5-7 days so that an MUA can be done within 10 days.
(c) She also complains of diplopia & on close inspect. she can’t look up properly on the Lt side, what is your
next course of action?
Answer: Arrange facial x-rays, need to consider orbital floor fracture
(d) Explain how you would manage prolonged epistaxis in an adult Pt?
Answer: Initial measures include ABCD: Aggressive TTT of hypovolaemia Check anticoagulant. status; if unsure or known
to be on anticoagulants then send Bl. for clotting. Pr. on the fleshy part of the nose for 10 min Could then move on to cautery
with silver nitrate & or put a cotton wall soaked in lidocaine with adrenaline in the nostril, never cauterize both sidfes
of the nasal septum If continues insert nasal pack Post. bleeding can respond to tamponade with a folley catheter.
‫ــــــــــــــــــــــــــــــــــــــــــــــــــــــــــــــــــــــــــــــــــــــــــــــــــــــــــــــــــــــــــــــــــــــــــــــــــــــــــــــــــــــــــــــــــــــــــــــــــــــــــــــــــــــــــــ‬
168- Question 4 of 10
An 18 yo man is brought to the ED after a hit & run. His GCS is 6/15 & the decision is made to intubate.

(a) What type of muscle relaxant is suxamethonium?


Answer: Depolarising muscle relaxant.
(b) What is the dose range for suxamethonium in adults?

67
Answer: 600micrograms-1mg/kg.
(c) In what circumstances is suxamethonium contraindicated? (Give two)
Answer: Hyperkalaemia, burns, paraplegia, crush injuries.
(d) What is the result of the administration of suxamethonium administration on intracranial pressure(ICP)?.
Answer: Suxamethonium causes a rise in ICP.
(e) What is the normal duration of action of suxamethonium?
Answer: About 5 minutes but longer in Pts with abnormal pseudo-cholinesterase enzymes.
‫ــــــــــــــــــــــــــــــــــــــــــــــــــــــــــــــــــــــــــــــــــــــــــــــــــــــــــــــــــــــــــــــــــــــــــــــــــــــــــــــــــــــــــــــــــــــــــــــــــــــــــــــــــــــــــــ‬
169- Question 8 of 10
A 70 yo lady fell on her outstretched Lt wrist. X Ray revealed a colles fracture & it was decided to manipulate the
fracture using a biers block technique.

(a) What should be recorded before the pocedure? (Name two)


Answer: Pt consent, BP, how long the Pt is fasted, pre-op assessment, ECG, SpO2.
(b) What are the contraindications to biers block?(Name four)
Answer: Severe HTN, obesity, peripheral vascular disease, raynauds syndrome, methaemoglobinaemia, children < 7
yrs, sickle cell disease or trait, uncooperative or confused Pt, & procedures needed in both arms.
(c) What is the local anaesthetic of choice?
Answer: Prilocaine 0.5%
(d) How much above the systolic BP should the pressure cuff be inflated to?
Answer: The tourniquet should be inflated to at least 100mmHg above the systolic BP.
(e) At least how long must the tourniquet be inflated for?
Answer: The tourniquet must be inflated for at least 20 minutes.
‫ــــــــــــــــــــــــــــــــــــــــــــــــــــــــــــــــــــــــــــــــــــــــــــــــــــــــــــــــــــــــــــــــــــــــــــــــــــــــــــــــــــــــــــــــــــــــــــــــــــــــــــــــــــــــــــ‬
170- Question 10 of 10
A 26 yo man is brought to the ED by ambulance after a MVA. He has a low GCS & is intubated on arrival. The
decision is made to insert a CV line. During the attempted insertion the Pts O2 saturation begins to drop.
(a) What are the clinical signs to suggest a tension pneumothorax?(Give four)
Answer: Absent breath sounds on the affected side, hyper-resonance over the affected lung, distended neck veins,
tachycardia, hypotension, & tracheal deviation.
(b) What would you do if you suspected a tension pneumothorax on the side of the central venous line?
Answer: Immediate decompression by inserting an IV cannula into the second intercostal space in the mid-clavicular
line just above the third rib.
(c) What is the next step if this initial TTT is successful?
Answer: Insertion of an axillary chest drain. Obtain a CXR.
(d) Why does tension pneumothorax cause cardiac arrest?
Answer: Movement of the mediastinum causes kinking of the great vessels & a decrease in venous return.
(e) If a Pt is receiving intermittent positive pressure ventilation (IPPV) what feature may cause the treating
clinician to suspect a tension pneumothorax?
Answer: A sudden increase in airway pressure.
‫ــــــــــــــــــــــــــــــــــــــــــــــــــــــــــــــــــــــــــــــــــــــــــــــــــــــــــــــــــــــــــــــــــــــــــــــــــــــــــــــــــــــــــــــــــــــــــــــــــــــــــــــــــــــــــــ‬
171- Question 5 of 10
An 18 yo man is brought to the ED after a hit & run. His GCS is 6/15 & the decision is made to intubate.

(a) What class of medication is thiopentone?


Answer: A barbituate.
(b) What are the effects of overdosage?
Answer: Hypotension, respiratory depression.
(c) What is the duration of action?
Answer: Duration of action of 5 to 10 minutes.
(d) What is the induction dose in an adult?
Answer: 4mg/kg
(e) What is the induction dose range in a child?
Answer: 2-7mg/kg.
‫ــــــــــــــــــــــــــــــــــــــــــــــــــــــــــــــــــــــــــــــــــــــــــــــــــــــــــــــــــــــــــــــــــــــــــــــــــــــــــــــــــــــــــــــــــــــــــــــــــــــــــــــــــــــــــــ‬
172- Question 2 of 5
A 35 yo man suffered a burns injury at work.

(a) Name four common signs of significant smoke inhalation injury?

68
Answer: Persistent cough, stridor, or wheezing, hoarseness, deep facial or circumferential neck burns, nares with
inflammat. or singed hair, carbonaceous sputum or burnt matter in the mouth or nose, blistering or edema of the oropharynx,
depressed mental status, including evidence of drug or alcohol use, resp. distress & hypoxia or hypercapnia.
(b) Describe immediate burn care & cooling? (3 steps)
Answer: Any hot or burned clothing, any jewelry, & any obvious debris should immediately be removed to prevent
further injury & to enable accurate assessment of the extent of injury, cool water or saline soaked gauze should be
applied, ice & freezing should be avoided to prevent frostbite & systemic hypothermia.
(c) What points are important to attain in the history? (Four points)
Answer: What burned (eg, chemicals, textiles). The locat. Of the fire (eg, enclosed or open space).Whether an explosion
occurred. Whether the Pt used alcohol or drugs. Whether there was associated trauma (eg, from falling debris)
(d) Describe burns in terms of superficial, superficial partial thickness, deep partial thickness & full thickness?
Answer: Superficial burns involve only the epidermal layer of skin. They are painful, dry, red, & blanch with
pressure. Superficial partial-thickness burns involve the epidermis & superficial portions of the dermis. They are
painful, red, & weeping, usually form blisters, & blanch with pressure. Deep partial-thickness burns extend into the
deeper dermis, damaging hair follicles & glandular tissue. They are painful to pressure only. They almost always
blister (easily unroofed), are wet or waxy dry, & have variable color from patchy cheesy white to red.Full-thickness
burns extend through & destroy the dermis. They are usually painless. Skin appearance can vary from waxy white to
leathery gray to charred & black. The skin is dry & inelastic, & does not blanch with pressure.
(e) In an adult what percentage of total body surface area does each leg, arm, ant. & post. Trunk & head
represent?
Answer: Each leg represents 18% TBSA; each arm represents 9% TBSA, the ant. & post. Trunk each represents 18%
TBSA, & the head represents 9% TBSA.
‫ــــــــــــــــــــــــــــــــــــــــــــــــــــــــــــــــــــــــــــــــــــــــــــــــــــــــــــــــــــــــــــــــــــــــــــــــــــــــــــــــــــــــــــــــــــــــــــــــــــــــــــــــــــــــــــ‬
173- Question 1 of 20
A 69 yrs old lady is brought into the resuscitat. Room. She is too unwell to give a history but her daughter tells you
she has complained abd. Pain for the last 3 days. Over the last 24 hrs she has become increasingly unwell. Clinical
exam. Reveals a tender abd. in the epigastrium & Rt upper quadrant. She is clearly icteric. PR 135, BP 88/45, temp
39.5, RR 35/min, Sats 98% (on high flow O2) Hb 12.2, WCC 21.9, Plt 290 Na 137, K 4.1, Cl 105, HCO3 12, Urea
10.0, Creatinine 125 AST 56, Gamma GT 37, Alk Phosphate 742, total Billi 65, T. protein 65, Albumin 35

(a) What is the most likely diagnosis?


Answer: Biliary sepsis: Ascending cholangitis
(b) What secondary condition has arisen as a result?
Answer: Septic shock
(c) What 3 physical findings are classically described as Charcot�s triad?
Answer: Jaundice, Pain (R upper quadrant) & Fever
(d) Give 2 commonly encountered bacterial pathogens that you might expect to isolate in this case?
Answer: E. Coli Klebseilla Enterococcus Bacteroids sp.
(e) The Pt is being resuscitated with fluids & inotropes. IV antibiotics have been commenced. What imaging
modality would be most appropriate in this unstable Pt at this time, to confirm the clinical diagnosis?
Answer: Abd. USS
‫ــــــــــــــــــــــــــــــــــــــــــــــــــــــــــــــــــــــــــــــــــــــــــــــــــــــــــــــــــــــــــــــــــــــــــــــــــــــــــــــــــــــــــــــــــــــــــــــــــــــــــــــــــــــــــــ‬
174- Question id: 2102
A 23 yo male from Poland presents to the department in the early hrs. He has been badly beaten with a wooden club
to the face. He is stable but you suspect facial fractures.

Fig 1
(a) What are the important things to test for & document?

69
Answer: Sensation of the face (Anaesthesia over the region supplied by the infraorbital nerve (lower lid, cheek, side
of nose, upper lip, upper teeth & gums). Check for tenderness over the zygomatic arches, maxilla, m&ible & TMJ
Assess mouth opening Look for bruising oedema, subcutaneous emphysema Nasal deviation Visual acuity Eye
movements any diplopia Uneven pupilary levels due to orbital floor damage) CSF rhinorrhoea Sunconjunctival
haemorrhage without a post. Border (suggests an orbital wall or ant. cranial fossa fracture)
(b) What name is given to the fracture type shown in the figure 1?
Answer: Le Fort II
(c) The Pt has the fracture shown above, you notice that he has Bl. & clear fluid coming from his nostrils
known as the tramline effect. What do you need to do?
Answer: Contact neurosurgeons immediately Ensure Pt doesn’t blow nose Give antibiotics Ensure TT prophylaxis
(d) He is unsure of his tetanus status what will you do?
Answer: If unsure then give a dose of combined DTP (ensure GP follow up) then make an assessment of if it is a
tetanus prone wound.
(e) What constitutes a tetanus prone wound?
Answer: Heavy contamination with soil or faeces Devitalised tissue Infection or wounds >6hrs old Puncture wounds
& animal bites
‫ــــــــــــــــــــــــــــــــــــــــــــــــــــــــــــــــــــــــــــــــــــــــــــــــــــــــــــــــــــــــــــــــــــــــــــــــــــــــــــــــــــــــــــــــــــــــــــــــــــــــــــــــــــــــــــ‬
175- Question id: 4544
A 21 yo female presents to the ED as she was sent by NHS direct due to tingling in her RT arm, it seems to worsen
when she lifts her arm upwards.

(a) What does the CXR in figure 1 reveal?


Answer: RT cervical rib
(b) How does this explain her symptoms?
Answer: Sometimes a cervical rib can press on the subclavian artery & brachial plexus causing transient vascular
insufficiency or paraesthesiae. This is known as thoracic outlet syndrome.
(c) What is the investigation of choice upon discovering this finding?
Answer: CT angiography with the arms down & the arms raised will show if there is any vascular compromise.
(d) In the ED what things do you need to arrange?
Answer: Although that vast majority of TOS is non-urgent a vascular & in this care arterial cause is more urgent as
the possibility of throwing off an embolus are increased. A vascular surgery consultation is needed. They may want
the Pt to be started on heparin & admitted?
(e) If this was to be addressed operatively what risks would you need to tell the Pt about?
Answer: Lymphocele & phrenic nerve damage are the main 2 significant ones.
‫ــــــــــــــــــــــــــــــــــــــــــــــــــــــــــــــــــــــــــــــــــــــــــــــــــــــــــــــــــــــــــــــــــــــــــــــــــــــــــــــــــــــــــــــــــــــــــــــــــــــــــــــــــــــــــــ‬
176- Question 5 of 20
The following CT scan (fig 1) was performed in the ED on a 75 yo man who had fallen earlier in the day. His GCS
was 13/15 due to him being confused & having his eyes shut.

(a) Describe the main findings (fig 1)


Answer: Large acute subdural haematoma on the RT extending from the frontal lobe to the occipital lobe. Midline
shift & surrounding oedema.
(b) What is the most common intracranial injury post traumatic head injury?
Answer: Subdural haematoma
(c) What is the pathophysiology of the findings seen on the CT scan? (Simple description required)
Answer: Essentially, injury to the bridging veins between the brain surface & the dura matter caused by shearing
forces that tear the veins.
(d) What is the approximately mortality for the condition seen on the CT scan?
Answer: Answer: 30-90% around 60% is typical1
(e) What is the definitive management?

70
Answer: Surgery for emergent decompression has been advocated if the acute subdural hematoma is associated with
a midline shift greater than or equal to 5 mm. Surgery also has been recommended for acute subdural hematomas
exceeding 1 cm in thickness. These indications have been incorporated into the Guidelines for the Surgical
Management of Acute Subdural Hematomas proposed by a joint venture between the Brain Trauma Foundation & the
Congress of Neurological Surgeons released in 2006.
‫ــــــــــــــــــــــــــــــــــــــــــــــــــــــــــــــــــــــــــــــــــــــــــــــــــــــــــــــــــــــــــــــــــــــــــــــــــــــــــــــــــــــــــــــــــــــــــــــــــــــــــــــــــــــــــــ‬
177- Question 6 of 20
A fit & well 68 yo lady is brought in to the ED early in the morning. She slipped whilst getting out of the shower &
now has a very painful Lt Hip. She thinks that she has dislocated her prosthesis again. X-rays confirm that the
prosthesis has dislocated posteriorly.

(a) What is the best method to reduce the hip?


Answer: There are no RCTs to address this quest. But recent Best Bets would suggest that sedat. With propofol in the
ED in the RT hands has got a very good success rate. This is highly dependant on the doctor having the necessary
Anaesthetic experience to manage any complicat. Of propofol sedat. The alternative is reducting. Under GA.
(b) What important questions do you need to ask in the history before considering the above?
Answer: This question is trying to test the candidate's ability to think ahead. It is important to ensure that the Pt is
starved if you are going to use propofol sedation.
(c) What is it paramount to test prior to considering reduction?
Answer: The function of the sciatic nerve. The finding of sciatic nerve dysfunction mandates surgical exploration to
release or repair the nerve.
(d) What method would you use to reduce the hip?
Answer: Allis method The Pt should be supine & under procedural sedat. The combined weight of the Pt & physician
may exceed the weight limit of the stretcher. It is generally unsafe for the physician to be standing on a stretcher. For
these reasons, placing the Pt on the floor rather than on the stretcher is often useful. An assistant should stabilize the
pelvis. The physician should initially be toward the Pt's feet, providing in-line tract. The physician should then gently
flex the hip 60-90o while maintaining in-line tract. At this point, the physician is standing directly above the Pt's hip,
providing traction in-line with the deformity. Gently adducting the hip can force the head of the femur laterally & help
it clear the acetabular rim. Alternately, gentle lateral traction can be applied to the proximal femur. Reduction can be
confirmed by a click that is felt & may be heard as well. The Pt should assume normal anatomical position. Stimson
method This method is mechanically the same as the Allis method, but the positioning is opposite. Although some
physicians prefer this method because of its technical ease & high success rate, this method has some important
disadvantages. It requires the Pt to be in a prone posited. Which may not be possible for the Pt with multiple trauma.
Monitoring the Pt during procedural sedat. Is also difficult. The prone Pt is placed so the pelvis on the affected side
hangs either over the end or over the side of the stretcher. The hip & knee are flexed to 90o. Downward pressure is
applied to the popliteal fossa, providing traction in-line with the deformity. An assistant stabilizes the pelvis & trunk
preventing the Pt from being pulled off the stretcher. Whistler technique The Pt is placed supine with ipsilateral knee
flexed to 120o. The physician st&s on the affected side & places an arm under the ipsilateral knee with his or her h&
resting on the contralateral knee. The pelvis & ankle are stabilized by an assistant or the physician's free h&. The
physician raises his or her arm, which applies an ant. Force to the knee & subsequently to the affected hip. Ant.
dislocations A modified Allis technique may be used. The Pt is placed supine. The physician stands at the foot of the
stretcher. Traction is applied to a neutral hip while an assistant stabilizes the pelvis. Gentle lateral traction applied to
the proximal femur facilitates the femoral head clearing the acetabular rim.
(e) What should the further care post reduction include?
Answer: After reduct. of the hip dislocate., obtain repeat AP & lateral radiographs of the hip, as well as repeat CT scans
or MRIs of the hip to verify proper reduct. After either open or closed reduct. of a hip dislocat., the Pt is instructed to
remain on bed rest with his or her legs abducted & with skeletal traction designed to keep the hip from displacing
post.ly. The durat. Of tract. is approximately 2 wks, but the recommended period with no weight bearing is
controversial & varies from 9 days to 3 months.
‫ــــــــــــــــــــــــــــــــــــــــــــــــــــــــــــــــــــــــــــــــــــــــــــــــــــــــــــــــــــــــــــــــــــــــــــــــــــــــــــــــــــــــــــــــــــــــــــــــــــــــــــــــــــــــــــ‬
178- Question 7 of 20
A 52 yo man presented with ant. Shoulder pain which had come on over the previous 24 hrs since a trip to the
gymnasium in which he had been using various exercise machines. The pain was agravated by lifting & overhead
reaching. He was tender in the bicipital groove.

(a) What is the most likely diagnosis?


Answer: Bicipital tendonitis.
(b) What clinical test may be used to aid in the diagnosis?

71
Answer: Yergason's test this test of supination against resistance is positive if it elicits pain in the bicipital groove.
Also the pain of bicipital tendonitis is frequently aggravated by the painful arc maneuver.
(c) What is the role of plain X-Rays in this condition?
Answer: Plain x-rays of the shoulder (including PA, external rotation, Y-outlet, & axillary views) are not necessary in
most Pts with bicipital tendonitis.
(d) How would you manage this Pt?
Answer: Ice , advise Pts to eliminate lifting & restrict over-the-shoulder positions & reaching, weighted pendulum
stretching exercise for 5 to 10 min. a day acutely, & then 3 times/wk as symptoms improve to reduce the chance of
recurrent tendonitis, isometric toning exercises of elbow flexion should begin 3 to 4 wks after the acute pain has
resolved. If symptoms persist then a corticosteroid injection or orthopaedic referral should be considered.
(e) The Pt went on to develop a lump just proximal to the antecubital fossa. What are the risk factors for this
compication? (Name 3)
Answer: Risk factors for rupture include recurrent tendonitis, previous rotator cuff or contralateral biceps tendon
rupture, age greater than 50, poor general shoulder conditioning, & rheumatoid arthritis.
‫ــــــــــــــــــــــــــــــــــــــــــــــــــــــــــــــــــــــــــــــــــــــــــــــــــــــــــــــــــــــــــــــــــــــــــــــــــــــــــــــــــــــــــــــــــــــــــــــــــــــــــــــــــــــــــــ‬
179- Question id: 2038
A 19 yo man had fallen on his Rt shoulder while playing soccer.His X Ray is shown.

Fig 1
(a) What is the diagnosis?
Answer: Ant. shoulder dislocation.
(b) Name 3 findings O/E?
Answer: An anteriorly dislocated shoulder causes the arm to be slightly abducted & externally rotated. The Pt resists
all movement. The acromion appears prominent in thin individuals & there is loss of the normal rounded appearance
of the shoulder. Axillary nerve dysfunction manifests as loss of sensation in a "shoulder badge" distribution, although
this finding is not reliably present.
(c) Name two factors associated with fracture?
Answer: Factors associated with fracture include age over 40, 1st time dislocate. & traumatic mechanism (eg, fights or
fall)
(d) Describe what finding would be expected on the Y radiographic view?
Answer: When an ant. Dislocation is present, the humeral head appears medial to the "Y".
(e) Name two associations of post. Shoulder dislocations?
Answer: Violent muscle contract. Following a seizure or electrocution represent common causes of post. Shoulder
dislocat.
‫ــــــــــــــــــــــــــــــــــــــــــــــــــــــــــــــــــــــــــــــــــــــــــــــــــــــــــــــــــــــــــــــــــــــــــــــــــــــــــــــــــــــــــــــــــــــــــــــــــــــــــــــــــــــــــــ‬
180- Question 9 of 20
A 38 yo man was punched in the face last night. He has come in today as he cannot see properly in his Lt eye it keeps
going blurry & he is seeing double.

(a) Explain how you would test visual acuity; write down how you would record it?
Answer: Use a Snellen chart 6 metres away from the Pt get them to read off the chart covering one eye at a time,
instruct them to go down the chart until they cannot read the letters any more. The line they reach will determine their
acuity i.e. 6/12 or 6/5 (best it could be) If Pts read additional letters form the line below record it as such: 6/12 +2.

72
(b) His RT eye appears to be slightly sunken & he had a subconjunctival haemorrhage. What do you need to
establish with regard to the subconjunctival haemorrhage?
Answer: Can you see the back of it? If not then it could represent an orbital wall fracture or an ant. Cranial fossa fracture.
(c) When you assess eye movements what are you looking out for?
Answer: Restriction of upward gaze due to the inferior rectus muscle being trapped in the broken orbital floor.
(d) What is the tear drop sign seen on facial x-rays?
Answer: It represents soft tissue mass in the top of the maxilla. Sinus from muscle that has slipped down through the
orbital floor
‫ــــــــــــــــــــــــــــــــــــــــــــــــــــــــــــــــــــــــــــــــــــــــــــــــــــــــــــــــــــــــــــــــــــــــــــــــــــــــــــــــــــــــــــــــــــــــــــــــــــــــــــــــــــــــــــ‬
181- Question 14 of 20
A 25 yo woman presented to the ED after coming home from a sking holiday. She had fallen on her last day & had
persistent pain in her RT hand at the base of her thumb. There was point tenderness over the ulnar side of the
metacarpophalangeal joint of the thumb.

(a) What is the diagnosis?


Answer: Ulnar collateral ligament injury (gamekeeper's thumb or skier's thumb).
(b) What is the mechanism of injury?
Answer: Forced radial deviation of the thumb.
(c) How is this condition tested for clinically?
Answer: Valgus stress testing determines the irritat. & integrity of the ulnar collateral ligament (stress is applied across
the MP joint to the collateral ligament located on the ulnar side of the thumb [ie the thumb is pushed away from the palm]).
(d) Why is this condition significant?
Answer: The strength or holding power of the thumb & first finger may be compromised.
(e) How should this Pt be treated if x ray is unremarkable but there is clinical evidence of a severe injury?
Answer: Referral to an orthopedist or hand surgeon is indicated for Pts with evidence of complete ligament tear. For
lesser injuries a thumb spica splint or a dorsal hood splint for approximately 3 weeks is appropriate.
‫ــــــــــــــــــــــــــــــــــــــــــــــــــــــــــــــــــــــــــــــــــــــــــــــــــــــــــــــــــــــــــــــــــــــــــــــــــــــــــــــــــــــــــــــــــــــــــــــــــــــــــــــــــــــــــــ‬
182- Question id: 2106
A 23 yo university rugby player was tackled whilst leaping to touch down a try he was airlifted by 2 players & l&ed
with some force on his Lt shoulder.

Fig 1 Fig 2

Fig 3
(a) What is shown in the radiograph (fig 1)?

73
Answer: AC joint dislocation, grade III. The AC ligament is ruptured along with the conoid & trapezoid ligaments.
(b) How would you manage this?
Answer: Good analgesia broad arm sling & orthopaedic follow up may require internal fixation. Type IV-VI:
Account for more than 10-15% of total acromioclavicular dislocat. & should be managed surgically. Failure to reduce
& fix these will lead to chronic pain & dysfunct. Type III the acromioclavicular joint capsule & coracoclavicular
ligaments are completely disrupted. The coracoclavicular interspace is 25-100%>the normal shoulder. Type IV This is
a type III injury with avulsion of the coracoclavicular ligament from the clavicle, with the distal clavicle displaced
posteriorly into or through the trapezius. Type V This is type III but with exaggerate. of the vertical displacement of
the clavicle from the scapula-coracoclavicular interspace 100-300%>the normal side, with the clavicle in a SC
position. Type VI This is a rare injury. This is type III with inf. Dislocat. of the lateral end of the clavicle below the coracoid.
(c) If a Pt sustains a fracture of the scapular what should you do?
Answer: Check for other injuries, it takes considerable force to fracture the scapula so look for rib fractures &
particularly any evidence of a lung contusion.
(d) What does the radiograph in figure 2 show?
Answer: Fracture of the humeral head, 3 part fracture
(e) What does the radiograph in figure 3 show? How would you manage this in the ED?
Answer: First do no harm! Do not pull this! You will make it worse, will need ORIF. Give good analgesia & support
in sling Answer: Salter Harris 4 fracture of the distal tibia, medial malleolus.
‫ــــــــــــــــــــــــــــــــــــــــــــــــــــــــــــــــــــــــــــــــــــــــــــــــــــــــــــــــــــــــــــــــــــــــــــــــــــــــــــــــــــــــــــــــــــــــــــــــــــــــــــــــــــــــــــ‬
183- Question 16 of 20
A 40 yo woman presented with severe abd. Pain. This had come on gradually over the course of the previous 24 hrs.
O/E her vital signs were within the normal range & her abd. Was soft. Bl. investigations revealed a serum amylase
Level which was five times the upper limit of normal? A diagnosis of pancreatitis was made.

(a) What is the most common cause of this condition in women?


Answer: Women-Gallstone pancreatitis.Men-Alcohol.
(b) Name five findings which may be present O/E in this condition?
Tachycardia, fever, shock, abd. Tenderness, abd. Distention, guarding, ecchymotic discoloration in the flank (Grey-
Turner's sign) or the periumbilical region (Cullen's sign), an epigastric mass due to pseudocyst formation may become
palpable in the course of the disease.
(c) How quickly does serum amylase rise in acute pancreatitis?
Serum amylase  within 6-12 hrs of onset. In uncomplicated attacks, serum amylase is usually  for 3 to 5 days
(d) What other conditions may cause a rise in serum amylase? (Give four conditions)
Post-ERCP, acute cholecystitis, parotitis, intestinal trauma, intestinal surgery, intestinal obstruction, intestinal
infarction, alcoholism, anorexia, cirrhosis, ruptured ectopic pregnancy, salpingitis, renal failure, acidosis, &
malignancy with ectopic amylase production.
(e) What is the role of CXR is this Pt?
Approximately 1/3 of Pts with acute pancreatitis have abnormalities visible on the chest X ray such as  of a
hemidiaphragm, pleural effusions, basal atelectasis, pulmonary infiltrates, or acute respiratory distress syndrome.
‫ــــــــــــــــــــــــــــــــــــــــــــــــــــــــــــــــــــــــــــــــــــــــــــــــــــــــــــــــــــــــــــــــــــــــــــــــــــــــــــــــــــــــــــــــــــــــــــــــــــــــــــــــــــــــــــ‬
184- Question 17 of 20
A 39 yo man comes into the department in triple immobilisation with a hard collar, blocks & tape. He was the driver
in a rear end shunt a few hrs earlier. He walked from the vehicle, which was hit at around 30 m.p.h.

(a) According to the Canadian c-spine rules what are the 3 high risk factors that mandate radiography? (3
marks)
Answer: 1. any dangerous mechanism of injury 2. Any paraesthesia in the extremities 3. Age >65
(b) Over what GCS range is it acceptable to apply the Canadian c-spine rules?
Answer: Only validated when GCS is 15/15
(c) He doesn't fit any of the high risk factors. Name as many of the low risk factors, which if present would
mean you could assess the range of movement in the neck. (3 marks if all 6, 2 marks 4-5, 1 mark 2-3)
Answer: Sitting in the ED -Simple rear end shunt* (excludes roll-over, hot by large truck, high speed crash, pushed into
traffic)-Ambulatory at any time-Delayed onset of neck pain (not immediate)-Absence of midline c-spine tenderness
(d) You decide that it is safe to ask him to rotate his neck. However he can't move his neck to 45 degrees. What
do you do now?
Answer: Needs x-ray
(e) Do you know of any other validated rules for assessing the need for radiography with regard to neck
injuries in the alert & stable trauma Pt?
Answer: The NEXUS rules.
74
‫ــــــــــــــــــــــــــــــــــــــــــــــــــــــــــــــــــــــــــــــــــــــــــــــــــــــــــــــــــــــــــــــــــــــــــــــــــــــــــــــــــــــــــــــــــــــــــــــــــــــــــــــــــــــــــــ‬
185- Question id: 2014
A 55 yrs old lady presents with a 2 day history of abd. pain. Clinically she is mildly tachypnoeic & has a sinus
tachycardia. Abd. Exam. Confirms generalised tenderness.

(a) What is the radiological diagnosis (see fig 1)?


Answer: Small bowel obstruction
(b) Name 3 symptoms associated with this x-rays appearance.
Answer: Persistant vomiting / bilious vomiting Colicky abd. Pain Abd. Distension Absolute constipation
(c) Name 3 potential causes of this x-rays appearance.
Answer: Adhesions Hernia Intraluminal obstruction (e.g. gallstones, or food bolus) Stricture Neoplasm
(d) Give 3 steps in the early management of this Pt. Post immediate ABCD assessment, assume O2 has been
started
Answer: Nil by mouth / NGT IV fluid resuscitation Analgesia Urinary catheter General surgical referral Bl. tests
‫ــــــــــــــــــــــــــــــــــــــــــــــــــــــــــــــــــــــــــــــــــــــــــــــــــــــــــــــــــــــــــــــــــــــــــــــــــــــــــــــــــــــــــــــــــــــــــــــــــــــــــــــــــــــــــــ‬
186- Question 20 of 20
A 26 yo presents to the ED after being hit in the face by a baseball bat. The treating clinician suspects a facial bone fracture

(a) What are the imporant points on inspection of the facial bones? (Give four)
Answer: Asymmetry, flattening of the cheek suggests a depressed zygomatic fracture; a flattened & elongated face
may be due to post. & downward displacement of the maxilla (the so called dish face deformity), nasal deviation,
saddle deformity, an orbital floor fracture may cause uneven pupil levels, CSF rhinorrhoea, and subconjunctival
haemorrhage without a post. Border.
(b) If there is hypo/anaesthesia of the cheek, side of the nose & upper lip which nerve may be affected?
Answer: Infraorbital nerve.
(c) What is the significance of subcutaneous emphysema in this Pt?
Answer: Subcutaneous emphysema suggests a compound fracture often of the maxillary sinus.
(d) If an mandibular fracture is suspected what X Ray should be requested?
Answer: Orthopantomogram.
(e) If there is no evidence of facial fracture on X ray but there is a strong clinical suspicion of facial fracture
how would you proceed?
Answer: Expert consultation or follow up.
‫ــــــــــــــــــــــــــــــــــــــــــــــــــــــــــــــــــــــــــــــــــــــــــــــــــــــــــــــــــــــــــــــــــــــــــــــــــــــــــــــــــــــــــــــــــــــــــــــــــــــــــــــــــــــــــــ‬
187- Question 5 of 20
A 36 yo man was hit by a car.He complained of a painful RT knee. X ray revealed a tibial plateau fracture.

(a) Which side of the plateau is usually involved?


Answer: Tibial plateau fractures most commonly involve the lateral plateau after a direct blow that produces a strong
force to the lateral knee.
(b) Besides an AP film which other films should be requested?
Answer: Lateral & oblique.
(c) What is the typical finding on radiography?
Answer: Radiographs typically reveal a depression of the lateral tibial plateau in moderate to severe fractures.
(d) When clinical suspicion is high but radiographs are equivocal how should the clinician proceed?
Answer: Further imaging should be with CT or MRI.

75
(e) Name four management steps?
Answer: Compression, icing, knee splinting in full extension, elevation, orthopaedic referral & strict non-weight
bearing are the initial phase of TTT of a tibial plateau fracture.
‫ــــــــــــــــــــــــــــــــــــــــــــــــــــــــــــــــــــــــــــــــــــــــــــــــــــــــــــــــــــــــــــــــــــــــــــــــــــــــــــــــــــــــــــــــــــــــــــــــــــــــــــــــــــــــــــ‬
188- Question id: 2071
A 60 yo woman presented with a 2 day history of abd. Pain & bloating. She had not passed a bowel motion for 24 hrs
& felt nauseated. Her past medical history was significant for an appendicetomy & a cholecystectomy. O/E she was
febrile at 38degrees & her abd. Was bloated & diffusely tender.

Fig 1
(a) What is the diagnosis?
Answer: Small bowel obstruction.
(b) What is the differential diagnosis? (Give two)
Answer: Intestinal pseudo-obstruction & paralytic ileus.
(c) What are the causes of this condition? (Give five causes)
Answer: Adhesions, hernia, volvulus, congenital malformat. Duplicate, atresia, stenosis, neoplasm, inflammatory
stricture, radiat. enteritis, intussusception, gallstones, feces or meconium, bezoar, & traumatic intramural hematoma.
(d) How would you further investigate this Pt? (Name four)
Answer: Urea & creatinine & the hematocrit can be used to gauge the degree of dehydrate. Leukocytosis with Lt
Ward shift may be present. Metabolic alkalosis can be seen in Pts who have frequent emesis. Metabolic (lactic)
acidosis can result if the bowel becomes ischemic or if dehydrate. Is severe enough to cause hypoperfusion of the gut
& other tissues. Serum lactate is found to be elevated in Pts with mesenteric ischemia & is a sensitive. CT abd. May
give more information on the level & cause of the obstruction
(e) How would you manage this Pt? (Name 3 steps)
Answer: Nasogastric tube, IV fluids, surgical consultation
‫ــــــــــــــــــــــــــــــــــــــــــــــــــــــــــــــــــــــــــــــــــــــــــــــــــــــــــــــــــــــــــــــــــــــــــــــــــــــــــــــــــــــــــــــــــــــــــــــــــــــــــــــــــــــــــــ‬
189- Question 13 of 20
A 44 yo presents with a short history of an RT sided neck swelling that seems to be going up & down. It is quite
tender & it seems to be most prominent when he eats. O/E he appears well, he is afebrile & there are no skin changes.
The mass appears to be just below the angle of the RT mandible & is approximately 3cm by 2cm.

(a) What further clinical exam. Would you perform in the ED?
Answer: Full exam. Of the oral cavity & neck. Need to look inside the mouth the check for any lesions/salivary
calculi/ signs of tooth decay/infection/. Also full systemic exam. To ensure no signs of sepsis.
(b) What investigations should you perform?
Answer: If you think that it is infected then it might be worth doing some baseline inflammatory markers & of course
Bl. cultures. Organise an OP silaogram through the maxillofacial team.
(c) What is the most likely diagnosis given the above history?
Answer: submandibular calculus.
(d) What are the potential TTTs?
Answer: Gentle probing into the duct from inside the mouth with a thin blunt instrument can sometimes free a stone
which then falls into the mouth. This is done by a doctor. Therapeutic sialendoscopy. This is a similar procedure to
that described above. It also uses a very thin endoscope (tube) with a camera & light at the tip. The tube is pushed into
76
the duct. If a stone is seen, then a tiny 'basket' or pair of 'grabbers' that are attached to the tube is used to grab the stone
& pull it out. This technique can successfully remove about 17 in 20 stones. Local anesthetic is usually injected into
the duct first to make this procedure painless. In some cases, where the stone is rather large, the stone is broken up
first & then the fragments are pulled out. A small operation to cut out the stone is the traditional TTT, but is done less
& less as therapeutic sialendoscopy has become available. It may still be needed if therapeutic sialendoscopy is not an
available option, or if it fails. 'Shock wave' TTT (lithotripsy) may be an option. This uses ultrasound waves to break
up stones. The broken fragments then pass out along the duct. This is a relatively new TTT for salivary stones
(although it has been used for some yrs to treat kidney stones). However, it is not done commonly. Sometimes shock
waves are used to break up a large stone when therapeutic sialendoscopy is done to make smaller fragments which
can be more easily removed.
‫ــــــــــــــــــــــــــــــــــــــــــــــــــــــــــــــــــــــــــــــــــــــــــــــــــــــــــــــــــــــــــــــــــــــــــــــــــــــــــــــــــــــــــــــــــــــــــــــــــــــــــــــــــــــــــــ‬
190- Question id: 4532
A 24 yo drug dealer has been shot in the abd, he comes in the resus room & is haemodynamically unstable. You have
been pre-alerted.

(a) As part of your primary assessment B seems normal clinically to you, although you have not yet seen the
back. In E what do you need to look for?
Answer: This question is stressing the importance of fully looking around the body including the back/loin/groin &
Sides for entry & exit points of the bullets. An abd. Emergency could fast become a thoraco-abd. Emergency if what
first seems like an isolated abd. Gun shot wound turns out to have an exit point high in the back of the chest.
(b) Where is this Pt going & what measures need to occur prior to that?
Answer: Needs urgent laparotomy, Needs good IV access first ideally central lines & arterial lines.
(c) The Pt is bleeding from the wound site- the Hb on the initial gas is 6.9. What will you do?
Answer: The Pt will need transfusion of whole Bl., FFP, cryoprecipitate & consideration should be given to rV111a,
evidence is emerging stating that giving it early although not yet proven to decrease mortality has been shown to
reduce ICU days, reduce the amount of Bl. required etc.
(d) With reference to figure 1: what is shown & what technique is being used here?
Answer: Damage control laparotomy shows a Penrose drain sutured to a Foley catheter through the liver. One could
use a Senstaken tube to create a tamponade effect. This is being used to stop intra-abd. Haemorrhage.
(e) What is it important to remember to give in all cases?
Answer: Tetanus prophylaxis & anaerobic antibiotic cover.
‫ــــــــــــــــــــــــــــــــــــــــــــــــــــــــــــــــــــــــــــــــــــــــــــــــــــــــــــــــــــــــــــــــــــــــــــــــــــــــــــــــــــــــــــــــــــــــــــــــــــــــــــــــــــــــــــ‬
191- Question id: 2039
A 28 yo man fell while ice skating with his son. His x ray is shown below.

(a) What is the diagnosis?


Answer: Scaphoid Fracture.
(b) Name two mechanisms of injury?
Answer: Fractures of the scaphoid can occur either with direct axial compression or with hyperextension of the wrist
such as a fall on the outstretched hand
(c) Name two findings O/E?
Answer: Grip strength is typically reduced. On palpation pain is typically in 1 of 3 places: The volar prominence at
the distal wrist crease for distal pole fractures; in the anatomical snuff box for waist fractures & just distal to Lister's
Tubercle (a longitudinal bony prominence on of the distal radius just to the ulnar side of the extensor carpi radialis
tendon) for proximal pole fractures.
(d) Name 3 radiographs which should be requested if this injury is suspected?
Answer: For suspected scaphoid fractures, stanard radiographs include a posteroant. (PA), true lateral, & a scaphoid
view. The scaphoid view is made in PA format with the wrist in full pronation, & ulnar deviation.
77
(e) What findings on a lateral radiograph are suggestive of this condition?
Answer: The scapholunate angle is formed by a line bisecting the scaphoid in its longitudinal axis & a line bisecting
the lunate. This angle should be between 40 & 60 degrees. More or lesser degrees indicate ligamentous instability
&/or fracture.
‫ــــــــــــــــــــــــــــــــــــــــــــــــــــــــــــــــــــــــــــــــــــــــــــــــــــــــــــــــــــــــــــــــــــــــــــــــــــــــــــــــــــــــــــــــــــــــــــــــــــــــــــــــــــــــــــ‬
192- Question 19 of 20
A 32 yo woman was bitten by a dog 2 days prior to presentation. She had attended her primary care physician who
had treated her with anti-tetanus prophylaxis but had not prescribed any anti-biotics.O/E there were bite marks on
either side of the thenar eminence of the RT hand. The area was red, swollen & tender. She was afebrile.

(a) Name two bacterial classes that may have caused this wound infection?
Answer: The predominant organisms in animal bite wounds are the oral flora of the biting animal (notable pathogens
include Pasteurella, Capnocytophaga & anaerobes) as well as human skin flora (such as staph. & streptococci).
(b) Name 3 possible complications of this wound infection?
Answer: Subcutaneous abscesses, associated crush injury, osteomyelitis, tenosynovitis, & septic arthritis.
(c) Name 3 management steps?
Answer: TTT of animal bites includes wound care, antibiotic therapy, and vaccinat. radiographic imaging & surgical
evaluat.
(d) What antibiotics should be started in this lady?
Answer: Options for empiric gram--ve & anaerobic coverage include 1.Monotherapy with a beta-lactam/beta-
lactamase inhibitor, such as one of the following: Ampicillin-sulbactam (3 g every six hrs), Piperacillin/tazobactam
(4.5 g every eight hrs), Ticarcillin-clavulanate (3.1 g every four hrs), or 2. A third generation cephalosporin such as
ceftriaxone (1 g IV every 24 hrs) PLUS metronidazole (500 mg IV every eight hrs).
(e) Name two circumstances where rabies vaccination should be considered?
Answer: Rabies prophylaxis should be considered in the setting of bites from unvaccinated pets, wild animals & in
geographic areas where the prevalence of rabies is high.
‫ــــــــــــــــــــــــــــــــــــــــــــــــــــــــــــــــــــــــــــــــــــــــــــــــــــــــــــــــــــــــــــــــــــــــــــــــــــــــــــــــــــــــــــــــــــــــــــــــــــــــــــــــــــــــــــ‬
193- Question id: 2066
A 56 yo man presented with a severe headache which had a sudden onset. His CT Brain scan is shown.
(a) What is the diagnosis?
Answer: SAH
(b) Name four risk factors for this condition?
Answer: Smoking, HTN, alcohol, family history, phenylpropanolamine in appetite suppressants & oestrogen deficiency

Fig 1
(c) How would you manage this Pt?
Answer: Neurosurgical consultation, intensive care setting for constant hemodynamic & cardiac monitoring, stool
softeners, bed rest,analgesia to diminish hemodynamic fluctuations & lower the risk of rebleeding & pneumatic
compression stockings to limit risk of deep vein thrombosis should be utilized while Pts are immobile.Antithrombotic
discontinuation, ICP monitoring & nimodipine therapy.
(d) What are the complications of this condition? (Name four)
Answer: Rebleeding, vasospasm & delayed cerebral ischemia, hydrocephalus, increased intracranial pressure,
seizures, hyponatremia, cardiac abnormalities, & hypothalamic dysfunction & pituitary insufficiency.
(e) Name two prognostic factors?

78
Answer: 1.Level of consciousness & neurologic grade on admission, 2.Pt age (inverse correlation), 3.Amount of Bl.
on initial head computed tomography (CT) scan (inverse correlation).
‫ــــــــــــــــــــــــــــــــــــــــــــــــــــــــــــــــــــــــــــــــــــــــــــــــــــــــــــــــــــــــــــــــــــــــــــــــــــــــــــــــــــــــــــــــــــــــــــــــــــــــــــــــــــــــــــ‬

194- Question 15 of 20
A 24 yo man was brought to the ED after being mugged. He was stabbed in the abd... His Bl. Pr. was 140/80mmHg &
his heart rate was 82bpm. Exam. Revealed a stab wound superior to his umbilicus.

(a) In relation to the stabbing instrument what points are important in the history? (Four points)
Answer: What instrument was used, how long it was, how wide it was, how he was positioned during the stabbing, &
what path the implement traveled.
(b) How would you investigate this Pt?
Answer: Local wound exploration, FBC, UE, Bl. group & hold, CT scan.
(c) How would you manage this Pt? (Four points)
Answer: Provide initial resuscitation based upon protocols from Advanced Trauma Life Support, Monitored bed, two
wide bore IV lines, IV fluids, surgical consultation.
(d) Name two points in the exam. That if present are strong indicators for urgent laporotomy?
Answer: Immediate laparotomy was traditionally indicated in the presence of hemodynamic instability, evisceration,
or unequivocal peritoneal signs on physical exam... Others are signs of GIT hemorrhage, & an implement in situ.
(e) What is the role of plain radiographs in this Pt?
Answer: Plain film radiographs add little to the management of stab wounds.
‫ــــــــــــــــــــــــــــــــــــــــــــــــــــــــــــــــــــــــــــــــــــــــــــــــــــــــــــــــــــــــــــــــــــــــــــــــــــــــــــــــــــــــــــــــــــــــــــــــــــــــــــــــــــــــــــ‬
195- Question id: 2037
A 30 yo man presented with severe Lt Flank pain, nausea, & difficulty urinating.He had microscopic Bl. in his urine.
(a) List 2 possible diagnoses?
Answer: Renal colic, pyelonephritis, & renal cell carcinoma.
(b) Name 3 risk factors for nephrolithiasis?
Answer: For calcium stones, urinary risk factors include hypercalciuria, hyperoxaluria, hyperuricosuria,
hypocitraturia, & dietary risk factors such as a low calcium intake, high oxalate intake, high animal protein intake,
high sodium intake, or low fluid intake. A history of prior nephrolithiasis, Pts with a family history of stones have an
increased risk of nephrolithiasis, frequent upper UTIs, & HTN.
(c) Name two complications of nephrolithiasis?
Answer: Nephrolithiasis may lead to persistent renal obstruction, staghorn calculi, & infection.
(d) Name one type of stone which is radiolucent on abd. X ray?
Answer: Uric acid stone.

Fig 1
(e) What investigation should be used in pregnant Pts?
Answer: Ultrasound is the initial diagnostic test in pregnant women or in Pts in whom cholecystitis or a gynecologic
process is a prominent consideration.
‫ــــــــــــــــــــــــــــــــــــــــــــــــــــــــــــــــــــــــــــــــــــــــــــــــــــــــــــــــــــــــــــــــــــــــــــــــــــــــــــــــــــــــــــــــــــــــــــــــــــــــــــــــــــــــــــ‬
196- Question 12 of 50
79
A 29 yo who has recently had a baby & is breast-feeding comes to see you as she has developed diarrhoea & is concerned

(a) She takes oral aminophyline for asthma. Which antibiotic should you avoid in this Pt?
Answer: Ciprofloxacin & erythromycin are both liver enzyme inhibitors & can therefore increase plasma
concentrations of theophyline.
(b) She came off lithium prior to conception but is worried about her depression what do you advice?
Answer: Cannot go on it whilst still breast feeding due to risks to baby of involuntary movements.
(c) What drug can cause cleft lip & palate if taken during pregnancy? & which drug should therefore be used
instead?
Answer: Phenytoin can cause cleft lip/palate. Monotherapy with carbamazepine is probably the safest. Risks of major
congenital malformations related to specific anti-epileptic drugs Carbamazepine taken as a single drug TTT (known
as monotherapy) carries the lowest risk, with 2.2 babies born with MCMs in 100 women taking the drug (2.2%)
Taking Na valproate as monotherapy at a daily dosage under 1000mg, carries a risk of 5.1% Taking Na valproate as
monotherapy at daily doses over 1000mg carries a risk of 9.1% Drug combinat. That include Na valproate have a
significantly higher risk of MCMs than combinations that don't include this drug. Taking lamotrigine as monotherapy
at daily dosages of 200 mg or less carries a risk of 3.2% Taking lamotrigine as monotherapy at a daily dosage above
200 mg carries a risk of 5.4% Taking carbamazepine & Na valproate together carries a risk of 8.8% Taking Na
valproate & lamotrigine together carries a risk of 9.6% The information from the study didn't include any specific data
on vigabatrin, gabapentin, topiramate, tiagabine, oxcarbazepine, levetiracetam & pregabalin.1
(d) She is sexually active again & doesnt want to conceive what advice do you give regarding contraception?
Answer: Can't go on the OCP due to risks associated with breast feeding. The progesterone only pill/condoms/cap etc
are other options.
‫ــــــــــــــــــــــــــــــــــــــــــــــــــــــــــــــــــــــــــــــــــــــــــــــــــــــــــــــــــــــــــــــــــــــــــــــــــــــــــــــــــــــــــــــــــــــــــــــــــــــــــــــــــــــــــــ‬
197- Question 18 of 50
A 29 yo lady with hyperemesis grvidarum attends the ED as she is on holiday in the local area visiting her mother.

(a) Considering hyperemesis gravidarum, when is it most common & when would you expect it to have
resolved by?
Answer: 8-12 weeks, settled by 20 weeks, more likely to get it if you are younger than 30.
(b) Is a family history relevant?
Answer: Yes there's a genetic component. It is more likely in sisters & daughters of women who have suffered with it
(c) How is it treated?
Answer: NB that it is a diagnosis of exclusion; need to rule out other things by investigat. Can use lots of antiemetics-
In the Uk normally start with antihistamine then proclorperazine or metocloparimde then ondansetron See
Cochrane review on what's the best evidenced based therapy: Anti-emetic medicat. appears to ↓ the frequency of
nausea in early preg. There is some evidence of adverse effects, but there is very little informat. On effects on fetal outcomes
from randomised controlled trials. Of newer TTT, pyridoxine (vit. B6) appears to be more effective in ↓ the severity of
nausea. The results from trials of P6 acupressure are equivocal. No trials of TTT for hyperemesis gravidarum show
any evidence of benefit. Evidence from observational studies suggests no evidence of teratogenicity from any of these
TTT.
‫ــــــــــــــــــــــــــــــــــــــــــــــــــــــــــــــــــــــــــــــــــــــــــــــــــــــــــــــــــــــــــــــــــــــــــــــــــــــــــــــــــــــــــــــــــــــــــــــــــــــــــــــــــــــــــــ‬
198- Question 36 of 50
A 29 yo woman presented with severe RT lower quadrant pain which had begun during exercise. She had no history
of PV bleeding & wasn't sexually active. She was at the mid-point of her menstrual cycle. O/E she had moderate
tenderness in her RT lower quadrant but had no guarding. She was afebrile & haemodynamically stable

(a) What is the differential diagnosis? (Give five)


Answer: Ruptured ovarian cyst, ectopic pregnancy, endometriosis, PID, appendicitis, renal calculi, mittelschmerz
pain, ovarian torsion & pain from a leiomyoma.
(b) How would you investigate this Pt in the ED? (Give five investigations)
Answer: Urinalysis, urine or serum HCG, FBC, UE, pelvic ultrasound, & a vaginal swab.
(c) How would you manage this Pt in the ED if she has an uncomplicated ovarian cyst rupture? (Give two
points)
Answer: Analgesia, IV fluids & organise follow up.
(d) What is the indication for surgery if this Pt has a complicated ovarian cyst rupture?
Answer: Emergency surgery is performed to control ongoing significant hemorrhage.
(e) How would the presentation of a dermoid cyst rupture vary from that of a simple cyst rupture?
Answer: Shock & hemorrhage are the immediate sequelae of rupture of a dermoid cyst due to spillage of sebaceous
material into the abd. Cavity.
80
‫ــــــــــــــــــــــــــــــــــــــــــــــــــــــــــــــــــــــــــــــــــــــــــــــــــــــــــــــــــــــــــــــــــــــــــــــــــــــــــــــــــــــــــــــــــــــــــــــــــــــــــــــــــــــــــــ‬
199- Question 2 of 10
A 25 yo woman presented with persistent vomiting at 9 wks gestation. U/S confirmed a viable intrauterine pregnancy.

(a) How would you investigate this Pt? (Give three investigations)
Answer: Measurement of weight, orthostatic Bl. Pr.s, serum free T4 concentrat, serum electrolytes, & urine ketones.
(b) How would you manage this Pt? (Name three points)
Answer: Gut rest, IV rehydration, avoidance of precipitants, anti-emetic medication.
(c) How is the diagnosis of hyperemesis gravidarum made?
Answer: The diagnosis of hyperemesis gravidarum is made clinically in a woman with onset of persistent vomiting accompanied
by weight loss exceeding 5% of prepreg. Body weight & ketonuria in the 1st trimester, unrelated to other causes.
(d) List three possible maternal complications of hyperemesis gravidarum?
Answer: Micronutrient deficiency, wernicke encephalopathy (from deficiency of vit. B1) & sequelae of malnutrition
(immunosuppression, poor wound healing) have been reported. Oesophageal tears & rupture are other possible complicat.
(e) What birth defects are associated with hyperemesis gravidarum?
Answer: There's no clear  in the risk of birth defects among offspring of gravida with hyperemesis gravidarum.
‫ــــــــــــــــــــــــــــــــــــــــــــــــــــــــــــــــــــــــــــــــــــــــــــــــــــــــــــــــــــــــــــــــــــــــــــــــــــــــــــــــــــــــــــــــــــــــــــــــــــــــــــــــــــــــــــ‬
200- Question 3 of 10

A 19 yo woman attends with a history of lower abd. Pain.

(a) What are the 3 common features of pelvic inflammatory disease?


Answer: Minimum criteria for the diagnosis of PID are listed below. Institute empiric TTT of PID when a Pt has all
of the following minimal clinical criteria in the absence of an established cause other than PID: Lower abd.
Tenderness on palpation Adnexal tenderness cervical motion tenderness Additional criteria, especially in women with
more severe clinical signs, can be used to  the specificity of the diagnosis. Oral temp. >38.3C -Abnormal cervical or
vaginal discharge - ESR - CRP –Lab. Documentat. of cervical infect. With N gonorrhoeae or C trachomatis. It is
important to note that PID can occur & cause serious harm without causing any noticeable symptoms.
(b) What combination of drugs would you use to treat this condition?
Answer: 1st line antibiotics for pelvic inflammatory disease: Ceftriaxone 250mg IM stat (for gonococcal cover) plus
Doxycycline 100mg bd for 14 days & Metronidazole 400 mg bd PO for 5 days
(c) In what circumstances would you give IV antibiotics?
Answer: IV therapy for severe disease is indicated if: A surgical emergency can't be excluded Lack of response to
oral therapy clinically severe disease (temp >38oC, signs of pelvic peritonitis, signs of a tubo-ovarian abscess)
Intolerance to oral therapy Disseminated gonococcal infection.
(d) What do you want to rule out in this Pt?
Answer: Ectopic pregnancy. Or other serious cause of abdo pain
(e) What investigations would you do?
Answer: FBC,U&E,CRP,LFTs,CXR,ECG, beta HCG, amylase/lipase, clotting screen, G&S=/- cross match.
‫ــــــــــــــــــــــــــــــــــــــــــــــــــــــــــــــــــــــــــــــــــــــــــــــــــــــــــــــــــــــــــــــــــــــــــــــــــــــــــــــــــــــــــــــــــــــــــــــــــــــــــــــــــــــــــــ‬
200- Question 5 of 10

A 26 yo woman presented with right iliac fossa pain & PV bleeding. She was afebrile. Urine HCG was positive.

(a) What is the diagnosis? What is the differential diagnosis?


Answer: Ectopic pregnancy. Normal pregnancy with threatened/missed or incomplete abortion, ruptured or torsed
corpus luteum cyst, & degenerating uterine leiomyoma.
(b) Name two investigations which should be performed?
Answer: Transvaginal +/- transabd. Ultrasound & serum quantitative HCG.
(c) Name three management steps?
Answer: Analgesia.Bl. Group. Anti-D if Pt is rhesus negative. Obstetrical consultation.
(d) Name five risk factors for this condition?
Answer: Previous ectopic, previous tubal surgery, tubal ligation, intrauterine DES exposure, current IUD use, tubal
pathology, infertility, previous cervicitis (gonorrhea, chlamydia), history of pelvic inflammatory disease, multiple
sexual partners, smoking, previous pelvic/abd. Surgery, vaginal douching, & early age of intercourse (<18 yrs).
(e) The above lady was haemodynamically stable & there was no fetal cardiac activity. It was decided to treat
this lady with methotrexate. Below what serum HCG is methotrexate suitable?
Answer: HCG <5000 mIU/mL
‫ــــــــــــــــــــــــــــــــــــــــــــــــــــــــــــــــــــــــــــــــــــــــــــــــــــــــــــــــــــــــــــــــــــــــــــــــــــــــــــــــــــــــــــــــــــــــــــــــــــــــــــــــــــــــــــ‬

81
201- Question 6 of 10
A 23 yo who is gravida 2 para 1 attends with PV bleeding in the 8th week of her current pregnancy. She has also had
some mild lower back & abd. pain.

(a) What is a threatened miscarriage?


Answer: Vaginal bleeding through a closed cervical os. 50% will go on to miscarry.
(b) What is cervical shock?
Answer: Severe pain & bleeding accompanied by hypotension & bradycardia might be due to cervical shock due to a
vagal response caused by the presence of retained products stuck in the cervical os.
(c) How do you treat it?
Answer: Remove any products of conception from the cervical os using sponge forceps.
(d) What is it important to do in a woman who has had an inevitable abortion?
Answer: Need to be referred to gynaecologists for in Pt care- may well go on to require a D+C. Also need to consider
Rheus status- if mother is -ve & non-immune then will need anti-D.
(e) List 4 risk factors for ectopic pregnancy
Answer: 1-Previous pelvic surgery 2-Previous ectopic 3-PID 4-Assisssted fertilisat. 5-Endometriosis 6-IUCD 7-POP.
‫ــــــــــــــــــــــــــــــــــــــــــــــــــــــــــــــــــــــــــــــــــــــــــــــــــــــــــــــــــــــــــــــــــــــــــــــــــــــــــــــــــــــــــــــــــــــــــــــــــــــــــــــــــــــــــــ‬
202- Question 1 of 10
A 26 yo Lady presents to the ED with vomiting. She is 13/40 preg. in her 1st preg. She tells you that she can't hold
anything down & that it has been getting worse over the last few days. The 12 wks scan showed a healthy intrauterine
foetus. O/E she appears dehydrated & is tachycardic.

(a) What are the possible diagnoses?


Answer: Any cause of vomiting- normal part of preg., GE, etc.. hyperemesis gravidarum Appendicitis, Acute Ovarian
Torsion Cholecystitis & Biliary Colic, Pancreatitis, DKA, Pregnancy, Preeclampsia Gastritis & PUD UTI, Female
Gastroenteritis Hepatitis, Pyelonephritis, Molar pregnancy, Pseudotumor cerebri Acute fatty liver of pregnancy
(b) What investigations should be performed?
Answer: Lab. Studies * Obtain electrolyte levels. * Measure urine gravity & ketones. * Perform LFT if hepatitis is a
concern. Of note, LFTs can be slightly  with hyperemesis gravidarum. * Perform a CBC & urinalysis to rule out
other causes, with particular concern for pyelonephritis. * Hyperthyroidism causing nausea & vomiting is rare, a T3 &
T4 level should be drawn if this is a concern. (TSH can be suppressed in hyperemesis gravidarum.) * Obtain serum
amylase-to-creatinine ratio if pancreatitis is a concern. * Serum hCG levels are not clinically useful in a Pt with a
known intrauterine pregnancy & hyperemesis. Imaging Studies * The Pt should have an ultrasonographic evaluation
of her pregnancy to look for molar pregnancy or multiple gestations.
(c) What would prompt you to admit the Pt?
Answer: Presence of ketones is an important marker clearly if the Bl. tests revealed significant renal failure or other
concerning features then the Pt would need to be admitted
(d) What analgesics are good in the first trimester?
Answer: Paracetamol or opiod based, avoid NSAIDS
‫ــــــــــــــــــــــــــــــــــــــــــــــــــــــــــــــــــــــــــــــــــــــــــــــــــــــــــــــــــــــــــــــــــــــــــــــــــــــــــــــــــــــــــــــــــــــــــــــــــــــــــــــــــــــــــــ‬
203- Question 3 of 5
A 24 yo woman presented with a 2 week history of lower abd. pain & pustular vaginal discharge.

(a) What is the differential diagnosis?(Name three)


Answer: PID, intrauterine pregnancy complication, obstetrical malignancy, cystitis, & urethritis.
(b) How would you investigate this Pt?(Name five)
Answer: Preg. test to rule out ectopic preg. & complications of an intrauterine pregnancy, urinalysis, vaginal swab,
gram stain & microscopic exam. of vaginal discharge, FBC, ESR, CRP, transabd. +/- transvaginal U/S, CT pelvis.
(c) What are the risk factors for pelvic inflammatory disease?(Name three)
Answer: Multiple partners, age (PID occurs in highest frequency among those 15 to 25 yrs of age) previous PID, IUD
insert. (the risk of PID is primarily limited to the 1st 3 wks after IUD insert.) while barrier contracept. protects against PID.
(d) Name the two most common pathogens which cause PID?
Answer: Neisseria gonorrhoeae & Chlamydia trachomatis.
(e) Name three complications of PID?
Answer: Infertility, ectopic pregnancy, & chronic pelvic pain.
‫ــــــــــــــــــــــــــــــــــــــــــــــــــــــــــــــــــــــــــــــــــــــــــــــــــــــــــــــــــــــــــــــــــــــــــــــــــــــــــــــــــــــــــــــــــــــــــــــــــــــــــــــــــــــــــــ‬
204- Question id: 2073
A 33 yo man presented with pleuritic chest pain which was relieved by sitting forward. O/E there was an audible frict. rub.

82
(a) What is the diagnosis?
Answer: Pericarditis.
(b) What is the aetiology of this condition?(Name three)
Answer: Viral (adenovirus, enteroviruses, CMV, influenza virus, HBV & HSV), TB, other bacterial, "autoreactive"
(immune-mediated), uremia, neoplastic, idiopathic.
(c) Name four laboratory findings which support the diagnosis?
Answer: CK-MB, troponin, CRP, elevated WBC count, elevated ESR.
(d) Name three features of the ECG which help to distinguish this condition from an acute MI.
Answer: The ST segment elevation in acute pericarditis begins at the J point, rarely exceeds 5 mm & usually retains
its normal concavity. The distribution of ST elevation is different. Acute STEMI is often associated with reciprocal
ST segment changes, which are not seen with pericarditis except in aVR & V1.
(e) How should this Pt be managed?
Answer: TTT of the underlying condition or if viral or idiopathic,NSAIDs.
‫ــــــــــــــــــــــــــــــــــــــــــــــــــــــــــــــــــــــــــــــــــــــــــــــــــــــــــــــــــــــــــــــــــــــــــــــــــــــــــــــــــــــــــــــــــــــــــــــــــــــــــــــــــــــــــــ‬
205- Question 14 of 50
You see a 74 yo gentleman with known hypertension, he is c/o feeling sick, & has a headache, he also appears to be
confused. His BP is 220/130.

(a) Is this mild, moderate or severe hypertension?


Answer: Severe: >125 diastolic = severe
(b) What is the diagnosis?
Answer: Hypertensive encephalophay.
(c) What must your physical examination look for? (What conditions need ruling out?)
Answer: Need to examine the retinal for changes (exudates, haemorrhages, papilodema) Need to examine for any
focal neurology, accurately record GCS & mental status. Need to rule out a stroke or subarachnoid haemorrhage. If
suspected needs urgent CT head.
(d) When & who should start therapy? How should it be commenced?
Answer: Not in the ED, medical team to start TTT may require ICU/HDU care. Aim is to reduce the BP slowly.
‫ــــــــــــــــــــــــــــــــــــــــــــــــــــــــــــــــــــــــــــــــــــــــــــــــــــــــــــــــــــــــــــــــــــــــــــــــــــــــــــــــــــــــــــــــــــــــــــــــــــــــــــــــــــــــــــ‬
206- Question 16 of 50
A 72 yo man presented with a low grade fever, weight loss, & fatigue. He complained of a severe new unilateral
temporal headache & jaw claudication.
(a) Name three other symptoms associated with this condition?
Answer: Temporal arteritis may cause visual complaints such as amurosis fugax, polymyalgia rheumatica, upper
respiratory tract symptoms, arm claudication, symptoms 2ry to aotic aneurysms & aortic dissection may occur.
(b) Name three possible findings on physical exam?
Answer: Pulses may be diminished in the setting of large vessel disease.Tender or thickened temporal or other cranial
arteries can occur.Some Pts have cotton wool spots in the retina, depending on the site of critical vascular lesions.
Fundoscopic examination shows changes of ischemic optic neuropathy with a swollen pale disc & blurred margins. In
Pts with PMR, active range of motion of the shoulders, neck, & hips is limited due to pain. Bruits may be heard on
auscultation of the carotid or supraclavicular areas.
(c) Name three likely laboratory findings?
Answer: A characteristic lab. abnormality seen in most Pts with GCA is a  ESR, which often reaches 100 mm/h or
more. Serum CRP levels in GCA tend to parallel those of the ESR while the WBCs is usually normal, even in the
setting of widespread systemic inflammat.  serum concentrate. of hepatic enzymes, such as aspartate
83
aminotransferase & alkaline phosphatase, occur in 25 to 35% of Pts. The serum albumin level is often moderately ↓ at
diagnosis & a normochromic anemia is generally present prior to therapy.
(d) What is the TTT for this condition?
Answer: If temporal arteritis is not complicated by symptoms or signs of ischemic organ damage (eg, visual loss) an
initial dose of glucocorticoid equivalent to 40 to 60 mg of prednisone in a single dose is appropriate. If potentially
reversible symptoms persist or worsen, the dose may increased until symptomatic control is achieved.
(e) What is the prognosis for this condition?
Answer: In most Pts temporal arteritis tends to run a self-limited course over several months to several yrs. The
glucocorticoid dose can eventually be reduced & discontinued in the majority of Pts.A sizable minority have more
chronic disease & require low doses of prednisone for a number of yrs to control symptoms.
‫ــــــــــــــــــــــــــــــــــــــــــــــــــــــــــــــــــــــــــــــــــــــــــــــــــــــــــــــــــــــــــــــــــــــــــــــــــــــــــــــــــــــــــــــــــــــــــــــــــــــــــــــــــــــــــــ‬
207- Question 17 of 50
A 25 yo lady is brought in by ambulance. She has taken 7 bottles of 650ml methadone & 2 bottles of vodka. The ED
doctor who laid eyes on her wrote looks dreadful in the notes. She has pinpoint pupils.
(a) What is your immediate management?
Answer: ABCD with focused history if available Main priority will usually be A in such Pts Ensure that airway is
maintained (may require adjuncts or actually ventilating them, always have bag valve mask on hand) Can give IM
naloxone if doesn't have IV access
(b) You cannot get IV access quickly in this lady what will you do?
Answer: Give IM naloxone likely better to start with a reasonable dose i.e. 600mcg. Can give 200mcg-2mg in
repeated doses up to 10mg.
(c) What is the dose of naloxone in children?
Answer: 10mcg/kg with subsequent doses 100mcg/kg
(d) You get a response to your TTT. However it appears transient what will you do? (Please describe exactly
showing the workings of any calculations)
Answer: (600/100 x 60) = 360mcg (This is the amount that you want to infuse over 1 hr. Make up a syringe
containing 4mg of naloxone in 20mls (therefore 200mcg per ml) Therefore 1.8ml per hr needs IVI of naloxone. BNF
says: By continuous IVI using an infusion pump, 4 mg diluted in 20 mL IV infusion solution [unlicensed concentrate.]
at a rate adjusted according to response (initial rate may be set at 60% of initial IV injection dose (see above) &
infused over 1 hr)
(e) You take an ABG & the results are shown below: pH 7.107 pCO2 8.93 pO2 36.05 HCO3 16.2 BE -10.6 HB
15.1 Glucose 13.2 Describe what it shows?
Answer: Metab. acidosis -ve BE & ↓ pH. Also comment on the glucose. Comment on the high CO2 Resp. depression
(f) What could explain these results?
Answer: Could be DKA, high CO2 due to respiratory depression. Effects of alcohol & methadone could explain these
results. Could be septic?
‫ــــــــــــــــــــــــــــــــــــــــــــــــــــــــــــــــــــــــــــــــــــــــــــــــــــــــــــــــــــــــــــــــــــــــــــــــــــــــــــــــــــــــــــــــــــــــــــــــــــــــــــــــــــــــــــ‬

84

Das könnte Ihnen auch gefallen